You are on page 1of 909

PREP® Self-Assessment PREPSA 2023

American Academy of Pediatrics 1


PREP® Self-Assessment PREPSA 2023

Question 1
A 2-week-old boy is brought to the clinic for persistent vomiting. The neonate is exclusively fed
a standard cow milk– based formula. He takes 4 oz every 2 hours and vomits after every feeding,
despite burping. The emesis is not bloody, bilious, or projectile. He has 4 yellow, seedy, soft
stools per day without blood. He is otherwise healthy. The neonate was born at term via an
uncomplicated delivery and had an unremarkable newborn nursery course. His weight today is
greater than his birth weight. His vital signs and physical examination findings are unremarkable.

Of the following, the MOST likely cause of this neonate’s symptoms is


A. gastroesophageal reflux disease
B. milk protein allergy
C. overfeeding
D. pyloric stenosis

American Academy of Pediatrics 2


PREP® Self-Assessment PREPSA 2023

Correct Answer: C
The neonate in the vignette has gastroesophageal reflux (GER) caused by overfeeding.
Gastroesophageal reflux, defined in the 2018 American Academy of Pediatrics Gastroesophageal
Reflux Clinical Report (Eichenwald) as “the passage of gastric contents into the esophagus…can
be distinguished from gastroesophageal reflux disease (GERD), which includes troublesome
symptoms or complications associated with GER.” Gastroesophageal reflux is a relatively
common, normal physiologic occurrence, affecting over two-thirds of healthy infants. Signs and
symptoms of GER consist of spitting up, regurgitation, or even emesis. If there is associated
pain, poor weight gain, or dysphagia, the infant would meet criteria for GERD; however, the
neonate in the vignette does not have these symptoms.

Overfeeding is a common cause of GER among infants. When the physiologic gastric volume
capacity is exceeded, increased intragastric pressure results in GER. Although the volume
consumed and timing of feedings varies among infants, full-term infants typically formula-feed 2
ounces every 3 to 4 hours (Item C1). The management of GER in infants includes reducing the
feeding volume, increasing feeding frequency, avoiding seated and supine positions post-
feedings, and assisting the infant in expelling gastric gas by patting the back (“burping”).
Gastroesophageal reflux typically resolves by 12 months of age.

Cow milk protein allergy, or allergic proctocolitis, results from hypersensitivity to the proteins in
cow milk–based formulas. Symptoms and signs commonly include fussiness; loose, mucousy
stools; hematochezia; emesis; and/or GERD.

Pyloric stenosis results from hypertrophy of the pylorus causing gastric obstruction. It typically
presents between 3 and 5 weeks of age with forceful, often projectile, nonbloody, nonbilious
emesis.

American Academy of Pediatrics 3


PREP® Self-Assessment PREPSA 2023

PREP Pearls
• Gastroesophageal reflux (GER) is a relatively common, normal physiologic occurrence,
affecting over two-thirds of
• healthy infants. Signs and symptoms of GER consist of spitting up, regurgitation, or even
emesis. If there is associated pain, poor weight gain, or dysphagia, the infant would meet
criteria for gastroesophageal reflux disease.
• Overfeeding is a common cause of gastroesophageal reflux (GER) among infants. When
the physiologic gastric volume capacity is exceeded, increased intragastric pressure
results in GER.
• The management of gastroesophageal reflux in infants includes reducing the feeding
volume, increasing feeding frequency, avoiding seated and supine positions post-
feedings, and assisting the infant in expelling gastric gas by patting the back (“burping”).
Gastroesophageal reflux typically resolves by 12 months of age.

ABP Content Specifications(s)


• Differentiate gastroesophageal reflux from gastroesophageal reflux disease

Suggested Readings
• American Academy of Pediatrics Committee on Nutrition. Feeding the infant. In
Kleinman RE, Greer FR, eds. Pediatric Nutrition. 7th ed. American Academy of
Pediatrics; 2013:13-110.
• DiMaggio DM, Cox A, Porto AF. Updates in infant nutrition. Pediatr Rev.
2017;38(10):449-462. doi:10.1542/pir.2016-0239
• Eichenwald EC; Committee on Fetus and Newborn; Cummings JJ, Aucott SW,
Goldsmith JP, et al. Diagnosis and management of gastroesophageal reflux in preterm
infants. Pediatrics. 2018;142(1):e20181061. doi:10.1542/peds.2018-1061
• Lightdale JR, Gremse DA; Section on Gastroenterology, Hepatoloy, and Nutrition.
Gastroesophageal reflux: management guidance for the pediatrician. Pediatrics.
2013;131(5):e1684-e1695. doi:10.1542/peds.2013-0421.
• Rosen R, Vandenplas Y, Singendonk M, et al. Pediatric gastroesophageal reflux clinical
practice guidelines: joint recommendations of the North American Society for Pediatric
Gastroenterology, Hepatology, and Nutrition and the European Society for Pediatric
Gastroenterology, Hepatology, and Nutrition. J Pediatr Gastroenterol
Nutr.2018;66(3):516-554. doi:10.1097/MPG.0000000000001889.

American Academy of Pediatrics 4


PREP® Self-Assessment PREPSA 2023

Question 2
A 12-year-old, 40-kg boy in the pediatric intensive care unit with acute respiratory distress
syndrome is currently intubated and mechanically ventilated, and has persistent hypoxemia. His
last arterial blood gas analysis showed a pH of 7.22, pCO2 of 65 mm Hg, and a pO2 of 58 mm
Hg. After adjusting the ventilator, his arterial blood gas analysis shows a pH of 7.25, pCO2 of 63
mm Hg, and pO2 of 80 mm Hg.

Of the following, the MOST likely change made to the mechanical ventilator settings was a(n)
A. decrease in positive end-expiratory pressure from 8 to 7 cm H2O
B. decrease in tidal volume from 300 to 240 mL
C. increase in inspiratory time from 0.7 to 0.9 seconds
D. increase in respiratory rate from 22 to 24 breaths/minute

American Academy of Pediatrics 5


PREP® Self-Assessment PREPSA 2023

Correct Answer: C
The child in the vignette has acute respiratory distress syndrome (ARDS) and persistent
hypoxemia. The change made to the mechanical ventilator settings resulted in an improvement in
oxygenation (pO2) with negligible change in ventilation (pCO2). Oxygenation is a function of
mean airway pressure. The main drivers of mean airway pressure are peak inspiratory pressure
(PIP), positive end-expiratory pressure (PEEP), and inspiratory time fraction (i-time) (Item
C2A). Thus, the most likely ventilator change that resulted in a higher pO2 was an increase in the
inspiratory time fraction. Decreasing the PEEP would have resulted in the opposite effect and
likely a lower pO2. A decrease in the tidal volume would not have increased the pO2 and would
have caused an increase in pCO2. An increase in the respiratory rate would have little effect on
oxygenation and would have decreased the pCO2.

Item C2A Determinants of oxygenation during pressure-limited, time-cycled ventilation. Shaded


circles represent ventilator-controlled variables.
Solid lines represent the simple mathematical relationships that determine mean airway pressure and
oxygenation, and dashed lines represent relationships that cannot be quantified with a simple
mathematical method. From Carlo WA, Greenough A, Chatburn RL. Advances in conventional
mechanical ventilation. In: Boynton BR, Carlo WA, Jobe AH, eds. New Therapies for Neonatal
Respiratory Failure. Boston, Mass: Cambridge University Press; 1994.

Children with acute lung disease require intubation and mechanical ventilation for 2 primary
causes of respiratory failure: (1) failure of oxygenation and (2) failure of ventilation. The
ventilator management of these 2 distinct pathologic states requires an understanding of the basic
mechanics of ventilation and ventilator function.

Failure of oxygenation or ventilation can occur as a result of new-onset lung disease, cardiac
dysfunction, neurologic abnormalities, or multi-organ system failure. Examples of primary
parenchymal lung disease include:
• Pneumonia
• ARDS
• Inhalation injury
• Chest trauma Drowning

American Academy of Pediatrics 6


PREP® Self-Assessment PREPSA 2023

• Aspiration
• Chronic lung disease
• Pulmonary fibrosis
• Asthma

The goals of mechanical ventilation include optimizing gas exchange, reducing work of
breathing, and limiting ongoing lung damage. Mechanical ventilation, if not prescribed correctly,
can lead to progressive and additional lung injury as a result of excessive pressure or
overdistension from high delivered tidal volume.

Alveolar pressure is the main determinant of oxygenation. Because alveolar pressure cannot be
directly measured, mean airway pressure measured by the mechanical ventilator can be used as a
surrogate for mean alveolar pressure. Mean airway pressure can be visualized schematically as
the area under the curve of a single breath cycle when ventilator pressure is plotted against time
(Item C2B). Mean airway pressure can be increased by providing a higher PIP, higher PEEP, or
longer inspiratory time. A higher mean airway pressure results in improved oxygenation.

Item C2B: Percentage change in pressure in relation to the time (in time constants) allowed
for equilibration. As a longer time is allowed for equilibration, a higher percentage change in
pressure will occur. The same rules govern the equilibration for step changes in volume

Excessive mean airway pressure, however, can result in lung damage from high peak/plateau
pressures (barotrauma) or high alveolar stretch and tidal volume (volutrauma). Children who are
intubated, are mechanically ventilated, and have ongoing hypoxemia ultimately require a higher
mean airway pressure in order to improve their oxygenation status. Although an increase in the
fraction of inspired oxygen (FiO2) can temporarily improve oxygenation, high concentrations of
oxygen are toxic to the alveoli due to the development of free radicals. Increasing the FiO2 can
temporize hypoxia; however, ventilator adjustments that increase mean airway pressure should
be made in order to optimize oxygen delivery while minimizing toxicity.

Ventilation refers to the clearance of CO2 from the body and is a function of minute ventilation.
Minute ventilation can be calculated as the mathematical product of respiratory rate and tidal
volume (MV= RR × TV) (Item C2C). If CO2 production is relatively constant, manipulating the
minute ventilation will have a predictable effect on the concentration of CO2 and rate of
elimination. Under steady state CO2 production, changes in minute ventilation result in a linear
and inversely proportional clearance of CO2. Thus, an increase in minute ventilation will result in

American Academy of Pediatrics 7


PREP® Self-Assessment PREPSA 2023

a proportional decrease in pCO2, and a decrease in minute ventilation will result in an increase in
pCO2.

Item C2C
alveolar minute ventilation = (tidal volume - dead space) X frequency

PREP Pearls
• Mean airway pressure is the driving force behind oxygenation in mechanically ventilated
patients and is affected primarily by changes in peak inspiratory pressure, positive end-
expiratory pressure, and inspiratory time fraction.
• Carbon dioxide is eliminated in a linear fashion, inversely proportional to minute
ventilation (minute ventilation = respiratory rate × tidal volume).
• Mechanical ventilation can induce lung injury by both excessive pressure (barotrauma)
and/or excessive volume (volutrauma).

ABP Content Specifications(s)


• Plan the appropriate ventilatory support for patients with various conditions

Suggested Readings
• Carlo WA, Ambalavanan N. Conventional mechanical ventilation: traditional and new
strategies. Pediatr Rev. 1999;20(12):e117-e126. doi:10.1542/pir.20-12-e117.
• Cheifetz IM. Invasive and noninvasive pediatric mechanical ventilation. Respir Care.
2003;48(4):442-458. http://rc.rcjournal.com/content/48/4/442.

American Academy of Pediatrics 8


PREP® Self-Assessment PREPSA 2023

Question 3
A 13-month-old girl is seen for a health supervision visit. She was diagnosed with perinatally
acquired HIV infection at 2 weeks of age and is currently receiving combination antiretroviral
therapy. She has no history of opportunistic infections. She received her routine childhood
immunizations at ages 2, 4, and 6 months including Haemophilus influenzae type b (Hib)
conjugate vaccine and pneumococcal conjugate vaccine (PCV13). Her physical examination
findings are normal. Laboratory data are notable for a CD4+ T lymphocyte count of 750 cells/µL
(reference range for age 1-5 years, ≥1,000 cells/µL), CD4+ percentage of 25% (reference range
for age 1-5 years, ≥30%), and an HIV viral load of 4,900 copies/mL.

Of the following, in addition to diphtheria-tetanus-pertussis (DTaP), Hib, PCV13, and hepatitis


A, the MOST appropriate vaccine(s) to administer today is
A. measles-mumps-rubella
B. measles-mumps-rubella and varicella
C. measles-mumps-rubella-varicella
D. varicella

American Academy of Pediatrics 9


PREP® Self-Assessment PREPSA 2023

Correct Answer: B
The girl in the vignette is living with HIV infection and has evidence of low-level
immunosuppression based on the absence of opportunistic infections, a CD4+ T lymphocyte
count greater than 500/µL, and CD4+ percentage greater than 22%. In this setting, both measles-
mumps-rubella (MMR) and varicella vaccines are indicated. In addition, all inactivated vaccines,
including diphtheria-tetanus-pertussis (DTaP), Haemophilus influenzae type b (Hib),
pneumococcal conjugate vaccine 13 (PCV13), and hepatitis A vaccines, should be administered
at this visit. Given the lack of safety data, children with HIV infection should not receive the
quadrivalent measles-mumps-rubellavaricella (MMRV) vaccine.

All infants with HIV infection should receive the rotavirus vaccine, irrespective of CD4+ T-
lymphocyte count or percentage.

Severe immunosuppression, as a general principle, is a contraindication to the administration of


live-virus vaccines (eg, MMR, varicella, and MMRV). This principle applies to individuals with
high-level immunosuppression categorized as follows:
• Children ages 1 through 13 years living with HIV with a CD4+ T lymphocyte percentage
of less than 15%
• Adolescents age 14 years and older living with HIV with a CD4+ T lymphocyte count of
less than 200/µL Children with cancer receiving chemotherapy
• Children with combined B- and T-lymphocyte primary immunodeficiency disorders (eg,
severe combined immunodeficiency disorder)
• Receipt of daily high doses of corticosteroids (dose of ≥20 mg/day or ≥2 mg/kg per day
for 2 weeks or longer)
• Receipt of certain biologic agents (eg, tumor necrosis factor–antagonists, anti–B-
lymphocyte or anti–Tlymphocyte monoclonal antibodies)
• Receipt of solid organ transplant in the past 2 months
• Receipt of hematopoietic stem cell transplant within the past 2 months (and often for a
longer duration)

Children with immunosuppression should receive killed vaccines as routinely recommended (eg,
Hib, pneumococcal, meningococcal, inactivated polio, diphtheria and tetanus toxoids and
acellular pertussis vaccine, hepatitis A, hepatitis B, and inactivated influenza vaccine).

PREP Pearls
• Administration of live vaccines, such as measles-mumps-rubella (MMR) and varicella, is
recommended for children and adolescents living with HIV infection who have low-level
or no immunosuppression.
• Children living with HIV with high-level immunosuppression (defined as a CD4+ T
lymphocyte percentage of less than 15% in children aged 1 through 13 years, or a CD4+
T lymphocyte count of less than 200 cells/µL in adolescents aged 14 years and older)
must not receive measles-mumps-rubella (MMR) or varicella vaccines.
• Children with immunosuppression, including HIV infection, should receive killed
vaccines.
American Academy of Pediatrics 10
PREP® Self-Assessment PREPSA 2023

ABP Content Specifications(s)


• Plan an immunization regimen for a patient with an immune deficiency, including an
immune deficiency as a result of chemotherapy

Suggested Readings
• American Academy of Pediatrics. Human immunodeficiency virus infection. In:
Kimberlin DW, Barnett ED, Lynfield R, Sawyer MH, eds. Red Book: 2021-2024 Report
of the Committee on Infectious Diseases. 32nd ed. American Academy of Pediatrics;
2021. Accessed September 1, 2022. Red Book Online.
• American Academy of Pediatrics. Immunization and other considerations in
immunocompromised children. In: Kimberlin DW, Barnett ED, Lynfield R, Sawyer MH,
eds. Red Book: 2021-2024 Report of the Committee on Infectious Diseases. 32nd ed.
American Academy of Pediatrics; 2021. Accessed September 1, 2022. Red Book Online.
• Humiston SG, Atkinson WL, Rand C, Szilagyi PG. Immunizations. In: McInerny TK,
Adam HM, Campbell DE, DeWitt TG, Foy JM, Kamat DM, eds. American Academy of
Pediatrics Textbook of Pediatric Care. American Academy of Pediatrics; 2021:chap 20.
Accessed September 1, 2022. Pediatric Care Online.
• Rubin LG, Levin MJ, Ljungman P, et al. 2013 IDSA clinical practice guideline for
vaccination of the immunocompromised host. Clin Infect Dis. 2014;58(3):309-318.
doi:10.1093/cid/cit816.

American Academy of Pediatrics 11


PREP® Self-Assessment PREPSA 2023

Question 4
A 3-year-old boy is evaluated for a 1-month history of intermittent fevers up to 40°C, night
sweats, and 2.2-kg weight loss. He has no known sick contacts and has not traveled recently. The
boy is afebrile. He is irritable but consolable and appears well nourished and well developed.
There are multiple subcentimeter cervical lymph nodes palpable, more on the left side than the
right, and a 4 × 3–cm, firm, immobile, nontender, left-sided, supraclavicular lymph node. His
spleen is palpable 1 cm below the left costal margin. The remainder of his physical examination
findings are unremarkable.

Of the following, the BEST next step in this boy’s management is referral to a(n)
A. emergency department
B. gastroenterologist
C. infectious disease specialist
D. otolaryngologist

American Academy of Pediatrics 12


PREP® Self-Assessment PREPSA 2023

Correct Answer: D
The boy in the vignette has cervical and supraclavicular lymphadenopathy. Cervical
lymphadenopathy is very common and has many possible etiologies. Supraclavicular
lymphadenopathy, however, is highly concerning for malignancy (eg, lymphoma) and requires
further investigation with a lymph node biopsy. Of the response choices, an otolaryngologist is
best suited to perform a biopsy. Interventional radiologists and pediatric surgeons also perform
lymph node biopsies, but these specialties are not among the response choices.

Cervical lymphadenopathy is most often caused by a viral infection. This type of


lymphadenopathy is usually transient, and the lymph nodes are small, mobile, and nontender.
Occasionally, lymphadenitis may occur, leading to a larger lymph node that is usually tender.
Lymphadenitis is treated with antibiotics. Further investigation is indicated if the lymphadenitis
persists after treatment or worsens.

The investigation of lymphadenopathy should include (a):


• Complete blood count (CBC) with differential: The CBC may be normal in the setting of
a malignancy. Leukocytosis with a left shift may indicate a bacterial infection.
Lymphocytosis may indicate a viral infection. Some viral infections cause leukopenia and
lymphopenia (eg, HIV).
• Epstein-Barr virus and cytomegalovirus titers
• Erythrocyte sedimentation rate and C-reactive protein—elevation supports inflammation
or infection.
• Liver transaminases—elevation supports a viral infection or infiltrative process.
• Lactate dehydrogenase and uric acid levels—elevation indicates an increase in cell
turnover, which is associated with malignancy.

Imaging studies are helpful in the evaluation of lymphadenopathy. Ultrasonography can provide
valuable information about the size and architecture of the lymph nodes. Chest radiography may
identify a mediastinal mass. Computed tomography with contrast may identify an abscess.
Although consultation with other subspecialists may be helpful, the immediate priority is to
obtain a biopsy specimen for diagnostic evaluation by a pathologist. It is important to obtain
sufficient tissue for diagnosis from the most pathologic part of the lymph node. For this reason,
an excisional biopsy may be preferred over a fine-needle aspiration biopsy. This child’s clinical
condition does not warrant urgent evaluation or management in an emergency department.

PREP Pearls
• Supraclavicular lymphadenopathy is usually pathologic and requires further investigation
with a lymph node biopsy.
• Persistent or worsening lymphadenopathy requires further investigation.

ABP Content Specifications(s)


• Plan the appropriate diagnostic evaluation of unexplained lymphadenopathy Recognize
clinical findings associated with lymphoma

American Academy of Pediatrics 13


PREP® Self-Assessment PREPSA 2023

Suggested Readings
• Friedmann AM. Evaluation and management of lymphadenopathy in children. Pediatr
Rev. 2008;29(2):53-60. doi:10.1542/pir.29-2-53.
• Sahai S. Lymphadenopathy. Pediatr Rev. 2013;34(5):216-227. doi:10.1542/pir.34-5-216.
• Weinberg GA, Sefel GB, Hall CB. Lymphadenopathy. In: McInerny TK, Adam HM,
Campbell DE, DeWitt TG, Foy JM, Kamat DM, eds. American Academy of Pediatrics
Textbook of Pediatric Care. American Academy of Pediatrics; 2021:chap 175. Accessed
September 1, 2022. Pediatric Care Online.
• Weinstock MS, Patel NA, Smith LP. Pediatric cervical lymphadenopathy. Pediatr Rev.
2018;39(9):433-443. doi:10.1542/pir.2017-0249.

American Academy of Pediatrics 14


PREP® Self-Assessment PREPSA 2023

Question 5
A 2-year-old girl with a noncontributory medical history is brought to the emergency department
for intermittent abdominal pain over the past 12 hours. The pain, though progressively
worsening, waxes and wanes. She had 3 episodes of nonbloody, nonbilious vomiting today. Over
the past 6 hours, the girl has had a significantly decreased activity level and intermittent
irritability.

Her vital signs include a temperature of 37.9°C, heart rate of 130 beats/min, respiratory rate of
24 breaths/min, and blood pressure of 108/76 mm Hg. On physical examination, the girl appears
tired and cries intermittently while drawing her legs in toward her chest. She has right lower
quadrant tenderness without rebound or guarding. The remainder of her physical examination
findings are unremarkable.

Abdominal ultrasonography findings are shown in Item Q5.

Of the following, the BEST next step in this girl’s management is a(n)
A. air enema procedure
B. computed tomography of the abdomen
C. emergency laparotomy
D. upper gastrointestinal imaging with small bowel series

American Academy of Pediatrics 15


PREP® Self-Assessment PREPSA 2023

Correct Answer: A
The girl in the vignette has signs and symptoms concerning for intussusception. Abdominal
ultrasonography demonstrates the characteristic “target sign” (Item C5), also known as a “bull’s
eye” or “coiled spring,” which represents layering of intestine within the intestine. Because the
girl is hemodynamically stable, performing an air enema is the best next management step. An
air enema can be both diagnostic and therapeutic. If the air enema fails to reduce the
intussusception, an emergency laparotomy may be necessary. Surgical intervention is indicated
as the initial treatment if free air is visualized on imaging studies or there is evidence of
peritonitis on physical examination.

Although computed tomography (CT) of the abdomen can be used to diagnose intussusception, it
is not therapeutic and may require the child to be sedated. In addition, it exposes the child to
unnecessary and significant amounts of radiation. Computed tomography is typically reserved
for children with suspected intussusception in whom other imaging modalities fail to reveal a
diagnosis or to identify a pathological lead point cause for intussusception. Upper
gastrointestinal imaging with small bowel series under fluoroscopy is the study of choice for
suspected malrotation with midgut volvulus, but it would not be appropriate in this scenario.
Intussusception is the invagination of one part of the intestine into another part, most commonly
occurring at the ileocecal junction (90% of cases). It most often occurs in children aged 3 months
to 3 years; it is the most common cause of bowel obstruction in children younger than 2 years.
Common symptoms of intussusception include sudden onset of severe, colicky abdominal pain
that progressively worsens, irritability, intermittent inconsolable crying, emesis, lethargy, and
altered mental status. In some instances, the lethargy may be profound and episodic, and it may
be the only presenting symptom. Intussusception must always be included in the differential
diagnosis for a child with altered mental status.

Physical examination findings may include abdominal tenderness and a palpable, sausage-shaped
mass in the right lower quadrant (~50%). Although up to 50% of children with intussusception
have grossly bloody stools, the classic finding of “currant jelly” (blood mixed with mucus) stool
is rare. Currant-jelly stool is a late and ominous finding, indicating bowel ischemia and necrosis.
The classic triad of abdominal pain, currant-jelly stools, and a palpable abdominal mass is
present in less than 15% of children at the time of presentation.

American Academy of Pediatrics 16


PREP® Self-Assessment PREPSA 2023

PREP Pearls
• The classic triad of abdominal pain, currant-jelly stools, and a palpable abdominal mass
is present in less than 15% of children with intussusception.
• An air enema can be both diagnostic and therapeutic for intussusception.
• Extreme lethargy or altered mental status may be the only presenting symptom of
intussusception.

ABP Content Specifications(s)


• Recognize the clinical features associated with intussusception, and manage appropriately

Suggested Readings
• Baker RD. Acute abdominal pain. Pediatr Rev. 2018;39(3):130-139.
doi:10.1542/pir.2017-0089.
• Chiu L, Dudgeon D, Stallion A. Gastrointestinal obstruction. In: McInerny TK, Adam
HM, Campbell DE, DeWitt TG, Foy JM, Kamat DM, eds. American Academy of
Pediatrics Textbook of Pediatric Care. 2nd ed. American Academy of Pediatrics;
2021:chap 257. Accessed September 1, 2022. Pediatric Care Online.
• Gayle, T. Sudden altered mental status in an 11-month-old boy. Pediatr Rev.
2020;41(2):88-89. doi:10.1542/pir.2018-0108.

American Academy of Pediatrics 17


PREP® Self-Assessment PREPSA 2023

Question 6
A 2-year-old girl was seen for a routine health supervision visit 2 weeks ago. At that time, her
weight was inadvertently entered into the medical record in pounds but labeled as kilograms. The
girl’s mother called the advice line last night because her daughter had a fever and was provided
an acetaminophen dose based on the incorrect weight. The pediatrician noticed the error when
reviewing the chart this morning and called the girl’s mother for follow-up. The girl received 1
dose of acetaminophen since the overnight phone call, is feeling better, and is not exhibiting
negative consequences of the dosing error. The dose administered was well below a hepatotoxic
level.

Of the following, this event is BEST categorized as a/an


A. adverse event
B. medical error
C. non-preventable adverse event
D. sentinel event

American Academy of Pediatrics 18


PREP® Self-Assessment PREPSA 2023

Correct Answer: B
The girl in the vignette received a larger than appropriate dose of acetaminophen because of an
error in the weight documentation. As this occurrence was a preventable error that did not result
in patient harm, it is best classified as a medical error. The term adverse event implies that the
patient was harmed as a result of an intervention. A sentinel event is an error that leads to a
patient’s death or serious injury.

Careful study is needed to better prevent medical errors. In order to thoroughly study medical
error, it is important to have a universal terminology. A medical error is defined by the Institute
of Medicine as “the failure to complete a planned action or the initiation of a wrong plan to
achieve a specific goal.” An adverse event is an injury caused by the medical management as
opposed to the patient’s underlying disease process. An adverse event causes harm to the patient
whereas a medical error does not necessarily do so. Medication error is the most common type of
medical error and can occur anywhere along the path of a medication reaching a patient (eg,
prescribing, prescription-filling, administration). An adverse drug event is a medication error that
results in patient injury; an adverse drug event is the most common type of adverse event.

Adverse drug events can be further categorized as preventable, non-preventable, and potential. A
preventable adverse drug event is one that could have been avoided, while a non-preventable
adverse drug event could not have been avoided. For example, a patient is prescribed a
medication to which they have a known allergy (preventable) versus a patient with no known
allergy who develops an allergic reaction to a prescribed medication (nonpreventable). A
potential adverse drug event, also known as a “near miss” event, is a medication error that places
the patient at significant risk of injury but does not result in harm.

A sentinel event is an unexpected event that results in death or serious injury, or the risk of death
or serious injury.

Not all sentinel events are the result of a medical error. A serious adverse drug event may meet
criteria for a sentinel event.

PREP Pearls
• The Institute of Medicine defines a medical error as “the failure to complete a planned
action or the initiation of a wrong plan to achieve a specific goal.”
• An adverse event is an injury caused by the medical management as opposed to the
patient’s underlying disease process.
• A sentinel event is an unexpected event that results in death or serious injury, or the risk
of death or serious injury.

ABP Content Specifications(s)


• Understand and apply the definition of a medical error
• Understand and apply the definition of a near-miss event
• Understand and apply the definition of a sentinel event
• Understand and apply the definition of a preventable adverse event
American Academy of Pediatrics 19
PREP® Self-Assessment PREPSA 2023

• Understand and apply the definition of a non-preventable adverse event

Suggested Readings
• Agency for Healthcare Research and Quality. Adverse events, near misses and errors.
Patient Safety Network. September 7, 2019. Accessed September 1, 2022.
https://psnet.ahrq.gov/primer/adverse-events-near-misses-and-errors.
• Institute of Medicine (US) Committee on Quality of Health Care in America, Kohn LT,
Corrigan JM, Donaldson MS, eds. To Err is Human: Building a Safer Health System.
National Academies Press (US); 2000. doi:10.17226/9728.
• Leonard, MS. Patient safety and quality improvement: medical errors and adverse events.
Pediatr Rev. 2010;31(4):151158. doi:10.1542/pir.31-4-151.
• Neuspiel DR. Medical errors, adverse events, and patient safety. In: McInerny TK, Adam
HM, Campbell DE, DeWitt TG, Foy JM, Kamat DM, eds. American Academy of
Pediatrics Textbook of Pediatric Care. American Academy of Pediatrics; 2021:chap 288.
Accessed September 1, 2022. Pediatric Care Online.

American Academy of Pediatrics 20


PREP® Self-Assessment PREPSA 2023

Question 7
A 3-month-old male infant born at term is brought to the emergency department via ambulance
for a seizure. Initial evaluation shows an ionized calcium level of 3.3 mg/dL (0.8 mmol/L)
(reference range, 4.5-5.3 mg/dL [1.1-1.3 mmol/L]). He is treated with intravenous calcium,
which aborts the seizure. The infant’s parents report that he has been jittery and irritable for the
past few days. He exclusively breastfed for the first 2 months after birth. Due to fussiness, his
parents recently started making formula using a recipe found on the internet containing hemp
seed hearts, coconut water, dates, and sea moss. The infant was recently diagnosed with
laryngomalacia after an evaluation for noisy breathing. His physical examination findings are
normal for age. His parents are of normal stature.

Laboratory evaluation drawn prior to treatment reveals the following:


Laboratory Test Result
Total calcium 4.5 mg/dL (1.1 mmol/L) (reference range, 9-11
mg/dL [2.2-2.8 mmol/L])
Phosphorous 1.2 mg/dL (reference range, 2.7-4.5 mg/dL)
Magnesium 2.1 mg/dL (0.9 mmol/L) (1.6-2.6 mg/dL [0.7-
1.1 mmol/L])
Parathyroid hormone 534 pg/mL (reference range, 10-65 pg/mL)
Alkaline phosphatase 1,021 U/L (reference range, 146-477 U/L)
25-hydroxyvitamin D Pending
1,25-dihydroxyvitamin D Pending

Of the following, the BEST next step in this infant’s management is oral administration of
A. calcium, cholecalciferol, and calcitriol
B. calcium and phosphorus
C. magnesium
D. phosphorus and calcitriol

American Academy of Pediatrics 21


PREP® Self-Assessment PREPSA 2023

Correct Answer: A
The infant in the vignette has hypocalcemia due to severe vitamin D deficiency. His noisy
breathing and seizure are manifestations of hypocalcemia. After treating his symptomatic
hypocalcemia with intravenous calcium, the infant should be treated with oral calcium and
cholecalciferol (dietary vitamin D3). Calcitriol (1,25-dihydroxyvitamin D, the
active form of vitamin D) should be added given the severity of his hypocalcemia. Calcitriol acts
immediately to absorb calcium from the intestine while vitamin D stores are replenished by
cholecalciferol. The infant should also be switched to a cow milk–based formula, given its higher
vitamin D content, once he can safely drink from a bottle.

Breast milk does not provide adequate vitamin D; infants who are exclusively or predominantly
breastfed must receive supplementation with this vitamin. Although commercial formulas are
fortified with vitamin D, homemade formulas, including the one described in the vignette, do not
contain additional vitamin D. The American Academy of Pediatrics recommends that breastfed
infants receive at least 400 IU of supplemental vitamin D daily.

Vitamin D is a steroid hormone that is integral in the regulation of calcium and phosphorus
homeostasis. Item C7A outlines vitamin D metabolism and action. Laboratory findings depend
on the severity and duration of the vitamin D deficiency (Item C7B). In mild vitamin D
deficiency, relative hypocalcemia will result in a rise in parathyroid hormone (PTH) to maintain
serum calcium in the normal range. Calcium is reabsorbed from the kidney, absorbed from the
gut (via conversion of remaining 25-hydroxyvitamin D stores to 1,25-dihydroxyvitamin D), and
released from the bones. As vitamin D deficiency becomes more severe and PTH rises higher,
calcium is depleted and levels begin to fall. Increased PTH levels cause phosphate wasting in the
kidney, resulting in low serum phosphorus levels. Parathyroid hormone has an indirect effect on
osteoclasts, resulting in increased bone resorption. Alkaline phosphatase levels rise as bone
turnover is increased. When 25-hydroxyvitamin D stores are extremely low, there is inadequate
vitamin D available for conversion to 1,25-dihydroxyvitamin D, which results in low levels of
this hormone. The infant in the vignette is expected to have low 25-hydroxyvitamin D and 1,25-
dihydroxyvitamin D levels.

Item C7B

American Academy of Pediatrics 22


PREP® Self-Assessment PREPSA 2023

Item C7A

Manifestations of severe vitamin D deficiency include rickets (widening of the wrists, rachitic
rosary, craniotabes) and symptoms of hypocalcemia. Infants may display irritability,
constipation, noisy breathing due to laryngospasm, and seizures. Older children may experience
paresthesias.

Treatment with phosphorus and calcitriol is indicated for hypophosphatemia due to defects in the
fibroblast growth factor 23 (FGF23) pathway, usually due to mutations in the PHEX gene.
Fibroblast growth factor 23 is the main hormone responsible for phosphorus wasting in the
kidney. Burosumab, a monoclonal antibody that inhibits FGF23, is approved by the US Food and
Drug Administration for the treatment of children aged 6 months and older with X-linked
hypophosphatemia. Although the infant in the vignette is hypophosphatemic, this presentation is
due to secondary hyperparathyroidism and the phosphate wasting effect that the elevated PTH
level has on the kidney. Additionally, both parents are of normal stature, which is evidence
against familial hypophosphatemia, an X-linked dominant disorder that negatively impacts
growth.

Magnesium is required for a normal PTH response to low calcium levels. In children with
hypomagnesemia, hypocalcemia may develop due to inadequate PTH action. In this case, the
PTH level would be inappropriately low. This situation commonly occurs in infants of diabetic
mothers.
American Academy of Pediatrics 23
PREP® Self-Assessment PREPSA 2023

Calcium and phosphorus is the treatment for metabolic bone disease of prematurity, which
occurs due to a deficiency of these minerals. The infant in the vignette was born at term, so this
would be an unlikely diagnosis. In addition, severe hypocalcemia is an unlikely finding in
metabolic bone disease of prematurity.

PREP Pearls
• The American Academy of Pediatrics recommends supplementation of 400 IU daily of
vitamin D for exclusively breastfed infants.
• Manifestations of severe vitamin D deficiency may include rickets (widening of the
wrists, rachitic rosary, craniotabes) and hypocalcemia with secondary
hyperparathyroidism.
• Homemade infant formulas may contain or lack ingredients that may affect mineral
homeostasis.

ABP Content Specifications(s)


• Recognize the effects of vitamin D deficiency in patients of various ages, including those
who are breast-fed

Suggested Readings
• Dawodu A, Wagner CL. Vitamin D inadequacy. In: McInerny TK, Adam HM, Campbell
DE, DeWitt TG, Foy JM, Kamat DM, eds. American Academy of Pediatrics Textbook of
Pediatric Care. American Academy of Pediatrics; 2021:chap 346. Accessed September 1,
2022. Pediatric Care Online.
• Holick MF, Binkley NC, Bischoff-Ferrari HA, et al; Endocrine Society. Evaluation,
treatment, and prevention of vitamin D deficiency: an Endocrine Society clinical practice
guideline. J Clin Endocrinol Metab. 2011;96(7):1911-30. doi:10.1210/jc.2011-0385.
• Misra M, Pacaud D, Petryk A, Collett-Solberg PF, Kappy M. Vitamin D deficiency in
children and its management: review of current knowledge and recommendations.
Pediatrics. 2008; 122(2):398-417. doi:10.1542/peds.2007-1894.
• Munns CF, Shaw N, Kiely M, et al. Global consensus recommendations on prevention
and management of nutritional rickets. J Clin Endocrinol Metab. 2016;101(2):394-415.
doi:10.1210/jc.2015-2175.
• Vieira MA, Kube PK, van Helmond JL, et al. Recipe for disaster: homemade formula
leading to severe complications in 2 infants. Pediatrics. 2021;148(3):e2021050947.
doi:10.1542/peds.2021-050947.
• Wagner CL, Greer FR; American Academy of Pediatrics Section on Breastfeeding;
American Academy of Pediatrics Committee on Nutrition. Prevention of rickets and
vitamin D deficiency in infants, children, and adolescents. Pediatrics. 2008;122(5):1142-
1152. doi:10.1542/peds.2008-1862.

American Academy of Pediatrics 24


PREP® Self-Assessment PREPSA 2023

Question 8
A 12-day-old male neonate in the neonatal intensive care unit develops apnea, bradycardia, and
oxygen desaturation requiring an increase in continuous positive airway pressure ventilation
(CPAP) from 6 to 8 cm H2O, and fraction of inspired oxygen (FiO2) from 0.23 to 0.40. His mean
upper limb cuff blood pressure has ranged from 18 to 25 mm Hg for the past hour. The neonate
was born at 26 weeks’ gestation with a birthweight of 940 g. Delivery was by cesarean section
due to maternal preeclampsia with rupture of membranes at delivery. The neonate has been
tolerating nasogastric feedings of 20 calorie/oz breast milk at 80 mL/kg/day and receiving
parenteral nutrition through a central intravenous line.

Laboratory data are shown:


Laboratory Test Result
Arterial blood gas base deficit 10
White blood cell count 22,000/µL (22.0 × 109/L)
Neutrophils 40%
Bands 25%
Lymphocytes 20%
Monocytes 11%
Eosinophils 4%
Hemoglobin 10.8 g/dL (108 g/L)
Hematocrit 30.5%
Platelet count 87 × 103/µL (87 × 109/L)

Blood cultures are pending.

Of the following, the BEST treatment for this neonate is intravenous


A. ampicillin and cefotaxime
B. ampicillin and gentamicin
C. meropenem and oxacillin
D. vancomycin and gentamicin

American Academy of Pediatrics 25


PREP® Self-Assessment PREPSA 2023

Correct Answer: D
The preterm neonate in the vignette has signs and symptoms of late-onset sepsis with clinical
deterioration and an elevated white blood cell count with a left shift. The most likely causative
organisms are coagulase-negative staphylococci and gram-negative bacteria. Of the response
choices, the combination of vancomycin (effective against coagulase-negative staphylococci)
and gentamicin (effective against gram-negative bacteria such as Escherichia coli and Klebsiella
pneumoniae) is the best treatment pending blood culture results.

Ampicillin, gentamicin, cefotaxime, and meropenem are all effective against gram-negative
bacteria, but they are not effective against coagulase-negative staphylococci. Oxacillin is also not
effective against coagulase-negative staphylococci. Ampicillin and gentamicin is the empiric
antibiotic combination of choice in early-onset sepsis and is effective against the common
bacterial causes of early onset sepsis (eg, group B Streptococcus, E coli, and Listeria
monocytogenes).

This neonate’s risk factors for sepsis include extreme prematurity, presence of an endotracheal
tube and central line, and parenteral nutrition. Extremely preterm neonates in the neonatal
intensive care unit are at increased risk for infection due to immature defense barriers (eg, skin
and mucous membranes), an immature immune system, invasive interventions (eg, mechanical
ventilation, intravascular catheters, and parenteral nutrition), and comorbidities (eg, patent ductus
arteriosus, necrotizing enterocolitis, and chronic lung disease). The more premature the neonate,
the higher the risk of infection.

Sepsis should always be considered as a cause of clinical deterioration in a preterm neonate.


Neonatal sepsis is classified as early-or late-onset. Early-onset sepsis is defined as sepsis in the
first week after birth, and late-onset sepsis occurs 7 or more days after birth. Early-onset sepsis is
most often caused by maternal vertical transmission of microorganisms; late-onset sepsis is most
often postnatally acquired.

PREP Pearls
• Risk factors for late-onset sepsis in preterm neonates include immature defense barriers,
an immature immune system, invasive interventions, and comorbidities of prematurity.
• The choice of empiric antibiotics for late-onset sepsis in a preterm neonate should be
based on the most likely causative organisms, which include coagulase-negative
staphylococci and gram-negative bacteria.

ABP Content Specifications(s)


• Plan appropriate antimicrobial therapy for suspected sepsis in the immediate newborn
period

Suggested Readings
• Bentlin MR, Rugolo, LMSS. Late-onset sepsis: epidemiology, evaluation and outcome.
NeoReviews. 2010;11(8):e426e435. doi:10.1542/neo.11-8-e426.

American Academy of Pediatrics 26


PREP® Self-Assessment PREPSA 2023

• Camacho-Gonzalez A, Spearman PW, Stoll BJ. Neonatal infectious diseases: evaluation


of neonatal sepsis. Pediatr Clin North Am. 2013;60(2):367-389.
doi:10.1016/j.pcl.2012.12.003.
• Chu A, Hageman JR, Schreiber M, Alexander K. Antimicrobial therapy and late-onset
sepsis. NeoReviews. 2012;13(2):e94-e102. doi:10.1542/neo.13-2-e94.
• Kojaoghlanian T. The newborn at risk of infection. In: McInerny TK, Adam HM,
Campbell DE, DeWitt TG, Foy JM, Kamat DM, eds. American Academy of Pediatrics
Textbook of Pediatric Care. American Academy of Pediatrics; 2021:chap 102. Accessed
September 1, 2022. Pediatric Care Online.
• Puopolo KM, Benitz WE, Zaoutis TE; Committee on Fetus and Newborn, Committee on
Infectious Diseases. Management of neonates born at less than or equal to 34 6/7 weeks'
gestation with suspected or proven early-onset bacterial sepsis. Pediatrics.
2018;142(6):e20182896. doi:10.1542/peds.2018-2896.

American Academy of Pediatrics 27


PREP® Self-Assessment PREPSA 2023

Question 9
The mother of a 10-year-old girl with cystic fibrosis (CF), jointly managed by her pediatrician
and a multidisciplinary CF center at a nearby children’s hospital, calls for advice about a positive
airway culture. The girl was diagnosed with CF after a positive newborn screening result. She
has been very healthy and has required no aggressive intervention for pulmonary infection. Her
routine airway cultures, obtained in the CF center by deep throat/gag swab, have always grown
normal flora or methicillin-sensitive Staphylococcus aureus. Her most recent culture was positive
for Pseudomonas aeruginosa. The girl has no symptoms of illness. She is eating well and has had
no weight loss. She is attending school regularly.

Of the following, the MOST appropriate management strategy in response to this girl’s
laboratory finding is
A. hospitalization for intravenous antibiotic administration
B. inhaled antibiotic treatment for eradication of the identified organism
C. no intervention while she is asymptomatic
D. repeat airway culture before implementing treatment

American Academy of Pediatrics 28


PREP® Self-Assessment PREPSA 2023

Correct Answer: B
The most appropriate management strategy for the girl in the vignette, who is experiencing her
first acquisition of Pseudomonas aeruginosa infection, is an inhaled anti-pseudomonal antibiotic.
Multiple studies demonstrate that proactive treatment to eradicate Pseudomonas from the airway
in children with cystic fibrosis (CF) helps preserve lung function. A single 28-day treatment
course of inhaled tobramycin given twice daily, with follow-up culture to document resolution
and re-treatment if the culture is still positive, is the standard of care in the United States. The
addition of oral antibiotics for acute eradication has not been shown to be more effective than an
inhaled antibiotic alone. However, concurrent oral azithromycin has been demonstrated to delay
symptomatic pulmonary exacerbation of cystic fibrosis in children with first acquisition of
Pseudomonas. There are no data to support hospitalization for treatment with intravenous
antibiotics as more effective than treatment with an inhaled antibiotic alone for eradication of
Pseudomonas in a child with asymptomatic first acquisition.

Because longitudinal epidemiologic studies demonstrate that acquisition of P aeruginosa as a


chronic colonizing agent is associated with more rapid decline in lung function, quarterly airway
cultures are the standard of care for individuals with CF. Even though the girl in the vignette is
asymptomatic, she should be treated for eradication promptly on identification of P aeruginosa in
her respiratory culture. There is no need to repeat the culture before treatment, but it should be
repeated after the treatment to document eradication or determine the need for retreatment. There
are no clear criteria defining the number of positive cultures required to diagnose chronic P
aeruginosa colonization, for which treatment includes long-term, alternate-month, inhaled
tobramycin. Nor are there clear guidelines for when to stop long-term treatment after some
period of negative cultures. With the advent of highly effective modulator therapies, the
incidence of P aeruginosa infection in individuals with CF has decreased, and many of those who
had been chronically colonized become decolonized. How, if, and when to stop chronic inhaled
antibiotic treatment in this population is unclear and requires ongoing investigation.

Chronic lung disease with recurrent bronchitis leading to bronchiectasis is the most common
respiratory complication of CF. The changing pattern of respiratory microorganisms cultured
from individuals with CF over the years is shown in Item C9. Chronic colonization with
methicillin-resistant Staphylococcus aureus (MRSA) has also been associated with a decline in
lung function in individuals with CF. Several studies have investigated the eradication of MRSA
airway infection, but there is no current widely accepted practice guideline recommending a
standardized approach.

American Academy of Pediatrics 29


PREP® Self-Assessment PREPSA 2023

Exocrine pancreatic insufficiency leading to malabsorption and malnutrition is the most common
gastrointestinal complication of CF, present in about 85% of affected individuals. As children
with CF age into adolescence and adulthood, the incidence of endocrine pancreatic dysfunction
increases and manifests as CF-related diabetes (CFRD). Although insulin is usually required for
management, CFRD differs from both type 1 and type 2 diabetes. Calorie restriction is not
appropriate in the management of CFRD, and weight loss does not improve outcomes. There is
no genetic predisposition to CFRD. However, some individuals with CF carry genetic modifiers
that have overlap with those of type 2 diabetes. Cystic fibrosis manifestations and complications
can affect the respiratory, gastrointestinal, and reproductive systems.

PREP Pearls
• Chronic colonization with Pseudomonas aeruginosa and/or methicillin-resistant
Staphylococcus aureus is associated with a decline in lung function in children and
adolescents with cystic fibrosis.
• The most appropriate treatment of first acquisition of Pseudomonas in an individual with
cystic fibrosis is 28 days of inhaled antipseudomonal antibiotic, usually tobramycin.
• Cystic fibrosis-related diabetes is a later-onset complication of progressive pancreatic
fibrosis.

ABP Content Specifications(s)


• Recognize the pulmonary and extrapulmonary complications of cystic fibrosis in children
of various ages
• Recognize the pathogens commonly associated with the pulmonary complications of
cystic fibrosis
American Academy of Pediatrics 30
PREP® Self-Assessment PREPSA 2023

• Identify the age-related gastrointestinal and hepatobiliary features in a patient with cystic
fibrosis

Suggested Readings
• Dasenbrook EC, Checkley W, Mero CAl, et al. Association between respiratory tract
methicillin-resistant Staphylococcus aureus and survival in cystic fibrosis. JAMA.
2010;303(23):2386-2392. doi:10.1001/jama.2010.791.
• Jackson L, Waters V. Factors influencing the acquisition and eradication of early
Pseudomonas aeruginosa infection in cystic fibrosis. J Cyst Fibros. 2021;20(1):8-16.
doi:10.1016/j.jcf.2020.10.008.
• Mogayzel PJ Jr, Naureckas ET, Robinson KA, et al; Cystic Fibrosis Foundation
Pulmonary Clinical Practice Guidelines Committee. Cystic Fibrosis Foundation
pulmonary guideline: pharmacologic approaches to prevention and eradication of initial
Pseudomonas aeruginosa infection. Ann Am Thorac Soc. 2014;11(10):1640-1650.
doi:10.1513/AnnalsATS.201404166OC.
• Paranjape SM, Mogayzel PJ Jr. Cystic fibrosis. Pediatr Rev. 2014;35(5):194-205.
doi:10.1542/pir.35-5-194.
• Willey-Courand DB, Marshall BC. Cystic fibrosis. In: McInerny TK, Adam HM,
Campbell DE, DeWitt TG, Foy JM, Kamat DM, eds. American Academy of Pediatrics
Textbook of Pediatric Care. American Academy of Pediatrics; 2021:chap 239. Accessed
September 1, 2022. Pediatric Care Online.

American Academy of Pediatrics 31


PREP® Self-Assessment PREPSA 2023

Question 10
A 5-year-old girl is seen for a health supervision visit. She attends kindergarten and is doing very
well. Her medical history is significant for recurrent joint subluxations of the hips, shoulders,
knees, and elbows bilaterally, easy bruising, and poor wound healing often requiring placement
of sutures to achieve closure. The family history is significant for similar findings in her father
and grandfather. The girl’s father had a recurrent incisional hernia at an appendectomy site.
There is no family history of arterial aneurysm or rupture. On physical examination, there are
bruises on her lower extremities, widened atrophic scars with evidence of previous sutures (Item
Q10), and skin and joint hypermobility. The remainder of her physical examination findings are
normal.

Of the following, the girl’s MOST likely diagnosis is


A. Ehlers-Danlos syndrome
B. homocystinuria
C. Loeys-Dietz syndrome
D. Marfan syndrome

American Academy of Pediatrics 32


PREP® Self-Assessment PREPSA 2023

Correct Answer: A
The girl in the vignette has classical Ehlers-Danlos syndrome (cEDS). Ehlers-Danlos syndrome
is a group of inherited connective tissue disorders. Cardinal features of EDS include joint
hypermobility, skin hyperextensibility, and tissue fragility. The 2017 international classification
describes 13 subtypes of EDS. The most common forms are classical, vascular, and hypermobile.
There is wide phenotypic variability and genetic heterogeneity among the different EDS
subtypes. The underlying genetic etiology is known for all subtypes of EDS except hypermobile
EDS. Classical Ehlers-Danlos syndrome is inherited in an autosomal dominant manner. The
clinical diagnosis of cEDS is made based on the presence of major and minor criteria.

Major criteria include:


• Skin hyperextensibility: assessed by pinching and lifting skin on the volar surface at the
middle of the nondominant forearm. Skin is called hyperextensible if it can be stretched
to 1.5 cm at the forearm and dorsum of hands, and to 3 cm for neck, elbows, and knees.
• Atrophic scarring
• Generalized joint hypermobility: assessed using the Beighton criteria for joint
hypermobility. Joint hypermobility is defined as a score of 5 or above with a total score
of 9 (Item C10A).

Minor criteria include:


• Easy bruising
• Epicanthal folds
• Family history (first-degree relative meeting clinical criteria)
• Hernia
• Joint hypermobility-related complications (eg, sprains, subluxation, pain, flexible
flatfoot)
• Molluscoid pseudotumors (fleshy lesions associated with scars over pressure points such
as the elbows and knees)
• Skin fragility
• Soft, doughy skin
American Academy of Pediatrics 33
PREP® Self-Assessment PREPSA 2023

• Subcutaneous spheroids (mobile spherical hard bodies that are palpable on the forearms
and shins)

The minimal clinical diagnostic criteria suggestive of the diagnosis of cEDS include:
• Presence of skin hyperextensibility and atrophic scarring; plus
• Generalized joint hypermobility; or
• At least 3 minor criteria

The diagnosis of cEDS is confirmed by the presence of minimal clinical criteria and the
identification of a pathogenic variant in one of the genes known to be associated with cEDS
(COL5A1, COL5A2, COL1A1). Disorders of connective tissue can have considerable clinical
overlap. While the absence of a pathogenic variant in one of the above genes does not exclude
the diagnosis of cEDS, it suggests consideration of another underlying etiology.

Evaluation and management for the established diagnosis of cEDS are listed in Item C10B.

Homocystinuria is characterized by the following features:


• Central nervous system: developmental delay, intellectual disability
• Eyes: ectopia lentis, severe myopia
• Vascular system: thromboembolic episodes
• Skeletal: tall stature, long limbs, scoliosis, pectus excavatum

Homocystinura is inherited in an autosomal recessive manner. Atrophic scars, easy bruising, and
joint hypermobility are not features of homocystinuria. Developmental delay is a universal
finding, often the first presenting feature; this is not seen in the girl in the vignette.

Clinical features of Loeys-Dietz syndrome are described below:


• Craniofacial: hypertelorism, bifid uvula/cleft palate, strabismus, craniosynostosis
• Cutaneous: velvety, translucent skin, easy bruising, dystrophic scars
• Skeletal: pectus excavatum/carinatum, scoliosis, joint hypermobility, arachnodactyly,
cervical spine malformation, clubfeet
American Academy of Pediatrics 34
PREP® Self-Assessment PREPSA 2023

• Vascular: cerebral, thoracic, and abdominal arterial aneurysms and/or dissections

Affected individuals are at increased risk of complications related to arterial aneurysms. While
there is some clinical overlap with EDS, distinguishing features of Loeys-Dietz syndrome
include craniofacial anomalies, pectus deformity, arachnodactyly, spine anomalies, and arterial
aneurysms.

Clinical features of Marfan syndrome are described below:


• Cardiovascular: aortic dilatation, mitral valve prolapse, tricuspid valve prolapse,
enlargement of the proximal pulmonary artery
• Ocular: myopia, ectopia lentis, risk for retinal detachment
• Skeletal: pectus excavatum/carinatum, tall stature, scoliosis, arachnodactyly,
disproportionately long extremities for the length of the trunk

Classical EDS is distinguished from Marfan syndrome by the presence of atrophic scars, skin,
and joint hyperextensibility.

PREP Pearls
• Cardinal features of Ehlers-Danlos syndromes include joint hypermobility, skin
hyperextensibility, and tissue fragility.
• In addition to the minimal clinical criteria, the diagnosis of classical Ehlers-Danlos
syndrome requires the identification of a pathogenic variant in one of the known
associated genes (COL5A1, COL5A2, COL1A1).
• Evaluation and management of classical Ehlers-Danlos syndrome includes assessment of
clotting parameters, baseline 2D echocardiography, evaluation and therapy for hypotonia
and gross motor delay, and management of pain and wound healing.

ABP Content Specifications(s)


• Recognize the clinical findings associated with Ehlers-Danlos syndrome

American Academy of Pediatrics 35


PREP® Self-Assessment PREPSA 2023

Suggested Readings
• Bowen JM, Sobey GJ, Burrows NP, et al. Ehlers-Danlos syndrome, classical type. Am J
Med Genet C Semin Med Genet. 2017;175(1):27-39. doi:10.1002/ajmg.c.31548.
• Dietz H. FBN1-related Marfan syndrome. GeneReviews. GeneReviews [Internet].
University of Washington; 2021. Accessed September 1, 2022.
https://www.ncbi.nlm.nih.gov/books/NBK1335/.
• Malfait F, Francomano C, Byers P, et al. The 2017 international classification of the
Ehlers-Danlos syndromes. Am J Med Genet C Semin Med Genet. 2017;175(1):8-26.
doi:10.1002/ajmg.c.31552.
• Malfait F, Wenstrup R, De Paepe A. Classic Ehlers-Danlos syndrome. GeneReviews
[Internet]. University of Washington; 2021. Accessed September 1, 2022.
https://www.ncbi.nlm.nih.gov/books/NBK1244/.
• Siegel DM, Marston B. Joint pain. In: McInerny TK, Adam HM, Campbell DE, DeWitt
TG, Foy JM, Kamat DM, eds. American Academy of Pediatrics Textbook of Pediatric
Care. American Academy of Pediatrics; 2021:chap 171. Accessed September 1, 2022.
Pediatric Care Online .

American Academy of Pediatrics 36


PREP® Self-Assessment PREPSA 2023

Question 11
A 7-year-old boy is seen for a health supervision visit after being lost to follow-up. His mother
reports that he has received special education services since kindergarten and is in a “separate
classroom.” She is unsure of his special education classification, but states that “he acts just like
my 3-year-old.” The boy was delayed with toilet training, and this past year learned to dress
himself. He only recently began identifying colors and does not know the alphabet.

During the physical examination, the boy is eager for attention and shows the examiner an action
figure he brought with him. He can provide his name and age, and speaks in 3-word sentences.
He shakes his head “no” or shrugs when asked open-ended questions, and maintains eye contact.
The remainder of the boy’s physical examination findings are normal. After the physical
examination is completed, he gestures to request the stethoscope so that he can listen to his
action figure.

Of the following, this boy’s MOST likely diagnosis is


A. autism spectrum disorder
B. hearing impairment
C. intellectual disability
D. language disorder

American Academy of Pediatrics 37


PREP® Self-Assessment PREPSA 2023

Correct Answer: C
The boy in the vignette is demonstrating impairments in cognitive and adaptive skills, consistent
with a diagnosis of intellectual disability. The boy’s behavioral and developmental history, along
with the speech, language, and social skills he demonstrates during the visit, are at a 3-year-old
level. The physician should confirm this diagnosis and its severity by reviewing the
neuropsychological testing report from the school.

Autism spectrum disorder is defined by social communication deficits including difficulties with
non-verbal communication. Repetitive and restricted behaviors are also required for a diagnosis
of autism. The boy in the vignette makes eye contact, uses gestures to support his
communication, and demonstrates social reciprocity by showing and sharing objects.
Additionally, by observation and parent report, he does not have repetitive or restricted behaviors
or interests. The boy’s social and play skills are at the same level as his cognitive and adaptive
skills, as expected in a child with intellectual disability. Although intellectual disability co-occurs
in about 30% of individuals with autism spectrum disorder, those with autism have more
prominent social communication deficits with social functioning that is lower than expected for
their cognitive level.

When assessing a child with language delays, including those with suspected autism spectrum
disorder, hearing impairment must be considered; formal audiology evaluation should be
completed. However, hearing impairment alone would not account for the cognitive and adaptive
impairments exhibited by the boy in the vignette. Similarly, a diagnosis of language disorder
does not explain the impairment in non-verbal and adaptive skills exhibited by this boy.

PREP Pearls
• Intellectual disability co-occurs in about 30% of individuals with autism spectrum
disorder. Individuals with both autism spectrum disorder and intellectual disability have
more prominent social communication deficits with social functioning that is lower than
expected for their cognitive level.
• Formal audiology evaluation is a crucial part of the initial evaluation for children with
language delays or concerns for autism spectrum disorder.

ABP Content Specifications(s)


• Distinguish findings associated with autism spectrum disorder from those of an
intellectual disability
• Distinguish findings associated with autism spectrum disorder from those of profound
hearing loss

American Academy of Pediatrics 38


PREP® Self-Assessment PREPSA 2023

Suggested Readings
• Hyman SL, Levy SE, Myers SM; Council on Children With Disabilities, Section on
Developmental and Behavioral Pediatrics. Identification, evaluation, and management of
children with autism spectrum disorder. Pediatrics. 2020;145(1):e20193447.
doi:10.1542/peds.2019-3447.
• Kryszak E, Mulick JA, Butter EM. Autism spectrum disorder. In: McInerny TK, Adam
HM, Campbell DE, DeWitt TG, Foy JM, Kamat DM, eds. American Academy of
Pediatrics Textbook of Pediatric Care. American Academy of Pediatrics; 2021:chap 221.
Accessed September 1, 2022. Pediatric Care Online.
• Long M, Register-Brown K. Autism spectrum disorder. Pediatr Rev. 2021;42(7):360-374.
doi:10.1542/pir.2020-000547.
• Phelps RA, Cohen WI. Intellectual disability. In: McInerny TK, Adam HM, Campbell
DE, DeWitt TG, Foy JM, Kamat DM, eds. American Academy of Pediatrics Textbook of
Pediatric Care. American Academy of Pediatrics; 2021:chap 278. Accessed September 1,
2022. Pediatric Care Online.
• Purugganan O. Intellectual disabilities. Pediatr Rev. 2018;39(6):299-309.
doi:10.1542/pir.2016-0116.

American Academy of Pediatrics 39


PREP® Self-Assessment PREPSA 2023

Question 12
A 16-year-old adolescent boy is seen as a new patient to initiate care and discuss his attention-
deficit/hyperactivity disorder (ADHD). He takes amphetamine-dextroamphetamine 30 mg each
morning and short-acting amphetaminedextroamphetamine 5 mg each afternoon. He was due for
his annual health supervision visit and medication refill visit 4 months ago, but his mother
recently had knee surgery and delayed bringing him in for his scheduled visit. The boy reports
weekly marijuana use on a substance use screening tool.

Of the following, this adolescent’s MOST likely source for nonmedical prescription medication
use would be
A. buying from a drug dealer
B. his mother’s medication
C. his own medication
D. stealing from a health care professional

American Academy of Pediatrics 40


PREP® Self-Assessment PREPSA 2023

Correct Answer: B
The adolescent in the vignette is most likely to participate in nonmedical use of prescription
medication obtained from his family member (ie, his mother). He is less likely to use his own
medication for nonmedical reasons, steal the prescription or medication from a health care
professional, or buy it from a drug dealer.

Pediatricians have the important task of providing anticipatory guidance around safe medication
use, storage, and disposal. According to the most recent National Survey on Drug Use and
Health (2019), nonmedical use of prescription pain relievers and benzodiazepines has declined
slightly among people aged 12 years and older (Item C12). Of those surveyed, 3.5% had used
prescription pain medication for a nonmedical reason at least once in the past year. Of the 9.7
million people aged 12 years or older who used a prescription pain reliever in an inappropriate
manner, over half of them reported obtaining the prescription medication from a friend or family
member.

Approximately one-third inappropriately used a pain reliever medication that had been
appropriately prescribed by their own health care professional. Of those individuals reporting
nonmedical use of a pain reliever in the past year, 6.2% acquired the medication from a drug
dealer.

Prevention of nonmedical prescription drug use is an important part of adolescent health care.
Physicians should counsel parents regarding the safe storage of all controlled substance
prescription medications in a lock box or other secure location. These medications should be
administered to the adolescent by a parent or guardian to ensure appropriate usage. Counseling
around safe medication use, storage, and disposal should be provided for all family members
(including grandparents and those outside the direct household). Health care professionals should
be aware of their own community’s resources for controlled substance (eg, opioids) disposal
such as pharmacy takeback days and at-home medication disposal options.

American Academy of Pediatrics 41


PREP® Self-Assessment PREPSA 2023

PREP Pearls
• Adolescents participating in nonmedical prescription drug use are most likely to obtain
them from a friend or family
• member.
• Counseling around safe medication use, storage, and disposal for all family members is
recommended.
• Health care professionals should be knowledgeable about their local safe medication
disposal options.

ABP Content Specifications(s)


• Recognize the risk of misuse of prescription medications

Suggested Readings
• Levy S, Bagley S. Substance use: initial approach in primary care. In: McInerny TK,
Adam HM, Campbell DE, DeWitt TG, Foy JM, Kamat DM, eds. American Academy of
Pediatrics Textbook of Pediatric Care. American Academy of Pediatrics; 2021:chap 239.
AccessedSeptember 1, 2022.Pediatric Care Online .
• Levy SJ, Williams JF; Committee on Substance Use and Prevention. Substance use
screening, brief intervention, and referral to treatment. Pediatrics.
2016;138(1):e20161211. doi:10.1542/peds.2016-1211.
• Substance Abuse and Mental Health Services Administration. (2020). Key substance use
and mental health indicators in the United States: results from the 2019 National Survey
on Drug Use and Health 2020. Accessed September 1, 2022.
• https://www.samhsa.gov/data/sites/default/files/reports/rpt29393/2019NSDUHFFRPDF
WHTML/2019NSDUHFFR090120.htm#topofpage.

American Academy of Pediatrics 42


PREP® Self-Assessment PREPSA 2023

Question 13
A 4-year-old girl is evaluated in the emergency department for 6 days of bloody diarrhea. Her
mother reports that her daughter appears tired and pale. The girl drank only 4 oz of fluids today
and has not urinated for more than 24 hours. The girl’s 2 older sisters also have diarrhea, but
their symptoms are improving. The family recently vacationed at a cabin on a lake in a rural
area. They did not drink any unfiltered lake water but drank the tap water.

The patient has a heart rate of 145 beats/min, a respiratory rate of 20 breaths/min, and blood
pressure of 85/55 mm Hg. She is ill-appearing, and her capillary refill time is 4 seconds. The
remainder of her physical examination findings are unremarkable.

Of the following, the MOST likely cause of this girl’s illness is


A. Escherichia coli
B. Giardia duodenalis
C. norovirus
D. Yersinia enterocolitica

American Academy of Pediatrics 43


PREP® Self-Assessment PREPSA 2023

Correct Answer: A
The girl in the vignette has bloody diarrhea that most likely resulted from drinking contaminated
well water, a common source of water in rural areas. The most likely cause of her illness is
infection with Escherichia coli (possibly hemolytic uremic syndrome). E coli, including E coli
0157:H7 and other forms of Shiga toxin–producing E coli, are common well-water contaminants.
Drinking water contaminated with Giardia duodenalis, norovirus, and Yersinia enterocolitica can
also cause diarrhea but is less likely to produce bloody diarrhea.

An estimated 15% to 20% of homes in the United States rely on private wells for their water
supply. In most states, families are responsible for maintaining and testing their own water
system. Well water should be tested at least annually for contaminants, including coliforms; if
results are positive, specific testing for E coli should be performed. Other common bacterial
contaminants include Salmonella, Yersinia, Shigella, and Campylobacter. Common viral
contaminants include norovirus, sapovirus, rotavirus, and hepatitis A and E. Parasites, such as
Giardia, can spread through water sources, and water filtration may not remove the spores.
Routine testing for these parasites is not recommended unless there is an outbreak in the
surrounding area or home.

Chemicals, such as arsenic, fluoride, lead, nitrates, radon, and uranium, can also contaminate
drinking water. Arsenic, radon, and uranium are carcinogenic. Arsenic is most commonly found
in water in the Slate Belt (parts of the southeastern United States, Nebraska, and Alaska).
Uranium is most commonly found in water in the Western mountain states (Arizona, Utah, New
Mexico, Montana, Colorado, Idaho, Nevada, and Wyoming) and in granite outcroppings in the
Eastern United States. Small amounts of fluoride in drinking water promote healthy tooth
enamel, but toxic levels can be found in some private sources of drinking water (eg, well water).

Lead can be found in private wells and public drinking water sources, typically as a result of
leaching from lead-containing pipes. In 2016, the American Academy of Pediatrics published a
policy statement on the prevention of childhood lead toxicity, which addressed the topic of lead
exposure in schools from water fountains. Nitrates pose a significant risk to young infants fed
formula mixed with contaminated water or homemade baby food made with vegetables
contaminated with nitrate-containing fertilizer. Because of their increased ability to convert
nitrates to nitrites, these infants are at risk of methemoglobinemia.

During health supervision visits, practitioners should inquire about the family’s water supply,
especially in rural areas, where private wells may be common. Families who depend on well
water should be counseled to have their water tested at least once a year and more often when
there are infants, pregnant women, or those with compromised immune systems drinking the
water.

American Academy of Pediatrics 44


PREP® Self-Assessment PREPSA 2023

PREP Pearls
• Safe water sources should be discussed at health supervision visits.
• Private well water sources should undergo annual testing for infectious or toxic
contaminants.
• Water from drinking fountains can be contaminated with lead; schools should test their
water sources.

ABP Content Specifications(s)


• Know the contaminants potentially found in drinking water

Suggested Readings
• American Academy of Pediatrics. Drinking water from private wells and risks to
children. Pediatrics. 2009;123(6):15991605. doi:10.1542/peds.2009-0751.
• Council on Environmental Health. Prevention of childhood lead toxicity. Pediatrics.
2016;138(1):e20161493. doi:10.1542/peds.2016-1493.
• US Environmental Protection Agency. Private drinking water wells. Accessed September
1, 2022. https://www.epa.gov/privatewells.

American Academy of Pediatrics 45


PREP® Self-Assessment PREPSA 2023

Question 14
A 14-year-old adolescent girl is evaluated in the emergency department for fever of 3 days’
duration, sore throat, dysphagia, and a slightly muffled voice. She is otherwise healthy, fully
immunized, and does not have a history of recurrent throat infections. Her family history is
unremarkable. On physical examination, her temperature is 39°C, heart rate is 100 beats/min,
respiratory rate is 18 breaths/min, and oxygen saturation is 99% in room air. She is in mild
discomfort due to her throat pain but is speaking comfortably in complete sentences. Her left
tonsil is 3+ enlarged and erythematous, her right tonsil is 1+, and her uvula is deviated to the
right. She has enlargement and tenderness of her left cervical lymph nodes. The remainder of her
physical examination findings are unremarkable.

Of the following, the MOST appropriate next management step for this adolescent is
A. drainage of the abscess
B. intravenous antibiotics
C. observation only
D. tonsillectomy

American Academy of Pediatrics 46


PREP® Self-Assessment PREPSA 2023

Correct Answer: A
The adolescent in the vignette has a peritonsillar abscess (PTA); drainage of the abscess is the
most appropriate first step in treatment. Intravenous antibiotics may be administered as the first
step in treatment for individuals with certain complications resulting from PTAs (eg, septic
thrombophlebitis). Observation alone is not appropriate management and may result in
complications. Tonsillectomy for an acute infected PTA may be performed in certain individuals,
such as those who would require general anesthesia for incision and drainage and those with a
history of recurrent tonsil infections, but would not be the appropriate first-line treatment for this
otherwise healthy adolescent.

Peritonsillar abscesses occur when pus collects between the pharyngeal muscles and the palatine
tonsil capsule. The most common inciting pathogens are Streptococcus and Fusobacterium
species. Common clinical manifestations include fever, sore throat, dysphagia, muffled voice,
asymmetric tonsils, and uvular deviation.

The diagnosis of a PTA is usually made clinically. If the diagnosis is uncertain, intraoral
ultrasonography or computed tomography of the neck with intravenous contrast may be
performed to assess for the presence of a fluid collection or neck mass, taking into consideration
the risks of sedation and radiation.

Treatment of an uncomplicated PTA is usually drainage under local anesthesia with a subsequent
10-day course of oral antibiotics, usually a penicillin, cephalosporin, or clindamycin. After
abscess culture and sensitivities are resulted, antibiotics can be directed at the causative
organism.

Some children and adolescents with PTA may require hospital admission, including those who
experience complications.

Indications for admission in PTA


• The need for intravenous hydration due to poor oral intake
• pain management
• no reliable outpatient follow-up
• management of complications after drainage such as severe bleeding or respiratory
distress secondary to aspiration of abscess contents into the patient’s airway

PREP Pearls
• Peritonsillar abscesses typically present with unilateral tonsillar enlargement and uvular
deviation.
• Uncomplicated peritonsillar abscesses may be treated with drainage under local
anesthesia and subsequent oral antibiotics.

American Academy of Pediatrics 47


PREP® Self-Assessment PREPSA 2023

ABP Content Specifications(s)


• Recognize the clinical findings associated with peritonsillar abscess
• Plan the appropriate management of a peritonsillar abscess
• Plan the appropriate diagnostic evaluation of a peritonsillar abscess, considering
commonly associated pathogens

Suggested Readings
• Bochner RE, Gangar M, Belamarich PF. A clinical approach to tonsillitis, tonsillar
hypertrophy, and peritonsillar and retropharyngeal abscesses. Pediatr Rev.
2017;38(2):81-92. doi:10.1542/pir.2016-0072.
• Conrad C, Cornfield DN. Airway obstruction. In: McInerny TK, Adam HM, Campbell
DE, DeWitt TG, Foy JM, Kamat DM, eds. American Academy of Pediatrics Textbook of
Pediatric Care. American Academy of Pediatrics; 2021:chap 348. Accessed September 1,
2022. Pediatric Care Online .
• Mitchell RB, Archer SM, Ishman SL, et al. Clinical practice guideline: tonsillectomy in
children (update). Otolaryngol Head Neck Surg. 2019;160(1_suppl):S1-S42.
doi:10.1177/0194599818801757.

American Academy of Pediatrics 48


PREP® Self-Assessment PREPSA 2023

Question 15
A 16-year-old adolescent girl is seen in the emergency department for evaluation of chest pain,
shortness of breath, and nausea of 1 day’s duration. She describes the pain as sharp, substernal,
and constant. She feels nauseated but has not vomited, and has no interest in eating or drinking.
The girl’s heart rate is 120 beats/min, respiratory rate is 40 breaths/min, blood pressure is 100/70
mm Hg, and oxygen saturation is 94% in room air. Her chest pain is not reproducible by
palpation. Cardiac examination reveals a normal S1, normal S2 with a gallop rhythm, and no
murmur. There are crackles heard bilaterally on auscultation of her lungs. Her liver is palpated 3
cm below the right costal margin. Her extremities are pale and cool with delayed capillary refill.
A chest radiograph (Item Q15A) and electrocardiogram (Item Q15B) are obtained.

Of the following, the BEST next step in the adolescent’s diagnostic evaluation is
A. cardiac catheterization
B. cardiac magnetic resonance imaging
C. transesophageal echocardiography
D. transthoracic echocardiography

American Academy of Pediatrics 49


PREP® Self-Assessment PREPSA 2023

Correct Answer: D
The adolescent in the vignette has findings significant for tachycardia, a gallop rhythm, bilateral
crackles, and hepatomegaly. Chest radiography demonstrates cardiomegaly and pulmonary
edema, and low-voltages and abnormal ST segments and T waves are noted on
electrocardiography. This constellation of signs and symptoms is consistent with myocarditis,
and additional diagnostic testing is warranted. Of the response choices, the best next test would
be transthoracic echocardiography.

Transesophageal echocardiography is not recommended for the initial evaluation of myocarditis


and is typically not needed in this clinical context. Cardiac magnetic resonance imaging (MRI)
can play an important role in the diagnosis of myocarditis, but it would not be recommended for
the initial evaluation. Cardiac catheterization may be warranted during the evaluation of heart
failure, but it would not be appropriate at this stage of diagnosis.

Myocarditis can present in several ways along a spectrum of severity. Individuals with fulminant
myocarditis, the most severe form, are critically ill with poor perfusion, a gallop rhythm, and
hepatomegaly. They may experience life-threatening arrhythmias and cardiovascular collapse.
Due to the acute onset, the left ventricle may not have had an opportunity to dilate, so chest
radiography may not show cardiomegaly nor will echocardiography show a dilated left ventricle,
although the cardiac function will be poor. Signs and symptoms of heart failure in the non-
fulminant form may include shortness of breath, dyspnea with exertion, nausea, vomiting, and
decreased appetite (poor oral intake in infants). Chest radiography will demonstrate
cardiomegaly and pulmonary edema. Echocardiography will show poor function and a dilated
left ventricle. Dilated cardiomyopathy, as a result of myocarditis causing a “burned out”
myocardium, presents with a gradual progression of heart failure signs and symptoms. The
muscle becomes injured by infection and chronic inflammation, resulting in a thinned out and
dilated left ventricle that functions poorly.

In cases of myocarditis, chest radiography findings can range from normal to cardiomegaly and
pulmonary edema, as seen in this girl’s imaging study. Echocardiography will show varying
degrees of ventricular dysfunction and possibly a pericardial effusion and valvular dysfunction.
Electrocardiography should be obtained on all patients with suspected myocarditis. Most affected
individuals will be tachycardic and some will have arrhythmias (eg, tachyarrhythmias and heart
block). Relatively low voltages are commonly seen, as are abnormal T waves. Rarely, there may
be changes suggestive of ischemia with ST segment elevations (Item C15). A cardiac MRI may
be used to look for evidence of myocardial inflammation and scarring. Laboratory studies that
may prove helpful in the evaluation of myocarditis include troponin, brain-natriuretic peptide,
end-organ function studies (eg, blood urea nitrogen, creatinine, transaminases), as well as viral
polymerase chain reaction studies and serologies.

American Academy of Pediatrics 50


PREP® Self-Assessment PREPSA 2023

PREP Pearls
• Myocarditis can present along a spectrum of severity ranging from cardiogenic shock to a
chronic picture with dilated cardiomyopathy.
• Electrocardiography can be very helpful in the diagnosis of myocarditis, often
demonstrating low voltage, ST- and Twave abnormalities, and arrhythmias.
• Transthoracic echocardiography can be used to support the diagnosis of myocarditis
demonstrating evidence of poor function with or without dilation of the left ventricle,
depending on the chronicity of disease.

MOCA-Peds Objective
• Recognize the clinical manifestations, etiology, and diagnosis of myocarditis.

ABP Content Specifications(s)


• Plan an appropriate diagnostic evaluation of myocarditis

Suggested Readings
• Dasgupta S, Iannucci G, Mao C, Clabby M, Oster ME. Myocarditis in the pediatric
population: a review. Congenit Heart Dis. 2019;14(5):868-877. doi:10.1111/chd.12835.
• McCulloch MA, Gajarski RJ. Congenital and acquired heart disease. In: McInerny TK,
Adam HM, Campbell DE, DeWitt TG, Foy JM, Kamat DM, eds. American Academy of
Pediatrics Textbook of Pediatric Care. American Academy of Pediatrics; 2021:chap 234.
Accessed September 1, 2022. Pediatric Care Online.
• Putschoegel A, Auerbach S. Diagnosis, evaluation and treatment of myocarditis in
children. Pediatr Clin North Am. 2020;67(5):855-874. doi:10.1016/j.pcl.2020.06.013.
• Tunuguntla H, Jeewa A, Denfield SW. Acute myocarditis and pericarditis in children.
Pediatr Rev. 2019;40(1):14-25. doi:10.1542/pir.2018-0044.
American Academy of Pediatrics 51
PREP® Self-Assessment PREPSA 2023

Question 16
A 13-year-old adolescent girl is seen in the office for evaluation of lower abdominal pain and
hematochezia of 3 months’ duration. Her symptoms have progressed from mild lower abdominal
pain and loose stools to severe, crampy lower abdominal pain with bloody, liquid stools 6 to 8
times per day. She has urgency and tenesmus, and passes nocturnal stools. Her family has noted
pallor over the last few weeks, and she reports dizziness. The girl is otherwise healthy and does
not take any medication. Her mother has rheumatoid arthritis and thyroid disease.
The girl is pale and quiet. She is afebrile, with a heart rate of 150 beats/min, blood pressure of
110/65 mm Hg, weight of 38 kg (13th percentile for age), height of 146 cm (4th percentile for
age), and body mass index of 18 kg/m2 (30th percentile for age). She has conjunctival pallor, dry
mucous membranes, and a soft systolic murmur. Her abdomen is soft, tender to palpation
diffusely without rebound or guarding, and grossly bloody stool is noted on the rectal
examination.

Laboratory evaluation in the office demonstrates a hemoglobin level of 7.8 g/dL (78 g/L) and
positive stool occult blood test.

Of the following, the BEST next step in the management of this adolescent is
A. immediate referral to the local emergency department for evaluation and care
B. oral iron supplementation with repeat hemoglobin test in 1 week
C. referral to pediatric gastroenterology for appointment within 1 week
D. stool infection studies with follow-up office appointment in 24 hours

American Academy of Pediatrics 52


PREP® Self-Assessment PREPSA 2023

Correct Answer: A
The adolescent in the vignette has significant anemia, evidence of ongoing lower gastrointestinal
bleeding, and vital signs that are concerning for hypovolemia (tachycardia). She needs to be
urgently evaluated and stabilized in the emergency department.

The initial step in the evaluation of children with gastrointestinal bleeding (either through
vomitus or stool) is the assessment of hemodynamic stability based on vital signs (tachycardia,
hypotension, orthostatic changes), capillary refill, and mental status. If hemodynamic instability
is noted, appropriate intravenous (IV) access and IV fluid will be necessary, with consideration
of transfusions of packed red blood cells, platelets, and/or coagulation factors as clinically
appropriate. Laboratory data, including hemoglobin level, platelet count, coagulation studies,
liver function tests, albumin level, and type and cross-match (in preparation for potential blood
transfusions), should be obtained. The presence of blood should be confirmed with a stool occult
blood test, as stool or vomitus can appear red with the ingestion of certain foods, dyes, and
medications. Urgent consultation with a pediatric gastroenterologist is indicated because children
with refractory gastrointestinal bleeding and ongoing hemodynamic instability despite the
administration of IV fluids and blood products may require emergent therapeutic endoscopy.
Once hemodynamic stability is established, attention should be given to the cause of the
gastrointestinal bleeding.

Upper gastrointestinal (UGI) bleeding may manifest as hematemesis (bright red blood in
vomitus), coffee-ground emesis, melena, or (with brisk UGI bleeding) hematochezia. Lower
gastrointestinal (LGI) bleeding may present as melena or hematochezia.

Common causes of UGI bleeding are as follows:


• Coagulopathy
• Esophageal varices
• Esophagitis
• Gastritis
• Mallory-Weiss tear
• Ulcer (gastric or duodenal, peptic or Helicobacter pylori–associated)

Common causes of LGI bleeding include:


• Arteriovenous malformation
• Coagulopathy
• Colitis (infectious, allergic, inflammatory bowel disease)
• Fissure
• Intussusception
• Meckel diverticulum Polyp

Evaluation to determine the source of the gastrointestinal bleeding should be tailored to the
suspected causes. Endoscopy should be used to evaluate UGI bleeding to both determine the

American Academy of Pediatrics 53


PREP® Self-Assessment PREPSA 2023

cause of bleeding and potentially treat the underlying condition. Treatment methods include
variceal banding, argon plasma coagulation, cautery, and epinephrine administration.

The approach to determining the cause of LGI bleeding should be tailored by the presenting
symptoms. A Meckel scan should be considered for painless LGI bleeding. A history supporting
colitis (abdominal pain, urgency, tenesmus, and/or passing of nocturnal stools) should prompt an
evaluation for infectious colitis. Colicky pain and vomiting in a young child would suggest
intussusception, and ultrasonography with surgical consultation should be considered. Ongoing
LGl bleeding would require further evaluation with endoscopy and colonoscopy. Assessing the
small bowel with a nuclear medicine scan (using technetium Tc 99m-labeled red blood cells),
angiography, and/or capsule endoscopy may be recommended when gastrointestinal bleeding
continues and endoscopy and colonoscopy are unable to reveal the source.

Starting oral iron supplementation with follow-up in 1 week or requesting pediatric


gastroenterology consultation within 1 week are not appropriate steps for this adolescent with
evidence of hypovolemia and anemia on physical examination. Although stool infection studies
would be indicated, follow-up in 24 hours would not be appropriate for this adolescent.

PREP Pearls
• Initial assessment of a child with gastrointestinal bleeding includes evaluation of
hemodynamic stability.
• Children with gastrointestinal bleeding and hemodynamic instability need urgent
intravenous access and intravenous fluid administration, and may require transfusion of
blood products.
• Upper gastrointestinal bleeding may manifest as hematemesis, melena, or hematochezia.
Lower gastrointestinal bleeding may present as melena or hematochezia.

ABP Content Specifications(s)


• Plan the appropriate evaluation of blood in vomitus or stool, including in a patient who
has hemodynamically significant blood loss

American Academy of Pediatrics 54


PREP® Self-Assessment PREPSA 2023

Suggested Readings
• Avner JR. Gastrointestinal hemorrhage. In: McInerny TK, Adam HM, Campbell DE,
DeWitt TG, Foy JM, Kamat DM, eds. American Academy of Pediatrics Textbook of
Pediatric Care. American Academy of Pediatrics; 2021:chap 155. Accessed September 1,
2022. Pediatric Care Online.
• Neidich GA, Cole SR. Gastrointestinal bleeding. Pediatr Rev. 2016;35(6):243-254.
doi:10.1542/pir.35-6-243.
• Sahn B, Bitton S. Lower gastrointestinal bleeding in children. Gastrointest Endoscopy
Clin North Am. 2016;26:75-98. doi:10.1016/j.giec.2015.08.007.
• Turner D, Ruemmele FM, Orlanski-Meyer E, et al. Management of paediatric ulcerative
colitis, part 2: acute severe colitis-an evidence-based consensus guideline from the
European Crohn’s and Colitis Organization and the European Society of Paediatric
Gastroenterology, Hepatology, and Nutrition. J Pediatr Gastrointest Nutr. 2018;67:292-
310. doi:10.1097/MPG.0000000000002036.

American Academy of Pediatrics 55


PREP® Self-Assessment PREPSA 2023

Question 17
A 4-month-old, full-term, developmentally normal infant is evaluated in the outpatient office for
paroxysmal episodes for the past 2 weeks. During the episodes, his body crunches up repeatedly,
“as though he is doing baby sit-ups.” Initially, it occurred once or twice per day, usually in the
evening, but it now occurs in clusters, at which point he becomes difficult to console. His mother
suspects that he is uncomfortable and might have gastroesophageal reflux. She is concerned
because the episodes are becoming more frequent.

The infant’s vital signs are normal. His physical examination findings, including neurological,
are normal. During the visit, the infant had a typical episode in which he stiffened, his arms
extended out suddenly and he bent forward. This occurred in a short cluster during which he
appeared uncomfortable and cried.

Of the following, the BEST next step in this infant’s management is


A. referral to a gastroenterologist for outpatient evaluation
B. referral to a neurologist for outpatient evaluation
C. reflux precautions and close follow-up in the pediatrician’s office
D. urgent neurological evaluation through the emergency department

American Academy of Pediatrics 56


PREP® Self-Assessment PREPSA 2023

Correct Answer: D
The infant in the vignette’s episodes are suggestive of infantile spasms (IS), a condition requiring
prompt diagnosis and treatment to optimize developmental outcome. Infantile spasms, the most
common epilepsy syndrome in infancy, is clinically characterized by a triad of:
• Epileptic spasms
• Electroencephalogram background of hypsarrhythmia
• Accompanying developmental plateau and regression

When all 3 of these features are present the eponym, “West syndrome,” is used. The diagnosis
requires a high index of suspicion and necessitates urgent evaluation with expedited
electroencephalogram (EEG) and pediatric neurology consultation.

Infantile spasms typically present during the first year after birth, with a peak age of onset
between 3 and 7 months. Classification is divided into 2 etiologic categories: cryptogenic and
symptomatic. Cryptogenic IS is diagnosed in 10% to 40% of affected infants, presenting in
developmentally normal infants whose diagnostic evaluation does not identify an underlying
cause. With prompt recognition and early initiation of treatment, this cohort of children is more
likely to have a favorable developmental outcome.

Symptomatic IS is diagnosed in 60% to 90% of affected infants; a greater percentage of infants


are now recognized as symptomatic due to improved genetic diagnostic testing. Symptomatic IS
can occur secondary to prenatal/perinatal causes including hypoxic-ischemic encephalopathy,
cortical malformations, and genetic syndromes (classically, tuberous sclerosis complex and
trisomy 21). Postnatal etiologies can include trauma and infection. Developmental outcome is
highly dependent on the underlying etiology. Item C17A summarizes the etiologic classification
of infantile spasms.

Often, when symptoms of IS begin, parents bring their children to the pediatrician with concerns
for colic or gastroesophageal reflux (GER). Classically, spasms involve symmetric contraction
with flexion of the trunk, neck and arms lasting up to 5 seconds and occurring in clusters.
However, there is a range of clinical appearance and severity of spasms, with some presenting as
tonic eye rolling or neck flexion/nodding, which can be subtle and easily missed. Spasms may
initially emerge while the infant is drowsy, either when falling asleep or waking from sleep, and
can be accompanied by crying, grimacing, pallor, flushing, or nystagmus.

Suspicion for infantile spasms should prompt urgent referral for evaluation and EEG rather than
outpatient referral due to the impact early diagnosis and treatment can have on developmental
outcome. While GER or colic can have a similar clinical appearance, IS should be considered
and excluded first.

American Academy of Pediatrics 57


PREP® Self-Assessment PREPSA 2023

Electroencephalography in cases of IS demonstrates a specific interictal background of


hypsarrhythmia: a chaotic, disorganized, asynchronous, nonrhythmic, high-voltage spike and
spike-and-slow wave pattern (Item C17B). While a routine EEG may capture hypsarrhythmia, a
recording including wakefulness and sleep is helpful as hypsarrhythmia initially emerges during
stage 2 and 3 of non-REM sleep; prolonged (24 hours) EEG monitoring allows exclusion of
movements mimicking infantile spasms.

American Academy of Pediatrics 58


PREP® Self-Assessment PREPSA 2023

Once the diagnosis of IS is confirmed, evaluation is directed at identifying an underlying cause.


This process typically includes magnetic resonance imaging of the brain as well as genetic and
metabolic testing, with an emphasis on treatable disorders with specific therapies. Treatment of
IS includes administration of adrenocorticotropic hormone (ACTH) or vigabatrin initiated as
soon as possible after diagnosis. Adrenocorticotropic hormone is preferred over vigabatrin for
first-line management of cryptogenic spasms. Vigabatrin can be used first-line and is the
preferred initial choice in children with tuberous sclerosis complex. Treatment considerations are
summarized in Item C17C. Treatment effect is often reassessed both clinically and with repeat
EEG monitoring to confirm resolution of the hypsarrhythmia pattern. Children with infantile
spasms often require long-term, multidisciplinary monitoring for neurodevelopmental progress
and subsequent development of other seizure types.

PREP Pearls
• Infantile spasms, the most common epilepsy syndrome in infancy, are clinically
characterized by a triad of: 1) epileptic spasms, 2) electroencephalogram background of
hypsarrhythmia, and 3) accompanying developmental plateau and regression.
• A high index of suspicion and urgent evaluation with expedited electroencephalography
are necessary to confirm the diagnosis of infantile spasms and quickly initiate treatment.
• Targeted evaluation for underlying causes of infantile spasms, focusing on treatable
conditions, should occur concomitantly with treatment initiation and is tailored toward
the individual patient. Long-term developmental outcome is impacted by time to
treatment and underlying etiology.

MOCA-Peds Objective
• Recognize the clinical features of rheumatic fever.

ABP Content Specifications(s)


• Understand the prognosis of infantile spasms
• Recognize the clinical findings associated with infantile spasms

American Academy of Pediatrics 59


PREP® Self-Assessment PREPSA 2023

Suggested Readings
• Fine A, Wirrell E. Seizures in children. Pediatr Rev. 2020;41(7):321-347.
doi:10.1542/pir.2019-0134.
• Go CY, Mackay MT, Weiss SK, et al. Evidence based guideline update: medical
treatment of infantile spasms. Report of the Guideline Development Subcommittee of the
American Academy of Neurology and the Practice Committee of the Child Neurology
Society. Neurology. 2012;78(24):1974-1980. doi:10.1212/WNL.0b013e318259e2cf.
• McBride M. Seizure disorders. In: McInerny TK, Adam HM, Campbell DE, DeWitt TG,
Foy JM, Kamat DM, eds. American Academy of Pediatrics Textbook of Pediatric Care.
American Academy of Pediatrics; 2021:chap 327. Accessed September 1, 2022. Pediatric
Care Online.
• Whelass JW, Gibson PA, Rosbeck KL, et al. Infantile spasms (West syndrome): update
and resources for pediatricians and providers to share with parents. BMC Pediatr.
2012;12:108. doi:10.1186/1471-2431-12-108.

American Academy of Pediatrics 60


PREP® Self-Assessment PREPSA 2023

Question 18
A 5-year-old, previously healthy boy is seen in the clinic for evaluation of testicular pain and
fever that started this morning. He had a cough and runny nose 2 weeks ago, which have
resolved. He has no other symptoms and no history of trauma. On physical examination, the boy
has a temperature of 38.5°C, heart rate of 117 beats/min, respiratory rate of 22 breaths/min, and
oxygen saturation of 98% in room air. Both testicles are palpated in the scrotum and the
cremasteric reflex is intact. The testicles appear swollen, with no abnormal coloring or masses
palpated. Ultrasonography with Doppler shows normal blood flow to both testicles.

Of the following, this boy’s MOST likely diagnosis is


A. inguinal hernia
B. postinfectious orchitis
C. testicular torsion
D. torsion of the appendix testis

American Academy of Pediatrics 61


PREP® Self-Assessment PREPSA 2023

Correct Answer: B
The boy in the vignette has fever, painful testicular swelling, and a normal cremasteric reflex. Of
the response choices, these signs and symptoms are most consistent with orchitis. In this age
group, orchitis is most commonly seen after viral infections. An inguinal hernia can cause scrotal
pain and swelling but would be palpated during the genitourinary examination. Testicular
torsion, a surgical emergency, is an unlikely diagnosis for the boy in the vignette given his
normal blood flow on Doppler ultrasonography and normal cremasteric reflex. In the case of
testicular torsion, the normal cremasteric reflex (retraction of the testis in response to touch on
the upper thigh) is usually absent. Torsion of the appendix testis is also unlikely given the normal
findings on ultrasonography and absence of a “blue dot sign” on physical examination.

Orchitis (inflammation of the testis) and epididymitis (inflammation of the epididymis) have
infectious or inflammatory etiologies. Symptoms and signs of orchitis and epididymitis include
testicular swelling and tenderness, dysuria, urinary frequency and urgency, and often systemic
symptoms (eg, fever). The etiology of orchitis and epididymitis varies by age. Children ages 2 to
13 years often have a postinfectious cause (most commonly Mycoplasma, enterovirus, or
adenovirus). Vasculitis (eg, Henoch-Schönlein purpura) is another common cause of orchitis and
epididymitis in this age group. Older children and adults are more likely to have an infectious
etiology with organisms that cause urinary tract or sexually transmitted infections. A urinalysis
may show the presence of nitrites and/or leukocyte esterase because the infection can start in the
urine and ascend into the epididymis and testes.

It is important to differentiate testicular pain caused by orchitis and epididymitis from testicular
torsion, which is an emergency. Testicular torsion can be difficult to diagnose based on physical
examination findings alone. Therefore, ultrasonography with Doppler should be obtained for all
children and adolescents with testicular pain.

Ultrasonography with Doppler is not 100% specific; urgent surgical or urologic consultation
should be obtained when there is concern for testicular torsion.

PREP Pearls
• All children and adolescents with testicular pain should have ultrasonography with
Doppler performed to evaluate for testicular torsion.
• Symptoms and signs of orchitis and epididymitis typically include testicular swelling and
tenderness, dysuria, urinary frequency and urgency, and often systemic symptoms.

ABP Content Specifications(s)


• Recognize the clinical findings associated with orchitis
• Identify common causes of orchitis

American Academy of Pediatrics 62


PREP® Self-Assessment PREPSA 2023

Suggested Readings
• Palmer LS. Scrotal swelling and pain. In: McInerny TK, Adam HM, Campbell DE,
DeWitt TG, Foy JM, Kamat DM, eds. American Academy of Pediatrics Textbook of
Pediatric Care. American Academy of Pediatrics; 2021:chap 190. Accessed September 1,
2022. Pediatric Care Online.
• Sexually Transmitted Infections Treatment Guidelines, 2021. Epididymitis. Centers for
Disease Control and Prevention.
• Updated July 22, 2021. Accessed September 1, 2022. https://www.cdc.gov/std/treatment-
guidelines/epididymitis.htm.
• Stewart A, Ubee SS, Davies H. Epididymo-orchitis. BMJ. 2011;342:d1543.
doi:10.1136/bmj.d1543.
• Trojian TH, Lishnak TS, Heiman D. Epididymitis and orchitis: an overview. Am Fam
Physician. 2009;79(7):583-587. https://pubmed.ncbi.nlm.nih.gov/19378875/.
• Wu WJ, Gitlin JS. The male genital system. Pediatr Rev. 2020;41(3):101-111.
doi:10.1542/pir.2017-0316.

American Academy of Pediatrics 63


PREP® Self-Assessment PREPSA 2023

Question 19
A 9-month-old girl is seen for follow-up 3 weeks after hospital admission for a febrile
Escherichia coli urinary tract infection. She received intravenous ceftriaxone in the hospital and
subsequently completed treatment with oral cefdinir. The girl is currently asymptomatic, feeding
well, and having regular bowel movements. At 6 months of age she had a febrile urinary tract
infection. She is alert and active, and has vital signs that are normal for age. Her physical
examination findings, including those of the genitourinary examination, are unremarkable.
Renal ultrasonography performed in the hospital was normal. A voiding cystourethrogram
showed bilateral grade III vesicoureteral reflux.

Of the following, the MOST appropriate next management step for this infant is
A. polyethylene glycol treatment, orally
B. repeat voiding cystourethrogram in 3 months
C. trimethoprim-sulfamethoxazole prophylaxis, orally
D. ureteral reimplantation surgery

American Academy of Pediatrics 64


PREP® Self-Assessment PREPSA 2023

Correct Answer: C
The infant in the vignette has recurrent febrile urinary tract infection (UTI) and a voiding
cystourethrogram (VCUG) that shows bilateral vesicoureteral reflux (VUR). The most
appropriate next management step for this girl is trimethoprim-sulfamethoxazole prophylaxis to
prevent recurrent UTI.

Vesicoureteral reflux is the retrograde passage of urine from the bladder to the upper urinary
tract. Vesicoureteral reflux is divided into primary and secondary types. Primary VUR occurs
due to a congenitally short segment of ureter within the bladder wall, which results in incomplete
closure of the ureterovesical junction during bladder contraction. Secondary VUR occurs as a
result of high bladder pressure associated with conditions such as posterior urethral valves,
bladder-bowel dysfunction, and neurogenic bladder.

Asymptomatic vesicoureteral reflux may be diagnosed during evaluation of a neonate with a


history of antenatal hydronephrosis or screening in children with a family history of VUR.
Vesicoureteral reflux may be diagnosed during the evaluation of a febrile UTI. In toilet-trained
children, VUR may present with bladder-bowel dysfunction (eg, urinary frequency and urgency,
day time enuresis, encopresis and constipation). Less commonly, VUR may present with
hypertension, renal insufficiency, or incidental detection of a small and scarred kidney on
ultrasonography or computed tomography.

Voiding cystourethrogram is the test of choice to confirm the presence of VUR and grade its
severity. Indications to perform a VCUG in neonates with a history of antenatal hydronephrosis
include presence of moderate to severe hydronephrosis or ureteral dilation on the postnatal
ultrasonography. A VCUG is also indicated in a child with a first febrile UTI with abnormal
renal ultrasonography, or a child with recurrent febrile UTI.

Vesicoureteral reflux is categorized as low grade (I-III) and high grade (IV-V). The International
Reflux study group classifies VUR into 5 grades as shown:

• Grade I: Reflux into the middle of the ureter without dilation


• Grade II: Reflux into the ureter and collecting system without dilation
• Grade III: Reflux into the ureter and collecting system with mild dilation and mild
blunting of calices
• Grade IV: Reflux into the ureter and collecting system with gross dilation and severe
blunting of calices
• Grade V: Massive reflux with gross dilation of ureter and collecting system, blunting of
all calices, and tortuosity of the entire collecting system

Spontaneous resolution, without any surgical intervention, occurs in most cases of VUR. The
likelihood of resolution is higher in low-grade as compared to high-grade VUR, unilateral as
compared to bilateral VUR, and in those patients presenting with asymptomatic antenatal
hydronephrosis compared to those presenting with febrile UTI. The medical management of
VUR includes watchful waiting, addressing any bladder-bowel dysfunction, and starting
American Academy of Pediatrics 65
PREP® Self-Assessment PREPSA 2023

antibiotic prophylaxis (if indicated). Antibiotic prophylaxis is indicated for non–toilet-trained


children, those with low-grade VUR with recurrent UTI, and those with high-grade VUR.
Medications commonly used for UTI prophylaxis include trimethoprim-sulfamethoxazole,
nitrofurantoin, or amoxicillin. Surgical correction (ureteral reimplantation) is recommended for
patients who have recurrent breakthrough infections on antibiotic prophylaxis or persistent high
grade VUR with significant risk of renal scarring.

Polyethylene glycol treatment might be considered for a child with secondary VUR associated
with chronic constipation. The child in the vignette has no history of constipation, therefore
polyethylene glycol treatment would not be appropriate for her. A repeat VCUG to assess for
resolution or persistence of VUR is generally recommended after 9 to 15 months, not after 3
months. The girl in the vignette does not meet criteria for surgical correction as she has bilateral
grade III VUR (low grade) and has not had a breakthrough UTI with the use of antibiotics.

PREP Pearls
• Vesicoureteral reflux may be diagnosed during the evaluation of a febrile urinary tract
infection, antenatally diagnosed hydronephrosis, or screening in children with family
history.
• Spontaneous resolution of vesicoureteral reflux is common when it is low grade,
unilateral, and in those with asymptomatic antenatally diagnosed hydronephrosis.
• Antibiotic prophylaxis is indicated for vesicoureteral reflux in non–toilet-trained children,
those with low-grade vesicoureteral reflux with recurrent urinary tract infection, and
those with high-grade vesicoureteral reflux.

ABP Content Specifications(s)


• Recognize the clinical findings associated with vesicoureteral reflux
• Understand the natural history of vesicoureteral reflux

Suggested Readings
• Balighian E, Burke M. Urinary tract infections in children. Pediatr Rev. 2018;39(1):3-12.
doi:10.1542/pir.2017-0007.
• Mattoo TK, Shaikh N, Nelson CP. Contemporary management of urinary tract infection
in children. Pediatrics. 2021;147(2):e2020012138. doi:10.1542/peds.2020-012138.
• Nguyen HT. Obstructive uropathy and vesicoureteral reflux. In: McInerny TK, Adam
HM, Campbell DE, DeWitt TG, Foy JM, Kamat DM, eds. American Academy of
Pediatrics Textbook of Pediatric Care. American Academy of Pediatrics; 2021:chap 299.
Accessed September 1, 2021. Pediatric Care Online.

American Academy of Pediatrics 66


PREP® Self-Assessment PREPSA 2023

Question 20
A 12-year-old boy is seen in the office for evaluation of bilateral anterior knee pain. His pain
began about 3 months ago during wrestling practice. There was no acute injury. He reports no
swelling, locking, or instability. The pain is worse with kneeling, running, and jumping. There is
tenderness over the inferior aspect of both patellae. He also has pain with resisted knee
extension. The remainder of the boy’s physical examination findings are unremarkable.

Of the following, the MOST likely cause of this boy’s pain is


A. Osgood-Schlatter disease
B. patellar tendinopathy
C. prepatellar bursitis
D. Sinding-Larsen-Johansson syndrome

American Academy of Pediatrics 67


PREP® Self-Assessment PREPSA 2023

Correct Answer: D
The most likely diagnosis for the boy in the vignette is Sinding-Larsen-Johansson syndrome
(SLJ) or inferior patellar pole apophysitis. An apophysis is a bony ossification center adjacent to
a minor growth plate. Apophyses are found at sites where tendons attach to bone. Contraction of
a muscle causes the tendon to pull on the apophysis. The physis, the growth area adjacent to the
apophysis, is made of soft bone that has not yet calcified and is especially vulnerable to injury.
With repetitive activity, traction on the apophysis can lead to pain. Overuse of the quadriceps
muscles and direct pressure on the bottom of the patella (eg, kneeling) cause irritation of the
apophysis, known as apophysitis.

Sinding-Larsen-Johansson syndrome typically occurs in children between the ages of 9 and 13


years. Because they experience rapid prepubertal growth later than girls, boys tend to have SLJ
at the older end of this age range, while girls typically have symptoms around age 9 or 10 years.
The boy in the vignette has classic history and physical examination findings for SLJ: insidious
onset of pain, bilateral symptoms, tenderness over the inferior pole of the patella, and pain with
resisted quadriceps contraction. About half of affected children have unilateral symptoms. Some
children recall an acute episode that provoked their pain, such as a fall directly onto the knee(s).
While clinicians can make the diagnosis of SLJ based on history and physical examination
findings, radiographs often show an irregular pattern or a slight separation at the inferior patella
apophysis. Item C20A shows a prominent apophysis at the inferior pole of the patella. Use of a
patellar strap over the patellar tendon (Item C20B) can act like a pulley mechanism and relieve
some of the tension on the apophysis. Strengthening and stretching exercises may help some
children, but there are no studies to show that exercises are efficacious. The application of ice
and the use of topical or oral nonsteroidal antiinflammatory medications (NSAIDs) may be
helpful for relief of symptoms.

American Academy of Pediatrics 68


PREP® Self-Assessment PREPSA 2023

Osgood-Schlatter disease (OSD) is another type of apophysitis. Children with OSD have pain
and tenderness at the tibial tubercle, the site where the patellar tendon attaches. The clinical
history and treatment of OSD and SLJ are very similar.

Patellar tendinopathy involves breakdown and fraying of the patellar tendon. Tendinopathy is
more common in adults, although adolescents who participate in jumping and landing sports
experience this condition as well. Young athletes with patellar tendinopathy may have symptoms
similar to those seen with SLJ and OSD. However, adolescents with patellar tendinopathy have
tenderness over the tendon, rather than the bone. Historically, clinicians used the term tendinitis
to describe tendon irritation. Currently, the term tendinopathy is preferred as this condition does
not have an inflammatory cause.

With repeated pressure or trauma to the front of the knee, the prepatellar bursa can become
irritated and swollen, a condition known as prepatellar bursitis. Prepatellar bursitis is more
common in adults but can affect adolescents and rarely, younger children. Because wrestlers
often kneel and land forcefully on their knees during practice and competition, they are at
increased risk of prepatellar bursitis. Affected individuals present with localized swelling over
the patella. The presence of redness and warmth over the front of the knee should prompt
evaluation for infectious bursitis which can be either primary, or secondary, following traumatic
bursitis.

PREP Pearls
• Apophysitis occurs because the physis, the growth area adjacent to the apophysis, is
made of soft bone and is especially vulnerable to injury.
• Sinding-Larsen-Johansson syndrome is an apophysitis of the inferior patellar pole
apophysis. Osgood-Schlatter disease is an apophysitis of the tibial tubercle.
• Tendinopathy and bursitis, while frequently seen in adults, are uncommon causes of knee
pain in children.

ABP Content Specifications(s)


• Recognize the clinical findings associated with sports-related prepatellar bursitis, and
manage appropriately

Suggested Readings
• Kannikeswaran N, Suresh S. Sports musculoskeletal injuries. In: McInerny TK, Adam
HM, Campbell DE, DeWitt TG, Foy JM, Kamat DM, eds. American Academy of
Pediatrics Textbook of Pediatric Care. American Academy of Pediatrics; 2021:chap 334.
Accessed September 1, 2022. Pediatric Care Online.
• Kiningham R, Monseau A. Caring for wrestlers. Curr Sports Med Rep. 2015;14(5):404-
412. doi:10.1249/JSR.0000000000000193.
• Overuse injuries. In: Sarwark JF, Labella CR, eds. Orthopaedics and Sports Medicine: A
Quick Reference Guide. 3rd ed. American Academy of Pediatrics. 2021:311-342.
• Wolf M. Knee pain in children: part I: evaluation. Pediatr Rev. 2016;37(1):18-23; quiz
24, 47. doi:10.1542/pir.2015-0040
American Academy of Pediatrics 69
PREP® Self-Assessment PREPSA 2023

Question 21
A 20-month-old, previously healthy girl is brought to the urgent care center with a 1-day history
of left otalgia and increased fussiness. She has had rhinorrhea and cough for the past week, and
developed a fever of 38.8°C 6 hours ago that resolved after 1 dose of acetaminophen. There has
been no otorrhea. She is drinking, eating, stooling, and voiding well. She has no past history of
otitis media. On physical examination, she is afebrile and has clear rhinorrhea in both nares and 2
tender, mobile, soft, 1-cm posterior cervical lymph nodes. The appearance of her left tympanic
membrane is shown in Item Q21. The remainder of her physical examination findings are
normal.

Of the following, the BEST next step in this girl’s management is

A. high-dose amoxicillin for 5 days


B. high-dose amoxicillin for 7 days
C. high-dose amoxicillin for 10 days
D. observation for 48 to 72 hours

American Academy of Pediatrics 70


PREP® Self-Assessment PREPSA 2023

Correct Answer: B
The girl in the vignette has acute otitis media (AOM), which is an acute bacterial infection of the
middle ear with fluid, otherwise known as suppurative otitis media. According to the American
Academy of Pediatrics Acute Otitis Media Clinical Practice Guidelines, the best treatment for
children younger than 24 months of age without severe signs or symptoms is observation for 48
to 72 hours with analgesic administration as needed followed by an antibiotic prescription if
symptoms worsen or persist over that time frame.

The diagnostic criteria for AOM include:


• Recent onset of ear pain (<48 hours)
• Bulging tympanic membrane
• Reduced mobility of the tympanic membrane with pneumatic otoscopy or tympanometry

Severe disease is defined as one of the following:


• Otalgia that is moderate or severe in nature lasting for at least 48 hours
• Temperature of 39°C (102.2°F) or higher

The treatment algorithm for children ≥ 6 months of age with AOM is outlined in Item C21A.
The recommended antibiotic therapy for initial or delayed treatment as well as failed treatment is
outlined in Item C21B. Recommended duration of oral antibiotic treatment of AOM by age:
• <2 years 10 days
• 2-5 years 7 days
• ≥6 years 5 to 7 days

Recommended duration for parenteral treatment of AOM with ceftriaxone is 3 days.

American Academy of Pediatrics 71


PREP® Self-Assessment PREPSA 2023

PREP Pearls
• The diagnostic criteria for acute otitis media include an acute onset of ear pain (<48
hours), a bulging tympanic membrane, and reduced mobility of the tympanic membrane
with pneumatic otoscopy.
• Severe acute otitis media is defined as otalgia that is moderate or severe in nature lasting
for at least 48 hours, or a temperature of 39°C (102.2°F) or higher.
• For children aged 6 months to less than 2 years with unilateral acute otitis media that is
not severe, observation for 48 to 72 hours is appropriate, after which antibiotic treatment
is indicated for persistent or worsening symptoms.

ABP Content Specifications(s)


• Recognize the various etiologies of diminished tympanic membrane mobility
• Plan the appropriate diagnostic evaluation of acute otitis media
• Plan the appropriate initial and follow-up management of acute otitis media in patients of
various ages, including when complications occur and when initial therapy is ineffective

Suggested Readings
• Lieberthal AS, Carroll AE, Chonmaitree T, et al. The diagnosis and management of acute
otitis media. Pediatrics. 2013;131(3):e964-e999. doi:10.1542/peds.2013-3791.
• Rosa-Olivares J, Porro A, Rodriguez-Varela M, Riefkohl G, Niroomand-Rad I. Otitis
media: to refer, to refer, to do nothing: a review for the practitioner. Pediatr Rev.
2015;36(11):480-488. doi:10.152/pir.36-11-480.
• Tan TQ. Otitis media and otitis externa, In: McInerny TK, Adam HM, Campbell DE,
DeWitt TG, Foy JM, Kamat DM, eds. American Academy of Pediatrics Textbook of
Pediatric Care. American Academy of Pediatrics; 2021:chap 305. Accessed September 1,
2022. Pediatric Care Online .

American Academy of Pediatrics 72


PREP® Self-Assessment PREPSA 2023

Question 22
A newborn is examined 4 hours after birth in the normal newborn nursery. She was born after 39
weeks’ gestation in an uncomplicated vaginal delivery to a 22-year-old woman with no
significant medical history. Apgar scores were 8 and 9 at 1 and 5 minutes, respectively. On
physical examination, the neonate has a soft boggy swelling over the occipital area that crosses
suture lines with bruising and scattered petechiae on the scalp. The remainder of the physical
examination findings are normal.

Of the following, the BEST next step in the management of this condition is
A. head ultrasonography
B. serial hemoglobin levels
C. serial physical examinations
D. serum bilirubin levels

American Academy of Pediatrics 73


PREP® Self-Assessment PREPSA 2023

Correct Answer: C
The newborn in the vignette has physical examination findings consistent with caput
succedaneum. Caput succedaneum is a collection of blood and serous fluid just beneath the skin
and above the periosteal layer of the scalp (Item C22A) in vertex presentation vaginal deliveries.
Caput is typically seen at or immediately after birth and resolves spontaneously within 48 to 72
hours. On physical examination, the characteristic findings are a diffuse, soft, edematous
swelling over the vertex of the scalp that may cross over suture lines and can be associated with
scalp bruising. This condition does not need further evaluation in cases with classic
presentations.
Item C22A

Item C22A

American Academy of Pediatrics 74


PREP® Self-Assessment PREPSA 2023

A cephalohematoma is a blood collection in the subperiosteal area caused by injury to the veins
beneath the periosteum from birth trauma (Item C22A). This finding is typically seen 24 hours
after birth as a firm swelling restricted to one area of the head, which increases in size over a few
days and resolves spontaneously over 2 to 3 weeks. Because of the cephalohematoma’s
subperiosteal location, the swelling does not cross suture lines; this differentiates it from a caput
succedaneum. A large cephalohematoma can increase the risk for significant jaundice; blood
breakdown in the lesion contributes to the neonate’s level of unconjugated bilirubin.
Subgaleal hemorrhage, a rare and serious condition causing blood collection beneath the galea
aponeurosis, results from tearing of the bridging veins connecting the scalp and the intradural
sinuses (Item C22A). It is most commonly seen after vacuum-assisted delivery and is
progressive from the time of birth. On physical examination, the swelling is ill-defined, may
have crepitus or fluid waves, and can extend into the neck or orbital region. Careful monitoring
of vital signs and serial hemoglobin levels is recommended; blood loss can be significant and
lead to anemia, hypovolemia, and shock. Item C22B illustrates the differences between the 3
conditions.

For this neonate with a caput succedaneum, no imaging is indicated, and the risk of bleeding is
minor, therefore, monitoring bilirubin or hemoglobin levels is not indicated.

PREP Pearls
• Caput succedaneum is a common newborn scalp swelling found immediately after birth.
• Caput succedaneum typically crosses suture lines and resolves spontaneously within 48
to 72 hours.
• Due to the risk of severe blood loss associated with subgaleal hemorrhage, which results
from tearing of the bridging veins connecting the scalp and the intradural sinuses, it is
important to differentiate this condition from caput succedaneum.

ABP Content Specifications(s)


• Distinguish between caput succedaneum, cephalohematoma, and subgaleal hemorrhage

American Academy of Pediatrics 75


PREP® Self-Assessment PREPSA 2023

Suggested Readings
• Adams-Chapman I, Stoll BJ. Nervous system disorders. In: Behrman, RE, Kliegman,
RM, Jenson, HB, Stanton BF. Nelson Textbook of Pediatrics. 18th ed. Saunders;
2007:713-720.
• Akangire G, Carter B. Birth injuries in neonates. Pediatr Rev. 2016;37(11):451-462.
doi:10.1542/pir.2015-0125.
• Kaur H, Campbell DE. Physical examination of the newborn. In: McInerny TK, Adam
HM, Campbell DE, DeWitt TG, Foy JM, Kamat DM, eds. American Academy of
Pediatrics Textbook of Pediatric Care. American Academy of Pediatrics; 2021:chap 94.
Accessed September 1, 2022. Pediatric Care Online.
• Whitesel E, Brodsky D. Fluctuant mass on an infant’s scalp. NeoReviews.
2018;19(8):e490-e492. doi:10.1542/neo.19-8-e490
• Wood KE. Solving the mystery of a 1-month-old infant with scalp swelling: search the
literature first. Hosp Pediatr. 2017;7(8):502-504. doi:10.1542/hpeds.2017-0015.

American Academy of Pediatrics 76


PREP® Self-Assessment PREPSA 2023

Question 23
A 3-year-old girl is admitted to the hospital for evaluation and treatment of acute-onset systemic
hypertension with an arterial blood pressure of 130/88 mm Hg. She is otherwise healthy. A
parenteral antihypertensive medication is prescribed for administration every 6 hours in order to
achieve optimum efficacy once the medication serum concentration is at steady state. The
elimination half-life of the medication is 6 hours and the medication follows firstorder kinetics.
After the second dose, the girl remains hypertensive with a blood pressure of 120/65 mm Hg but
is otherwise asymptomatic.

Of the following, the MOST likely time-frame, after administration of the initial dose, for this
medication to achieve optimum efficacy is
A. 12 hours
B. 18 hours
C. 24 hours
D. 30 hours

American Academy of Pediatrics 77


PREP® Self-Assessment PREPSA 2023

Correct Answer: D
The medication administered to the girl in the vignette follows first-order kinetics and has a 6-
hour elimination halflife. When given in regular intervals, this medication will achieve steady
state concentration around 30 hours after the initial dose (6-hour elimination half-life × 5 half-
lives). Elimination half-life is defined as the time it takes for the plasma concentration of the
drug or the total amount of drug in the body to be reduced by 50%. It is only applicable to drugs
that exhibit first-order kinetics, in which a constant fraction of drug is eliminated per unit time.
The steady state of a medication concentration is achieved after the fifth half-life. It takes 5
elimination half-lives for ~97% of the bioavailable medication dose to be eliminated from the
body. Thus, in this vignette, it is not surprising that after the second dose this girl has not
achieved optimal blood pressure control.

First-order kinetics occur when a constant proportion of the drug is eliminated per unit time. In
contrast, zero-order kinetics occur when a constant amount of drug is eliminated per unit time.
For drugs eliminated by first-order kinetics, half-life is constant regardless of serum
concentration.

Elimination half-life (in first-order kinetics) is determined by both medication clearance (CL)
and volume of distribution (Vd), as noted in the following equation:

T (½) = (0.693 × Vd)/CL

Factors that increase a medication’s half-life include decreased renal or hepatic clearance
(depending on the pharmacodynamics of the drug) or a decrease in overall metabolic rate. Drug
half-life may be decreased by increases in drug metabolism or rate of excretion. Changes in
elimination half-life are directly proportional to the volume of distribution of the drug. The larger
the volume of distribution, the more the drug is concentrated in the tissues rather than in the
plasma. Because only the plasma concentration of the drug is exposed to the elimination
mechanisms, an increase in volume of distribution results in an increase in elimination half-life.
Some medications have a narrow therapeutic window, meaning that the range of serum
concentrations required for the drug to be effective without causing toxicity is limited. Many
parenteral antibiotics (eg, vancomycin, aminoglycosides) have narrow therapeutic windows.
Following serum drug concentrations allows clinicians to monitor for risk of toxicity while
ensuring adequate medication concentration for efficacy. Levels obtained immediately before
dosing (trough levels) provide the most accurate assessment of serum medication concentration.
Peak levels, obtained shortly after drug administration, are less reliable because they are subject
to differences in absorption and distribution rates. Measurement of peak levels is usually
reserved for monitoring of medications in which peak levels are specifically associated with
efficacy or toxicity.

American Academy of Pediatrics 78


PREP® Self-Assessment PREPSA 2023

PREP Pearls
• Elimination half-life is defined as the time it takes for plasma drug concentration to be
reduced by 50%.
• It takes 5 half-lives for a drug to be roughly 97% eliminated, and 5 half-lives to achieve
steady state concentration.
• Changes in elimination half-life are proportional to volume of distribution, and inversely
proportional to clearance.

ABP Content Specifications(s)


• Understand the number of half-lives required to reach steady-state serum drug
concentrations
• Recognize the association between half-life, therapeutic range, and drug toxicity Plan the
appropriate timing for measurement of serum drug concentrations

Suggested Readings
• Loebstein R, Koren G. Clinical pharmacology and therapeutic drug monitoring in
neonates and children. Pediatr Rev. 1998;19(12):423-428. doi:10.1542/pir.19-12-423.
• Wagner J, Abdel-Rahman SM. Pediatric pharmacokinetics. Pediatr Rev. 2013;34(6):258-
269. doi:10.1542/pir.34-6-258.

American Academy of Pediatrics 79


PREP® Self-Assessment PREPSA 2023

Question 24
A 2-month-old infant is brought to the emergency department in January for trouble breathing.
He has had progressively worsening rhinorrhea and cough for the past 2 days. Over the past 8
hours, he has not been tolerating his breast milk feedings due to congestion and rapid breathing.
The infant was born at 31 weeks’ gestation and has otherwise been healthy.

On physical examination, he has a temperature of 38.0°C, heart rate of 149 beats/min, respiratory
rate of 46 breaths/min, and oxygen saturation of 89% in room air. He appears to be in mild
respiratory distress. Copious clear rhinorrhea and subcostal retractions are present. End
expiratory wheezing is heard in both lung fields. The remainder of his physical examination
findings are normal.

Of the following, the MOST likely pathogen causing this infant’s illness is
A. coronavirus
B. influenza virus
C. parainfluenza virus
D. respiratory syncytial virus

American Academy of Pediatrics 80


PREP® Self-Assessment PREPSA 2023

Correct Answer: D
The history and physical examination findings of the infant in the vignette, with an antecedent
upper respiratory tract infection, low-grade fever, and respiratory distress, are consistent with the
diagnosis of acute bronchiolitis. Bronchiolitis is a common inflammatory illness of the lower
respiratory tract in infants and young children. Although many community-acquired respiratory
viruses can cause bronchiolitis, respiratory syncytial virus (RSV) is the most common cause in
infants followed by human rhinovirus, human metapneumovirus, and parainfluenza virus. Other
viral pathogens such as influenza and coronavirus rarely cause bronchiolitis. In addition,
influenza is associated with high-grade fever.

Respiratory syncytial virus is a negative-strand, enveloped, nonsegmented RNA virus of the


genus Orthopneumovirus of the Paramyxoviridae family. In temperate climates, RSV causes
annual epidemics primarily in the winter and early spring.

Respiratory syncytial virus bronchiolitis can lead to hospitalization in 1% to 3% of all children


during the first year after birth, with most hospital admissions occurring before age 6 months.
The major risk factors for severe disease include:
• Premature birth before 29 weeks’ gestation
• Chronic lung disease of prematurity
• Hemodynamically significant congenital heart disease (eg, pulmonary hypertension)
• Immunodeficiency disorders

Other risk factors for severe RSV disease include low birth weight, maternal smoking, formula
feeding, exposure to secondhand smoke in the home, history of atopy, and household crowding.
Transmission of RSV occurs through direct or close contact with upper respiratory tract
secretions via large-particle droplets or fomites. The incubation period is typically 4 to 6 days.
Virus replication in the nasal epithelium is followed by an influx of immune cells into the
respiratory tract epithelium. The pathogenesis of bronchiolitis involves airway edema, increased
mucus secretions, and epithelial cell necrosis due to the direct cytotoxic effect of the virus. The
duration of viral shedding ranges from 3 to 8 days in immunocompetent hosts; shedding for up to
3 or 4 weeks may occur in young infants and immunocompromised hosts.

Respiratory syncytial virus causes respiratory tract infections in individuals of all ages. Young
infants with RSV infection often develop upper respiratory tract symptoms (rhinorrhea,
congestion) associated with low-grade fever; approximately 20% to 30% progress to lower
respiratory tract disease, especially bronchiolitis and/or pneumonia.

Lower respiratory tract disease manifests as cough, tachypnea, wheezing, rales, and increased
work of breathing (intercostal or subcostal retractions, nasal flaring, grunting or head bobbing).
Irritability, lethargy, feeding difficulties, and dehydration are common due to airway obstruction
resulting from mucus secretions and airway edema. Young infants, especially premature infants,
may present with apneic spells as the sole clinical feature. The disease course of bronchiolitis can
range from mild, self-limited illness to acute hypoxic respiratory failure requiring invasive
ventilator support. Among immunocompromised patients, especially recipients of hematopoietic
American Academy of Pediatrics 81
PREP® Self-Assessment PREPSA 2023

stem cell transplants, RSV can cause severe lower respiratory tract infection complicated by
respiratory failure, disseminated disease, and death.

Nearly all children acquire primary RSV infection at least once by 2 years of age. Immunity
resulting from RSV infection in childhood is incomplete; reinfection with RSV can occur
throughout life. Subsequent infection tends to be less severe, often presenting as a mild upper
respiratory tract infection in older children and adults. Severe lower respiratory tract disease due
to RSV can occur and recur in immunocompromised children and older adults.

The diagnosis of bronchiolitis is clinical, based primarily on the history and physical
examination findings. Routine laboratory testing for viral detection or chest radiography do not
affect management and therefore are not recommended in most ambulatory settings. Rapid
antigen detection methods such as direct fluorescent antibody assays and enzyme immunoassays
can reliably detect RSV in nasopharyngeal specimens with reported sensitivities of 80% to 90%
compared with culture. Coinfection with other community-acquired respiratory viruses may
occur in up to 30% of infants with RSV bronchiolitis. In hospitalized infants with severe disease,
more sensitive and specific reverse-transcriptase multiplex polymerase chain reaction assays may
be performed to rapidly detect RSV and other community respiratory viruses in nasopharyngeal
specimens. This information may be important for management of severe disease and infection
control in the inpatient setting.

Treatment for RSV bronchiolitis includes respiratory support in conjunction with fluid and
nutrition management. The routine use of bronchodilators, corticosteroids, or antibiotics is not
recommended; these therapies are not effective in shortening or modifying the disease course.
Hospitalized infants and children with RSV infection should be placed on contact precautions (in
addition to standard precautions) for the duration of illness.

For certain high-risk infants and young children (eg, preterm infants with chronic lung disease
and infants with hemodynamically significant congenital heart disease), prevention of RSV
infection using palivizumab, a humanized recombinant monoclonal immunoglobulin G1K
antibody, is recommended.

PREP Pearls
• Respiratory syncytial virus is the most common cause of acute bronchiolitis in infants
and young children, followed by human rhinovirus.
• Risk factors for severe respiratory syncytial virus infection include: premature birth
before 29 weeks’ gestation, chronic lung disease of prematurity, hemodynamically
significant congenital heart disease, and immunodeficiency disorders.
• The diagnosis of bronchiolitis is clinical, based primarily on the history and physical
examination findings.

American Academy of Pediatrics 82


PREP® Self-Assessment PREPSA 2023

ABP Content Specifications(s)


• Plan the appropriate diagnostic evaluation for respiratory syncytial virus infection
• Understand the epidemiology of respiratory syncytial virus
• Recognize the clinical features associated with respiratory syncytial virus infection, and
manage appropriately

Suggested Readings
• American Academy of Pediatrics, Committee on Infectious Diseases, Bronchiolitis
Guideline Committee. Technical report: updated guidance for palivizumab prophylaxis
among infants and young children at increased risk of hospitalization for respiratory
syncytial virus infection. Pediatrics. 2014;134(2):e620-e638. doi:10.1542/peds.2014-
1666.
• American Academy of Pediatrics. Respiratory syncytial virus. In: Kimberlin DW, Barnett
ED, Lynfield R, Sawyer MH, eds. Red Book: 2021–2024 Report of the Committee on
Infectious Diseases. 32nd ed. American Academy of Pediatrics; 2021. Accessed
December 7, 2021. Red Book Online.
• Hall CB. Bronchiolitis. In: McInerny TK, Adam HM, Campbell DE, DeWitt TG, Foy
JM, Kamat DM, eds. American Academy of Pediatrics Textbook of Pediatric Care.
American Academy of Pediatrics; 2021:chap 224. Accessed September 1, 2022. Pediatric
Care Online .
• Ralston SL, Lieberthal AL, Meissner HC. Clinical practice guideline: the diagnosis,
management, and prevention of bronchiolitis. Pediatrics. 134(5):e1474-e1502.
doi:10.1542/peds.2015-2862.
• Silver AH, Nazif JM. Bronchiolitis. Pediatr Rev. 2019;40(11):568-576.
doi:10.1542/pir.2018-0260.

American Academy of Pediatrics 83


PREP® Self-Assessment PREPSA 2023

Question 25
A 16-year-old adolescent girl is brought to the emergency department for evaluation of frequent
voiding for 2 months. She has 10 to 12 large-volume voids in a day. She has no burning with
urination or urinary incontinence. She typically drinks 5 to 6 L of water per day. The girl’s heart
rate is 82 beats/min, respiratory rate is 16 breaths/min, and blood pressure is 110/70 mm of Hg.
Her weight is at the 10th percentile and height is at the 25th percentile for age. She has no
neurologic deficits, and the rest of the examination findings are unremarkable.

Laboratory data are shown:


Blood Result
Sodium 133 mEq/L (133 mmol/L)
Potassium 4.1 mEq/L (4.1 mmol/L)
Chloride 98 mEq/L (98 mmol/L)
Bicarbonate 24 mEq/L (24 mmol/L)
Blood urea nitrogen 7 mg/dL (2.5 mmol/L)
Creatinine 0.6 mg/dL (53 µmol/L)
Glucose 72 mg/dL (4 mmol/L)
Serum osmolality 270 mOsm/kg (reference range, 275-305 mOsm/kg)
Urine Result
Specific gravity 1.005
Leukocyte esterase Negative
Nitrite Negative
Blood Negative
Protein Negative
Glucose Negative
Urine osmolality 150 mOsm/kg (reference range, 300-900 mOsm/kg)

Of the following, the MOST likely diagnosis for this adolescent girl is
A. diabetes insipidus
B. diabetes mellitus
C. primary polydipsia
D. syndrome of inappropriate antidiuretic hormone

American Academy of Pediatrics 84


PREP® Self-Assessment PREPSA 2023

Correct Answer: C
The girl in the vignette has polyuria, polydipsia, low serum sodium, low serum osmolality, and
dilute urine, favoring a diagnosis of primary polydipsia.

Polyuria is defined as urine volume production of more than 2 L/m2 or 40-50 mL/kg in 24 hours.
Primary polydipsia is characterized by the consumption of an excessive amount of fluids, leading
to polyuria and dilute urine. It is commonly seen in individuals with psychiatric conditions (eg,
schizophrenia, depression, and bipolar disorders) and is often referred to as psychogenic
polydipsia. Primary polydipsia can be seen in healthy individuals who drink large quantities of
water out of habit; this condition is referred to as habitual polydipsia or compulsory water
drinking.

Primary polydipsia can be asymptomatic or may be associated with symptoms of hyponatremia


(eg, nausea, vomiting, lethargy, confusion, ataxia, and seizures). Laboratory evaluation shows a
low to normal serum sodium level, low to normal serum osmolality, and low urine specific
gravity. A low urine osmolality is seen due to loss of the renal medullary concentration gradient
from excess water intake.

The differential diagnosis of primary polydipsia includes diabetes insipidus, which also presents
with polyuria, polydipsia, and dilute urine. The child in the vignette has hyponatremia (hypo-
osmolality), whereas the serum sodium level is high or normal in diabetes insipidus. A water
deprivation test can be performed to differentiate primary polydipsia from diabetes insipidus
when the diagnosis is not clear. In primary polydipsia, polyuria decreases, and the urine
osmolality increases after water deprivation. However, in diabetes insipidus, polyuria persists,
and the urine osmolality is low after water deprivation.

In diabetes mellitus, polyuria, polydipsia, and dilute urine result from a high urine glucose level
(secondary to a high serum glucose level), which causes an osmotic diuresis. The child in the
vignette has a normal serum glucose level, and her urine glucose result is negative, thus making
it unlikely that she has diabetes mellitus. The syndrome of inappropriate secretion of antidiuretic
hormone is characterized by a concentrated urine and reduced urine volume, unlike the results
obtained for the child in the vignette.

The treatment for primary polydipsia is restriction of water intake. Children with severe
hyponatremia (serum sodium <120 mEq/L [120 mmol/L]) and those with associated neurologic
symptoms should be given an intravenous infusion of 3% normal saline, with close monitoring
of serum sodium levels in an intensive care unit.

PREP Pearls
• Primary polydipsia is characterized by consumption of an excessive amount of fluids,
leading to polyuria, low serum sodium level, low serum osmolality, and dilute urine.
• Primary polydipsia can be asymptomatic or associated with symptoms of hyponatremia
(eg, nausea, vomiting, lethargy, confusion, ataxia, and seizures).
• The treatment for primary polydipsia is restriction of water intake.
American Academy of Pediatrics 85
PREP® Self-Assessment PREPSA 2023

ABP Content Specifications(s)


• Recognize the clinical findings associated with water intoxication in patients of various
ages

Suggested Readings
• Driano JE, Lteif AN, Creo AL. Vasopressin-dependent disorders: what is new in
children? Pediatrics. 2021;147(5):e2020022848. doi:10.1542/peds.2020-022848.
• Miller RS, Libber SM, Plotnick L. Polyuria. In: McInerny TK, Adam HM, Campbell DE,
DeWitt TG, Foy JM, Kamat DM, eds. American Academy of Pediatrics Textbook of
Pediatric Care. American Academy of Pediatrics; 2021: chap 182. Accessed September 1,
2022. Pediatric Care Online.
• Weiner A, Vuguin P. Diabetes insipidus. Pediatr Rev. 2020;41(2):96-99.
doi:10.1542/pir.2018-0337.

American Academy of Pediatrics 86


PREP® Self-Assessment PREPSA 2023

Question 26
A 10-month-old female infant is brought for a health supervision visit. She was last seen 4
months ago. She has been well since that visit, with no significant illness. She has been feeding
well, taking 4 to 5 ounces of formula 4 to 5 times/day plus pureed foods and yogurt. At birth, her
weight was at the 30th to 40th percentile; at her previous health supervision visit, her weight was
at the 15th to 20th percentile. She says “mama” and “dada,” and is able to pull up to stand but
does not crawl. She is followed by cardiology for a heart murmur. The girl’s temperature is
36.5°C, heart rate is 100 beats/min, respiratory rate is 28 breaths/min, blood pressure is 96/45
mm Hg, and oxygen saturation is 100% in room air. Her weight is 7.3 kg (8th percentile) and
length is 65 cm (<3rd percentile). She appears pale and is in no apparent distress. There is a 3/6
systolic ejection murmur, lungs are clear to auscultation bilaterally, and her abdomen is soft and
nontender with no hepatosplenomegaly. There are no focal neurologic deficits.

The remainder of her physical examination findings are normal.

Laboratory data are shown:


Laboratory Test Result
White blood cell count 7,400/µL (7.4 × 109/L)
Hemoglobin 3.2 g/dL (32 g/L)
Platelet count 314 × 103/µL (314 × 109/L)
Mean corpuscular volume 115 fL
Neutrophils 22%
Lymphocytes 71%
Monocytes 7%
Reticulocyte 0.8%
Absolute reticulocyte count 6 × 103/µL
Hemoglobin A 81.6%
Hemoglobin A2 2.4%
Hemoglobin F 16%
Adenosine deaminase 1.98 (reference range, 0.33-0.95 U/g of hemoglobin)

Of the following, the MOST appropriate treatment for this girl’s condition is
A. erythropoietin administration
B. long-term transfusion therapy
C. no intervention with continued observation
D. single transfusion with continued observation

American Academy of Pediatrics 87


PREP® Self-Assessment PREPSA 2023

Correct Answer: B
The girl in the vignette has a history and laboratory findings that are consistent with a pure red
blood cell aplasia, most likely Diamond-Blackfan anemia (DBA). This diagnosis is supported by
severe macrocytic anemia without reticulocytosis, elevated fetal hemoglobin level, and elevated
erythrocyte adenosine deaminase (eADA) level. Her cardiac murmur may be evidence of a
congenital heart defect (CHD); there is an increased prevalence of congenital anomalies (eg,
cardiac, genitourinary, thumb) in children with DBA. Genetic testing may reveal a mutation in a
ribosomal protein, however, not all mutations have been identified.

The treatment for DBA is long-term blood transfusion therapy, which is required to support the
child’s growth. Leucodepleted red blood cells should be administered to avoid febrile reactions,
cytomegalovirus transmission, and human leukocyte antigen (HLA) alloimmunization. It is
important to obtain a blood sample for hemoglobin electrophoresis and eADA testing before
administering a blood transfusion, as those tests must be performed on the child’s native blood
cells and would otherwise not be accurate until 3 months after a blood transfusion.

Iron chelation therapy should also be administered to prevent iron overload from long-term
transfusion therapy; most chelators cannot be administered until a child is at least 2 years of age.
Abdominal magnetic resonance imaging is used to assess the liver iron concentration and iron
deposition in other organs (eg, heart). Some forms of DBA are steroid responsive. However,
corticosteroids should be avoided in children younger than about 1 year because of the potential
side effects, including its effect on growth, and to allow safe administration of live vaccines.
However, in areas where there is a low or unsafe (risk for infection) blood supply, administration
may begin as early as 6 to 9 months of age. Hematopoietic stem cell transplantation is a curative
option, however, 20% of DBA cases have spontaneous remission. A matched sibling donor is
preferred but must be screened to ensure they are not a silent carrier. Children with DBA have an
increased risk for several types of cancer, which is not completely eliminated by stem cell
transplantation.

Transient erythroblastopenia of childhood (TEC) is a more common form of pure red blood cell
aplasia. It usually occurs after a viral infection, most commonly in very young children (typically
>1 year of age), and may be associated with other cytopenias. Transient erythroblastopenia of
childhood spontaneously resolves in a few months, and long-term transfusion therapy is not
required. However, if a child with TEC has significant symptoms of anemia, a red blood cell
transfusion may be warranted at that time. There is no malignancy risk associated with TEC.
Erythropoietin administration is not recommended for the treatment of DBA; the anemia is not
due to a lack of erythropoietin. Observation with no intervention is not appropriate for the infant
in the vignette as she has severe anemia and requires medical intervention. A single transfusion
with continued observation would be the treatment of choice for anemia in a significantly
symptomatic child with TEC, due to its likelihood of early spontaneous resolution. However, this
is not a treatment of choice in individuals with newly diagnosed DBA.

American Academy of Pediatrics 88


PREP® Self-Assessment PREPSA 2023

PREP Pearls
• Diamond-Blackfan anemia is a rare form of pure red blood cell aplasia with severe
macrocytic anemia without reticulocytosis, elevated fetal hemoglobin level, and elevated
erythrocyte adenosine deaminase level.
• There is an increased prevalence of congenital anomalies (eg, cardiac, genitourinary,
thumb) in children with Diamond-Blackfan anemia.
• Transient erythroblastopenia of childhood is a form of pure red blood cell aplasia that
most often occurs in very young children (typically >1 year of age) after a viral infection.
The condition spontaneously resolves within a few weeks.

ABP Content Specifications(s)


• Plan the appropriate management of transient erythroblastopenia of childhood
• Distinguish between the clinical findings of Diamond-Blackfan syndrome and transient
erythroblastopenia of childhood
• Recognize the clinical and laboratory findings associated with transient
erythroblastopenia of childhood

Suggested Readings
• Lanzkowsky P, Lipton J, Fish JD. Bone marrow failure. Lanzkowsky’s Manual of
Pediatric Hematology and Oncology. 6th ed. Elsevier; 2016:121-125.
• Noronha SA. Aplastic and hypoplastic anemias. Pediatr Rev. 2018;39(12):601-611.
doi:10.1542/pir.2017-0250.
• Segel GB, Hirsh MG, Feig SA. Managing anemia in pediatric office practice: part 1.
Pediatr Rev. 2002;23(3):75-84. doi:10.1542/pir.23-3-75.
• Vlachos A, Muir E. How I treat Diamond-Blackfan anemia. Blood. 2010;116(19):3715-
3723. doi:10.1182/blood-2010-02251090.

American Academy of Pediatrics 89


PREP® Self-Assessment PREPSA 2023

Question 27
A 13-year-old adolescent boy is being seen for follow-up after an urgent care visit several days
ago for 1 week of fatigue preceded by 2 days of fever and sore throat.

Results of the laboratory tests obtained at that visit are shown:


Laboratory Test Result
Complete blood cell count with Normal with mildly elevated atypical
differential lymphocytes
Rapid streptococcal antigen Negative
Heterophile antibody spot test for Positive
infectious mononucleosis
Epstein-Barr virus capsule antigen IgG Positive
Epstein-Barr virus capsule antigen IgM Positive
Epstein-Barr virus early antigen Negative
Epstein-Barr virus nuclear antigen Negative
Antinuclear antibody (ANA) titer 1:40
Thyrotropin Normal

The boy’s mother is very concerned about the ANA titer result because her husband’s sister was
recently diagnosed with systemic lupus erythematosus. The boy’s symptoms are significantly
improved. He has no significant medical history, and the results of a review of systems are
negative. His vital signs and findings on a physical examination today are normal.

Of the following, the BEST next step in this boy’s management is to

A. advise that the test result is likely due to a viral infection


B. obtain an antinuclear antibody profile evaluation
C. refer him to a pediatric rheumatologist
D. treat him with a 5-day course of corticosteroids

American Academy of Pediatrics 90


PREP® Self-Assessment PREPSA 2023

Correct Answers: A
Epstein-Barr virus infection. Low ANA titers (<1:320) are usually not indicative of an
autoimmune process, so he should not be referred to a rheumatologist at this time and should not
have an ANA profile evaluation performed, unless he exhibits other specific signs of
rheumatologic disease. He also does not need to receive corticosteroids for mononucleosis
because his symptoms have significantly improved.

Antinuclear antibodies are a collection of antibodies directed against various macromolecules


that reside in the cell nucleus. A positive ANA test result can be indicative of a rheumatologic
disease, but by itself is not diagnostic; for example, a positive ANA test result is one of the many
criteria required to make a diagnosis of systemic lupus erythematosus (SLE). Clinicians must be
careful in interpreting positive ANA titer results, which may be seen in up to one-third of healthy
children and may be associated with viral illnesses, such as mononucleosis and parvovirus.
Clinicians should order ANA titers only in the appropriate clinical context.

If, after an appropriate evaluation, a diagnosis of an autoimmune disease is highly suspected,


clinicians can use criteria developed for specific conditions to help guide further decision-
making. Some autoimmune diseases are, however, inherently difficult to diagnose because of
their naturally waning and waxing courses. Systemic lupus erythematosus is an example of a
diagnosis that may be delayed by years; 2 criteria exist for SLE to aid in making this diagnosis
(Item C27A and Item C27B). If there is strong suspicion for an autoimmune disease, a
diagnosis is often best made in consultation with appropriate subspecialists.

American Academy of Pediatrics 91


PREP® Self-Assessment PREPSA 2023

American Academy of Pediatrics 92


PREP® Self-Assessment PREPSA 2023

American Academy of Pediatrics 93


PREP® Self-Assessment PREPSA 2023

Viral infections (eg, EBV, cytomegalovirus, and parvovirus) can cause a transiently positive
ANA.

PREP Pearls
• Up to one-third of healthy children may have a positive antinuclear antibody titer result.
• Low antinuclear antibody titers (<1:320) are usually not indicative of an autoimmune
process and may be due to viral illnesses, such as mononucleosis or parvovirus.

ABP Content Specifications(s)


• Plan the appropriate diagnostic evaluation of systemic lupus erythematosus, and interpret
the results appropriately

Suggested Readings
• Dalrymple AM, Moore TL. Laboratory evaluation in pediatric autoimmune diseases.
Pediatr Rev. 2015;36(11):496-502. doi:10.1542/pir.36-11-496.
• Siegel DM, Gewanter HL, Sahai S. Rheumatologic diseases. In: McInerny TK, Adam
HM, Campbell DE, DeWitt TG, Foy JM, Kamat DM, eds. American Academy of
Pediatrics Textbook of Pediatric Care. American Academy of Pediatrics; 2021:chap 324.
Accessed September 1, 2022. Pediatric Care Online.
• Vehe RK, Riskalla MM. Collagen vascular diseases: SLE, dermatomyositis, scleroderma,
and MCTD. Pediatr Rev. 2018;39(10)501-515. doi:10.1542/pir.2017-0262.

American Academy of Pediatrics 94


PREP® Self-Assessment PREPSA 2023

Question 28
A 3-year-old girl is brought to the emergency department after drinking an unknown amount of
mouthwash. The girl’s mother found her with an empty bottle of mouthwash in the bathroom.
She is unsure how much had been in the bottle. The girl had 4 episodes of nonbloody, nonbilious
vomiting prior to arriving at the emergency department. She has a temperature of 36.7°C, heart
rate of 138 beats/min, respiratory rate of 12 breaths/min, and blood pressure of 108/66 mm Hg.
She appears lethargic and is poorly responsive to painful stimuli. The remainder of the girl’s
physical examination findings are unremarkable. Her airway, breathing, and circulation are
stabilized.

Of the following, the BEST next step in this girl’s management is to


A. administer activated charcoal
B. administer fomepizole
C. order an arterial blood gas analysis
D. order a bedside blood glucose level

American Academy of Pediatrics 95


PREP® Self-Assessment PREPSA 2023

Correct Answer: D
The girl in the vignette has ingested an unknown amount of mouthwash, which contains ethanol
(ethyl alcohol). In addition to beer, wine, and liquor, ethanol is found in a wide variety of
household items including hand sanitizer, liquid cough and cold medications, cooking extracts
(eg, vanilla), mouthwash, and perfumes/colognes. Because ethanol inhibits hepatic
gluconeogenesis, children (especially young children) who ingest ethanol can become
profoundly hypoglycemic. Therefore, the best next management step for the girl in the vignette is
to obtain a bedside blood glucose level.

The girl has signs of severe mental status alteration, therefore, administration of activated
charcoal is contraindicated due to aspiration risk. Although ethanol can cause anion-gap acidosis,
which would be detected on an arterial blood gas analysis, it is important to first address life-
threatening complications such as severe hypoglycemia and those affecting the airway,
breathing, and circulation. Fomepizole is the antidote for ethylene glycol and methanol toxicity;
it is not indicated for treatment of ethanol ingestion.

Signs and symptoms of ethanol ingestion or overdose include:


• Agitation
• Altered mental status
• Ataxia
• Central nervous system depression or coma
• Hypoglycemia
• Hypothermia
• Lethargy
• Nausea and vomiting
• Respiratory depression
• Seizures
• Slurred speech
• Tachycardia or bradycardia

The initial management of acute alcohol toxicity is focused on stabilizing the airway and
cardiopulmonary system. Respiratory depression and hypoglycemia are typically the most urgent
concerns. Prompt administration of glucose is indicated if hypoglycemia is present; monitoring
serial glucose levels until the child has recovered is a critical aspect of care. Hemodialysis may
be considered for children with toxic ingestions displaying evidence of end-organ damage or
severe acidosis that does not respond to conventional medical therapy.

Alcohol is the substance most widely abused by adolescents. The signs and symptoms of acute
ethanol overdose may mask those of other ingested toxic substances. It is important to consider
co-ingestion in older children and adolescents with a suspected intentional ingestion.

American Academy of Pediatrics 96


PREP® Self-Assessment PREPSA 2023

PREP Pearls
• In addition to beer, wine, and liquor, ethanol is found in a wide variety of household
products (eg, mouthwash, cough and cold medications) that are easily accessible to
children of all ages.
• Ethanol inhibits hepatic gluconeogenesis; children (especially young children) who ingest
ethanol can become profoundly hypoglycemic.
• Alcohol is the substance most widely abused by adolescents. It is important to consider
coingestion in an older child or adolescent with a suspected intentional ingestion.

MOCA-Peds Objective
• Evaluate and manage a child with hypoglycemia.

ABP Content Specifications(s)


• Recognize the signs and symptoms of ethanol intoxication, and manage appropriately
• Understand that ethanol intoxication may mask toxicity caused by ingestion of other
drugs

Suggested Readings
• Fein DM, Sue YJ. Methanol ingestion. Pediatr Rev. 2011;32(12):549-550.
doi:10.1542/pir.32.12.549.
• Fine JS. Poisoning. In: McInerny TK, Adam HM, Campbell DE, DeWitt TG, Foy JM,
Kamat DM, eds. American Academy of Pediatrics Textbook of Pediatric Care. American
Academy of Pediatrics; 2021:chap 369. Accessed September 1, 2022. Pediatric Care
Online.
• Khine H. Ethanol in over-the-counter drugs. Pediatr Rev. 1996;17(10):342-343.
doi:10.1542/pir.17.10.342.
• Sheridan DC, Hughes A, Horowitz Z. Adolescent ingestions: various reasons and
medications. Pediatr Rev. 2021;42(6):279-289. doi:10.1542/pir.2019-0310.
• Wang GS, Hoyte C. Common substances of abuse. Pediatr Rev. 2018;39(8):403-414.
doi:10.1542/pir.2017-0267.

American Academy of Pediatrics 97


PREP® Self-Assessment PREPSA 2023

Question 29
A pediatrician is called to an urgent cesarean delivery at 38 weeks’ gestation due to fetal
bradycardia. The 36-year-old mother’s first prenatal visit was at 37 weeks’ gestation, when
ultrasonography showed severe oligohydramnios; the fetal kidneys could not be visualized. The
findings were discussed with the mother, and she was scheduled to see the maternal-fetal
specialist but went into labor before the appointment. The clinician arrives for the delivery and
prepares for resuscitation. There was no opportunity to talk to the mother before delivery about
the fetus’ condition and plans for care.

Of the following, the BEST next management step after delivery will most likely be
A. administration of intravenous antibiotics
B. endotracheal intubation for positive-pressure ventilation
C. performance of renal ultrasonography
D. umbilical line placement for intravenous glucose

American Academy of Pediatrics 98


PREP® Self-Assessment PREPSA 2023

Correct Answers: B
Of the response choices, the best next management step after delivery of this neonate will most
likely be endotracheal intubation to provide positive-pressure ventilation. The fetal
ultrasonography findings are consistent with renal agenesis. Neonates born with bilateral renal
agenesis have significant pulmonary hypoplasia resulting from severe oligohydramnios. Fetal
urine contributes significantly to the amniotic fluid volume that is needed for fetal lung
development. Immediate resuscitative efforts include positive-pressure ventilation through an
endotracheal tube.

The combination of oligohydramnios and renal agenesis is known as Potter syndrome. These
neonates also have limb and facial anomalies due to maternal pelvic compression from the lack
of amniotic fluid. Neonates with Potter syndrome do not usually survive, even with the highest
levels of medically complex technological care. In an ideal situation, a maternal-fetal medicine
specialist would be consulted before delivery, at which time there would be discussion and
shared decision-making regarding the newborn’s diagnosis and prognosis. However, in this
vignette, the neonate was delivered before such a consultation could take place, therefore, it is
appropriate to provide initial resuscitation and stabilization and have that discussion when the
mother is awake and able to fully participate.

Although renal ultrasonography is needed to confirm the diagnosis of bilateral renal agenesis, it
should be performed after the neonate’s condition is stabilized.
Prenatally diagnosed cases of renal agenesis are often now managed at fetal health centers,
facilities specializing in complex fetal care. New fetal therapies are becoming available for the
management of renal agenesis (eg, amnioinfusion, long-term neonatal ventilation, neonatal
dialysis), creating opportunities for later transplantation and presenting physicians and families
with challenging ethical and moral questions.

Umbilical line placement for intravenous glucose may be needed, but would be performed after
endotracheal intubation and appropriate ventilation is provided. Similarly, a decision regarding
administration of intravenous antibiotics will be made later based on perinatal risk factors for
sepsis.

PREP Pearls
• Fetal urine contributes significantly to amniotic fluid volume.
• Amniotic fluid is necessary for fetal lung development.
• Fetal renal agenesis results in severe oligohydramnios which leads to pulmonary
hypoplasia.

ABP Content Specifications(s)


• Identify the features associated with the oligohydramnios tetrad (Potter syndrome), and
manage appropriately
• Recognize the renal findings associated with Potter syndrome (pulmonary hypoplasia)

American Academy of Pediatrics 99


PREP® Self-Assessment PREPSA 2023

Suggested Readings
• Feldenberg R, Beck A. Congenital diseases of the kidneys prognosis and treatments.
NeoReviews. 2017;18(6):e345e356. doi:10.1542/neo.18-6-e345.
• Janjua HS, Lam SK, Gupta V, Krishna S. Congenital anomalies of the kidneys, collecting
system, bladder and urethra. Pediatr Rev. 2019;40(12):619-626. doi:10.1542/pir.2018-
0242.
• Kelsey KM, Thorvilson MJ, Collura CA.mBilateral renal agenesis: redefining complex
palliative care across the continuum of a shifting disease paradigm. Pediatrics.
2020;146(1_MeetingAbstract):215-216. doi:10.1542/peds.146.1MA3.215b .
• Wolfe DS, Suskin B. Prenatal diagnosis. In: McInerny TK, Adam HM, Campbell DE,
DeWitt TG, Foy JM, Kamat DM, eds. American Academy of Pediatrics Textbook of
Pediatric Care. American Academy of Pediatrics; 2021:chap 82. Accessed September 1,
2022. Pediatric Care Online.
• Wong CJ. Fetal and neonatal renal failure and renal transplant. NeoReviews.
2016;17(12):e744-e752. doi:10.1542/neo.17-12-e744.

American Academy of Pediatrics 100


PREP® Self-Assessment PREPSA 2023

Question 30
A chief medical officer (CMO) discovers that a physician at the hospital is receiving
compensation for recommending a medical device to his patients. The CMO informs the
physician about the institution’s conflict of interest policies.

Of the following, the BEST next step for the CMO is to


A. require creation of a consulting agreement that stipulates that the physician must disclose
the financial arrangement to his patients
B. require creation of a document that discloses the financial relationship between the
physician and the company to his employer
C. require the physician and his staff to perform remedial training on the institution’s
relevant policies
D. terminate the physician from practice immediately for unethical behavior

American Academy of Pediatrics 101


PREP® Self-Assessment PREPSA 2023

Correct Answer: A
The physician in the vignette has a conflict of interest (COI). The Institute of Medicine in 2009
defined COI as “a set of circumstances that create a risk that professional judgment or actions
regarding a primary interest will be unduly influenced by a secondary interest.” Receiving
compensation for recommending a medical device to patients is a COI because medical
judgment could be influenced by secondary financial gain. The physician might be more inclined
to recommend the medical device even if it is not the best choice for the patient.
The best next step for the chief medical officer to mitigate the COI is to require creation of a
consulting agreement that stipulates that the physician must disclose the arrangement to his
patients. The agreement would require the physician to inform patients of the financial
relationship before prescribing the medical device. Those already using the medical device must
also be informed.

Creation of a document that discloses the financial relationship to his employer is also important.
However, the physician still must disclose the arrangement to his patients. Creation of a
disclosure document alone would not suffice. Education regarding institutional policies that
pertain to COIs is also important. However, education alone would not mitigate the COI that is
occurring in the vignette. Depending on institutional policy, disciplinary action for not
appropriately disclosing a COI may include termination. Disclosing the financial relationship to
patients who have already been prescribed the medical device, however, is still required to help
mitigate the COI.

Other examples of COIs include acceptance of gifts from industry (eg, meals and drug samples),
writing for or promoting an industrial product, and financial interest in a medical product that the
physician prescribes, uses, or recommends. Conflicts of interest can be managed by instituting
COI policies that address disclosure content, restricting participation of researchers with COIs,
and providing ongoing education at all levels of participation (faculty, students, residents, and
fellows).

PREP Pearls
• The Institute of Medicine in 2009 defined conflict of interest as “a set of circumstances
that create a risk that
• professional judgment or actions regarding a primary interest will be unduly influenced
by a secondary interest.”
• Examples of conflict of interest include acceptance of gifts from industry (eg, meals and
drug samples), writing for or promoting an industrial product, and financial interest in a
medical product that the physician prescribes, uses, or recommends.

ABP Content Specifications(s)


• Recognize and apply ethical principles regarding conflicts of interest

American Academy of Pediatrics 102


PREP® Self-Assessment PREPSA 2023

Suggested Readings
• Muth CC. Conflict of interest in medicine. JAMA. 2017;317(17):1812.
doi:10.1001/jama.2017.4044.
• US Institute of Medicine Committee on Conflict of Interest in Medical Research,
Education, and Practice; Lo B, Field MJ, eds. Conflicts of interest and medical practice.
In: Conflict of Interest in Medical Research, Education, and Practice. National
Academies Press; 2009:6.
• Wilfond BS. Duenas DM, Johnson L-M. Conflicts of interest and recommendations for
clinical treatments that benefit researchers. Am J Bioethics. 2020;20(10):90-91.
doi:10.1080/15265161.2020.1806380.

American Academy of Pediatrics 103


PREP® Self-Assessment PREPSA 2023

Question 31
A 16-year-old adolescent boy is seen for follow-up the morning after an emergency department
visit for an asthma exacerbation. The adolescent has known asthma and allergies to grass and
ragweed. Although prescribed daily, he takes his controller medication only as needed and has
not felt the need to use any recently. Yesterday, he helped a friend mow a large field. About 15
minutes after starting to cut the tall grass he felt chest tightness and began to cough. He used his
albuterol inhaler, which brought some relief, and took 2 to 4 puffs several times over the next 4
hours until the job was done. After going home and showering he felt better for a while. About 7
to 8 hours after he started cutting the grass he developed wheezing that was not fully relieved by
his albuterol inhaler, prompting a visit to the emergency department. In the emergency
department he was treated with oral corticosteroids and bronchodilators. Today, he feels much
better, and his physical examination findings are normal.

Of the following, the BEST next step in this adolescent’s management is


A. an albuterol inhaler as needed and before known allergen exposure
B. as-needed use of inhaled corticosteroid with an albuterol inhaler
C. daily and as-needed use of a combined budesonide/formoterol inhaler
D. daily use of inhaled tiotropium and formoterol

American Academy of Pediatrics 104


PREP® Self-Assessment PREPSA 2023

Correct Answer: C
The adolescent in the vignette had a biphasic asthma response to allergen exposure, with early
bronchospasm amenable to bronchodilator treatment followed hours later by a more intense
airway inflammatory reaction responsive only to corticosteroids. This biphasic response is
characteristic of IgE-mediated allergic asthma. The best next management step for this
adolescent is daily and as-needed use of a combined budesonide/formoterol inhaler. A combined
inhaled corticosteroid/long-acting β-agonist (ICS/LABA) inhaler used daily and additionally
before allergen exposure prevents both early and late responses in those with IgE-mediated
allergic asthma. Formoterol is an LABA appropriate for use in asthma management only when
combined with an ICS. It is the only LABA with a short enough onset of action to provide acute
relief, making it appropriate for as-needed administration (unlike salmeterol). Under the Same
Maintenance And Rescue Therapy (SMART) paradigm from the 2020 National Asthma
Education and Prevention Program guidelines, the combination of an ICS with formoterol as the
LABA is appropriate for both daily maintenance and as-needed treatment.

Inhaled bronchodilators both prevent and treat the immediate IgE-mediated asthma reaction, but
alone they are inadequate to prevent and treat the late inflammatory reaction. Inhaled
corticosteroids prevent the late response but have no effect on the early bronchospastic reaction.
The use of an ICS after the onset of a reaction, with or without a bronchodilator, is unlikely to
prevent the delayed response.

Tiotropium is an anticholinergic agent indicated as an add-on controller therapy for moderate to


severe asthma not adequately controlled by an ICS, with or without a LABA. It does not have
antiinflammatory properties, so it would not prevent a delayed response. Combined use of
inhaled tiotropium and formoterol would engage 2 different mechanisms for bronchodilation, but
this combination lacks the anti-inflammatory properties of an ICS, making it a poor choice to
prevent and treat IgE-mediated asthma.

The biphasic response of IgE-mediated allergic asthma is physiologically similar to the waves of
symptoms in biphasic anaphylaxis triggered by IgE. In IgE-mediated allergic asthma, histamine
and leukotrienes released from airway mast cells act directly on airway smooth muscle to cause
bronchospasm within a few minutes of exposure to a triggering allergen. Additional mediators
promote chemotaxis of eosinophils and neutrophils which invade the airway submucosa over the
next several hours and set up the inflammatory cascade responsible for the delayed
bronchospasm. Mediators generated as part of the late response perpetuate the cycle of
bronchospasm and inflammation, which can continue for several days with waves of cough,
wheezing, and chest tightness unless pharmacologically interrupted. This prolonged process does
not occur with every exposure in everyone with allergic asthma, but in controlled laboratory
challenges, the biphasic response is demonstrable even in those with mild asthma.

Other therapies beneficial in preventing the biphasic IgE-mediated response include injections of
the anti-IgE monoclonal antibody, omalizumab, and oral administration of a leukotriene modifier
drug. Omalizumab binds to circulating IgE and prevents the linkage of mast-cell bound allergens
by IgE, preventing the initiation of the bronchospastic and inflammatory cascade. Leukotriene
American Academy of Pediatrics 105
PREP® Self-Assessment PREPSA 2023

mediators are important in both the early and late asthma responses, so leukotriene modifier
drugs can help interrupt the inflammatory cascade. However, in laboratory challenges these
drugs have not been uniformly successful, highlighting the complexity of the process. Inhaled
corticosteroids are more effective than the leukotriene modifiers in most allergic asthma
challenge studies. The leukotriene modifier drugs are best used as alternative primary treatment
for mild asthma or as secondary or additional treatment with an ICS in moderate or severe
asthma. It is important to note that the US Food and Drug Administration issued a boxed
warning for montelukast for serious mental health side effects.

PREP Pearls
• IgE-mediated allergic asthma is characterized by a biphasic response including early
bronchospasm and delayed airway inflammation.
• The early response in IgE-mediated allergic asthma is amenable to prevention and
treatment with bronchodilators, but corticosteroids are the most effective therapy for both
prevention and treatment of the delayed inflammatory response.
• Injectable anti-IgE monoclonal antibodies (eg, omalizumab) can prevent both the early
and late phases of allergic asthma by binding circulating IgE and preventing its binding to
allergens on mast cells.

MOCA-Peds Objective
• Plan the appropriate use of controller therapy in a patient with asthma.
• Manage a child with an acute asthma exacerbation.

ABP Content Specifications(s)


• Recognize the early and late effects of an IgE-mediated allergen in a patient with asthma

American Academy of Pediatrics 106


PREP® Self-Assessment PREPSA 2023

Suggested Readings
• Dinakar C. Asthma. In: McInerny TK, Adam HM, Campbell DE, DeWitt TG, Foy JM,
Kamat DM, eds. American Academy of Pediatrics Textbook of Pediatric Care. American
Academy of Pediatrics; 2021:chap 239. Accessed September 1, 2022. Pediatric Care
Online.
• Djukanovic R, Hanania N, Busse W, Price D. IgE-mediated Asthma: new revelations and
future insights. Respir Med. 2016;112:128-129. doi:10.1016/j.rmed.2015.10.013.
• Expert Panel Working Group of the National Heart, Lung, and Blood Institute (NHLBI)
administered and coordinated National Asthma Education and Prevention Program
Coordinating Committee (NAEPPCC); Cloutier MM, Baptist AP, Blake KV, et al. 2020
focused updates to the asthma management guidelines: a report from the National
Asthma Education and Prevention Program Coordinating Committee Expert Panel
Working Group. J Allergy Clin Immunol. 2020;146(6):12171270.
doi:10.1016/j.jaci.2020.10.003.
• Holgate S, Casale T, Wenzel S, et al. The anti-inflammatory effects of omalizumab
confirm the central role of IgE in allergic inflammation. J Allergy Clin Immunol.
2005;115(3):459-465. doi:10.1016/j.jaci.2004.11.053.
• Patel SJ, Teach SJ. Asthma. Pediatr Rev. 2019;40(11):549-567. doi:10.1542/pir.2018-
0282.

American Academy of Pediatrics 107


PREP® Self-Assessment PREPSA 2023

Question 32
A 14-year-old adolescent girl is seen in the office for a health supervision visit. Her menarche
was 1 year ago; her cycles are irregular, with her last menstrual period occurring 5 weeks before
this visit. After her first cycle, she did not have any bleeding for about 40 days; since then, her
cycle intervals have ranged from 25 to 45 days. Her blood flow lasts for 4 to 6 days with the use
of 2 to 3 pads per day. The adolescent is otherwise healthy with no history of medical problems.
She is concerned about her irregular cycles. Her height is 165 cm (75th percentile) and weight is
50 kg (50th percentile); the percentiles are similar to those documented at her annual visit a year
ago. Her physical examination findings are normal.

Of the following, the BEST explanation for this adolescent’s irregular menstrual cycles is
A. anorexia nervosa
B. hypothyroidism
C. immature hypothalamic-pituitary axis
D. polycystic ovarian syndrome

American Academy of Pediatrics 108


PREP® Self-Assessment PREPSA 2023

Correct Answer: C
It is common for adolescents to have irregular menstrual cycles, especially in the first 2 years
after menarche. This phenomenon is due to immaturity of the hypothalamic-pituitary axis, which
leads to anovulatory cycles. The interval between menstrual cycles can range from 21 to 45 days;
5% to 10% of girls have intervals shorter than 20 days or longer than 60 days. Prolonged
intervals are most commonly seen between the first and second cycle. Bleeding normally occurs
for 3 to 7 days. After the initial 2 years, most adolescents begin to have more regular cycles with
intervals of 21 to 34 days, typical of adult women.

Irregular cycles can be either prolonged or shortened. Bleeding may be excessive. Causes of a
prolonged interval between cycles include anovulatory cycles, pregnancy, polycystic ovarian
syndrome (PCOS), hypothyroidism, prolactinoma, and eating disorders. Shortened intervals
between cycles may be the result of an immature hypothalamic-pituitary axis, systemic disorders,
and medications. It is important to distinguish irregular menstrual cycles from secondary
amenorrhea, which is defined as the cessation of regular menstrual cycles for 6 months.
Clinicians should start evaluating for an etiology when menstrual cycles have been absent for 3
months. Although a cause is not always identified, excessive bleeding can be the result of a
coagulation disorder; von Willebrand disease is a common cause.

For the girl in the vignette, PCOS is unlikely because of the absence of features of
hyperandrogenism and hyperinsulinism. The fact that her growth percentiles remain unchanged
from her previous visit is not consistent with a diagnosis of hypothyroidism or an eating disorder.

PREP Pearls
• For the first 2 years after menarche, irregular menstrual cycles are normal because of the
immature hypothalamic-
• pituitary axis leading to anovulatory cycles.
• The typical interval between menstrual cycles in adolescents ranges from 21 to 45 days ,
with an average blood flow duration of 3 to 7 days.
• Although normal, irregular menstrual cycles after menarche are a frequent cause of
anxiety among young girls and should be addressed.

ABP Content Specifications(s)


• Recognize normal variations in the menstrual cycle in adolescent girls

American Academy of Pediatrics 109


PREP® Self-Assessment PREPSA 2023

Suggested Readings
• ACOG Committee Opinion No. 651: menstruation in girls and adolescents: using the
menstrual cycle as a vital sign. Obstet Gynecol. 2015;126(6):e143-e146.
doi:10.1097/AOG.0000000000001215.
• American Academy of Pediatrics Committee on Adolescence; American College of
Obstetricians and Gynecologists Committee on Adolescent Health Care; Diaz A, Laufer
MR, Breech LL. Menstruation in girls and adolescents: using the menstrual cycle as a
vital sign. Pediatrics. 2006;118(5):2245-2250. doi:10.1542/peds.2006-2481.
• Gray SH. Menstrual disorders. Pediatr Rev. 2013;34(1):6-18. doi:10.1542/pir.34-1-6 .
• Trent M, Joffe A. Vaginal bleeding. In: McInerny TK, Adam HM, Campbell DE, DeWitt
TG, Foy JM, Kamat DM, eds. American Academy of Pediatrics Textbook of Pediatric
Care. American Academy of Pediatrics; 2021:chap 204. Accessed September 1, 2022.
Pediatric Care Online.

American Academy of Pediatrics 110


PREP® Self-Assessment PREPSA 2023

Question 33
An 8-year-old boy with attention-deficit/hyperactivity disorder, combined type, is brought to the
office for medication follow-up. His mother reports that the stimulant medication is working
well, but she and his teacher are concerned because the boy’s grades have dropped and his
frustration with school has increased since entering the third grade. He takes up to 2 hours to
complete his homework. His mother tries to limit distractions by having the television off and
sitting with him while he works, but it has not helped. The boy’s teacher allows extra time for
him to complete his work, and he takes tests in a quiet office to limit distractions. He has been
much less hyperactive and impulsive since starting the medication. The boy’s physical
examination findings are unremarkable.

Of the following, the BEST next step in this boy’s management is to


A. increase the dose of his stimulant medication
B. recommend a school evaluation for a learning disability
C. recommend parent behavior-management training
D. request a 504 plan through the school for additional accommodations

American Academy of Pediatrics 111


PREP® Self-Assessment PREPSA 2023

Correct Answer: B
The boy in the vignette, with known attention-deficit/hyperactivity disorder (ADHD), is
exhibiting signs of a learning disability. When he advanced to a higher grade in school, he began
to have significant academic difficulty (poor grades, frustration with school, and prolonged time
to complete homework despite behaviorally effective treatment with a stimulant medication).
Common comorbidities in children with ADHD include learning disabilities, anxiety, depression,
and oppositional defiant disorder. To assess the boy for a learning disability, an evaluation that
includes both cognitive function and academic achievement should be performed.

Attention-deficit/hyperactivity disorder is the most common behavioral disorder in children, with


a prevalence of 7%; it is a chronic and lifelong condition. The neurochemical basis of ADHD is
thought to be a deficiency of the neurotransmitter norepinephrine. Some individuals with ADHD
have structural and functional differences in several areas of the brain including the prefrontal
cortex (controls attention, organization, and executive functioning), the limbic system (controls
emotional regulation), and basal ganglia (regulates communication within the brain).

There are 3 subtypes of ADHD:


• Predominantly inattentive presentation
• Predominantly hyperactive-impulsive presentation
• Combined presentation

A diagnosis of ADHD requires that the child or adolescent meet the Diagnostic and Statistical
Manual of Mental Disorders, Fifth Edition (DSM-5) criteria by exhibiting at least 6 out of 9
symptoms/behaviors in either the inattentive or hyperactive-impulsive categories (for primarily
inattentive or primarily hyperactive-impulsive type), or symptoms in both categories for the
combined type. These symptoms must persist for at least 6 months, be present
in 2 or more settings, and result in functional impairment (social, academic, occupational).
Confirmation of these criteria can be made with parent and teacher behavior ratings forms; these
same forms can be utilized to assess a child’s response to medication and need for dose
adjustment.

Once a diagnosis of ADHD has been established, management depends on the age of the child.
For preschool-aged children (4 to 6 years), behavior management training (specifically, parent
management training) is the first-line treatment. Parent management training teaches
developmentally appropriate expectations, how to communicate about emotions, and promotes
positive parent–child interaction skills. Parent management training would not be the best next
step in management of the boy in the vignette, as he is of school age, and this approach would
not address his learning challenges.

For school-age children, medication along with parent-training and/or behavioral classroom
interventions are recommended. Stimulant medications are recommended over non-stimulants
(eg, atomoxetine, extended-release guanfacine, extended-release clonidine). Medication dosage
is not weight dependent. A low dose should be started and slowly increased to maximize benefit
and limit side effects. The most common side effects of stimulant medications include decreased
American Academy of Pediatrics 112
PREP® Self-Assessment PREPSA 2023

appetite, headache, upset stomach, and sleep disturbance. Heart rate and blood pressure can
increase with stimulant medication. Prior to initiating a stimulant medication, any family history
of sudden death, hypertrophic cardiomyopathy, or arrhythmia should be elicited. If the family
history reveals any of these conditions or there are abnormal cardiac findings on examination, an
electrocardiogram (ECG) should be obtained. Otherwise, an ECG is not routinely indicated.
Children with ADHD may be eligible for additional support through their school system. A 504
Rehabilitation Act Plan (504 plan) for a child or adolescent with ADHD may include the
following accommodations:
• Extended time to complete work and tests
• Reduced assignments
• Use of notes during tests
• A quiet space in which to take tests

When a learning disability is identified, an individualized education plan (IEP) must be


developed. The IEP defines the educational interventions (eg, small group/pull-out instruction) to
be put in place and ensures that progress is closely monitored through an annual review. The boy
in the vignette is receiving accommodations in the classroom for his ADHD, but is still
struggling academically. A 504 plan may add to or modify these accommodations, but would not
provide an evaluation to assess him for a learning disability or allow for an IEP to be created.

PREP Pearls
• Effective treatment of attention-deficit/hyperactivity disorder varies by age and includes
both medication and behavioral therapy.
• Ongoing assessment for comorbidities is an important part of attention-
deficit/hyperactivity disorder management.
• 504 accommodations may provide enough support for some individuals with attention-
deficit/hyperactivity disorder, but if there is also a learning disability resulting in
functional impairment, an evaluation through the school is required to determine the need
for educational intervention(s).

ABP Content Specifications(s)


• Understand the neurochemical basis of ADHD
• Plan the appropriate diagnostic evaluation of ADHD
• Plan the appropriate management of ADHD

American Academy of Pediatrics 113


PREP® Self-Assessment PREPSA 2023

Suggested Readings
• Leslie LK, Guevara JP. Attention-deficit/hyperactivity disorder. In: McInerny TK, Adam
HM, Campbell DE, DeWitt TG, Foy JM, Kamat DM, eds. American Academy of
Pediatrics Textbook of Pediatric Care. American Academy of Pediatrics; 2021:chap 220.
Accessed September 1, 2022. Pediatric Care Online.
• Rajaprakash M, Leppert ML. Attention-deficit/hyperactivity disorder. Pediatr Rev.
2022;43(3):135-147. doi:10.1542/pir.2020-000612.
• Wolraich ML, Chan E, Froehlich T, et al. ADHD diagnosis and treatment guidelines: a
historical perspective. Pediatrics. 2019;144(4):e20191682. doi:10.1542/peds.2019-1682.
• Wolraich ML, Hagan JF Jr, Allan C, et al. Clinical practice guideline for the diagnosis,
evaluation, and treatment of attention-deficit/hyperactivity disorder in children and
adolescents. Pediatrics. 2019;144(4):e20192528. doi:10.1542/peds.2019-2528.

American Academy of Pediatrics 114


PREP® Self-Assessment PREPSA 2023

Question 34
A 3-month-old male infant born at term is admitted to the hospital with a 2-week history of
fevers without a source.

Laboratory data are shown:


Laboratory test Result
White blood cell count 1,500/µL (1.5 × 109/L)
Hemoglobin 8.0 g/dL (80 g/L)
Platelet count 50 × 103/µL (50 × 109/L)
Alanine aminotransferase 5,500 U/L
Aspartate aminotransferase 4,500 U/L
Serum bilirubin 6 mg/dL (102.6 µmol/L)
Serum triglycerides 500 mg/dL (5.7 mmol/L)
Erythrocyte sedimentation rate 150 mm/h
Serum ferritin 12,000 ng/mL (12,000 µg/L)
Interleukin 2 soluble receptor 2,000 pg/mL (reference range,
levels 175.3-858.2 pg/mL)

Of the following, the MOST likely physical examination finding expected in this condition is
A. hearing loss
B. snuffles
C. splenomegaly
D. vesicular rash

American Academy of Pediatrics 115


PREP® Self-Assessment PREPSA 2023

Correct Answer: C
The infant in the vignette has prolonged fevers, cytopenia, evidence of inflammation, elevated
transaminases, and elevated soluble interleukin 2 (IL-2) receptor levels. This clinical
presentation is consistent with hemophagocytic lymphohistiocytosis (HLH). Although seen in all
age groups, the highest incidence is in infants younger than 3 months. This condition can be
familial or acquired. The most common infectious trigger of acquired HLH is Epstein-
Barr virus (EBV), and the most common physical examination finding is splenomegaly (89% of
cases).

Cytomegalovirus (CMV) and herpes simplex viruses (HSV) are much less common triggers for
HLH than EBV. The clinical manifestations of CMV (human herpesvirus 5) depend on the age
and immunocompetent status of the host, ranging from asymptomatic infection in children to a
mononucleosis-like illness associated with hepatitis in adolescents and adults. Congenital CMV
is the most common cause of sensorineural hearing loss in the United States; only 10% of
congenital infections are apparent at birth. The clinical manifestations and severity of HSV
infection also depend on age and host factors. In the newborn, HSV can lead to serious infection;
45% of cases manifest as skin, eye, and mouth (SEM) disease, 30% as central nervous system
disease, and 25% as disseminated infection. Eighty percent of neonates with SEM have a
vesicular rash.

Snuffles (copious nasal secretions) is one of the clinical manifestations of congenital syphilis
along with splenomegaly, lymphadenopathy, and a maculopapular rash with small, dark red
copper spots on hands and feet. Syphilis is not a trigger for HLH in children.

Epstein-Barr virus (human herpesvirus 4) is a DNA virus that affects the B cells. Unlike other
herpes viruses, EBV is easily degradable and has a latent as well as a lytic cycle. Epstein-Barr
virus is shed in oropharyngeal secretions and transmitted through close personal contact.
Manifestations of EBV infection depend on host factors such as genetic predisposition, age, and
immunocompetent status (Item C34). Young children who acquire EBV infection usually are
asymptomatic or have a mild disease course. Infected adolescents and young adults with primary
EBV infection develop infectious mononucleosis (IM). Major clinical manifestations of IM
include fever, pharyngitis, and lymphadenopathy. Generalized lymphadenopathy is a hallmark of
infectious mononucleosis, with anterior and posterior cervical regions being the most common
sites involved. Hepatosplenomegaly, jaundice, and rash can also be seen in acute infectious
mononucleosis.

Epstein-Barr virus–associated diseases affecting immunosuppressed children include:


• Posttransplantation lymphoproliferative disorder: Children who have undergone solid
organ transplantation and hematopoietic stem cell transplantation are at risk. The
incidence is highest in the first year after transplantation. Increasing levels of EBV viral
load correlate with development of posttransplantation lymphoproliferative disorder. The
clinical manifestations range from self-limiting, mild, IM-like illness to lifethreatening
solid tumors similar to large cell non-Hodgkin lymphoma.

American Academy of Pediatrics 116


PREP® Self-Assessment PREPSA 2023

• Manifestations in children with HIV infection:


o lymphocytic interstitial pneumonitis
o oral “ hairy” leukoplakia
o leiomyosarcoma

• X-linked lymphoproliferative syndrome: Occurs in children with an X-linked genetic


predisposition, most commonly due to a genetic defect in the signal lymphocyte
activation molecule associated protein (SAP). More than 90% of these children have
evidence of hemophagocytosis (phagocytosis of red blood cells, lymphocytes by
macrophages or histiocytes). The major manifestations of X-linked lymphoproliferative
syndrome include fatal IM, lymphoma, or HLH.

• Hemophagocytic lymphohistiocytosis:
a. Familial: due to a gene mutation in perforin; more common in infants younger than 1
year with X-linked lymphoproliferative syndrome; mean age of diagnosis is 5
years.
b. Acquired: most commonly triggered by infection; the majority are secondary to EBV
infection; mean age at diagnosis is 4 years. It also occurs in children with
underlying rheumatologic disease or malignancy.

American Academy of Pediatrics 117


PREP® Self-Assessment PREPSA 2023

PREP Pearls
• Clinical manifestations of Epstein-Barr virus (EBV) infection depend on the age and
immunocompetent status of the host. Young children with EBV are usually asymptomatic
or have mild disease; adolescents and young adults with primary EBV infection develop
infectious mononucleosis.
• Hemophagocytic lymphohistiocytosis is a rare clinical entity seen in genetically
predisposed children. Epstein-Barr virus is the most common infectious trigger.
• The most common findings in children with hemophagocytic lymphohistiocytosis are
splenomegaly; cytopenias; and elevated transaminases, inflammatory markers,
triglycerides, ferritin, and soluble interleukin 2 receptor levels.

ABP Content Specifications(s)


• Understand the importance of host factors in the outcome of Epstein-Barr virus infection
• Understand the epidemiology of Epstein-Barr virus

Suggested Readings
• American Academy of Pediatrics. Epstein-Barr virus infections (infectious
mononucleosis). In: Kimberlin DW, Barnett ED, Lynfield R, Sawyer MH, eds. Red
Book: 2021–2024 Report of the Committee on Infectious Diseases. 32nd ed. American
Academy of Pediatrics; 2021. Accessed September 1, 2022. Red Book Online.
• Jenson HB. Epstein-Barr virus. Pediatr Rev. 2011;32(9):375-383; quiz 384.
doi:10.1542/pir.32-9-375.
• Katz BZ. Epstein-Barr virus (mononucleosis and lymphoproliferative disorders). In:
Long SS, Prober CG, Fischer M, eds. Principles and Practice of Pediatric Infectious
Diseases. 5th ed. Elsevier; 2018:1088-1095.
• Krilov LR. Infectious mononucleosis and other Epstein-Barr viral infections. In:
McInerny TK, Adam HM, Campbell DE, DeWitt TG, Foy JM, Kamat DM, eds.
American Academy of Pediatrics Textbook of Pediatric Care. American Academy of
Pediatrics; 2021:chap 275. Accessed September 1, 2022. Pediatric Care Online.
• Pinninti S, Hough-Telford C, Pati S, Boppana S. Cytomegalovirus and Epstein-Barr virus
infections. Pediatr Rev. 2016;37(6):223-234. doi:10.1542/pir.2015-0072.

American Academy of Pediatrics 118


PREP® Self-Assessment PREPSA 2023

Question 35
A 6-year-old boy is brought to the office for school concerns. After an evaluation by the school
his mother was told that he has an intellectual disability. She was surprised to hear this diagnosis
as she thought the testing was “just for his reading.”
Review of the boy’s medical record reveals that he received early intervention services (speech
and occupational therapy) beginning at age 24 months to address limited speech, difficulties with
independent feeding, and fine motor skills. The boy’s mother decided to “take a break from
therapies to let him catch up” after he aged out of early intervention at 3 years of age. During all
subsequent health supervision visits, global impairments in speech, fine motor skills, problem
solving, and adaptive skills were documented, and the boy’s mother was strongly advised to have
him evaluated by the school when he entered prekindergarten.

The boy is not able to dress independently, is unable to tell a story, struggles with verbal
instructions involving multiple steps, and prefers to play with younger children. He was toilet
trained at age 5 years. The boy’s teacher raised academic concerns including his inability to
identify letters or numbers, or write his name. Behavioral concerns at school include inattention,
hyperactivity, frequent emotional outbursts, and rare aggressive behaviors.
The boy’s mother reports that her brother did not start talking until he was 5 years old, struggled
with learning to read, required special education services, and did not graduate from high school.
The boy’s mother does not want to “let the same thing happen” to her son. She plans to have him
retained in the first grade because she feels that he is “not trying hard enough” and “only needs
help with reading.”

Of the following, the finding that MOST supports the school’s diagnosis is the
A. family history of speech and learning difficulties
B. history of inattention, hyperactivity, and externalizing behaviors
C. receipt of speech therapy at an early age
D. significant impairments in cognitive and adaptive functioning from an early age

American Academy of Pediatrics 119


PREP® Self-Assessment PREPSA 2023

Correct Answer: D
The boy in the vignette has an intellectual disability. He has a history of developmental delays
from a young age, and these impairments have persisted in all areas, including cognitive and
adaptive functioning, thus supporting the diagnosis made with the formal school evaluation. The
other response choices are not specifically associated with intellectual disability.

Individuals with intellectual disability frequently have delayed acquisition of receptive and
expressive language, fine motor, and adaptive skills; deficits in problem solving; social
immaturity; and behavioral difficulties. Levels of severity of intellectual disability include mild,
moderate, severe, and profound. Individuals with mild intellectual disability may not be seen
with difficulties until early school age, whereas most individuals with severe to profound
intellectual disability are identified in the first year of life.

Although children with learning disabilities and intellectual disability can both display
behavioral challenges, those with a learning disability do not have impairments in cognitive and
adaptive functioning. A child with a reading disability or dyslexia may have a history of delayed
speech and language milestones, and may have received speech therapy; however, they would
have achieved other cognitive and adaptive milestones appropriately. At school entry, a child
with a reading disability or dyslexia may struggle with letter/word recognition and decoding new
words. These struggles may progress to deficits in reading fluency and comprehension.
Previously, learning disabilities were diagnosed when there was a discrepancy between
intellectual ability and academic performance. More recently, with the introduction of the
“response to intervention” approach, those individuals with a suspected learning disability are
provided with increasing levels of support and interventions with close monitoring of their
progress. For those individuals who show limited progress until significant supports are in place,
a classification of learning disability is typically given.

When a child has school performance concerns, information should be obtained regarding
attainment of developmental milestones, social history, birth history, past medical history, and
family history (family members with similar difficulties or concerns). For the boy in the vignette,
it is unclear from the history provided if his uncle had a reading disability or intellectual
disability.

Pediatricians provide ongoing developmental surveillance within the medical home and support
the identification of children requiring further evaluation. Psychoeducational assessment is
needed for both the diagnosis of intellectual disability and learning disabilities. A developmental
history of global impairment with deficits in both cognitive and adaptive functioning
differentiates intellectual disability from a learning disability. Behavioral challenges, frequently
seen with both diagnoses, indicate the need for further assessment. Grade retention has not been
shown to be beneficial for children with intellectual disability or learning disabilities and should
be discouraged.

American Academy of Pediatrics 120


PREP® Self-Assessment PREPSA 2023

PREP Pearls
• Children with learning disabilities do not have the impairments in both cognitive and
adaptive functioning seen in children with intellectual disability.
• Children with intellectual disability and as well as those with a learning disability may
exhibit behavioral difficulties.
• Children with mild intellectual disability may not be identified until school age.

ABP Content Specifications(s)


• Distinguish the findings associated with learning disabilities from those of intellectual
disabilities

Suggested Readings
• Phelps RA, Cohen WI. Intellectual disability. In: McInerny TK, Adam HM, Campbell
DE, DeWitt TG, Foy JM, Kamat DM, eds. American Academy of Pediatrics Textbook of
Pediatric Care. American Academy of Pediatrics; 2021:chap 278. Accessed September 1,
2021. Pediatric Care Online.
• Purugganan O. Intellectual disabilities. Pediatr Rev. 2018;39(6):299-309.
doi:10.1542/pir.2016-0116.
• Rey-Casserly C, McGuinn L, Lavin A; Committee on Psychosocial Aspects of Child and
Family Health, Section on Developmental and Behavioral Pediatrics. School-aged
children who are not progressing academically: considerations for pediatricians.
Pediatrics. 2019;144(4):e20192520. doi:10.1542/peds.2019-2520.
• Rimrodt SL, Lipkin PH. Learning disabilities and school failure. Pediatr Rev.
2011;32(8):315-324. doi:10.1542/pir.32-8315.

American Academy of Pediatrics 121


PREP® Self-Assessment PREPSA 2023

Question 36
A 4-day-old male infant is brought to the emergency department for evaluation of fast breathing
and poor feeding. He was born at term via vaginal delivery and went home from the hospital the
next day. His mother’s breast milk came in a couple of days ago, and he had been eating well
until 8 hours before presentation. He latches well but seems to be breathing too quickly to suck
and swallow. On physical examination, his heart rate is 170 beats/min, respiratory rate is 70
breaths/min, blood pressure is 70/40 mm Hg in the right arm, and oxygen saturation is 65% in
room air measured in the right hand. He is tachypneic but not in distress. His lungs are clear to
auscultation bilaterally, heart sounds are a normal S1 and a single S2 with a II/VI holosystolic
murmur at the left sternal border, abdomen is benign, and capillary refill is brisk. A chest
radiograph is obtained (Item Q36).

Of the following, the BEST next management step for this neonate is to
A. initiate prostaglandin infusion
B. intubate for impending respiratory failure
C. perform echocardiography
D. perform electrocardiography

American Academy of Pediatrics 122


PREP® Self-Assessment PREPSA 2023

Correct Answers: A
The best next management step for the neonate in the vignette, who has tachypnea, desaturation,
and poor feeding, is to initiate treatment with prostaglandin. His physical examination findings
are significant for tachycardia, tachypnea, clear lungs, a single second heart sound, and a
holosystolic murmur. His chest radiograph demonstrates a right aortic arch and a boot-shaped
heart. These findings are concerning for congenital heart disease with insufficient pulmonary
blood flow. Prostaglandin needs to be initiated urgently to open the closing ductus arteriosus to
increase pulmonary blood flow and increase the oxygen saturation. Echocardiography and
electrocardiography will be required to understand the details of his cardiac diagnosis, but they
can be performed after the prostaglandin infusion is started. There is no indication for intubation
of this child at this time.

The mnemonic “5 Ts and H,” which represents truncus arteriosus, tricuspid atresia, transposition
of the great arteries, tetralogy of Fallot, total anomalous pulmonary venous connection, and
hypoplastic left heart syndrome, is helpful for remembering the types of cyanotic congenital
heart disease. However, knowledge of the physiology of each patient is needed to truly
understand their expected oxygen saturations. Children who are cyanotic because of their cardiac
anatomy have insufficient pulmonary blood flow, a mixing lesion, or transposition physiology.
Children with obstruction to systemic blood flow will have signs of shock and decreased cardiac
output; their oxygen saturations can be variable.

The term “critical congenital heart disease” means that the child’s circulation is dependent on a
patent ductus arteriosus. Examples of ductal-dependent pulmonary blood flow include
pulmonary atresia or critical pulmonary valve stenosis. Ductal-dependent systemic blood flow
occurs in hypoplastic left heart syndrome. In transposition of the great arteries, 2 parallel
circulations are seen. The deoxygenated blood returns to the heart from the systemic
veins to the right atrium, into the right ventricle, and exits the aorta. The oxygenated blood
returns to the heart from the pulmonary veins to the left atrium, into the left ventricle, and out of
the pulmonary artery. There needs to be a means of mixing for oxygenated blood to get to the
body. Patency of the ductus arteriosus allows for shunting of blood from the aorta to the
pulmonary artery, which increases the blood returning to the left atrium and increases shunting
across an atrial level defect. This can be sufficient to maintain adequate oxygen saturation in
these patients. When the atrial defect size is insufficient, balloon atrial septostomy is required.
Patency of the ductus arteriosus is achieved with a continuous infusion of prostaglandin. When
the ductus is already closed at presentation, higher doses will be required. Side effects, which can
be dose dependent, may include apnea, fever, tachycardia, vasodilation, and hypotension.

PREP Pearls
• Congenital cyanotic heart disease occurs when there is insufficient pulmonary blood
flow, intracardiac mixing, or transposition physiology.
• Prostaglandin infusion maintains patency of the ductus arteriosus. Higher doses are
needed when the ductus has closed.
• The side effects of prostaglandin infusion include apnea, fever, tachycardia, vasodilation,
and hypotension.
American Academy of Pediatrics 123
PREP® Self-Assessment PREPSA 2023

ABP Content Specifications(s)


• Understand the role of the ductus arteriosus in cyanotic congenital heart disease, and
manage appropriately

Suggested Readings
• Desai K, Rabinowitz EJ, Epstein S. Physiologic diagnosis of congenital heart disease in
cyanotic neonates. Curr Opin Pediatr. 2019;31(2):274-283.
doi:10.1097/MOP.0000000000000742.
• Puri K, Allen HD, Qureshi AM. Congenital heart disease. Pediatr Rev. 2017;38(10):471-
486. doi:10.1542/pir.2017-0032.
• Silberbach M, Hannon D. Presentation of congenital heart disease in the neonate and
young infant. Pediatr Rev. 2007;28(4):123-31. doi:10.1542/pir.28-4-123.
• Ushay HM. Cyanosis. In: McInerny TK, Adam HM, Campbell DE, DeWitt TG, Foy JM,
Kamat DM, eds. American Academy of Pediatrics Textbook of Pediatric Care. American
Academy of Pediatrics; 2021:chap 136. Accessed September 1, 2021. Pediatric Care
Online .

American Academy of Pediatrics 124


PREP® Self-Assessment PREPSA 2023

Question 37
A clinic team has been working to improve previsit planning for children with complex medical
conditions. Using plando-study-act (PDSA) cycles, the team has sequentially implemented
several interventions. To assess the impact of each intervention, the team has been following 4-
week moving averages of the percentage of visits for which previsit planning was completed
(Item Q37). The arrow at week 63 (Item Q37) shows one time point at which the team
implemented a new intervention with the goal of increasing the percentage of completed previsit
planning. The team analyzes data points over a defined time frame after each intervention to
determine the next steps.

Of the following, the BEST description of how the data are displayed is as a
A. control chart
B. failure mode and effects analysis
C. key driver diagram
D. run chart

American Academy of Pediatrics 125


PREP® Self-Assessment PREPSA 2023

Correct Answer: D
The clinic team in this vignette is using quality improvement science to improve previsit
planning for children with complex medical conditions. The data are displayed using a run chart,
which involves plotting individual points over time and monitoring for change from the median.
Run charts are one method used to determine if changes seen are caused by random (common
cause) variation within expected distribution or if the results are from a special
(uncommon) cause variation caused by external factors. In this vignette, the intervention tested at
week 63 resulted in a special cause variation and suggests that the team has achieved sustained
improvement.

Rules defining special cause variation in run charts include:


1) Six consecutive points, all on one side of the median (a shift) or
2) Five or more consecutive points, all increasing or decreasing (a trend)

Control charts (Item C37) also plot data over time, with the addition of upper and lower control
limits (3 standard deviations [SDs] from the mean). Rules defining special cause variation in
control charts include:
1. Any point beyond 3 SDs from the mean
2. Two points more than 2 SDs from the mean
3. Six or more consecutive points increasing or decreasing (a trend)
4. Eight or more consecutive points above or below the mean

Creation of an aim or goal, determining outcomes to measure, and testing the process are key
components of quality improvement science. Setting goals by choosing Specific, Measurable,
Actionable, Relevant, Timely (SMART) aims is an important first step. Using Plan-Do-Study-
Act (PDSA) cycles, initially with small changes, and measuring outcomes over time with run or
control charts will help to inform teams about whether to adopt, adapt, or abandon the
intervention.

American Academy of Pediatrics 126


PREP® Self-Assessment PREPSA 2023

Failure modes and effects analysis is a quality improvement science tool used to better
understand causes and effects of system failures. Key driver diagrams visually display
contributors (or drivers) critical to the achievement of an aim.

PREP Pearls
• Quality improvement projects should include a Specific, Measurable, Actionable,
Relevant, and Timely (SMART) aim.
• Plan-Do-Study-Act (PDSA) cycles are an efficient and effective way to evaluate the
impact of quality improvement efforts.
• Run charts and control charts can be used to monitor quality improvement data over time
and determine if special cause variation is present.

ABP Content Specifications(s)


• Recognize that analysis of variation in data is critical in quality improvement to
understand whether the variation is actually improvement
• Recognize that quality improvement requires looking at data or processes (ie, trends)
over time

Suggested Readings
• Bartman T, McClead RE. Core principles of quality improvement and safety. Pediatr
Rev. 2016;37(1):407-417. doi:10.1542/pir.2015-0091.
• Wolfe HA, Taylor A, Subramanyam R. Statistics in quality improvement: measurement
and statistical process control. Pediatr Anesth. 2021;31(5):539-547.
doi:10.1111/pan.14163.

American Academy of Pediatrics 127


PREP® Self-Assessment PREPSA 2023

Question 38
A 14-year-old adolescent boy is seen in the emergency department with acute flaccid paralysis of
the lower extremities, ascending numbness to the mid-trunk, urinary retention, and constipation.
His vital signs are normal for age. On physical examination, his mental status and cranial nerve
function are normal. He has normal upper extremity strength and reflexes. Strength in both lower
extremities is 0/5, reflexes are brisk at the patella and ankle, and toes are upgoing to plantar
stimulation. There is a sensory level at the mid-thorax below which he has diminished sensation
to light touch and pinprick. The adolescent has normal coordination in his upper extremities. His
lower extremity coordination and gait cannot be assessed due to his weakness. The remainder of
his physical examination findings are unremarkable.

Magnetic resonance imaging (MRI) of the spinal cord shows an extensive longitudinal T2
hyperintensity extending from T3 to the conus with no enhancement. Brain MRI is normal.
Lumbar puncture is performed, and cerebrospinal fluid analysis shows an elevated white blood
cell count of 100 cells/µL with a lymphocytic predominance. Antibody tests are ordered.
The adolescent is admitted to the intensive care unit and treated with intravenous
methylprednisolone daily for 5 days with minimal improvement. He is subsequently treated with
plasma exchange. Rehabilitation services are engaged early in his disease course.

Of the following, the BEST additional supportive measure(s) in the acute management of this
adolescent is/are
A. anticoagulation with therapeutic intravenous heparin infusion
B. bladder ultrasonography with intermittent catheterization
C. permissive hypertension with intravenous phenylephrine
D. prophylactic neuropathic pain management with oral gabapentin

American Academy of Pediatrics 128


PREP® Self-Assessment PREPSA 2023

Correct Answer: B
The adolescent in the vignette has acute transverse myelitis, a disorder of the spinal cord
characterized by acute onset of motor, sensory, and sphincter deficits with a range of underlying
etiologies. Urinary retention is a common complication, which increases the risk of urinary tract
infection. Therefore, of the response choices, the best additional supportive measures in this
adolescent’s acute management are bladder ultrasonography and intermittent catheterization.
Children and adolescents with transverse myelitis may have radicular or back pain preceding the
acute onset of weakness, sensory disturbances (numbness, paresthesias) and sphincter
dysfunction. Diagnostic criteria include:
• Bilateral, symmetric, or asymmetric, spinal cord dysfunction (motor, sensory, autonomic)
• Symptoms that evolve over 4 hours to 21 days
• Trunk level below which there is sensory dysfunction
• Radiographic or laboratory evidence of inflammation (gadolinium enhancement on
magnetic resonance imaging
• [MRI], cerebrospinal fluid analysis with pleocytosis or elevated IgG index) Exclusion of
alternative etiologies

Prompt identification of an acute myelopathy and exclusion of alternate causes requiring


emergency treatment (eg, spinal cord compression) is critical for appropriate management.
Autoimmune and inflammatory etiologies are most common; neoplastic, vascular (hematoma or
ischemic), or infectious causes may also occur. Magnetic resonance imaging of the spine can
both exclude alternate etiologies and provide radiographic evidence of inflammation.
Radiographic findings in transverse myelitis are variable with the most common being a
centrally located hyperintensity spanning multiple contiguous vertebral levels (Item C38). It is
important to note that up to 50% of patients with transverse myelitis will have normal findings
on spine MRI. Brain MRI aids in the diagnostic evaluation, as some autoimmune inflammatory
disorders causing myelitis can have brain involvement.

In the absence of an alternate diagnosis, expert consensus recommends urgent administration of


high-dose intravenous steroids (methylprednisolone). The 5-day course of treatment should not
be delayed for antibody tests. If there is minimal to no improvement in response to the steroid
course and severe neurologic deficits are present, plasma exchange can be considered.

Supportive management aimed at preventing complications is critical. Bowel and bladder


dysfunction are common in myelopathies, in particular those which involve the conus. Urinary
retention increases the risk for urinary tract infections and more serious conditions such as
pyelonephritis. Frequent routine bladder ultrasonography and intermittent catheterizations are
important components of management to prevent retention and avoid urinary complications. The
incidence of infection is lower with intermittent catheterization compared to indwelling
catheters. Constipation is common and should be monitored and managed with bowel care,
which can include increased fiber and water intake, physical activity, biofeedback, and oral
osmotic laxatives. Bowel and bladder dysfunction can have a significant impact on long-term
quality of life, requiring surveillance and management beyond the acute period.

American Academy of Pediatrics 129


PREP® Self-Assessment PREPSA 2023

Item C38: Sagittal T2-weighted magnetic resonance image showing a long segment of abnormal
hyperintensity and expansion of the visualized spinal cord, characteristic of transverse myelitis

Additional complications of acute myelopathy include deep vein thrombosis, autonomic


instability, orthostatic hypotension, neuropathic pain, and decubitus ulcers. There is an increased
risk of deep vein thrombosis due to immobility related to the neurological deficit. Deep vein
thrombosis prophylaxis can include prophylactic heparin or lovenox and sequential compression
devices; heparin infusion for therapeutic anticoagulation is not indicated in an inflammatory
disorder. Autonomic instability due to disruption of the sympathetic nerve fibers may occur;
common clinical manifestations include orthostatic hypotension and central neuropathic pain.
Orthostatic hypotension can be treated with compression stockings, increased fluid and salt
intake, fludrocortisone, or midodrine. Permissive hypertension is not used to treat this
complication. Neuropathic pain is not treated prophylactically. If neuropathic pain develops,
medications such as gabapentin can be used in addition to nonpharmacological approaches such
as physical therapy and stretching. Immobility and sensory deficits confer a risk for decubitus
American Academy of Pediatrics 130
PREP® Self-Assessment PREPSA 2023

ulcers, so meticulous skin care and careful monitoring for breakdown are critical components of
care.

PREP Pearls
• Acute transverse myelitis is a disorder of the spinal cord characterized by acute onset of
motor, sensory, and sphincter deficits with a range of underlying etiologies, most
commonly inflammatory or autoimmune. Radicular or back pain may precede the acute
onset of neurologic symptoms.
• Urgent evaluation of suspected transverse myelitis should include spine magnetic
resonance imaging (MRI) to exclude cord compression. Most commonly, imaging shows
centrally located hyperintensity spanning multiple contiguous vertebral levels; up to 50%
have normal findings on MRI.
• In addition to disease-specific treatment of transverse myelitis with high-dose intravenous
corticosteroids or plasmaexchange, supportive management includes bowel and bladder
care, deep vein thrombosis prophylaxis, decubitus ulcer prevention, and treatment of
autonomic instability.

ABP Content Specifications(s)


• Recognize the significance of bladder and bowel dysfunction in spinal cord disease

Suggested Readings
• Absoud M, Greenburg B, Lim M, Lotze T, Thomas T, Deiva K. Pediatric transverse
myelitis. Neurology. 2016;87(9 Supplement 2):S46-S52.
doi:10.1212/WNL.0000000000002820.
• Bigi S, Banwell B, Yeh EA. Outcomes after early administration of plasma exchange in
pediatric central nervous system inflammatory demyelination. J Child Neurol.
2015;30(7):874-880. doi:10.1177/0883073814545883.
• Lu Victor, Niazi T. Pediatric spinal cord diseases. Pediatr Rev. 2021;42(9):486-499.
doi:10.1542/pir.2020-000661.

American Academy of Pediatrics 131


PREP® Self-Assessment PREPSA 2023

Question 39
An 11-year-old girl is being evaluated for an injury to her left ankle that she sustained while
playing basketball. On the day before her visit to the clinic, she inverted her ankle when she
landed awkwardly from a jump. On physical examination, she has moderate swelling over the
lateral aspect of her left ankle. There is tenderness to palpation over the anterior aspect of the
distal lateral malleolus and just anterior and inferior to this spot. She has mild laxity with anterior
drawer testing. She has an antalgic gait and reports mild pain when walking.

Of the following, the BEST next step in this girl’s management is


A. cast immobilization
B. gradual return to basketball as tolerated
C. limiting weight-bearing with the use of crutches
D. use of an air stirrup splint

American Academy of Pediatrics 132


PREP® Self-Assessment PREPSA 2023

Correct Answer: D
The most likely diagnosis for the girl in the vignette is a sprain of the anterior talofibular
ligament (ATFL). Item C39A illustrates the position of the ATFL, calcaneofibular ligament
(CFL) and posterior talofibular ligament (PTFL) on the lateral aspect of the ankle. She had an
ankle inversion injury and has tenderness over the anterior aspect of the distal lateral malleolus,
where the ligament inserts, and over the ligament itself.

Following an ankle sprain, recovery is faster with early mobilization of the ankle. A stirrup style
ankle brace (Item C39B) permits dorsiflexion and plantarflexion at the ankle, which allows for a
more normal gait, while limiting eversion and inversion. For individuals who cannot tolerate
ambulation due to pain, full immobilization with a cast or walker boot may be required in the
early period after injury. Use of crutches with limited weight-bearing is rarely indicated.
Physicians should encourage early mobilization with the use of a less restrictive brace when
injured athletes can tolerate it.

American Academy of Pediatrics 133


PREP® Self-Assessment PREPSA 2023

Based on the Ottawa ankle rules (Stiell), ankle radiographs are not indicated for the girl in the
vignette; she does not have tenderness over the posterior aspect of the lateral or medial
malleolus, and she is able to ambulate. The Ottawa ankle rules were developed for adults but
have been shown to be highly sensitive in detecting ankle fractures in children as well. These
rules recommend radiographs for those with pain near the malleoli who are either unable to bear
weight immediately and at the time of examination or have tenderness on palpation of the bone
at the posterior or distal aspect of either malleolus. For children with tenderness over the
navicular or base of the fifth metatarsal bones, clinicians should obtain foot radiographs.

In addition to the use of a stirrup brace, the initial phase of management for an ankle sprain
includes applying ice to decrease inflammation and range-of-motion exercises. Strength
exercises are added next. As ankle strength improves, children can add exercises that promote
proprioception, such as single-leg balance skills, and begin progression back to sport-specific
activities.

PREP Pearls
• Early mobilization after an ankle sprain leads to a faster recovery.
• The Ottawa ankle rules for obtaining radiographs after an ankle injury have been
validated in children.

ABP Content Specifications(s)


• Plan the appropriate management of an uncomplicated sports-related ankle injury

Suggested Readings
• Coleman N. Sports Injuries. Pediatr Rev. 2019;40(6):278-290. doi:10.1542/pir.2018-
0221.
• Dowling S, Spooner CH, Liang Y, et al. Accuracy of Ottawa ankle rules to exclude
fractures of the ankle and midfoot in children: a meta-analysis. Acad Emerg Med.
2009;16(4):277-287. doi:10.1111/j.1553-2712.2008.00333.x.
• Kannikeswaran N, Suresh S. Sports musculoskeletal injuries. In: McInerny TK, Adam
HM, Campbell DE, DeWitt TG, Foy JM, Kamat DM, eds. American Academy of
Pediatrics Textbook of Pediatric Care. American Academy of Pediatrics; 2021:chap 334.
Accessed September 1, 2022. Pediatric Care Online.
• Stiell IG, Greenberg GH, McKnight RD, et al. Decision rules for the use of radiography
in acute ankle injuries. Refinement and prospective validation. Jama. 1993;269(9):1130.
• Strains, sprains, and dislocations. In: Sarwark J, Labella C, eds. Orthopaedics and Sports
Medicine: A Quick Reference Guide. 3rd ed. American Academy of Pediatrics;
2021:283-299.

American Academy of Pediatrics 134


PREP® Self-Assessment PREPSA 2023

Question 40
A male infant is seen for a routine health supervision visit. His parents have no concerns. He sits
unassisted on the examination table. He becomes excited when his mother hands him a favorite
toy, reaches up to shake it, and places it in his mouth. While the examiner is talking to his
mother, the infant babbles and smiles when his mother looks at him.

Of the following, this infant’s age is MOST likely


A. 3 months
B. 4 months
C. 6 months
D. 9 months

American Academy of Pediatrics 135


PREP® Self-Assessment PREPSA 2023

Correct Answer: C
The infant in the vignette is exhibiting developmental milestones most consistent with a 6-
month-old. He responds to his mother, babbles, sits without support, and brings an object to his
mouth.

Assessment and discussion of infant developmental milestones during each routine health
supervision visit are crucial, allowing for early identification of developmental delays and
counseling of the parents on ways to support developmental progress and what skills to expect
by the next visit. Identifying delays early allows for initiation of early intervention services and
evaluation for underlying conditions.

There are 4 domains of infant development: social, language, gross motor, and fine motor.
Mastered social developmental skills of a 6-month-old should include recognizing their parents
and responding to others’ emotions. Infants this age are often described as happy or joyful. They
enjoy looking at themselves in a mirror and begin to understand cause and effect (eg, dropping a
toy to have it picked up, pushing a button to make a noise). It is concerning if, by 6 months of
age, an infant is not showing affection to caregivers, reaching out to them, or becoming excited
when they are talked to.

The speech development of a 6-month-old should include babbling with vowel sounds (eg, ah,
eh, and oh) and some consonant sounds (eg, m, b, d). It is concerning if infants do not make
squealing sounds or laugh by 6 months of age. If they do not respond to noises, especially loud
noises in the environment, their hearing should be evaluated.

Fine motor skills at 6 months of age include bringing objects to their mouth; parents should be
counseled about choking hazards. Infants are able to pass objects between their hands (transfer
objects), continue to use a raking grasp, and progress toward a fine pincer grasp by 9 months of
age. It is concerning if, at 6 months of age, they are not reaching for objects or they are reaching
with only 1 hand; infants should not demonstrate a hand preference.

Gross motor skills at 6 months of age include rolling from front to back and back to front. Infants
at this age should be able to sit with little or no support of their own hands and support their
weight on their legs when being held upright. If the infant seems weak or stiff, they should be
evaluated for neurologic abnormalities.

PREP Pearls
• By 6 months of age, infants should be able to sit independently or with their own hands
for support.
• By 6 months of age, infants should babble with vowel sounds and begin to add consonant
sounds.
• By 6 months of age, infants should be able to cross midline to reach for objects.

American Academy of Pediatrics 136


PREP® Self-Assessment PREPSA 2023

ABP Content Specifications(s)


• Evaluate the motor developmental progress/status of an infant at 6 months of age,
including recognition of abnormalities
• Evaluate the cognitive and behavioral developmental progress/status of an infant at 6
months of age

Suggested Readings
• American Academy of Pediatrics. Cognitive development: 4 to 7 months. Accessed
September 1, 2022. https://www.healthychildren.org/English/ages-
stages/baby/Pages/Cognitive-Development-4-to-7-Months.aspx.
• American Academy of Pediatrics. Cognitive development: 7 months. Accessed
September 1, 2022. https://www.healthychildren.org/English/ages-
stages/baby/Pages/Developmental-Milestones-7-Months.aspx.
• Centers for Disease Control and Prevention. Important milestones: your baby by six
months. Accessed September 1, 2022.
https://www.cdc.gov/ncbddd/actearly/milestones/milestones-6mo.html.
• Feldman HM. How young children learn language and speech. Pediatr Rev.
2019;40(8):398-411. doi:10.1542/pir.20170325.
• Scharf RJ, Scharf GJ, Stroustrup A. Developmental milestones. Pediatr Rev.
2016;37(1):25-37; quiz 38, 47. doi:10.1542/pir.2014-0103.

American Academy of Pediatrics 137


PREP® Self-Assessment PREPSA 2023

Question 41
A 6-year-old boy is brought to the emergency department with tea-colored urine for 1 day. He
has had no flank pain, burning, or passage of blood clots with urination. He had a sore throat 4
weeks ago. On physical examination, the boy’s heart rate is 110 beats/min, respiratory rate is 18
breaths/min, and blood pressure is 136/86 mm Hg. He has mild puffiness over his eyes; the rest
of the physical examination findings are unremarkable.

Results of a urinalysis with microscopy are shown:


Urine Result
Specific gravity 1.030
Leukocyte esterase Negative
Nitrite Negative
Blood 3+
Protein 3+
Red blood cells > 100/HPF
White blood cells < 5/HPF

Of the following, the MOST likely diagnosis for this boy is


A. cystitis
B. glomerulonephritis
C. nephrolithiasis
D. rhabdomyolysis

American Academy of Pediatrics 138


PREP® Self-Assessment PREPSA 2023

Correct Answer: B
The child in the vignette had a sore throat 4 weeks before presentation with macroscopic
hematuria (tea-colored urine without blood clots), hypertension, and edema. Urinalysis with
microscopy shows hematuria and proteinuria favoring a diagnosis of acute poststreptococcal
glomerulonephritis (PSGN).

Macroscopic or gross hematuria is the presence of blood in the urine that is visible to the naked
eye along with the presence of red blood cells on urine microscopy. Macroscopic hematuria
should be differentiated from other causes of discolored urine. Urine that is dipstick test–positive
for blood but is without red blood cells on microscopy.

indicates myoglobinuria (eg, rhabdomyolysis) or hemoglobinuria (eg, intravascular hemolysis).


Discolored urine with a urine dipstick test that is negative for blood could be the result of
medications (eg, sulfonamides, pyridium, rifampin), other pigments (eg, beet, food coloring), or
urate crystals.

The causes of macroscopic hematuria are divided into 3 categories: glomerular disease,
interstitial or tubular disease, and urinary tract or vascular disease (Item C41). Bright red, red, or
pink urine with blood clots is suggestive of hematuria originating from the urinary collecting
system. Passage of blood in urine, with or without clots, at the beginning or at the end of
urination is suggestive of lower urinary tract etiology (eg, urethritis, urethral trauma, or bladder
stone). The presence of urinary symptoms like urgency, frequency, dysuria, and flank pain is
suggestive of urinary tract infection and nephrolithiasis. Brown, tea, or cola-colored urine
without blood clots favors a glomerular cause. A decrease in urine output, swelling of
extremities, or headaches related to high blood pressure also suggest a glomerular cause. A
recent history of sore throat or skin infection (eg, poststreptococcal glomerulonephritis),
recurrent episodes of gross hematuria (eg, IgA nephritis), trauma, sickle cell disease, or systemic
symptoms (eg, rheumatologic disease) help in identifying the cause of macroscopic hematuria.

American Academy of Pediatrics 139


PREP® Self-Assessment PREPSA 2023

Physical examination in a child with macroscopic hematuria should include measurement of


blood pressure and examination for edema, costovertebral angle tenderness, evidence of trauma
to the lower urinary tract, skin rash, and joint swelling. Hematuria with hypertension and edema,
as seen in the boy in the vignette, is suggestive of glomerulonephritis.

Laboratory evaluation of macroscopic hematuria should include urinalysis with microscopic


examination of a freshly voided, centrifuge-spun urine specimen. The presence of red blood cell
casts in the urine specimen is diagnostic of acute glomerulonephritis. Further evaluation depends
on the suspected etiology of macroscopic hematuria based on history and physical examination
findings. Serum electrolyte, blood urea nitrogen, serum creatinine, and serum albumin levels
should be obtained in a child with acute glomerulonephritis. Serum complement 3 (C3) level and
complement 4 (C4) level, antinuclear antibody, antistreptolysin O (ASO), and anti–double-
stranded DNA tests should be ordered to further classify the type of glomerulonephritis. A low
C3 level, normal C4 level, and elevated ASO titer are characteristic of PSGN. Proteinuria can be
confirmed on a first-morning urine specimen by evaluating the urine protein–to–urine creatinine
ratio. A urine culture specimen should be obtained for any child with urinary symptoms
suspicious for pyelonephritis and/or cystitis.

Renal and bladder ultrasonography is used to confirm nephrolithiasis, cystic kidney disease,
obstructive uropathy, tumors, or renal parenchymal disease. Computed tomography may be
required to detect nephrolithiasis. A renal Doppler study may be required to evaluate for renal
vascular thrombosis as a cause of macroscopic hematuria.

Cystitis is an unlikely diagnosis for the child in the vignette, who has tea-colored urine without
dysuria, and urine microscopy without leukocyte esterase or white blood cells. The absence of
bright red urine with blood clots, flank pain, and renal colic make nephrolithiasis an unlikely
diagnosis. Rhabdomyolysis is also unlikely, due to the presence of red blood cells noted on urine
microscopy.

Management of macroscopic hematuria depends on the cause. Glomerulonephritis, including


PSGN, should be managed in consultation with a pediatric nephrologist. Management of
glomerulonephritis includes supportive care, treatment of edema and hypertension with a
diuretic, symptomatic management of hyperkalemia and acute kidney injury, and prevention of
complications of fluid overload and pulmonary edema. The other glomerulonephritides may
require administration of steroids. The management of nonglomerular causes of hematuria may
include antibiotics (cystitis), intravenous hydration (nephrolithiasis, sickle cell–related papillary
necrosis), and urologic consultation (nephrolithiasis, renal trauma, obstructive uropathy).

American Academy of Pediatrics 140


PREP® Self-Assessment PREPSA 2023

PREP Pearls
• Brown, tea, or cola-colored urine without blood clots favors a glomerular etiology of
macroscopic hematuria.
• Bright red, red, or pink urine with blood clots is suggestive of hematuria originating from
the urinary collecting system.
• A urine that is dipstick test–positive for blood without red blood cells on urine
microscopy indicates myoglobinuria (eg, rhabdomyolysis) or hemoglobinuria (eg,
intravascular hemolysis).

ABP Content Specifications(s)


• Formulate a differential diagnosis of gross hematuria
• Plan the appropriate clinical evaluation of gross hematuria
• Recognize the disorders associated with hematuria

Suggested Readings
• Reidy KJ, Rio MD. Hematuria. In: McInerny TK, Adam HM, Campbell DE, DeWitt TG,
Foy JM, Kamat DM, eds. American Academy of Pediatrics Textbook of Pediatric Care.
American Academy of Pediatrics; 2021:chap 160. Accessed September 1, 2022. Pediatric
Care Online.
• Stotter B. Hematuria in children. Accessed September 1, 2022.
https://aap2.silverchaircdn.com/aap2/content_public/cms/resources/15681/2020-jul-
pediatric-hematuria.pdf.
• Viteri B, Reid-Adam J. Hematuria and proteinuria in children. Pediatr Rev. 2018;39
(12):573-587. doi:10.1542/pir.20170300.

American Academy of Pediatrics 141


PREP® Self-Assessment PREPSA 2023

Question 42
A 3-day-old male infant is seen in the emergency department for decreased feeding, vomiting,
and lethargy. He has been taking 2 ounces of formula every 2 hours but vomited his last feed and
has been difficult to arouse. His vital signs include a blood pressure of 68/40 mm Hg,
temperature of 37.3°C, heart rate of 170 beats/min, and respiratory rate of 80 breaths/min. His
newborn metabolic screen results are pending.

Laboratory data are shown:


Laboratory Test Result
Glucose 38 mg/dL (2.1 mmol/L)
Ammonia 35 µg/dL (25 µmol/L)
Carbon dioxide 16 mEq/L (16 mmol/L)
Lactic acid 3 mmol/L
Urine ketones Positive
Urine reducing substances Positive
Serum total bilirubin 14 mg/dL (239.4 µg/dL)
Aspartate aminotransferase 90 U/L
Alanine aminotransferase 60 U/L
White blood cell count 8,000/µL (8.0 × 109/L)
C-reactive protein 0.2 mg/dL (2 mg/L)

Of the following, the neonate’s MOST likely diagnosis is


A. fatty acid oxidation disorder
B. galactosemia
C. organic acidemia
D. urea cycle defect

American Academy of Pediatrics 142


PREP® Self-Assessment PREPSA 2023

Correct Answer: B
The neonate in the vignette has galactosemia as evidenced by hypoglycemia, hyperbilirubinemia,
elevated liver enzymes, and the presence of reducing substances in the urine. The diagnosis of
classic galactosemia is made through newborn screening or after the onset of clinical signs and
symptoms consistent with the condition. In an untreated neonate, classic galactosemia presents
with a life-threatening illness that can involve the liver, brain, and kidney. Neonates are normal
at birth; symptoms develop after ingestion of breast milk or standard formula, which leads to
accumulation of galactose and its by-products (galactose-1-phosphate, galactitol, and
galactonate) in the body.

The clinical presentation of classic galactosemia includes weight loss, poor feeding, vomiting,
and lethargy. On physical examination, signs of liver failure are seen, including jaundice, liver
enlargement, excessive bleeding, and sometimes a full anterior fontanelle. Laboratory findings
include hyperbilirubinemia and elevated liver enzyme levels. Excess galactose in the urine leads
to a positive finding of urine-reducing substances with the glucose oxidase method. Renal
tubular disease can manifest with metabolic acidosis and the presence of glucose and galactose in
the urine. Affected neonates are prone to gram-negative sepsis, particularly due to Escherichia
coli, with resultant septic shock and death. Critical intervention includes initiation of glucose-
containing intravenous fluids and the immediate restriction of dietary galactose, requiring a
change to soy or other lactose-free formula.

Galactosemia is caused by a deficiency of the erythrocyte enzyme galactose-1-phosphate


uridylyltransferase (GALT), which is inherited in an autosomal recessive manner. The diagnosis
is confirmed with an elevated erythrocyte galactose-1-phosphate concentration and detection of 2
pathogenic variants in the GALT gene. Although classic galactosemia can be detected on
newborn screening, the clinical manifestations of the disorder may appear before the metabolic
screening result becomes available.

Management and long-term care of galactosemia includes:


• Restriction of dietary lactose and galactose-containing foods–all milk products must be
removed, including human milk and cow milk; standard infant formula is replaced with
soy-based formula. Dietary restriction must continue throughout life.
• Annual assessment for nutritional deficiencies, including plasma calcium, phosphorous,
and 25-hydroxyvitamin D levels.
• Periodic developmental and growth assessment.
• Ophthalmologic evaluation at ages 1 year, 5 years, and during adolescence, with affected
children having an increased risk for cataracts.
• Assessment of bone mineral density at age 6 years, from puberty through adolescence,
and then every 5 years.
• Affected girls have an increased risk for premature ovarian insufficiency, delayed
menarche, and primary or secondary amenorrhea. Measurement of plasma 17-beta-
estradiol and follicle-stimulating hormone levels is recommended if there is delayed
menarche or secondary amenorrhea.

American Academy of Pediatrics 143


PREP® Self-Assessment PREPSA 2023

Laboratory findings differentiating disorders of inborn errors of metabolism are outlined in Item
C42. Bilirubin levels are normal and reducing substances are absent in the urine in each of the
conditions noted in the other responses, distinguishing them from classic galactosemia.
Fatty acid oxidation (FAO) disorders present with hypoketotic hypoglycemia with high anion
gap acidosis. Common FAO disorders include very long-chain acyl-CoA dehydrogenase
deficiency, medium-chain acyl-CoA dehydrogenase deficiency, carnitine palmitoyltransferase I
and II deficiency, and carnitine transporter defects.

Organic acidemia presents in the neonatal period with hyperammonemia and high anion gap
acidosis in the presence of urine ketones. Common organic acid disorders include isovaleric
aciduria, methylmalonic aciduria, propionic aciduria, and maple syrup urine disease.
Urea cycle defects present in the neonatal period with hyperammonemia and respiratory
alkalosis. Common urea cycle defects include ornithine transcarbamylase deficiency,
citrullinemia, and argininosuccinic aciduria.

PREP Pearls
• The acute neonatal presentation of the various inborn errors of metabolism can be
distinguished based on laboratory data.
• The findings of hypoglycemia, hyperbilirubinemia, elevated liver enzymes, and the
presence of reducing substances in the urine is a hallmark of classic galactosemia.
• Neonates with galactosemia should be fed soy formula; lactose- and galactose-containing
foods must be avoided, including both human and cow milk.

American Academy of Pediatrics 144


PREP® Self-Assessment PREPSA 2023

ABP Content Specifications(s)


• Recognize the clinical features associated with galactosemia
• Plan the appropriate immediate and long-term management of galactosemia, while
considering the long-term prognosis
• Recognize the laboratory features associated with galactosemia

Suggested Readings
• AAP Committee on Genetics; Saul RA, ed. Treating genetic disorders. In: Medical
Genetics in Pediatric Practice. American Academy of Pediatrics; 2013;chap 16.
• Berry G. Classic galactosemia and clinical variant galactosemia. GeneReviews [Internet].
https://www.ncbi.nlm.nih.gov/books/NBK1518/.
• Cerone J, Rios A. Galactosemia. Pediatr Rev. 2019;40(Suppl 1):24-27.
doi:10.1542/pir.2018-0150.
• Rice GM, Steiner RD. Inborn errors of metabolism. Pediatr Rev. 2016;37(1):3-16.
doi:10.1542/pir.2014-0122.
• Welling L, Bernstein LE, Berry GT, et al; Galactosemia Network (GalNet). International
clinical guideline for the management of classical galactosemia: diagnosis, treatment, and
follow-up. J Inherit Metab Dis. 2017;40(2):171-176. doi:10.1007/s10545-016-9990-5.

American Academy of Pediatrics 145


PREP® Self-Assessment PREPSA 2023

Question 43
A 9-year-old girl, who is chronically debilitated, is brought to the office by her parents to discuss
end-of-life options. She has been requiring hospitalization more frequently for minor illnesses
and does not return to her prior baseline after each hospitalization. The girl was born at 23
weeks’ gestation after a placental abruption that resulted in severe hypoxic-ischemic
encephalopathy. During her 9-month neonatal intensive care unit course, she was found to have a
grade IV intraventricular hemorrhage with post-hemorrhagic hydrocephalus requiring a
ventriculoperitoneal shunt. Her past medical history is also significant for spastic quadriplegia
and chronic respiratory failure with tracheostomy and ventilator dependency.

The girl is awake, with spontaneous eye-opening, and is in no acute distress. Her vital signs are
at baseline. She has no spontaneous respiratory effort, is mechanically ventilated, does not
interact with her environment, and is nonverbal other than incoherent vocalizations. She has
severe spasticity, hypertonicity, and minimal response to painful stimulation except for increased
tone posturing. The girl’s parents believe their daughter is suffering and ask about end-of-life
options, specifically withdrawal of support or limitation of curative medical care in the event of
another acute illness requiring in-patient hospital care.

Of the following, the BEST next option to offer this girl’s family is referral to a(n)
A. inpatient admission for planned withdrawal of technological support
B. institutional ethics committee for review of ethics of withdrawal
C. local hospice for palliative care and establishment of advance directives
D. regional organ procurement organization

American Academy of Pediatrics 146


PREP® Self-Assessment PREPSA 2023

Correct Answer: C
The girl in the vignette has a severe and static hypoxic brain injury as a result of her traumatic
and premature birth. Although she appears to be in a persistent vegetative state, she does not
meet the clinical criteria for brain death; her decorticate posturing suggests intact brainstem
reflexes, and thus determination of brain death cannot be made. Given that the girl’s parents
perceive her to be suffering and wish to limit additional interventions to prolong her life, the best
option is to refer the girl and her family to hospice and palliative care and establish advance
directives including a do not attempt resuscitation order and limitation of care.

For chronically ill children, consultation with palliative care and hospice clinicians can be
advantageous for physicians and families both during an acute life-threatening illness and when
children are at their baseline state of health. It is often challenging for families to discuss end-of-
life issues or limitations of care during an acute hospitalization as their primary focus tends to be
on the goal of acute recovery. Long-term planning including advance directives, limitations of
care, and do not attempt resuscitation orders are best discussed when children are in a relatively
healthy (baseline) state.

Pediatric euthanasia is currently illegal in the United States. However, withdrawal of life-
sustaining technological support may take place under the direction of experts in end-of-life care,
specifically those with hospice or palliative care training. The active withdrawal of life-
sustaining technology can be done ethically when caregiver goals are clearly established and
made in the best interest of the child. This action is ethically comparable to not initiating the
same therapy in children who have underlying life-limiting illnesses.

Dying children frequently experience pain and/or dyspnea at the end of life. When this occurs,
there may be a reluctance to give medications that will relieve pain but also hasten death.
Morally, this challenge falls under the Doctrine of Double Effect which states that if an action is
morally justified, the unintended side effect does not negate the value of the intended action. The
Doctrine of Double Effect justifies medication administration in these cases such that symptoms
may be relieved, even if doing so will precipitate an otherwise inevitable and imminent death. In
some cases, medications may be administered to alleviate symptoms of pain, air hunger, and
dyspnea, even though these medications (eg, opiates) may cause respiratory depression and
hasten death. In order to be ethically justified, a number of criteria must exist simultaneously:
• The intended effect must be beneficial (eg, relief of pain or suffering) Only the beneficial
effect of the action is considered (eg, the goal is to relieve pain and not to hasten death)
• The negative effect is not the only means of achieving the good effect (eg, the desired
good effect does not justify unethical means (eg, giving high-dose potassium to
precipitate death as a means to end pain and suffering would not be ethical)
• The good effect must outweigh the bad effect (eg, if death is otherwise inevitable and
imminent, the negative effect can be justified)

In 1968, the Uniform Anatomical Gift Act provided the legal foundation upon which human
organs and tissues can be donated for transplantation. The Uniform Brain Death Act of 1978
aimed to clarify legal ambiguity surrounding the determination of death and established that the
American Academy of Pediatrics 147
PREP® Self-Assessment PREPSA 2023

“irreversible cessation of all functioning of the brain, including the brain stem” is death. In 1987,
guidelines for the determination of brain death in children were published cooperatively and in
2011 were revised and again promulgated by the Society of Critical Care Medicine, the
American Academy of Pediatrics, and the Child Neurology Society. The decision about organ
donation can be an emotionally challenging discussion for any physician and family.

The diagnosis of brain death requires confirmation of irreversible loss of all functions of the
brain, including the brainstem. Unlike coma or persistent vegetative state, brain death is
permanent, with no chance of recovery of neurologic function. In clinical practice, the requisite
findings in brain death are persistent coma of a known cause in the absence of metabolic
derangements or pharmacologic confounders, absence of brainstem reflexes, and apnea.
Reversible conditions that can interfere with the neurologic examination must be excluded prior
to brain death testing in order for the clinical determination of brain death to be valid. A child
determined to be brain dead is legally and clinically dead, and thus organs and tissues are eligible
for donation.

At present, euthanasia is not a legal option for children in the United States, and thus referral to
an ethics committee for consideration would be inappropriate. Additionally, as the girl is not
brain dead, referral to an organ procurement organization for organ donation would be
premature. Finally, although withdrawal of technological support could be facilitated by an
elective inpatient admission, engagement with hospice or palliative care clinicians would be
preferred prior to an elective admission.

PREP Pearls
• Consultation with palliative care and hospice experts can be a valuable resource for
families to help establish goals of care and advance directives.
• Brain death, unlike coma or persistent vegetative state, is irreversible; there is no
possibility that children with brain death will regain consciousness.
• Medically appropriate interventions may hasten death; if the intent is to provide
symptomatic relief, the intervention may be ethically justified under the Doctrine of
Double Effect.

ABP Content Specifications(s)


• Recognize and apply ethical principles when caring for a patient who is in a persistent
vegetative state

American Academy of Pediatrics 148


PREP® Self-Assessment PREPSA 2023

Suggested Readings
• McLaughlin BT, Filiano JJ. Pediatric brain death. Pediatr Rev. 2021;42(3):161-163.
doi:10.1542/pir.2020-0133.
• Nakagawa TA, Ashwal SA, Mathur M, et al. Clinical report—Guidelines for the
determination of brain death in infants and children: an update of the 1987 task force
recommendations. Pediatrics. 2011;128(3):e720-e740. doi:10.1542/peds.2011-1511.
• Okun AL. Palliative, end-of-life, and bereavement care. In: McInerny TK, Adam HM,
Campbell DE, DeWitt TG, Foy JM, Kamat DM, eds. American Academy of Pediatrics
Textbook of Pediatric Care. American Academy of Pediatrics; 2021:chap 67. Accessed
September 1, 2022. Pediatric Care Online.
• Opel DJ, Olson ME. Ethics for the pediatrician. Pediatr Rev. 2012;33(8):370-373.
doi:10.1542/pir.33-8-370.

American Academy of Pediatrics 149


PREP® Self-Assessment PREPSA 2023

Question 44
A 5-week-old male infant is seen in the clinic for a routine follow-up visit. He has had worsening
emesis over the past 2 weeks that occurs after all feedings and is yellow in color. He has had no
fever or respiratory symptoms, but he is progressively becoming more fussy. The infant has lost
110 g since his last visit 2 weeks ago. He is too fussy to obtain an accurate abdominal
examination.

Of the following, this infant’s diagnostic evaluation would MOST likely show
A. abnormal upper gastrointestinal series with small bowel follow through
B. elevated total body potassium
C. esophageal erosion
D. hypochloremic metabolic alkalosis

American Academy of Pediatrics 150


PREP® Self-Assessment PREPSA 2023

Correct Answers: D
The infant in the vignette has a clinical presentation that is consistent with a diagnosis of
hypertrophic pyloric stenosis (HPS). On abdominal examination of an infant with pyloric
stenosis, an “olive-sized” epigastric mass is palpable in 60% to 80% of cases, and peristaltic
waves may be visible.

Affected infants develop hypochloremic metabolic alkalosis resulting from:


• Vomiting that leads to a loss of hydrochloric acid and potassium chloride
• Pyloric obstruction that prevents pancreatic stimulation to produce bicarbonate (HCO3),
which leads to a worsening metabolic alkalosis
• Dehydration that results in an increased concentration of HCO3 and a “contraction”
alkalosis

Infants with HPS have a low total body potassium, though plasma levels might be normal
because of a shift to the extracellular space.

Abdominal ultrasonography is the first step recommended to confirm the diagnosis of


hypertrophic pyloric stenosis. A barium swallow would be indicated if ultrasonography were
nondiagnostic. If this infant had bilious emesis, an upper gastrointestinal series with small bowel
follow through would be indicated to evaluate for malrotation with volvulus or other cause of
distal obstruction.

Severe gastrointestinal reflux can cause pain and esophageal erosion, which can be diagnosed on
endoscopic evaluation. Although severe gastroesophageal reflux with esophagitis can cause
weight loss, the infant in the vignette should first be evaluated for pyloric stenosis. Endoscopy
would not be indicated at this time.

PREP Pearls
• Abdominal examination of an infant with hypertrophic pyloric stenosis may demonstrate
a palpable epigastric “olive”
• or visible peristaltic waves.
• Hypertrophic pyloric stenosis causes a hypochloremic metabolic alkalosis.
• Abdominal ultrasonography is the imaging study of choice to diagnose hypertrophic
pyloric stenosis.

ABP Content Specifications(s)


• Recognize the acid-base changes associated with pyloric stenosis, and manage
appropriately
• Recognize the clinical features associated with pyloric stenosis, and manage
appropriately

American Academy of Pediatrics 151


PREP® Self-Assessment PREPSA 2023

Suggested Readings
• Reddy S, Kamat DM. Pyloric stenosis. In: McInerny TK, Adam HM, Campbell DE,
DeWitt TG, Foy JM, Kamat DM, eds. American Academy of Pediatrics Textbook of
Pediatric Care. American Academy of Pediatrics; 2021:chap 321. Accessed September 1,
2022. Pediatric Care Online.
• Rich BS, Dolgin SE. Hypertrophic pyloric stenosis. Pediatr Rev. 2021;42(10):539-545.
doi:10.1542/pir.2020-003277.

American Academy of Pediatrics 152


PREP® Self-Assessment PREPSA 2023

Question 45
A 2-week-old male infant, born at 35 weeks’ gestation, is seen at the office for a weight check.
He is breastfeeding well with normal urination and stooling. He has gained 5 ounces since birth.
On physical examination, the neonate is comfortable in room air, with a respiratory rate of 40
breaths/min and a temperature of 37.5°C. There is bilateral watery eye discharge, palpebral and
bulbar injection, and mild eyelid swelling. The neonate received eye prophylaxis at birth. The
conjunctiva and nasopharynx are swabbed for bacterial culture and a polymerase chain reaction
test for gonorrhea and chlamydia.

Of the following, the BEST next step in management of this neonate is


A. azithromycin eye ointment
B. azithromycin oral suspension
C. ceftriaxone, intramuscular
D. external digital massage of the tear ducts

American Academy of Pediatrics 153


PREP® Self-Assessment PREPSA 2023

Correct Answer: B
The neonate in the vignette has findings consistent with chlamydial conjunctivitis. Neonatal
chlamydial infection is acquired during passage through the infected birth canal. Early in the
course, chlamydial conjunctivitis can be watery, and later it becomes mucopurulent. There may
be swelling of the eyelids and chemosis (swelling and redness of conjunctiva). Eye prophylaxis
with erythromycin ointment given at birth is not effective in preventing chlamydial infection.
Oral azithromycin or oral erythromycin is the treatment of choice for both chlamydial
conjunctivitis and pneumonia in neonates.

Topical ophthalmic therapy is not indicated for neonatal chlamydial conjunctivitis, due to high
rates of treatment failure as well the need to concurrently eradicate nasopharyngeal carriage in
order to prevent subsequent pneumonia.

A single dose of intramuscular ceftriaxone, 50 mg/kg, is recommended for treatment of


gonococcal ophthalmia, but is not effective for treatment of a chlamydial infection. Although
increased tearing and matting of the eyelashes is commonly seen with nasolacrimal duct
obstruction, conjunctival erythema, edema, and swelling are not common. The presence of eyelid
swelling and chemosis should prompt further investigation and consideration of treatment for
neonatal conjunctivitis.

Chlamydia trachomatis is an obligate intracellular organism that infects squamocolumnar or


transitional epithelial cells, such as the mucous membranes of the conjunctivae, nasopharynx,
cervix, urethra, and rectum. It is the most common notifiable sexually transmitted infection in the
United States, with the highest incidence in adolescents and adults, aged 15 to 24 years. The type
of chlamydial infection (eg, genital, ocular, pneumonia) is dependent on the serotype and
susceptibility of the host.

• Serotype A, C: trachoma, chronic follicular keratoconjunctivitis


• Serotype B, D: genital and ocular infections
• Serotype L1, L2 and L3: lymphogranuloma venereum

The diagnostic test of choice is a Chlamydia nucleic acid amplification test (NAAT). In cases of
chlamydial conjunctivitis, swabs of the conjunctiva rather than the discharge should be sent. In
neonates with suspected chlamydial pneumonia, a posterior pharyngeal swab should be sent for a
NAAT. Samples for NAAT in postpubertal girls should be either a cervical swab or a first catch
(initial stream) urine sample; in postpubertal boys, a first catch urine sample is appropriate.

Treatment of adolescents with genital chlamydial infection is with either


• 1 g of azithromycin, orally as a single dose OR
• 100 mg of doxycycline, orally, twice a day for 7 days OR
• 500 mg of levofloxacin, orally, for 7 days

American Academy of Pediatrics 154


PREP® Self-Assessment PREPSA 2023

Test of cure immediately after treatment is not recommended. However, because reinfection is
common, retesting is recommended 3 months after treatment or at the patient’s next medical care
visit within 12 months.

PREP Pearls
• Chlamydia, an obligate intracellular bacterium, causes the most common sexually
transmitted notifiable infection in the United States.
• Neonates exposed to Chlamydia through an infected birth canal develop conjunctivitis
typically 5 to 14 days after birth; chlamydial pneumonia occurs between 3 and 12 weeks
of age.
• The recommended treatment for any form of neonatal chlamydial infection is oral
azithromycin or erythromycin.

ABP Content Specifications(s)


• Understand the epidemiology of Chlamydia trachomatis
• Plan appropriate diagnostic evaluation for chlamydial infection depending on the site of
infection (eg, genital, respiratory)
• Recognize the clinical features associated with chlamydial infection in patients of various
ages Plan the management of chlamydial infection in patients of various ages

Suggested Readings
• American Academy of Pediatrics. Chlamydia trachomatis. In: Kimberlin DW, Barnett
ED, Lynfield R, Sawyer MH, eds. Red Book: 2021–2024 Report of the Committee on
Infectious Diseases. 32nd ed. American Academy of Pediatrics; 2021. Accessed
September 1, 2022. Red Book Online .
• Darville T, Rours IJG. Chlamydia trachomatis. In: Long SS, Prober CG, Fischer M, eds.
Principles and Practice of Pediatric Infectious Diseases. 5th ed. Elsevier; 2018:908-
914.e2.
• Joffe A. Sexually transmitted infections. In: McInerny TK, Adam HM, Campbell DE,
DeWitt TG, Foy JM, Kamat DM, eds. American Academy of Pediatrics Textbook of
Pediatric Care. American Academy of Pediatrics; 2021:chap 239. Accessed September 1,
2022. Pediatric Care Online.
• Richards A, Guzman-Cottrill JA. Conjunctivitis. Pediatr Rev. 2010;31(5):196-208.
doi:10.1542/pir.31-5-196.
• Siqueira LM. Chlamydia infections in children and adolescents. Pediatr Rev.
2014;35(4):145-52: quiz 153-154. doi:10.1542/pir.35-4-145.

American Academy of Pediatrics 155


PREP® Self-Assessment PREPSA 2023

Question 46
A 6-week-old infant with no significant birth or past medical history is brought to the emergency
department by ambulance for decreased responsiveness. He appears lethargic, pale, and cyanotic
with poor respiratory effort. His capillary refill time is greater than 5 seconds. His vital signs
include a temperature of 36.5°C, heart rate of 198 beats/min, respiratory rate of 8 breaths/min,
blood pressure of 65/42 mm Hg, and oxygen saturation of 86% in room air.

Of the following, the BEST next step in this infant’s management is to


A. administer a 20 mL/kg normal saline fluid bolus
B. administer broad-spectrum antibiotics intravenously
C. begin bag-valve mask ventilation and prepare for intubation
D. obtain a point-of-care glucose level

American Academy of Pediatrics 156


PREP® Self-Assessment PREPSA 2023

Correct Answer: C
The infant in the vignette is displaying altered mental status, for which the initial management is
the ABCs (Airway, Breathing, Circulation) of resuscitation. Assessment and stabilization of the
airway is the first step, followed by stabilization of breathing and circulation. The infant in the
vignette is cyanotic with poor respiratory effort and a respiratory rate of 8 breaths/min; thus, bag-
valve mask ventilation and preparing for intubation is the best next management step. Although
obtaining a point-of-care glucose level and administration of intravenous fluids and broad-
spectrum antibiotics will be important steps in the infant’s care, addressing the airway and
breathing concerns must take priority.

The signs and symptoms of altered mental status in a child can range from subtle to overt and
vary depending upon the age of presentation. These may include:
• Abnormal motor movements
• Abnormal pupil size or reaction to light
• Abnormal respiratory pattern (eg, tachypnea, bradypnea, apnea, or Cheyne-Stokes
respirations)
• Abnormal tone
• Confusion
• Excessive sleepiness or poor responsiveness to the environment or caregiver
• Irritability or extreme fussiness
• Lethargy
• Minimal or no response to painful stimuli
• Persistent eye deviation or disconjugate gaze
• Weak or absent cry

The mnemonic AEIOU TIPS is helpful for remembering the broad differential diagnosis for
causes of altered mental status (Item C46).
Reye syndrome should be considered in the differential diagnosis for children with altered
mental status.

American Academy of Pediatrics 157


PREP® Self-Assessment PREPSA 2023

After the child with altered mental status has been stabilized, a thorough history and physical
examination should be performed. The history should include:
• Onset and time course (acute or gradual) of symptoms
• Associated symptoms
• History of trauma
• Travel history
• Assessment for potential ingestion of toxic substances
• Any predisposing past medical history

Initial evaluation of a child with altered mental status should include (a):
• Point-of-care glucose level
• Arterial or venous blood gas analysis
• Complete blood count
• Electrolyte panel
• Blood culture
• Urinalysis
• Urine culture
• Toxicology screen
• Cerebrospinal fluid studies (cell count, protein, glucose, culture)
• Ammonia level
• Serum amino acids, urine organic acids, and carnitine levels (if severe hypoglycemia is
present or an inborn error of metabolism is suspected)
• Head computed tomography if trauma or increased intracranial pressure is suspected
(highly recommended in ANY child with severe alteration of consciousness)
Electrocardiography

Initial treatment of a child with altered mental status includes:


• Assessment and immediate intervention for Airway, Breathing, and Circulation (ABCs)
abnormalities (intubation for any child that is severely obtunded or unable to protect their
airway)
• Isotonic fluid bolus, intravenously: 20 mL/kg (10 mL/kg if a cardiac etiology is
suspected)
• Dextrose administration if hypoglycemic
• Correction of temperature abnormalities
• Correction of electrolyte or acid-base imbalances
• Avoiding hypotension and hyperventilation
• Administration of broad-spectrum antibiotics
• Consideration for naloxone administration
• Initiation of other specific therapy for probable diagnoses

American Academy of Pediatrics 158


PREP® Self-Assessment PREPSA 2023

PREP Pearls
• The initial management of altered mental status is the ABCs (Airway, Breathing,
Circulation) of resuscitation.
• The signs and symptoms of altered mental status in a child can range from subtle to overt
and vary depending upon the age of presentation.
• The mnemonic AEIOU TIPS is helpful for remembering the broad differential diagnosis
for causes of altered mental status.

ABP Content Specifications(s)


• Identify the various etiologies of an altered level of consciousness
• Plan the appropriate initial evaluation of an altered level of consciousness

Suggested Readings
• Avner JR. Altered states of consciousness. Pediatr Rev. 2006;27(9):331-338.
doi:10.1542/pir.27.9.331.
• Forti RJ, Avner JR. Altered mental status. In: McInerny TK, Adam HM, Campbell DE,
DeWitt TG, Foy JM, Kamat DM, eds. American Academy of Pediatrics Textbook of
Pediatric Care. American Academy of Pediatrics; 2021:chap 349. Accessed September 1,
2022. Pediatric Care Online.

American Academy of Pediatrics 159


PREP® Self-Assessment PREPSA 2023

Question 47
A 3-day-old neonate is seen in the office after his newborn screening test showed a thyroxine
(T4) level of 1.2 µg/dL (15.5 nmol/L) (reference range, ≥6.5 µg/dL [≥83.7 nmol/L]. The reflex
thyroid-stimulating hormone (TSH) level is 2.5 mIU/L (reference range, ≤20 mIU/L). The
neonate was born at term via uncomplicated labor and delivery and had no problems in the
newborn nursery. He is exclusively breastfed, and his weight today is 3% below birth weight.
The neonate appears vigorous and has mild jaundice. He does not have a goiter or umbilical
hernia. The remainder of his physical examination findings are normal.

Of the following, the BEST next step in the management of this neonate is to
A. begin levothyroxine therapy
B. obtain thyroid-stimulating hormone and free thyroxine levels
C. order magnetic resonance imaging of the pituitary gland
D. repeat the newborn screening test when the neonate is 5 to 7 days old

American Academy of Pediatrics 160


PREP® Self-Assessment PREPSA 2023

Correct Answer: B
The neonate in the vignette has an abnormal newborn screening test with a low thyroxine (T4)
and normal thyroidstimulating hormone (TSH) level. Based on the neonate’s physical
examination findings, the next best step in management is to obtain confirmatory serum TSH and
free thyroxine (FT4) levels. The most likely diagnosis for this healthy male neonate is congenital
thyroxine-binding globulin (TBG) deficiency, a benign X-linked condition. Confirmatory testing
would demonstrate normal TSH and FT4 levels.

Newborn screening tests vary by state and primarily measure total T4 with a reflex TSH level or
vice versa. Some states automatically measure both TSH and T4 levels. The primary TSH screen
is designed to detect the high TSH levels in primary hypothyroidism. The primary T4 screen, in
addition to detecting low T4 levels due to primary hypothyroidism, detects low levels due to
central hypothyroidism (TSH deficiency) and TBG deficiency. Of note, abnormal newborn
screening test results should always be confirmed with appropriate diagnostic testing.
Thyroid hormones are bound to TBG, transthyretin, and albumin. When an individual is deficient
in TBG, the total pool of circulating T4 is decreased. However, the active free portion of T4
(FT4) remains normal in the circulation.

Thyroxine-binding globulin deficiency is a harmless condition; treatment should not be pursued.


Although the pattern of a low T4 and “normal” TSH level may be seen in central
hypothyroidism, this condition is usually accompanied by signs of other pituitary hormone
deficiencies such as hypoglycemia, microphallus, or midline defects. The neonate in the vignette
is healthy, making central hypothyroidism less likely; however, confirmatory serum testing is
required. For neonates with confirmatory serum testing consistent with central hypothyroidism
(low free T4 level and normal or low TSH level), evaluation for other pituitary hormone
deficiencies and magnetic resonance imaging of the pituitary gland is indicated.

Levothyroxine therapy should be started promptly in neonates with congenital hypothyroidism to


prevent adverse developmental outcomes. However, confirmatory serum thyroid function tests
should be ordered (although results may not necessarily need to be obtained) before treatment is
initiated. Additionally, without evidence of central hypothyroidism, starting thyroid hormone
replacement therapy for the neonate in the vignette would not be appropriate prior to performing
further evaluation.

Neonates who have newborn screening performed too soon after birth may have elevated TSH
levels, given the expected surge of thyroid hormones that occurs after delivery. In such a case,
repeating the newborn screening test may be recommended. However, this would not be true for
the neonate in the vignette, as his pattern of thyroid function results is not consistent with the
newborn screening test being drawn too soon after birth.

American Academy of Pediatrics 161


PREP® Self-Assessment PREPSA 2023

PREP Pearls
• Neonates with an abnormal thyroid newborn screening test result should be evaluated
promptly with a physical examination and confirmatory serum thyroid-stimulating
hormone and free thyroxine testing.
• A newborn screening test result showing a low thyroxine and normal thyroid-stimulating
hormone level is consistent with thyroxine-binding globulin (TBG) deficiency or, less
frequently, central hypothyroidism. The confirmatory serum free thyroxine level
distinguishes these conditions (normal in TBG deficiency and low in central
hypothyroidism).
• Thyroxine-binding globulin deficiency is a benign, X-linked condition that does not
require treatment.

ABP Content Specifications(s)


• Recognize the clinical and laboratory features associated with thyroid-binding globulin
deficiency

Suggested Readings
• American Academy of Pediatrics, Rose SR; Section on Endocrinology and Committee on
Genetics, American Thyroid Association, Brown RS; Public Health Committee and
Lawson Wilkins Pediatric Endocrine Society. Update of newborn screening and therapy
for congenital hypothyroidism. Pediatrics. 2006;117(6):2290-2303.
doi:10.1542/peds.2006-0915.
• Diaz A, Lipman Diaz EG. Hypothyroidism. Pediatr Rev. 2014;35(8):336-347.
doi:10.1542/pir.35-8-336.
• Léger J, Olivieri A, Donaldson M, et al; ESPE-PES-SLEP-JSPE-APEG-APPES-ISPAE;
Congenital Hypothyroidism Consensus Conference Group. European Society for
Paediatric Endocrinology consensus guidelines on screening, diagnosis, and management
of congenital hypothyroidism. J Clin Endocrinol Metab. 2014; 99(2):363-384.
doi:10.1210/jc.2013-1891.
• Orlowski CC. Hypothyroidism. In: McInerny TK, Adam HM, Campbell DE, DeWitt TG,
Foy JM, Kamat DM, eds. American Academy of Pediatrics Textbook of Pediatric Care.
American Academy of Pediatrics; 2021:chap 273. Accessed September 1, 2021. Pediatric
Care Online.
• Wassner AJ, Smith JR. Disorders of thyroxine-binding globulin. In: Kliegman RM, St.
Geme JW, Blum NJ, Shah SS, Tasker RC, Wilson, KM, eds. Nelson Textbook of
Pediatrics. 21st ed. Elsevier; 2020:2914.

American Academy of Pediatrics 162


PREP® Self-Assessment PREPSA 2023

Question 48
A 6-month-old infant is seen for a health supervision visit. Her family uses bicycling as their
primary mode of transportation within the community. The infant’s mother wants to know if
there are specific safety recommendations regarding riding with her.

Of the following, the MOST accurate anticipatory guidance to offer is that


A. a bicycle helmet will first be required when she rides a tricycle
B. a child should not be in a bicycle seat until 9 months of age
C. this child can most safely ride in a bicycle trailer
D. US federal law requires that children 16 years and younger wear a helmet while riding a
bicycle

American Academy of Pediatrics 163


PREP® Self-Assessment PREPSA 2023

Correct Answer: C
Studies have shown that there are fewer injuries to children when riding in bicycle-towed trailers
than in bicycle-mounted child seats, therefore, the infant in the vignette would be safest riding in
a bicycle trailer. Some bicycle seats are marketed for infants as young as 9 months of age, but the
American Academy of Pediatrics recommends not using a bicycle seat until 12 months of age. A
bicycle seat should sit over the rear wheel and have spoke guards to prevent injuries to
the feet and hands, as well as a shoulder harness and lap belt. All children, regardless of age,
should wear a bicycle helmet when riding a bicycle or tricycle, and in a bicycle seat or trailer.
About 100 children die from bicycle crash-related injuries each year in the United States. In
general, these injuries are more common in older children, males, and members of lower
socioeconomic communities. Those who survive are 3 times as likely to have been wearing a
helmet. Behaviors that increase the risk for injury include riding against traffic, riding at night,
and failing to obey traffic laws and conventions.

A 2012 survey of bicycle helmet use in the United States showed that only about 42% of
children always wore helmets. The reasons most often cited for not using a helmet are discomfort
and ugliness. Lack of helmet ownership is less often a barrier. Helmet use by children is
increased when parents also use a helmet. Many studies have correlated children’s bicycle
helmet use with parents' attitudes about helmet use and parent ownership of their own helmets.
Of children who owned helmets, those in families with strict rules about use were much more
likely to use the helmets than in households without strict parental enforcement. In one study,
only one-third of parents reported that helmet use was discussed by their pediatrician.

There are no federal bicycle helmet laws in the United States. Helmet laws vary by state and
county. States began adopting helmet laws in 1987; most are limited to children younger than 16
to 18 years. Thirteen states have no state or local helmet laws. Legislation on the use of bicycle
helmets for children has increased helmet use.

PREP Pearls
• It is safer to have a child in a bicycle-towed trailer than a bicycle-mounted seat.
• Parents’ use of helmets and strict enforcement of use increases children's use of bicycle
helmets.
• There are no federal laws regarding bicycle helmet use in the United States.

MOCA-Peds Objective
• Provide anticipatory guidance regarding safety and injury prevention for school-age
children (ages 5 to 10 years).
ABP Content Specifications(s)
• Counsel parents and children regarding bicycle safety

American Academy of Pediatrics 164


PREP® Self-Assessment PREPSA 2023

Suggested Readings
• Dellinger AM, Kresnow MJ. Bicycle helmet use among children in the United States: the
effects of legislation, personal and household factors. J Safety Res. 2010;41(4):375-380.
doi:10.1016/j.jsr.2010.05.003.
• Jewett A, Beck LF, Taylor C, Baldwin G. Bicycle helmet use among persons 5 years and
older in the United States, 2012. J Safety Res. 2016;59:1-7.
doi:10.1016/j.jsr.2016.09.001.
• Miller PA, Binns HJ, Christoffel KK. Children's bicycle helmet attitudes and use.
Association with parental rules. The Pediatric Practice Research Group. Arch Pediatr
Adolesc Med. 1996;150(12):1259-1264. doi:10.1001/archpedi.1996.02170370037005.
• Okun A. Bicycle safety. Pediatr Rev. 2015;36(3):138-139. doi:10.1542/pir.36-3-138.
• Powell EC, Tanz RR. Tykes and bikes: injuries associated with bicycle-towed child
trailers and bicycle-mounted child seats. Arch Pediatr Adolesc Med. 2000;154(4):351-
353. doi:10.1001/archpedi.154.4.351.

American Academy of Pediatrics 165


PREP® Self-Assessment PREPSA 2023

Question 49
A 38-year-old, gravida 7, para 5 woman who is 38 weeks’ pregnant is brought to the labor and
delivery unit for severe abdominal pain. She has a history of diabetes and hypertension. She
received limited prenatal care. Ultrasonography 1 month ago showed that the placenta was
abnormally placed. The woman’s blood pressure is 230/140 mm Hg. The health care team is
preparing to perform an urgent cesarean section because of the fetal heart tracing shown in Item
Q49. Before delivery, the team is planning for the neonate’s potential management needs beyond
standard resuscitation.

Of the following, in the delivery room this neonate is MOST likely to require
A. blood transfusion
B. chest radiography
C. intravenous calcium gluconate
D. surfactant administration

American Academy of Pediatrics 166


PREP® Self-Assessment PREPSA 2023

Correct Answers: A
The clinical scenario described in the vignette is suggestive of a placental abruption with fetal
blood loss. The neonate in the vignette is most likely to require a blood transfusion of
uncrossmatched O-negative blood.

It is essential to develop an action plan before delivery of this newborn based on the perinatal
history and the fetal heart pattern. The fetal heart pattern shows recurrent late decelerations
consistent with fetal distress. The maternal history of severe abdominal pain and an abnormal
placenta position suggest a placental abruption in a setting of placenta previa; additional risk
factors for placental abruption include the mother’s history of hypertension and current critically
high blood pressure. The fetal bradycardia noted on prenatal monitoring suggests fetal distress
caused by fetal hypoxia, which is likely secondary to blood loss. Volume expansion with a blood
transfusion may be needed if the newborn does not respond to appropriate ventilation, chest
compressions, and epinephrine. It may be beneficial, before delivery, to inform the blood bank of
the potential need for uncrossmatched O-negative blood.

Most neonates transition to the extrauterine environment without difficulty. Eighty-five percent
of term neonates start spontaneous breathing within 30 seconds of delivery; an additional 10%
start breathing in response to drying and stimulation. Five percent of term newborns require
some form of resuscitation, which may include positivepressure ventilation, chest compressions,
medication administration (eg, epinephrine), and/or administration of volume expanders. A pre-
resuscitation team briefing is important to inform all team members of the clinical situation,
assign roles and responsibilities, and prepare an action plan for potential complications. Calcium
gluconate is not indicated as part of the initial resuscitation of a neonate. Because this will be a
delivery of a term infant, surfactant is not indicated in the delivery room. There is no indication
for chest radiography in the delivery room for the neonate in the vignette.

The most important step in neonatal resuscitation is ventilation. Most neonates who require
resuscitation in the delivery room respond to the provision of adequate ventilation. The
indications for positive-pressure ventilation are apnea, gasping, or a heart rate less than 100
beats/min. If the newborn does not respond to positive-pressure ventilation with an increased
heart rate and observed chest rise within 15 seconds, the practitioner should evaluate the
situation with the following ventilation corrective steps: Mask adjustment, Reposition the head
and neck, Suction the mouth and nose, Open the mouth, Pressure increase, Alternative airway
(MR SOPA). Inadequate ventilation leads to hypoxemia, hypoperfusion, and acidosis, which
affects vital organs, especially the newborn brain, resulting in hypoxic ischemic encephalopathy
with long-term neurodevelopmental consequences.

PREP Pearls
• Most neonates transition to the extrauterine environment without difficulty. Five percent
require some form of resuscitation.
• Anticipation and planning are key components of neonatal resuscitation.
• Ventilation is the most important step in neonatal resuscitation.

American Academy of Pediatrics 167


PREP® Self-Assessment PREPSA 2023

ABP Content Specifications(s)


• Recognize the indications for immediate positive-pressure ventilation in a neonate
• Understand the metabolic consequences of continued poor perfusion in a newborn infant

Suggested Readings
• Hainstock LM, Raval GR. Neonatal resuscitation. Pediatr Rev. 2020;41(3):155-158.
doi:10.1542/pir.2018-0203.
• Muoser CA, Kroll H, Wolfe DS. An unexpected cause of recurrent fetal decelerations.
NeoReviews. 2022;23(2):e128– e135. doi:10.1542/neo.23-2-e128.
• Pinheiro JMB. Assessment and stabilization at delivery. In: McInerny TK, Adam HM,
Campbell DE, DeWitt TG, Foy JM, Kamat DM, eds. American Academy of Pediatrics
Textbook of Pediatric Care. American Academy of Pediatrics; 2021:chap 109. Accessed
September 1, 2022. Pediatric Care Online.
• Weiner GM, Zaichkin J. Textbook of Neonatal Resuscitation. 8th ed. American Academy
of Pediatrics; 2021.
• Yager P, Noviski N. Shock. Pediatr Rev. 2010;31(8):311-319. doi:10.1542/pir.31-8-311.

American Academy of Pediatrics 168


PREP® Self-Assessment PREPSA 2023

Question 50
A 13-year-old adolescent girl is seen for a health supervision visit. Her mother is concerned that
the adolescent has been looking into the mirror more often and commenting negatively on the
way her stomach looks. Privately, the adolescent shares that she has been trying to eat less bread
to lose weight and have a flatter stomach. She reports no vomiting and takes no medications. She
has not yet begun her menses. On physical examination, the adolescent’s weight is at the 40th
percentile, 1.4 kg less than it was 3 months ago, and her height is at the 50th percentile. Sexual
maturity rating is 3 for breast development. The remainder of the physical examination findings
are normal.

Of the following, the BEST next step in this adolescent’s management is to


A. prescribe a nutritional supplement drink and have her follow up in 2 months
B. provide reassurance that the adolescent’s thoughts are normal for her age
C. refer her to a cognitive behavioral therapist and a dietitian
D. refer her to an endocrinologist for evaluation of her delayed puberty

American Academy of Pediatrics 169


PREP® Self-Assessment PREPSA 2023

Correct Answer: C
The adolescent in the vignette is experiencing developmental changes both cognitively (concerns
about her body) and physically (breast development, height progression). Her negative thoughts
about her body have led to nutritional restriction and weight loss. Of the response choices, the
best next step in management is to refer her for cognitive behavioral therapy (CBT) and to a
dietitian. Cognitive behavioral therapy has been shown to improve negative body image in
adolescents and adults. Individuals with behaviors related to body image concerns (eg, restriction
or purging, and/or weight loss), such as the adolescent in the vignette, should be referred to a
mental health professional trained in CBT.

Adolescents commonly experience negative thoughts about themselves and their bodies; these
are not always problematic. However, restrictive nutritional intake, purging behaviors (eg,
vomiting, overexercise), and lack of appropriate weight and height progression are concerning
signs that suggest an adolescent needs additional intervention including mental health care. Lack
of weight loss should not reassure physicians, especially in this age group where physical growth
and development is ongoing.

Health care professionals should routinely screen adolescents for body image concerns with
questions such as:
• “Are there body areas that cause you stress?”
• “What do you think your weight should be?”
• “Do you do any body checking, like mirror checks or weighing?”

While a nutritional supplement drink can be helpful in the treatment of malnutrition, the
adolescent in the vignette is not malnourished and does not require this intervention. Her
pubertal progression is normal, so an endocrinology referral is not indicated at this time.

PREP Pearls
• Development of a negative body image is common in adolescence.
• Cognitive behavioral therapy and guidance from a dietitian can be helpful when a
negative body image affects health behaviors or delays growth and development.

ABP Content Specifications(s)


• Understand the effect of rapid body changes on an adolescent’s sense of self
• Identify outcomes and plan the management of a poor self-image in adolescence

Suggested Readings
• Hagan JF, Shaw JS, Duncan PM, eds. Bright Futures: Guidelines of Health Supervision
of Infants, Children and Adolescents. 4th ed. American Academy of Pediatrics; 2017.
• Hornberger LL, Lane MA; AAP The Committee on Adolescence. Identification and
management of eating disorders in children and adolescents. Pediatrics.
2021;147(1):e2020040279. doi:10.1542/peds.2020-040279.
• Seetharaman S, Fields EL. Avoidant/restrictive food intake disorder. Pediatr Rev.
2020;41(12):613-622. doi:10.1542/pir.2019-0133.
American Academy of Pediatrics 170
PREP® Self-Assessment PREPSA 2023

Question 51
A 7-year-old boy is seen in the emergency department for evaluation of a 2-day history of
abdominal pain, watery diarrhea, malaise, and loss of appetite. Five days ago, he visited a
family-owned farm with his siblings. He swam in a chlorinated pool located on the farm and had
exposure to livestock, including calves. Other members of the family are also ill with diarrhea.
The boy’s temperature is 37.4°C; his vital signs are normal for age. His physical examination
findings are unremarkable except for dry oral mucous membranes.

Of the following, the MOST likely cause of this boy’s illness is


A. Clostridium perfringens
B. Cryptosporidium parvum
C. enteropathogenic Escherichia coli
D. norovirus

American Academy of Pediatrics 171


PREP® Self-Assessment PREPSA 2023

Correct Answer: B
The boy in the vignette had onset of watery diarrhea 3 days after exposure to treated (ie,
chlorinated, filtrated) recreational water. This presentation is most suggestive of
Cryptosporidium parvum gastroenteritis. The incubation period of cryptosporidiosis ranges from
3 to 14 days.

Recreational water-associated outbreaks can be caused by a variety of pathogens including


Escherichia coli O157:H7, Shigella, norovirus, and Giardia. Sporadic outbreaks of norovirus
gastroenteritis are common. However, the incubation period of norovirus infection is short (12 to
24 hours) and characterized by sudden onset of vomiting followed by watery diarrhea.
Clostridium perfringens has been implicated in toxin-mediated foodborne illness characterized
by abrupt onset of watery diarrhea and abdominal pain, but it has a shorter incubation period (8
to 12 hours) than Cryptosporidium parvum. Acute diarrhea due to enteropathogenic E coli
(EPEC) is very rare in children living in high-income countries, but it is a frequent cause of acute
and chronic diarrhea in infants living in low- and middle-income countries.

Cryptosporidium parvum is an obligate intracellular coccidian protozoan that infects humans and
animals. Oocysts shed in stools are infectious; person-to-person and animal-to-human (zoonotic)
fecal-oral transmission can occur. Infection can result from ingestion of 10 or fewer oocysts,
which are hardy and can survive in chlorinated water for more than 1 week. Waterborne
outbreaks of diarrheal illness due to cryptosporidiosis are most often linked to swallowing
contaminated public drinking water or water at chlorinated recreational water areas (eg,
swimming pools, waterslides), followed by contact with infected cattle or individuals in child
care centers.

Animal-to-human (zoonotic) transmission has occurred in farmers, veterinarians, and children


visiting petting zoos who had contact with livestock (eg, young calves). Other risk factors for
infection include travel to endemic areas and immunocompromised status (eg, HIV, solid organ
transplantation). Foodborne transmission of cryptosporidiosis has been reported after
consumption of contaminated raw produce, unpasteurized apple cider, and milk.

Cryptosporidiosis is most commonly reported in children aged 1 through 9 years with seasonal
peaks in late summer and early fall (when children are often exposed to outdoor recreational
water). The most common manifestations of cryptosporidiosis include non-bloody watery
diarrhea, abdominal pain, nausea, vomiting, low-grade fever, anorexia, and weight loss.
Asymptomatic gastrointestinal infection can occur, resulting in failure to thrive. The diarrheal
illness is typically self-limited in immunocompetent hosts with symptom resolution by 2 weeks.
In immunocompromised hosts, especially in individuals living with HIV, profuse, watery
(cholera-like) diarrhea or chronic diarrhea for weeks to months associated with weight loss and
extraintestinal disease (eg, biliary tract disease) has been reported. The duration of oocyst
shedding is usually 2 weeks after symptom resolution in immunocompetent hosts but may last
for months in immunocompromised hosts.

American Academy of Pediatrics 172


PREP® Self-Assessment PREPSA 2023

Conventional stool examination for ova and parasites may not detect the small oocysts (4-6 µm
in diameter) of Cryptosporidium species. Detection may be enhanced by using a modified
Kinyoun acid-fast stain on concentrated stool specimens. More specific testing on stool samples
include detection of oocysts by direct immunofluorescent antibody or detection of
Cryptosporidium antigen by enzyme immunoassay. Given the intermittent shedding of oocysts, a
minimum of 3 stool specimens collected on separate days is recommended prior to excluding
infection. Molecular diagnostic methods using nucleic acid amplification (eg, gastrointestinal
pathogen panel) are available in many health care settings that target several intestinal bacterial,
viral, and parasitic pathogens including Cryptosporidium species in a single multiplex
polymerase chain reaction platform.

Anti-parasitic therapy is not indicated for mild illness in immunocompetent individuals, as it is


usually self-limiting. For moderate to severe illness, a 3-day course of oral nitazoxanide is
recommended for immunocompetent individuals aged 1 year and older. Immunocompromised
individuals are treated with nitazoxanide for 14 days or longer based on clinical response.
Cases of cryptosporidiosis should be reported to the local or state public health department.
Control measures at recreational water venues should include education of the public and staff on
healthy swimming behaviors such as:
• Avoidance of swimming and other recreational water activities when ill with diarrhea
• Avoidance of swallowing recreational water
• Hand hygiene with soap and water (Cryptosporidium species are resistant to alcohol-
based hand sanitizers)

Information on Healthy Swimming from the Centers for Disease Control and Prevention can be
found here:
https://www.cdc.gov/healthywater/swimming/swimmers/steps-healthy-swimming.html.

PREP Pearls
• Cryptosporidium is a leading cause of diarrheal outbreaks associated with exposure to
chlorinated recreational water (eg, swimming pools, waterslides).
• Fecal-oral transmission of Cryptosporidium infection occurs via ingestion of oocysts
excreted by infected hosts.
• Manifestations of cryptosporidiosis include non-bloody, watery diarrhea with abdominal
pain, nausea, vomiting, lowgrade fever, anorexia, and weight loss; asymptomatic
gastrointestinal infection can occur resulting in failure to thrive.

ABP Content Specifications(s)


• Recognize which pathogens can be transmitted by contaminated recreational water, while
providing guidance about prevention of such infections

American Academy of Pediatrics 173


PREP® Self-Assessment PREPSA 2023

Suggested Readings
• Abdel-Haq N, Chearskul P, Rafee Y, Asmar BI. Parasitic infections. In: McInerny TK,
Adam HM, Campbell DE, DeWitt TG, Foy JM, Kamat DM, eds. American Academy of
Pediatrics Textbook of Pediatric Care. American Academy of Pediatrics; 2021:chap 308.
Accessed September 1, 2022. Pediatric Care Online.
• American Academy of Pediatrics. Cryptosporidiosis. In: Kimberlin DW, Barnett ED,
Lynfield R, Sawyer MH, eds. Red Book: 2021–2024 Report of the Committee on
Infectious Diseases. 32nd ed. American Academy of Pediatrics; 2021. Accessed
September 1, 2022. Red Book Online.
• Custodio H. Protozoan parasites. Pediatr Rev. 2016;37(2):59-69. doi:10.1542/pir.2015-
0006.
• Gharpure R, Perez A, Miller AD, Wikswo ME, Silver R, Hlavsa MC. Cryptosporidiosis
outbreaks - United States, 2009-2017. MMWR Morb Mortal Wkly Rep.
2019;68(25):568-572. doi:10.15585/mmwr.mm6825a3.
• Hlavsa MC, Cikesh BL, Roberts VA, et al. Outbreaks associated with treated recreational
water — United States, 20002014. MMWR Morb Mortal Wkly Rep. 2018;67(19):547-
551. Accessed September 1, 2022.
https://www.cdc.gov/mmwr/volumes/67/wr/mm6719a3.htm.

American Academy of Pediatrics 174


PREP® Self-Assessment PREPSA 2023

Question 52
An 8-month-old boy is evaluated for a 3-day history of worsening cough, nasal congestion, and
low-grade fever. His mother reports that he is irritable, has difficulty breathing, and is eating and
drinking less than usual. He has good urine output.

On physical examination, the infant is ill-appearing and fussy but consolable. His temperature is
38.5°C, heart rate is 102 beats/min, respiratory rate is 55 breaths/min, and oxygen saturation is
95% in room air. He has mild intercostal retractions and nasal flaring. Breath sounds are coarse
throughout with crackles in the right base extending into the right axilla; no wheezing is heard.
The remainder of the physical examination findings are unremarkable.

Of the following, the BEST next step in this infant’s management is to


A. administer antibiotics
B. obtain computed tomography scan of the chest
C. perform rapid intubation and start mechanical ventilation
D. recommend he return for reevaluation tomorrow

American Academy of Pediatrics 175


PREP® Self-Assessment PREPSA 2023

Correct Answer: A
The infant in the vignette with fever, tachypnea, increased work of breathing, and focal crackles
on physical examination most likely has community-acquired pneumonia (CAP). Although most
CAP in children is caused by viruses, this infant’s focal findings and worsening symptoms make
a bacterial etiology more likely. Based on the 2011 Infectious Diseases Society of America
(IDSA) guidelines for management of CAP, administration of antibiotics is the best next
intervention for this infant with likely bacterial pneumonia.

The infant in the vignette appears ill and meets the IDSA guideline criteria for respiratory
distress (Item C52A). Admission to the hospital should be considered, and chest radiography
would be appropriate; however chest computed tomography is not indicated as a first-line
imaging study. Although the infant is in mild distress, he has normal oxygen saturation and does
not show signs of imminent respiratory failure; therefore, there is no indication for intubation
and mechanical ventilation. However, given his level of distress and the concern for bacterial
pneumonia, he does need intervention beyond observation and reevaluation. An important
consideration regarding his need for hospital admission is his risk for dehydration, as his mother
reports poor oral intake.

Item C52A: Criteria for Respiratory Distress in Children With Pneumonia


1. Tachypnea, respiratory rate, breaths/mina
1.1. Age 0–2 months: .60
1.2. Age 2–12 months: .50
1.3. Age 1–5 Years: .40
1.4. Age .5 Years: .20
2. Dyspnea
3. Retractions (suprasternal, intercostals, or subcostal)
4. Grunting
5. Nasal flaring
6. Apnea
7. Altered mental status
8. Pulse oximetry measurement ,90% on room air

The criteria for normal respiratory rate is variable within and across age groups, especially in
infants. However, a rate of 55 breaths/min is at or above the accepted limit for an 8-month-old
(depending on the reference used), and is an indicator of respiratory distress. Item C52B
displays percentile ranges of respiratory rate by age.

American Academy of Pediatrics 176


PREP® Self-Assessment PREPSA 2023

Respiratory rate centile chart in awake children (centiles—3rd, 10th, 25th, 50th, 75th, 90th, 97th).

Respiratory rate centile chart in asleep children (centiles—3rd, 10th, 25th, 50th, 75th, 90th, 97th).

An elevated respiratory rate (tachypnea) is a sensitive indicator of illness at any age. A careful
assessment of respiratory rate and work of breathing (eg, use of accessory muscles, retractions,
nasal flaring, head bobbing) can help determine illness severity in a child. An elevated
respiratory rate is most suggestive of a respiratory infection or other airway and pulmonary
parenchymal disease; however, it can also be associated with non-respiratory conditions
including metabolic derangements, cardiac dysfunction, anxiety, and hyperventilation. A
decreased respiratory rate (hypopnea) raises concern for a condition affecting central control of
respiration.

Measurement of respiratory rate in the sleeping child may provide the most accurate assessment
of the child’s status by eliminating anxiety and other external factors that may affect the rate of
breathing. Because of natural variability in the respiratory cycle, it is important to count the
respiratory rate for a full minute. Significant elevation or depression of the respiratory rate on
physical examination should be investigated beginning with a careful history and physical
examination. The need for additional evaluation should be determined based on findings on the
initial assessment.
American Academy of Pediatrics 177
PREP® Self-Assessment PREPSA 2023

PREP Pearls
• Normal respiratory rate varies widely across ages; infants show greater variability than
older children.
• Respiratory rate is a sensitive indicator of illness; abnormal sleeping respiratory rate is
the best measure of true dysfunction.
• Any child with unexplained tachypnea or hypopnea should undergo careful evaluation for
underlying illness.

ABP Content Specifications(s)


• Plan the appropriate clinical and diagnostic evaluation of tachypnea of various etiologies
• Recognize the presence of tachypnea as a sensitive indicator of respiratory disease

Suggested Readings
• Bloomfield D. Tachypnea. Pediatr Rev. 2002;23(8):294-295;discussion 294-295.
doi:10.1542/pir.23-8-294.
• Bradley JS, Byington CL, Shah SS, et al. Executive summary: the management of
community-acquired pneumonia in infants and children older than 3 months of age:
clinical practice guidelines by the Pediatric Infectious Diseases Society and the Infectious
Diseases Society of America. Clin Infect Dis. 2011;53(7):617-630.
doi:10.1093/cid/cir625.
• Light M. Pneumonia. In: McInerny TK, Adam HM, Campbell DE, DeWitt TG, Foy JM,
Kamat DM, eds. American Academy of Pediatrics Textbook of Pediatric Care. American
Academy of Pediatrics; 2021:chap 315. Accessed September 1, 2022. Pediatric Care
Online.
• Vo P, Karash VS. Respiratory failure. Pediatr Rev. 2014;35(11):476-484.
doi:10.1542/pir.35-11-476.

American Academy of Pediatrics 178


PREP® Self-Assessment PREPSA 2023

Question 53
The mother of a 9-year-old girl provides the office with a copy of her recent school evaluation,
which includes cognitive and academic achievement testing. The girl currently receives school
interventions to help with reading. Since the interventions were implemented, her mother has
noted some improvement when working on homework, but the girl’s grades continue to be
below those of her twin sister.

Cognitive testing results are shown:


Area Result Quotient
Verbal comprehension 104
Perceptual reasoning 108
Working memory 87
Processing speed 89
General ability index 106

Academic achievement testing results are shown:


Area Result Quotient
Reading
Letter-word identification 82
Passage comprehension 84
Word attack 83
Math
Math calculation 110
Applied problems 102
Writing
Spelling 85
Writing sample 83

The girls’ mother states, “I know she is a smart girl just like her twin, but I don’t understand why
these numbers are so different.” She asks how to best support her daughter at home and at
school.

Of the following, the BEST next step in this girl’s management is to


A. recommend time extensions for tests/assignments involving reading or writing
B. recommend transfer to a magnet school for math and technology
C. refer her for occupational therapy
D. start stimulant medication to help with attention and improve working memory

American Academy of Pediatrics 179


PREP® Self-Assessment PREPSA 2023

Correct Answer: A
The girl in the vignette has average non-verbal and verbal cognitive ability. Her academic
achievement testing is consistent with a specific learning disorder in the domains of reading and
written expression. Her scores in these areas are significantly lower (more than 15 points) than
her cognitive scores. In addition to the reading interventions she is currently receiving,
implementing accommodations, including extended time for completing her work, is the best
next step in this girl’s management. Extra time to complete her work may be beneficial given the
girl’s slower processing speed and working memory difficulties.

Cognitive testing and achievement testing are recommended to evaluate for a learning disability
and eligibility for special education support. Cognitive testing measures verbal, visual-spatial,
and problem solving skills; scores typically remain consistent over time and are usually
predictive of school performance. Results of cognitive testing are influenced by both genetic and
environmental factors. Identical twins have more similar cognitive test results compared to
fraternal twins. The environment, including home setting, parental education level, parental
mental health, nutrition, and school attendance, also plays a role. Performance on cognitive
testing may be impacted by cultural or language bias, the setting testing was performed in,
interaction between the examiner and test-taker, and motivation or other emotional factors in the
test-taker. Differences of greater than 15 points between cognitive subscores or between
cognitive and academic achievement scores requires further investigation, but it does not confirm
the presence of a learning disability.

While achievement test results for the girl in the vignette are consistent with a language-based
learning disorder, which includes written expression, there is no information regarding the
presence of fine motor impairment or difficulties with handwriting that would warrant referral
for occupational therapy. Monitoring fine motor skills, including handwriting, is important;
however, the girl’s achievement scores in spelling suggest phonetic deficits that limit her ability
to sound out words and form sentences.

Identifying areas of strength and encouraging activities (both academic and extracurricular)
supporting these domains can enhance self-esteem. However, while the girl in the vignette has
average scores for math calculation and applied problems, transfer to a magnet school focused on
mathematics would not be recommended in place of intervention and accommodations to address
her specific learning disorder.

Working memory is the ability to retain information and apply it to cognitive tasks. Processing
speed is a measure of how quickly information is processed and the rate at which cognitive tasks
are completed. Both of these scores are discrepant compared to cognitive scores for the girl in
the vignette. This type of discrepancy is common in children with attention-deficit/hyperactivity
disorder (ADHD). More information from the girl’s mother and her teacher regarding her
behavior at home and in the classroom is needed to evaluate her for ADHD. A stimulant
medication should be considered only after a diagnosis of ADHD is confirmed. In a child with
ADHD, stimulant medications do not increase processing speed or improve working memory,
but can improve the completion of tasks by increasing attention.
American Academy of Pediatrics 180
PREP® Self-Assessment PREPSA 2023

Pediatricians play a valuable role in referring children and supporting parents seeking school
evaluation for a learning disability, as well as in helping families understand and interpret
evaluation results. It is important to perform a thorough history and physical examination to
assess for comorbid conditions (eg, ADHD, fine motor impairment) that may affect academic
performance. Identified strengths and areas requiring support should guide health care
professionals’ recommendations for the school and home settings.

PREP Pearls
• Pediatricians should facilitate the school evaluation process for a learning disability and
assist the family in understanding and interpreting the results.
• Discrepancies between cognitive and academic achievement scores on psychoeducational
testing help to identify areas requiring intervention, but alone they do not define a
learning disorder.
• Identifying activities and areas of strength for children with learning disorders is vital in
supporting self-esteem and promoting a positive attitude toward school.

ABP Content Specifications(s)


• Interpret the results of specialized and standardized achievement tests, with emphasis on
understanding the significance of discrepancies between categories
• Identify factors that can influence the results of intelligence quotient tests

Suggested Readings
• Braaten EB, Norman D. Intelligence (IQ) testing. Pediatr Rev. 2006;27(11):403-408.
Accessed September 1, 2022. https://pubmed.ncbi.nlm.nih.gov/17079505/.
• Frankowski BL. Learning difficulty. In: McInerny TK, Adam HM, Campbell DE, DeWitt
TG, Foy JM, Kamat DM, eds. American Academy of Pediatrics Textbook of Pediatric
Care. American Academy of Pediatrics; 2021:chap 172. Accessed September 1, 2022.
Pediatric Care Online.
• Rey-Casserly C, McGuinn L, Lavin A; Committee on Psychosocial Aspects of Child and
Family Health,section on
• Developmental and Behavioral Pediatrics. School-aged children who are not progressing
academically: considerations for pediatricians. Pediatrics. 2019;144(4):e20192520.
doi:10.1542/peds.2019-2520.
• Rimrodt SL, Lipkin PH. Learning disabilities and school failure. Pediatr Rev.
2011;32(8):315-324. doi:10.1542/pir.32-8-315

American Academy of Pediatrics 181


PREP® Self-Assessment PREPSA 2023

Question 54
A 6-month-old infant, born at 27 weeks’ gestation, with grade 2 vesicoureteral reflux,
gastroesophageal reflux, and chronic lung disease is seen in the emergency department for
lethargy. His daily medications include fluticasone, furosemide, lansoprazole, and nitrofurantoin.
His temperature is 36.8°C, heart rate is 120 beats/min, respiratory rate is 30 breaths/min, blood
pressure is 85/65 mm Hg, and oxygen saturation is 99% on his home oxygen support of 0.5
L/min via nasal cannula. The infant’s anterior fontanelle is slightly sunken, and his mucous
membranes are moist and pink. On lung auscultation, fine rhonchi are heard throughout without
wheezing or rales. There are no retractions or other signs of increased work of breathing. The
remainder of his physical examination findings are normal.

Laboratory data are shown:


Laboratory Test Result
Sodium 130 mEq/L (130 mmol/L)
Potassium 2.8 mEq/L (2.8 mmol/L)
Chloride 89 mEq/L (89 mmol/L)
Carbon dioxide 38 mEq/L (38 mmol/L)
Blood urea nitrogen 10 mg/dL (3.6 mmol/L)
Creatinine 0.3 mg/dL (26.5 µmol/L)

Of the following, the medication MOST likely to have led to this infant’s findings is
A. fluticasone
B. furosemide
C. lansoprazole
D. nitrofurantoin

American Academy of Pediatrics 182


PREP® Self-Assessment PREPSA 2023

Correct Answers: B
The infant in the vignette, with chronic lung disease related to prematurity, has signs of mild
dehydration (mildly sunken anterior fontanelle), and electrolyte abnormalities including
hyponatremia, hypokalemia, hypochloremia, and an elevated bicarbonate level. These findings
are consistent with a contraction metabolic alkalosis induced by chronic diuretic therapy. Thus,
of the response choices, furosemide (used to control excess fluid in the lungs) is the most likely
medication to have contributed to this infant’s condition.

Fluticasone (an inhaled corticosteroid used to manage pulmonary inflammation), lansoprazole (a


proton-pump inhibitor used to treat gastroesophageal reflux), and nitrofurantoin (a bactericidal
antibiotic used for urinary tract infection prophylaxis), commonly used to treat complications of
prematurity, are not associated with the electrolyte abnormalities seen in the infant in the
vignette.

Furosemide is a loop diuretic that inhibits reabsorption of sodium and chloride in the ascending
loop of Henle and the proximal and distal renal tubules by interfering with the chloride-binding
co-transport system. Both natriuresis (sodium loss) and diuresis (water loss) result. Diuresis
decreases extracellular fluid volume and concentrates extracellular serum bicarbonate,
contributing to metabolic alkalosis. Historically, this process was known as “contraction
alkalosis,” and the primary mechanism was thought to be due to decreased fluid volume and the
resulting concentration of serum bicarbonate. However, newer studies have demonstrated that
chloride plays a major role in the development of metabolic alkalosis, and some have suggested
that “chloride depletion alkalosis” replace the term “contraction alkalosis” to more accurately
reflect the underlying mechanism.

Metabolic alkalosis is often associated with chloride anion loss, either through gastric fluid or
excessive urinary excretion. Infants with severe emesis syndromes (eg, pyloric stenosis)
classically present with a hypochloremic metabolic alkalosis. Additionally, chloride-wasting
diuretics can induce a metabolic alkalosis. Infants and children with chronic lung disease can
have a chronic respiratory acidosis from retention of carbon dioxide, and may develop a
compensatory metabolic alkalosis to maintain a normal acid-base balance.

Gitelman and Bartter syndromes are associated with metabolic alkalosis, a high urinary excretion
of serum potassium, and hypokalemia. Bartter syndrome is an autosomal recessive condition
characterized by hypokalemia, excessive renal wasting of sodium chloride, and metabolic
alkalosis due to reduced activity of one of several electrolyte transporters in the ascending loop
of Henle. In addition to the expected electrolyte abnormalities associated with salt-wasting,
impaired sodium chloride reabsorption leads to volume depletion and increased activity of the
renin-angiotensin-aldosterone system. Secondary hyperaldosteronism and increased sodium
excretion enhances potassium and hydrogen ion loss in the urine leading to hypokalemia and
metabolic alkalosis. Children with Bartter syndrome are often seen with failure to thrive,
lethargy, polydipsia, polyuria, dehydration, hypotonia, and developmental delay.

American Academy of Pediatrics 183


PREP® Self-Assessment PREPSA 2023

PREP Pearls
• Metabolic alkalosis is often caused by hypochloremia.
• Infants and children on chronic diuretic therapy with loop diuretics (eg, furosemide) are
at risk for developing a chloride depletion alkalosis.
• Bartter syndrome can cause hypokalemic, hypochloremic, metabolic alkalosis; this
diagnosis should be suspected in infants with hyponatremia, hypokalemia, failure to
thrive, and dehydration.

ABP Content Specifications(s)


• Identify factors contributing to metabolic alkalosis

Suggested Readings
• Hsu BS, Lakhani SA, Wilhelm M. Acid-base disorders. Pediatr Rev. 2016;37(9):361-369.
doi:10.1542/pir.2015-0093.
• Mahajan P. Fluids, electrolytes, and acid-base composition. In: McInerny TK, Adam
HM, Campbell DE, DeWitt TG, Foy JM, Kamat DM, eds. American Academy of
Pediatrics Textbook of Pediatric Care. American Academy of Pediatrics; 2021:chap 58.
Accessed September 1, 2022. Pediatric Care Online.
• Schwaderer AL, Schwartz GJ. Back to basics: acidosis and alkalosis. Pediatr Rev.
2004;25(10):350-357. doi:10.1542/pir.25-10-350.
• Tergestina M, Chandran S, Kumar M. Case 2: Metabolic alkalosis in a neonate 12 hours
after birth. NeoReviews. 2018;19(10):e613-e615. doi:10.1542/neo.19-10-e613.
• Thakore P, Anderson M, Yosypiv IV. Classic Bartter syndrome: a cause of severe
hypokalemic metabolic alkalosis. Clin Pediatr (Phila). 2019;58(14):1557-1561.
doi:10.1177/0009922819857535.

American Academy of Pediatrics 184


PREP® Self-Assessment PREPSA 2023

Question 55
A 6-month-old male infant is seen for a health supervision visit. His parents are concerned that
he seems to have a lazy eye. The infant was born at full term after an unremarkable pregnancy
and delivery. He has been healthy and is developing normally. His family history is
unremarkable. The infant’s growth parameters are normal. Corneal light reflex examination of
his eyes is shown in Item Q55. The remainder of his physical examination findings are normal.

Of the following, the BEST next step in this infant’s management is to


A. initiate intermittent eye patching of the left eye
B. provide reassurance that no treatment is needed
C. refer him to early intervention
D. refer him to a pediatric ophthalmologist

American Academy of Pediatrics 185


PREP® Self-Assessment PREPSA 2023

Correct Answer: B
The infant in the vignette does not have strabismus; his corneal light reflex is symmetric. Instead,
he has pseudostrabismus, and his parents can be reassured that no treatment is needed. Neither
referral to an ophthalmologist nor eye patching is indicated for pseudostrabismus. He does not
need to be referred to early intervention because there is no current concern for developmental
delay or visual deficit.

Performing regular vision screening is critical, especially in early childhood, in order to


effectively detect and treat vision problems. For example, amblyopia treatment is almost
completely ineffective after about 7 to 9 years of age. Methods for vision screening at specific
ages are outlined in Item C55A.

American Academy of Pediatrics 186


PREP® Self-Assessment PREPSA 2023

Several examination techniques can be used to identify eye misalignments. The red reflex test
should be performed starting at birth and is ideally performed so that both eyes can be visualized
at the same time to detect any asymmetry (Loh). This technique can detect not only
retinoblastomas and cataracts but also strabismus.

The corneal light reflex is predictably asymmetric if there is any misalignment. Esotropia will
result in a temporal displacement of the light in the affected eye and exotropia will result in a
nasal displacement.

The cover test is more accurate for detecting strabismus than the corneal light reflex but is more
complex to perform. The cover test is demonstrated in Item C55B. Detection of refixation
indicates strabismus.

Instrument-based testing can also screen for eye misalignments. However, not all devices have
this capability, so physicians must refer to their devices’ instructions.

Intermittent eye crossing can be normal in newborns, but should resolve by 2 to 4 months of age.
Persistent eye crossing, however, is never normal at any age, and affected children should be
referred to an ophthalmologist.

PREP Pearls
• A symmetric corneal light reflex makes strabismus highly unlikely.
• Amblyopia treatment is almost completely ineffective after about 7 to 9 years of age.
• Intermittent eye crossing can be normal in newborns, but should resolve by 2 to 4 months
of age.

American Academy of Pediatrics 187


PREP® Self-Assessment PREPSA 2023

ABP Content Specifications(s)


• Evaluate a patient for ocular tropias and phorias
• Plan the appropriate evaluation of strabismus, including timing of evaluation to prevent
complications
• Differentiate the clinical findings associated with strabismus from those of
pseudostrabismus

Suggested Readings
• American Academy of Pediatrics; Section on Ophthalmology; American Association for
Pediatric Ophthalmology And Strabismus; American Academy of Ophthalmology;
American Association of Certified Orthoptists. Red reflex examination in neonates,
infants, and children. Pediatrics. 2008;122(6):1401-1404. doi:10.1542/peds.2008-2624.
• Loh AR, Chiang MF. Pediatric vision screening. Pediatr Rev. 2018;39(5):225-234.
doi:10.1542/pir.2016-0191.
• Nelson LB, Johnson BB, O’Hara M. Amblyopia and strabismus. In: McInerny TK, Adam
HM, Campbell DE, DeWitt TG, Foy JM, Kamat DM, eds. American Academy of
Pediatrics Textbook of Pediatric Care. American Academy of Pediatrics; 2021:chap 214.
Accessed September 1, 2022. Pediatric Care Online.
• Wallace DK, Morse CL, Melia M, et al. Pediatric eye evaluations preferred practice
pattern®: I. Vision screening in the primary care and community setting; II.
Comprehensive ophthalmic examination. Ophthalmology. 2018;125(1):184-227.
doi:10.1016/j.ophtha.2017.09.032.

American Academy of Pediatrics 188


PREP® Self-Assessment PREPSA 2023

Question 56
A 1-day-old neonate is seen for routine newborn care in the nursery. He was born via precipitous
vaginal delivery at 39 weeks’ gestation. His birth weight was 3,000 g. His mother emigrated
from West Africa 2 weeks ago. On review of her prenatal laboratory data, it is noted that the
hepatitis B test result is pending. The findings on the neonate’s physical examination are normal
with no palpable hepatomegaly. He received the hepatitis B vaccine shortly after delivery. The
nursing student caring for him asks what results on the mother’s hepatitis B testing would place
him at greatest risk of perinatal transmission.

Of the following, the BEST response to the student’s question is


Hepatitis B Hepatitis B Hepatitis B e Hepatitis B e
surface antigen surface antibody antigen antibody
(HBsAg) (HBsAb) (HBeAg) (HBeAb)
Row A Positive Negative Negative Negative
Row B Positive Negative Positive Negative
Row C Negative Positive Negative Negative
Row D Negative Positive Negative Positive

A. Row A
B. Row B
C. Row C
D. Row D

American Academy of Pediatrics 189


PREP® Self-Assessment PREPSA 2023

Correct Answer: B
Perinatal hepatitis B transmission occurs at the time of labor and delivery. The risk of
transmission to the newborn is determined based on the presence of maternal hepatitis B surface
antigen (HBsAg) and hepatitis B e antigen (HBeAg). The presence of HBsAg implies either an
acute (<6 months) or chronic (≥6 months) infection in the mother. The presence of HBeAg
indicates a high level of transmissibility. For a mother positive for both HBsAg and HBeAg, the
risk of perinatal transmission is as high as 90%. For a mother that is only HBsAg positive, the
risk is around 30%. Other risk factors associated with high transmissibility include a high viral
DNA level >2,000 IU/mL, age <25 and <3 doses of maternal hepatitis B vaccine. Detection of
hepatitis B surface antibody (>10 mIU/ML) indicates disease immunity, either from
immunization or natural infection. Detection of hepatitis B e antibody along with hepatitis B
surface antibody indicates past infection.

Hepatitis B is a partially double-stranded DNA virus that belongs to the Hepadnaviridae family.
Other hepatotropic viruses (hepatitis A, C, D, E) have an RNA genome. Hepatitis B virus is
transmitted through body fluids and blood. Common modes of transmission include
percutaneous (sharing contaminated needles, tattooing), permucosal (sexual), and perinatal.
Transmission through blood products is rare due to universal screening of donors for HBsAg.

Perinatal transmission occurs with exposure to blood and maternal secretions during labor and
delivery. All pregnant women are routinely screened for HBsAg at the first prenatal visit. If
HBsAg is positive, further testing is performed including HBeAg, HBeAb, hepatitis B DNA
levels, and liver enzyme levels. A comprehensive strategy to eliminate perinatal hepatitis B
transmission has been implemented in the United States that includes:
1. Universal immunization of all infants at birth
2. Postexposure prophylaxis with hepatitis B immunoglobulin for neonates born to mothers that
test positive for hepatitis B surface antigen or with unknown hepatitis B status

• If a mother’s HBsAg status is positive, in addition to hepatitis B vaccine at birth, the


infant should be administered hepatitis B immunoglobulin within 12 hours of delivery
irrespective of the birth weight.
• If a mother’s HBsAg status is unknown at the time of delivery of a neonate with a birth
weight less than 2,000 g, hepatitis B immunoglobulin should be administered within 12
hours after birth.
• If a mother’s HBsAg status is unknown at the time of delivery of a neonate with a birth
weight of greater than or equal to 2,000 g, hepatitis B immunoglobulin should be
administered within 7 days of birth in cases when the HBsAg result is found to be
positive or is still unknown.

American Academy of Pediatrics 190


PREP® Self-Assessment PREPSA 2023

PREP Pearls
• For a mother positive for both HBsAg and HBeAg, the risk of perinatal transmission is as
high as 90%.
• In addition to hepatitis B vaccine at birth, a neonate born to a mother that tested positive
for hepatitis B surface antigen should receive hepatitis B immunoglobulin within 12
hours of delivery, irrespective of the neonate’s birth weight.
• If a mother’s hepatitis B surface antigen status is unknown at the time of delivery,
neonates with a birth weight of less than 2,000 g should receive hepatitis B
immunoglobulin within 12 hours after birth and those with a birth weight greater than or
equal to 2000 g should receive hepatitis B immunoglobulin within 7 days of birth, if the
status remains unknown or is found to be positive.

ABP Content Specifications(s)


• Understand the epidemiology of the hepatitis B virus
• Understand the risks associated with perinatally acquired hepatitis B virus infections

Suggested Readings
• American Academy of Pediatrics. Hepatitis B. In: Kimberlin DW, Barnett ED, Lynfield
R, Sawyer MH, eds. Red Book: 2021– 2024 Report of the Committee on Infectious
Diseases. 32nd ed. American Academy of Pediatrics; 2021. Accessed September 1, 2022.
Red Book Online.
• Byrd KK, Murphy TV, Hu DJ. Hepatitis B and hepatitis D viruses. In: Long SS, eds.
Principles and Practice of Pediatric Infectious Diseases. 4th ed. Elsevier; 2012:1077-
1087.e6.
• Hardikar W, Schwarz KB. Hepatitis. In: McInerny TK, Adam HM, Campbell DE,
DeWitt TG, Foy JM, Kamat DM, eds. American Academy of Pediatrics Textbook of
Pediatric Care. American Academy of Pediatrics; 2021:chap 265. Accessed September 1,
2022. Pediatric Care Online.
• Noor A, Fiorito T, Krilov LR. Viral infections in the nursery. In: Martin GI, Rosenfield
W, eds. Common Problems in the Newborn Nursery: An Evidence and Case Based
Guide. Springer; 2019:81-88.
• Schillie S, Vellozzi C, Reingold A, et al. Prevention of hepatitis B virus infection in the
United States: recommendations of the Advisory Committee on Immunizations. MMWR
Recomm Rep. 2018;67(1):1-31. Accessed September 1, 2022.
https://www.cdc.gov/mmwr/volumes/67/rr/pdfs/rr6701-H.PDF.

American Academy of Pediatrics 191


PREP® Self-Assessment PREPSA 2023

Question 57
A 12-year-old girl is seen in the clinic for evaluation of a 3-month history of abdominal pain and
diarrhea. She has 5 to 8 loose, malodorous, floating, greasy-appearing stools daily. She lost 5 kg
unintentionally during the 3-month period. Physical examination reveals a weight of 36.3 kg
(24th percentile for age), height of 150 cm (44th percentile for age), and a body mass index of 16
kg/m2 (19th percentile for age). She appears pale. Her abdomen is distended but soft and
nontender. The remainder of the girl’s physical examination findings are unremarkable.

Laboratory data are shown:


Laboratory Test Result
Stool
pH 7.0
Occult blood Negative
Reducing substances Negative
Ova and parasite Negative
Bacterial pathogen testing Negative
Blood
Hemoglobin 10.1 g/dL (101 g/L)
Erythrocyte sedimentation rate 2 mm/h
25-Hydroxyvitamin D 9 ng/mL (22.46 nmol/L)
Albumin 4.3 g/dL (43 g/L)

Of the following, the BEST treatment for this girl is


A. albumin 25% infusion
B. gluten-free diet
C. lactase enzyme replacement therapy
D. low-sucrose diet

American Academy of Pediatrics 192


PREP® Self-Assessment PREPSA 2023

Correct Answer: B
The girl in the vignette has evidence of increased intestinal fat losses (steatorrhea and fat-soluble
vitamin D deficiency), resulting from fat maldigestion and/or fat malabsorption. Of the response
choices, a gluten-free diet is the only treatment for a fat malabsorption disorder.

On the basis of the girl’s laboratory and physical examination findings, a diagnosis of celiac
disease was suspected. Further testing revealed an elevated tissue transglutaminase IgA antibody
level, and findings on endoscopy with biopsy confirmed the diagnosis. A gluten-free diet is the
treatment for celiac disease. Albumin infusions are used to treat symptomatic hypoalbuminemia
caused by severe protein-losing enteropathy. The albumin level of the girl in the vignette is
within the reference range, and an albumin infusion would not address her fat malabsorption and
steatorrhea. Lactase enzyme replacement therapy can be used to treat diarrhea that results from
malabsorption of lactose, and a low-sugar diet may resolve diarrhea caused by malabsorption of
sucrose, but neither treatment would decrease this girl’s fat malabsorption and steatorrhea.

Normal function of the pancreas, liver, and small intestine is critical for normal fat metabolism
and absorption. The pancreas produces digestive enzymes (eg, lipase), which break down
ingested fats (eg, long-chain triglycerides [LCTs]) to insoluble fatty acids. Bile acids form
micelles with the fatty acids, rendering them water soluble and, therefore, able to be absorbed
through the intestinal brush border. Medium-chain triglycerides (MCTs) are water soluble and do
not require pancreatic enzymes or bile for absorption. Common dietary sources of LCTs include
most fats (animal and plant origin). Natural sources of MCTs include coconut and palm oils;
synthetically produced MCT oils are also available.

Steatorrhea (excessive fat in the stool) indicates fat maldigestion or malabsorption. Signs and
symptoms of steatorrhea include oily or greasy-appearing stools, floating stools that are often
difficult to flush, frequent stooling, stool urgency, and severe diaper rash in infants and diapered
children. Weight loss or poor weight gain and fatsoluble vitamin (A, D, E, and K) deficiencies
may occur from fat maldigestion or malabsorption. Stool testing reveals an elevated fecal fat
concentration.

Conditions that cause fat maldigestion are listed in Item C57. If fat malabsorption is suspected,
disorders of the small intestine (enteropathies) should be investigated. Causes of malabsorption
include infections (eg, Giardia duodenalis [formerly lamblia]), inflammatory enteropathies (eg,
Crohn disease, celiac disease, eosinophilic enteropathy, postinfectious diarrhea, bacterial
overgrowth, and autoimmune enteropathy), and anatomical causes (eg, malrotation with
intermittent volvulus).

Treatment of steatorrhea is tailored to the cause; treatment of the underlying condition helps to
alleviate the complications of fat malabsorption. Attention should be given to the nutritional
status of the child. Achieving optimal nutritional status by maximizing caloric intake (eg,
supplementation with MCT oil) and identifying and treating fatsoluble vitamin deficiencies are
imperative.

American Academy of Pediatrics 193


PREP® Self-Assessment PREPSA 2023

PREP Pearls
• Steatorrhea (Increased fecal fat) occurs when ingested fat is maldigested and/or
malabsorbed.
• Complications of fat maldigestion or malabsorption include diarrhea, abdominal pain,
diaper dermatitis in infants and diapered children, weight loss or poor weight gain, and
fat-soluble vitamin deficiencies.
• Treatment of steatorrhea includes identifying and managing the underlying cause while
supporting nutrition and growth through adequate caloric intake and fat-soluble vitamin
supplementation.

ABP Content Specifications(s)


• Recognize clinical situations in which bacterial overgrowth may play a role in
malabsorption Plan the appropriate management of fat malabsorption

Suggested Readings
• Ammoury RF, Croffie JM. Malabsorptive disorders of childhood. Pediatr Rev.
2010;31(10):407-416. doi:10.1542/pir.3110-407.
• Diab L, Krebs NF. Vitamin excess and deficiency. Pediatr Rev. 2018;39(4):161-179.
doi:10.1542/pir.2016-0068.
• Ediger TR, Hill ID. Celiac disease. Pediatr Rev. 2021;42(10):529–538.
doi:10.1542/pir.2020-000711.
• Safta AM, Kerner JA Jr. Gluten-sensitive enteropathy (celiac sprue). In: McInerny TK,
Adam HM, Campbell DE, DeWitt TG, Foy JM, Kamat DM, eds. American Academy of
Pediatrics Textbook of Pediatric Care. American Academy of Pediatrics; 2021:chap 199.
Accessed September 1, 2022. Pediatric Care Online.

American Academy of Pediatrics 194


PREP® Self-Assessment PREPSA 2023

Question 58
A 4-year-old boy with trisomy 21 is seen in the emergency department with a femur fracture. His
mother brought him to the emergency department because he would not walk when he awoke
from his nap. There is no reported history of trauma. The boy’s past medical history is significant
for an atrial septal defect that was repaired 9 months ago. He has an individualized education
program and receives speech therapy through the school district twice weekly. He is incontinent
of urine and stool. The boy lives with his mother and 2 older developmentally typical siblings.
His mother states that she is unemployed because she must care for the boy during the day. The
boy’s grandparents sometimes help with childcare. The family receives public assistance and has
Medicaid insurance. Further evaluation in the emergency department reveals multiple healed rib
fractures.

Of the following, the MOST significant risk factor for this boy’s injuries is
A. family need for public assistance
B. male sex
C. single-parent household
D. trisomy 21

American Academy of Pediatrics 195


PREP® Self-Assessment PREPSA 2023

Correct Answer: D
The boy in the vignette has injuries consistent with non-accidental trauma (NAT) or physical
abuse (unexplained femur fracture, multiple healed rib fractures). Having trisomy 21 is his most
significant risk factor for experiencing physical abuse. Children with developmental disabilities
and chronic medical conditions are at increased risk for abuse. One study (Paul) found that
children with disabilities are nearly 3 times more likely to be physically abused than children
without a disability. Children with mild cognitive disabilities and without motor disabilities were
found to be at the highest risk.

The risk of NAT is inversely related to the child’s age, with children younger than 2 years at the
highest risk. The incessant crying associated with colic is a risk factor for physical abuse,
specifically abusive head trauma (ie, shaken baby syndrome). Sex is not a risk factor for
experiencing NAT, although studies have suggested that fracturing injuries are more common in
males.

Family structure and economic stressors are important components of risk for child abuse.
However, a single-parent household is not, by itself, the relevant variable. Parental depression,
isolation from community and family, personal history of physical abuse as a child, substance
abuse, and domestic violence are strong risk factors. Multiple caretakers can either increase or
decrease the risk for abuse, depending on the support they provide. Low socioeconomic status
alone is not a risk factor for child abuse, however, combined with other environmental stressors,
it can increase risk.

Non-accidental trauma is a significant cause of injury and death for children in the United States.
In 2010, 2.07/100,000 children are known to have died from non-accidental injury. However, the
number of children who experience maltreatment is underreported. Self-reports by adolescents
and adults suggest that up to 40% of people have suffered some form of maltreatment during
childhood. Health care workers are mandated reporters and are required to report suspicion of all
forms of child abuse to child protective services.

PREP Pearls
• Non-accidental trauma is a significant cause of childhood injury and death; its incidence
is underreported.
• Children with developmental disabilities and chronic medical conditions are at increased
risk for physical abuse.
• Parental depression, isolation from community and family, a personal history of physical
abuse as a child, substance abuse, and domestic violence are strong risk factors for child
physical abuse.

ABP Content Specifications(s)


• Understand the common trigger events (eg, incessant crying) for physical abuse
• Understand the epidemiology of and the psychosocial and environmental risk factors for
physical abuse

American Academy of Pediatrics 196


PREP® Self-Assessment PREPSA 2023

Suggested Readings
• Austin AE, Lesak AM, Shanahan ME. Risk and protective factors for child maltreatment:
a review. Curr Epidemiol Rep. 2020;7(4):334-342. doi:10.1007/s40471-020-00252-3.
• Dubowitz H, Finkel MA. Physical abuse and neglect. In: McInerny TK, Adam HM,
Campbell DE, DeWitt TG, Foy JM, Kamat DM, eds. American Academy of Pediatrics
Textbook of Pediatric Care. American Academy of Pediatrics; 2021:chap 367. Accessed
September 1, 2022. Pediatric Care Online.
• Glick JC, Lorand MA, Bilka KR. Physical abuse of children. Pediatr Rev.
2016;37(4):146-158. doi:10.1542/pir.2015-0012.
• Jones L, Bellis MA, Wood S, et al. Prevalence and risk of violence against children with
disabilities: a systematic review and meta-analysis of observational studies. Lancet.
2012;380(9845):899-907. doi:10.1016/S0140-6736(12)60692-8.
• Legano LA, Desch LW, Messner SA, et al; Council on Child Abuse and Neglect, Council
on Children with Disabilities. Maltreatment of children with disabilities. Pediatrics.
2021;147(5):e2021050920. doi:10.1542/peds.2021-050920.
• Paul AR, Adamo MA. Non-accidental trauma in pediatric patients: a review of
epidemiology, pathophysiology, diagnosis and treatment. Transl Pediatr. 2014;3(3):195-
207. doi:10.3978/j.issn.2224-4336.2014.06.01.

American Academy of Pediatrics 197


PREP® Self-Assessment PREPSA 2023

Question 59
A 12-month-old, full-term male infant is being evaluated for gross motor delay. His mother first
became concerned when he was 7 months of age because he was not yet rolling over. At that
time laboratory testing demonstrated a serum creatine kinase level of 50 U/L and a thyroid-
stimulating hormone level of 1.1 mIU/L. Newborn screening results were re-verified as normal.
The infant was enrolled in physical therapy. He rolled (supine to prone and prone to supine) at
age 9 months and was able to sit with support at 10 months of age. At age 12 months, he cannot
sit independently. He reaches for toys with both hands, transfers objects, and is beginning to use
2 fingers in a pincer grasp to pick up cereal. He smiles, laughs, and babbles with multiple
consonants. He enjoys playing peek-a-boo and with toys that he causes to light up and play
music. There has been no developmental regression.

On physical examination, the infant’s vital signs are normal. His head circumference is 46 cm
(41st percentile for age). On neurological examination, he is alert and playful with age-
appropriate stranger anxiety. His cranial nerve findings are normal. He has truncal hypotonia
with head lag when pulled to sit, slippage at the shoulders on vertical suspension, and mild
appendicular hypotonia in all 4 extremities. His strength appears normal and symmetric. When
placed in a supported sit, he has slight titubation of the head. Deep tendon reflexes are 2+ in the
biceps, brachioradialis, and triceps, 3 at the patella bilaterally, and 2+ at the achilles bilaterally.
His toes are downgoing to plantar stimulation. He reaches for toys without dysmetria/ataxia. The
remainder of the infant’s physical examination findings are normal.

Of the following, the BEST next step in this infant’s evaluation is


A. brain magnetic resonance imaging
B. electromyography
C. nerve conduction studies
D. survival motor neuron 1 gene testing

American Academy of Pediatrics 198


PREP® Self-Assessment PREPSA 2023

Correct Answer: A
The infant in the vignette has an isolated gross motor delay in the setting of predominantly axial
hypotonia with preserved reflexes and strength. These examination findings are suggestive of a
central etiology for his hypotonia. Magnetic resonance imaging (MRI) of the brain would be an
appropriate next step in his diagnostic evaluation to evaluate for cerebral malformations or
neuroimaging markers of specific genetic and/or metabolic disorders. The other response choices
would be more appropriate when testing for an infant with findings suggestive of hypotonia
secondary to a peripheral etiology.

Tone, the resistance of the muscle to passive range of motion/stretch, is distinct from strength,
the amount of force a muscle group can generate. This distinction is important in the assessment
of an infant with gross motor delay, as the presence or absence of weakness can be a helpful
localization indicator. The differential diagnosis of hypotonia in infancy is broad, encompassing
the entirety of the nervous system. In this age group, however, central etiologies are more
common than peripheral.

The signs and symptoms accompanying hypotonia provide important clues for localization that
can be used to plan diagnostic evaluations (Item C59A). The evaluation of hypotonia begins
with a thorough history, with particular attention to birth/perinatal history, family history, and
developmental history (Item C59B). A complete physical examination with attention to head
circumference, dysmorphology, skin examination, and presence of organomegaly should be
performed. Neurological examination can provide key information for localization; close
attention should be paid to the pattern of tone abnormalities, strength examination, and reflexes.
Item C59C summarizes key examination findings for localization in the evaluation of hypotonia.
A video library of signs of motor weakness can be found at:
https://childmuscleweakness.org/video-library/.

Infants suspected of a central etiology of hypotonia should undergo neuroimaging, preferably


brain MRI, which can help guide further genetic and metabolic evaluations.

The initial diagnostic testing for infants with hypotonia suspected to have a peripheral etiology is
focused on timesensitive and treatable conditions. Evaluation usually includes a serum creatinine
kinase (CK) level, SMN1 gene deletion testing, and thyroid studies to evaluate for an underlying
myopathy, spinal muscular atrophy, or thyroid disease. If the results of initial laboratory testing
are negative, electromyography and nerve conduction studies should be performed for more
precise localization within the peripheral nervous system prior to proceeding to targeted disease
tests or muscle biopsy.

Management of hypotonia in infancy includes therapy services and targeted disease treatment,
when available, in order to optimize the child’s development.

American Academy of Pediatrics 199


PREP® Self-Assessment PREPSA 2023

PREP Pearls
• The signs and symptoms accompanying hypotonia provide important clues for accurate
localization, which can encompass any part of the nervous system.
• In infants, central etiologies of hypotonia are more common than peripheral.
• Tone, the resistance of the muscle to passive range of motion/stretch, is distinct from
strength, the amount of force a muscle group can generate. The presence or absence of
weakness can be a helpful localization indicator.
American Academy of Pediatrics 200
PREP® Self-Assessment PREPSA 2023

ABP Content Specifications(s)


• Differentiate the findings associated with central nervous system causes of hypotonia
from those of peripheral nervous system causes
• Plan the appropriate evaluation of hypotonia in patients of various ages

Suggested Readings
• Ahmed MI, Iqbal M, Hussain N. A structured approach to the assessment of a floppy
neonate. J Pediatr Neurosci. 2016;11(1):2-6. doi:10.4103/1817-1745.181250.
• Noritz G, Murphy N; Neuromotor Screening Expert Panel. Motor delays: early
identification and evaluation. Pediatrics. 2013;131(6):e2016-e2027.
doi:10.1542/peds.2013-1056.
• Peredo DE, Hannibal MC. The floppy infant: evaluation of hypotonia. Pediatr Rev.
2009;30(9):e66-e76. doi:10.1542/pir.30-9-e66.
• Spiro A. Hypotonia. In: McInerny TK, Adam HM, Campbell DE, DeWitt TG, Foy JM,
Kamat DM, eds. American Academy of Pediatrics Textbook of Pediatric Care. American
Academy of Pediatrics; 2021:chap 167. Accessed September 1, 2022. Pediatric Care
Online.

American Academy of Pediatrics 201


PREP® Self-Assessment PREPSA 2023

Question 60
A 14-year-old adolescent boy is seen for a preparticipation physical examination prior to his high
school soccer season. The boy’s medical history is significant for a seizure 2 years ago that
occurred during a middle school basketball game. His family history is remarkable for a sibling
who died of sudden infant death syndrome and his mother and maternal uncle who have type 1
diabetes mellitus. The adolescent’s physical examination findings are normal.

Of the following, the BEST next management step for this boy is
A. clearance for soccer participation
B. referral to a cardiologist
C. referral to a endocrinologist
D. referral to a neurologist

American Academy of Pediatrics 202


PREP® Self-Assessment PREPSA 2023

Correct Answer: B
The boy in the vignette had an apparent seizure while exercising. Myoclonic activity associated
with an arrhythmia can be mistaken for seizure activity. Seizures can also be associated with
dysrhythmia events and can be the initial presenting sign of an underlying channelopathy, such
as long QT syndrome. The boy had a sibling who died of sudden infant death syndrome, which
can also be the sentinel event for a child with a congenital arrhythmia syndrome. The boy’s
personal and family history are very concerning for a genetic heart condition, and he should be
evaluated by cardiology prior to clearing him for sports participation.

A child or adolescent with a well-controlled seizure disorder should be cleared for participation
in most sports without qualification; neurology referral is not indicated. Historically, children
with epilepsy were excluded from sports due to inaccurate beliefs that physical activity could
worsen seizures and that their risk of sports injury was increased. Children and adolescents with
epilepsy report lower rates of sports participation compared to their peers. Young athletes with a
history of seizures may be at increased risk of injury with sports that have an increased risk of
falls (eg, tumbling), equipment risks (eg, archery, weight lifting), and water sports. Clearance for
participation in these activities may need to be contingent on having additional safeguards in
place.

The boy in the vignette has a family history of type 1 diabetes mellitus (DM). Screening for type
1 DM is not recommended and there is no indication for endocrinology evaluation. The boy’s
family history of DM is not a contraindication to full sports clearance.

PREP Pearls
• A history of unexplained seizure activity could signify arrhythmogenic heart disease.
• A child or adolescent with a well-controlled seizure disorder should be cleared for
participation in most sports without qualification.

ABP Content Specifications(s)


• Identify factors that influence participation in contact sports by healthy children and
adolescents
• Understand the guidelines for sports participation for patients who have type 1 diabetes
• Understand the guidelines for sports participation for patients who have a seizure disorder

American Academy of Pediatrics 203


PREP® Self-Assessment PREPSA 2023

Suggested Readings
• Bernhardt DT, Roberts WO; American Academy of Family Physicians. PPE:
Preparticipation Physical Evaluation. 5th ed. American Academy of Pediatrics; 2019.
• Bernhardt DT. Sports preparticipation physical evaluation. In: McInerny TK, Adam HM,
Campbell DE, DeWitt TG, Foy JM, Kamat DM, eds. American Academy of Pediatrics
Textbook of Pediatric Care. American Academy of Pediatrics; 2021:chap 17. Accessed
September 1, 2022. Pediatric Care Online.
• Maron BJ, Levine BD, Washington RL, Baggish AL, Kovacs RJ, Maron MS. Eligibility
and disqualification recommendations for competitive athletes with cardiovascular
abnormalities: Task Force 2: preparticipation screening for cardiovascular disease in
competitive athletes: a scientific statement from the American Heart Association and
American College of Cardiology. Circulation. 2015;132(22):e267-272.
doi:10.1161/CIR.0000000000000238.
• Miller SM, Peterson AR. The sports preparticipation evaluation. Pediatr Rev.
2019;40(3):108-128. doi:10.1542/pir.20160216.
• Wong J, Wirrell E. Physical activity in children/teens with epilepsy compared with that in
their siblings without epilepsy. Epilepsia. 2006;47(3):631-639. doi:10.1111/j.1528-
1167.2006.00478.x.

American Academy of Pediatrics 204


PREP® Self-Assessment PREPSA 2023

Question 61
A 3-month-old infant is seen for a weight check. Her weight today is 5 kg, which is the same as
her weight at an appointment 2 weeks ago. She is scheduled for surgical repair of a large
ventricular septal defect at the end of the month and is on a high-dose diuretic regimen. She has
persistent comfortable tachypnea and is otherwise asymptomatic. Her diet consists of expressed
breast milk by bottle. Attempts at increasing the volume of feedings have failed secondary to
disinterest from the infant.

Of the following, the BEST next dietary intervention for this infant is to
A. add electrolyte solution to aid in hydration
B. change to a 30-kcal/oz formula
C. fortify the breast milk
D. limit feeding time to 20 minutes

American Academy of Pediatrics 205


PREP® Self-Assessment PREPSA 2023

Correct Answer: C
The infant in the vignette has a large ventricular septal defect with increased pulmonary blood
flow, leading to pulmonary edema and tachypnea. Her symptoms are managed with high-dose
diuretics to decrease the pulmonary edema. Ultimately, she needs to undergo surgical repair of
this lesion. Until then, the pediatrician and pediatric cardiologist can improve her surgical
candidacy by maintaining appropriate weight gain. Fortifying the expressed breast milk is the
best option to limit fluid intake while maximizing calories. Neither adding an electrolyte solution
for hydration nor limiting feeding time to minimize energy use will provide the needed
additional calories. Changing the infant’s diet to a 30-kcal/oz formula may result in diarrhea and
excessive stool losses.

Growth failure is common in children with congenital heart disease and can begin in utero. There
is an increased risk of malnutrition in children with unrepaired lesions, residual lesions after
surgical repair, single-ventricle anatomy, chronic hypoxemia, pulmonary hypertension, low
cardiac output, and feeding intolerance. Children with more complicated lesions often require
additional calories (a mean of 26 kcal/oz of formula or fortified breast milk) while needing to
limit their overall fluid intake. Many children with heart disease are treated with diuretics to
further minimize the effect of fluid on the strained heart. Children with congenital heart disease
have an increased incidence of oral aversion and feeding intolerance and may require nasogastric
tube or gastrostomy tube feedings to supplement their oral intake.

For chronic cardiac conditions, a multidisciplinary team that consists of a pediatrician,


cardiologist, dietician, and speech therapist may be needed to optimize children’s nutrition and
growth.

PREP Pearls
• Children with heart disease have increased caloric needs and often require limitation of
fluid intake.
• Increased caloric density formula or fortified breast milk is often needed for appropriate
weight gain in infants with
• heart disease.
• A multidisciplinary approach is needed to optimize nutrition and growth for many
children with heart disease.

ABP Content Specifications(s)


• Plan the dietary management of cardiac disease in a patient who is receiving a fluid-
restricted diet

American Academy of Pediatrics 206


PREP® Self-Assessment PREPSA 2023

Suggested Readings
• Kaufman J, Vichayavilas P, Rannie M, et al. Improved nutrition delivery and nutrition
status in critically ill children with heart disease. Pediatrics. 2015;135(3):e717-e725.
doi:10.1542/peds.2014-1835.
• Puri K, Allen H, Qureshi AM. Congenital heart disease. Pediatr Rev. 2017:38(10):471-
486. doi:10.1542/pir.2017-0032.
• Sevilla WMA. Nutritional considerations in pediatric chronic disease. Pediatr Rev.
2017;38(8):343-352. doi:10.1542/pir.2016-0030.

American Academy of Pediatrics 207


PREP® Self-Assessment PREPSA 2023

Question 62
A 15-year-old adolescent boy is evaluated in the office for right flank pain and bright red urine
with blood clots for 1 day. His pain is intermittent and colicky, and is radiating to the right lower
abdomen. He has no dysuria, urgency, or fever. The boy’s heart rate is 120 beats/min, respiratory
rate is 16 breaths/min, and blood pressure is 110/70 mm Hg. His weight is greater than the 95th
percentile, and his height is at the 50th percentile for age. He has right costovertebral angle
tenderness. The remainder of his physical examination findings are unremarkable.

A urinalysis with microscopy is shown:


Laboratory Test Result
Specific gravity 1.030
Leukocyte esterase Positive
Nitrite Negative
Blood 3+
Protein Negative
Red blood cells >100/HPF
White blood cells 10-20/HPF

Of the following, the BEST next step in confirming this adolescent’s suspected diagnosis is
A. abdominal computed tomography with contrast
B. abdominal magnetic resonance imaging without contrast
C. plain abdominal radiography
D. renal and bladder ultrasonography

American Academy of Pediatrics 208


PREP® Self-Assessment PREPSA 2023

Correct Answer: D
The adolescent boy in the vignette has symptoms consistent with renal colic, macroscopic
hematuria with blood clots, and right costovertebral tenderness, favoring a diagnosis of
nephrolithiasis. Renal ultrasonography is the firstline imaging modality recommended to detect
nephrolithiasis. Advantages of ultrasonography over the other options listed include the
avoidance of exposure to ionizing radiation and the potential to detect obstruction in the urinary
system. A noncontrast spiral computed tomography (CT) scan is the most sensitive test to
diagnose nephrolithiasis, especially in symptomatic patients, however, the significant radiation
exposure makes this a less optimal choice. A CT scan with contrast is not commonly
recommended as the contrast may decrease the sensitivity for stone detection and extend the time
required to perform the test. Renal calculi are often not detected on magnetic resonance imaging.
Abdominal radiography has a lower sensitivity and specificity than does ultrasonography and
detects only radio-opaque renal stones.

Urolithiasis is the presence of stones (calculi) in the kidney, bladder, or urethra. Nephrolithiasis
refers to the formation of stones in the kidney. Nephrocalcinosis is the deposition of calcium
salts in the renal parenchyma; it may be associated with discrete renal stones. The incidence of
nephrolithiasis is rising in the pediatric population. Although there is no clear evidence available,
some studies attribute this phenomenon to the increasing prevalence of obesity and dietary
changes.

Renal calculi are commonly composed of various combinations of calcium oxalate, calcium
phosphate, struvite, cysteine, and uric acid. Factors that increase the risk for stone formation
include decreased urine volume, increased solute excretion (calcium, uric acid), decreased levels
of stone inhibitors (citrate, magnesium), urinary tract infection, and renal structural abnormality
(eg, renal cyst). Children with nephrolithiasis typically have a urinary metabolic abnormality,
such as hypercalciuria, hyperoxaluria, hypocitraturia, hyperuricosuria, or cystinuria.

The presentation of nephrolithiasis varies with age. Adolescents generally have a typical
presentation with flank pain, renal colic, and macroscopic hematuria. Nausea, vomiting, dysuria,
and urgency may be present. Younger children may have nonspecific abdominal pain or be
asymptomatic. The history obtained should focus on recurrent urinary tract infection,
malabsorption symptoms, diet, medications, and family history of renal stones. Physical
examination findings suggestive of nephrolithiasis include poor growth, which may indicate a
metabolic disorder, and costovertebral angle tenderness.

Laboratory evaluation of suspected nephrolithiasis should include urinalysis with microscopy to


evaluate for hematuria, pyuria, and the presence of crystals; blood urea nitrogen, creatinine,
serum electrolyte, calcium, phosphorus, and uric acid levels; and a spot urine calcium-to-
creatinine ratio. A urine culture may be required.

The treatment of symptomatic nephrolithiasis includes hydration, pain control with opioids
and/or nonsteroidal antiinflammatory drugs, and antibiotics if associated with urinary tract
infection. Medical expulsion therapy with an αblocker (eg, tamsulosin), which relaxes the
American Academy of Pediatrics 209
PREP® Self-Assessment PREPSA 2023

ureteral smooth muscle, may facilitate the passage of distal ureteral stones (<5 mm). A urology
consultation is recommended in the case of an obstructed stone or large stone requiring
ureteroscopy or extracorporeal shock wave lithotripsy.

Children are at risk of recurrent nephrolithiasis. A renal stone, when retrieved, should be sent for
analysis regarding its composition. A 24-hour urine collection should be performed to help
identify any treatable urinary metabolic abnormalities. Long-term treatment includes increased
fluid intake to increase the urine volume. For children with hypercalciuria, reduced sodium
intake, moderate protein intake, the daily recommended allowance of calcium and vitamin D,
and a thiazide diuretic may be advised. Moderate oxalate restriction and avoidance of excess
vitamin C intake are required for the management of hyperoxaluria. Citrate supplementation may
be required in those with hypocitraturia to decrease the risk of stone formation.

PREP Pearls
• Factors that increase the risk for nephrolithiasis include decreased urine volume,
increased solute excretion (calcium, uric acid), and decreased levels of stone inhibitors
(citrate, magnesium).
• Renal and bladder ultrasonography is the first-line imaging modality recommended to
diagnose nephrolithiasis in children.
• A noncontrast spiral computed tomography scan is the most sensitive test to diagnose
nephrolithiasis in symptomatic children.

ABP Content Specifications(s)


• Plan the appropriate management of urinary tract stones
• Recognize factors contributing to the development of urinary tract stones

Suggested Readings
• Gellin CE. Urinary tract stones. Pediatr Rev. 2019;40(3):154-156. doi:10.1542/pir.2017-
0235.
• McKay CP. Renal stone disease. Pediatr Rev. 2010;31(5):179-188. doi:10.1542/pir.31-5-
179.
• Tayaba M, Kamat D. Pediatric nephrolithiasis: a review. Pediatr Ann. 2017;46(6):e242-
e244. doi:10.3928/1938235920170517-02.

American Academy of Pediatrics 210


PREP® Self-Assessment PREPSA 2023

Question 63
A 12-year-old girl is seen for a health supervision visit. Developmental milestones were delayed
for speech, and she received speech therapy from age 2 to 4 years. She is in a 7th-grade, regular
education classroom with an individualized education plan for reading and comprehension.
Menarche was achieved at age 11 years. Physical examination findings are unremarkable except
that she is a tall girl. Family history is noncontributory.

Of the following, the MOST likely diagnosis for this girl is


A. Klinefelter syndrome
B. Marfan syndrome
C. trisomy X
D. Turner syndrome

American Academy of Pediatrics 211


PREP® Self-Assessment PREPSA 2023

Correct Answer: C
Of the response choices, the most likely diagnosis for the girl in the vignette is trisomy X.
Trisomy X or triple X (47,XXX) is a sex chromosome aneuploidy with an incidence of
approximately 1 in 1,000 liveborn females. Clinical features are highly variable without clearly
defined malformations.

Common reported features include:


• Tall stature
• Developmental delay: Primarily speech delay; intellectual disability in the mild-to-
moderate range (average IQ of 55); learning disabilities in verbal and expressive
language
• Behavior: Conduct disorder, depression, anxiety, autism spectrum disorder, mood
disorder

Other reported features include:


• Neurologic: Seizure disorder, headaches, tremor
• Dysmorphic features: Midface hypoplasia, upward-slanting palpebral fissures, mild
hypertelorism Eye: Congenital cataracts, strabismus
• Ear-nose-throat: Cleft palate, dental disorders
• Cardiac: Congenital heart defects (atrial septal defect, ventricular septal defect)
• Gastrointestinal: Reflux, constipation, functional abdominal pain
• Genitourinary: Cystic dysplastic kidneys, solitary kidney, hydronephrosis, premature
ovarian insufficiency Musculoskeletal: Scoliosis, congenital hip dysplasia, radioulnar
synostosis, clubfoot

Triple X syndrome arises when a nondisjunction event results in an extra X chromosome in the
embryo. Approximately 60% occur during maternal meiosis I and are associated with advanced
maternal age. These are sporadic events and the chromosomal anomaly would not be expected to
be passed on to the next generation. The diagnosis is usually made either via noninvasive
prenatal screening or postnatally during evaluation for developmental delay or learning
disabilities. It is believed that trisomy X is a highly underdiagnosed condition because of its
phenotypic variability and lack of clear phenotypic features. Recommended evaluation upon
diagnosis includes developmental and behavioral assessment, renal ultrasonography, and 2-
dimensional echocardiography to look for structural defects, as well as education and counseling
about the syndrome.

Klinefelter syndrome (47,XXY) is a sex chromosome aneuploidy seen in males. Clinical features
include tall stature, hypogonadism, and infertility caused by aspermatogenesis. Androgen
deficiency is the hallmark of the disorder. Affected males can show delayed speech and motor
skills with learning disabilities.

Marfan syndrome is a connective tissue disorder. Affected individuals may have involvement of
several systems including:

American Academy of Pediatrics 212


PREP® Self-Assessment PREPSA 2023

• Cardiovascular: Aortic dilation with risk for tear and rupture, mitral valve prolapse and
regurgitation, aortic valve insufficiency, tricuspid valve prolapse, and enlargement of the
proximal pulmonary artery
• Skeletal: Reduced upper-to–lower body segment ratio, increased arm span–to-height
ratio, scoliosis, rib-cage asymmetry with pectus carinatum and/or excavatum, and joint
laxity
• Ocular: Progressive high myopia, and ectopia lentis associated with an increased risk for
glaucoma, retinal detachment, and early cataracts

Developmental delay and learning disabilities are not associated with Marfan syndrome.

Girls with Turner syndrome (45,X) may have learning difficulties in the visuospatial domain.
Features of Turner syndrome commonly include short stature, webbed neck, dysmorphic facial
features (down-slanted palpebral fissures, low-set ears, depressed nasal bridge), cardiac defects
(coarctation of aorta, bicuspid aortic valve), widespaced nipples, and pectus excavatum.

PREP Pearls
• Trisomy X or triple X (47,XXX) is a sex chromosome aneuploidy with highly variable
clinical features. The most common features include tall stature, speech delay, and
behavioral issues.
• Triple X syndrome arises when a nondisjunction event results in an extra chromosome X
in the embryo; approximately 60% occur during maternal meiosis I and are associated
with advanced maternal age.

ABP Content Specifications(s)


• Recognize the clinical features associated with a 47,XXX chromosome abnormality

Suggested Readings
• Jones KL, Jones M, del Campo M. Recognizable patterns of malformations. In: Smith’s
Recognizable Patterns of Human Malformation. 8th ed. Elsevier; 2021:68-69.
• Parisi MA. Genetics over the life cycle. In: AAP Committee on Genetics; Saul RA, ed.
Medical Genetics in Pediatric Practice. American Academy of Pediatrics; 2013:21-50.
• Tartaglia NR, Howell S, Sutherland A, Wilson R, Wilson L. A review of trisomy X
(47,XXX). Orphanet J Rare Dis. 2010;5:8. doi:10.1186/1750-1172-5-8 .
• Wigby K, D'Epagnier C, Howell S, et al. Expanding the phenotype of triple X syndrome:
a comparison of prenatal versus postnatal diagnosis. Am J Med Genet.
2016;170(11):2870-2881. doi:10.1002/ajmg.a.37688.

American Academy of Pediatrics 213


PREP® Self-Assessment PREPSA 2023

Question 64
A previously healthy, 8-year-old, fully immunized boy is seen in the office for malaise,
headache, a 2-day history of fever, and a 10-day history of worsening, nonproductive cough. One
week ago he was seen in the emergency department and diagnosed with a viral illness. The boy
has no sick contacts, no pets at home, and has not traveled recently.

The boy appears non-toxic. His temperature is 38.2°C, heart rate is 102 beats/min, respiratory
rate is 38 breaths/min, and oxygen saturation is 92% in room air. His blood pressure is normal.
He has pharyngeal erythema and crackles auscultated in both lung fields. The remainder of his
physical examination findings are unremarkable.

Of the following, the MOST likely pathogen causing this boy’s illness is
A. Legionella pneumophila
B. Mycoplasma pneumoniae
C. Staphylococcus aureus
D. Streptococcus pneumoniae

American Academy of Pediatrics 214


PREP® Self-Assessment PREPSA 2023

Correct Answer: B
The clinical presentation of the well-appearing boy in the vignette with a gradual onset of illness
characterized by prolonged cough, low-grade fever, malaise, pharyngitis, and bilateral crackles
on lung examination is suggestive of atypical pneumonia caused by Mycoplasma pneumoniae.

It can be difficult to reliably distinguish bacterial etiologies of community-acquired pneumonia


(CAP) from viral etiologies based on clinical findings alone. However, CAP caused by
Staphylococcus aureus and Streptococcus pneumoniae is associated with high fever, chills,
marked tachypnea, and focal auscultatory findings, which is not consistent with the clinical
findings for the boy in the vignette. Atypical infection due to Legionella pneumophila is rare in
children, accounting for ≤1% of all pneumonia cases, making this diagnosis unlikely for the boy
in the vignette.

Mycoplasma pneumoniae is a pleomorphic bacteria without a cell wall; it is a frequent cause of


upper respiratory tract infection and CAP in school-aged children, adolescents, and young adults.
Mycoplasma pneumoniae is responsible for an estimated 2 million annual infections in the
United States and 10% to 20% of hospital admissions for CAP. Viral pathogens cause the
majority of CAP in preschool-aged children; infection with M pneumoniae is uncommon in this
age group.

Mycoplasma pneumoniae infection is acquired year-round via respiratory droplets during contact
with symptomatic individuals. Pneumonia may develop in up to 30% of household contacts.
Outbreaks are common in health care settings, colleges, camps, and military bases. Following
infection, prolonged (weeks to months) asymptomatic carriage of the organism in the respiratory
tract may occur. Immunity after infection is not lifelong.

The incubation period of M pneumoniae ranges from 1 to 4 weeks (typically 2 to 3 weeks). The
upper respiratory tract is primarily affected. Pneumonia develops in up to 25% of school-aged
children infected with M pneumoniae. The illness is characterized by a gradual onset. Cough
(present in 90% to 100% of cases) can persist up to 4 weeks. Other symptoms include fever,
malaise, headache, and sore throat. Non-exudative pharyngitis is common and may be associated
with cervical adenopathy. Acute otitis media, with or without bullous myringitis, is unusual. On
lung auscultation, crackles and wheezes may be heard.

M pneumoniae can cause severe, complicated pneumonia in children with sickle cell disease,
trisomy 21, immunodeficiency disorders, and chronic cardiac or pulmonary disease. Acute chest
syndrome in children with sickle cell disease and asthma exacerbations can be triggered by M
pneumonia infection.

Extrapulmonary manifestations of M pneumoniae infection, affecting the dermatological,


hematologic, neurologic, or cardiac systems, are rare. Ten percent of infected children develop
an erythematous maculopapular rash. Unusual and severe dermatologic manifestations of M
pneumoniae include immune-mediated disorders such as erythema multiforme and Stevens-
Johnson syndrome. In contrast, M pneumoniae–induced rash and mucositis (MIRM) is a benign
American Academy of Pediatrics 215
PREP® Self-Assessment PREPSA 2023

disease characterized by involvement of more than 2 mucosal sites (with or without the skin).
Hematological manifestations include hemolytic anemia, thrombocytopenic purpura, and
hemophagocytic syndrome. Central nervous system disease may include acute encephalitis, acute
disseminated encephalomyelitis, cerebellar ataxia, transverse myelitis, and peripheral
neuropathy. Cardiac manifestations include myocarditis and pericarditis. Arthritis has been
reported in immunocompromised hosts.

The laboratory diagnosis of M pneumoniae infection can be challenging, especially in


ambulatory settings, and is not routinely recommended. In hospitalized patients with suspected
M pneumoniae infection, laboratory testing of respiratory tract specimens by nucleic acid
amplification tests (NAAT) is preferred. The sensitivity and specificity of NAAT range from
80% to 100%. However, NAAT are only helpful in the evaluation of patients with active
symptoms of classic clinical disease as M pneumoniae can colonize the respiratory tract for
prolonged periods after infection (resulting in a positive test from a prior infection). Serologic
diagnosis using immunofluorescence and enzyme immunoassays that detect IgM and IgG is less
useful due to low sensitivity compared to NAAT and low specificity of IgM results. Detection of
a fourfold or greater increase in IgG titers in serum obtained during acute and convalescent
periods (4 weeks apart) can establish the diagnosis. Chest radiograph findings in Mycoplasma
pneumonia are nonspecific (eg, bilateral interstitial infiltrates, focal consolidation, pleural
effusion).

Mycoplasma pneumoniae infection is mild and self-limited in the majority of children. There is
no evidence that antimicrobial therapy alters the disease course in upper respiratory tract
infection due to M pneumoniae. In ambulatory settings, routine administration of antimicrobial
therapy for school-aged children for respiratory tract infection due to suspected mycoplasma is
not indicated. In hospitalized children with moderate to severe M pneumoniae pneumonia or
other severe manifestation (neurologic disease, infection in compromised hosts), antimicrobial
therapy with a macrolide (eg, azithromycin, clarithromycin) may be indicated. The increasing
incidence of macrolide-resistant M pneumoniae strains is a concern.

PREP Pearls
• Mycoplasma pneumoniae is a frequent cause of upper respiratory tract infection and
community-acquired pneumonia in school-aged children, adolescents, and young adults.
• In ambulatory settings, routine administration of antimicrobial therapy for school-aged
children for respiratory tract infection due to suspected Mycoplasma is not indicated.
• In children with sickle cell disease, trisomy 21, immunodeficiency disorders, and chronic
cardiac or pulmonary disease, Mycoplasma pneumoniae can cause severe, complicated
pneumonia.
• In hospitalized children with moderate to severe Mycoplasma pneumoniae pneumonia or
other severe manifestations, antimicrobial therapy with a macrolide (eg, azithromycin,
clarithromycin) may be indicated.

American Academy of Pediatrics 216


PREP® Self-Assessment PREPSA 2023

ABP Content Specifications(s)


• Understand the epidemiology of Mycoplasma pneumoniae
• Recognize the clinical features associated with Mycoplasma pneumoniae infection

Suggested Readings
• American Academy of Pediatrics. Mycoplasma pneumoniae and other Mycoplasma
species infections. In: Kimberlin DW, Barnett ED, Lynfield R, Sawyer MH, eds. Red
Book: 2021–2024 Report of the Committee on Infectious Diseases. 32nd ed. American
Academy of Pediatrics; 2021. Accessed September 1, 2022. Red Book Online.
• Gereige RS, Laufer PM. Pneumonia. Pediatr Rev. 2013;34(10):438-456; quiz 455-456.
doi:10.1542/pir.34-10-438.
• Krafft C, Christy C. Mycoplasma pneumonia in children and adolescents. Pediatr Rev.
2020;41(1):12-19. doi:10.1542/pir.2018-0016.
• Light M. Pneumonia. In: McInerny TK, Adam HM, Campbell DE, DeWitt TG, Foy JM,
Kamat DM, eds. American Academy of Pediatrics Textbook of Pediatric Care. American
Academy of Pediatrics; 2021:chap 315. Accessed September 1, 2022. Pediatric Care
Online.

American Academy of Pediatrics 217


PREP® Self-Assessment PREPSA 2023

Question 65
A 12-month-old girl is seen for a health supervision visit. She has been thriving, and her parents
do not have any concerns. On physical examination, there is a white pupillary reflex in the right
eye. The remainder of her physical examination findings are normal.

Of the following, the BEST next step in this child’s management would be referral to a
A. geneticist
B. pediatric oncologist
C. pediatric ophthalmologist
D. radiation oncologist

American Academy of Pediatrics 218


PREP® Self-Assessment PREPSA 2023

Correct Answer: C
The child in the vignette has a white pupillary reflex, known as leukocoria (Item C65A).
Leukocoria is a concerning finding, suggestive of retinoblastoma (RB; an abnormal growth of
retinoblasts), and must be confirmed by an ophthalmologist. Though rare, RB is the most
common intraocular pediatric malignancy. It occurs in young children, typically younger than 5
years. Leukocoria is present at the time of diagnosis in 50% to 60% of cases of RB. The chance
of survival in children with early diagnosis and treatment of local disease is greater than 90%;
late diagnosis of RB may lead to loss of vision and a very poor prognosis. It is therefore critically
important that any child with leukocoria undergo prompt examination by a pediatric
ophthalmologist. In most cases, examination with the patient under anesthesia is required to
allow for thorough examination and to acquire necessary images. Retinoblastoma may also
present with new-onset strabismus. As it progresses, RB may also lead to decreased visual
acuity, inflammation, hyphema, and vitreous hemorrhage (a black pupil on eye examination).

Retinoblastoma is one of the first cancers explained by Alfred Knudson’s 2-hit hypothesis. In RB
there is an initial germline or somatic mutation in the RB1 gene (a tumor suppressor) located on
chromosome 13, followed by a second somatic mutation in retinoblast cells, leading to tumor
development. Mutations in RB1 can lead to additional malignancies, most commonly
osteosarcoma, followed by soft tissue sarcomas and melanoma. Radiation therapy for RB
increases the risk of secondary malignancies. Individuals with 13q deletion syndrome
(microcephaly, high forehead, short nose, hypertelorism, micrognathia, and developmental
delay) are at increased risk of developing RB due to the location of the RB1 gene in the deleted
segment. There is a highly aggressive type of RB that occurs in very young infants (mean age,
4.5 months), which is associated with an MYCN oncogene mutation instead of RB1.

Retinoblastoma may be inherited or sporadic; unilateral or bilateral; unifocal or multifocal. Item


C65B compares heritable and non-heritable forms of RB.

Hereditable RB
Eighty-five percent of heritable RB cases are due to new spontaneous mutations (sporadic), while the
remaining 15% have a positive family history (familial). For the sporadic cases the paternal allele is
affected in approximately 90% of cases. Post conception, a second somatic mutation occurs in the
remaining RB1 gene.
There are a small number of patients with heritable RB where RB1 mutations cannot be found. In some of
these, MYCN amplification is identified in the tumor. Such tumors exhibit aggressive histologic features
with a median age at diagnosis of 4 months. Mosaicism is a postzygotic mutation found only in a portion
of cells. It is often difficult to diagnose and is heritable only if the reproductive germ cells are affected.

American Academy of Pediatrics 219


PREP® Self-Assessment PREPSA 2023

Nonhereditable RB
These patients inherit two normal copies of the RB1 gene. Post conception two somatic mutations occur,
one in each copy of the RB1 gene in a retinoblast with resulting tumor formation. These cases have
unifocal tumors.

Advances in the early diagnosis and treatment of RB have significantly improved the prognosis
in high-income countries. However, in low- and middle-income countries, advanced RB is
prevalent, and the prognosis is graver due to delayed diagnosis and limited treatment resources.
The International Classification of Retinoblastoma (ICRB) places tumors in categories A through
E, based on the location and extent of the tumor. Tumors of groups A and B are confined to the
retina, group C has local spread, group D has diffuse spread, and in group E the eye has the
lowest chance of being spared.

Treatment of RB consists of a combination of chemotherapy (systemic, intravitreal, or intra-


arterial), radiation therapy, and enucleation. The main goals of treatment are to prevent mortality,
preserve vision, and remove the tumor while avoiding damage to other structures. Efforts are
made to prevent secondary malignancies, such as by avoiding radiation therapy in individuals
with heritable RB.

The diagnostic evaluation for children with RB should include the following:
• Hearing evaluation (if abnormal newborn hearing screen or family history of hearing
loss)
• Imaging studies (magnetic resonance imaging of the brain and orbits, bone scan)
• Complete blood cell count (CBC; to assess for cytopenias indicating bone marrow
involvement)
• Comprehensive metabolic panel (assessment before initiating chemotherapy)
• Lumbar puncture (if there is concern for central nervous system disease)
• Bone marrow biopsy (if the CBC is abnormal)

All children should have screening for a bilateral red reflex at every health supervision visit. The
American Academy of Pediatrics guidelines recommend screening from the newborn to 6
months of age, 6 to 12 months of age, 1 to 3 years of age, 4 to 5 years of age, and 6 years and
older visits. If there is a family history of RB, genetic testing and a comprehensive
ophthalmologic examination must be performed; genetic testing can be obtained prenatally. The
results of genetic testing and which family member had RB are used together to determine the
required screening, which occurs up to approximately 7 years of age.
The geneticist, radiation oncologist, and pediatric oncologist play a role in caring for a child with
RB. However, the specialist who must evaluate the child first and make the diagnosis of RB is
the pediatric ophthalmologist.

American Academy of Pediatrics 220


PREP® Self-Assessment PREPSA 2023

PREP Pearls
• Leukocoria is the presenting finding in 50% to 60% of cases of retinoblastoma, and new-
onset strabismus may be the presenting finding of retinoblastoma.
• A pediatric ophthalmologist must promptly and thoroughly examine a child with
leukocoria to evaluate for retinoblastoma.
• All children should have screening for a bilateral red reflex at every health supervision
visit. The American Academy of Pediatrics guidelines recommend screening from the
newborn to 6 months of age, 6 to 12 months of age, 1 to 3 years of age, 4 to 5 years of
age, and 6 years and older visits.

ABP Content Specifications(s)


• Differentiate the historical and clinical findings associated with hereditary retinoblastoma
from those of sporadic retinoblastoma
• Recognize the inheritance pattern associated with retinoblastoma and the significance of
the family history in planning management
• Recognize the clinical and laboratory findings associated with retinoblastoma

Suggested Readings
• Donahue S, Nixon C; Section on Ophthalmology, American Academy of Pediatrics;
Committee on Practice and Ambulatory Medicine, American Academy of Pediatrics;
American Academy of Ophthalmology; American Association for Pediatric
Ophthalmology and Strabismus; American Association of Certified Orthoptists. Visual
system assessment in infants, children, and young adults by pediatricians. Pediatrics.
2016;137(1):e20153596. doi:10.1542/peds.2015-3596.
• Leahey A. Retinoblastoma. Lanzkowsky P, Lipton J, Fish JD, eds. Lanzkowsky’s Manual
of Pediatric Hematology and Oncology. 6th ed. Elsevier Inc; 2016:544-554.
• Manrique M, Akinbolue D, Madigan WP, Bregman J. Update on the treatment of
retinoblastoma. Neoreviews. 2021;22(7):e423-e437. doi:10.1542/neo.22-7-e423.
• Wilson WG. Retinoblastoma. Pediatr Rev. 2007;28(1):37-38. doi:10.1542/pir.28-1-37.

American Academy of Pediatrics 221


PREP® Self-Assessment PREPSA 2023

Question 66
A 16-year-old adolescent boy is brought to the emergency department by ambulance after he was
involved in a motor vehicle collision. He was the unrestrained driver of a vehicle that hit a tree
traveling approximately 40 mph. He complains of nasal and right-sided facial pain. He did not
lose consciousness during the accident and was able to selfextricate from the vehicle. He denies
neck or back pain. Physical examination reveals swelling and ecchymosis of his right cheek and
nasal bridge, and a small amount of blood in his nares. His pupils are equal and reactive to light,
and his extraocular movements are intact bilaterally. His dentition is intact, and there is no
bleeding from his mouth. Head computed tomography scan does not demonstrate any skull
fractures or intracranial hemorrhage. Computed tomography scan of his face shows displaced,
comminuted fractures of his right zygomatic arch and ethmoid bones. Of the following, the
BEST next step in this boy’s management is

A. intravenous ampicillin/sulbactam
B. magnetic resonance imaging with contrast
C. oral analgesia, cold compresses, and nasal decongestant
D. urgent evaluation by an otolaryngologist

American Academy of Pediatrics 222


PREP® Self-Assessment PREPSA 2023

Correct Answers: D
The boy in the vignette has displaced, comminuted facial fractures and requires urgent evaluation
by a pediatric otolaryngologist.

The management of paranasal sinus trauma and facial bone fractures is dependent upon the
complexity, location, and displacement of the fracture(s). Displaced, complex, or injuries with
high risk of complication may require surgical repair. For isolated, non-displaced, or minimally
displaced facial fractures, conservative management with cold compresses, oral analgesia, and
nasal decongestants is appropriate.

Routine antimicrobial prophylaxis has not been demonstrated to be beneficial in the treatment of
paranasal sinus trauma; therefore, antibiotic therapy with ampicillin/sulbactam is not the best
next management step for the boy in the vignette.

Maxillofacial computed tomography is the gold standard imaging modality for facial trauma.
This study evaluates the facial bones including the orbits, sinuses, mandible, and teeth.
Computed tomography with contrast can be used to evaluate for fracture and vascular injury;
therefore, evaluation with magnetic resonance imagining is not routinely used in the evaluation
of facial trauma. Radiography of the facial bones (eg, Waters projection and Caldwell projection)
is no longer routinely used to evaluate for facial fractures.

Facial fractures may occur in isolation or as fractures of multiple facial bones. Complications of
facial bone fractures can include nerve injury, intracranial infection, cerebral spinal fluid leak,
olfactory nerve damage, sinus infection, and facial deformities.

PREP Pearls
• Comminuted or displaced facial fractures require urgent evaluation by an
otolaryngologist.
• Maxillofacial computed tomography (facial bones including orbits, sinuses, mandible,
teeth) is the gold standard imaging modality for facial trauma.
• For isolated, non-displaced, or minimally displaced facial fractures, conservative
management with cold compresses, oral analgesia, and nasal decongestants is
appropriate.

ABP Content Specifications(s)


• Plan the appropriate management of trauma to any of the paranasal sinuses

American Academy of Pediatrics 223


PREP® Self-Assessment PREPSA 2023

Suggested Readings
• Cole P, Kaufman Y, Hollier LH Jr. Managing the pediatric facial fracture.
Craniomaxillofac Trauma Reconstr. 2009;2(2):77-83. doi:10.1055/s-0029-1202592.
• Fein DM, Fagan MJ. Overall approach to trauma in the emergency department. Pediatr
Rev. 2018;39(10):479-489. doi:10.1542/pir.2017-0246.
• Gussack GS, Luterman A, Powell RW, Rodgers K, Ramenofsky ML. Pediatric
maxillofacial trauma: unique features in diagnosis and treatment. Laryngoscope.
1987;97(8 Pt 1):925-930. https://pubmed.ncbi.nlm.nih.gov/3613792/.
• Haug RH, Foss J. Maxillofacial injuries in the pediatric patient. Oral Surg Oral Med Oral
Pathol Oral Radiol Endod. 2000;90(2):126-134. doi:10.1067/moe.2000.107974.
• Kühnel TS, Reichert TE. Trauma of the midface. GMS Curr Top Otorhinolaryngol Head
Neck Surg. 2015;14:Doc06. doi:10.3205/cto000121.

American Academy of Pediatrics 224


PREP® Self-Assessment PREPSA 2023

Question 67
A 2-day-old, 38 weeks’ gestation male neonate is seen in the emergency department. This
morning, while changing his diaper, his mother noticed blood mixed in the stool. The infant has
been fussy for a few hours. He was delivered vaginally during a planned home delivery attended
by a neighbor who has experience in performing home births but is not a qualified midwife. The
neonate did not receive any medications at the time of or after delivery. He is exclusively
breastfed and has been feeding well. The infant’s heart rate is 156 beats/min and blood pressure
is 88/35 mm Hg (mean = 43 mm Hg). The infant is awake and fussy. He appears pink and well
perfused, with scattered petechiae over the abdomen and blood oozing from the umbilical cord.
His abdomen is soft and nontender. Laboratory tests are ordered, and a peripheral intravenous
line is placed.

Of the following, the BEST next step in this neonate’s management is to


A. administer ampicillin and gentamicin, intravenously
B. administer vitamin K, intravenously
C. infuse fresh frozen plasma
D. infuse platelets

American Academy of Pediatrics 225


PREP® Self-Assessment PREPSA 2023

Correct Answers: B
The neonate in the vignette most likely has hemorrhagic disease of the newborn, and it is evident
from the perinatal history that he did not receive 1 mg intramuscular vitamin K after delivery. To
prevent further bleeding, it is important to give this patient vitamin K as soon as possible. If no
intravenous line is available, intramuscular injection is an appropriate alternative. Coagulation
factor replacement and blood product administration, if needed, can occur later.

Vitamin K is an essential cofactor for carboxylation of clotting factors II, VII, IX, and X.
Newborns are at risk of developing vitamin K deficiency because they have low vitamin K stores
and a sterile intestinal tract at birth. Clinical presentation includes bruising, petechiae, melena,
intracranial bleeding, and bleeding from other areas such as the umbilicus, circumcision site,
mouth, and scalp. There are 3 forms of hemorrhagic disease of the newborn due to vitamin K
deficiency: classic, early-onset, and late-onset. The classic form, as seen in the neonate in the
vignette, is the most common type. This type occurs in exclusively breastfed infants who did not
receive vitamin K prophylaxis at birth. Breast milk has a low level of vitamin K compared to
formula. Additionally, intestinal bacteria produce vitamin K, and in exclusively breastfed infants,
bacterial colonization of the intestinal tract takes longer to occur. This typically occurs between
the second to seventh day after birth. The early-onset form presents in the first 24 hours after
birth and is most often due to maternal medications (eg, phenobarbital, phenytoin, rifampicin,
isoniazid) that interfere with in-utero vitamin K storage. The late-onset form usually presents at 2
to 12 weeks after birth and may be seen as late as 6 months of age. Infants at risk for late-onset
hemorrhagic disease of the newborn are those with cholestatic liver dysfunction or chronic
intestinal malabsorption. The earliest coagulation abnormality seen in hemorrhagic disease of the
newborn is a prolonged prothrombin time; eventually, if untreated, the partial thromboplastin
time will become prolonged.

The treatment for hemorrhagic disease of the newborn is 1 mg of intravenous or intramuscular


vitamin K. There is no indication for antibiotics in the management of hemorrhagic disease of
the newborn.

PREP Pearls
• Hemorrhagic disease of the newborn due to vitamin K deficiency can be easily prevented
with intramuscular injection of 1 mg of vitamin K after birth.
• Vitamin K deficiency should be strongly considered for a newborn presenting with any
form of bleeding, especially those exclusively breastfeeding, those delivered at home, and
those who did not receive vitamin K prophylaxis after birth.

ABP Content Specifications(s)


• Plan appropriate vitamin K administration for a newborn infant
• Recognize the presenting signs and symptoms of hemorrhagic disease of the newborn in
an infant who did not receive vitamin K, and manage appropriately

American Academy of Pediatrics 226


PREP® Self-Assessment PREPSA 2023

Suggested Readings
• American Academy of Pediatrics Committee on Fetus and Newborn. Controversies
concerning vitamin K and the newborn. Pediatrics. 2003;112(1 pt 1):191-192. PMID:
12837888.
• Araki S, Shirahata A. Vitamin K deficiency bleeding in Infancy. Nutrients.
2020;12(3):780. doi:10.3390/nu12030780.
• Hand I, Noble L, Abrams SA; Committee on Fetus and Newborn, Section on
Breastfeeding, Committee on Nutrition. Vitamin K and the newborn infant. Pediatrics.
2022;149(3):e2021056036. doi:10.1542/peds.2021-056036.
• Phillippi JC, Holley SL, Morad A, Collins MR. Prevention of vitamin K deficiency
bleeding. J Midwifery Womens Health. 2016;61(5):632-636. doi:10.1111/jmwh.12470.
• Shearer MJ. Vitamin K deficiency bleeding (VKDB) in early infancy. Blood Rev.
2009;23(2):49-59. doi:10.1016/j.blre.2008.06.001.

American Academy of Pediatrics 227


PREP® Self-Assessment PREPSA 2023

Question 68
A 13-year-old adolescent boy with newly diagnosed type 1 diabetes is seen for follow-up. He is
doing well with his insulin regimen of multiple daily subcutaneous injections. He takes 12 units
of long-acting basal insulin analog once daily at bedtime and boluses of a rapid-acting insulin
analog before meals, dosed based on carbohydrate counts (1 unit for every 15 g of carbohydrate)
and blood glucose measurements (1 unit for every 50 mg/dL over 150 mg/dL). The boy’s
physical examination findings are unremarkable.

His blood glucose log is shown (glucose levels are in mg/dL):


Breakfast Lunch Dinner Bedtime
Day 1 253 118 142 117
Day 2 322 165 133 154
Day 3 196 122 102 125
Day 4 217 131 95 111
Day 5 284 110 118 132

Of the following, the BEST next step in this boy’s management is to


A. ask the boy to check his blood glucose at 3 AM
B. ask the boy to check for urine ketones each morning
C. increase the basal insulin dose to 13 units
D. split the basal insulin dose into 6 units twice daily

American Academy of Pediatrics 228


PREP® Self-Assessment PREPSA 2023

Correct Answer: A
The boy in the vignette has newly diagnosed type 1 diabetes, and his blood glucose log
demonstrates that he is experiencing elevated blood glucoses in the morning. This pattern is most
commonly due to too little (dawn effect) or too much (Somogyi effect) bedtime basal insulin. A
blood glucose level checked in the middle of the night (usually between 2-3 AM) can help
distinguish between these two effects. An elevated overnight glucose level indicates too little
basal insulin, and a low level indicates too much basal insulin.

The dawn effect is due to the body’s circadian rhythm. As a child or adolescent is preparing to
rise in the morning, regulatory hormones such as cortisol and growth hormone start to rise. In
children and adolescents without diabetes, insulin works in concert with these regulatory
hormones resulting in a rise in glucose level without frank hyperglycemia. However, in a child or
adolescent with diabetes who has too little circulating basal insulin, the glucose level rises into
the hyperglycemic range.

The Somogyi effect is due to rebound hyperglycemia after an episode of hypoglycemia. When
too much circulating basal insulin results in low blood glucose levels overnight, counter-
regulatory hormones such as epinephrine, cortisol, and growth hormone are secreted. This
response can result in elevated blood glucose levels in the morning. The solution is to decrease
the basal insulin dose to prevent episodes of hypoglycemia. This phenomenon is rare; most
children and adolescents with overnight hypoglycemia due to excess basal insulin remain
hypoglycemic.

Most long-acting basal insulin analogs work for at least 24 hours. In rare situations, the basal
insulin analog wears off sooner, and splitting the basal insulin dose can help stabilize the glucose
levels. The glucose log for the boy in the vignette is not consistent with the basal insulin analog
wearing off too soon. The boy takes his long-acting basal insulin analog at bedtime, so high
glucose levels in the evenings, not mornings, could be a sign that it is wearing off too soon.
Although the boy in the vignette may need an increased basal insulin dose, he should first check
a middle-of-thenight blood glucose level to ensure he is not experiencing hypoglycemia and the
Somogyi effect.

When children and adolescents with type 1 diabetes have hyperglycemia, they are usually
advised to check for urine or blood ketones as hyperglycemia can signify impending diabetic
ketoacidosis. However, the boy in the vignette is asymptomatic and is not experiencing sustained
hyperglycemia. Although the boy should check for ketones, this approach will not help
determine why his morning blood glucose levels are high.

American Academy of Pediatrics 229


PREP® Self-Assessment PREPSA 2023

PREP Pearls
• In type 1 diabetes, elevated morning blood glucose levels may be due to either too little
(dawn effect) or too much basal insulin (Somogyi effect).
• Checking a blood glucose level in the middle of the night (2-3 AM) can help distinguish
between the dawn and Somogyi effects.
• Understanding the pharmacodynamics of insulin analogs is essential for making effective
insulin dose adjustments.

MOCA-Peds Objective
• Evaluate and manage a child with hypoglycemia.

ABP Content Specifications(s)


• Plan the appropriate management of type 1 diabetes to effectively achieve good control
and to avoid long-term complications
• Counsel patients regarding self-management of type 1 diabetes

Suggested Readings
• Jackson S, Creo A, Al Nofal A. Management of type 1 diabetes in children in the
outpatient setting. Pediatr Rev. 2022;43(3):160-170. doi:10.1542/pir.2020-001388.
• Javed A, Schwenk WF, II, Tebben P. Diabetes mellitus. In: McInerny TK, Adam HM,
Campbell DE, DeWitt TG, Foy JM, Kamat DM, eds. American Academy of Pediatrics
Textbook of Pediatric Care. American Academy of Pediatrics; 2021:chap 241. Accessed
September 1, 2022. Pediatric Care Online.
• Kliegman RM, St Geme JW, Blum NJ, Shah SS, Tasker RC, Wilson KM. Diabetes
mellitus. In: Nelson Textbook of Pediatrics. 21st ed. Elsevier; 2020:3019-3052.

American Academy of Pediatrics 230


PREP® Self-Assessment PREPSA 2023

Question 69
A faculty member is reviewing a journal article with a group of medical students. The article
describes a randomized controlled trial comparing 2 warm-up exercise programs designed to
prevent knee injuries in high school soccer athletes. The researchers found that 5% of the athletes
who participated in a strengthening warm-up program sustained knee injuries during their soccer
season compared to 15% of control group participants who used a static stretching warm-up
program. The researchers concluded that for every 10 athletes who participated in the
strengthening program, there was 1 less injury.

Of the following, the concept BEST expressed by the researchers’ conclusion is


A. number needed to treat
B. odds ratio
C. relative risk
D. significance level

American Academy of Pediatrics 231


PREP® Self-Assessment PREPSA 2023

Correct Answer: A
The authors of the journal article in the vignette describe a reduction of 1 injury for every 10
athletes who participate in the strength training program. This expression of the intervention
outcome is known as the number needed to treat (NNT), the number of individuals that must be
treated for one to have the outcome of interest. The NNT is calculated by dividing 100 by the
difference in effect size between the two groups. In this case, 100/(15-5) = 10.

A corollary to the NNT is the number needed to harm (NNH). The NNH is useful for describing
the number of individuals that would need to be treated in order for one to have an adverse
outcome (eg, an adverse effect occurring in individuals taking a study medication as compared to
a placebo). The advantage of using NNT and NNH as outcome measures is their intuitive way of
expressing effect size. The NNT and NNH calculations describe a dichotomous outcome, in this
case injury or no injury. However, the NNT and NNH do not express the clinical importance of
an outcome or relate the outcome to other variables; these factors that must be considered. For
example, a medication designed to prevent sepsis with an NNT of 100 may appear beneficial
based on this calculation. However, if the drug has a high risk of serious adverse effects, an
alternate drug would be a better choice. Clinicians must consider the impact of additional
relevant factors when using NNT and NNH as outcome measures.

The relative risk compares the probability of an outcome between 2 groups. In the vignette, 5%
of the athletes in the strengthening group sustained an injury, while 15% of athletes in the
stretching group sustained an injury. The relative risk (RR) of injury for the stretching group is
15 divided by 5, or 3.

The odds ratio is another method of examining the relationship between interventions and binary
outcomes. The odds ratio compares the odds of each of 2 possible outcomes in the control and
intervention groups.

In the strengthening group, 95% of the athletes were free of injury (good outcome). The odds of
a good outcome in this group is the probability of no injury (0.95) divided by the probability of
injury (0.05) [0.95 ÷ 0.05 = 19]; so, the odds are 19:1.

In the static group, 85% of the athletes were free of injury. The odds of a good outcome for this
group is 0.85 ÷ 0.15 = 0.57; so the odds are 5.7:1.
The odds ratio is the ratio of the odds in each of these groups, the odds of a good outcome in the
strengthening group: static group = 19:5.7 = 3.3.

In statistics, the null hypothesis (H0) is the hypothesis that any observed difference in outcomes
between groups is due to chance. A type I error refers to rejecting the null hypothesis when it is
correct. The significance level (α) of a statistical test is the chance of making a type I error. In
medical studies, the significance level is typically set at 0.05, which means that the investigators
will consider any differences in outcomes to be related to the intervention(s) unless the chance of
a type I error is 0.05 or greater.

American Academy of Pediatrics 232


PREP® Self-Assessment PREPSA 2023

PREP Pearls
• The number needed to treat calculation is an intuitive method of describing a
dichotomous outcome; it defines the number of individuals that need to be treated to see
the outcome of interest.
• The number needed to treat does not express the clinical importance of an outcome or
relate the outcome to other variables.

ABP Content Specifications(s)


• Understand the concept of number-needed-to-treat when utilized to describe therapeutic
interventions

Suggested Readings
• Norton EC, Dowd BE, Maciejewski ML. Odds ratios-current best practice and use.
JAMA. 2018;320(1):84-85. doi:10.1001/jama.2018.6971.
• Saver JL, Lewis RJ. Number needed to treat: conveying the likelihood of a therapeutic
effect. JAMA. 2019;321(8):798799. doi:10.1001/jama.2018.21971.
• Smith TK, Johnson SB. Research and statistics: distribution, variability, and statistical
significance. Pediatr Rev. 2010;31(10):431-432. doi:10.1542/pir.31-10-431.
• Tschudy MM, Rowe PC. Research and statistics: number needed to treat and intention to
treat analysis. Pediatr Rev. 2010;31(9):380-382. doi:10.1542/pir.31.9.380.

American Academy of Pediatrics 233


PREP® Self-Assessment PREPSA 2023

Question 70
A 6-year-old, previously healthy boy is brought to the urgent care center with a 2-week history
of cough and lowgrade fever. On physical examination, his temperature is 38°C, his oxygen
saturation is 94% in room air, and he has crackles over his left lung base. Chest radiography
shows a left lower lobe consolidation and a radiolucent lesion with a thin cortical rim with a
fallen leaf fragment in the metaphyseal region of the right proximal humerus (Item Q70). The
boy has no history of arm pain. Right upper extremity physical examination findings are normal.

Item Q70: Findings for the boy described in the vignette.


Reprinted with permission from Rosenblatt J, Koder A. Pediatr Rev. 2019;40(2):54

Of the following, the BEST next step in management of this boy’s incidental finding is
A. bone biopsy
B. bone scan
C. magnetic resonance imaging
D. observation

American Academy of Pediatrics 234


PREP® Self-Assessment PREPSA 2023

Correct Answer: D
The boy in the vignette has an incidental radiographic finding of a simple bone cyst, also known
as a solitary bone cyst or unicameral bone cyst (UBC). This is a benign pediatric bone tumor
representing 3% of primary bone tumor biopsies. Unicameral bone cysts occur most often
between ages 4 and 10 years; the etiology remains unknown. Most UBCs are located in the
metaphysis of the proximal humerus or femur but can occur in other appendicular or axial bones.
They are generally painless and discovered incidentally but can be mildly painful, reflecting a
microscopic fracture. Trauma to the area may result in an abrupt pathologic fracture presenting
with pain, edema, erythema, warmth, decreased range of motion, and deformity.

The diagnosis of UBC can be made with radiography alone. Typical features include location in
the metaphysis that extends to, but does not cross, the physis; radiolucency with a thin cortical
rim; and the pathognomonic sign of the “fallen leaf” fragment (Item C70) representing a portion
of fractured cortical bone settling to the dependent part of the fluid-filled cyst. The differential
diagnosis includes aneurysmal bone cyst (ABC), fibrous dysplasia, and enchondroma.
Observation is reasonable for asymptomatic and small UBCs. Indications for surgical
intervention include persistent pain, repeated fractures, and deformity prevention.

The diagnosis of UBC rarely requires more imaging than radiography, though if the diagnosis is
equivocal, a bone scan or magnetic resonance imaging may be useful to identify other
abnormalities, or help distinguish the lesion from an ABC. Because of the benign nature of
UBCs and the characteristic findings on radiography, a bone biopsy is not necessary for
diagnosis.

PREP Pearls
• The diagnosis of a unicameral bone cyst can be made using radiography; typical features
include location in the metaphysis that extends to, but does not cross, the physis;
radiolucency with a thin cortical rim; and the pathognomonic sign of the “fallen leaf”
fragment representing a portion of fractured cortical bone settling to the dependent part of
the fluid-filled cyst.
• The differential diagnosis of a unicameral bone cyst includes an aneurysmal bone cyst,
fibrous dysplasia, and enchondroma.
• Observation is reasonable for asymptomatic and small unicameral bone cysts. Indications
for surgical intervention include persistent pain, recurrent fracture, and prevention of
deformity.

ABP Content Specifications(s)


• Understand the natural history of a bone cyst
• Formulate a differential diagnosis of a bone cyst

American Academy of Pediatrics 235


PREP® Self-Assessment PREPSA 2023

Suggested Readings
• Pretell-Mazzini J, Murphy RF, Kushare I, Dormans JP. Unicameral bone cysts: general
characteristics and management controversies. J Am Acad Orthop Surg. 2014;22:295-
303. doi:10.5435/JAAOS-22-05-295.
• Rosenblatt J, Koder A. Understanding unicameral and aneurysmal bone cysts. Pediatr
Rev. 2019;40(2):51-59. doi:10.1542/pir.2015-0128.
• Wilkins RM. Unicameral bone cysts. J Am Acad Orthop Surg. 2000;8(4):217-224.
doi:10.5435/00124635-20000700000002.

American Academy of Pediatrics 236


PREP® Self-Assessment PREPSA 2023

Question 71
A 14-year-old adolescent girl is seen in the office for follow up of dysmenorrhea. Based on the
results of laboratory evaluation performed at her initial visit for this condition 9 months ago, she
was diagnosed with iron-deficiency anemia and type 2 diabetes (hemoglobin A1C, 6.6%). Today,
she reports ongoing heavy and painful menstrual cycles as well as increasing fatigue; she is
otherwise well. She has never had sexual intercourse. The girl’s vital signs are normal. Her body
mass index has increased over the past 3 months from 98% to 99% for age and sex. Her current
prescribed medications are ferrous sulfate 325 mg every other day, naproxen sodium 440 mg
twice daily at the time of her menses, and metformin 1,000 mg twice daily.
Laboratory data are shown:

Laboratory Test Result


Hemoglobin 9.3 g/dL (93 g/L)
Mean corpuscular volume 78 fL
Mean corpuscular hemoglobin concentration 34.2 g/dL (342 g/L)
Ferritin 2 ng/mL (2 µg/L)
Hemoglobin A1C 6.7%

Of the following, the BEST next step in this girl’s management is


A. assess adherence by asking open-ended questions
B. decrease the dose of metformin for 2 weeks
C. increase her dose of ferrous sulfate to 325 mg daily
D. refer her to a hematologist for possible iron infusion therapy

American Academy of Pediatrics 237


PREP® Self-Assessment PREPSA 2023

Correct Answers: A
The adolescent in the vignette has recently been diagnosed with chronic medical conditions that
require daily medications for long-term management. Medication adherence is difficult for
adolescents, especially if parents are not monitoring medications closely to increase
accountability. The best next step in the management of the girl in the vignette is to assess
medication adherence by asking her open-ended questions. For example, “Are you in charge of
your own medications?” “Do you ever accidentally forget to take them?” “Do any of them cause
you problems?” The physician should not increase or decrease the dosage of her medications
until it is assessed if the patient is taking their medications correctly. While referral to
hematology for iron infusion therapy may be needed at some point, this step should not occur
before open-ended inquiry regarding her medication adherence.

While adolescents are developing autonomy and independence and are able to take some
responsibility for their actions, their struggle with peer factors and a desire to be “normal” may
decrease adherence to medication or medical recommendations. Data show that adolescents who
are solely responsible for their own medications are less adherent than those who share
responsibility with their parents. Shared decision-making among adolescents, their parents, and
their physicians is an important aspect of chronic medical condition management, including
adherence. Physicians should work to minimize conflict between caregivers and adolescents and
develop treatment plans as a team.

Barriers to adherence include transition times (eg, young adults transitioning to adult services),
prolonged intervals between appointments, lack of knowledge regarding medication uses,
systems issues (eg, pharmacy or insurance problems), guilt regarding missed doses, and
medication side effects. The World Health Organization published a report on adherence that
outlines dimensions of adherence in 5 broad categories (Item C71). The report highlights the
importance of promoting support (strengths-based practice) instead of blame and recognition of
the negative effect of poverty (and other social determinants of health) on adherence.

American Academy of Pediatrics 238


PREP® Self-Assessment PREPSA 2023

PREP Pearls
• Adherence to medications and medical treatment plans can be challenging for adolescents
due to various factors including peer factors, striving for “normalcy,” forgetting the
medication, physical well-being (eg, side effect concerns), and parent conflict.
• Physicians can assess medication adherence with open-ended, nonjudgmental questions.
• Physicians should participate as team members with parents and adolescents to problem
solve how to improve adherence.

ABP Content Specifications(s)


• Understand factors that can affect adherence to health maintenance activities by
adolescents
• Understand how to improve adherence to medical regimens by adolescent patients,
including those with chronic illness, and the barriers to such adherence

Suggested Readings
• Hanghøj S, Boisen KA. Self-reported barriers to medication adherence among
chronically ill adolescents: a systematic review. J Adol Health. 2014;54(2):121-138.
doi:10.1016/j.jadohealth.2013.08.009.
• World Health Organization. Improving adherence rates: guidance for countries. Geneva,
Switzerland:2003.

American Academy of Pediatrics 239


PREP® Self-Assessment PREPSA 2023

Question 72
A 16-month-old boy is seen for a routine health supervision visit. He has been well since his last
visit 4 months ago and has not had any recent illness. The boy was breastfed until 12 months of
age, when he transitioned to cow milk. He started eating solid foods at 6 months of age, and
occasionally eats crackers, but mainly consumes milk (about 48 oz/day). He also drinks juice.
The boy’s activity level, stools, and urine output are normal. His heart rate is 100 beats/min. He
looks well nourished. His lips, tongue, and conjunctiva are pale. There is no scleral icterus, and
his mucous membranes are moist. He has a 2/6 systolic ejection murmur at the left sternal
border. The remainder of his physical examination findings are normal.

Laboratory data are shown:


Laboratory Test Result
White blood cell count 6,800/µL (6.8 × 109/L)
Hemoglobin 5.5 g/dL (55 g/L)
Platelet count 480 × 103/µL (480 × 109/L)
Mean corpuscular volume 65 fL
Red cell distribution width 18%
Neutrophils 41.2%
Lymphocytes 54.3%
Monocytes 4.2%
Eosinophils 0.3%
Reticulocyte 1%
Absolute reticulocyte count 44 × 103/µL (44 × 109/L)
Iron 20 µg/dL (3.6 µmol/L) (reference range, 45-165 µg/dL
[8.1-30 µmol/L])
Total iron-binding capacity 530 µg/dL (95 µmol/L) (reference range, 220-430 µg/dL
[39.4-77 µmol/L)
Unsaturated iron-binding 507 µg/dL (90.8 µmol/L) (reference range, 110-370
capacity µg/dL [19.7-66.2 µmol/L])
Iron saturation 3.7% (reference range, 14-50%)
Ferritin <5 ng/mL (<5 µg/L) (reference range, 30-400 ng/mL
[30-400 µg/L])

Of the following, the BEST next best step in this boy’s management is to
A. give a 20-mL/kg normal saline bolus, intravenously
B. give a 20-mL/kg transfusion of packed red blood cells
C. give a dose of iron, intravenously
D. recommend dietary changes and close follow-up

American Academy of Pediatrics 240


PREP® Self-Assessment PREPSA 2023

Correct Answer: C
The boy in the vignette has severe iron-deficiency anemia (hemoglobin level of 5.5 g/dL [55
g/L]) due to inadequate dietary intake of iron. The best next management step is to give a dose of
iron intravenously. Parenteral iron is indicated for children who require a rapid and/or short
treatment course, such as the boy in the vignette, and for children who do not tolerate oral iron
supplementation or have ongoing blood loss or gastrointestinal disease.

The boy in the vignette is hemodynamically stable so he does not require an immediate increase
in his hemoglobin level. A transfusion of packed red blood cells would be indicated to rapidly
increase the hemoglobin level if the boy had symptoms such as severe fatigue, syncope or near
syncope, or significant tachycardia. An intravenous normal saline bolus would be indicated if the
boy was hemodynamically unstable. Normal saline boluses should be given with caution in the
setting of anemia because the dilutional effect causes worsening of the anemia and decreased
oxygen-carrying capacity, and could lead to heart failure. Although dietary changes should be
recommended for the boy, his severe anemia necessitates more rapid iron replacement.

Oral iron supplementation would also be appropriate for the boy in the vignette. The ferrous
form of iron, at a minimum dose of 3 mg/kg/day, is sufficient to replenish iron stores over
several months (3 or more). Very high doses of oral iron (eg, 6 mg/kg/day) can cause bowel
irritation and constipation, which may lead to decreased adherence.

Inadequate dietary intake is a common cause of iron-deficiency anemia. The boy in the
vignette’s diet is deficient in iron-containing foods such as meats, legumes, and fortified cereals.
Cow milk has a low iron content and excessive cow milk intake may interfere with intestinal iron
absorption; therefore, cow milk intake should be limited to 24 oz per day. Gastrointestinal blood
loss due to cow milk protein allergy can also lead to iron-deficiency anemia. In cases where cow
milk protein allergy is suspected, it is important to eliminate all cow milk from the diet until the
evaluation is complete. To ensure their infants have adequate iron intake, mothers of solely
breastfed infants should continue to take their daily iron-containing prenatal vitamins.
Breastfeeding mothers who have a history of iron deficiency should continue to take iron
supplementation even when the child is supplementing the breast milk with other foods.

The boy in the vignette has thrombocytosis, which is likely a result of his iron deficiency.
Thrombocytopenia may also be seen in severe iron deficiency. While an increase in reticulocytes
might be expected in a child with anemia, this is dependent on an adequate iron supply; this
boy’s low reticulocyte percentage is likely due to his iron deficiency. An increase in the
reticulocyte percentage is one of the earliest changes observed after treatment with iron is
implemented.

American Academy of Pediatrics 241


PREP® Self-Assessment PREPSA 2023

PREP Pearls
• Parenteral iron is indicated for children with iron deficiency who are hemodynamically
stable and require a rapid and/or short treatment course, do not tolerate oral iron
supplementation, or have ongoing blood loss or gastrointestinal disease.
• A packed red blood cell transfusion is indicated for the treatment of symptomatic anemia
(eg, severe fatigue, syncope or near syncope, or significant tachycardia).
• An increase in the reticulocyte percentage is an early indicator of recovery during the
treatment of iron-deficiency anemia.

ABP Content Specifications(s)


• Plan the appropriate management of severe anemia of various etiologies, while
considering the risks associated with various therapies

Suggested Readings
• Lanzkowsky P, Lipton J, Fish JD. Lanzkowsky’s Manual of Pediatric Hematology and
Oncology. 6th ed. Elsevier; 2016:215-216.
• McFarren AK, Levy AS. Anemia and pallor. In: McInerny TK, Adam HM, Campbell
DE, DeWitt TG, Foy JM, Kamat DM, eds. American Academy of Pediatrics Textbook of
Pediatric Care. American Academy of Pediatrics; 2021:chap 128. Accessed September 1,
2022. Pediatric Care Online.
• Segel GB, Hirsh MG, Feig SA. Managing anemia in pediatric office practice: Part 1.
Pediatr Rev. 2002;23(3):75-84. doi:10.1542/pir.23-3-75.
• Tong S, Vichinsky E. Iron deficiency: implications before anemia. Pediatr Rev.
2021;42(1):69-83. doi:10.1542/pir.20180134.

American Academy of Pediatrics 242


PREP® Self-Assessment PREPSA 2023

Question 73
A 4-year-old boy is seen in the office with fever, rash, and decreased oral intake. He has had
fevers for 5 days, and a new skin rash was noted this morning. He has had no nasal congestion,
cough, vomiting, or diarrhea. His temperature is 38.5°C, heart rate is 130 beats/min, respiratory
rate is 30 breaths/min, blood pressure is 85/55 mm Hg, and oxygen saturation is 99% in room air.
He is irritable but consolable. His conjunctivae are injected without drainage bilaterally, lips and
tongue are erythematous without blisters or exudate, and he has cervical lymphadenopathy on the
right. A diffuse maculopapular erythematous rash is noted on his trunk and extremities as well as
erythema of his hands and feet. The remainder of his physical examination findings are
unremarkable.

Of the following, the BEST next management step for this boy is to
A. administer acetylsalicylic acid orally at 10 mg/kg per day
B. administer intravenous immunoglobulin at 2 g/kg
C. obtain a complete echocardiogram
D. obtain laboratory data including C-reactive protein level

American Academy of Pediatrics 243


PREP® Self-Assessment PREPSA 2023

Correct Answer: B
The boy in the vignette has a history and physical examination findings that meet criteria for
classic Kawasaki disease (KD). The best next step for him is treatment with 2 mg/kg of
intravenous immunoglobulin to reduce the risk of coronary artery aneurysm formation. Although
children with KD are treated with acetylsalicylic acid at the time of diagnosis, a dose of 10
mg/kg per day is not adequate. Echocardiography and laboratory evaluation are important to the
complete evaluation of KD, but they are not as urgent as the administration of immunoglobulin.

Classic KD is a clinical diagnosis made when a child has fever for 5 days plus at least 4 of the
following 5 criteria, per the American Heart Association:
• Erythema and cracked lips, strawberry tongue (Item C73), and/or erythema of oral and
pharyngeal mucosa Bilateral bulbar conjunctival injection sans exudate
• Maculopapular, diffuse, erythematous, or erythema multiforme-like rash
• Erythema and edema of the feet and hands in the acute phase and/or periungual
desquamation in the subacute phase
• Cervical lymphadenopathy (≥1.5-cm diameter), usually unilateral

Children with KD may have other evidence of multisystem involvement (Son) including
myocarditis, pericarditis, shock, pulmonary nodules, hydrops of the gallbladder, sterile pyuria,
and aseptic meningitis. Children with a prolonged fever with fewer than 4 of the aforementioned
classic features may have atypical or incomplete KD; this diagnosis may be supported by
laboratory and echocardiographic findings consistent with KD. A high index of suspicion is
necessary in such cases as it is important to make the diagnosis and begin treatment in a timely
manner to minimize the risk of complications. An algorithm for the evaluation and diagnosis of
incomplete KD is available (McCrindle).

Involvement of the cardiovascular system in children with KD can lead to significant morbidity
and mortality. Clinical findings may include a hyperdynamic state with tachycardia, a new or
accentuated innocent murmur, a new pathologic murmur due to valvular pathology, or a gallop.
The performance of echocardiography is important to evaluate the coronary arteries, myocardial
function, valvular function, and to assess for pericardial effusion.

Treatment is directed at preventing cardiovascular complications including the development of


coronary artery aneurysms. Upon diagnosis, and within 10 days of illness onset, 2 g/kg of
intravenous immunoglobulin should be administered to reduce the incidence of aneurysms. If a
patient is diagnosed after 10 days of illness and is still having fevers, it is reasonable to
administer intravenous immunoglobulin. Acetylsalicylic acid is given to treat inflammation and
as an antiplatelet agent, but has not been demonstrated to prevent aneurysm formation. Although
expert consensus is to treat with acetylsalicylic acid, the recommended dose varies
internationally from 30-50 mg/kg (low dose) to 80-100 mg/kg per day (high dose). High-dose
acetylsalicylic acid is typically used in the United States, while low dose is typically used in
Japan and Western Europe. There are currently no data to support superior efficacy of high-dose
vs low-dose acetylsalicylic acid.

American Academy of Pediatrics 244


PREP® Self-Assessment PREPSA 2023

PREP Pearls
• Classic Kawasaki disease is diagnosed when a child has fever for 5 or more days and at
least 4 of the following 5 clinical findings:
o Erythema and cracking of the lips, strawberry tongue, and/or erythema of oral and
pharyngeal mucosa
o Bilateral bulbar conjunctival injection without exudate
o Maculopapular diffuse erythematous or erythema multiforme-like rash
o Erythema and edema of the hands and feet in acute phase and/or periungual
desquamation in subacute phase
o Cervical lymphadenopathy (≥1.5-cm diameter), usually unilateral
• Children with a prolonged fever with fewer than 4 of the classic features may have
atypical or incomplete Kawasaki disease; this diagnosis may be supported by laboratory
and echocardiographic findings.
• Treatment of children with Kawasaki disease with 2 mg/kg of intravenous
immunoglobulin is required to reduce the incidence of coronary artery aneurysms;
acetylsalicylic acid is used to treat inflammation and block platelet function.

ABP Content Specifications(s)


• Identify cardiac complications associated with Kawasaki disease and how to prevent their
occurrence

Suggested Readings
• Jackson MA. Kawasaki disease. In: McInerny TK, Adam HM, Campbell DE, DeWitt
TG, Foy JM, Kamat DM, eds. American Academy of Pediatrics Textbook of Pediatric
Care. American Academy of Pediatrics; 2021:chap 280. Accessed September 1, 2022.
Pediatric Care Online.
• McCrindle BW, Rowley AH, Newburger JW, et al. Diagnosis, treatment and long-term
management of Kawasaki disease. A Scientific Statement from the American Heart
Association. Circulation. 2017;135(17):e927-e999. doi:10.1161/CIR.0000000000000484.
• Son MBF, Newburger J. Kawasaki disease. Pediatr Rev. 2018;39(2):78-90.
doi:10.1542/pir.2016-0182.

American Academy of Pediatrics 245


PREP® Self-Assessment PREPSA 2023

Question 74
A 14-year-old adolescent boy is admitted to the hospital for management of anemia,
hematochezia, and hypovolemia. He has a 6-month history of recurrent hematochezia and diffuse
abdominal pain with worsening symptoms over the past 3 weeks, now with 5 to 6 episodes of
bloody diarrhea daily. He reports bilateral knee pain, a 5-lb weight loss, fevers, and mouth sores
over the past 2 weeks. Physical examination reveals a pale and tired-appearing adolescent. His
heart rate is 120 beats/min, blood pressure is 130/70 mm Hg, weight is 57 kg (60th percentile for
age), height is 160 cm (18th percentile for age), and body mass index is 25 kg/m2 (90th
percentile for age). He has a soft systolic murmur, a diffusely tender abdomen without rebound
or guarding, and his rectal examination findings are notable for grossly bloody stools without
other abnormality.

Laboratory data are shown:


Laboratory Test Result
White blood cell count 11,100/µL (11.1 × 109/L)
Hemoglobin 7.8 g/dL (78 g/L)
Mean corpuscular volume 63 fL
Platelet count 478 × 103/µL (478× 109/L)
Prothrombin time 12 s
Stool pathogens Negative
Clostridium difficile Negative

After the administration of intravenous fluids and packed red blood cells, his heart rate
normalizes. Endoscopy with colonoscopy is performed after a bowel cleanout. The upper
endoscopy findings are normal. The colonoscopy demonstrates severe mucosal erythema,
friability and edema, with exudate from the rectum to the cecum (Item Q74); pathology
demonstrates severe chronic colitis without granulomas with a normal terminal ileum. Magnetic
resonance enterography reveals a thickened colonic bowel wall with a normal-appearing small
intestine. Of the following, the BEST treatment for the adolescent’s condition is

A. exclusive enteral nutrition


B. intravenous corticosteroids
C. oral azathioprine
D. oral tacrolimus

American Academy of Pediatrics 246


PREP® Self-Assessment PREPSA 2023

Correct Answer: B
The adolescent in the vignette has findings consistent with ulcerative colitis (UC); the
constellation of signs, symptoms, and laboratory findings meets the criteria for acute severe
colitis (ASC). The initial therapy recommended for ASC includes intravenous corticosteroids
and supportive care (including treatment of anemia and nutritional support as clinically
warranted).

Inflammatory bowel disease (IBD) is a chronic, immune-mediated disease of the gastrointestinal


tract, classified by the major subtypes Crohn disease (CD) and UC. Crohn disease can affect any
part of the gastrointestinal tract from the mouth to the anus; it is characterized by patchy,
transmural intestinal inflammation with the presence of granulomas. Ulcerative colitis is limited
to the colon and is characterized by contiguous superficial inflammation beginning in the rectum
and extending proximally as far as the cecum.

The presentation of IBD can include both intestinal and extraintestinal signs and symptoms
(Item C74A). Intestinal signs/symptoms can include chronic diarrhea, hematochezia, vomiting,
and abdominal pain. Signs of intestinal obstruction and/or perianal disease (skin tags, fissures,
fistulae, and/or abscess) can be present with CD. Extraintestinal signs/symptoms can include

American Academy of Pediatrics 247


PREP® Self-Assessment PREPSA 2023

rashes (erythema nodosum, pyoderma gangrenosum), arthritis, ankylosing spondylitis, uveitis,


growth failure, anorexia, weight loss or poor weight gain, and fevers.

The initial management of IBD is dependent on the subtype and severity of the disease (Item
C74B).
• For mild to moderate UC: Aminosalicylates (sulfasalazine or mesalamine) can be used
as first-line therapy; oral and rectal formulations are available. Immunomodulators
(azathioprine [AZA], 6-mercaptopurine [6-MP], methotrexate) can be considered to
maintain remission but are not useful for induction of remission. It is important for
clinicians to consider the association of AZA and 6-MP with hepatosplenic T-cell
lymphoma before usage.
• For acute severe UC: Intravenous corticosteroids are recommended first-line therapy.
Treatment with biologic agents (infliximab, adalimumab) or colectomy should be
considered if the condition is not improving after the first 5 to 7 days of intravenous
corticosteroids.
• For mild to moderate CD: Aminosalicylates, prednisone, and immunomodulators can
be considered. Exclusive enteral nutrition (EEN), the use of polymeric or semi-elemental
formula to meet 95% to 100% of caloric needs, may also be considered for induction and
maintenance of remission.
• For severe CD (including children with growth failure and stricturing/fistulizing
disease): Infliximab or adalimumab (with or without an immunomodulator) is generally
used to induce and maintain remission. Corticosteroids and EEN may also be used for
induction of remission, and EEN can also be used to maintain remission.

American Academy of Pediatrics 248


PREP® Self-Assessment PREPSA 2023

Medical management of inflammatory bowel disease. BUN ¼ blood urea nitrogen; CBC ¼ complete
blood cell count; 6-MP ¼ 6-mercaptopurine; PPD ¼ tuberculosis skin test; QO ¼ once per week; SQ ¼
subcutaneously; TPMT ¼ thiopurine methyltransferase.

Recommendations for health supervision of pediatric patients with IBD, including office
management and medication side effects, are detailed in Rufo.

American Academy of Pediatrics 249


PREP® Self-Assessment PREPSA 2023

For the adolescent in the vignette, azathioprine is not recommended as it does not quickly induce
remission.

Exclusive enteral nutrition is indicated for the treatment of CD, but is not indicated for the
management of UC. Tacrolimus could be considered as a second- or third-line treatment for
acute UC; it would not be used as initial therapy.

PREP Pearls
• Inflammatory bowel disease is a chronic, immune-mediated disease of the
gastrointestinal tract, with 2 major subtypes: Crohn disease and ulcerative colitis.
• Crohn disease can occur anywhere within the gastrointestinal tract and is characterized by
transmural, usually patchy, chronic intestinal inflammation. Ulcerative colitis is limited
to the colon and is characterized by superficial inflammation beginning in the rectum and
extending proximally as far as the cecum.
• Medical and nutritional therapies are options for induction and maintenance of remission
for children with inflammatory bowel disease (IBD); types of therapies are dependent on
the location, severity, and classification of the child’s IBD.

ABP Content Specifications(s)


• Recognize the association of arthritis with inflammatory bowel disease
• Distinguish the clinical features associated with Crohn disease from those of ulcerative
colitis

Suggested Readings
• Dykes DMH, Saeed SA. Inflammatory bowel disease. In: McInerny TK, Adam HM,
Campbell DE, DeWitt TG, Foy JM, Kamat DM, eds. American Academy of Pediatrics
Textbook of Pediatric Care. American Academy of Pediatrics; 2021:chap 276. Accessed
September 1, 2022. Pediatric Care Online.
• Rufo PA, Denson LA, Sylvester FA, et al. Health supervision in the management of
children and adolescents with IBD: NASPGHAN recommendations. J Pediatr
Gastroenterol Nutr. 2012;55(1):93-108. doi:10.1097/MPG.0b013e31825959b8.
• Shapiro J, Subedi S, LeLeiko N. Inflammatory bowel disease. Pediatr Rev.
2016;37(8):337-347. doi:10.1542/pir.20150110.
• Turner D, Ruemmele FM, Orlanski-Meyer E, et al. Management of paediatric ulcerative
colitis, part 2: acute severe colitis-an evidence-based consensus guideline from the
European Crohn’s and Colitis Organization and the European Society for Paediatric
Gastroenterology, Hepatology, and Nutrition. J Pediatr Gastroenterol Nutr.
2018;67(2):292-310. doi:10.1097/MPG.0000000000002036.

American Academy of Pediatrics 250


PREP® Self-Assessment PREPSA 2023

Question 75
A 7-year-old boy is seen in the emergency department for left knee swelling. He sustained a fall
2 weeks ago, at which time he developed left knee swelling and tenderness. He was seen by an
orthopedic surgeon who aspirated 40 mL of blood from that knee. Magnetic resonance imaging
did not reveal any significant musculoskeletal abnormality other than hemarthrosis with evidence
of some hemosiderin. Over the past few days, the knee swelling has worsened. There is increased
swelling and pain, for which he has been taking ibuprofen, that makes it difficult for him to
ambulate. He has no fever or erythema at the site of swelling.

Of the following, the BEST test to confirm this child’s diagnosis is


A. coagulation profile with mixing studies
B. complete blood cell count with reticulocyte count
C. joint fluid analysis
D. magnetic resonance imaging

American Academy of Pediatrics 251


PREP® Self-Assessment PREPSA 2023

Correct Answer: A
The boy in the vignette has evidence of bleeding into his joint. Results of a coagulation profile,
including prothrombin time (PT), international normalized ratio (INR), and activated partial
thromboplastin time (aPTT) with mixing studies, can help determine the diagnosis and guide
further testing. The tests are initially performed on the patient’s blood sample. The sample is
then mixed with a normal donor sample, and the tests are repeated. After 2 hours of incubation,
the mixed sample testing is repeated. If an individual has a prolonged PT or aPTT due to a
missing clotting factor, the prolonged time will correct once the sample is mixed with a normal
sample, and this correction will persist after the 2-hour incubation period. However, if an
individual produces the clotting factor, but has created an antibody against it (eg, a lupus
anticoagulant), the initially corrected time will become prolonged again. For the child in the
vignette with hemophilia, the initial aPTT will be prolonged, and the mixing study will be
corrected at both hours 0 and 2 (Item C75).

Basic process for evaluating coagulation profile abnormalities. If available, a mixing study should be
performed to differentiate a coagulation factor deficiency from an inhibitor. The possible factors involved
may then be narrowed down on the basis of which coagulation study or studies are abnormal. Note:
PTand APTTare normal in FXIII deficiency.61 H/LMWK, high-/low-molecular-weight kininogen.

The hemosiderin noted on magnetic resonance imaging suggests that there is blood around the
joint. An individual with hemophilia can sustain bleeding in a joint from trauma. That joint may
become a target joint, one that is more likely to have repeated bleeding episodes.
The most common form of hemophilia is factor VIII deficiency. The goal of treatment is to
increase the factor level to prevent ongoing bleeding and promote healing. Administration of
recombinant or purified plasma-derived factor VIII products is the treatment of choice for the
boy in the vignette. He will also need prophylactic factor replacement to prevent future damage
to the joint and eventual arthropathy.

A complete blood cell count (CBC) might be helpful in determining whether the boy in the
vignette has an infection by demonstrating an elevated white blood cell count. However, it is
unlikely that this child has a serious knee infection without fever or erythema. Although a CBC
American Academy of Pediatrics 252
PREP® Self-Assessment PREPSA 2023

may reveal anemia from the previous bleeding episode, it will not confirm the boy’s underlying
diagnosis.

Analysis of the joint fluid would be helpful to diagnose a septic joint or osteomyelitis. However,
as noted above, without fever or joint erythema, it is unlikely that the child in the vignette has a
serious knee infection. Additionally, the hemosiderin noted on magnetic resonance imaging
would not be seen in the case of bone or joint infection.

Magnetic resonance imaging may reveal evidence of musculoskeletal injury from the fall 2
weeks earlier or signs of infection (eg, septic join, osteomyelitis). However, a significant injury
should have been evident on the previous imaging study; the boy’s clinical picture is not
consistent with a serious knee infection, such as osteomyelitis or a septic joint.
Of note, aspirin and nonsteroidal antiinflammatory drugs (eg, ibuprofen) should be avoided in
individuals with known or suspected hemophilia due to the antiplatelet effects of these drugs.

PREP Pearls
• A coagulation profile with mixing studies is helpful in evaluating a child with a suspected
bleeding disorder.
• An individual with hemophilia with bleeding into a joint after trauma may develop a
target joint (a joint more likely to have repeated bleeding episodes).
• Aspirin and nonsteroidal antiinflammatory drugs (eg, ibuprofen) should be avoided in
individuals with known or suspected hemophilia due to the antiplatelet effects of these
drugs.

ABP Content Specifications(s)


• Recognize the clinical manifestations and complications associated with hemophilia, and
manage appropriately

Suggested Readings
• Acharya SS. Rare bleeding disorders in children: identification and primary care
management. Pediatrics. 2013;132(5):882-892. doi:10.1542/peds.2012-3662.
• Callaghan MU, MD, Rajpurkar M. Coagulation disorders. In: McInerny TK, Adam HM,
Campbell DE, DeWitt TG, Foy JM, Kamat DM, eds. American Academy of Pediatrics
Textbook of Pediatric Care. American Academy of Pediatrics; 2021:chap 230. Accessed
September 1, 2022. Pediatric Care Online.
• Rodriguez V, Warad D. Pediatric coagulation disorders. Pediatr Rev. 2016;37(7):279-
291. doi:10.1542/pir.2015-0062 .
• Zimmerman B, Valentino LA. Hemophilia: in review. Pediatr Rev. 2013;34(7):289-294;
quiz 295. doi: 10.1542/pir.34-7-289

American Academy of Pediatrics 253


PREP® Self-Assessment PREPSA 2023

Question 76
A 5-year-old boy is brought to the emergency department for fever and rash. The family returned
to New York after a 2-week camping trip to North Carolina in August. He has been having
persistent fevers for 3 days, and is complaining of a severe headache. He developed a rash over
his wrists and ankles this morning. He is up to date with vaccinations. On physical examination,
the boy has a temperature of 40°C, heart rate of 140 beats/min, respiratory rate of 44
breaths/min, blood pressure of 80/40 mm Hg, and oxygen saturation of 98% in room air. He has
a pinpoint blanchable rash over the trunk and distal extremities.

Of the following, the laboratory findings MOST consistent with the boy’s condition are:
Response Serum Platelet Transaminases
choice sodium count
Row A decreased increased increased
Row B decreased decreased increased
Row C increased increased normal
Row D increased decreased normal

A. Row A
B. Row B
C. Row C
D. Row D

American Academy of Pediatrics 254


PREP® Self-Assessment PREPSA 2023

Correct Answer: B
The boy in the vignette has Rocky Mountain spotted fever (RMSF). This condition is difficult to
diagnose in the early stages of illness, as in the first 3 days the signs, symptoms, and laboratory
findings overlap with other common viral infections, bacterial infections, and inflammatory
conditions that present as acute sepsis syndrome. Epidemiologic clues such as tick exposure in
an endemic region or outdoor activity are helpful, however they are absent in approximately half
of RMSF cases. This child has high fevers, headaches, and a maculopapular rash that started at
the wrists and ankles. This presentation, along with a history of outdoor activity during the
summer in North Carolina, is suggestive of RMSF. Laboratory tests typically reveal
hyponatremia, thrombocytopenia, and elevated transaminases.

In the United States, RMSF is most prevalent in the Southeastern and South Central states. Five
states account for about 63% of the infections: Arkansas, Missouri, North Carolina, Oklahoma,
and Tennessee. Peak prevalence is in the summer season. Rocky Mountain spotted fever is
caused by the bacteria Rickettsia rickettsii, an intracellular gram-negative bacillus. The principal
vectors are the American dog tick (Dermacentor variabilis), Rocky Mountain wood tick
(Dermacentor andersoni), and brown dog tick (Rhipicephalus sanguineus) (Item C76A).

The clinical presentation of RMSF frequently starts with a fever (99% to 100% of cases),
myalgia, and headache (79% to 91%). The temperature exceeds 38.9°C for the majority of
children during the first 3 days. In a few cases, the classic rash is noted on the first day of illness;
however, it is more likely to be seen on the second or third day. Eventually, the rash will be
present in 88% to 90% of cases. The rash may be diffuse initially; it usually begins around the
wrists and ankles (Item C76B); however, it may start on the trunk. Rash on the palms and soles
is often considered a typical feature; however, it may not be present in all affected children and
occurs later in the clinical course of the disease. Rocky Mountain “spotless” fever may occur in
older patients and in patients with dark skin, which can make the diagnosis more challenging.
The laboratory findings of RMSF reflect vascular injury. A profound thrombocytopenia may be
seen secondary to platelet consumption along damaged surfaces. Slightly increased transaminase
levels and hyponatremia may be noted.

American Academy of Pediatrics 255


PREP® Self-Assessment PREPSA 2023

Rocky Mountain spotted fever is a life-threatening infection; empiric treatment should be


expedited in children with clinical features and epidemiology suggestive of RMSF. Doxycycline
(or other tetracyclines) is the antibiotic of choice, irrespective of the child's age. Treatment
started within 5 days of initial symptoms is associated with a decreased rate of hospitalization,
intensive care unit admission, and mortality. Confirmation of RMSF is usually made by positive
indirect immunofluorescence assay; this may be nonreactive early in the disease course.
Confirmation of the diagnosis requires a 4-fold rise in titer between acute and convalescent sera
2 to 6 weeks apart.

PREP Pearls
• Rocky Mountain spotted fever presents with nonspecific signs and symptoms early in the
disease course. Clinical features include high fever, headache, and a rash that begins at
the wrists and ankles, with laboratory findings of thrombocytopenia, hyponatremia, and
elevated transaminases.
• Rocky Mountain spotted fever is caused by the intracellular bacteria Rickettsia rickettsii;
peak prevalence is during the summer months.
• Treatment with doxycycline, irrespective of a child’s age, should be initiated promptly
when Rocky Mountain spotted fever is suspected.

ABP Content Specifications(s)


• Understand the epidemiology of Rocky Mountain spotted fever
• Plan the appropriate management of suspected or confirmed Rocky Mountain spotted
fever
• Recognize the clinical features associated with Rocky Mountain spotted fever

American Academy of Pediatrics 256


PREP® Self-Assessment PREPSA 2023

Suggested Readings
• American Academy of Pediatrics. Rocky Mountain spotted fever. In: Kimberlin DW,
Barnett ED, Lynfield R, Sawyer MH, eds. Red Book: 2021–2024 Report of the
Committee on Infectious Diseases. 32nd ed. American Academy of Pediatrics; 2021.
Accessed September 1, 2022. Red Book Online.
• Noor A, Triche AB, Krilov LR. Rocky Mountain spotted fever and other Rickettsioses.
In: Domachowske J, ed. Introduction to Clinical Infectious Diseases. 2019. Accessed
September 1, 2022. doi:10.1007/978-3-319-91080-2_33.
• Paddock CD, Alvarez-Hernández G. Rickettsia rickettsii (Rocky Mountain spotted fever).
In: Long SS, Prober CG, Fischer M, eds. Principles and Practice of Pediatric Infectious
Diseases. 5th ed. Elsevier; 2018:952-957.
• Razzaq S, Schutze GE. Rocky Mountain spotted fever: a physician's challenge. Pediatr
Rev. 2005;26(4):125-130. doi:10.1542/pir.26-4-125.
• Seese RT, Danziger-Isakov L. Rocky Mountain spotted fever. In: McInerny TK, Adam
HM, Campbell DE, DeWitt TG, Foy JM, Kamat DM, eds. American Academy of
Pediatrics Textbook of Pediatric Care. 2nd ed. Itasca, IL: American Academy of
Pediatrics; 2017:2592-2596.
https://publications.aap.org/redbook/book/347/chapter/5755620/Rocky-Mountain-
Spotted-Fever?searchresult=1.

American Academy of Pediatrics 257


PREP® Self-Assessment PREPSA 2023

Question 77
A 14-year-old adolescent girl is seen in the office for evaluation of right thigh pain, which began
1 month ago during soccer practice. The pain is located in her anterior right thigh over the femur
midshaft. Initially, the pain occurred only with high-impact activities (eg, running), but now also
occurs while walking. She recalls a collision with another soccer player around the time her pain
began, but is unsure if this was directly related to the onset of her pain. She denies fever, rash,
joint swelling, or change in appetite or activity level. The girl’s physical examination findings
are remarkable only for mild pain with hip internal rotation and when hopping on her right leg.
There are no areas of tenderness. Radiographs of the right hip and femur are unremarkable.

Of the following, the BEST next step in this girl’s evaluation and management is
A. magnetic resonance imaging of the femur
B. physical therapy for soft tissue mobilization
C. splinting the leg in a position of knee flexion for 1 to 2 days
D. use of a thigh compression wrap during soccer

American Academy of Pediatrics 258


PREP® Self-Assessment PREPSA 2023

Correct Answer: A
The adolescent in the vignette has a history of gradually worsening, activity-related pain and
participates in a highimpact sport. Her history and physical examination findings suggest a femur
stress fracture. The best next step in her evaluation and management is magnetic resonance
imaging (MRI) of the femur. Item C77 is a T2-weighted MRI scan of the femur demonstrating a
right femoral neck stress fracture with edema of the inferior aspect of the femoral neck and a
subtle fracture line.

Conditions affecting the hip often cause pain that refers to the anterior thigh, as is the case for the
girl in the vignette. If an athlete with a stress fracture continues to participate in high-impact
physical activity, the stress fracture can become a complete fracture. Stress fractures of the
femoral neck are especially worrisome because complete fractures of the femoral neck can lead
to avascular necrosis of the femoral head.

Other orthopedic problems that can cause pain in the anterior thigh include slipped capital
femoral epiphysis (SCFE), bone cysts or tumors, quadriceps contusion, and quadriceps strain.
The girl in the vignette has normal hip and femur radiography, making SCFE and bone tumors
unlikely diagnoses; these conditions are usually seen on radiography. A “preslip” that occurs
before the onset of a SCFE may not be visible on radiography. This girl does not have decreased
hip internal rotation that might suggest a SCFE or a preslip condition.

The girl in the vignette reports a collision with another athlete. A direct hit to the thigh can cause
a hematoma in the muscle (quadriceps contusion) which would present with anterior thigh pain.
For quadriceps strains and contusions, pain is elicited with palpation of the quadriceps and with
quadriceps contraction. With a mild quadriceps contusion, it is not uncommon for affected
individuals to be unsure of the exact event that led to injury. However, mild contusions usually
improve within 1 week so this is an unlikely diagnosis for the girl in the vignette.

Treatment for mild muscle contusions includes ice, stretching, a compression wrap, and rest from
activities for 1 to 2 days. Children and adolescents with moderate to severe contusions often have
American Academy of Pediatrics 259
PREP® Self-Assessment PREPSA 2023

swelling, pain, and longer-lasting symptoms. Moderate to severe contusions are treated with
icing, wrapping the knee in a flexed position for 1 to 2 days, and rest from sports. Massage and
soft tissue mobilization should be avoided as these techniques can worsen a quadriceps
contusion. A thigh compression wrap may provide comfort when returning an athlete with a
thigh contusion back to sports.

Myositis ossificans is a complication of quadriceps contusion. Calcification develops within the


hematoma, typically 3 to 6 weeks after the initial injury. There is increased pain with activity and
often night pain as well. The calcified tissue may be palpable on physical examination and is
visible on radiography. Myositis ossificans can lead to prolonged pain and time out of sport
activity.

PREP Pearls
• Signs and symptoms of a femoral neck stress fracture include pain with high-impact
activities, groin and/or anterior thigh pain, and pain with internal rotation of the hip.
• Femoral neck stress fractures are important to recognize and manage carefully because
complete fractures can lead to avascular necrosis of the femoral head.
• Conditions that affect the hip in children and adolescents often cause pain that refers to
the anterior thigh.

ABP Content Specifications(s)


• Plan the appropriate management of bruises and hematomas
• Identify complications associated with a deep hematoma of the thigh

Suggested Readings
• Coleman N. Sports injuries. Pediatr Rev. 2019;40(6):278-290. doi:10.1542/pir.2018-
0221.
• Kannikeswaran N, Suresh S. Sports musculoskeletal injuries. In: McInerny TK, Adam
HM, Campbell DE, DeWitt TG, Foy JM, Kamat DM, eds. American Academy of
Pediatrics Textbook of Pediatric Care. American Academy of Pediatrics; 2021:chap 334.
Accessed September 1, 2022. Pediatric Care Online.
• Kerbel YE, Smith CM, Prodromo JP, Nzeogu MI, Mulcahey MK. Epidemiology of Hip
and Groin Injuries in Collegiate Athletes in the United States. Orthop J Sports Med.
2018;6(5):2325967118771676. doi:10.1177/2325967118771676.
• Sarwark JF, Labella CR. Overuse injuries. Orthopaedics and Sports Medicine: A Quick
Reference Guide. 3rd ed. American Academy of Pediatrics. 2021:311-342.
• Sarwark JF, Labella CR. Traumatic muscle injuries. In: Orthopaedics and Sports
Medicine: A Quick Reference Guide. 3rd ed. American Academy of Pediatrics.
2021:301-309.

American Academy of Pediatrics 260


PREP® Self-Assessment PREPSA 2023

Question 78
A 15-year-old adolescent girl with anorexia nervosa is admitted to the hospital with weight loss
of 6 kg over the past 2 months. She has no history of vomiting, diarrhea, or abdominal pain. She
is not taking any medications. The girl’s heart rate is 50 beats/min, respiratory rate 16
breaths/min, and blood pressure 100/60 mm Hg. Her weight is less than the third percentile,
height is at the 25th percentile, and body mass index is 15 kg/m2. She has pallor, temporal
wasting, and a decrease in subcutaneous fat. The remainder of her physical examination findings
are unremarkable.

Laboratory data on admission are shown:


Laboratory Test Result
Sodium 137 mEq/L (137 mmol/L)
Potassium 4 mEq/L (4 mmol/L)
Chloride 98 mEq/L (98 mmol/L)
Bicarbonate 22 mEq/L (22 mmol/L)
Blood urea nitrogen 7 mg/dL (2.5 mmol/L)
Creatinine 0.5 mg/dL (44.2 µmol/L)
Calcium 8.8 mg/dL (2.2 mmol/L)
Magnesium 1.8 mg/dL (0.7 mmol/L)
Phosphorus 3.9 mg/dL (1.3 mmol/L)
Glucose 90 mg/dL (5 mmol/L)
Hemoglobin 10.5 g/dL (105 g/L)

Of the following, the electrolyte abnormality MOST likely to develop in this adolescent is
A. hypermagnesemia
B. hyperphosphatemia
C. hypokalemia
D. hyponatremia

American Academy of Pediatrics 261


PREP® Self-Assessment PREPSA 2023

Correct Answers: C
The girl in the vignette has normal serum electrolytes at admission, but her total body potassium
is most likely depleted because of malnutrition from anorexia nervosa. Once enteral nutrition is
started, she is at risk for hypokalemia, hypophosphatemia, and hypomagnesemia from refeeding
syndrome.

Potassium is predominantly an intracellular cation with a normal physiologic requirement of 2 to


3 mEq/kg per day. The normal concentration of potassium in the intracellular compartment is
140 to 150 mEq/L, while the extracellular compartment (serum) concentration is 4 to 5 mEq/L.
The serum potassium levels are determined by daily intake, losses from the renal or
gastrointestinal tract, and shift of potassium between the intracellular and extracellular fluid
compartments.

The factors that determine the distribution of potassium between the extracellular and
intracellular compartment include hormones regulating glucose metabolism, action of adrenergic
receptors, and acid-base status. Insulin promotes entry of potassium into the cell and glucagon
inhibits the intracellular shift. β-Adrenergic agonists (eg, albuterol, epinephrine) promote entry
into the cell leading to hypokalemia, whereas β-blockers inhibit entry into the cell leading to
hyperkalemia. Alkalosis causes a shift of potassium into the cells; acidosis causes a shift from
the intracellular to the extracellular compartment leading to hyperkalemia. As a result of
electrolyte shifts, the serum potassium level does not always reflect the total body content of
potassium.

Children with malnutrition generally have low total body potassium. The liver undergoes
gluconeogenesis and breaks down protein during the initial period of malnutrition. Electrolyte
shifts occur from the intracellular to extracellular compartment to maintain normal serum levels.
For the child in the vignette, the commencement of nutrition (refeeding) makes carbohydrates the
main source of energy. The resulting increase in insulin level shifts the potassium, magnesium,
and phosphorus to the intracellular compartment leading to hypokalemia, hypomagnesemia, and
hypophosphatemia. Hypernatremia can occur to balance the cations, which in turn can lead to
fluid overload.

Feedings are usually initiated at 50% of estimated caloric need, advanced gradually over 3 to 7
days with a caloric increase of 10% to 25% per day, and electrolytes carefully monitored to
prevent complications of refeeding syndrome.

PREP Pearls
• Potassium is predominantly an intracellular cation with a normal physiologic requirement
of 2 to 3 mEq/kg per day.
• Insulin, β-adrenergic agonists, and alkalosis promote potassium entry into the
intracellular compartment and cause hypokalemia.
• Once nutrition is started, a malnourished child is at risk for hypokalemia,
hypophosphatemia, and hypomagnesemia from refeeding syndrome.

American Academy of Pediatrics 262


PREP® Self-Assessment PREPSA 2023

ABP Content Specifications(s)


• Understand that serum potassium concentration does not reflect total body content of
potassium
• Identify the physiologic requirements for sodium and potassium in patients of various
ages

Suggested Readings
• Mahajan P, Felt JR. Fluids, electrolytes, and acid-base composition. In: McInerny TK,
Adam HM, Campbell DE, DeWitt TG, Foy JM, Kamat DM, eds. American Academy of
Pediatrics Textbook of Pediatric Care. American Academy of Pediatrics; 2021:chap 239.
Accessed September 1, 2022. Pediatric Care Online.
• Pulcini CD, Zettle S, Srinath A. Refeeding syndrome. Pediatr Rev. 2016;37(12):516-523.
doi:10.1542/pir.2015-0152.

American Academy of Pediatrics 263


PREP® Self-Assessment PREPSA 2023

Question 79
A 15-year-old adolescent girl is evaluated in the emergency department for worsening headache
and gait changes. Over the past 2 years her walking has progressively looked and felt more
awkward and unbalanced. Over the past 6 months she has developed a constant dull headache
with associated photophobia, nausea, and vomiting. Her headache has escalated in severity over
the past 3 weeks. The headache wakes her from sleep 4 times per week. The pain is worse in the
morning, while lying down, and when she coughs or sneezes. Over the past few weeks she
developed blurring of her vision and intermittent paresthesias in both hands.

She is alert and oriented. Her visual acuity is 20/50 bilaterally. She has bilateral papilledema and
is unable to fully abduct her eyes. The remainder of her cranial nerve findings are normal. She
has full strength in all 4 extremities with diffuse hyperreflexia. Her toes are downgoing with
plantar stimulation. Coordination testing shows mild dysmetria on finger to nose bilaterally. Her
gait is shuffling, wide based, and hesitant; she prefers to hold on to her mother for support while
ambulating. The remainder of her physical examination findings are normal.

Of the following, the BEST next step in this adolescent’s management is


A. lumbar puncture with opening pressure measurement
B. magnetic resonance imaging of the brain with and without contrast
C. neurosurgical consultation for placement of a ventriculoperitoneal shunt
D. ophthalmology consultation for evaluation of fundus and eye movements

American Academy of Pediatrics 264


PREP® Self-Assessment PREPSA 2023

Correct Answer: B
The adolescent in the vignette is exhibiting symptoms and signs of increased intracranial
pressure (ICP) with worsening headache, blurry vision, paresthesias, papilledema, and abnormal
gait. Of the response choices, the best next management step is to obtain magnetic resonance
imaging (MRI) of the brain with and without contrast to guide further diagnostic work-up. Head
computed tomography (CT) without contrast would also be an appropriate option.

The skull is a fixed space. Brain, cerebrospinal fluid (CSF), and blood constitute the 3
compartments of that space. Because the space is fixed, if the volume of any one compartment is
altered beyond the ability for a compensatory change in one or both of the other two, intracranial
pressure builds, leading to increased ICP and the potential for life-threatening brain herniation.

The neurological history and physical examination findings are critical to identifying the
presence of increased ICP and formulating the differential diagnosis. Depending on the
underlying etiology (Item C79A), children and adolescents with increased ICP can present
acutely, sub-acutely, or chronically, and there is potential for a lifethreatening neurological
emergency. Signs and symptoms may vary according to the timeline of increase in ICP (Item
C79B). Together, the timeline and nature of symptoms aid in defining the differential diagnosis
and management plan. This adolescent’s case, evolving over months to years, is subacute to
chronic, suggesting a gradual increase in her ICP.

Item C79A: Causes of Increased Intracranial Pressure in Children


• HEAD TRAUMA
• Cerebral edema
• Intracerebral hemorrhage
• Extracerebral hemorrhage (subdural, epidural)
• VASCULAR CAUSES
• Arterial or venous infarctions
• Intracerebral hemorrhage
• Dural sinus thrombosis
• Subarachnoid hemorrhage
• Vascular anomalies (vein of Galen malformation,
• arteriovenous malformations)
• NEOPLASTIC CAUSES
• Primary brain tumors
• Metastatic (intracerebral, meningeal infiltration)
• Hydrocephalus (congenital or acquired,
• communicating or noncommunicating)
• Pseudotumor cerebri (benign intracranial
• hypertension)
• Central nervous system infections
• Meningitis (bacterial, fungal, mycobacterial)
• Encephalitis (focal or diffuse)
• Abscess
• METABOLIC CAUSES
• Inborn errors of metabolism (hyperammonemia)
American Academy of Pediatrics 265
PREP® Self-Assessment PREPSA 2023

• Hepatic encephalopathy
• Diabetic ketoacidosis
• Renal failure
• Reye syndrome
• Status epilepticus
• Hypoxic-ischemic encephalopathy
• Fluid-electrolyte abnormalities (hyponatremia,
• hypernatremia)
• STRUCTURAL CAUSES
• Craniosynostosis

Increased ICP should be managed by a multidisciplinary team including pediatric critical care,
neurosurgery, and neurology specialists. The acute management of ICP is focused on assessing
airway, breathing, and circulation, and stabilization of the patient. The immediate goal is to
lower the ICP while maintaining adequate cerebral perfusion pressure (CPP). Once the child’s
condition is stabilized, focus can shift to identifying the underlying cause.

Symptoms of uncal herniation include coma and unilateral pupillary dilation with ipsilateral
paralysis due to compression of cranial nerve III and displacement of the cerebral peduncles and
brainstem. Herniation of the cerebellar tonsils downward presents with coma, decerebrate
posturing, and the autonomic changes of Cushing triad (bradycardia, hypertension with widened
pulse pressure, and abnormally slow respirations). Herniation is a neurological emergency that is
diagnosed clinically; no diagnostic testing is required. Herniation should be treated immediately
with hypertonic saline and/or other osmotic agents and bedside maneuvers (eg, hyperventilation).
Definitive treatment may include decompression (ie, craniotomy) or external ventricular drain
placement depending on the underlying cause.

Once the patient is stabilized, diagnostic testing aimed at identifying the underlying etiology
should be performed. Head CT without contrast is a reliable study for quickly evaluating for
mass effect, hemorrhage, hydrocephalus, or bone fracture and is often obtained in the acute
setting. Brain MRI is useful for evaluation of parenchymal injury, cerebral edema, and the
posterior fossa, as well as further delineating mass lesions. Contrast is useful in evaluating
conditions associated with the breakdown of the blood–brain barrier such as neoplasm or
infection.

Lumbar puncture (LP) can be both a diagnostic and therapeutic procedure. Typically,
neuroimaging is performed before the LP to exclude a mass that, if present, can produce a
pressure gradient leading to herniation. In the setting of suspected meningitis or encephalitis, if
the LP is delayed to obtain neuroimaging, antibiotics should be empirically started to avoid
treatment delay. An opening pressure measurement can be obtained with the LP. Normal
cerebrospinal fluid (CSF) opening pressure in children is between 12 and 28 cm of water; the
normal range is impacted by age, body habitus, and level of sedation. In the setting of idiopathic
intracranial hypertension (IIH), removal of CSF can provide temporary symptom relief which
simultaneously supports the diagnosis.

American Academy of Pediatrics 266


PREP® Self-Assessment PREPSA 2023

Although placement of a ventriculoperitoneal shunt may be necessary for management of the


adolescent in the vignette’s ICP, her condition is subacute to chronic and diagnostic evaluation
with brain imaging is a more appropriate next step. Ophthalmologists are often involved in the
acute and long-term care of children and adolescents with IIH. However, neuroimaging is a
higher priority for the adolescent in the vignette.

PREP Pearls
• The neurological history and physical examination findings are critical in identifying
increased intracranial pressure. The presenting signs and symptoms can be acute,
subacute, or chronic depending on the underlying cause.
• Herniation syndromes are neurological emergencies. They should be diagnosed clinically
and treated without delay, prior to obtaining definitive diagnostic testing.
• After clinical stabilization, neuroimaging is often the initial diagnostic test to identify an
underlying cause of increased intracranial pressure and to assess for mass effect. Lumbar
puncture, which allows for measurement of intracranial pressure, cerebrospinal fluid
analysis, and can provide temporary symptom relief, may be delayed until neuroimaging
is performed.

American Academy of Pediatrics 267


PREP® Self-Assessment PREPSA 2023

ABP Content Specifications(s)


• Recognize the clinical findings associated with increased intracranial pressure in patients
of various ages
• Plan the appropriate diagnostic evaluation of increased intracranial pressure, and manage
appropriately
• Understand the indications and contraindications for examination of the cerebrospinal
fluid in a patient who has increased intracranial pressure

Suggested Readings
• Aylward S, Reem R. Pediatric intracranial hypertension. Pediatr Rev. 2018;39(3):121-
129. doi:10.1542/pir.2016-0226.
• Klein J, Koch T. Headache in children. Pediatr Rev. 2020;41(4):159-171.
doi:10.1542/pir.2017-0012.
• Kochanek PM, Tasker RC, Carney N, et al. Guidelines for the management of pediatric
severe traumatic brain injury, third edition: update of the brain trauma foundation
guidelines. Pediatr Crit Care Med. 2019;20(3S Suppl 1):S1-S82.
doi:10.1097/PCC.0000000000001735.
• Kukreti V, Mohseni-Bod H, Drake J. Management of raised intracranial pressure in
children with traumatic brain injury. J Pediatr Neurosci. 2014;9(3):207-215.
doi:10.4103/1817-1745.147572.
• Shah M, Smyth M. Increased intracranial pressure. In: McInerny TK, Adam HM,
Campbell DE, DeWitt TG, Foy JM, Kamat DM, eds. American Academy of Pediatrics
Textbook of Pediatric Care. American Academy of Pediatrics; 2021:chap 364.
• Accessed September 1, 2022. Pediatric Care Online.

American Academy of Pediatrics 268


PREP® Self-Assessment PREPSA 2023

Question 80
A 7-year-old-girl is brought to the emergency department for evaluation of severe abdominal
pain, nausea, vomiting, and muscle cramping. She was feeling well earlier in the day and had
helped her family clean the garage. On physical examination, her temperature is 36.7°C, heart
rate is 128 beats/min, respiratory rate is 22 breaths/min, and blood pressure is 130/80 mm Hg.
She is lying in the fetal position, crying, and appears to be in pain. There is a 2-cm, circular
lesion with central pallor and a central punctum on her right hand. The remainder of her physical
examination findings are unremarkable.

Of the following, the envenomation MOST likely responsible for this girl’s symptoms is a
A. black widow spider bite
B. brown recluse spider bite
C. emperor scorpion sting
D. yellow jacket sting

American Academy of Pediatrics 269


PREP® Self-Assessment PREPSA 2023

Correct Answer: A
The girl in the vignette has signs and symptoms concerning for envenomation from a black
widow spider (Item C80, Latrodectus) bite. Black widow spider venom is neurotoxic and causes
release of acetylcholine and norepinephrine at the nerve terminals. Signs and symptoms begin
suddenly including nausea, vomiting, pain and pallor at the bite site, paresthesias, tenderness in
regional lymph nodes, sweating, abdominal pain, and muscle cramping (abdomen, flank, chest,
thighs). Severe signs and symptoms include muscle rigidity, difficulty speaking, ptosis, agitation,
hypertension, and respiratory distress. Treatment of a black widow envenomation includes
cleansing the bite wound with soap and water, hydration, tetanus immunization, and analgesia.
Intravenous opioids for pain control and benzodiazepines for anxiolysis and smooth muscle
relaxation may be necessary for severe symptoms. For severe hypertension, intravenous anti-
hypertensive medications should be given. Anti-venom should be administered for life-
threatening hypertension, cardiac, or respiratory compromise. As the anti-venom is equine
derived, children should be tested for horse serum hypersensitivity prior to administration.

Venom from the brown recluse (Loxosceles) spider is a cytotoxin with localized effects; it rarely
produces systemic symptoms. Signs and symptoms include redness and itching around the bite
site which can progress to swelling, pain, vesicles, or hemorrhagic blisters. Local ischemia and
tissue necrosis can result in the formation of a progressively enlarging necrotic ulcer over several
weeks after the initial envonomation. Uncommon systemic manifestations include fever, chills,
malaise, vomiting, diarrhea, and arthralgias. Severe envenomation may result in coagulopathy;
symptoms may include hemolytic anemia, disseminated intravascular coagulation, jaundice,
renal failure, seizures, and shock.

Treatment of brown recluse spider envenomation is primarily aimed at good wound care
(washing the site with soap and water and debriding necrotic tissue), and tetanus immunization.
Antibiotics should only be used if there are signs and symptoms of cellulitis. Consultation with a
wound care specialist is indicated if the wound becomes large,
disfiguring, or is located in an area that may compromise functionality if scarring occurs.
Prophylactic antibiotics, systemic corticosteroids, dapsone, metronidazole, colchicine, surgical
excision, and hyperbaric oxygen therapy have not demonstrated treatment efficacy and should
not be routinely used.

The emperor scorpion is non-venomous; a sting may cause brief pain and redness at the site. The
bark scorpion is the only venomous scorpion, producing a potent neurotoxin. A yellow jacket
sting usually causes pain, redness, and swelling at the site but can progress to signs and
symptoms of anaphylaxis in some children.
American Academy of Pediatrics 270
PREP® Self-Assessment PREPSA 2023

PREP Pearls
• Black widow spider venom is neurotoxic. Signs and symptoms begin suddenly, and may
include nausea, vomiting, pain and pallor at the bite site, paresthesias, tenderness in
regional lymph nodes, sweating, abdominal pain, and muscle cramping.
• Severe signs and symptoms of a black widow spider bite include muscle rigidity,
difficulty speaking, ptosis, agitation, hypertension, and respiratory distress.
• Brown recluse spider venom is a cytotoxin that produces localized effects; systemic
symptoms are rare. Treatment is aimed at localized wound care.

ABP Content Specifications(s)


• Plan the appropriate management of a spider bite

Suggested Readings
• Banerji S, Bronstein AC. Envenomations. In: McInerny TK, Adam HM, Campbell DE,
DeWitt TG, Foy JM, Kamat DM, eds. American Academy of Pediatrics Textbook of
Pediatric Care. American Academy of Pediatrics; 2021:chap 358. Accessed September 1,
2022. Pediatric Care Online.
• Bond GR. Snake, spider, and scorpion envenomation in North America. Pediatr Rev.
1999;20(5):147-151. doi:10.1542/pir.20.5.147.

American Academy of Pediatrics 271


PREP® Self-Assessment PREPSA 2023

Question 81
An 18-month-old boy is brought to the emergency department by his parents after they found
him in the garage coughing and sputtering next to an open bottle of lamp oil. Prior to arrival his
parents removed the clothing that was soiled with lamp oil. The boy is awake, alert, and
interactive. His vital signs include a heart rate of 125 beats/min, respiratory rate of 35
breaths/min, and blood pressure of 85/60 mm Hg. He has audible wheezing but no stridor and is
in mild distress. The boy’s oral mucosa is pink and moist with no lesions. His skin is warm and
well perfused, and he does not have a rash. The remainder of the boy’s physical examination
findings are normal. A chest radiograph is obtained and shown in Item Q81.

Item Q81: Chest radiograph for the boy in the vignette.


Courtesy of J. Kane

Of the following, the BEST intervention for this boy is the administration of
A. activated charcoal
B. corticosteroids
C. ipecac syrup
D. oxygen

American Academy of Pediatrics 272


PREP® Self-Assessment PREPSA 2023

Correct Answer: D
The boy in the vignette has ingested and aspirated lamp oil, which contains hydrocarbons. Given
that the boy has respiratory distress and bilateral infiltrative disease on his chest radiograph,
oxygen should be administered immediately, and he should be admitted to the hospital for
further observation and management.

Hydrocarbons are organic compounds containing hydrogen and carbon atoms. They are found in
many common household chemical-based cleaners and fuels. Examples of items containing
hydrocarbons include industrial and model glues, nail polishes, paints and paint solvents, pine
oil, gasoline, kerosene, furniture polishes, lamp oil, and lighter fluid.

The risk of toxicity from ingestion or aspiration of hydrocarbon-containing substances is related


to both the quantity of the ingestion and the chemical characteristics of the substance itself.
Hydrocarbons with low viscosity and low surface tension (eg, lamp oil) are most likely to result
in aspiration.

When ingested, hydrocarbons can cause systemic toxicity and affect multiple organ systems. The
most common and severe systemic effects of ingested hydrocarbons include cardiac arrhythmias
secondary to myocardial sensitization, neurologic depression, encephalopathy, acute liver injury,
and acute renal tubular necrosis.

In children with unintentional hydrocarbon ingestion, pulmonary aspiration is much more


common than systemic toxicity. Respiratory distress develops as a result of direct lung injury.
Low viscosity and low surface tension of aspirated hydrocarbons contribute to chemical
pneumonitis. The main clinical complication is severe necrotizing pneumonia. Other pulmonary
complications include atelectasis, interstitial inflammation, and formation of hyaline
membranes. Fever often results from the inflammatory response to chemical irritation in the
lungs. Affected children are also at risk of bacterial superinfection; development of fever outside
of the acute post-ingestion period should raise suspicion for a new infection.

The acute management of a child with hydrocarbon poisoning depends on the degree of
symptoms, particularly the development of respiratory distress or altered mental status. External
decontamination, which includes removing all contaminated clothing and cleaning affected hair
and skin to reduce risk of additional irritation and inhalation, should be performed as soon as
possible. Asymptomatic children seen in the emergency department after hydrocarbon exposure
with normal vital signs should be observed for at least 6 hours post exposure before discharge.
Children with any respiratory symptoms should receive oxygen and a trial of a β2-agonist if
there are signs of bronchospasm.

Chest radiography should be performed. Children with any respiratory or neurological symptoms
should be admitted to the hospital for observation and supportive care; they may progress to
respiratory failure requiring endotracheal intubation and mechanical ventilation. In severe cases,
extracorporeal membrane oxygenation has been used to treat refractory hypoxemia and severe
acute respiratory distress syndrome.
American Academy of Pediatrics 273
PREP® Self-Assessment PREPSA 2023

Gastric lavage, induced emesis with ipecac administration, and decontamination with activated
charcoal are not recommended after hydrocarbon ingestion because of the increased risk of
vomiting and additional pulmonary aspiration. There are no compelling data to support the use of
prophylactic antibiotics to prevent infection or corticosteroids to reduce the inflammatory
response.

PREP Pearls
• Aspiration of hydrocarbon-containing substances is related to the hydrocarbon’s low
viscosity and low surface tension.
• Asymptomatic children seen after hydrocarbon exposure should be observed for at least 6
hours with serial examinations; if they remain asymptomatic they can be safely
discharged to home.
• Children with any respiratory symptoms after hydrocarbon exposure should receive
oxygen and a trial of β2-agonists if there are signs of bronchospasm. Chest radiography
should be performed.
• Neither prophylactic antibiotics nor systemic corticosteroids should be administered
routinely in pediatric hydrocarbon aspiration.

ABP Content Specifications(s)


• Plan the management of a patient who has ingested or aspirated a substance containing
hydrocarbons

Suggested Readings
• Fine JS. Poisoning. In: McInerny TK, Adam HM, Campbell DE, DeWitt TG, Foy JM,
Kamat DM, eds. American Academy of Pediatrics Textbook of Pediatric Care. American
Academy of Pediatrics; 2021:chap 369. Accessed September 1, 2022. Pediatric Care
Online.
• Jolliff HA, Fletcher E, Roberts KJ, Baker SD, McKenzie LB. Pediatric hydrocarbon-
related injuries in the United States: 2000–2009. Pediatrics. 2013;131(6):1139-1147.
doi:10.1542/peds.2012-3913.
• Makrygianni, EA, Palamidou, F, Kaditis, AG. Respiratory complications following
hydrocarbon aspiration in children. Pediatr Pulmonol. 2016;51(6):560-569.
doi:10.1002/ppul.23392.
• Victoria, MS, Nangia, BS. Hydrocarbon poisoning: a review. Pediatr Emerg Care.
1987;3(3):184-186. doi:10.1097/00006565-198709000-00014.

American Academy of Pediatrics 274


PREP® Self-Assessment PREPSA 2023

Question 82
A 16-month-old girl is evaluated in the emergency department for a rash and difficulty breathing.
She was in her usual state of good health until, while eating a peanut butter and jelly sandwich,
she suddenly developed a cough, swelling of her lips, and a rash. On physical examination, she
has a heart rate of 140 beats/min, respiratory rate of 30 breaths/min, blood pressure of 70/40 mm
Hg, and an oxygen saturation of 82% in room air. She has swelling of her lips, an urticarial rash,
and diffuse wheezing on chest auscultation.

Of the following, this girl’s allergic reaction is BEST described as


A. type 1, IgE mediated
B. type 2, cytotoxic
C. type 3, immune complex mediated
D. type 4, T-cell mediated

American Academy of Pediatrics 275


PREP® Self-Assessment PREPSA 2023

Correct Answer: A
The girl in the vignette had an immediate reaction to the food she was consuming (most likely
peanut butter), manifesting as urticaria, wheezing, and hypotension. These findings are
consistent with anaphylaxis, a type 1, IgE-mediated allergic reaction. Type 2 reactions are
antibody driven (eg, autoimmune hemolytic diseases). Type 3 reactions are caused by
antigenantibody complexes (eg, drug induced serum sickness). Type 4 reactions are cell
mediated and often described as a delayed reaction (eg, proctocolitis in infants).
Food allergy is a growing concern around the world, especially in industrialized countries such
as the United States. It is estimated that 8% of children have a food allergy, 3% have experienced
an anaphylactic reaction, and over 2% have allergies to more than one food. The most common
foods involved are milk, soy, and egg; peanut, seafood, wheat, and tree nuts account for most of
the remaining food allergens.

Food allergies are categorized as type 1 (IgE-mediated) reactions, type 4 (non-IgE mediated)
reactions, or combined reactions (Item C82).

IgE-mediated (type 1) reactions manifest with anaphylaxis and urticaria. This reaction requires
exposure to the food for sensitization; it occurs upon repeat exposure. During a type 1 reaction,
mast cells and basophils release histamine, tryptase, prostaglandins, and leukotrienes leading to
the development of hives, wheezing, and hypotension.

Pollen-food syndrome is characterized by an IgE-mediated mucosal reaction after eating raw


foods. The pollen contained within the raw food stimulates local release of inflammatory
mediators through the type 1 pathways.

However, when the food reaches the stomach, acid neutralizes the pollen, and there are no
systemic symptoms. Cooking the food prevents this local reaction.

Serum IgE levels or skin prick testing are the tests of choice for diagnosing IgE-mediated
allergies. The class of reaction or size of the wheal determines the likelihood of an allergy but
does not correlate with the severity of allergic reaction. Serum IgE panels can show cross
reactivity and are more sensitive than they are specific. If a test result is negative, there is a 90%
chance the child does not have an allergy to foods included in the panel.

Delayed hypersensitivity (type 4) allergic reactions are non-IgE-mediated reactions. Examples of


type 4 reactions include proctocolitis of infancy and food-protein-induced enterocolitis
syndrome. Proctocolitis is often seen in infancy, due to an inflammatory response to soy or cow
milk protein; eosinophils are present in the colon and rectum. Proctocolitis presents with bloody
stools and can cause fussiness, but does not lead to weight loss or systemic symptoms. Food-
protein-induced enterocolitis syndrome is also commonly seen in infancy, due to soy or cow
milk protein. It affects the small intestines and can lead to flattened villi with subsequent
decreased nutrient absorption. These conditions are diagnosed clinically, based on history and
physical examination findings; they are treated by avoiding the offending agent(s). Children

American Academy of Pediatrics 276


PREP® Self-Assessment PREPSA 2023

often outgrow this sensitivity. Celiac disease (gluten sensitivity) is a type 4 allergic reaction
managed by removal of gluten-containing products from the diet; it is a lifelong condition.

A combination of IgE and non-IgE-mediated inflammatory effects are seen in eosinophilic


esophagitis, gastroenteritis, and atopic dermatitis.

PREP Pearls
• The most common food allergies include milk, soy, and egg; peanut, seafood, wheat, and
tree nuts account for most of the remaining allergens.
• Serum IgE levels or skin prick testing are the tests of choice for diagnosing IgE-mediated
allergies.

MOCA-Peds Objective
• Know the indications and limitations of allergy testing (including skin and in vitro tests).

ABP Content Specifications(s)


• Understand the natural history of various food allergies
• Know the common foods that cause allergic reactions

Suggested Readings
• Cosme-Blanco W, Arroyo-Flores E, Ale H. Food allergies. Pediatr Rev. 2020;41(8):403-
415. doi:10.1542/pir.2019-0037.
• Food protein-induced enterocolitis syndrome. National Organization for Rare Disorders.
2019. Accessed February 11, 2022. https://rarediseases.org/rare-diseases/food-protein-
induced-enterocolitis-syndrome/.
• Le-Carlson M, Kerner JA. Gastrointestinal allergy. In: McInerny TK, Adam HM,
Campbell DE, DeWitt TG, Foy JM, Kamat DM, eds. American Academy of Pediatrics
Textbook of Pediatric Care. American Academy of Pediatrics; 2021:chap 256. Accessed
September 1, 2022. Pediatric Care Online.
• Second E, Simon MR. Anaphylaxis. In: McInerny TK, Adam HM, Campbell DE, DeWitt
TG, Foy JM, Kamat DM, eds. American Academy of Pediatrics Textbook of Pediatric
Care. American Academy of Pediatrics; 2021:chap 350. Accessed September 1, 2022.
Pediatric Care Online.

American Academy of Pediatrics 277


PREP® Self-Assessment PREPSA 2023

Question 83
A 12-year-old boy is seen for concerns about weight gain and fatigue. His parents are worried
that he has a thyroid problem and request laboratory evaluation. Review of systems is significant
for bilateral knee pain. Family history reveals a parent who underwent bariatric surgery. The
boy’s vital signs are normal except for a blood pressure of 132/92 mm Hg. His growth charts are
shown in Item Q83A and Item Q83B. The boy’s body mass index is 24.8 kg/m2 (96th
percentile). He has acanthosis nigricans on his posterior neck and thin, flesh-colored striae on
both flanks. Sexual maturity rating is 2 for pubic hair and genital development. The remainder of
his physical examination findings are unremarkable.

Of the following, the BEST next management step for this boy is to
A. begin treatment with levothyroxine
B. begin treatment with liraglutide
C. discuss intensive lifestyle modification
D. refer him for bariatric surgery

American Academy of Pediatrics 278


PREP® Self-Assessment PREPSA 2023

Correct Answer: C
The boy in the vignette has obesity with a body mass index (BMI) greater than the 95th
percentile for age and sex. The best next management step is to discuss intensive lifestyle
modification. Although the boy’s family is concerned about thyroid disease, his normal linear
growth makes this diagnosis unlikely. Current guidelines regarding the evaluation of obesity
recommend measuring thyroid function only in the setting of slow growth. Levothyroxine has
not been shown to reduce weight and is inappropriate to prescribe for this boy for the purpose of
weight loss.

In 2014, the prevalence of obesity in children and adolescents was estimated to be 17%. Pediatric
obesity is typically due to genetic predisposition exacerbated by environmental factors.
Endocrine causes of obesity, such as growth hormone deficiency, hypothyroidism, and Cushing
syndrome, are rare and are typically accompanied by poor linear growth. The first-line treatment
of pediatric obesity is intensive lifestyle modification. A multifaceted approach that involves the
entire family and includes healthy eating habits, increased exercise, decreased inactivity (eg,
nonacademic screen time), and psychological support is optimal. It has been proposed that
pharmacological therapy be limited to those with 1) a BMI greater than or equal to 95th
percentile with comorbidity or 2) a BMI greater than or equal to 120% of the 95th percentile.
Liraglutide is a glucagon-like peptide 1 (GLP-1) receptor agonist that was approved by the US
Food and Drug Administration (FDA) in 2020 for treatment of obesity in children and
adolescents aged 12 years and older.

Glucagon-like peptide 1 receptor agonists work to decrease food intake by acting on the appetite
centers in the brain and slowing gastric emptying. Liraglutide is recommended as adjunctive
therapy for those who have not been successful in losing weight with lifestyle intervention alone.
The most common side effects include nausea and vomiting. Treatment is contraindicated in
those with pancreatitis or a personal or family history of medullary thyroid carcinoma.
Orlistat is a gastrointestinal lipase inhibitor that reduces intestinal fat absorption. Prior to the
2020 FDA-approval of liraglutide, it was the only FDA-approved medication for treatment of
obesity in pediatrics. Although it has been demonstrated to reduce BMI, diarrhea is a major side
effect that has limited its use.

Bariatric surgery is reserved for adolescents with a BMI >40 kg/m2 or over 35 kg/m2 with
severe comorbidities such as type 2 diabetes or obstructive sleep apnea. Surgery is limited to
those who have completed puberty, been unsuccessful with lifestyle modification with or without
pharmacotherapy, undergone psychological evaluation, and demonstrated adherence to healthy
dietary and activity habits.

PREP Pearls
• Intensive lifestyle modification is the first-line therapy for children and adolescents with
obesity.
• Endocrine causes of obesity are rare, and evaluation should only be pursued in the setting
of weight gain and disrupted linear growth or developmental concerns.

American Academy of Pediatrics 279


PREP® Self-Assessment PREPSA 2023

• Pharmacologic and surgical interventions for pediatric obesity should be limited to those
with severe obesity and comorbidities.

ABP Content Specifications(s)


• Counsel a family with regard to obesity prevention and treatment
• Know which interventions have been effective and ineffective in caring for patients of
various ages with obesity

Suggested Readings
• Srivastava G, Fox CK, Kelly AS, et al. Clinical considerations regarding the use of
obesity pharmacotherapy in adolescents with obesity. Obesity. 2019; 27(2):190-204.
doi:10.1002/oby.22385.
• Styne DM, Arslanian SA, Connor EL, et al. Pediatric obesity—assessment, treatment,
and prevention: an Endocrine Society Clinical Practice Guideline. J Clin Endocrinol
Metab. 2017;102(3):709-757. doi:10.1210/jc.2016-2573.
• Wang YC, Gahagan S. Obesity and metabolic syndrome. In: McInerny TK, Adam HM,
Campbell DE, DeWitt TG, Foy JM, Kamat DM, eds. American Academy of Pediatrics
Textbook of Pediatric Care. American Academy of Pediatrics; 2021:chap 298. Accessed
September 1, 2022. Pediatric Care Online.

American Academy of Pediatrics 280


PREP® Self-Assessment PREPSA 2023

Question 84
Two hours after delivery, a neonate born at 39 weeks’ gestation has a choking episode while
breastfeeding. On physical examination, the neonate is gagging on oral secretions. An orogastric
tube is placed and a chest radiograph is obtained (Item Q84). The prenatal history and delivery
were uncomplicated other than significant polyhydramnios.

Of the following, evaluation of this neonate will MOST likely include


A. computed tomography of the chest
B. echocardiography
C. esophagoscopy
D. upper gastrointestinal tract contrast study

American Academy of Pediatrics 281


PREP® Self-Assessment PREPSA 2023

Correct Answers: B
The neonate in the vignette is choking with feeding, gagging on oral secretions, and has a
prenatal history of polyhydramnios. This clinical picture is suggestive of a diagnosis of
esophageal atresia (EA) with or without tracheoesophageal fistula (TEF), which is confirmed on
chest radiography showing the orogastric tube terminating at the cervical level of the esophagus
(EA) and air in the stomach (TEF). Radiography of isolated esophageal atresia will not show air
in the stomach.

Approximately 50% of neonates with TEF/EA will have associated vertebral, anorectal, cardiac,
renal and limb anomalies. This association is given the acronym VACTERL. It is important to
look for these other anomalies in patients with TEF/EA due to their potential impact (especially
cardiac anomalies) on the neonate’s morbidity risk. In addition to a thorough physical
examination, diagnostic evaluation for neonates with TEF/EA should include echocardiography,
renal ultrasonography, and spinal ultrasonography. While vertebral anomalies may be seen on
plain radiography, spinal ultrasonography should be performed to rule out a tethered spinal cord.
Plain chest radiography with an orogastric tube is the appropriate imaging to confirm the
diagnosis of EA/TEF. Upper gastrointestinal tract contrast study, esophagoscopy, and computed
tomography of chest are not warranted in this situation.

PREP Pearls
• Tracheoesophageal fistula or esophageal atresia should be considered in a newborn with
choking episodes, increased oral secretions, and a prenatal history of polyhydramnios.
• Chest radiography showing an orogastric tube terminating at the cervical level of the
esophagus is diagnostic for esophageal atresia; if there is also air in the stomach, the
diagnosis is tracheoesophageal fistula.
• Fifty percent of newborns with tracheoesophageal fistula or esophageal atresia have
associated vertebral, anorectal, cardiac, renal, and limb anomalies. This is known as
VACTERL association.

ABP Content Specifications(s)


• Recognize the clinical and laboratory features associated with tracheoesophageal fistula
in a newborn infant
• Recognize other anomalies associated with tracheoesophageal fistula in the newborn
infant

American Academy of Pediatrics 282


PREP® Self-Assessment PREPSA 2023

Suggested Readings
• Keckler SJ, St Peter SD, Valusek PA, et al. VACTERL anomalies in patients with
esophageal atresia: an updated delineation of the spectrum and review of the literature.
Pediatr Surg Int. 2007;23(4):309-313. doi:10.1007/s00383007-1891-0.
• Lee S. Basic knowledge of tracheoesophageal fistula and esophageal atresia. Adv
Neonatal Care. 2018;18(1):14-21.
• doi:10.1097/ANC.0000000000000464.
• Morris MW, Blewett CJ. Tracheoesophageal fistula. NeoReviews. 2017;18(8):e472-e479.
doi:10.1542/neo.18-8-e472.
• Solomon BD. VACTERL/VATER Association. Orphanet J Rare Dis. 2011;6:56.
doi:10.1186/1750-1172-6-56.

American Academy of Pediatrics 283


PREP® Self-Assessment PREPSA 2023

Question 85
A 15-year-old adolescent girl is seen in the emergency department for dyspnea, dizziness, and
tingling of her fingers. She has known asthma and has used her albuterol inhaler several times in
the past hour without benefit. She has no other known health problems and takes no regular
medications. The adolescent plays volleyball, and her team has just lost a hotly contested game
to a cross town rival. She denies any specific triggers preceding her difficulty breathing.
On physical examination, the adolescent appears anxious but not in distress. Her temperature is
36.8°C, heart rate is 120 beats/min, blood pressure 90/65 mm Hg, respiratory rate is 30
breaths/min, and oxygen saturation by pulse oximeter is 99% in room air. Her pupils are round
and reactive to light. There is no nasal flaring or use of accessory muscles. She is mildly
hyperpneic. There is good air exchange with no stridor or wheeze. Inspiratory-to-expiratory ratio
is 1:1. The remainder of her physical examination findings are normal.

Of the following, the test MOST likely to confirm the diagnosis is


A. capillary blood gas analysis
B. peak expiratory flow rate
C. serum drug screen
D. serum electrolyte panel

American Academy of Pediatrics 284


PREP® Self-Assessment PREPSA 2023

Correct Answer: A
The adolescent in the vignette has hyperventilation syndrome. Her visible anxiety, hyperpnea,
mild tachypnea, and paresthesias without other respiratory abnormalities in the context of an
emotional trigger (loss of an important volleyball game), strongly suggest hyperventilation as the
underlying cause of her symptoms. Hyperventilation causes an acute respiratory alkalosis. The
alkalosis causes a decreased ionized calcium level (due to increased protein binding of serum
calcium, decreasing the free component), which manifests as paresthesias. A diagnosis of
hyperventilation is often apparent from the history and physical examination findings, in which
case no additional testing is needed. However, if the diagnosis is unclear, a capillary blood gas
analysis would be the most informative test, showing an elevated pH and decreased PCO2 level.

Peak expiratory flow rate (PEFR) is an effort-dependent test; decreased airflow demonstrated by
an individual with hyperventilation syndrome may not represent airway obstruction. The girl in
the vignette has no physical examination findings (wheeze, prolonged expiration) consistent with
an acute asthma exacerbation. There are no mental status or physical examination findings to
suggest substance use; if performed, a serum drug screen would not likely give helpful
information. There are no findings to suggest an electrolyte abnormality or an acid-base
imbalance other than acute respiratory alkalosis. It is possible that the girl’s serum bicarbonate
(HCO3-) would be decreased as metabolic compensation if there was prolonged respiratory
alkalosis. However, a capillary blood gas analysis would provide more direct evidence of
respiratory alkalosis with an increased pH and a low PCO2.

Capillary blood gas analysis can be used to evaluate a child’s acid-base status, but is not a good
test for evaluation of oxygenation. While it is generally accepted that venous pH is
approximately 0.05 units higher than the equivalent arterial pH, and venous PCO2 is about 5 mm
Hg higher than the corresponding arterial value, there is no currently accepted factor for
translation of venous or capillary PO2 to an arterial equivalence. Some use arterialized capillary
blood gas determinations drawn from a fingertip or ear lobe (after warming the site to increase
arterial mixture) in an attempt to mimic arterial blood gas determination by a less invasive route
than arterial puncture; however, there is significant potential for error in interpretation of
oxygenation status. Direct arterial samples are the most reliable for this assessment.

PREP Pearls
• A capillary blood gas analysis can accurately estimate acid-base balance but not
oxygenation status.
• Respiratory alkalosis is the physiologic response to hyperventilation.
• Hyperventilation syndrome can usually be diagnosed by history and physical examination
findings; if the diagnosis is unclear, a capillary blood gas analysis is the most informative
test.

ABP Content Specifications(s)


• Understand the limitations of capillary blood gas analysis

American Academy of Pediatrics 285


PREP® Self-Assessment PREPSA 2023

Suggested Readings
• Hsu BS, LakhaniSA, Wilhelm M. Acid-base disorders. Pediatr Rev. 2016;37(9):361-367.
doi:10.1542/pir.2015-0093.
• Kongstad HK, Rosendal CAH, Rasmussen BS, Weinreich UM. Agreement between
arterial and non-arterialized fingertip capillary blood gas and acid-base values. Eur Clin
Respir J. 2019;6(1):1644892. doi:10.1080/20018525.2019.1644892.
• Mahajan P, Felt JR. Fluids, electrolytes, and acid-base composition. In: McInerny TK,
Adam HM, Campbell DE, DeWitt TG, Foy JM, Kamat DM, eds. American Academy of
Pediatrics Textbook of Pediatric Care. American Academy of Pediatrics; 2021:chap 58.
Accessed September 1, 2022. Pediatric Care Online.

American Academy of Pediatrics 286


PREP® Self-Assessment PREPSA 2023

Question 86
A 15-year-old adolescent girl is seen for evaluation of neck enlargement, which she first noticed
several months ago. Her neck has not been red or tender to touch. She has not had a recent fever
or illness, difficulty swallowing, fatigue, palpitations, unexpected weight changes, or menstrual
irregularity. Her vital signs are normal for age. On physical examination, the adolescent’s thyroid
gland is asymmetrically enlarged with a firm nodule palpated on the left when she swallows.
There is no cervical lymphadenopathy. The remainder of her physical examination findings are
unremarkable. A thyroid-stimulating hormone level is normal, and thyroid ultrasonography is
performed (Item Q86).

Of the following, the BEST next step in this adolescent’s management is


A. fine-needle aspiration and biopsy of the nodule
B. reassurance and repeat physical examination in 6 months
C. repeat ultrasonography in 6 months
D. treatment with radioactive iodine therapy

American Academy of Pediatrics 287


PREP® Self-Assessment PREPSA 2023

Correct Answer: A
The adolescent in the vignette has a 2.1-cm, left-sided thyroid nodule and a normal thyroid-
stimulating hormone (TSH) level. The best next step in management is a fine-needle aspiration
and biopsy (FNAB) of the nodule to determine if it is malignant. Thyroid ultrasonography and
thyroid function tests are warranted when a thyroid nodule is palpated or there is thyroid
asymmetry. Current guidelines recommend ultrasound-guided FNAB for nodules ≥1 cm with a
solid component, or nodules with suspicious ultrasonographic features (hypoechogenicity,
irregular margins, increased intranodular blood flow, microcalcifications, abnormal cervical
lymph nodes) when the TSH level is non-suppressed. If the TSH level is suppressed, the best
next step would be nuclear thyroid scintigraphy to evaluate for a hyperfunctioning nodule.

Given the size (>1 cm) of the solid nodule for the adolescent in the vignette, reassurance and
reexamination in 6 months or repeat ultrasonography in 6 months without further evaluation
would not be appropriate. Reassurance may be appropriate for nodules that are small (<1 cm)
and without suspicious features on ultrasonography, or nodules that are cystic.

Subsequent management of thyroid nodules is based on results of the FNAB. Benign nodules are
followed by ultrasonography to monitor for concerning changes. Total thyroidectomy is
recommended for children and adolescents with biopsy results suspicious for or consistent with
malignancy. Subsequent need for radioactive iodine therapy is determined by postoperative
disease staging. Radioactive iodine is also used to treat hyperthyroidism due to Graves disease. It
is not first-line therapy for thyroid nodules.

PREP Pearls
• Thyroid ultrasonography and thyroid function tests are warranted when a thyroid nodule
is palpated or there is thyroid asymmetry.
• Fine-needle aspiration and biopsy of a thyroid nodule is indicated if the thyroid-
stimulating hormone level is not suppressed and the nodule is  ≥1 cm in size with a solid
component, or suspicious features are present on ultrasonography (hypoechogenicity,
irregular margins, increased intranodular blood flow, microcalcifications, abnormal
cervical lymph nodes).
• Reassurance and repeat ultrasonography in 6 to 12 months may be appropriate for thyroid
nodules that are cystic or small (<1 cm) and without suspicious features on
ultrasonography.

ABP Content Specifications(s)


• Recognize the clinical features associated with a thyroid cyst/tumor
• Plan the appropriate evaluation and management of a thyroid mass/nodule

American Academy of Pediatrics 288


PREP® Self-Assessment PREPSA 2023

Suggested Readings
• Allen-Rhoades W, Whittle SB, Rainusso N. Pediatric solid tumors in children and
adolescents: an overview. Pediatr Rev. 2018;39(9):444-453. doi:10.1542/pir.2017-0268.
• Francis GL, Waguespack SG, Bauer AJ, et al. Management guidelines for children with
thyroid nodules and differentiated thyroid cancer. Thyroid. 2015; 25(7):716-759.
doi:10.1089/thy.2014.0460.
• Osipoff JN, Wilson TA. Consultation with the specialist: thyroid nodules. Pediatr Rev.
2012;33(2):75-81. doi:10.1542/pir.33-2-75.

American Academy of Pediatrics 289


PREP® Self-Assessment PREPSA 2023

Question 87
A 2-year-old girl is seen for a health supervision visit. She has normal growth parameters and
development. Her physical examination findings are remarkable only for a short neck with low
posterior hairline and reduced movement at the neck. Cervical spine radiographs show fusion of
the C2-C3 vertebrae.

Of the following, the girl’s MOST likely diagnosis is


A. Klippel-Feil syndrome
B. Pierre-Robin sequence
C. Turner syndrome
D. VACTERL association

American Academy of Pediatrics 290


PREP® Self-Assessment PREPSA 2023

Correct Answer: A
The girl in the vignette has Klippel-Feil syndrome (KFS). Klippel-Feil syndrome is characterized
by the triad of short neck, reduced neck mobility, and low posterior hairline secondary to
congenital fusion of the cervical vertebrae (2 or more). The frequency of KFS is approximately 1
in 42,000 births; it is more common in females than males. Approximately half of affected
individuals do not present with the classic triad and are identified through an incidental finding
on an imaging study or when they present with acute neurological symptoms. The clinical
findings associated with anomalies of the upper cervical spine often bring affected children to
medical attention sooner. Clinical findings associated with KFS are shown in Item C87.

Upon diagnosis of KFS, anterior-posterior, lateral, flexion-extension, and open-mouth odontoid


view radiographs of the cervical spine should be performed in order to determine the mobility of
each open space. Renal ultrasonography and 2-dimensional echocardiography are recommended
to identify associated congenital anomalies. New-onset pain, trauma, or symptoms of
radiculopathy or neuropathy should prompt magnetic resonance imaging of the entire spine. The
majority of children with KFS do not have any family history and are sporadic in occurrence.
Autosomal dominant and recessive forms of KFS are known to occur.

Pierre-Robin sequence (PRS) is characterized by micrognathia and glossoptosis, with or without


cleft palate. Cervical vertebral fusion anomalies, as seen in the girl in the vignette, are not a
feature of PRS. Common underlying genetic etiologies of PRS include 22q11.2 deletion and
Stickler syndrome.

Turner syndrome is characterized by short stature, webbed neck, dysmorphic facial features
(down-slanted palpebral fissures, low-set ears, depressed nasal bridge), cardiac defects
(coarctation of aorta, bicuspid aortic valve), widespaced nipples, and pectus excavatum. While a
low posterior hairline can be seen in Turner syndrome, cervical fusion anomalies with restricted
neck movement are not a feature of this condition.

American Academy of Pediatrics 291


PREP® Self-Assessment PREPSA 2023

VACTERL is an association of anomalies that includes vertebral defects, anal atresia, cardiac
defects, tracheoesophageal fistula, renal anomalies, and limb defects. VACTERL is a clinical
diagnosis of exclusion that requires involvement of at least 3 of the organ systems included in the
acronym.

PREP Pearls
• Klippel-Feil syndrome (congenital fusion of 2 or more cervical vertebrae) is
characterized by the triad of short neck, reduced neck mobility, and low posterior
hairline. Approximately half of affected individuals do not have the classic triad of
associated features.
• Klippel-Feil syndrome can be associated with congenital anomalies of the brain, spine,
heart, kidneys, and musculoskeletal system.
• Most cases of Klippel-Feil syndrome are sporadic in occurrence.

ABP Content Specifications(s)


• Recognize the clinical and radiologic findings associated with Klippel-Feil syndrome

Suggested Readings
• Frikha R. Klippel-Feil syndrome: a review of the literature. Clin Dysmorphol.
2020;29(1):35-37. doi:10.1097/MCD.0000000000000301.
• Litrenta J, Bi AS, Dryer JW. Klippel-Feil syndrome: pathogenesis, diagnosis, and
management. J Am Acad Orthop Surg. 2021;29(22):951-960. doi:10.5435/JAAOS-D-21-
00190.
• Marion RW, Samanich J. Facial dysmorphism. In: McInerny TK, Adam HM, Campbell
DE, DeWitt TG, Foy JM, Kamat DM, eds. American Academy of Pediatrics Textbook of
Pediatric Care. American Academy of Pediatrics; 2021:chap 148. Accessed September 1,
2022. Pediatric Care Online.

American Academy of Pediatrics 292


PREP® Self-Assessment PREPSA 2023

Question 88
A 2-month-old infant is seen for a routine health supervision visit. She did not pass her newborn
hearing test, and further diagnostic evaluation completed last week confirmed severe bilateral
hearing loss. An audiologist will be fitting her for a hearing aid next month and following her
closely. She may be eligible for cochlear implantation at 24 months of age. The infant is
scheduled for evaluation by an early intervention specialist next week and will then begin
services. She is also scheduled to see a developmental-behavioral pediatrician at 6 months of
age. Genetic testing results are pending.

The infant’s parents are feeling heartbroken and overwhelmed over her diagnosis. There is no
family history of hearing loss, and they have not had any previous contact or exposure to anyone
who is deaf or hard of hearing. Her mother expresses fear about the future including how the
child will manage when she enters school and learns to read.

Of the following, the BEST approach to maximize this infant’s language development is
A. early placement of a hearing amplification device
B. her parents learning American Sign Language
C. initiation of individualized, family-centered early intervention services
D. standardized developmental assessments every 6 months

American Academy of Pediatrics 293


PREP® Self-Assessment PREPSA 2023

Correct Answer: C
The infant in the vignette has severe bilateral hearing loss. The best approach to maximize her
language development is initiation of individualized, family-centered early intervention services.
In addition to the degree or severity of hearing loss and age at identification, the timing of
treatment, including early intervention services, determines prognosis for children with hearing
loss. It is recommended that early intervention services be initiated no later than 6 months of age
and ideally within 1 week of confirmatory testing.

The role of early intervention programs in the care of children with hearing impairment is to
provide services from trained professionals who are experienced working with children with
hearing loss and their families. Interventions should be family centered, provided in a natural
setting, and individualized to the mode of communication preferred by the family; this may be
spoken language, American Sign Language, visually supported spoken language, or a
combination. Early intervention specialists should communicate with members of the child’s
medical team including their audiologist. This dialogue may include feedback on hearing
amplification devices to help optimize usage or monitoring for comorbidities such as visual or
cognitive impairment.

The educational approach for hearing-impaired children is multi-faceted and must begin as soon
as possible after diagnosis. Outcomes in expressive and receptive speech and reading skills at
school age are improved when treatments, including early intervention and hearing amplification
devices, are initiated within the first 3 to 9 months. Without such interventions, decreased
language and communication skills result in lower literacy, limited participation in mainstream
education, and increased risk for grade retention and poorer overall academic achievement.

Hearing aids can provide amplification and increase the range of speech and language sounds a
child is exposed to. The effectiveness of amplification devices has been tied to the number of
hours per day they are used, as well as close monitoring and frequent adjustments to ensure
safety and comfort. Neither amplification device use nor cochlear implantation alone guarantees
language development. Education on all communication modes, including American Sign
Language, should be provided for families to consider. Information on support groups and
connecting with other individuals with hearing loss can be an important component of the
intervention plan. It is recommended that children with hearing impairment have a
developmental assessment with standardized tools to assess cognitive and language abilities
completed through their early intervention program every 6 months during the first 3 years of
life.

Although the implementation of newborn hearing screening has significantly lowered the age at
diagnosis for children with hearing loss, the timing of treatment initiation is of equal importance
to ensure optimal outcomes. Parents and caregivers of children with hearing loss are critical
members of the early intervention team, and the mode of communication must work within the
family system. Longitudinal monitoring during these early years can be facilitated by the
pediatrician, other clinicians (including ear, nose, and throat specialists), and the early
intervention team.
American Academy of Pediatrics 294
PREP® Self-Assessment PREPSA 2023

PREP Pearls
• The prognosis for children with hearing loss depends on the degree of hearing loss and
the timing of diagnosis and treatment.
• Education for children with hearing loss involves early interventions that are
individualized to the families’ preferred mode of communication.
• Developmental assessments utilizing standardized tools to evaluate cognitive and
language abilities should be performed every 6 months during the first 3 years of life in
children living with hearing loss.

ABP Content Specifications(s)


• Understand the major approaches to education for children living with hearing loss

Suggested Readings
• Grindle CR. Pediatric hearing loss. Pediatr Rev. 2014;35(11):456-463
doi:10.1542/pir.35-11-456.
• Joint Committee on Infant Hearing of the American Academy of Pediatrics; Muse C,
Harrison J, Yoshinaga-Itano C, et al. Supplement to the JCIH 2007 position statement:
principles and guidelines for early intervention after confirmation that a child is deaf or
hard of hearing. Pediatrics. 2013;131(4):e1324-e1349. doi:10.1542/peds.2013-0008.
• LeClair KL, Saunders JE. Meeting the educational needs of children with hearing loss.
Bull World Health Organ. 2019;97(10):722-724. doi:10.2471/BLT.18.227561.
• Stewart JE, Bentley JE. Hearing loss in pediatrics: what the medical home needs to know.
Pediatr Clin North Am. 2019;66(2):425-436. doi:10.1016/j.pcl.2018.12.010 .
• Tharpe AM, Sladen DP, Rothpletz A. Hearing loss. In: McInerny TK, Adam HM,
Campbell DE, DeWitt TG, Foy JM, Kamat DM, eds. American Academy of Pediatrics
Textbook of Pediatric Care. American Academy of Pediatrics; 2021:chap 158. Accessed
September 1, 2022. Pediatric Care Online.

American Academy of Pediatrics 295


PREP® Self-Assessment PREPSA 2023

Question 89
A 16-year-old adolescent boy is seen in the emergency department for a 5-day history of
abdominal pain, bloody diarrhea, and fever. The abdominal pain is intermittent and associated
with cramping and nausea. The boy immigrated to the United States from Nicaragua with his
family 2 weeks ago. His family members are experiencing nausea and occasional loose stools.
There is no history of animal bites or exposure to farm animals. His immunizations are up to
date.

On physical examination, the boy has a temperature of 38.5°C, heart rate of 120 beats/min,
respiratory rate of 20 breaths/min, blood pressure of 116/64 mm Hg, and oxygen saturation of
96% in room air. He has lower abdominal tenderness with mild distention. There is no
hepatomegaly or splenomegaly. The remainder of the adolescent’s examination findings are
unremarkable.

Laboratory data are shown:


Laboratory Test Result
White blood cell count 23,000/µL (23.0 × 109/L)
Neutrophils 79%
Bands 9%
Lymphocytes 10%
Monocytes 2%
Hemoglobin 11.0 g/dL (110 g/L)
Platelet count 376 × 103/µL (376 × 109/L)

A comprehensive metabolic profile is normal except for mild hyponatremia.

Of the following, the MOST likely pathogen causing this adolescent’s illness is
A. Entamoeba histolytica
B. enterotoxigenic Escherichia coli
C. Schistosoma mansoni
D. Vibrio cholerae

American Academy of Pediatrics 296


PREP® Self-Assessment PREPSA 2023

Correct Answer: A
The adolescent in the vignette has a clinical presentation that includes acute onset of bloody
diarrhea, crampy abdominal pain, and fever after recently immigrating from Nicaragua. This
presentation is suggestive of amoebic colitis caused by the parasite Entamoeba histolytica.
Enterotoxigenic Escherichia coli (ETEC) typically presents with watery diarrhea and
predominantly affects infants living in under-resourced countries as well as susceptible travelers.
The noninflammatory diarrheal illness associated with ETEC occurs 1 to 3 days after exposure.
Fever is low-grade or absent, and the duration of illness is short (3 to 4 days).

Cholera due to Vibrio cholerae is sporadically reported in the United States in travelers returning
from choleraendemic areas or those who have ingested contaminated food products from
cholera-endemic countries. The clinical presentation of cholera differs from that of the boy in the
vignette. The incubation period is usually 1 to 3 days and characterized by voluminous watery
diarrhea (“rice-water” appearance), emesis, and severe dehydration.

Chronic intestinal schistosomiasis caused by Schistosoma mansoni may result in abdominal pain
and bloody diarrhea and is common in immigrants from sub-Saharan Africa and South America.
Typical findings include hepatosplenomegaly and peripheral eosinophilia, which were not
identified in the adolescent in the vignette.

Amoebiasis due to E histolytica is a leading cause of diarrheal illness globally, primarily


affecting people residing in low-resource countries where sanitation is poor. Amoebiasis is
endemic with a high disease burden in the tropical and subtropical settings of Central and South
America, Africa, and Asia. In the United States and other wellresourced countries, amoebiasis
occurs mostly in travelers or immigrants from endemic nations, institutionalized individuals, and
men who have sex with men. Transmission of E histolytica infection occurs by the fecal-oral
route.

Ingested cysts undergo excystation in the small intestine, releasing trophozoites that migrate to
the large intestine where they produce more cysts by binary fusion. Cysts and trophozoites are
excreted in feces. Amoebic colitis may occur, and the intestinal epithelium may be invaded by
the trophozoites leading to extraintestinal infection (eg, liver infection through the hepatic portal
circulation or hematogenous spread to brain and lung). The amoebic cysts can survive in the
environment for weeks to months and have a low infectious dose, predisposing to disease spread
and community outbreaks. The cysts are resistant to gastric acid and are relatively resistant to
chlorine.

The incubation period of amoebiasis is typically 2 to 4 weeks, but can be highly variable, ranging
from a few days to months or years. Asymptomatic infections are common. Amoebic colitis
typically manifests with diarrhea and crampy lower abdominal pain. The diarrhea is watery or
bloody and may be associated with tenesmus, fever (temperature, >38°C), and weight loss.
Complications of severe disease include colonic perforation, peritonitis, and toxic megacolon.
Amoebic liver abscess may present acutely or have a subacute presentation with fever, right-
upper quadrant abdominal pain, hepatomegaly, and weight loss. Usually, a solitary abscess
American Academy of Pediatrics 297
PREP® Self-Assessment PREPSA 2023

affects the right lobe of the liver. Associated symptoms may include cough and right-sided
pleuritic chest pain.

The differential diagnosis of amoebic colitis includes invasive bacteria that cause bloody
diarrhea (eg, Shigella spp, Salmonella spp, Campylobacter spp, Yersinia spp, Shiga toxin–
producing E coli, and enteroinvasive E coli). Amoebic colitis may mimic inflammatory bowel
disease or acute appendicitis. Amoebic liver abscess cannot be distinguished from pyogenic liver
abscess on imaging studies alone.

A polymerase chain reaction (PCR) test on a stool specimen is highly sensitive (92% to 100%)
and specific (89% to 100%) for E histolytica and is the recommended test to make the diagnosis
of amoebic colitis. Serology using an indirect hemagglutination assay can be a useful adjunct to
the stool PCR assay. Serology has a sensitivity of 70% and 95% for the diagnosis of amoebic
infection in cases of colitis and extraintestinal disease, respectively, with a specificity of more
than 80%. Compared with PCR assay, stool microscopy for detection of trophozoites and cysts
has limited diagnostic usefulness, given its poor sensitivity, specificity, and inability to reliably
distinguish E histolytica from the other non-pathogenic species (eg, Entamoeba dispar spp). The
histologic examination of endoscopic colonic biopsy specimens using special stains may
demonstrate tissue trophozoites. Sigmoidoscopy/colonoscopy may reveal friable colonic mucosa
with discrete ulcers.

Extraintestinal disease (eg, amoebic liver abscess) may be detected with imaging studies (eg,
ultrasonography, abdominal computed tomography scan, magnetic resonance imaging), but
trophozoites are usually not detected in the aspirate obtained from an abscess. Peripheral
leukocytosis is common in children and adolescents with amoebic liver abscess.

Treatment of amoebic colitis is with intravenous (or oral) metronidazole (35-50 mg/kg per day
every 8 hours) or oral tinidazole (age ≥3 years: 50 mg/kg, max 2 g orally, once daily for 5 days).
Following the treatment of acute colitis, children and adolescents should receive a course of an
intraluminal amebicide (eg, iodoquinol) to eradicate luminal cysts.

PREP Pearls
• In the United States and other well-resourced countries, amoebiasis occurs mostly in
travelers or immigrants from endemic nations.
• Amoebic colitis typically presents with crampy lower abdominal pain, watery or bloody
diarrhea, tenesmus, fever (temperature, >38°C), and weight loss.
• Treatment of amoebic colitis is with metronidazole or tinidazole followed by an
intraluminal amoebicide.

ABP Content Specifications(s)


• Understand the epidemiology of amoebiasis
• Recognize the clinical features associated with amoebiasis, and manage appropriately

American Academy of Pediatrics 298


PREP® Self-Assessment PREPSA 2023

Suggested Readings
• Abdel-Haq N, Chearskul P, Rafee Y, Asmar BI. Parasitic infections. In: McInerny TK,
Adam HM, Campbell DE, DeWitt TG, Foy JM, Kamat DM, eds. American Academy of
Pediatrics Textbook of Pediatric Care. American Academy of Pediatrics; 2021;chap 308.
Accessed September 1, 2022. Pediatric Care Online.
• American Academy of Pediatrics. Amoebiasis. In: Kimberlin DW, Barnett ED, Lynfield
R, Sawyer MH, eds. Red Book: 2021–2024 Report of the Committee on Infectious
Diseases. 32nd ed. American Academy of Pediatrics; 2021. Accessed September 1, 2022.
Red Book Online.
• Custodio H. Protozoan parasites. Pediatr Rev. 2016;37(2):59-71. doi:10.1542/pir.2015-
0006.
• Shirley DT, Farr L, Watanabe K, Moonah S. A review of the global burden, new
diagnostic, and current therapeutics for amebiasis. Open Forum Infect Dis.
2018;5(7):ofy161. doi:10.1093/ofid/ofy161.

American Academy of Pediatrics 299


PREP® Self-Assessment PREPSA 2023

Question 90
A 9-week-old female infant (35 weeks’ postconceptual age) in the neonatal intensive care unit is
ready for discharge home. She was born at 26 weeks’ estimated gestational age weighing 750 g.
Her delivery was precipitated by maternal chorioamnionitis. After initial management with nasal
continuous positive airway pressure (CPAP), due to increasing oxygen needs, she required
endotracheal intubation with delivery of 1 dose of intratracheal surfactant on the second day after
delivery and ongoing volume regulated mechanical ventilation. Over the next week, her
respiratory support requirement declined, and she was placed back on nasal CPAP with 25%
oxygen. By 4 weeks of age, the infant required 0.5 L/min oxygen via nasal cannula. On the day
of discharge, she requires 0.25 L/min oxygen via nasal cannula. She is taking full feedings orally
and gaining weight well. The parents are counseled regarding their child’s risk for long-term
respiratory effects and neurocognitive morbidity at 18 to 24 months corrected age.

Of the following, this infant’s MOST predictive risk factor for these outcomes is
A. delayed surfactant administration
B. nasal cannula oxygen requirement at discharge
C. oxygen requirement at 28 days after birth
D. very low birth weight

American Academy of Pediatrics 300


PREP® Self-Assessment PREPSA 2023

Correct Answer: B
The infant in the vignette has mild bronchopulmonary dysplasia (BPD) requiring very low
respiratory support from a low-flow nasal cannula and fraction of inspired oxygen (FiO2) <30%
(Item C90A). The level of respiratory support required at 36 weeks’ postmenstrual age (PMA) is
a functional marker for the severity of BPD and the best predictor of respiratory and/or
neurocognitive morbidity at 18 to 24 months of age, corrected (Item C90B). An infant with mild
BPD, with no other significant risk factors, has the likelihood of a good respiratory and
neurocognitive outcome in the second year after birth.

Since its first description in the 1960s, the definition and classification of BPD has evolved. The
most recently accepted standards are revised from a 2001 National Institutes of Health consensus
conference which established a severity-graded definition of BPD determined by level of
respiratory support needed at 36 weeks’ postmenstrual age (PMA) or hospital discharge,
whichever occurs first.

Intrauterine growth retardation, prenatal and postnatal infection, requirement for invasive
ventilation with high initial oxygen concentration, and more than 1 dose of surfactant are risk
factors for the development and severity of BPD. Additional risk factors for the development of
BPD include male sex, delivery at <26 weeks’ estimated gestational age, and/or <1,100 g birth
weight. The long-term respiratory and neurocognitive outcomes for neonates and infants
diagnosed with BPD is the most associated with the functional classification of BPD at 36 weeks
PMA or discharge (independent of other risk factors for morbidity).

The current classification of BPD does not reference oxygen requirement at 28 days of age,
which is a change from prior iterations. This change results from advances in the treatment of
neonatal respiratory distress syndrome. Advances in resuscitation and respiratory management
have decreased the level of lung injury in many extremely premature neonates and thus modified
the evolution of BPD in this population.

American Academy of Pediatrics 301


PREP® Self-Assessment PREPSA 2023

The pathophysiology of BPD has changed over time. Early pathologic descriptions of BPD
included hypertrophy of airway smooth muscle, necrotizing bronchiolitis, vascular changes
consistent with pulmonary hypertension, and infiltration of inflammatory cells. The introduction
of prenatal corticosteroids and use of surfactant allowed survival of more premature infants and
American Academy of Pediatrics 302
PREP® Self-Assessment PREPSA 2023

led to the development of less aggressive approaches to ventilation. With the increased use of
noninvasive ventilation, lower ventilator pressures, and lower oxygen concentrations, the
pathophysiology of BPD has evolved from inflammation and fibrosis to immature
alveolarization of the lungs and arrested vasculature with fewer but larger alveoli. The overall
incidence of BPD has not changed in many years, but the severity and long-term morbidity has
decreased.

Many therapies other than ventilator support and oxygen have been proposed for treatment of
premature newborns at risk for BPD, but only a few have been demonstrated to change long-term
outcomes. The most effective prevention of BPD is prevention of premature delivery. Postnatal
management strategies to prevent BPD include gentle respiratory resuscitation with limitation of
airway pressure and oxygen as allowed by the situation. Variations in surfactant administration
have been studied. As respiratory support has evolved, the use of noninvasive ventilation and
delaying surfactant or using it selectively in very preterm infants has become the accepted
standard.

Several systemic pharmacological approaches have been studied; long-term data support only
intramuscular vitamin A administration, early introduction of caffeine, and early systemic
corticosteroid use in preventing BPD. Systemic corticosteroids are associated with a significant
adverse effect profile, and thus the risk:benefit ratio must be carefully weighed in each situation.
Systemic corticosteroids are often used late in the course of established BPD as a rescue therapy
to decrease the ongoing inflammatory response to lung injury. Antenatal corticosteroid
administration to the mother at risk for delivery of a very premature infant has not reduced the
risk for BPD, but is associated with improvement in the neonate’s overall morbidity and
mortality, including decreased risk for development of respiratory distress syndrome, necrotizing
enterocolitis, intraventricular hemorrhage, and early septicemia.

PREP Pearls
• The severity of bronchopulmonary dysplasia is defined by the level of respiratory support
required at 36 weeks’ postmenstrual age or hospital discharge, whichever comes first.
• The functional classification of bronchopulmonary dysplasia based on respiratory support
is a risk factor for death and adverse respiratory or neurodevelopmental outcome at less
than 2 years of age.
• Risk factors for development of bronchopulmonary dysplasia include birth weight <1,100
g, estimated gestational age <26 weeks, male sex, prenatal and postnatal infection, high
initial oxygen need, and receipt of multiple surfactant doses.

ABP Content Specifications(s)


• Plan appropriate inpatient and outpatient management of bronchopulmonary dysplasia
• Recognize situations that may lead to bronchopulmonary dysplasia
• Recognize the clinical features of bronchopulmonary dysplasia and its associated
sequelae

American Academy of Pediatrics 303


PREP® Self-Assessment PREPSA 2023

Suggested Readings
• AAP Committee on Fetus and Newborn. Respiratory support in preterm infants at birth.
Pediatrics. 2014;133(1):171-174. doi:10.1542/peds.2013-3442.
• Bamat NA, Zhang H, McKenna KJ, et al. The clinical evaluation of severe
bronchopulmonary dysplasia. NeoReviews. 2020;21(7):e442-e452. doi:10.1542/neo.21-
7-e442.
• Campbell DE. Continuing care of the infant after transfer from neonatal intensive care.
In: McInerny TK, Adam HM, Campbell DE, DeWitt TG, Foy JM, Kamat DM, eds.
American Academy of Pediatrics Textbook of Pediatric Care. American Academy of
Pediatrics; 2021:chap 112. Accessed September 1, 2022. Pediatric Care Online.
• Jensen EA, Dysart K, Gantz MG, et al. The diagnosis of bronchopulmonary dysplasia in
very preterm infants. an evidencebased approach. Am J Resp Crit Care Med.
2019;200(6):751-759. doi:10.1164/rccm.201812-2348OC.
• Jensen EA, Edwards EM, Greenberg LT, et al. Severity of bronchopulmonary dysplasia
among very preterm infants in the United States. Pediatrics. 2021;148(1):e2020030007.
doi:10.1542/peds.2020-030007.
• Jensen EA. Prevention of bronchopulmonary dysplasia: a summary of evidence-based
strategies. NeoReviews. 2019;20(4):e189-e201. doi:10.1542/neo.20-4-e189.
• Kair LR, Leonard DT, Anderson JM. Bronchopulmonary dysplasia. Pediatr Rev.
2012;33(6):255-63; doi:10.1542/pir.33-6255.

American Academy of Pediatrics 304


PREP® Self-Assessment PREPSA 2023

Question 91
A 14-year-old adolescent girl running a 10-km road race on a hot, humid day stops at the medical
tent for evaluation of weakness, dizziness, headache, and cramps in her calf muscles. Her heart
rate is 120 beats/min and temperature is 38.9°C. She is alert and oriented to time and place, and
appears diaphoretic. The remainder of her examination findings are unremarkable.

Of the following, this girl’s MOST likely diagnosis is


A. heat cramps
B. heat edema
C. heat exhaustion
D. heat stroke

American Academy of Pediatrics 305


PREP® Self-Assessment PREPSA 2023

Correct Answer: C
The girl in the vignette has signs and symptoms of heat exhaustion. Individuals with heat
exhaustion may have mild to moderate temperature elevation (under 40°C/104°F), and do not
exhibit major mental status changes.

Other signs and symptoms can include:


• Mild confusion
Dizziness
• Gastrointestinal symptoms (eg, diarrhea, emesis)
• Headache
• Mild hypotension
• Muscle cramps
• Tachycardia

Heat stroke is a more serious illness. Affected individuals have a body temperature of ≥40°C
(104°F) and mental status changes. Heat stroke can lead to multisystem organ failure.

Additional heat stroke signs and symptoms can include:


• Delirium
• Heart failure
• Moderate to severe hypotension
• Liver failure
• Seizures
• Unresponsiveness

Children affected with heat exhaustion are typically diaphoretic, while with heat stroke, the body
no longer produces sweat, and the skin appears dry.

Heat cramps are more accurately described as exercise-associated muscle cramping. These
cramps are localized to the muscles involved in the specific exercise. Dehydration and lack of
proper conditioning are risk factors for exercise-associated muscle cramping. The girl in the
vignette has additional signs and symptoms of heat illness, making heat exhaustion the more
likely diagnosis.

Heat edema results in swelling of the distal extremities due to venous pooling and is not
associated with elevated body temperature. The girl in the vignette does not have edema.
Children and adolescents with suspected heat illness should have vital signs urgently assessed,
including rectal temperature, heart rate, respiratory rate, and blood pressure. Treatment of heat
exhaustion includes stopping exercise, moving to a cooler environment, and oral rehydration. For
young athletes with suspected heat stroke, rapid cooling should be initiated. Immersion in a cold-
water bath with frequent temperature monitoring is the preferred method of cooling for children

American Academy of Pediatrics 306


PREP® Self-Assessment PREPSA 2023

old enough to respond to questions and follow directions. This method is not indicated in cases
of heat exhaustion.

Promoting proper acclimatization (gradually increasing exercise time and intensity during hot
weather periods), limiting sporting events and practices on hot, humid days, and ensuring
frequent access to fluids for hydration can help to reduce the incidence of heat illness in young
athletes.

Organizers of mass participation events held on warm weather days should have an emergency
action plan in place to treat heat illness, including:

• The presence of medical personnel qualified to identify and treat heat illness
• Means of rapid cooling
• Hydration stations
• Shady or cool areas where athletes can rest
• Provisions for accessing emergency medical care in the event of severe illness

PREP Pearls
• Individuals with heat exhaustion may have mild to moderate temperature elevation (up to
40°C [104°F]), and do not exhibit major mental status changes.
• Individuals with heat stroke have a body temperature of ≥40°C (104°F), and often exhibit
mental status changes.
• Immersion in a cold-water bath with frequent temperature monitoring is the preferred
method of cooling for children with heat stroke old enough to respond to questions and
follow directions.

ABP Content Specifications(s)


• Plan the appropriate evaluation of heat illness, and manage appropriately
• Recognize the clinical findings associated with heat illness, including complications
• Understand the mechanisms of heat-related illness, including age-related factors

Suggested Readings
• Council on Sports Medicine and Fitness and Council on School Health; Michael F
Bergeron, Cynthia Devore, Stephen G Rice, American Academy of Pediatrics. Policy
statement-Climatic heat stress and exercising children and adolescents. Pediatrics.
2011;128(3):e741-e747. doi:10.1542/peds.2011-1664.
• Mangus CW, Canares TL. Heat-related illness in children in an era of extreme
temperatures. Pediatr Rev. 2019;40(3):97107. doi:10.1542/pir.2017-0322.

American Academy of Pediatrics 307


PREP® Self-Assessment PREPSA 2023

Question 92
A 14-year-old adolescent boy is evaluated for a 2-day history of brown-colored urine. He had a
sore throat 3 days ago. He has no burning, urgency, or passage of blood clots with urination. He
has no history of rash, abdominal pain, or vomiting. Microscopic hematuria was noted on a
screening urine test performed during a health supervision visit 1 year ago. The boy’s family
history is significant for a maternal grandfather who was deaf and underwent kidney
transplantation at age 40 years. The boy’s heart rate is 80 beats/min, respiratory rate is 20
breaths/min, and blood pressure is 120/80 mm Hg. The rest of his examination findings are
unremarkable.

Results of urinalysis with microscopy are shown:


Urine Result
Specific gravity 1.020
Leukocyte esterase Negative
Nitrite Negative
Blood 3+
Protein 3+
Red blood cells >100/HPF
White blood cells 5/HPF

Of the following, the MOST likely diagnosis for this adolescent is


A. Alport syndrome
B. benign familial hematuria
C. Berger disease
D. poststreptococcal glomerulonephritis

American Academy of Pediatrics 308


PREP® Self-Assessment PREPSA 2023

Correct Answer: A
The boy in the vignette has macroscopic hematuria, proteinuria, a history of microscopic
hematuria, and a male family member with early-onset end-stage renal disease and deafness,
which favors a diagnosis of Alport syndrome Alport syndrome, or hereditary nephritis, is a
progressive glomerular disease that is associated with sensorineural hearing loss and ocular
abnormalities. Alport syndrome has an X-linked inheritance pattern in 80% to 85% of patients,
but can have autosomal recessive or autosomal dominant inheritance. A mutation in the gene
encoding the α-5 chain of type IV collagen (COL4A5) is seen in X-linked AS, and mutations in
the α-3 (COL4A3) and α-4 chain (COL4A4) are seen in autosomal recessive and dominant AS.
Abnormalities in the basement membrane in the glomerulus, eye, and inner ear lead to the
clinical findings of AS.

The initial renal presentation of AS includes asymptomatic microscopic hematuria and recurrent
episodes of macroscopic hematuria, which may occur after an upper respiratory infection. Renal
function and blood pressure may be normal early in the disease course. Affected individuals
typically develop worsening proteinuria, hypertension, and renal failure around 16 to 35 years of
age. Renal progression may be slower in certain families, especially those with autosomal
dominant AS, with end-stage renal disease delayed until 45 to 60 years of age. Bilateral
sensorineural hearing loss typically begins in the high-frequency range; the rate of progression is
usually similar to that of renal failure. The eye manifestations in AS include anterior lenticonus,
retinopathy, and corneal dystrophy.

Benign familial hematuria, or thin basement membrane nephropathy (TBMN), is an autosomal


dominant condition characterized by persistent microscopic hematuria. Proteinuria, progressive
renal failure, hearing loss, and eye findings are typically absent in TBMN.

Similar to AS, Berger disease, or immunoglobulin A nephropathy, presents with recurrent


episodes of macroscopic hematuria that may occur after upper respiratory infection. However,
this condition is not associated with a family history of progressive renal failure and deafness.
Poststreptococcal glomerulonephritis (PSGN) typically presents with macroscopic hematuria 2 to
4 weeks after a sore throat and is not associated with a family history of deafness and renal
failure. Thus, PSGN is a less likely diagnosis for the child in the vignette.

The diagnosis of AS is confirmed with molecular genetic testing. A skin or renal biopsy
specimen can also be used to diagnose AS. Treatment of AS is supportive. Proteinuria and
hypertension are managed using an angiotensinconverting enzyme inhibitor or angiotensin
receptor blocker. Renal replacement therapy, either dialysis or renal transplantation, is required
for patients with AS who have end-stage renal disease.

American Academy of Pediatrics 309


PREP® Self-Assessment PREPSA 2023

PREP Pearls
• Alport syndrome has an X-linked pattern of inheritance in most cases but can have
autosomal recessive or autosomal dominant inheritance.
• The clinical presentation of Alport syndrome includes asymptomatic microscopic
hematuria, macroscopic hematuria, worsening proteinuria, hypertension, and progressive
renal failure.
• Nonrenal findings in Alport syndrome include bilateral sensorineural hearing loss and
anterior lenticonus.

ABP Content Specifications(s)


• Recognize the renal findings associated with Alport syndrome

Suggested Readings
• Varade WS. Nephritis. In: McInerny TK, Adam HM, Campbell DE, DeWitt TG, Foy JM,
Kamat DM, eds. American Academy of Pediatrics Textbook of Pediatric Care. American
Academy of Pediatrics; 2021:chap 294. Accessed September 1, 2022. Pediatric Care
Online.
• Viteri B, Reid-Adam J. Hematuria and proteinuria in children. Pediatr Rev.
2018;39(12);573-587. doi:10.1542/pir.2017-0300

American Academy of Pediatrics 310


PREP® Self-Assessment PREPSA 2023

Question 93
A 4-year-old girl is evaluated for thigh muscle pain a few nights per week that wakes her from
sleep. The pain usually starts in the evening, especially after a physically active day; it is relieved
by massage and resolves by morning. She is otherwise healthy and remains very active. She has
had no fever, joint swelling, rash, weight loss, or fatigue. Findings on physical examination are
normal.

Of the following, the MOST appropriate next step in management is to


A. obtain lower extremity radiographs
B. prescribe a vitamin D supplement
C. provide reassurance
D. refer her to a physical therapist

American Academy of Pediatrics 311


PREP® Self-Assessment PREPSA 2023

Correct Answer: C
The girl in the vignette has growing pains, one of the most common causes of musculoskeletal
pain in children. Growing pains usually occur between ages 3 to 12 years; the etiology is
unknown. The diagnosis is made clinically, by exclusion, after performing a thorough history
and physical examination. Item C93A lists common clinical features of growing pains, as well
as red flag findings that warrant further evaluation in order to exclude a pathologic diagnosis.
Imaging and laboratory testing are not indicated to make the diagnosis. Because growing pains is
a diagnosis of exclusion, it is important to exclude other causes of lower extremity pain; the
differential diagnosis is extensive (Item C93B).

Treatment for growing pains is supportive, including massage, application of heat, analgesia with
acetaminophen or ibuprofen, stretches, and reassurance. Administration of vitamin D has not
been rigorously studied as a treatment for growing pains. Physical therapy has not been shown to
be an effective treatment for growing pains.

American Academy of Pediatrics 312


PREP® Self-Assessment PREPSA 2023

PREP Pearls
• Growing pains is one of the most common causes of musculoskeletal pain in children,
usually occurring between ages 3 to 12 years.
• The diagnosis of growing pains is made clinically, by exclusion, based on a thorough
history and physical examination.
• Treatment for growing pains is supportive, including massage, application of heat,
analgesia, stretches, and reassurance.

American Academy of Pediatrics 313


PREP® Self-Assessment PREPSA 2023

ABP Content Specifications(s)


• Recognize the typical presentation of a patient with growing pains

Suggested Readings
• Kanta P, Gopinathan NR. Idiopathic growing pains in pediatric patients: review of the
literature. Clin Pediatr. 2019;58(1):5-9. doi:10.1177/0009922818784956.
• Lehman PJ, Carl RL. Growing pains: when to be concerned. Sports Health.
2017;9(2):132-138. doi:10.1177/1941738117692533.
• Wolf M. Knee pain in children, part III: stress injuries, benign bone tumors, growing
pains. Pediatr Rev. 2016;37(3):114119. doi:10.1542/pir.2015-0042.

American Academy of Pediatrics 314


PREP® Self-Assessment PREPSA 2023

Question 94
A child is admitted to the hospital with a diagnosis of chickenpox. Oral hydroxyzine is ordered
for severe pruritus. A second child in the hospital room next door is undergoing therapy for
essential hypertension and is prescribed parenteral hydralazine. The 2 children are being cared
for by the same physician and bedside nurse. A medication error occurs when these similarly
sounding medications are inadvertently placed in the wrong patient medication bin, and
subsequently administered to the wrong child.

Of the following, the systems-based medication safety practice that MOST likely would have
prevented this medication error is
A. black box safety labels
B. color-coded medication vials
C. multi-dose medication containers
D. tall man lettering

American Academy of Pediatrics 315


PREP® Self-Assessment PREPSA 2023

Correct Answer: D
The vignette describes a “look-alike, sound-alike” medication error that occurred because 2
drugs with similarly sounding names, hydralazine and hydroxyzine, were confused. The systems-
based medication safety practice that most likely would have prevented this medication error is
the use of tall man lettering.

Tall man lettering uses uppercase letters, often in boldface print, to help differentiate medication
names that appear or sound similar. Tall man lettering helps distinguish between similar drug
names by capitalizing dissimilar letters (eg, hydrALAZINE and hydrOXYzine; DOPamine and
DOBUTamine). In addition to being a specific visual clue that can assist the clinician in
performing a visual double check to verify the correct medication, tall man lettering in and of
itself can alert the health care professional that the drug name can be confused with another
similarly sounding drug name.

The Institute for Safe Medication Practices (ISMP) coined the term “tall man lettering” and since
2008, has maintained a list of drug names with recommended, bolded tall man letters. In the
United States, if the US Food and Drug Administration (FDA) determines that a medication
name warrants tall man lettering, they will request that the manufacturer voluntarily revise their
labels to include tall man lettering. Additionally, the FDA may disseminate safety
communications to alert health care professionals of the potential medication name confusion.
Medication errors are extremely common and are the leading cause of medical error. One in
every 1,000 drugs prescribed in both inpatient and ambulatory settings have been associated with
a wrong drug selection during the process of prescribing, transcribing, dispensing, or
administering medications.

Black box warning labels issued by the FDA serve to warn the public and prescribers of
potentially life-threatening medication side effects. The use of single-dose vials (as opposed to
multi-dose vials) or single-dose drug packaging can reduce medication error, however neither
directly impacts the risk of a look-alike, sound-alike medication error. Color-coded medication
vials or vial tops can help distinguish medications from one another, however this practice is less
likely than tall man lettering to reduce the risk of a look-alike, sound-alike error.

PREP Pearls
• Medication errors are the most common medical error, with a frequency of nearly 1 in
1,000 inpatient medication orders and ambulatory prescriptions.
• Look-alike, sound-alike medication errors arise from confusion in the prescribing,
dispensing, or administration of medications with similar sounding names.
• Tall man lettering can help mitigate look-alike, sound-alike errors by providing a visual
cue that the particular medication name carries a high risk for confusion and medication
error.

ABP Content Specifications(s)


• Understand the role of medical device design in prevention of medical error
• Understand the impact of product naming and packaging on medication safety
American Academy of Pediatrics 316
PREP® Self-Assessment PREPSA 2023

Suggested Readings
• Bryan R, Aronson JK, Williams AJ, Jordan S. A systematic literature review of LASA
error interventions. Br J Clin Pharmacol. 2021;87(2):336-351. doi:10.1111/bcp.14644.
• DeHenau C, Becker MW, Bello NM, Liu S, Bix L. Tallman lettering as a strategy for
differentiation in look-alike, sound-alike drug names: the role of familiarity in
differentiating drug doppelgangers. Appl Ergon. 2016;52:77-84.
doi:10.1016/j.apergo.2015.06.009.
• Lambert BL, Schroeder SR, Galanter WL. Does tall man lettering prevent drug name
confusion errors? Incomplete and conflicting evidence suggest need for definitive study.
BMJ Qual Saf. 2016;25(4):213-217. doi:10.1136/bmjqs-2015004929.

American Academy of Pediatrics 317


PREP® Self-Assessment PREPSA 2023

Question 95
An 18-year-old adolescent girl is seen in the emergency department after an episode of syncope.
The adolescent was running in gym class this afternoon when she apparently experienced a loss
of consciousness and woke up on the floor. She never had anything like this event happen before
and is otherwise well with no other symptoms. She has no significant past medical history, she
has been healthy with normal development, and her immunizations are up to date. The girl
appears comfortable and well-nourished. Her vital signs demonstrate mild tachycardia and a
normal blood pressure. On chest auscultation, she has a normal S1, a loud S2, no murmur, rub, or
gallop; and clear breath sounds with good aeration throughout. Her liver edge is palpated 3 cm
below the right costal margin. She has no edema or cyanosis. An electrocardiogram is obtained
(Item Q95). Her chest radiograph is normal.

Of the following, the MOST likely diagnosis is


A. atrial septal defect
B. atrial tachycardia
C. pulmonary hypertension
D. vasovagal syncope

American Academy of Pediatrics 318


PREP® Self-Assessment PREPSA 2023

Correct Answers: C
The adolescent in the vignette had an episode of syncope with exertion. She had no preceding
symptoms. Her physical examination is significant for a loud second heart sound and
hepatomegaly. Her electrocardiogram (ECG) is abnormal, demonstrating right atrial
enlargement, right axis deviation, and right ventricular hypertrophy with ST and T wave
abnormalities. These findings suggest a process in the right heart leading to ventricular strain and
hypertrophy and a dilated right atrium. Among the response choices, these findings are best
explained by pulmonary hypertension. An atrial septal defect may lead to right atrial
enlargement, but not the abnormalities suggested by the other ECG findings. The adolescent’s
ECG demonstrates sinus rhythm, and she is unlikely to have had syncope from atrial tachycardia.
The lack of preceding symptoms and the abnormal ECG are not consistent with vasovagal
syncope.

Pulmonary hypertension (PH), defined by a pulmonary arterial pressure greater than or equal to
25 mm Hg at rest, is rare in the pediatric population and carries a high risk of morbidity and
mortality. Pulmonary hypertension can develop secondary to the physiologic effects of various
diseases including congenital heart disease, thromboembolic disease, and lung disease, or it can
be idiopathic. Regardless of etiology, PH results in vascular abnormalities (intimal hyperplasia
and medial hypertrophy), thrombosis, and inflammation which lead to right ventricular
dysfunction and ultimately right ventricular failure.

Children with PH typically have exertional dyspnea and progressive fatigue, while infants
generally have poor appetite, failure to thrive, and diaphoresis. Syncope and chest pain are
indicative of advanced disease; hemoptysis is a late finding. Findings on physical examination of
a child with PH may include a loud and single second heart sound and a right ventricular lift.
There may be a murmur of pulmonary insufficiency (early diastolic decrescendo) and/or
tricuspid regurgitation (holosystolic), as well as a gallop. Right-sided heart failure can cause
hepatomegaly and peripheral edema. The diagnostic evaluation of a child with suspected PH
includes electrocardiography and echocardiography and may include cardiac catheterization.
Recent advances in medical therapy for PH have improved survival and quality of life; the long-
term prognosis varies considerably based on underlying disease.

Eisenmenger syndrome is a severe type of PH associated with congenital heart disease,


specifically, large, unrepaired left-to-right shunts (eg, atrial septal defect, ventricular septal
defect, patent ductus arteriosus), and more complex lesions with unobstructed pulmonary blood
flow and large aortopulmonary connections. Chronic leftto-right shunting ultimately leads to
pulmonary hypertensive disease and elevation in pulmonary vascular resistance. Subsequently,
the resistance causes the left-to-right shunt to become a right-to-left shunt. The resultant systemic
desaturation impacts all other organs; the long-term prognosis is poor.

American Academy of Pediatrics 319


PREP® Self-Assessment PREPSA 2023

PREP Pearls
• Pulmonary hypertension is rare in the pediatric population and carries a high degree of
morbidity and mortality.
• Pulmonary hypertension may be idiopathic, or it can develop secondary to the
physiologic effects of various diseases, including congenital heart disease, pulmonary
disease, and thromboembolic disease.
• Eisenmenger syndrome is pulmonary hypertension in the setting of a long-standing left-
to-right shunt that results in elevated pulmonary vascular resistance.

ABP Content Specifications(s)


• Identify risks associated with an untreated large left-to-right shunt and pulmonary
hypertension

Suggested Readings
• Abman SH, Hansmann G, Archer SL, et al. Pediatric pulmonary hypertension guidelines
from the American Heart Association and American Thoracic Society. Circulation.
2015;132(21):2037-2099. doi:10.1161/CIR.0000000000000329.
• Arvanitaki A, Giannakoulas G, Baumgartner H, Lammers AE. Eisenmenger syndrome:
diagnosis, prognosis and clinical management. Heart. 2020;106(21):1638-1645.
doi:10.1136/heartjnl-2020-316665.
• Frank BS, Ivy DD. Pediatric pulmonary arterial hypertension. Pediatr Clin North Am.
2020;67(5):903-921. doi:10.1016/j.pcl.2020.06.005.
• Rajagopal H, Karnik R, Sahulee R. Pediatric pulmonary hypertension. Pediatr Rev.
2016;37(3):129-131. doi:10.1542/pir.2015-0067.

American Academy of Pediatrics 320


PREP® Self-Assessment PREPSA 2023

Question 96
An 8-year-old girl with myelomeningocele repaired at birth is evaluated for abnormal gait. She
has pain and increased tightness in both of her legs. Over the past 6 months she has experienced
intermittent, dull, low-back pain which is relieved with over-the-counter nonsteroidal anti-
inflammatory medications and rest. The girl denies bowel or bladder incontinence. She was
recently treated for a urinary tract infection.

The girl’s physical examination findings are notable for bilateral lower extremity hypertonia with
diminished range of motion of her ankles. Her deep tendon reflexes are absent at the ankles, 2+
at the patella bilaterally, and her toes are mute to plantar stimulation. Sensation is intact to light
touch and pinprick throughout. She is able to ambulate independently with ankle-foot orthotics,
and walks on her tiptoes without them. The remainder of her physical examination findings are
unremarkable.

Of the following, the BEST next step in this girl's management is


A. magnetic resonance imaging of the brain
B. magnetic resonance imaging of the lumbar spine
C. plain radiography of the spine
D. referral to orthopedics for heel cord tenotomy

American Academy of Pediatrics 321


PREP® Self-Assessment PREPSA 2023

Correct Answer: B
The girl in the vignette with myelomeningocele has signs and symptoms suggestive of tethered
cord syndrome (changes in lower extremity tone and gait, and a recent urinary tract infection).
The best next step in her management is to obtain magnetic resonance imaging (MRI) of the
lumbar spine.

Tethered cord syndrome occurs when there is an abnormally low-lying conus medullaris due to a
short, thick filum terminale, which results in an immobile spinal cord. Tethered cord is almost
universally present in children with myelomeningocele, but a minority (approximately 25%)
develop symptoms.

Symptoms of tethered cord include:


• Changes in urination
• Frequent urinary tract infections
• Lower extremity weakness
• Lower extremity tone or sensory changes
• Worsening of pre-existing foot deformities
• Spinal deformities
• Development of leg length discrepancy

If a tethered cord is suspected, MRI of the lumbar spine should be performed. A finding of low-
lying conus medullaris with a fatty or fibrous filum terminale on MRI is consistent with tethered
cord. When symptoms are present, earlier surgical repair (untethering) is associated with an
improved functional prognosis. Re-tethering, necessitating repeat procedures, can occur until late
adolescence when the spine has fully matured.

Care of children with myelomeningocele is individualized. Each child’s needs may differ
depending on their motor function level, level of the lesion, social structure and support, and
associated complications or conditions. Children with higher lesion levels and more impaired
mobility require more support. Management is aimed at optimizing quality of life and
independence, and prompt identification and treatment of complications. Many children benefit
from care in multidisciplinary collaborative clinics that may include neurosurgery, neurology,
orthopedic surgery, physiatry and rehabilitation services, developmental-behavioral pediatrics
and urology; the teams work together to optimize care.

Children with spinal dysraphisms, such as myelomeningocele, are at risk for a variety of
complications. Common complications include:
• Obstructive hydrocephalus secondary to Chiari malformation (especially in infancy)
• Seizures
• Tethered cord syndrome
• Spinal curvatures
• Neurogenic bowel/bladder
• Orthopedic deformities/contractures of the lower extremities

American Academy of Pediatrics 322


PREP® Self-Assessment PREPSA 2023

• Skin breakdown

Children with spinal dysphrasms are at increased risk for anxiety and depression due to social
and developmental challenges related to their underlying condition. Providing ongoing support in
anticipation of barriers and challenges as they grow up is an important part of their care.
When a child with myelomeningocele experiences a change in their neurological status, a broad
differential diagnosis should be considered. If increased intracranial pressure or worsening
hydrocephalus is suspected, brain MRI should be obtained. If changes in spine curvature are
suspected, plain radiography may be appropriate; however, in the setting of a rapid change, MRI
of the spine is preferred to evaluate for interval development of tethered cord or syringomyelia.
If there is persistent tightness at the achilles tendon that is limiting mobility and does not
improve with physical therapy and bracing, heel cord tenotomy should be considered. However,
excluding alternate diagnoses prior to surgery is important, in particular if the heel cord tightness
is a new finding.

PREP Pearls
• While the majority of children with myelomeningocele will have a tethered cord, only
25% will develop symptoms (eg, changes in urination, frequent urinary tract infections,
lower extremity weakness, lower extremity tone or sensory changes, worsening of pre-
existing foot deformities, spinal deformities, or development of leg length discrepancy).
• Children with myelomeningocele are at risk for development of complications such as
obstructive hydrocephalus secondary to Chiari malformation (especially in infancy),
seizures, tethered cord syndrome, spinal curvatures, neurogenic bowel/bladder,
orthopedic deformities/contractures of the lower extremities, and skin breakdown.
• Care of children with myelomeningocele is individualized and each child’s needs may
differ depending on their motor function level, level of the lesion, social structure and
support, and associated complications or conditions.

ABP Content Specifications(s)


• Recognize the clinical manifestations of and complications associated with spinal
dysraphism, and manage appropriately

Suggested Readings
• Burke R, Liptak GS; American Academy of Pediatrics Council on Children with
Disabilities. Providing a primary care medical home for children and youth with spina
bifida. Pediatrics. 2011;128(6):e1645-e1657. doi:10.1542/peds.2011-
• Houtrow A. Spina bifida. In: McInerny TK, Adam HM, Campbell DE, DeWitt TG, Foy
JM, Kamat DM, eds. American
• Academy of Pediatrics Textbook of Pediatric Care. American Academy of Pediatrics;
2021:chap 332. Accessed September 1, 2022. Pediatric Care Online.
• Lu VM, Nizai TN. Pediatric spinal cord disease. Pediatr Rev. 2021;42(9):486-497.
doi:10.1542/pir.2020-000661.

American Academy of Pediatrics 323


PREP® Self-Assessment PREPSA 2023

Question 97
A 17-year-old adolescent boy is seen for a routine health supervision visit. He has a history of
generalized anxiety disorder for which he takes a selective serotonin reuptake inhibitor daily. On
routine adolescent screening, he admits to drinking alcohol until he is intoxicated almost every
weekend. He reports no driving after drinking. He lives with his biological parents and 2
siblings. His mother states that she provides him and his friends alcohol for parties at their home
in order to “make sure he’s safe.” She feels that drinking alcohol is a normal rite of passage for
high school students. The adolescent’s physical examination findings are normal.

Of the following, the MOST appropriate next step is to


A. advise the adolescent’s mother to stop providing her son alcohol
B. ask the adolescent to provide a urine sample for a drug screen
C. decrease the adolescent’s selective serotonin reuptake inhibitor dose
D. reassure the adolescent’s mother that her actions are keeping her son safe

American Academy of Pediatrics 324


PREP® Self-Assessment PREPSA 2023

Correct Answer: A
The adolescent in the vignette is displaying binge drinking behavior, and his mother is providing
alcohol to him and his friends with the intention of preventing drinking and driving. The most
appropriate next step is to advise his mother to stop providing alcohol. Parents who supply their
children with alcohol and other drugs (AOD) in an effort to protect them are actually
normalizing dangerous and risky behaviors.

The pediatrician should also explain the effects of AOD on the developing brain of the
adolescent. Binge drinking (4 or more standard drinks within a 2-hour period for females; 5 or
more standard drinks for males) has been shown to negatively alter adolescent brain
development patterns.

There is no level of “normal” alcohol or drug use in adolescence. A strong recommendation


against any AOD use should be made, as well as counseling adolescents to never ride in a car
with a driver who has been using drugs or alcohol. A discussion should occur with the
adolescent’s mother on how to best support her and her son to achieve a goal of no alcohol or
substance use. Parents and adolescents may find it helpful to set expectations, consequences, and
rewards for behaviors before any negative behaviors occur. Physicians should be knowledgeable
about risk and protective factors for adolescent alcohol and other drug use in order to improve
identification of substance use problems and to enhance counseling and management skills.
Chapter 3 of Facing Addiction in America: The Surgeon General’s Report on Alcohol, Drugs,
and Health provides tables listing risk and protective factors for substance use.

When adolescents are using substances or alcohol as poor coping strategies, the physician should
assist the family in connecting with a mental health care professional who can address these
concerns. Medications such as selective serotonin reuptake inhibitors (SSRIs) can be helpful in
the treatment of comorbid psychiatric disorders such as anxiety, depression, and posttraumatic
stress disorder.
The SSRI medication dose for the adolescent in the vignette may need adjustment, but this is not
the best next management step. It is unclear if SSRI medications are affected by concurrent
alcohol use. In any case, they should not be stopped or decreased if an adolescent has a diagnosis
(eg, anxiety) that may be contributing to the use of recreational AODs. Instead of performing a
urine drug screen, the physician should screen the adolescent for other substance use in the past
12 months with a validated screening tool such as the Screening to Brief Intervention (S2BI) or
the Car, Relax, Alone, Forget, Friends, Trouble (CRAFFT) screen.

American Academy of Pediatrics 325


PREP® Self-Assessment PREPSA 2023

PREP Pearls
• There is no level of “normal” alcohol or substance use in adolescence. Adolescent
alcohol and substance use should not be considered a normal “rite of passage.”
• Binge drinking (ie, 4 or more standard drinks within a 2-hour period for females; 5 or
more standard drinks for males) has been shown to negatively alter adolescent brain
development patterns.
• When an adolescent is using substances or alcohol as a poor coping strategy, the
physician should assist the family in connecting with a mental health provider who can
address this concern.

ABP Content Specifications(s)


• Understand environmental/familial factors contributing to substance use
• Understand the importance of the primary care physician’s involvement in school- and
community-based educational initiatives with regard to substance use and the value of
such initiatives

Suggested Readings
• Facing Addiction in America: The Surgeon General’s Report on Alcohol, Drugs, and
Health. U.S. Department of Health and Human Services. Office of the Surgeon General;
2016. Accessed September 1, 2022.
https://store.samhsa.gov/sites/default/files/d7/priv/surgeon-generals-report.pdf.
• Bagley S, Levy S. Substance use disorders. In: McInerny TK, Adam HM, Campbell DE,
DeWitt TG, Foy JM, Kamat DM, eds. American Academy of Pediatrics Textbook of
Pediatric Care. American Academy of Pediatrics; 2021:chap 336. Accessed September 1,
2022. Pediatric Care Online.
• Hagan JF, Shaw JS, Duncan P, eds. Bright Futures: Guidelines for Health Supervision of
Infants, Children, and Adolescents. 3rd ed. American Academy of Pediatrics; 2008.
• Levy SJ, Williams JF; Committee on Substance Use and Prevention. Substance use
screening, brief intervention, and referral to treatment. Pediatrics.
2016;138(1):e20161211. doi:10.1542/peds.2016-1211.

American Academy of Pediatrics 326


PREP® Self-Assessment PREPSA 2023

Question 98
A 5-year-old girl is evaluated in the emergency department for 6 days of left neck swelling,
fever, and decreased oral intake. The left side of her neck is red and painful when touched and
when she moves her head left and right. She recently had a cough and congestion which resolved
about 1 week ago.

Her temperature is 38.9°C, heart rate is 120 beats/min, respiratory rate is 18 breaths/min, and
oxygen saturation is 97% in room air. There is a 1.5-cm area of erythema and swelling over the
left side of her neck that is tender and warm. The remainder of her physical examination findings
are normal. She is given an intravenous normal saline bolus for dehydration and admitted to the
hospital for treatment with a parenteral antibiotic.

Of the following, the BEST treatment for this girl’s infection is


A. ampicillin/sulbactam
B. azithromycin
C. ceftriaxone
D. vancomycin

American Academy of Pediatrics 327


PREP® Self-Assessment PREPSA 2023

Correct Answers: A
The girl in the vignette has symptoms and physical examination findings consistent with acute
cervical lymphadenitis due to a secondary bacterial infection following a viral infection. The
most common bacteria isolated in these infections are Staphylococcus aureus, Streptococcus
pyogenes, other Streptococcus species, and anaerobes. Ampicillin/sulbactam or clindamycin are
recommended as first-line treatment for uncomplicated bacterial acute cervical lymphadenitis. If
there is no improvement with first-line therapy, or the child is critically ill, treatment with
parenteral ceftriaxone and vancomycin is indicated. Intravenous azithromycin is not
recommended for children with suspected bacterial acute cervical lymphadenitis.

Acute cervical lymphadenitis is defined as swelling of one or more cervical lymph nodes for less
than 3 weeks. The most common cause is reactive lymphadenitis due to a virus, often a
respiratory virus such as rhinovirus, respiratory syncytial virus, or influenza. In these cases,
swollen lymph nodes occur concurrent with other viral symptoms and resolve in 4 to 7 days with
the viral syndrome. Lymphadenitis associated with viruses such as cytomegalovirus or Ebstein-
Barr virus may have a subacute (3 to 6 weeks) or chronic (>6 weeks) course.

Secondary bacterial cervical lymph node infections typically present with fever persisting
beyond the time of a viral illness, painful lymphadenopathy, surrounding erythema, and
decreased range of motion of the neck. In addition to the most common bacterial etiologies listed
above, infectious etiologies include Bartonella henselae (cat scratch disease), atypical
mycobacterium, fungal, and parasitic infections. These etiologies should be considered in
children with persistent symptoms despite antibiotic treatment.

The least common but most concerning cause for cervical lymphadenitis is malignancy (eg,
lymphoma, rhabdomyosarcoma, and neuroblastoma). Symptoms and physical examination
findings in these cases may include easy bruising, fatigue, weight loss, and fever. A thorough
physical examination should be performed, assessing for other areas of lymphadenopathy,
especially subclavicular.

Imaging modalities to evaluate lymphadenopathy include ultrasonography, computed


tomography (CT), and magnetic resonance imaging (MRI). If bacterial lymphadenitis is
suspected, ultrasonography is useful to evaluate for concerning features or abscess. Although the
sensitivity and specificity of ultrasonography varies with the experience of the ultrasonographer,
the test is quick, non-invasive, and does not expose the child to radiation. If there is concern for a
congenital anomaly (eg, branchial cleft or thyroglossal duct cyst), lymphatic malformation,
malignancy, or involvement of deeper structures (eg, under the sternocleidomastoid muscle), CT
or MRI is indicated. Computed tomography is often preferred, as sedation is not required and
most locations have this readily available. Magnetic resonance imaging is preferred for
evaluation of soft tissue structures and when there is concern for a malignancy; however,
younger children may require sedation and the study may take longer to arrange depending on
the clinical setting.

American Academy of Pediatrics 328


PREP® Self-Assessment PREPSA 2023

Abscesses <1 cm in diameter will typically resolve with antibiotic therapy alone. Abscesses
measuring 1.5-2 cm in diameter may improve with antibiotics alone; in many cases a 24- to 48-
hour trial of appropriate antimicrobial therapy can be attempted, potentially averting the need for
incision and drainage. For abscesses >2 cm, incision and drainage is indicated along with
antibiotic therapy.

PREP Pearls
• Viral reactive lymphadenitis is the most common cause of acute cervical
lymphadenopathy in children.
• Secondary bacterial lymphadenitis should be considered in children with prolonged fever,
neck tenderness, overlying erythema, or decreased range of motion after or during viral
infection.
• Children with weight loss, fatigue, easy bruising, and cervical lymphadenopathy should
be evaluated for malignancy.

ABP Content Specifications(s)


• Formulate a differential diagnosis of cervical lymphadenopathy
• Plan the appropriate management of acute cervical lymphadenopathy
• Plan the appropriate clinical evaluation of acute cervical lymphadenopathy

Suggested Readings
• Gaddy H, Riegel A. Unexplained lymphadenopathy: evaluation and differential
diagnosis. Am Fam Physician. 2016;94(11):896-903.
https://pubmed.ncbi.nlm.nih.gov/27929264/.
• Herendeen NE, Szilagyi PG. Cystic and solid masses of the face and neck. In: McInerny
TK, Adam HM, Campbell DE, DeWitt TG, Foy JM, Kamat DM, eds. American
Academy of Pediatrics Textbook of Pediatric Care. American Academy of Pediatrics;
2021:chap 238. Accessed September 1, 2022. Pediatric Care Online.
• Weinstock MS, Patel NA, Smith LP. Pediatric cervical lymphadenopathy. Pediatr Rev.
2018;39(9):433-443. doi:10.1542/pir.2017-0249.

American Academy of Pediatrics 329


PREP® Self-Assessment PREPSA 2023

Question 99
A 23-year-old, previously healthy woman delivers a female infant at 38 weeks, 2 days of
gestation. There were no complications during the pregnancy or delivery. The newborn pediatric
team is called to the delivery because the mother received fentanyl for pain 60 minutes before
delivery. At 1 minute, the neonate is noted to have acrocyanosis, with a heart rate of 90
beats/min, breathing spontaneously with weak cry on stimulation, and flexion of her extremities.
As the team provides stimulation and dries the neonate, she becomes more vigorous. At 5
minutes, she continues to have acrocyanosis, her heart rate is 140 beats/min, she has a strong cry,
and she is moving all extremities.

Of the following, this neonate’s Apgar scores are

Apgar Score
Response choice 1 minute 5 minutes
Row A 5 8
Row B 6 8
Row C 5 9
Row D 6 9

A. Row A
B. Row B
C. Row C
D. Row D

American Academy of Pediatrics 330


PREP® Self-Assessment PREPSA 2023

Correct Answer: D
Components of the Apgar score include heart rate, respiratory effort, muscle tone, reflex
irritability, and color (Item C99). The neonate in the vignette has a 1-minute Apgar score of 6 (1
point each for heart rate <100 beats/min, weak cry, flexion of extremities, and acrocyanosis, and
2 points for breathing spontaneously) and a 5-minute Apgar score of 9 (2 points each for heart
rate >100 beats/min, strong cry, crying with stimulation, and moving extremities, and 1 point for
acrocyanosis).

Abbreviations: PPV/NCPAP, positive-pressure ventilation/nasal continuous positive airway pressure;


ETT, endotracheal tube.
Record the score in the appropriate place at specific time intervals. The additional resuscitative measures
(if appropriate) are recorded at the same time that the score is reported using a check mark in the
appropriate box. Use the comment box to list other factors including maternal medications and/or the
response to resuscitation between the recorded times of scoring.
Reprinted with permission from American Academy of Pediatrics, Committee on Fetus and Newborn,
American College of Obstetricians and
Gynecologists and Committee on Obstetric Practice. The Apgar score. Pediatrics. 2006;117(4)1446.
ITEM C99: Expanded Apgar score form.

This scoring system was first described in 1952 by Dr Virginia Apgar with a 1-minute score used
to assess the need for prompt intervention to establish breathing in newborns. In 1958, a second
report was published, and since then a 1- and 5-minute Apgar score is assessed routinely for all
neonates at the time of delivery. It is important to note that the current Neonatal Resuscitation
Program (NRP) guidelines state “Apgar scores should not be used to dictate appropriate
resuscitative actions, nor should interventions for depressed infants be delayed until the 1-minute
assessment.” Neonates should be assessed immediately after birth and interventions provided
according to NRP guidelines. Expanded Apgar score cards can be used to record the Apgar
American Academy of Pediatrics 331
PREP® Self-Assessment PREPSA 2023

score, and resuscitation interventions can be made at 5-minute intervals up to 20 minutes after
birth.

Preterm or very-low-birthweight neonates often have low Apgar scores; the significance of that
score is not established outside of its use in the delivery room. In term neonates, the score at 1
minute alone cannot be used to predict the outcome for the infant. A very low score at 5 minutes
is associated with increased mortality but cannot be used alone to predict long-term outcomes
such as cerebral palsy or anoxic brain injury.

Exposure to medication such as analgesics during delivery can lead to a depressed Apgar score.
Other factors associated with low scores in term neonates include prenatal illicit drug use,
trauma, congenital anomalies, hypoxia, placental abruption or other causes of hypovolemia, and
infections.

PREP Pearls
• Components of the Apgar score include heart rate, respiratory effort, muscle tone, reflex
irritability, and color.
• Apgar scores should be assessed at 1 and 5 minutes after birth for all neonates.
• Premature or extremely low–birthweight infants often have low Apgar scores; this status
does not predict their longterm outcome.

ABP Content Specifications(s)


• Identify the effects of neurologic immaturity on the Apgar score in a very-low-birth-
weight infant Know the components and significance of the Apgar score

Suggested Readings
• American Academy of Pediatrics Committee on Fetus and Newborn, American College
of Obstetricians and Gynecologists Committee on Obstetric Practice. The Apgar score.
Pediatrics. 2015;136(4)819-822. doi:10.1542/Peds.2015-2651.
• American Academy of Pediatrics, Committee on Fetus and Newborn, American College
of Obstetricians and Gynecologists and Committee on Obstetric Practice. The Apgar
score. Pediatrics. 2006;117(4)1444-1447. doi:10.1542/peds.2006-0325.
• American Heart Association. Part 5: Neonatal Resuscitation 2020 American Heart
Association Guidelines for Cardiopulmonary Resuscitation and Emergency
Cardiovascular Care Program. Accessed September 1, 2022.
https://cpr.heart.org/en/resuscitation-science/cpr-and-ecc-guidelines/neonatal-
resuscitation.
• Apgar scores. Healthychildren.org. Updated September 25, 2015. Accessed September 1,
2022.
• https://www.healthychildren.org/English/ages-stages/prenatal/delivery-
beyond/Pages/Apgar-Scores.aspx.

American Academy of Pediatrics 332


PREP® Self-Assessment PREPSA 2023

Question 100
A 5-day-old male infant is brought to the emergency department with decreased oral intake,
urine output, and energy level. He was born at term with a normal birthweight. His mother
received the recommended prenatal care, and her pregnancy was uncomplicated. Both the mother
and neonate are blood type B+. He is exclusively breastfed. There are sick contacts at home with
respiratory illnesses. The neonate has developed nasal congestion and cough but is afebrile.
Physical examination reveals a sleeping but arousable neonate with an open soft and flat anterior
fontanelle; bilateral scleral icterus; lungs clear to auscultation bilaterally; normal cardiovascular
examination findings; soft, nontender, nondistended abdomen with no palpable
hepatosplenomegaly; and jaundiced skin. He has generalized hypotonia, with spasticity in the
lower extremities greater than the upper extremities. Because of concern for sepsis, a blood
specimen is sent for culture and additional laboratory testing.

Laboratory Test Result


White blood cell count 11,600/µL (11.6 × 109/L)
Hemoglobin 12.6 g/dL (126 g/L)
Platelet count 177 × 103/µL (177 × 109/L)
Mean corpuscular volume 102 fL
Neutrophils 77.2%
Lymphocytes 8.6%
Monocytes 13.1%
Reticulocyte 4.88%
Absolute reticulocyte count 170 × 103/µL
Total bilirubin 27 mg/dL (461.8 µmol/L)
Direct bilirubin 2.07 mg/dL (35.4 µmol/L)

Of the following, the BEST test to confirm this child’s suspected diagnosis is
A. direct antiglobulin (Coombs)
B. G6PD enzyme level
C. hemoglobin electrophoresis
D. osmotic fragility

American Academy of Pediatrics 333


PREP® Self-Assessment PREPSA 2023

Correct Answer: B
The boy in the vignette, with indirect hyperbilirubinemia, most likely has a glucose-6-phosphate
dehydrogenase (G6PD) deficiency. Enzyme level measurements are used to diagnose red blood
cell enzymopathies such as G6PD and pyruvate kinase deficiency. A low G6PD level would
make this diagnosis very likely. (A false-negative result may occur in the case of significant
reticulocytosis, as reticulocytes have a higher level of the enzyme.) Glucose-6phosphate
dehydrogenase deficiency has an X-linked recessive pattern of inheritance (rarely, spontaneous
mutations may occur). Most affected individuals are male; females are carriers (though females
may have a low enzyme level due to lyonization).

In the newborn period, G6PD deficiency may cause severe hyperbilirubinemia, in some cases
requiring an exchange transfusion. Close monitoring of affected neonates is warranted as severe
hyperbilirubinemia, such as precipitated by illness, may require prompt intervention to avoid
serious complications such as kernicterus. Kernicterus may occur in this setting not only due to
hemolysis, but possibly also due to reduced glucuronidation of bilirubin caused by the defective
G6PD activity in hepatocytes.

A direct Coombs test or direct antiglobulin test is used to identify antibodies on the surface of an
individual’s red blood cells, which may lead to hemolysis. An antihuman globulin is incubated
with the individual’s blood sample.

Agglutination confirms the presence of an antibody. Anticomplement agglutination usually


indicates an IgM antibody. The newborn in the vignette has the same blood type as his mother’s,
so he is unlikely to have red blood cell antibodies causing hemolysis.

Hemoglobin electrophoresis is used to detect hemoglobinopathies, such as sickle cell disease or


thalassemia. In the current era of newborn screening, most hemoglobinopathies are detected
early and confirmed with hemoglobin electrophoresis, allowing appropriate management of their
disorder from a young age. Genetic testing may also be performed to identify any α or β gene
mutations. Hemoglobinopathies do not usually cause immediate complications in the first few
months after birth because of the presence of fetal hemoglobin. This test would therefore not be
necessary.

Osmotic fragility testing is used to diagnose hereditary spherocytosis, a red blood cell membrane
protein mutation disorder in which the unstable red blood cell membrane leads to extravascular
hemolysis. In this disorder, red blood cells lyse when placed in a hypotonic solution. However,
this test has low sensitivity and specificity. Eosin-5maleimide flow cytometry, in which a
fluorescent dye binds to band 3 and the level of fluorescence is used to identify cases with a low
level of protein, has better sensitivity and specificity. The osmotic fragility test may also be
negative in the presence of fetal hemoglobin, of which the boy in the vignette would have a high
amount because of his age; therefore, this would not be an appropriate test to perform. Genetic
testing is also available to identify red blood cell membrane protein disorders.

American Academy of Pediatrics 334


PREP® Self-Assessment PREPSA 2023

PREP Pearls
• Glucose-6-phosphate dehydrogenase deficiency may cause severe hyperbilirubinemia in
the newborn period.
• Close monitoring of neonates with glucose-6-phosphate dehydrogenase deficiency is
warranted as severe hyperbilirubinemia, such as precipitated by illness, may require
prompt intervention to avoid serious complications such as kernicterus.

ABP Content Specifications(s)


• Recognize the clinical findings associated with G6PD deficiency
• Recognize the inheritance pattern associated with G6PD deficiency

Suggested Readings
• Anderson NB, Calkins KL. Neonatal indirect hyperbilirubinemia. NeoReviews.
2020;21(11):e749-e760. doi:10.1542/neo.21-11-e749 .
• Bhutani VK. Jaundice due to glucose-6-phosphate dehydrogenase deficiency.
NeoReviews. 2012;13(3):e166-e177. doi:10.1542/neo.13-3-e166.
• Lanzkowsky P, Lipton J, Fish JD. Lanzkowsky’s Manual of Pediatric Hematology and
Oncology. 6th ed. Elsevier Inc; 2016:154-158.
• Maxwell RR, Driscoll MC, Manwani D. The newborn with hematologic abnormalities.
In: McInerny TK, Adam HM, Campbell DE, DeWitt TG, Foy JM, Kamat DM, eds.
American Academy of Pediatrics Textbook of Pediatric Care. American Academy of
Pediatrics; 2021:chap 103. Accessed September 1, 2022. Pediatric Care Online.
• Noronha SA. Acquired and congenital hemolytic anemia. Pediatr Rev. 2016;37(6):235-
246. doi:10.1542/pir.2015-0053 .

American Academy of Pediatrics 335


PREP® Self-Assessment PREPSA 2023

Question 101
A 2-year-old, previously healthy boy is evaluated in the emergency department for acute onset of
facial paralysis. His mother noticed that when he smiles, the left side of his face appears to
droop, and he is unable to fully close his left eye. Two weeks ago he had mild rhinorrhea and a
cough, which self-resolved. His mother recalls removing a tick from his left ear while giving him
a bath around that time. She also reports he was hit on the left side of the face with a ball
yesterday.

On physical examination, the boy’s vital signs are normal for age. He is alert, talkative, and
cooperative with the examination. Otologic examination reveals normal tympanic membranes
bilaterally with no lesions or erythema. He has no skin rashes or lesions. On neurological
examination, the boy’s pupils are equal, round, and reactive to light with full extraocular
movements and no nystagmus. He has flattening of the nasolabial fold on the left when his face
is at rest, and when he smiles there is minimal movement of the left side of the mouth. He is
unable to completely close his left eye or raise his left eyebrow. His sensation is intact to light
touch and hearing is intact to finger rub bilaterally. Palatal elevation is symmetric and the tongue
rests midline. His strength is full in all 4 extremities with 2+ deep tendon reflexes throughout and
his toes are downgoing to plantar stimulation. Sensation is intact in all 4 extremities. He reaches
for toys with both hands with no dysmetria or ataxia. He has a normal, narrow-based gait. The
remainder of his physical examination findings are normal.

Of the following, the BEST next step in this boy’s diagnostic evaluation is
A. electromyography with nerve conduction studies
B. head computed tomography without contrast
C. Lyme immunoglobulin G/immunoglobulin M serological testing
D. magnetic resonance imaging of the brain with and without contrast

American Academy of Pediatrics 336


PREP® Self-Assessment PREPSA 2023

Correct Answer: C
The boy in the vignette has an acute isolated peripheral facial nerve palsy with known tick
exposure, necessitating evaluation for early disseminated Lyme disease. In children with facial
nerve palsy, the history and physical examination findings are crucial for guiding diagnostic
testing and management decisions. Bell palsy was the term originally used to denote idiopathic
peripheral facial nerve palsy; no identifiable cause is found in approximately 50% of cases. In
children, acute otitis media is the most common identifiable cause of peripheral facial nerve
palsy with a notable exception in Lyme-endemic areas, where early disseminated Lyme disease
is a common cause.

In the pediatric population, clinical features that suggest the diagnosis of idiopathic peripheral
facial nerve palsy include:

1) Clinical Findinigs
a) Facial paralysis in the expected distribution of a lower motor neuron/peripheral lesion
(involving both the upper and lower face); and
b) Absence of additional neurological deficits
2) Progression and time course:
a) Acute onset (over 1 to 2 days)
b) Progression in severity for up to 3 weeks with subsequent gradual improvement over
months
3) Associated prodrome (eg, ear pain or impaired hearing) may be seen

The history obtained should focus on the evolution and onset of symptoms, potential etiologic
clues (eg, infectious risk factors, tick exposure, trauma, systemic symptoms, or additional
neurological symptoms). A thorough ear, nose, throat (ENT) and skin examination should be
performed. A complete neurological examination, with close attention to the function of the
adjacent cranial nerves, is imperative to exclude multiple cranial neuropathies and confirm the
presence of a peripheral (rather than central) facial nerve paralysis.

The diagnosis of acute facial nerve paralysis is made clinically. Diagnostic evaluation is aimed at
identifying the etiology and guided by history and examination. Careful ENT evaluation with
otoscopy is usually sufficient to evaluate for acute otitis media, mastoiditis, or vesicular lesions
due to reactivation of varicella-zoster (Ramsay-Hunt syndrome). Serological testing for Lyme
disease should be performed if the diagnosis is suspected in endemic regions, and it should be
considered in non-endemic areas. Endemic areas, regions where the blacklegged tick (Ixodes
scapularis or Ixodes pacificus) reside, include the northeast and mid-atlantic states, north central
states (primarily Wisconsin and Minnesota), and the west coast (Northern California).
Neuroimaging can be considered to exclude inflammatory, demyelinating, compressive, or mass
lesions, and should be pursued if the neurological examination reveals additional deficits or
cranial neuropathies. Inflammation of the affected facial nerve can be identified by magnetic
resonance imaging but is not required to confirm the diagnosis. Electromyography and nerve
conduction studies are not useful in the acute evaluation of a facial nerve palsy but are

American Academy of Pediatrics 337


PREP® Self-Assessment PREPSA 2023

occasionally used after 3 to 4 weeks in severe cases to guide prognostic conversations and
surgical decisions.

A foundational aspect of management in peripheral facial nerve palsy is eye care to prevent
drying of the eye and corneal abrasions. Artificial tears during the day and eye ointment with
patching at night are mainstays of ophthalmological care. In adults, early use of short-term
glucocorticoids is recommended as it has been shown to be beneficial in long-term recovery.
Antiviral medications (eg, aciclovir) may improve the long-term recovery, but the evidence is
not strong. There is limited evidence in pediatric populations regarding the use of glucocorticoid
and antiviral medications; treatment is not well established with clinical practice extrapolated
from adult guidelines. If there is high suspicion for early disseminated lyme disease, empiric
antimicrobial treatment can be initiated while awaiting serologic testing confirmation. Acute
otitis media and/or mastoiditis are treated with antimicrobial therapy. In pediatric idiopathic
facial nerve palsy the prognosis is favorable, with some series reporting 90% of children
recovering with no or minimal facial nerve dysfunction.

PREP Pearls
• Bell palsy is an idiopathic acute peripheral facial nerve paralysis characterized by
weakness of the upper and lower facial muscles on the affected side. Approximately 50%
of cases are idiopathic, and in those with an identified cause, acute otitis media is the
most common etiology in children.
• Peripheral facial paralysis can occur in the setting of early disseminated Lyme disease
and should be a diagnostic consideration in Lyme-endemic areas or known tick exposure.
• Prognosis is favorable in the majority of cases of idiopathic acute peripheral facial nerve
palsy. Efficacy of early use of glucocorticoid +/- antiviral medications in the pediatric
population is not well established.

ABP Content Specifications(s)


• Recognize the clinical findings associated with Bell palsy, and manage appropriately

Suggested Readings
• Gronseth G, Paduga R. Evidence-based guideline update: steroids and antivirals for Bell
palsy: Report of the Guideline Development Subcommittee of the American Academy of
Neurology Neurology. 2012;79(22):2209-2213. doi:10.1212/WNL.0b013e318275978c.
• Hanci F, Türay S, Bayraktar Z, Kabakus N. Childhood facial palsy: etiologic factors and
clinical findings, an observational retrospective study. J Child Neurol. 2019;34(14):907-
912. doi:10.1177/0883073819865682.
• Meissner HC. Lyme disease. In: McInerny TK, Adam HM, Campbell DE, DeWitt TG,
Foy JM, Kamat DM, eds. American Academy of Pediatrics Textbook of Pediatric Care.
American Academy of Pediatrics; 2021:chap 287. Accessed September 1, 2022. Pediatric
Care Online.
• Psillas G, Antoniades E, Ieridou F, Constantinidis J. Facial nerve palsy in children: a
retrospective study of 124 cases. J Paediatr Child Health. 2019;55(3):299-304.
doi:10.1111/jpc.14190.
American Academy of Pediatrics 338
PREP® Self-Assessment PREPSA 2023

Question 102
A 16-year-old adolescent girl with a history of anxiety is brought to the office by her mother for
evaluation of a 6.8-kg weight loss over the past 2 months and sleep difficulties. The girl recently
transferred schools, and a group of girls at her new school have been posting negative comments
on her social media accounts.

Privately, the girl admits that she is being bullied at school and on social media. She is no longer
responding to texts from her friends at her old school, and she has quit playing volleyball. The
girl reports having suicidal thoughts and states, “It would be easier if I was not around anymore.”
She denies having a plan or a method to kill herself. She has no prior history of suicidal
behavior.
Mental health resources are provided.

Of the following, the BEST next step in this girl’s management is to


A. complete a safety plan and follow up in 1 week
B. recommend that the school guidance counselor address the bullying
C. refer her to the local emergency department for psychiatric evaluation
D. start an antidepressant and follow up in 1 week

American Academy of Pediatrics 339


PREP® Self-Assessment PREPSA 2023

Correct Answer: A
The girl in the vignette has suicidal ideation but does not currently have a method or plan to
harm herself. She would be considered “low risk,” and appropriate management includes
completion of a safety plan, lethal means restriction (locking up of medications and sharp
objects, and removal of firearms from the home), providing mental health resources, and close
follow-up (eg, 1 week).

Individuals with suicidal ideation, a method, and a plan within the past month or a history of
suicidal behavior in the past 3 months are classified as “high risk.” In that situation, management
includes transfer to a local emergency department to determine if psychiatric hospitalization is
required. Individuals with suicidal ideation and a method, but no plan or intent, are considered
“intermediate risk,” and management includes all of the “low-risk” interventions plus
involvement of a social worker and speaking with the individual’s mental health care
professional (if applicable) before sending them home. Starting an antidepressant without a clear
safety plan in place would not be the best next step in management.

Among adolescents, girls are more likely to consider or attempt suicide, while boys have more
frequent completion. Risk factors for suicide include a:
• Personal history of
o previous suicide
o attempt mental illness
o substance use disorder, childhood abuse or neglect
o trauma
o impulsive or aggressive behaviors
o being the victim of bullying
• Family history of suicide

For the girl in the vignette, addressing the bullying alone is not appropriate management, given
her suicidal ideation. Even when there are protective factors from suicide, such as engagement in
school and other activities as well as having social supports in place (family members and
friends), for an individual exhibiting warning signs, further management is required.

Annual screening for depression is recommended by the United States Preventive Services Task
Force (USPSTF) for all adolescents ages 12 to 18 years. While many of the depression screening
instruments can identify suicide risk, not all adolescents who die by suicide have depression.
Direct questioning is needed to assess the presence and frequency of suicidal thoughts, and
whether the individual has a method and/or plan for hurting themselves. Additionally, questions
regarding previous suicidal behavior should be asked.

Pediatricians can play an active role in suicide prevention by promoting resilience. Examples
include highlighting a child or adolescent’s strengths and encouraging parents and caregivers to
support the development of self-efficacy and problem solving skills for when adolescents are
faced with life’s challenges. Family and peer connectedness should be encouraged as additional
preventative factors.
American Academy of Pediatrics 340
PREP® Self-Assessment PREPSA 2023

PREP Pearls
• Assessment of suicide risk should include direct questioning to determine the presence
and frequency of suicidal thoughts, the availability of a method for self-harm, and
whether or not there is a plan or intent to act.
• Close monitoring, including the completion of a safety plan, at all risk levels is required
for those with suicidal ideation; transfer to an emergency department is indicated for
those in the high-risk category.
• Pediatricians play a critical role in identifying risk and protective factors for suicide as
well as promoting resilience.

ABP Content Specifications(s)


• Understand risk factors associated with suicidal behavior/completed suicide
• Identify factors that are protective against suicidal behavior
• Plan the appropriate assessment and management of suicidal ideation in patients of
various ages

Suggested Readings
• Baum R, Green CM, Wissow L, Kaplan S. Self-harm and suicide. In: Implementing
Mental Health Priorities in Practice.
• American Academy of Pediatrics; 2015. Accessed September 1, 2022.
https://publications.aap.org/pediatriccare/book/345/chapter/5748242/Self-Harm-and-
Suicide.
• Horowitz L, Tipton MV, Pao M. Primary and secondary prevention of youth suicide.
Pediatrics. 2020;145(Suppl 2):S195S203. doi:10.1542/peds.2019-2056H.
• Maslow GR, Dunlap K, Chung RJ. Depression and suicide in children and adolescents.
Pediatr Rev. 2015;36(7):299-310; quiz 309-310. doi:10.1542/pir.36-7-299.
• Suicide Prevention Campaign Toolkit. American Academy of Pediatrics. Updated
December 10, 2020. Accessed September 1, 2022. https://www.aap.org/en/news-
room/campaigns-and-toolkits/suicide-prevention/.

American Academy of Pediatrics 341


PREP® Self-Assessment PREPSA 2023

Question 103
A 10-month-old male infant with sickle cell disease is seen in the emergency department for
evaluation of fever and fussiness of 3 days’ duration. He has reduced his formula intake and had
a few episodes of nonbloody, nonbilious emesis. This morning it was hard to wake him up.
On physical examination, the infant is irritable. His temperature is 40°C, heart rate is 150
beats/min, respiratory rate is 25 breaths/min, and oxygen saturation is 100% in room air.

Laboratory data are shown:


Laboratory Test Result
Serum
White blood cell count 25,000/µL (25.0 × 109/L)
Hemoglobin 10 g/dL (100 g/L)
Platelet count 90 × 103/µL (90 × 109/L)
Cerebrospinal fluid
White blood cell count 550/µL (0.55 × 109/L)
Protein 190 mg/dL
Glucose 15 mg/dL (0.83 mmol/L)
Gram stain gram-negative coccobacilli
Polymerase chain reaction Haemophilus species detected

Three days later, the cerebrospinal fluid culture grew Haemophilus influenzae serotype b. His
parents are concerned and ask how to prevent transmission to his 5-year-old sister who is
currently undergoing chemotherapy for acute lymphoblastic leukemia.

Of the following, the MOST appropriate chemoprophylaxis regimen for this child’s household
members is
A. amoxicillin
B. azithromycin
C. rifampin
D. trimethoprim-sulfamethoxazole

American Academy of Pediatrics 342


PREP® Self-Assessment PREPSA 2023

Correct Answer: C
The infant in the vignette has meningitis from Haemophilus influenzae type b. Secondary disease
accounts for less than 2% of invasive Haemophilus influenzae b infections. The highest-risk
individuals are unimmunized household contacts younger than 4 years. Children with sickle cell
disease, asplenia, HIV infection, cancer, or a primary immunodeficiency are more susceptible to
invasive infection. Chemoprophylaxis for close contacts protects susceptible individuals from
acquiring infection by eliminating colonization. Rifampin achieves high concentrations in the
respiratory secretions. At a dose of 20 mg/kg, orally, once a day for 4 days, rifampin is one of the
most effective antimicrobial agents to eradicate nasopharyngeal carriage of H influenzae b.
Amoxicillin, trimethoprim sulfamethoxazole, and azithromycin are ineffective antimicrobial
prophylaxis agents in this situation.

Haemophilus influenzae is a gram-negative coccobacillus, typically considered normal flora of


the upper respiratory tract in humans. Encapsulated strains (a, b, c, d, e, and f) cause invasive
infections, and the unencapsulated strains are commonly associated with mucosal infections but
are also responsible for invasive infections.

Before the widespread use of the conjugate Haemophilus influenzae b vaccine (pre-1987),
invasive disease was the leading infectious diseases problem in young children, with those
between ages 6 and 18 months at highest risk. Haemophilus influenzae was responsible for more
than 95% of invasive infections in children (meningitis, bacteremia, epiglottitis, pneumonia,
septic arthritis). The incidence of invasive disease decreased by 99% in the post-vaccine era
(1989-2000). Non–type b capsular strains (a, c, d, e, and f) and nontypeable strains are
responsible for most cases of invasive infection in infants and young children since the universal
use of Haemophilus influenzae b conjugate vaccination. The non-b encapsulated H influenzae
serotypes cause invasive disease similar to that seen with H influenzae b. Vaccination against H
influenzae b does not confer immunity against other encapsulated or unencapsulated strains.
Nontypeable H influenzae is currently responsible for half of the cases of mucosal infections
such as otitis media, sinusitis, conjunctivitis, and bronchitis.

Secondary cases after H influenzae b meningitis are rare. Rifampin prophylaxis is recommended
for all household members, including adults, if the household has a contact younger than 4 years
who is not fully immunized. Rifampin prophylaxis is also recommended for all household
members if anyone younger than 18 years has an immunocompromised condition, irrespective of
vaccination status. Rifampin prophylaxis for the index case is suggested if an antibiotic other
than ceftriaxone or cefotaxime is used to treat invasive infection. Chemoprophylaxis in a daycare
or school setting is advised if there are 2 or more cases among the attendees exposed within 60
days.

Clinical manifestations and management of Haemophilus influenzae infections are described in


Item C103.

American Academy of Pediatrics 343


PREP® Self-Assessment PREPSA 2023

PREP Pearls
• Haemophilus influenzae is a gram-negative coccobacillus. Infections in children are
caused by encapsulated and unencapsulated strains.
• Type b Haemophilus influenzae was a common cause of invasive infections before the
widespread use of the conjugated vaccine. Invasive infections are now caused by other
encapsulated strains (a, c, d, e, and f) and nontypeable strains. The nontypeable strains
account for 50% of mucosal infections such as otitis media and sinusitis.
• In cases of Haemophilus influenzae type b meningitis, rifampin prophylaxis is
recommended for all household members, including adults, if the household has a child
younger than 4 years who is not fully immunized.

ABP Content Specifications(s)


• Plan the appropriate management of a typable and nontypable Haemophilus influenzae
infection
• Recognize the clinical features associated with typable and nontypable Haemophilus
influenzae infection
• Plan appropriate prophylaxis for individuals exposed to invasive Haemophilus influenzae
type B
• Understand the epidemiology of Haemophilus influenzae infection

Suggested Readings
• American Academy of Pediatrics. Haemophilus influenzae infections. In: Kimberlin DW,
Barnett ED, Lynfield R, Sawyer MH, eds. Red Book: 2021–2024 Report of the
Committee on Infectious Diseases. 32nd ed. American Academy of Pediatrics; 2021.
Accessed September 1, 2022. Red Book Online.
• Briere EC, Rubin L, Moro PL, et al. Prevention and control of Haemophilus influenzae
type b Disease Recommendations of the Advisory Committee on Immunization Practices
(ACIP). Centers for Disease Control and Prevention. MMWR Recomm Rep.
2014;63(RR-1):1-14. https://pubmed.ncbi.nlm.nih.gov/24572654/.
• Murphy TF. Haemophilus species, including H. influenzae and H. ducreyi (chancroid).
In: Bennett JE, Dolin R, Blaser MJ, eds. Mandell, Douglas and Bennett's Principles and
Practice of Infectious Diseases. 8th ed. Saunders; 2015:25752583.e2.
• St. Geme JW, Rempe KA. Haemophilus influenzae. In: Long SS, Prober CG, Fischer M,
eds. Principles and Practice of Pediatric Infectious Diseases. 5th ed. Elsevier; 2018:926-
931.e3.
• Weinberg GA, Buchanan AM. Meningitis. In: McInerny TK, Adam HM, Campbell DE,
DeWitt TG, Foy JM, Kamat DM, eds. American Academy of Pediatrics Textbook of
Pediatric Care. American Academy of Pediatrics; 2021:chap 289. Accessed September 1,
2022. Pediatric Care Online.

American Academy of Pediatrics 344


PREP® Self-Assessment PREPSA 2023

Question 104
A 10-year-old boy with epilepsy is evaluated for increasing lethargy. He had been doing well on
titrated phenobarbital monotherapy until 6 months ago when he started having breakthrough
seizures. One month ago, valproic acid was added to his medication regimen to optimize his
seizure control. The boy has remained seizure free for the past 2 weeks but has become
increasingly lethargic and difficult to arouse.

On physical examination, the boy has a temperature of 36.5°C, heart rate of 85 beats/min,
respiratory rate of 16 breaths/min, and blood pressure of 105/65 mm Hg. He appears sleepy but
arouses to voice. He is oriented to person and place and speaks with a clear voice. His sclera are
anicteric and pupils are briskly reactive. There is no hepatomegaly. He has normal deep tendon
reflexes throughout. The remainder of his physical examination findings are unremarkable.
Laboratory data are obtained.

Of the following, this boy’s MOST likely laboratory finding is an elevated serum level of
A. ammonia
B. blood urea nitrogen
C. phenobarbital
D. valproic acid

American Academy of Pediatrics 345


PREP® Self-Assessment PREPSA 2023

Correct Answer: C
The boy in the vignette is experiencing a drug interaction between phenobarbital and valproic
acid. Both medications are eliminated by cytochrome p450 hepatic enzymes, which are inhibited
by valproic acid. The addition of valproic acid to his medication regimen decreased the boy’s
metabolism and clearance of phenobarbital, resulting in a higher serum concentration and the
clinical effects of somnolence and lethargy. Although azotemia and hyperammonemia can cause
somnolence and lethargy, an elevated phenobarbital level is a more likely etiology in this
scenario.

Phenobarbital is an inducer of cytochrome p450 enzymes so the boy’s valproic acid level is more
likely to be low, not high.

Antiepileptic drug interactions are common in children and have increased with the introduction
of new medications and the increased use of multiple medications to optimize seizure control.
Antiepileptic drugs may also produce unintended interactions with other medications. Other than
levetiracetam, vigabatrin, and gabapentin, which are eliminated primarily via renal excretion,
most antiepileptic drugs are metabolized by hepatic enzymes.

Drugs that undergo hepatic metabolism via the cytochrome p450 pathway are responsible for
many clinically significant drug interactions. When metabolic pathways are inhibited by the
addition of a new medication, enzyme activity is reduced. This inhibition may be reversible or
irreversible. Clinical effects result from higher serum concentrations of drugs eliminated by the
inhibited metabolic pathway. Valproic acid inhibits the cytochrome p450 hepatic enzyme system
and displaces the protein-bound fraction of some medications.

Drug interactions causing enzyme induction are not as common as inhibition-based interactions
but can have an equally detrimental impact. The most common mechanism of enzyme induction
is increased synthesis of cytochrome p450 enzyme proteins. Induction increases the amount of
p450 available, which results in more rapid elimination and clearance. Phenobarbital, phenytoin,
and carbamazepine are potent inducers of hepatic metabolism via the cytochrome p450 pathway.
Individual variability, the presence of comorbidities, and medication interactions can result in
fluctuations in drug serum concentrations in children. Serum drug level monitoring is key in
preventing adverse medication interactions while ensuring appropriate therapeutic serum
concentrations. When indicated, monitoring serum medication concentrations can help clinicians
establish optimal therapy while avoiding an insufficient response or toxic effects.

American Academy of Pediatrics 346


PREP® Self-Assessment PREPSA 2023

PREP Pearls
• Monitoring serum drug concentrations allows clinicians to establish optimal therapy
regimens while avoiding an insufficient response or toxic effects.
• Many medications, including antiepileptics, interact with cytochrome p450 metabolic
pathways, which can alter the serum concentration of other medications.
• It is important to consider potential drug interactions when starting a new medication.
Cytochrome p450 pathways may be inhibited (potentially increasing concentrations of
concurrently administered medications), or induced (potentially decreasing
concentrations of concurrently administered medications).

ABP Content Specifications(s)


• Understand that concomitant administration of certain drugs can alter the serum
concentrations of other drugs Understand which drugs stimulate or inhibit hepatic
metabolism

Suggested Readings
• Fine A, Wirrell EC. Seizures in children. Pediatr Rev. 2020;41(7):321-347.
doi:10.1542/pir.2019-0134.
• Iapadre G, Balagura G, Zagaroli L, et al. Pharmacokinetics and drug interaction of
antiepileptic drugs in children and adolescents. Paediatr Drugs. 2018;20(5):429-453.
doi:10.1007/s40272-018-0302-4.
• Larsen G, Cash J. Drug interactions and adverse effects. In: McInerny TK, Adam HM,
Campbell DE, DeWitt TG, Foy JM, Kamat DM, eds. American Academy of Pediatrics
Textbook of Pediatric Care. American Academy of Pediatrics; 2021:chap 246. Accessed
September 1, 2022. Pediatric Care Online.

American Academy of Pediatrics 347


PREP® Self-Assessment PREPSA 2023

Question 105
A 24-month-old girl is brought to the clinic for a health supervision visit. Her parents ask what
technique is most helpful in the toilet training process. The girl is otherwise developmentally
appropriate and healthy. Her mother is 39 weeks pregnant.

Of the following, the MOST appropriate technique to recommend is

A. frequent reminders
B. initiation as soon as possible
C. negative reinforcement
D. training pants for the transition

American Academy of Pediatrics 348


PREP® Self-Assessment PREPSA 2023

Correct Answers: D
The American Academy of Pediatrics (AAP) has published guidelines to advise pediatricians on
parental toilet training anticipatory guidance. The parent-directed approach involves structural
behavioral training that includes increased fluid intake, scheduled toilet times, and both positive
and negative reinforcement. The child-directed approach, based on Dr T. Berry Brazelton’s
methods and recommended by the AAP, involves initiating training only when the child is
physiologically, developmentally, and behaviorally ready to toilet train (Item C105). It is
recommended that pediatricians initiate toilet training readiness assessment and discussion
beginning at the 24-month health supervision visit.

Toilet training methods in the United States have evolved over time and vary depending on
family culture, child behavior and development, sex of the child, parental beliefs, and family
circumstances. The age at which children achieve daytime continence has increased over time,
with 98% continent by age 36 months. There is little benefit to initiating training before 27
months of age. The AAP recommends avoiding: 1) pressure or negative reinforcement, and 2)
initiation during times of transition, such as a new school, moving, new caregivers, or new
siblings, and 3) keeping toddlers in wet or soiled diapers. The child-directed approach
recommends that children should: 1) be encouraged to use the toilet when feeling the urge to go
rather than at scheduled times; 2) have a safe, comfortable toilet-training environment, including
an accessible toilet or chair at the appropriate size, and 3) use training paints as a transition from
diapers to underwear.

To avoid toilet-training refusal, it is recommended that parents avoid parent–child conflict over
using the toilet, which may be counterproductive and potentially damaging. This conflict can
lead to the development of withholding behaviors, chronic constipation, and encopresis. Any
stressors in the child’s life should be addressed, pressure on toilet training should be avoided,
and the child should be the one in charge of the process. Positive feedback systems, such as
encouragement and sticker and star charts, may be helpful.

American Academy of Pediatrics 349


PREP® Self-Assessment PREPSA 2023

PREP Pearls
• The child-directed approach to toilet training involves initiating training when the child is
physiologically,
• developmentally, and behaviorally ready.
• The toilet training child should: 1) have a safe, comfortable toilet environment, including
an accessible toilet or chair at the appropriate size, 2) use training paints as a transition
from diapers to underwear, and 3) be encouraged and provided positive reinforcement.
• To avoid toilet training refusal, parents should avoid parent–child conflict over toileting,
which may be counterproductive and potentially damaging, leading to development of
withholding behaviors, chronic constipation, and encopresis.

ABP Content Specifications(s)


• Understand the factors that contribute to readiness for toilet training
• Recognize the normal age ranges in boys and girls during which bowel and bladder
control occurs Plan the appropriate management of delayed toilet training

Suggested Readings
• Blum NJ, Taubman B, Nemeth N. Relationship between age at initiation of toilet training
and duration of training: a prospective study. Pediatrics. 2003;111(4):810-814.
doi:10.1542/peds.111.4.810.
• Groski PA. Toilet training guidelines: clinicians—the role of the clinician in toilet
training. Pediatrics. 1999;103(Supplement_3):1364-1366. doi:10.1542/peds.103.S3.1364.
• Michel RS. Toilet training. Pediatr Rev. 1999;20(7):240-245. doi:10/154/pir.20-7-240.
• Schonwald A, Hotz A. Toilet training. In: Augustyn M, Zuckerman B, eds. Zuckerman
Parker Handbook of Developmental and Behavioral Pediatrics for Primary Care. 4th ed.
Wolters Kluwer; 2019:431.
• Stadtler AC, Gorski PA, Brazelton TB. Toilet training methods, clinical interventions,
and recommendations. Pediatrics.1999;103(suppl 3):1359-1361.
https://pubmed.ncbi.nlm.nih.gov/10353954/.

American Academy of Pediatrics 350


PREP® Self-Assessment PREPSA 2023

Question 106
A 7-year-old boy is brought to the emergency department for evaluation of right leg pain. Two
days ago, he sustained a closed right distal tibia fracture after being hit by a car while riding his
bicycle. He was evaluated at a local emergency department and underwent fracture reduction and
cast placement under sedation. Initially, the boy required only occasional acetaminophen for
pain. However, over the past 12 hours, he has had increasing pain. His parents have given him 2
doses of hydrocodone, but he has become increasingly difficult to console. On physical
examination, the boy appears agitated. He has a cast on his right lower extremity. The boy is able
to move his toes, which appear pink and well perfused. The remainder of his physical
examination findings are normal.

Of the following, the boy’s MOST likely diagnosis is


A. acute compartment syndrome
B. cellulitis
C. displacement of fracture fragments
D. nerve entrapment

American Academy of Pediatrics 351


PREP® Self-Assessment PREPSA 2023

Correct Answer: A
The boy in the vignette has a closed tibia fracture, is agitated and difficult to console, and has
worsening pain despite increased use of pain medications. These findings cause concern for
acute compartment syndrome. The boy’s cast should be removed for immediate evaluation. He
requires urgent orthopedic surgery consultation for likely fasciotomy surgery to relieve the
increased pressure in the muscle compartments of his leg. Failure to act quickly could result in
permanent muscle and nerve damage.

With compartment syndrome, increased pressure in the fascial compartments leads to decreased
blood flow to the muscle. Children with high-velocity mechanisms of injury are at increased risk
for compartment syndrome. This condition can occur after fracture, circumferential burns, crush
injuries, animal bites, and other soft tissue injuries. The incidence of compartment syndrome is
higher with tibia and forearm/wrist fractures.

Classically, the mnemonic of the “5 P’s” (pallor, paresthesia, paralysis, pain, and pulselessness)
has been used to describe the signs and symptoms of acute compartment syndrome. In a study by
Bae, the authors report on a series of 33 children and adolescents with compartment syndrome.
Only 1 child in the study exhibited the 5 P’s. Most children had 2 or 3 of these features, and 5
had pain only. The authors propose that the “3 A’s” (anxiety, agitation, need for increased
analgesics) are a more reliable indicator of compartment syndrome in children.

Cellulitis causes redness of the skin, pain, and swelling. However, cellulitis is unlikely to cause
the rapid, dramatic increase in pain seen in the boy in the vignette. Additionally, the boy had an
apparent closed fracture and did not require surgical management, so he would not have an
increased risk of infection. Displacement of a fracture following reduction is not uncommon;
however, this condition would occur gradually and be unlikely to cause severe pain. Nerve
entrapment can cause weakness and paresthesia, but it is not a common occurrence with distal
tibia fractures.

PREP Pearls
• Acute compartment syndrome occurs when increased pressure in the myofascial
compartments leads to decreased blood flow and subsequent ischemia.
• The “3 A’s” (anxiety, agitation, and increased analgesic use) are useful indicators of the
presence of acute compartment syndrome in children.
• Children who have signs and symptoms of compartment syndrome require urgent
orthopedic surgery consultation.

ABP Content Specifications(s)


• Recognize the clinical findings associated with compartment syndrome

American Academy of Pediatrics 352


PREP® Self-Assessment PREPSA 2023

Suggested Readings
• Bae DS, Kadiyala RK, Waters PM. Acute compartment syndrome in children:
contemporary diagnosis, treatment, and outcome. J Pediatr Orthop. 2001;21(5):680-688.
https://pubmed.ncbi.nlm.nih.gov/11521042/.
• Gresh M. Compartment syndrome in the pediatric patient. Pediatr Rev. 2017;38(12):560-
565. doi:10.1542/pir.2016-0114
• Lin JS, Samora JB. Pediatric acute compartment syndrome: a systematic review and
meta-analysis. J Pediatr Orthop B. 2020;29(1):90-96.
doi:10.1097/BPB.0000000000000593.
• Sarwark JF, LaBella CR. Compartment syndrome. In: Pediatric Orthopaedics and Sports
Injuries: A Quick Reference Guide. 3rd ed. American Academy of Pediatrics; 2021:483-
487.
• Strahlman RS. Fractures and dislocations. In: McInerny TK, Adam HM, Campbell DE,
DeWitt TG, Foy JM, Kamat DM, eds. American Academy of Pediatrics Textbook of
Pediatric Care. American Academy of Pediatrics; 2021:chap 252. Accessed September 1,
2022. Pediatric Care Online.

American Academy of Pediatrics 353


PREP® Self-Assessment PREPSA 2023

Question 107
Thirty minutes after birth, a neonate has a point-of-care glucose level of 52 mg/dL (2.89
mmol/L). A repeat point-ofcare glucose level at 1 hour after birth is 35 mg/dL (1.94 mmol/L).
He was born at 40 weeks’ gestation with a birth weight 3.98 kg (large for gestational age). The
neonate’s mother does not have a history of diabetes, and her glucose tolerance test result during
pregnancy was normal. The birth weights of her other 4 children were all above 3.8 kg. The
neonate is asymptomatic and has normal physical examination findings.

Of the following, the BEST next step in this neonate’s management is to


A. administer 10% dextrose intravenously
B. administer 25% dextrose orally
C. repeat the point-of-care glucose level in 1 hour
D. send blood to the laboratory for a plasma glucose level

American Academy of Pediatrics 354


PREP® Self-Assessment PREPSA 2023

Correct Answer: D
The large-for-gestational-age, full-term neonate in the vignette is at high risk for hypoglycemia
and has a point-ofcare (POC) glucose level of 35 mg/dL (1.94 mmol/L). Blood glucose levels
measured using POC glucose meters are less accurate than plasma glucose levels measured in the
laboratory, especially in the low blood glucose range. Because the neonate is asymptomatic, the
best next step in management is to send blood to the laboratory for a confirmatory plasma
glucose level.

Neonates at high-risk for hypoglycemia include those who are small for gestational age large for
gestational age born to mothers with diabetes late preterm (34 to 36 6/7 weeks’ gestation)
Treatment of hypoglycemia in neonates at high risk for the condition is dependent on whether
the infant is symptomatic or asymptomatic (Item C107). Symptoms of neonatal hypoglycemia
may include lethargy, apnea, poor feeding, jitteriness, and seizures. While awaiting confirmatory
test results, the asymptomatic neonate in the vignette should be fed donor breast milk, formula,
or oral dextrose gel. A 40% dextrose gel (0.5 mL/kg) massaged into the buccal mucosa, followed
by a feeding, is more effective than feeding alone for treatment of hypoglycemia in at-risk
newborns. There is no evidence that rapid treatment will prevent neurologic sequelae.

Reprinted with permission from Adamkin DH; Committee on Fetus and Newborn. Postnatal glucose
homeostasis in late-preterm and term infants. Pediatrics. 2011;127(3):576.
ITEM C107: Screening for and management of postnatal glucose homeostasis in late-preterm (34 –36 6⁄
7weeks) and term small-for-gestational age infants and infants who were born to mothers with
diabetes/large-for-gestational age infants. Late- preterm and small for gestational age infants (screen 0 –

American Academy of Pediatrics 355


PREP® Self-Assessment PREPSA 2023

24 hours); infants who were born to mothers with diabetes and large-for gestational age infants !34
weeks (screen 0 –12 hours).

Repeating the POC glucose level in 1 hour without a confirmatory plasma level and treatment is
not appropriate management for the high-risk neonate in the vignette. Oral 25% dextrose is not a
recommended treatment for neonatal hypoglycemia. Administration of 10% dextrose
intravenously is not indicated for this asymptomatic neonate but may be needed later, depending
on the repeat glucose levels and/or symptoms.

Hypoglycemia occurs in 10% of healthy term neonates, mainly during the first 24 to 48 hours
after birth. For neonates at high risk for hypoglycemia, the American Academy of Pediatrics
recommends screening for hypoglycemia and maintaining glucose levels greater than 40 mg/dL
(2.22 mmol/L) in the first 4 hours after birth and greater than 45 mg/dL (2.50 mmol/L) in hours 4
to 24 after birth.

Point-of-care glucose meters measure whole blood glucose levels compared with the laboratory
glucose test, which measures plasma glucose levels. The glucose meters used in most nurseries
measure blood glucose levels using the glucose oxidase and peroxidase chromogen method,
which is less accurate (especially in the low blood glucose range) than laboratory measurements
using a hexokinase-based method. All POC glucose meter measurements are susceptible to
alteration by heat and cold because the enzymes (proteins) can denature and become inactivated
at extreme temperatures. Glucose is unstable in whole blood. The plasma glucose concentration
is higher than whole blood glucose by 10% to 18%. It is important that separation of plasma
occurs within 30 minutes of collection. Erythrocytes metabolize glucose; glycolysis will decrease
the glucose concentration at a rate of 5% to 7% per hour if the plasma remains in contact with
the red blood cells.

PREP Pearls
• Neonates at high risk for hypoglycemia include those who are small for gestational age,
large for gestational age, born to mothers with diabetes, and late preterm (34 to 36 and
6/7 weeks’ gestation); these neonates should have their glucose levels monitored shortly
after birth.
• Neonates with hypoglycemia detected on a point-of-care glucose meter should have a
plasma glucose level confirmed in the laboratory. Point-of-care blood glucose levels can
be up to 18% lower than plasma levels.
• Treatment of neonatal hypoglycemia depends on the glucose level and the presence or
absence of symptoms.

ABP Content Specifications(s)


• Understand the limitations associated with the rapid assessment of whole blood glucose
concentrations utilizing glucose oxidase test strips in newborn infants

American Academy of Pediatrics 356


PREP® Self-Assessment PREPSA 2023

Suggested Readings
• Adamkin DH; Committee on Fetus and Newborn. Postnatal glucose homeostasis in late-
preterm and term infants. Pediatrics. 2011;127(3):575-579. doi:10.1542/peds.2010-3851.
• Alsaleem M, Saadeh L, Kamat D. Neonatal hypoglycemia: a review. Clin Pediatr (Phila).
2019;58(13):1381-1386. doi:10.1177/0009922819875540.
• American Academy of Pediatrics. Hypoglycemia. In: PCEP Book 1: Maternal and Fetal
Evaluation and Immediate Newborn Care. American Academy of Pediatrics; 2021.
• Bloomfield DE, Dinolfo E, Kokotos F. Care of the newborn after delivery. In: McInerny
TK, Adam HM, Campbell DE, DeWitt TG, Foy JM, Kamat DM, eds. American
Academy of Pediatrics Textbook of Pediatric Care. American Academy of Pediatrics;
2021:chap 87. Accessed September 1, 2022. Pediatric Care Online.
• Tonyushkina K, Nichols JH. Glucose meters: a review of technical challenges to
obtaining accurate results. J Diabetes Sci Technol. 2009;3(4):971-980.
doi:10.1177/193229680900300446.

American Academy of Pediatrics 357


PREP® Self-Assessment PREPSA 2023

Question 108
A 5-year-old girl with no significant medical history is seen in the emergency department for
abdominal pain and vomiting. After eating dinner, she developed acute-onset, severe, left-sided
abdominal pain associated with several episodes of nonbloody, nonbilious emesis and inability to
tolerate anything by mouth. She has had no fever, chills, diarrhea, constipation, or rash. There
was no recent illness or travel. Vital signs include a temperature of 36.8°C, heart rate of 115
beats/min, respiratory rate of 23 breaths/min, blood pressure of 120/70 mm Hg, and oxygen
saturation of 99% in room air. A large, tender mass is palpable on the left side of her abdomen.
The remainder of her physical examination findings are normal. Abdominal sonography reveals
a large renal mass (11.4 × 8.4 × 11 cm).

Results of laboratory tests include:


Laboratory Test Result
White blood cell count 11,800/µL (11.8 × 109/L)
Hemoglobin 7.6 g/dL (76 g/L)
Platelet count 314 × 103/µL (314 × 109/L)
Mean corpuscular volume 85 fL
Neutrophils  80%
Lymphocytes  12%
Monocytes  8%
Reticulocyte 1.2%
Absolute reticulocyte count 33 × 103/µL (33 × 109/L)
Prothrombin time 13.4 s
International normalized ratio 1.1
Activated partial thromboplastin time 30.8 s

Of the following, the BEST next step in this girl’s management is to


A. administer a transfusion of packed red blood cells
B. administer intravenous morphine
C. consult pediatric surgery
D. perform computed tomography of the chest and abdomen

American Academy of Pediatrics 358


PREP® Self-Assessment PREPSA 2023

Correct Answer: B
The girl in the vignette most likely has Wilms tumor. This diagnosis is supported by the findings
of a palpable abdominal mass, elevated blood pressure, and a renal mass visualized on
sonography. Her abdominal pain and anemia suggest that the mass has ruptured. It is important
to address her abdominal pain before continuing her cancer staging evaluation. As the girl is
hemodynamically stable, she does not require immediate surgical intervention or blood
transfusion.

Wilms tumor (nephroblastoma) is the most common primary renal tumor of childhood. It usually
presents as a painless abdominal mass incidentally palpated by a caregiver or health care
professional. Associated signs and symptoms include hypertension, hematuria, obstipation, and
weight loss.

Children with certain congenital syndromes have an increased risk for Wilms tumor including
Wilms tumor, aniridia, genitourinary malformations, and mental retardation (WAGR), Denys-
Drash, and Beckwith-Wiedemann. Children with these syndromes require screening abdominal
sonography every 3 months until 8 years of age for BeckwithWiedemann and until 5 years for
Denys-Drash and WAGR.

The most common areas of metastasis are the lungs and liver. Therefore, imaging obtained at the
time of diagnosis should include computed tomography of the chest and abdomen. Additional
investigations which should be obtained at the time of diagnosis of Wilms tumor include:
• Complete blood cell count
• Urinalysis
• Comprehensive metabolic profile
• Coagulation profile
• Doppler imaging of the renal veins and inferior vena cava
• Genetic analysis of peripheral blood in individuals with congenital anomalies or familial
Wilms tumor
• Cardiac assessment including echocardiography and electrocardiography

Treatment of Wilms tumor is guided by tumor staging (Item C108) and consists of surgical
resection, chemotherapy, and radiation. In the United States, surgical resection (nephrectomy) is
the first treatment performed (in lower stages of disease), making sure the contralateral kidney is
functional and intact. If the tumor size prevents surgical resection, or there are additional
findings making surgical resection inappropriate, neoadjuvant chemotherapy is administered and
surgery revisited when the tumor size decreases after a few cycles of chemotherapy. The
chemotherapy administered includes vincristine, actinomycin-D, doxorubicin,
cyclophosphamide, and etoposide. Radiation therapy is administered to individuals with higher
stage disease.

The prognosis for survival is 90% for stages I and II disease, 85% for stage III, and 66% for
stage IV or unfavorable histology. There is still a chance of cure in individuals who relapse.

American Academy of Pediatrics 359


PREP® Self-Assessment PREPSA 2023

The girl in the vignette has anemia and will need a transfusion of packed red blood cells.
However, because she is hemodynamically stable, this should be done after her pain is
addressed.

Surgical consultation will be needed, however, not urgently, as the girl in the vignette is in stable
condition. Because the tumor has ruptured, nephrectomy is not appropriate at this time. She will
first require neoadjuvant chemotherapy.

Computed tomography of the chest and abdomen will be required for staging purposes, however,
not before addressing the girl's pain.

PREP Pearls
• Wilms tumor is the most common pediatric renal tumor.
• Wilms tumor usually presents as a painless abdominal mass.
• Serial screening abdominal ultrasonography should be obtained in children with WAGR
(Wilms tumor, aniridia, genitourinary malformations, and mental retardation), Denys-
Drash, or Beckwith-Wiedemann syndrome due to their increased risk of Wilms tumor.

ABP Content Specifications(s)


• Understand the prognosis for a patient who has Wilms tumor Recognize the clinical
findings associated with Wilms tumor

American Academy of Pediatrics 360


PREP® Self-Assessment PREPSA 2023

Suggested Readings
• Friedman AD. Wilms tumor. Pediatr Rev. 2013;34(7):328-30; discussion 330.
doi:10.1542/pir.34-7-328.
• Lanzkowsky P, Lipton J, Fish JD.? Lanzkowsky’s Manual of Pediatric Hematology and
Oncology. 6th ed. Elsevier; 2016:491-502.

American Academy of Pediatrics 361


PREP® Self-Assessment PREPSA 2023

Question 109
A 7-year-old girl is brought to the office for concerns about inattention and decreased
engagement in her schoolwork. Since infancy, the girl has been advanced in her verbal abilities
and problem-solving skills. She started kindergarten at age 4 years. She is currently in the 3rd
grade and has been provided with in-classroom enrichments (eg, working ahead of the class,
additional assignments) during her school years. Despite these adaptations, the girl’s teacher
reports that she appears inattentive and poorly engaged with her work.

Previously, the girl was constantly curious and seeking to learn more. Her mother expresses
concern that the girl’s current school setting is not meeting her needs and reports that “she is
academically and cognitively at a 12-year-old level.” She asks if her daughter should skip a
grade or if they should consider transferring her to a different school.

Outside of school, the girl participates in a math enrichment class 3 days a week and Mandarin
lessons twice a week. She spends her weekends practicing coding or playing chess with her
grandfather. Her parents are concerned that these activities have resulted in them having less
time to spend with the girl’s 10-year-old sister. They also worry that she does not play or spend
time with friends. Socially, the girl has struggled to make friends at school and will become
worked up about topics such as global warming so much that other children tend to avoid her.
The girl’s physical examination findings are unremarkable.

Of the following, the BEST next management step for this girl is to
A. advise that she transfer to a magnet school specializing in science
B. begin stimulant medication to treat her inattention
C. obtain parent and teacher questionnaires to assess for comorbidities
D. recommend psychological assessment to advise on grade placement

American Academy of Pediatrics 362


PREP® Self-Assessment PREPSA 2023

Correct Answer: C
The girl in the vignette is an academically gifted child whose parents have concerns regarding
inattention, her socialemotional development, and effects on her family. The girl’s risk for
depression should also be considered. Of the response choices, the best next management step is
to obtain parent and teacher questionnaires to assess for comorbidities. However, multiple
recommendations are needed given there are several areas of concern.

Given the reports of inattention and decreased engagement, the girl should be assessed for
attentiondeficit/hyperactivity disorder (ADHD) and depression. This assessment may be
facilitated through completion of behavioral questionnaires. Based on the results, plans can be
made for any appropriate medication or recommendations for behavioral interventions. Stimulant
medication may be considered if a diagnosis of ADHD is made but should not be started without
first confirming the diagnosis.

Giftedness is defined as having exceptional abilities or competence in one or more domains


including academics, the arts, music, and athletics. It can be influenced by both sociocultural
differences and limited in some cases by discrimination or poverty. For some gifted children
there is a discrepancy between their cognitive/academic abilities and their social and emotional
development, sometimes referred to as asynchronous development. Behavioral expectations
should be developmentally appropriate. Overscheduling should be avoided, and the importance
of unstructured time should be stressed along with ensuring siblings are supported and not
ignored. Alternative options to skipping a grade should be considered for this girl given her
current social-emotional challenges. Optimization of learning may include increasing in-
classroom enrichments and small group instruction with peers at a similar academic/cognitive
level.

Anticipatory guidance for the gifted child should include:


1. Optimizing learning in the classroom setting and with extra-curricular activities (eg,
enrichments such as special projects; working ahead of the class; small group instruction
for gifted children; participation in a gifted classroom)
2. The social-emotional well-being of the child
3. Family dynamics
4. Assessment for comorbidities (eg, learning disabilities, ADHD, anxiety, and depression)

Parents play a large role in facilitating receipt of enrichments in the school setting and
participation in extracurricular activities. With the increased time and energy expended by
parents to provide opportunities for the gifted child, siblings may feel left out, parents may
become tired or frustrated, and tensions amongst family members may build. All of these factors
need to be considered when providing recommendations.

Additional psychological assessment may provide information on this girl’s current cognitive
and academic functioning, but it does not address her current social difficulties in the classroom;
these may be exacerbated by skipping a grade. Transferring schools only addresses the girl’s
engagement with her school work and would likely add to the family’s stress and concern for
American Academy of Pediatrics 363
PREP® Self-Assessment PREPSA 2023

lack of attention paid to her older sister. It would be important to ensure that her older sister
spends one-on-one time with their parents.

PREP Pearls
• Assessing for learning disabilities, attention-deficit/hyperactivity disorder, anxiety, and
depression should be part of the longitudinal follow-up of gifted children.
• Dynamics in families with a gifted child, including parental and sibling relationships and
potential stressors, should be assessed; unstructured time both individually and as a
family should be encouraged.

ABP Content Specifications(s)


• Understand the effects of gifted children on family dynamics
• Provide anticipatory guidance with respect to management of a gifted child

Suggested Readings
• Intagliata VJ, Scharf RJ. The gifted child. Pediatr Rev. 2017;38(12):575-577.
doi:10.1542/pir.2017-0088.
• Robinson NM, Olszewski-Kubilius PM. Gifted and talented children: issues for
pediatricians. Pediatr Rev.1996;17(12):427-434.
https://pubmed.ncbi.nlm.nih.gov/8973123/.

American Academy of Pediatrics 364


PREP® Self-Assessment PREPSA 2023

Question 110
A 5-year-old boy is seen in the emergency department with a 2-day history of bloody diarrhea
associated with abdominal cramps and malaise. One week ago, he visited a petting zoo at the
state fair and had contact with goats and sheep. He did not consume raw or unpasteurized milk or
milk products. He is afebrile with dry mucous membranes and sunken eyes. Capillary refill time
is 3 seconds.

Laboratory data are shown:


Laboratory Test Result
White blood cell count 13,100/µL (13.1 × 109/L)
Hemoglobin 12 g/dL (120 g/L)
Platelet count 410 × 103/µL (410 × 109/L)
Sodium concentration 128 mEq/L (128 mmol/L)

Of the following, the MOST likely Escherichia coli strain causing this boy’s illness is
A. enteroinvasive
B. enteropathogenic
C. enterotoxigenic
D. Shiga-toxin producing

American Academy of Pediatrics 365


PREP® Self-Assessment PREPSA 2023

Correct Answer: D
The boy in the vignette developed bloody diarrhea without fever after a recent visit to a petting
zoo where he had direct contact with animals and animal manure. This scenario is most
consistent with Shiga-toxin producing Escherichia coli (STEC) infection.

Pathogenic Escherichia coli causes urinary tract infection, enteric infection, and invasive
infection including bacteremia and meningitis. Diarrhea-producing E coli are subdivided into 5
pathotypes based on distinct epidemiology, clinical features, pathogenesis, and genetic
characteristics (Item C110).

Shiga-toxin producing E coli colonize the gastrointestinal tracts of cattle and other ruminants and
are shed in their stools. Transmission of diarrheagenic E coli infection occurs via ingestion of
contaminated water and food (eg, undercooked ground beef, raw leafy vegetables, unpasteurized
milk, or apple cider) or via contact with infected humans, infected animals, or animal manure.
Outbreaks of STEC serotype 0157:H7 have been linked to petting zoos, contaminated drinking
water, and recreational water. Person-to-person spread of STEC infection occurs frequently, and
has resulted in outbreaks of gastroenteritis in households and day care centers. The incubation
period of E coli O157:H7 is 3 to 4 days (range, 1 to 10 days). Diarrhea due to STEC is often
nonbloody at the onset of illness; hemorrhagic colitis develops after 2 to 3 days and is associated
with abdominal pain and low-grade fever.

American Academy of Pediatrics 366


PREP® Self-Assessment PREPSA 2023

The clinical course of diarrhea due to STEC may be complicated by hemolytic uremic syndrome
(HUS). Hemolytic uremic syndrome occurs in 15% of children younger than 5 years with STEC
infection compared with 6% in individuals of all ages. Children aged 1 to 4 years are at highest
risk for development of HUS. The classic manifestations of HUS include the triad of
microangiopathic hemolytic anemia, thrombocytopenia, and acute renal insufficiency; HUS
typically occurs within 1 week after a STEC diarrheal illness. Neurologic complications of HUS
include seizures and intracranial thrombosis. Chronic renal failure occurs in 4% to 10% of
affected children. The case fatality rate of HUS ranges from 3% to 5%. Vasculitis induced by
Shiga toxin and systemic activation of the complement cascade are implicated in the
pathogenesis of HUS.

Diarrhea due to enteropathogenic E coli (EPEC) and enterotoxigenic E coli (ETEC) is watery
because of adherence of the organism to the small bowel. The incubation period of EPEC and
ETEC is 6 to 48 hours. Diarrhea due to EPEC and ETEC strains is common in infants and young
children residing in countries where sanitation is poor and transmission occurs via contaminated
water. Enterotoxigenic E coli is a frequent cause of traveler’s diarrhea in children and adults.
Diarrhea due to enteroinvasive E coli is often watery (without blood or mucus) and associated
with fever; dysentery may occur in some cases.

The diagnosis of diarrheagenic E coli strains can be made with culture-independent, multiplex
polymerase chain reaction (PCR) diagnostic panels. Shiga-toxin producing E coli is diagnosed
using PCR assays on detection of the Stx1 and Stx2 genes. Stool culture (or broth-enriched stool
specimens) can also be used to detect Shiga toxins. Compared with culture and enzyme
immunoassay, PCR-based tests have 100% sensitivity and specificity for detection of O157:H7
and non-O157:H7 STEC strains, are more widely available, and have a more rapid turnaround
time.

Hydration is the mainstay of management of acute enteritis caused by E coli. Use of antimotility
agents in STEC 0157:H7 infection have been associated with increased risk of disease
progression and neurologic complications.

Some studies have demonstrated an association of HUS with the use of some classes of
antimicrobial agents. Therefore, some experts recommend against the use of antimicrobial agents
for treatment of enteritis due to STEC infection.

American Academy of Pediatrics 367


PREP® Self-Assessment PREPSA 2023

PREP Pearls
• Shiga-toxin producing Escherichia coli colonize the gastrointestinal tracts of cattle and
other ruminants and are shed in their stools; outbreaks have been linked to petting zoos,
contaminated drinking water, and recreational water.
• Diarrhea due to Shiga-toxin producing Escherichia coli is often nonbloody at the onset of
illness; hemorrhagic colitis develops after 2 to 3 days and is associated with abdominal
pain and low-grade fever.
• Hydration is the mainstay of management of enteritis due to Shiga-toxin producing
Escherichia coli infection. The use of antimotility agents and antimicrobial agents is not
recommended.

ABP Content Specifications(s)


• Understand the epidemiology of Escherichia coli infection
• Plan appropriate management for a patient with Escherichia coli infection

Suggested Readings
• Adrogue HE, Angelo J. Hemolytic-uremic syndrome. In: McInerny TK, Adam HM,
Campbell DE, DeWitt TG, Foy JM, Kamat DM, eds. American Academy of Pediatrics
Textbook of Pediatric Care. American Academy of Pediatrics; 2021:chap 263. Accessed
September 1, 2022. Pediatric Care Online.
• American Academy of Pediatrics. Escherichia coli diarrhea. In: Kimberlin DW, Barnett
ED, Lynfield R, Sawyer MH, eds. Red Book: 2021–2024 Report of the Committee on
Infectious Diseases. 32nd ed. American Academy of Pediatrics; 2021. Accessed
September 1, 2022. Red Book Online.
• Centers for Disease Control and Prevention. Outbreaks of Escherichia coli O157:H7
associated with petting zoos --- North Carolina, Florida, and Arizona, 2004 and 2005.
MMWR Morb Mortal Wkly Rep. 2005;54(50)1277-1280. Accessed September 1, 2022.
https://www.cdc.gov/mmwr/preview/mmwrhtml/mm5450a1.htm.
• Makvana S, Krilov LR. Escherichia coli infections. Pediatr Rev. 2015;36(4):167-171.
doi:10.1542/pir.36-4-167.

American Academy of Pediatrics 368


PREP® Self-Assessment PREPSA 2023

Question 111
A 16-year-old adolescent girl is seen for dysmenorrhea. Her medical history is significant for
migraine headaches with aura. She had menarche at age 12 years, and her menstrual cycles have
been monthly since that time, lasting 7 days each month. During a confidential discussion, she
reports 2 lifetime sexual partners, both males, and her last sexual encounter was 2 months ago.
She used a condom with every encounter. She reports no dysuria or vaginal discharge, and has
never been tested for sexually transmitted infections.

A urine pregnancy test result is negative. Screening tests for chlamydia, gonorrhea, HIV, and
syphilis are obtained.

Of the following, the BEST next step in this adolescent’s management is to


A. discuss all contraceptive options and have her follow up in 1 month
B. provide her with a prescription for emergency contraception
C. recommend combined oral contraceptive pills
D. recommend a long-acting reversible contraceptive method

American Academy of Pediatrics 369


PREP® Self-Assessment PREPSA 2023

Correct Answers: D
The adolescent in the vignette is sexually active and has dysmenorrhea and migraine headaches
with aura. Of the response choices, the best next step is to recommend a long-acting reversible
contraceptive (LARC) for pregnancy prevention. Long-acting reversible contraceptive methods
(eg, implant, intrauterine device) are recommended for adolescents because of their high efficacy
rate, ease of use, and safety.

Because the adolescent in the vignette is sexually active, a contraceptive option should be
recommended and initiated as soon as possible. Thus, having her follow up in 1 month is not the
preferred response. She does not require emergency contraception because her last sexual
encounter was 2 months ago. Emergency contraception pills include ulipristal acetate and
levonorgestrel; they should be used within 120 hours of the sexual encounter for optimal
efficacy. Combined oral contraceptive pills (OCPs) contain estrogen and progesterone. While
effective for dysmenorrhea management, estrogen-containing OCPs are contraindicated in
individuals who experience migraine with aura The adolescent in the vignette’s primary
dysmenorrhea can be treated with a nonsteroidal antiinflammatory medication.
Adolescents should be provided confidential care for and counseling on sexual health. Sexual
health should be assessed at every adolescent health visit. Physicians should be comfortable with
contraceptive counseling and sexual risk behavior assessments, including sexually transmitted
infection (STI) care and prevention. It is important to encourage 100% condom use for STI
prevention regardless of contraceptive method.

Contraceptives requiring little intervention by the adolescent are the most effective options.
Long-acting reversible contraceptive methods include the etonogestrel implant (replaced every 3
years) and the levonorgestrel-containing intrauterine devices (replaced every 3 to 5 years). Other
progesterone-only forms of contraception include depot medroxyprogesterone acetate injections
(given every 12 weeks) or progesterone-only oral pills (taken daily).
Estrogen-containing contraceptive options include a combined estrogen-progestin OCP, patch,
and vaginal ring.

Estrogen provides uterine lining stability and thus more predictable menstrual bleeding. The
Centers for Disease Control and Prevention’s Summary Chart of US Medical Eligibility Criteria
for Contraceptive Use outlines which contraceptive forms can and cannot be recommended for
adolescents with various medical comorbidities and conditions. Before starting an estrogen-
containing contraceptive, adolescents must be assessed for any contraindications.

PREP Pearls
• Long-acting reversible contraceptives are the most commonly recommended methods of
contraception for adolescents because of their high efficacy rate, ease of use, and safety.
• Health care professionals should know the advantages, disadvantages, and
contraindications for various forms of contraception.
• Estrogen-containing oral contraceptive pills are contraindicated in individuals who
experience migraine with aura.

American Academy of Pediatrics 370


PREP® Self-Assessment PREPSA 2023

ABP Content Specifications(s)


• Understand the forms of contraception available to adolescents and their associated
effectiveness and complications
• Identify relative and absolute contraindications to the use of oral contraceptives
Understand the non-contraceptive benefits of oral contraceptives
• Understand factors associated with contraceptive use or lack of use in adolescents

Suggested Readings
• ACOG Committee on Adolescent Health Care. Counseling adolescents about
contraception. Number 710, August 2017. Reaffirmed 2019. Accessed September 1,
2022. https://www.acog.org/clinical/clinical-
guidance/committeeopinion/articles/2017/08/counseling-adolescents-about-
contraception.
• Centers for Disease Control and Prevention. Summary Chart of U.S. Medical Eligibility
Criteria for Contraceptive Use. Updated 2020. Accessed September 1, 2022.
https://www.cdc.gov/reproductivehealth/contraception/pdf/summary-chartus-medical-
eligibility-criteria_508tagged.pdf.
• Curtis KM, Jatlaoui TC, Tepper NK, et al. U.S. Selected Practice Recommendations for
Contraceptive Use, 2016. MMWR Recomm Rep. 2016;65(No. RR-4):1-66.
doi:10.15585/mmwr.rr6504a1.
• Curtis KM, Tepper NK, Jatlaoui TC, et al. U.S. Medical Eligibility Criteria for
Contraceptive Use, 2016. MMWR Recomm Rep. 2016;65(No. RR-3):1-104.
doi:10.15585/mmwr.rr6503a1.
• Rastogi R, Rome ES. Adolescent gynecology in the office setting. Pediatr Rev.
2021;42(8):427-438. doi:10.1542/pir.2019-0122.
• Schaff EA. Contraception and abortion. In: McInerny TK, Adam HM, Campbell DE,
DeWitt TG, Foy JM, Kamat DM, eds. American Academy of Pediatrics Textbook of
Pediatric Care. American Academy of Pediatrics; 2021:chap 123. Accessed September 1,
2022. Pediatric Care Online.

American Academy of Pediatrics 371


PREP® Self-Assessment PREPSA 2023

Question 138
A 17-year-old adolescent boy is evaluated for burning with urination for the past 3 days. He has
had 3 lifetime sexual partners and is interested in women only. His last sexual encounter was 3
weeks ago when he had vaginal and oral sex without a condom. He has never been tested for
sexually transmitted infections. His physical examination findings are normal except for a scant
amount of mucoid urethral discharge. Urine and oral swabs are obtained for nucleic acid
amplification testing (NAAT) for Neisseria gonorrhoeae and Chlamydia trachomatis.
Of the following, the BEST next step in this adolescent's management is to

A. have him follow-up for treatment in 2 days when the test results are available
B. instruct him to notify all of his sexual partners from the past year
C. provide presumptive antibiotic treatment for gonorrhea and chlamydia
D. send a urinalysis and urine culture

American Academy of Pediatrics 372


PREP® Self-Assessment PREPSA 2023

Correct Answers: C
The adolescent in the vignette has symptoms and signs of urethral inflammation including
dysuria and urethral mucoid discharge. After obtaining urine and oral swabs for nucleic acid
amplification testing (NAAT) for Chlamydia trachomatis and Neisseria gonorrhoeae, the best
next step in management is to treat him with an antibiotic regimen effective against chlamydia
and gonorrhea (eg, doxycycline, orally, and ceftriaxone, intramuscularly). Treatment should
begin right away; it should not be held until test results are available.

Urethritis, inflammation of the urethral epithelial tissue, can have an infectious or non-infectious
etiology. Centers for Disease Control and Prevention (CDC) guidelines recommend that
physicians complete objective diagnostic testing for urethral inflammation when possible.
Criteria for urethral inflammation include one of the following:
• Presence of leukocyte esterase in a first-void urine specimen
• Presence of >10 white blood cells per HPF in a spun first-void urine specimen
• Evidence of gonococcal infection on a urine Gram stain
• Urethral discharge on physical examination

The adolescent in the vignette meets criteria for urethral inflammation with the presence of
mucoid discharge on physical examination, and he should be treated with an appropriate
antibiotic regimen before the chlamydia and gonorrhea NAAT test results return. Thus, having
him follow up in 2 days for treatment after results have returned would not be appropriate.
Urinalysis and urine culture would not aid in the diagnosis; he already meets criteria for urethral
inflammation with the presence of mucoid discharge. Partner notification is imperative after
confirmation of the presence of a sexually transmitted infection (STI). All sexual partners from
the 60 days preceding the onset of symptoms should be notified and advised to be tested and
treated. It is not recommended to notify all partners from the past year.

Nongonococcal urethritis (NGU) is diagnosed when urethral inflammation is caused by an


infectious organism other than N gonorrhoeae. These may include C trachomatis, Mycoplasma
genitalium, Trichomonas vaginalis, as well as other viruses and bacteria. Mycoplasma genitalium
accounts for almost 1 out of 4 cases of NGU in the United States. However, specific testing for
M genitalium should be performed only if symptoms persist after testing and treatment for C
trachomatis.

Confidential sexual history-taking is essential in adolescent health care and should include the 5
P’s (Item C138):

Syphilis and HIV screening should also be completed for the adolescent in the vignette.

Human papillomavirus (HPV) is another important STI to consider when caring for adolescents,
and physicians should recommend HPV vaccination for all young men and women starting at
age 11 years.

American Academy of Pediatrics 373


PREP® Self-Assessment PREPSA 2023

Item C138: The 5 P’s: Partners, Prevention of Pregnancy, Protection From STDs, Practices, and Past
History of STDs
1. Partners
• “Do you have sex with men, women, or both?”
• “In the past 2 months, how many partners have you had sex with?”
• “In the past 12 months, how many partners have you had sex with?”
• “Is it possible that any of your sex partners in the past 12 months had sex with someone else while
they were still in a sexual relationship with you?”
2. Prevention of pregnancy
• “What are you doing to prevent pregnancy?”
3. Protection from STDs
• “What do you do to protect yourself from STDs and HIV?”
4. Practices
• “To understand your risks for STDs, I need to understand the kind of sex you have had recently.”
• “Have you had vaginal sex, meaning ‘penis in vagina sex’?” If yes, “do you use condoms: never,
sometimes, or always?”
• “Have you had anal sex, meaning ‘penis in rectum/anus sex’?” If yes, “do you use condoms: never,
sometimes, or always?”
• “Have you had oral sex, meaning ‘mouth on penis/vagina’?”
For condom answers
• If “never:” “why don’t you use condoms?”
• If “sometimes:” “in what situations (or with whom) do you not use condoms?”
5. Past history of STDs
• “Have you ever had an STD?”
• “Have any of your partners had an STD?”
Additional questions to identify HIV and viral hepatitis risk include
• “Have you or any of your partners ever injected drugs?”
• “Have any of your partners exchanged money or drugs for sex?”
• “Is there anything else about your sexual practices that I need to know about?”

Adapted from Workowski KA, Bolan GA; Centers for Disease Control and Prevention. Sexually
transmitted diseases treatment guidelines, 2015. MMWR Recomm Rep. 2015;64(RR-03):1–137.

PREP Pearls
• Urethritis is commonly caused by sexually transmitted infections including gonorrhea
and chlamydia. Mycoplasma genitalium causes almost 25% of nongonococcal urethritis
cases in the United States.
• Health care professionals should treat individuals with objective signs of urethral
inflammation with antibiotics effective against gonorrhea and chlamydia, even before test
results are available.

ABP Content Specifications(s)


• Recognize the clinical findings associated with urethritis in male adolescents
• Formulate a differential diagnosis of urethritis in male adolescents
• Plan the appropriate evaluation and management of urethritis in male adolescents

American Academy of Pediatrics 374


PREP® Self-Assessment PREPSA 2023

Suggested Readings
• Joffe A. Sexually transmitted infections. In: McInerny TK, Adam HM, Campbell DE,
DeWitt TG, Foy JM, Kamat DM, eds. American Academy of Pediatrics Textbook of
Pediatric Care. American Academy of Pediatrics; 2021:chap 330. Accessed September 1,
2022. Pediatric Care Online .
• Lemly D, Gupta N. Sexually transmitted infections part 2: discharge syndromes and
pelvic inflammatory disease. Pediatr Rev. 2020;41(10):522-537. doi:10.1542/pir.2019-
0078.
• Marcell AV, Burstein GR Braverman P, et al; AAP Committee on Adolescence. Sexual
and reproductive health care services in the pediatric setting. Pediatrics.
2017;140(5):e20172858. doi:10.1542/peds.2017-2858.
• Reno H, Park I, Workowski K, Machefsky A, Bachmann L. A guide to taking a sexual
history. Centers for Disease Control. Updated July 2021. Accessed September 1, 2022.
https://www.cdc.gov/std/treatment/SexualHistory.pdf.
• Workowski, KA, Bachmann LH, Chan, PA, et al. Sexually Transmitted Infections
Treatment Guidelines, 2021. MMWR Recomm Rep. 2021;70(No. RR 3-99). Accessed
September 1, 2022. https://www.cdc.gov/std/treatmentguidelines/toc.htm.

American Academy of Pediatrics 375


PREP® Self-Assessment PREPSA 2023

Question 112
An 8-year-old girl with a history of congenital heart disease and heterotaxy syndrome is seen in
the office to discuss recent abdominal imaging results obtained to evaluate a several month
history of intermittent abdominal pain. An upper gastrointestinal tract series demonstrates an
abnormally positioned duodenal jejunal junction, meeting radiographic criteria for malrotation.
She is not currently experiencing vomiting, abdominal pain, or diarrhea. She has had
cardiothoracic surgery to correct her congenital heart disease and has a history of environmental
allergies. Her father has a history of Crohn disease, and her mother has a history of depression
and has been chronically treated with a selective serotonin reuptake inhibitor. Her family
recently moved, and she is starting at a new school. The girl’s physical examination findings are
unremarkable.

Of the following, the risk factor MOST likely to be associated with this girl’s imaging findings is
A. heterotaxy syndrome
B. history of environmental allergies
C. prenatal exposure to selective serotonin reuptake inhibitors
D. paternal history of Crohn disease

American Academy of Pediatrics 376


PREP® Self-Assessment PREPSA 2023

Correct Answer: A
Of the response options, the girl in the vignette’s history of heterotaxy syndrome is the most
likely risk factor for intestinal malrotation. Heterotaxy syndrome is associated with an increased
risk of intestinal rotation abnormalities, including intestinal malrotation and nonrotation. Up to
83% of children with heterotaxy syndrome have intestinal rotation abnormalities, compared to
1% of the general population. It is important to be aware of this association when evaluating
children with heterotaxy syndrome and gastrointestinal symptoms. A history of environmental
allergies, a family history of inflammatory bowel disease, or prenatal exposure to selective
serotonin reuptake inhibitors do not increase the likelihood of having intestinal rotation
abnormalities.

Normal intestinal rotation occurs by week 12 of gestation. By week 5, the small intestine and
proximal colon extend out of the abdominal wall and into the umbilical cord. Over the next
several weeks, the duodenum rotates counterclockwise 180 degrees, and the intestines return
back into the abdominal cavity. The cecum returns to the abdominal cavity last, causing an
additional 90-degree rotation of the duodenum and jejunum. Fixation of the duodenum and
proximal jejunum in the left upper quadrant then occurs, with the cecum positioned in the right
lower quadrant (Item C112A).

Item C112A: Normal rotation and fixation of the intestine. The small-bowel mesentery is fixed to the
duodenojejunal junction just to the left of L2 to the ileocecal junction at the sacroiliac joint. This
arrangement prevents twisting or volvulus.

Intestinal malrotation occurs when small intestinal rotation fails while the intestines are located
external to the abdominal cavity. The malrotated intestines return to the abdominal cavity with
the duodenal jejunal junction located in the right upper quadrant and the cecum in the mid-
abdomen. Adhesions (Ladd bands) fix the gallbladder and duodenum to the right abdominal
wall, resulting in a narrow mesentery that predisposes to a midgut volvulus as well as to small
bowel obstruction without midgut volvulus (Item C112B).

Midgut volvulus is a surgical emergency. An infant or child with sudden onset of bilious
(occasionally nonbilious) emesis, abdominal pain, and/or abdominal distention should be
urgently evaluated for midgut volvulus, as peritonitis, ischemic bowel, and sepsis can quickly
ensue. Hemodynamically stable infants and children may undergo a confirmatory upper
American Academy of Pediatrics 377
PREP® Self-Assessment PREPSA 2023

gastrointestinal series; a “corkscrew” sign is characteristic of midgut volvulus (Item C112C).


Hemodynamically unstable infants and children may require urgent operative management
without definitive imaging. Item C112D demonstrates the intraoperative appearance of a midgut
volvulus. Surgical management (Ladd procedure) includes lysis of the Ladd bands, mobilizing
the duodenum, division of adhesions around the superior mesenteric artery, and detorsioning of
the midgut volvulus (if present); appendectomy is also recommended.

Item C112B: Malrotation with midgut volvulus. The small-bowel mesentery has no attachment, and, with
the peristaltic motion of the intestine, the entire small bowel can twist around the axis of the superior
mesenteric artery.

Malrotation with Ladd bands. No attachment of the small-bowel mesentery to the posterior abdominal
wall is seen. The avascular adhesions that would normally perform this task instead extend across the
duodenum,causing an obstruction

Intestinal nonrotation occurs when the cecum returns to the abdomen before the small intestine,
preventing the final 90-degree rotation. Because the colon remains on the left side of the
abdomen and the small intestine remains on the right, the mesentery is wide and there is no risk
American Academy of Pediatrics 378
PREP® Self-Assessment PREPSA 2023

of midgut volvulus. However, the cecum is displaced; if appendicitis occurs, it will present in an
abnormal location with increased risk for misdiagnosis.

Item C112C: Upper gastrointestinal contrast study demonstrating malrotation with midgut volvulus—the
absence of the normally positioned ligament of Treitz and the corkscrew appearance of the duodenum

Item C112E summarizes the presentation and management of intestinal rotation abnormalities.
• The gold standard for the diagnosis of a rotation abnormality is an upper gastrointestinal
• contrast study, specifically looking for the location of the DJJ. Ultrasonography may be useful
• as a screening tool.
• The 3 potential causes of duodenal obstruction in children with a rotation abnormality are
midgut volvulus, Ladd bands, and an intrinsic duodenal obstruction. Surgeons should look for
all 3 in children undergoing surgery for a rotation abnormality with duodenal obstruction.
• The distance between the DJJ and the ileocecal junction represents the length of the base of
the small bowel mesentery, and can be estimated by contrast study or more accurately by
laparoscopy. If this distance is less than half the width of the abdomen, the patient may be at
• risk for midgut volvulus and should have a Ladd procedure.

American Academy of Pediatrics 379


PREP® Self-Assessment PREPSA 2023

• A laparoscopic approach is advantageous for older children without clinical or radiological


evidence of midgut volvulus. Infants and children with midgut volvulus should be approached
by laparotomy

PREP Pearls
• There is an increased risk of Intestinal rotational abnormalities in infants and children
with heterotaxy syndrome.
• With intestinal malrotation, adhesions (Ladd bands) that fix the gallbladder and
duodenum to the right abdominal wall result in a narrow mesentery that predisposes to
midgut volvulus, a surgical emergency.
• Surgical management with a Ladd procedure is generally indicated for infants and
symptomatic children with intestinal malrotation.

ABP Content Specifications(s)


• Recognize the clinical features associated with volvulus, and manage appropriately
• Recognize the clinical features associated with malrotation, and manage appropriately

Suggested Readings
• Klein MD. Acute surgical abdomen. In: McInerny TK, Adam HM, Campbell DE, DeWitt
TG, Foy JM, Kamat DM, eds. American Academy of Pediatrics Textbook of Pediatric
Care. American Academy of Pediatrics; 2021:chap 347. Accessed September 1, 2022.
Pediatric Care Online.
• Langer J. Intestinal rotation abnormalities and midgut volvulus. Surg Clin N Am.
2017;97:147-159. doi:10.1016/j.suc.2016.08.011.
• Midura D, Statter MB. Surgical emergencies in the pediatric office. Pediatr Rev.
2022;43(3):148-159. doi:10.1542/pir.2020-004916.
• Ryerson LM, Pharis S, Pockett C, et al. Heterotaxy syndrome and intestinal rotation
abnormalities. Pediatrics. 2018;142(2):e20174267. doi:10.1542/peds.2017-4267.

American Academy of Pediatrics 380


PREP® Self-Assessment PREPSA 2023

Question 113
A 3-year-old girl (Item Q113) is seen for a health supervision visit with her adoptive parents.
Prenatal history is not known. Gross motor skills are mildly delayed. Her other developmental
milestones are appropriate for age. Her weight and height are at the 3rd percentile, and her head
circumference is at the 5th percentile.

Of the following, the girl’s clinical findings are MOST likely due to
A. congenital cytomegalovirus infection
B. folate deficiency during pregnancy
C. maternal alcohol consumption during pregnancy
D. uncontrolled maternal diabetes during pregnancy

American Academy of Pediatrics 381


PREP® Self-Assessment PREPSA 2023

Correct Answers: C
The girl in the vignette demonstrates features of fetal alcohol spectrum disorder (FASD)
including growth restriction, gross motor delay, and the physical findings shown in Item
C113A (midface hypoplasia, a relatively smooth philtrum, small palpebral fissures, and a narrow
upper lip with a poorly defined “cupid’s bow”). Fetal alcohol spectrum disorder results from
maternal alcohol consumption during pregnancy. There is no safe limit for alcohol consumption
during pregnancy; FASD can occur with any level of alcohol intake. Both the American
Academy of Pediatrics and the American College of Obstetricians and Gynecologists
recommend complete abstinence from alcohol for preconceptional and pregnant women. The
clinical features of FASD include:
• Growth restriction (height and/or weight ≤10%)
• Dysmorphic features (small palpebral fissures, thin vermillion border, flat nasal bridge,
smooth philtrum, railroad track appearance to the ear, hockey stick palmar crease, and
midface hypoplasia)
• Central nervous system abnormalities
o Structural changes, head circumference ≤10%
o Functional deficits: developmental delay, cognitive deficits, problems with
attention or hyperactivity, social deficits, psychiatric problems

Diagnostic categories of FASD include fetal alcohol syndrome (FAS), partial fetal alcohol
syndrome (PFAS), alcoholrelated neurodevelopmental disorder (ARND), alcohol-related birth
defects (ARBD), and neurobehavioral disorder associated with prenatal alcohol exposure (ND-
PAE). A FASD diagnostic algorithm is outlined in Item C113B.
Alcohol-related birth defects may include:
• Cardiac: atrial septal defects, aberrant great vessels, ventricular septal defects,
conotruncal heart defects
• Skeletal: radioulnar synostosis, vertebral segmentation defects, large joint contractures,
scoliosis
• Renal: aplastic/hypoplastic/dysplastic kidneys, horseshoe kidneys, ureteral duplications
• Eye: strabismus, ptosis, retinal vascular anomalies, optic nerve hypoplasia Ear:
conductive hearing loss, sensorineural hearing loss

Management of FASD is focused on early developmental intervention strategies and behavioral


modification therapies.

Congenital cytomegalovirus infection can present with prematurity, intrauterine growth


retardation, microcephaly, poor feeding, thrombocytopenia, hepatosplenomegaly, intracranial
calcifications, sensorineural hearing loss, and chorioretinitis. While developmental delay may be
seen, there are no dysmorphic facial features associated with the condition.

Folate deficiency during pregnancy is associated with an increased risk of neural tube defects.
Developmental milestones are normal within the constraints of the location of the neural tube
defect. There are no facial dysmorphisms associated with folate deficiency.

American Academy of Pediatrics 382


PREP® Self-Assessment PREPSA 2023

Uncontrolled maternal diabetes mellitus increases the risk of a major birth defect up to 6% to
9%. Neonates can be large for gestational age and require postnatal glucose monitoring. There
are no associated dysmorphic facial features. Associated congenital malformations include
caudal regression, situs inversus, spinal anomalies, cardiovascular anomalies (transposition of
the great arteries, truncus arteriosus, tricuspid atresia), renal anomalies, and neural tube defects.

Item C113B: FASD diagnostic algorithm

PREP Pearls
• Clinical features of fetal alcohol spectrum disorder include growth restriction, facial
dysmorphic features, and central nervous system abnormalities in structure and/or
function.
• Dysmorphic features associated with fetal alcohol spectrum disorder include small
palpebral fissures, thin vermillion border, flat nasal bridge, smooth philtrum, railroad
track appearance to the ear, hockey stick palmar crease, and midface hypoplasia.
• The American Academy of Pediatrics and the American College of Obstetricians and
Gynecologists recommend complete abstinence from alcohol for preconceptional and
pregnant women.

American Academy of Pediatrics 383


PREP® Self-Assessment PREPSA 2023

MOCA-Peds Objective
• Understand the clinical features, approach to evaluation, and differential diagnosis of
fetal alcohol spectrum disorder.

ABP Content Specifications(s)
• Recognize the clinical and laboratory features associated with fetal alcohol syndrome,
and manage appropriately

Suggested Readings
• Fetal alcohol spectrum disorders (FASDs). Centers for Disease Control and Prevention.
Updated January 6, 2022. Accessed September 1, 2022.
https://www.cdc.gov/ncbddd/fasd/.
• Flore LA. Fetal alcohol spectrum disorders. In: McInerny TK, Adam HM, Campbell DE,
DeWitt TG, Foy JM, Kamat DM, eds. American Academy of Pediatrics Textbook of
Pediatric Care. American Academy of Pediatrics; 2021:chap 250. Accessed September 1,
2022. Pediatric Care Online.
• Hoyme HE, Kalberg WO, Elliott AJ, et al. Updated clinical guidelines for diagnosing
fetal alcohol spectrum disorders. Pediatrics. 2016;138(2):e20154256.
doi:10.1542/peds.2015-4256.
• Hoyme HE; AAP Committee on Genetics. Assessing dysmorphology in primary care. In:
Saul RA, ed. Medical Genetics in Pediatric Practice. American Academy of Pediatrics;
2013:chap 8. Accessed September 1, 2022.
https://publications.aap.org/aapbooks/book/379/Medical-Genetics-in-Pediatric-Practice.
• Smith VC, Turchi R. Fetal alcohol spectrum disorders: FAQs of parents and families.
Healthychildren.org. Updated August
• 20, 2019. Accessed September 1, 2022.
https://www.healthychildren.org/English/healthissues/conditions/chronic/Pages/Fetal-
Alcohol-Spectrum-Disorders-FAQs-of-Parents-and-Families.aspx.

American Academy of Pediatrics 384


PREP® Self-Assessment PREPSA 2023

Question 114
A 5-year-old boy who recently emigrated from a refugee camp in Burma is brought to the office
with complaints of abdominal pain, anorexia, and occasional watery diarrhea for more than 3
weeks. He is afebrile with normal vital signs. He has abdominal distention but no hepatomegaly
or splenomegaly. The remainder of his physical examination findings are normal.

Laboratory data are shown:


Laboratory Test Result
White blood cell count 12,000/µL (12.0 × 109/L)
Neutrophils 68%
Lymphocytes 26%
Monocytes 2%
Eosinophils 4%
Hemoglobin 11.7 g/dL (117 g/L)
Platelet count 274 × 103/µL (274 × 109/L)

A stool specimen is obtained for microscopic examination (Item Q114).

Item Q114: Microscopy of the stool sample from the child in the vignette.
Courtesy of Mae Melvin MD and the Centers for Disease Control and Prevention. For critique say
Ascaris lumbricoides egg. Courtesy of Mae Melvin MD and the Centers for Disease Control and
Prevention.

Of the following, the MOST likely pathogen causing this boy’s illness is
A. Ancylostoma duodenale
B. Ascaris lumbricoides
C. Necator americanus
D. Trichuris trichura

American Academy of Pediatrics 385


PREP® Self-Assessment PREPSA 2023

Correct Answer: B
The boy in the vignette is infected with Ascaris lumbricoides, an intestinal roundworm
commonly found in locations with tropical climates (eg, Burma). The boy’s symptoms
(abdominal pain, anorexia, watery diarrhea), physical examination finding (abdominal
distention), and characteristic appearance of the fertilized eggs on microscopic examination of a
stool specimen (oval shape surrounded by a thick shell with an external proteinaceous layer) are
consistent with A lumbricoides infection.

Ascaris lumbricoides, an intestinal nematode (roundworm), is a soil-transmitted helminth


parasite that infects more than 1.2 billion people globally. Ascariasis is a common infection in
tropical and subtropical regions of the world (eg, Southeast Asia, sub-Saharan Africa), especially
in settings of poverty, inadequate sanitation, and poor personal hygiene. Children younger than 5
years living in impoverished areas are most commonly affected. Infection is common in
immigrant and refugee children from resource-limited settings.

Ascaris lumbricoides infection is acquired through the ingestion of infective embryonated eggs
in fecal contaminated soil. The eggs hatch and release the larvae in the small bowel. The larvae
penetrate the gut mucosa and travel to the lung and liver via the lymphatic and portal venous
circulation. Larvae migrate into the smaller airways via alveolar capillaries and move up along
the tracheobronchial tree to the pharynx where they are swallowed. The larvae mature into adult
worms in the small bowel lumen (primarily the jejunum). The lifespan of the worm is typically 1
to 2 years. Female roundworms produce approximately 200,000 eggs per day. The excreted eggs
become infectious after incubation in soil. Because the parasite cannot replicate in humans, there
is no person-to-person transmission.

Asymptomatic infection is common with intestinal ascariasis. Symptomatic infection is


characterized by anorexia, nausea, abdominal pain, bloating, abdominal distention, and
intermittent diarrhea. Chronic helminth infection can result in malnutrition, developmental delay,
and cognitive deficits in children.

Complications of ascariasis are generally related to the migration of adult worms and include
intestinal or biliary obstruction, leading to biliary colic, cholangitis, cholelithiasis, appendicitis,
and pancreatitis. An acute transient pneumonitis (Löeffler syndrome) may occur during the
initial migration of larvae into the lung.

Diagnosis of ascariasis is made by visualizing the characteristic eggs on microscopic


examination of a fecal specimen. The fertilized eggs are round or oval in shape and surrounded
by a thick shell with an external proteinaceous layer (Item C114A).

Antihelminthic treatment is recommended for individuals with A lumbricoides infection,


including those who are asymptomatic, to prevent complications from worm migration.
Albendazole (given as a single-dose with food) is the drug of choice for treatment of ascariasis in
children. Other treatment options include mebendazole and pyrantel pamoate. Alternative agents
to treat ascariasis are ivermectin and nitazoxanide. Data to ascertain the safety profile of
American Academy of Pediatrics 386
PREP® Self-Assessment PREPSA 2023

ivermectin in pregnant women and children weighing less than 15 kg are insufficient. Follow-up
stool examination for ova and parasites is recommended 2 weeks and 2 to 3 months after
albendazole therapy to document cure.

Item C114A

When ascariasis infection is complicated by small bowel obstruction, management should be


conservative, with the administration of nasogastric suction and hydration before initiation of
albendazole therapy. Surgery may be warranted to treat intestinal or biliary tract obstruction.
Extraction of worms from the biliary tract via endoscopic retrograde cholangiopancreatography
may be necessary.

Other soil-transmitted helminths, including hookworm infections (due to Necator americanus or


Ancylostoma duodenale) and trichuriasis (due to Trichuris trichiura or whipworm), must be
considered in immigrant or refugee children who present with chronic abdominal symptoms.
Chronic hookworm infection causes moderate to severe hypochromic, microcytic anemia. The
hookworm eggs are oval or ellipsoidal with a thin shell (Item C114B). Chronic trichuriasis
infection often presents with dysentery syndrome mimicking inflammatory bowel disease with
bloody diarrhea, mucus, tenesmus, and abdominal pain. Rectal prolapse commonly occurs. The
whipworm eggs are barrelshaped with a thick shell and the appearance of a pair of “plugs” at
each end (Item C114C).

Item C114B : hookworm eggs are oval or ellipsoidal with a thin shell

Item C114C: whipworm eggs are barrelshaped with a thick shell and the appearance of a pair of “plugs”
at each end

American Academy of Pediatrics 387


PREP® Self-Assessment PREPSA 2023

PREP Pearls
• Ascaris lumbricoides, an intestinal nematode (roundworm), is a common soil-transmitted
helminth parasite found in tropical and subtropical regions.
• Symptomatic infection with Ascaris lumbricoides is characterized by anorexia, nausea,
abdominal pain, bloating, abdominal distention, and intermittent diarrhea.
• The diagnosis of ascaris infection is made by visualizing the characteristic eggs on
microscopic examination of a stool specimen; fertilized eggs are round or oval in shape
and surrounded by a thick shell with an external proteinaceous layer.

ABP Content Specifications(s)


• Understand the epidemiology of ascariasis
• Recognize the clinical features associated with ascariasis, and manage appropriately

Suggested Readings
• Abdel-Haq N, Chearskul P, Rafee Y, Asmar BI. Parasitic infections. In: McInerny TK,
Adam HM, Campbell DE, Foy JM, Kamat DM, eds. American Academy of Pediatrics
Textbook of Pediatric Care. 2nd ed. Itasca, IL: American Academy of Pediatrics; 2021;
chap 308. Accessed September 1, 2022. Pediatric Care Online.
• American Academy of Pediatrics. Ascaris lumbricoides infections. In: Kimberlin DW,
Barnett ED, Sawyer MH, eds. Red Book: 2021-2024 Report of the Committee on
Infectious Diseases. 32nd ed. American Academy of Pediatrics; 2021. Accessed
September 1, 2022. Red Book Online.
• Conterno LO, Turchi MD, Corrêa I, Anthelmintic drugs for treating ascariasis. Cochrane
Database Syst Rev. 2020;14;4(4):CD010599. doi:10.1002/14651858.CD010599.pub2.
• Weatherhead JE, Hotez PJ. Worm infections in children. Pediatr Rev. 2015;36(8):341-
354. doi:10.1542/pir.36-8-341.

American Academy of Pediatrics 388


PREP® Self-Assessment PREPSA 2023

Question 115
A 3-year-old girl is seen for a health supervision visit. Her mother is concerned because her
daughter continues to suck her thumb while many of the girl’s peers have stopped this behavior.
She tends to suck her thumb during quiet rest times (eg, watching a movie). Her mother has tried
scolding her for the behavior and applying unpleasant-tasting substances to her thumb. The girl
does not seem bothered by her thumb-sucking, and her peers do not tease her.

The girl appears well. Her development and social interactions are appropriate for age. Her
physical examination findings are normal. She has not lost any primary teeth and has a normal
palate. Her speech is 75% understandable during the examination.

Of the following, the BEST advice for this girl’s mother is to


A. have a dental appliance fitted
B. implement consequences for the behavior
C. not allow her to watch movies with peers
D. provide positive reinforcement

American Academy of Pediatrics 389


PREP® Self-Assessment PREPSA 2023

Correct Answers: D
The 3-year-old girl in the vignette is exhibiting thumb-sucking, a form of non-nutritive sucking
that is common beginning as early as 29 weeks’ gestation in utero. By 4 years of age only 12%
of children continue to suck their thumb or other digit and 4% continue to use a pacifier. The
most effective management strategy and best advice for the girl’s mother is to provide positive
reinforcement (eg, praise for not sucking her thumb during a movie or making a reward chart).

Negative consequences, such as criticizing (eg, telling her she is acting like a baby) or punishing
(eg, not allowing her to watch movies with friends) can increase the undesirable behavior and
should be avoided. A dental device is not recommended until other methods have been tried or
there are secondary mouth anatomy changes. The girl in the vignette has had no changes in the
shape of her mouth or teeth alignment, so having a dental appliance fitted is not indicated. Dental
consultation is recommended when considering use of a dental appliance.

Sucking on a digit/thumb or a pacifier are forms of non-nutritive sucking. Non-nutritive sucking


is part of normal neonatal and infant development. It is a commonly used and safe method to
soothe or calm an infant when they are not hungry. In infancy, pacifier use is the most common
form of non-nutritive sucking and is typically used with sleep. As children become older, thumb
or digit sucking becomes more prevalent and is more common while awake. Most children stop
this behavior on their own, but if it continues into the preschool years, there are potential
negative consequences including social (eg, being thought of as “babyish” or being teased by
peers) and dental (eg, changes in the shape of the mouth and palate or poor teeth alignment).
Changes in primary teeth alignment are likely reversible, while permanent teeth alignment may
need orthodontic intervention for correction. There is also an increased risk of infection (eg,
paronychia and herpetic whitlow).

When a child experiences negative sequelae of thumb-sucking, intervention should be targeted to


the times the child most often exhibits the behavior (eg, when they are bored, nervous, or tired).
Positive reinforcement is recommended over negative reinforcement. Negative reinforcement
can prolong thumb-sucking behavior, especially in children who do it when they are nervous.

Children who use thumb sucking as a method of coping with boredom may benefit from
redirection to an alternative activity such as holding or squeezing an object (eg, a ball) when they
have the urge to suck their thumb. The application of unpleasant-tasting substances to the
thumb/digit can be useful, but it should be made clear to the child that this is a reminder and not
a punishment.

PREP Pearls
• Thumb sucking is a common form of non-nutritive sucking in infants and young children.
• Negative consequences for children who suck their thumb often prolongs the behavior,
while positive reinforcement or providing alternative activities decrease the behavior.
• Dental appliances should be considered if the secondary teeth are becoming misaligned
or the shape of the mouth is being affected by thumb-sucking.

American Academy of Pediatrics 390


PREP® Self-Assessment PREPSA 2023

ABP Content Specifications(s)


• Understand the natural history of thumb sucking, and manage appropriately

Suggested Readings
• American Academy of Pediatrics. Pacifiers and thumb sucking. Healthychildren.org.
Updated November 11, 2020. Accessed September 1, 2022.
https://www.healthychildren.org/English/ages-stages/baby/crying-
colic/Pages/Pacifiersand-Thumb-Sucking.aspx.
• Davidson L. Thumb and finger sucking. Pediatr Rev. 2008;29(6):207-208.
doi:10.1542/pir.29-6-207.
• For the dental patient. Thumb sucking and pacifier use. J Am Dent Assoc.
2007;138(8):1176. doi:10.14219/jada.archive.2007.0331.
• Sarles RM, Edwards S. Self-stimulating behaviors. In: McInerny TK, Adam HM,
Campbell DE, DeWitt TG, Foy JM, Kamat DM, eds. American Academy of Pediatrics
Textbook of Pediatric Care. American Academy of Pediatrics; 2021:chap 192. Accessed
September 1, 2022. Pediatric Care Online.

American Academy of Pediatrics 391


PREP® Self-Assessment PREPSA 2023

Question 116
A 13-year-old adolescent girl is seen for an annual health supervision visit. The girl is a talented
softball player, and her parents would like advice about weight training as an adjunct to her
sport-specific training. The girl has been working with a certified strength and conditioning
specialist for the past 2 years. She has been engaging in a supervised, moderate-intensity weight
training program that involves performing 3 sets of 6 to 8 repetitions of each exercise. She would
like to advance to a higher-intensity weight training program with higher weight and fewer
repetitions. She has no chronic medical conditions and has never had a serious injury from
weight training. The girl’s physical examination findings are normal.

Of the following, the BEST recommendation for this girl is that she
A. change to a lower intensity weight training regimen with lower weight and more
repetitions
B. continue at her current level of moderate-intensity training
C. discontinue weight training and begin a strengthening program with bodyweight
exercises
D. gradually add lifting sets with higher weights and lower repetitions with supervision

American Academy of Pediatrics 392


PREP® Self-Assessment PREPSA 2023

Correct Answer: D
The girl in the vignette has experience with weight training and has been working under the
guidance of a certified coach. She can advance to a higher-intensity weight training program
involving fewer repetitions with higher weights. She should continue to be supervised by an
accredited strength and conditioning coach who can provide guidance on proper technique and
safe handling of equipment.

In the past, sports medicine professionals recommended only low-intensity/high-repetition


resistance training for young athletes. Many clinicians believed that weight training carried a
high risk of physical injury and subsequent growth disturbance in children. The most recent
American Academy of Pediatrics statement on resistance training for young athletes advises that
children and teenagers can safely engage in high-intensity weight training. When advising
children and their families about weight training, pediatricians need to assess a child’s maturity,
prior weight training experience, and quality of supervision.

The incidence of injury with weight training in the pediatric age group is low. Additionally,
resistance training has been shown to reduce many types of injuries. Many injuries from weight
training are due to improper handling of equipment. This finding underscores the need for
supervision by an experienced trainer. The ratio of athletes to coaches should be low enough to
allow the coaches to monitor each child carefully.

The term “training age” refers to a child’s accumulated experience of resistance training with
proper technique. Pediatricians should determine a child’s training age when making a
recommendation about the intensity of weight training. For example, a child who has never
engaged in resistance training should start by learning the proper form for lifting exercises
without using weights, then progress to low-intensity training. Because the girl in the vignette
has been weight training for 2 years under the guidance of a strength and conditioning
professional, she does not need to decrease the intensity of her regimen or discontinue weight
training. She can advance her program to include higher-intensity weight training sets.

PREP Pearls
• The incidence of injuries related to weight training in the pediatric age group is low.
• Children with an appropriate level of maturity, experience, and supervision can gradually
progress to high-intensity weight training.

ABP Content Specifications(s)


• Understand the importance of skeletal maturity in determining the appropriate type of
physical training

American Academy of Pediatrics 393


PREP® Self-Assessment PREPSA 2023

Suggested Readings
• Faigenbaum AD, Lloyd RS, Myer GD. Youth resistance training: past practices, new
perspectives, and future directions. Pediatr Exerc Sci. 2013;25(4):591-604.
doi:10.1123/pes.25.4.591.
• Myer GD, Quatman CE, Khoury J, Wall EJ, Hewett TE. Youth versus adult
“weightlifting” injuries presenting to United States emergency rooms: accidental versus
nonaccidental injury mechanisms. J Strength Cond Res. 2009;23(7):20542060. doi:
10.1519/JSC.0b013e3181b86712.
• Suggested Reading 3 Stricker PR, Faigenbaum AD, McCambridge TM; Council on
Sports Medicine and Fitness. Resistance training for children and adolescents. Pediatrics.
2020;145(6):e20201011. doi:10.1542/peds.2020-1011.

American Academy of Pediatrics 394


PREP® Self-Assessment PREPSA 2023

Question 117
A previously healthy, 7-year-old girl is seen in the office for right knee pain and swelling that
developed and has been worsening over the past several days. She also has pain while walking.
She had an episode of acute pharyngitis 3 weeks ago that resolved without intervention. On
physical examination, the girl has normal vital signs for her age. She has erythema and swelling
of the right knee with pain on motion. On chest auscultation, there is a long, highpitched systolic
murmur at the apex. There is an erythematous, serpiginous rash throughout her trunk that
blanches. The remainder of her examination findings are unremarkable.

Of the following, the BEST test to establish the child’s diagnosis is


A. antistreptolysin O antibody titer
B. C-reactive protein
C. echocardiography
D. rheumatoid factor

American Academy of Pediatrics 395


PREP® Self-Assessment PREPSA 2023

Correct Answers: A
The child in the vignette meets several of the modified Jones criteria for diagnosing acute
rheumatic fever (ARF): arthritis, erythema marginatum, and carditis. To definitively make the
diagnosis, however, one needs to also demonstrate evidence of a prior group A streptococcal
(GAS) infection. The best test to establish the diagnosis of ARF is antistreptolysin O titer. A C-
reactive protein level and echocardiography may be useful in the complete assessment and
management of the condition but will not clarify the underlying diagnosis. In general, a
rheumatoid factor level is unlikely to be helpful in the diagnosis of knee pain and swelling in a
young child and is not recommended to make the diagnosis in this child with a clinical picture
suggestive of ARF.

Acute rheumatic fever is a systemic inflammatory autoimmune reaction that manifests 2 to 4


weeks after a GAS infection of the pharynx. It most commonly occurs in children 5 to 15 years
of age and in those with crowded living conditions. Children with ARF can develop rheumatic
carditis that can result in rheumatic heart disease (RHD). Worldwide, RHD has a profound effect
on morbidity and mortality in this age group.

Children with an acute GAS pharyngitis typically have symptoms such as acute sore throat,
odynophagia, fever, and headache. In addition, some will experience nausea, vomiting, and
abdominal pain. Physical examination findings may be notable for tonsillopharyngeal erythema
with or without exudate, anterior cervical lymphadenopathy, soft palate petechiae, erythema and
swelling of the uvula, and/or scarlatiniform rash. Group A streptococcal pharyngitis can be
diagnosed by rapid antigen test or throat culture. Diagnosis of a recent, resolved GAS infection
can be made with antistreptolysin O titers or other streptococcal antibody levels (eg, anti–
deoxyribonuclease B).

The diagnosis of ARF is made using the modified Jones criteria (Item C117A), including
documentation of a previous GAS infection and whether the child is from a high- or low-risk
area. Low risk is defined as having an ARF incidence less than 2 per 100,000 school-aged
children (usually 5-14 years of age) per year or an all-age RHD prevalence of 1 or less per 1000
population per year.

Treatment of ARF is mostly supportive and includes anti-inflammatory medications (eg, aspirin)
for the arthritis and carditis. Physicians may consider prescribing prednisone in this context,
although there is a lack of evidence to support its use over aspirin. The natural history of ARF is
that the symptoms last for approximately 3 months and span the spectrum from complete
recovery to severe valvular disease, leading to heart failure. Some affected children will need
short-term treatment of heart failure, including diuretics and angiotensin-converting enzyme
inhibitors; a small number will need long-term treatment. The core pillar of therapy is prevention
of ARF with timely antibiotic treatment of GAS pharyngitis (Item C117B). Treatment of GAS
pharyngitis (typically intramuscular penicillin G benzathine) should be administered at the time
of diagnosis of ARF, even if there is no active pharyngitis and throat culture results are negative.
Recurrent GAS infections significantly increase the risk of ARF and subsequent cardiac
involvement. Therefore, secondary prophylaxis with intramuscular (preferred) or oral penicillin
American Academy of Pediatrics 396
PREP® Self-Assessment PREPSA 2023

is the standard of care; the duration (years) is dependent on the severity of disease; alternatives
for those who are allergic to penicillin include sulfadiazine or a macrolide or azalide.

Item C117A

American Academy of Pediatrics 397


PREP® Self-Assessment PREPSA 2023

Item C117B: Antibiotic treatment of GAS pharyngitis

PREP Pearls
• Acute rheumatic fever is a systemic inflammatory autoimmune reaction that manifests 2
to 4 weeks after a group A streptococcal infection of the pharynx, most commonly in
children 5 to 15 years of age and in those who live in crowded conditions.
• Diagnosis of acute rheumatic fever is made using the modified Jones criteria, which
require diagnosis of a recent or resolved group A streptococcal infection; if an acute
infection was not documented by rapid antigen test or throat culture, the diagnosis may
be made with antistreptolysin O titers or other streptococcal antibodies (eg, anti–
deoxyribonuclease B).
• Management of acute rheumatic fever includes treatment with anti-inflammatory
medications (eg, aspirin), eradication of residual group A streptococcal infection, as well
as secondary prophylaxis with long-term penicillin therapy.

ABP Content Specifications(s)


• Plan the appropriate initial management of rheumatic fever
• Plan appropriate diagnostic evaluation of rheumatic fever

American Academy of Pediatrics 398


PREP® Self-Assessment PREPSA 2023

Suggested Readings
• Ganesan K, Guilherme L. Acute rheumatic fever. Lancet. 2018;392:161-174.
doi:10.1016/S0140-6736(18)30999-1.
• Gewitz MH, Baltimore RS, Tani LY, et al. Revision of the Jones criteria for the diagnosis
of acute rheumatic fever in the era of Doppler echocardiography: a scientific statement
from the American Heart Association. Circulation. 2015;131:18061818.
doi:10.1161/CIR.0000000000000205.
• Lahrir S, Sanyahumbi A. Acute rheumatic fever. Pediatr Rev. 2021;42(5):221-232.
doi:10.1542/pir.2019-0288.

American Academy of Pediatrics 399


PREP® Self-Assessment PREPSA 2023

Question 118
A 2-year-old, previously healthy boy is brought to the emergency department for difficulty
walking. Two weeks ago he developed an upper respiratory infection that self-resolved. Over the
past 4 days, he has had difficulty standing and walking, and requires support to take steps such as
holding onto furniture or his parents’ hands. When he sits, he nods his head and has trouble
feeding himself. His parents deny recent trauma or ingestion.

On physical examination, the boy is alert and playful in his mother’s lap, resting against her. He
orients when his name is called. His pupils are equal, round, and reactive to light, extraocular
movements are intact with a few beats of end gaze nystagmus bilaterally, facies are symmetric,
and tongue is midline. Muscles have normal bulk, tone, and strength with symmetric, full
movements. Deep tendon reflexes are initially difficult to elicit but are 2 throughout; toes are
downgoing to plantar stimulation. Sensation is intact. When reaching for objects he has mild
dysmetria bilaterally. He maintains a seated posture without support but has titubation. He can
walk with significant support (holding his mother’s hands) with a wide-based gait.

Results of a complete blood cell count, comprehensive metabolic panel, toxicology screen (urine
and serum), and brain magnetic resonance imaging are normal.

Of the following, the MOST likely diagnosis for this boy is


A. acute cerebellar ataxia
B. acute disseminated encephalomyelitis
C. developmental coordination disorder
D. Miller-Fisher syndrome

American Academy of Pediatrics 400


PREP® Self-Assessment PREPSA 2023

Correct Answers: A
The boy in the vignette has acute cerebellar ataxia, a self-limited, postinfectious disorder
primarily affecting children younger than 5 years. Affected children exhibit truncal and gait
ataxia, often coming to medical attention due to the inability to stand or sit without support.
Dysmetria and titubation (head nodding) while sitting are further evidence of cerebellar
dysfunction. Affected children are otherwise well appearing with normal mental status.
Symptoms are usually maximal at onset and gradually improve over days to weeks, rarely
months. Management is supportive. There is no evidence that immune therapy (eg,
corticosteroids or intravenous immunoglobulin [IVIG]) affects outcome.

Ataxia is a specific type of balance disorder characterized by an absence of coordinated


movements. The differential diagnosis of ataxia is broad, and tailored based on the child’s age,
accompanying signs and symptoms, and time course (Item C118). Ataxia can be localized to
multiple levels of the nervous system. A careful history and physical examination help to
localize the causative lesion, narrow the differential diagnosis, and aid in planning the diagnostic
evaluation. Initial testing, especially with acute ataxia, is aimed at the most serious diagnostic
considerations followed by the most treatable. In addition to the broad neurologic differential
diagnosis of ataxia, non-neurological causes, such as toxic ingestion or labrynthitis, should be
considered.

American Academy of Pediatrics 401


PREP® Self-Assessment PREPSA 2023

The diagnostic evaluation of ataxia is tailored to the individual differential diagnosis. Toxicology
screening should be obtained in the setting of acute ataxia, even in the absence of known
ingestion. Lumbar puncture with cerebrospinal fluid analysis should be performed if infectious
or post-infectious etiologies (eg, Miller-Fisher variant of Guillain-Barré syndrome [GBS]) are
suspected. Neuro-imaging should be performed in the presence of focal neurological findings or
if there is an inability to perform a reliable neurological examination. Brain magnetic resonance
imaging (MRI) is indicated to evaluate for inflammation, ischemia, hemorrhage, or mass lesion.
Magnetic resonance imaging of the spine is indicated if inflammatory conditions such as acute
disseminated encephalomyelitis (ADEM), GBS, or transverse myelitis are suspected.

Acute disseminated encephalomyelitis is an immune-mediated, post/para-infectious


demyelinating disorder of the central nervous system that can present with balance disturbances
and ataxia. Neurological signs and symptoms of ADEM are broad and can include headache,
focal weakness, ataxia, cranial nerve palsies, vision loss, or seizures. Affected children are
encephalopathic. Brain and spine MRI in children and adolescents with ADEM demonstrates
hyperintensities in the involved white matter. Treatment is with intravenous corticosteroids;
IVIG or plasmapheresis are used in more refractory cases.

Developmental coordination disorder (DCD) describes chronic coordination disturbances (ie,


clumsiness) in otherwise normal children. Affected children typically have gross and fine motor
skill deficits that impact their activities of daily living and academic performance.

Developmental coordination disorder can co-occur with mood disorders,


attentiondeficit/hyperactivity disorder, and learning disabilities, and negatively affect self-esteem
and peer relationships. Evaluation should exclude alternate causes for the coordination
disturbance. Management is supportive and may include physical and occupational therapy,
screening for and treating comorbidities, and educational and physical accommodations.

Miller-Fisher syndrome is a variant of GBS characterized by the triad of ataxia, oculomotor


palsies, and absent deep tendon reflexes. In young children, GBS can present with an acute
progressive ataxia. In these cases, serial assessment of deep tendon reflexes is an important
diagnostic tool. Cerebrospinal fluid analysis classically demonstrates a high protein level with a
normal white blood cell count, however, normal results do not exclude the condition. Treatment
with IVIG hastens recovery, but has not been clearly shown to affect overall prognosis.

PREP Pearls
• The differential diagnosis of balance disturbances in childhood is broad and can localize
anywhere within the nervous system. A careful history and physical examination are
critical for appropriate localization, evaluation, and diagnosis.
• Ataxia refers to an absence of coordination and can be acute, subacute, or chronic
depending on the time course of symptoms.
• Acute cerebellar ataxia is a self-limited, post-infectious disorder presenting with acute
onset of truncal and gait ataxia, often in young children. Treatment is supportive.

American Academy of Pediatrics 402


PREP® Self-Assessment PREPSA 2023

ABP Content Specifications(s)


• Formulate a differential diagnosis of balance disturbance in patients of various ages

Suggested Readings
• Blank R, Barnett AL, Cairney J, et al. International clinical practice recommendations on
the definition, diagnosis, assessment, intervention, and psychosocial aspects of
developmental coordination disorder. Dev Med Child Neurol. 2019;61(3):242-285.
doi:10.1111/dmcn.14132.
• Overby P, Kapklein M, Jacobson R. Acute ataxia in children. Pediatr Rev.
2019;40(7):332-343. doi:10.1542/pir.2017-
• Overby P. Ataxia. In: McInerny TK, Adam HM, Campbell DE, DeWitt TG, Foy JM,
Kamat DM, eds. American Academy of Pediatrics Textbook of Pediatric Care. American
Academy of Pediatrics; 2021:chap 130. Accessed September 1, 2022. Pediatric Care
Online.

American Academy of Pediatrics 403


PREP® Self-Assessment PREPSA 2023

Question 119
A 13-year-old adolescent girl is seen in the office for concern about absent menses. She has been
healthy except for multiple ear infections as an infant. The girl is doing well in school and
participates in club soccer. Her sexual maturity rating is 1 for breast development and 3 for pubic
hair. The remainder of the girl’s physical examination findings are unremarkable. Her growth
charts are shown in Item Q119A and Item Q119B.

Of the following, the test/assessment MOST likely to reveal this girl’s diagnosis is a
A. bone age radiograph
B. calorie intake and expenditure assessment
C. karyotype
D. tissue transglutaminase IgA antibody level

American Academy of Pediatrics 404


PREP® Self-Assessment PREPSA 2023

Correct Answer: C
The girl in the vignette has Turner syndrome. The best test to reveal her diagnosis is a karyotype,
which would show an absent or missing part of a sex chromosome (45,X or a variant). Her short
stature, absent breast development, and history of multiple ear infections as an infant are all
features of Turner syndrome.

Although Turner syndrome may be implied by the clinical presentation, a karyotype is integral in
making the diagnosis. In addition, it is important to determine whether there is any mosaicism
with Y chromosomal material present, as this increases the risk of gonadal tumors.
Turner syndrome affects 25 to 50 per 100,000 females. Classically, in the newborn period, girls
with Turner syndrome have swelling of the hands and feet.

Physical examination findings may include short stature, low-set ears, a high-arched palate, low
posterior hairline, webbed neck, short fourth metacarpal bones, a shield chest, and increased
carrying angle of the elbow. Intelligence is typically normal, although there may be difficulty
with spatial tasks. A hallmark of classic Turner syndrome is primary gonadal failure, although
some girls will start puberty and later have premature ovarian failure. Girls with genetic
mosaicism may have only a few features of Turner syndrome including isolated short stature.
One should investigate for heart murmur and peripheral pulses given the risk for aortic and aortic
valve disease. Echocardiography should also be performed and repeated periodically as
hypertension and aortic root dilatation can arise over time. At the time of diagnosis, girls with
Turner syndrome should have renal ultrasonography performed to look for abnormal findings
such as horseshoe kidney.

Delayed puberty in girls is defined as lack of breast development by age 13 years. In adolescents
with delayed puberty, measurement of gonadotropins (luteinizing hormone and follicle-
stimulating hormone [FSH]) is indicated to determine whether the pubertal failure is primary
(ovarian/testicular) or secondary (hypothalamic-pituitary). In adolescent girls with Turner
syndrome, the FSH is often elevated due to primary ovarian failure. The girl in the
vignette has a sexual maturing rating of 3 for pubic hair, which is secondary to adrenal androgen
production.

Bone age radiography is an important part of the evaluation of short stature and delayed puberty.
Typically, conditions that delay puberty also delay bone age as estradiol is the primary hormone
responsible for growth plate maturation. Although bone age radiography may help in treatment
planning for the girl in the vignette, it is not specific enough to determine the etiology of her
delayed puberty. A delayed bone age in an otherwise healthy child or adolescent with a normal
growth rate aids in the diagnosis of constitutional delay of growth and puberty.

Apart from her short stature, the girl in the vignette also has low weight-for-age and sex. She is
also an athlete. At first glance, one may want to perform a dietary and exercise history to be sure
undernutrition is not the cause of her short stature. This history is important because adequate
caloric intake is vital for child and adolescent growth. Although the girl in the vignette has a low

American Academy of Pediatrics 405


PREP® Self-Assessment PREPSA 2023

weight compared to her peers, her body mass index is normal, indicating that her weight is
appropriate for her short stature.

As part of the evaluation for poor growth, especially in the setting of low weight, chronic
illnesses must be ruled out. This evaluation may include measurement of inflammatory markers
and screening for celiac disease with a tissue transglutaminase IgA antibody level. Some children
with celiac disease present only with poor growth and will not have the classic abdominal
symptoms associated with this condition. The body mass index of the girl in the vignette is
normal, making celiac disease less likely.

PREP Pearls
• Girls with short stature and delayed puberty should be evaluated for Turner syndrome.
• Turner syndrome should not be diagnosed clinically; a karyotype is important to make a
definitive diagnosis and identify any Y chromosomal material (which is associated with
an increased risk of gonadal tumors).
• After the diagnosis of Turner syndrome is made, echocardiography and renal
ultrasonography are indicated to look for associated anomalies.

ABP Content Specifications(s)


• Plan the appropriate evaluation of an adolescent boy or girl who has no signs of the onset
of puberty

Suggested Readings
• Gravholt CH, Andersen NH, Conway GS, et al. Clinical practice guidelines for the care
of girls and women with Turner syndrome: proceedings from the 2016 Cincinnati
International Turner Syndrome Meeting. Eur J of Endocrinol. 2017;177(3):G1-G70.
doi:10.1530/EJE-17-0430.
• Howell JC, Backeljauw P. Turner syndrome and Noonan syndrome. In: McInerny TK,
Adam HM, Campbell DE, DeWitt TG, Foy JM, Kamat DM, eds. American Academy of
Pediatrics Textbook of Pediatric Care. American Academy of Pediatrics; 2021:chap 342.
Accessed September 1, 2022. Pediatric Care Online.
• Kaplowitz PB. Delayed puberty. Pediatr Rev. 2010;31(5):189-195. doi:10.1542/pir.31-5-
189.
• Kritzler RK, Long D, Plotnick L. Puberty: normal and abnormal. In: McInerny TK,
Adam HM, Campbell DE, DeWitt TG, Foy JM, Kamat DM, eds. American Academy of
Pediatrics Textbook of Pediatric Care. American Academy of Pediatrics; 2021:chap 185.
Accessed September 1, 2022. Pediatric Care Online.
• Milbrandt T, Thomas E. Turner syndrome. Pediatr Rev. 2013;34(9):420-421.
doi:10.1542/pir.34-9-420.
• Samango-Sprouse C, Song SQ, Lin AE, et al. Klinefelter syndrome and Turner
syndrome. Pediatr Rev. 2021;42(5):272274. doi:10.1542/pir.2020-004028.

American Academy of Pediatrics 406


PREP® Self-Assessment PREPSA 2023

Question 120
A 2-day-old neonate is transferred to the neonatal intensive care unit for a dusky appearance. On
physical examination, he has a heart rate of 160 beats/min, respiratory rate of 60 breaths/min,
blood pressure of 70/55 mm Hg in the right arm, and right upper and lower extremity oxygen
saturations of 78% in room air. He is tachypneic but comfortable, without increased work of
breathing. Breath sounds are clear. He has a normal S1, a single S2, and a continuous murmur
heard in the left side of the chest; the liver edge is palpable 2 cm below the right costal margin;
and his extremities are warm and well perfused. A chest radiograph is obtained (Item Q120).

Of the following, the BEST next step in this neonate’s management is to


A. initiate epinephrine infusion
B. initiate inhaled nitric oxide
C. initiate prostaglandin infusion
D. intubate for respiratory failure

American Academy of Pediatrics 407


PREP® Self-Assessment PREPSA 2023

Correct Answer: C
The neonate in the vignette is cyanotic, with a room air oxygen saturation of 78% measured in
the right upper and lower extremities. He is comfortably tachypneic with good perfusion. On
auscultation, he has clear breath sounds, a single S2, and a continuous murmur. The chest
radiograph (Item C120A) shows oligemic lungs without infiltrate, a boot-shaped heart, and a
right aortic arch. These findings are consistent with tetralogy of Fallot with pulmonary atresia.
Although this diagnosis will need to be confirmed by echocardiography, for this ductal-
dependent condition it is important to first initiate prostaglandin infusion to prevent further
closure of the patent ductus arteriosus (PDA). The neonate does not have evidence of low cardiac
output, so administration of epinephrine is not indicated. He does not have evidence of
respiratory failure, so intubation is not indicated. Inhaled nitric oxide would not correct his
hypoxemia.

The mnemonic 5 Ts and H is helpful for remembering many of the types of cyanotic congenital
heart disease:
• Tetralogy of Fallot
• Total anomalous pulmonary venous connection
• Transposition of the great arteries
• Tricuspid atresia
• Truncus arteriosus
• Hypoplastic left heart syndrome

Knowledge of each child’s physiology is needed to understand the expected oxygen saturations.
Children who are cyanotic because of their cardiac anatomy (Item C120B) have insufficient
pulmonary blood flow, a mixing lesion, or transposition physiology. Children with obstruction to
systemic blood flow will have signs of shock and decreased cardiac output; their oxygen
saturations can be variable.

American Academy of Pediatrics 408


PREP® Self-Assessment PREPSA 2023

The term critical congenital heart disease indicates that the child’s circulation is dependent on a
PDA. The neonate in the vignette has ductal-dependent pulmonary blood flow, which is
pulmonary blood flow dependent on systemic blood flow through the PDA. In congenital heart
disease with ductal-dependent systemic blood flow, the systemic blood flow is dependent on
blood flow from the pulmonary artery through a PDA (eg, hypoplastic left heart syndrome).
In transposition of the great arteries (TGA), the 2 parallel circulations result in inadequate
mixing of oxygenated and deoxygenated blood. Patency of the ductus arteriosus allows for
shunting of blood from the aorta to the pulmonary artery, which increases the blood returning to
the left atrium and increases shunting across an atrial-level defect (eg, patent foramen ovale,
atrial septal defect). The shunting at the level of the PDA, alone, for TGA, is insufficient, and an
unrestrictive atrial septal defect is needed. Neonates with a restrictive or no atrial septal defect
will need to undergo urgent atrial septostomy for adequate mixing.

Patency of the ductus arteriosus is maintained with a continuous infusion of prostaglandin; if the
ductus has already closed, higher doses are required to reopen it. Adverse effects of
prostaglandin infusion, which can be dose dependent, include apnea, fever, tachycardia,
vasodilation, and hypotension.

Tetralogy of Fallot consists of a ventricular septal defect with anterior malalignment of the
interventricular septum, overriding aorta, pulmonary stenosis, and right ventricular hypertrophy.
Like many types of congenital heart disease, the physiology of tetralogy of Fallot is dependent
on each child’s anatomy. At one extreme, there is minimal pulmonary stenosis and anterior
malalignment of the interventricular septum. These children have normal oxygen saturations
with physiology similar to that of a child with a ventricular septal defect. At the other extreme is
tetralogy of Fallot with pulmonary atresia and major aortopulmonary collateral arteries, where
abnormal collateral vessels are the sources of pulmonary blood flow and pulmonary arteries do
not develop. The more the pulmonary blood flow is inhibited, the more hypoxemic the child will
be. In addition, there is a form of tetralogy of Fallot with absent pulmonary valve that results in
American Academy of Pediatrics 409
PREP® Self-Assessment PREPSA 2023

to-and-fro flow and abnormal airway development; these children often have breathing problems
at birth and many require tracheostomy and long-term mechanical ventilatory support.

Children with critical congenital heart disease are symptomatic in the newborn period, typically
in the first few days to weeks after birth. Children with congenital heart disease who have new
hypoxemia later in life have palliated single-ventricle disease, Eisenmenger syndrome, or other
causes of pulmonary hypertension with an intracardiac shunt.

PREP Pearls
• Cyanotic congenital heart disease due to decreased pulmonary blood flow can present
with comfortable tachypnea, desaturation, and a chest radiograph showing clear lungs.
• Prostaglandin infusion should be initiated before further diagnostic imaging when critical
congenital heart disease is suspected to maintain patency of the ductus arteriosus.
• Tetralogy of Fallot presents along a continuum of physiologies based on the underlying
anatomy.

ABP Content Specifications(s)


• Identify cardiac causes of cyanosis in children of various ages, including those who have
tetralogy of Fallot

Suggested Readings
• Desai K, Rabinowitz EJ, Epstein S. Physiologic diagnosis of congenital heart disease in
cyanotic neonates. Curr Opin Pediatr. 2019;31(2):274-283.
doi:10.1097/MOP.0000000000000742.
• Puri K, Allen HD, Qureshi AM. Congenital heart disease. Pediatr Rev. 2017;38(10):471-
486. doi:10.1542/pir.2017-0032.
• Silberbach M, Hannon D. Presentation of congenital heart disease in the neonate and
young infant. Pediatr Rev. 2007;28(4):123-31. doi:10.1542/pir.28-4-123.
• Ushay HM. Cyanosis. In: McInerny TK, Adam HM, Campbell DE, DeWitt TG, Foy JM,
Kamat DM, eds. American Academy of Pediatrics Textbook of Pediatric Care. 2nd ed.
Itasca, IL: American Academy of Pediatrics; 2021:1252-1258. Pediatric Care Online .

American Academy of Pediatrics 410


PREP® Self-Assessment PREPSA 2023

Question 121
A 15-year-old adolescent girl is brought to the emergency department for a 48-hour history of
blurry vision and pain with movement of the left eye. Today, she is unable to see anything out of
her left eye. She complains of mild, left frontal headache but denies fever or eye discharge.
There is no history of recent illness or eye injury. Her vital signs are normal, and she is in no
acute distress. Her cornea and conjunctivae are clear without discharge or injection. She has mild
pain with movement of her left eye but has full range of extraocular movement. When light is
directed at her right pupil, there is normal constriction of both pupils. When light is directed at
her left pupil, both pupils remain dilated. Fundoscopy shows blurred optic disc margins on the
left with sharp margins on the right. The remainder of her physical examination findings are
normal.

Of the following, the MOST likely diagnosis for this adolescent is


A. migraine variant
B. optic neuritis
C. orbital cellulitis
D. pseudotumor cerebri

American Academy of Pediatrics 411


PREP® Self-Assessment PREPSA 2023

Correct Answer: B
The adolescent in the vignette has findings consistent with left optic neuritis. The swinging
flashlight test, also known as the Marcus Gunn pupil test, is performed to detect an afferent
pupillary defect. The test is performed by shining a light on one eye for 2 seconds and then
alternating the light from eye to eye. Normally, both pupils constrict in response to shining the
light on one pupil. If the pupils remain dilated, an afferent pupillary defect is likely, indicating
disease of the retina or optic nerve. The adolescent has a lack of pupillary response on the left
side with a normal direct and consensual response on the right side, indicating a left afferent
pupillary defect. This finding, along with blurring of her left optic disc, makes optic neuritis the
most likely diagnosis.

Optic neuritis is relatively uncommon in children. It presents as acute or subacute vision loss.
Younger children are more likely to present with bilateral vision loss. The etiologies of optic
neuritis can be grouped into infectious (eg, Lyme disease, cat scratch disease, mycoplasma, and
tuberculosis), autoimmune (eg, systemic lupus erythematosus), and demyelinating diseases (eg,
multiple sclerosis, acute disseminated encephalomyelitis). Optic neuritis can be a clinical
manifestation of sarcoidosis. At initial presentation for about 20% of children and adolescents
with multiple sclerosis and neuromyelitis optica, optic neuritis is the sole clinical feature.

Magnetic resonance imaging of the brain and orbits should be performed in children and
adolescents who have symptoms and signs of optic neuritis to look for structural brain lesions
and white matter changes. Optic nerve enhancement on contrast imaging confirms the diagnosis
of optic neuritis. Cerebrospinal fluid should be analyzed for oligoclonal bands, IgG index, and
evidence of infection. Further evaluation for infections or autoimmune conditions should be
performed based on individual history and physical examination findings.

The first-line treatment of pediatric optic neuritis is high-dose, pulsed methylprednisolone for 3
to 5 days followed by a taper over 2 weeks. Children and adolescents resistant to steroid therapy
may benefit from treatment with intravenous immunoglobulin and plasmapheresis.

The differential diagnosis of an acute visual deficit includes trauma to the eye, orbital infection,
increased intracranial pressure, intracranial lesions, stroke, optic neuritis, migraine variants,
cavernous sinus thrombosis, and exposure to carbon monoxide. The adolescent in the vignette
has no history of upper respiratory symptoms and no findings of periorbital swelling or
restriction of eye movements to support a diagnosis of orbital cellulitis. Although migraine could
be a consideration for the girl in the vignette with her headache and blurry vision, it would not
explain her afferent pupillary defect. Pseudotumor cerebri presents with headache and a gradual
history of blurry vision progressing to vision loss with blurred optic discs bilaterally on
fundoscopic examination; this clinical picture is not consistent with the presentation of the
adolescent in the vignette.

American Academy of Pediatrics 412


PREP® Self-Assessment PREPSA 2023

PREP Pearls
• Optic neuritis can present with acute vision loss and findings of an afferent pupillary
defect with blurring of the optic disc margins.
• Evaluation of children and adolescents with suspected optic neuritis should include
magnetic resonance imaging of the brain and orbits.
• Optic neuritis can be the initial clinical presentation of demyelinating disorders such as
multiple sclerosis.

ABP Content Specifications(s)


Recognize the clinical findings associated with visual impairment

Suggested Readings
• Borchert M, Liu GT, Pineles S, Waldman AT. Pediatric optic neuritis: what is new. J
Neuroophthalmol. 2017;37 Suppl 1(Suppl 1):S14-S22.
doi:10.1097/WNO.0000000000000551.
• Chang MY, Pineles SL. Pediatric optic neuritis. Semin Pediatr Neurol. 2017;24(2):122-
128. doi:10.1016/j.spen.2017.04.004.
• Irwin W, Kienstra A, Vezzetti R. Sudden unilateral vision loss in a teenage girl. Pediatr
Rev. 2021;42:(Supplement 1) S112-S116. doi:10.1542/pir.2019-0110.
• Patel KN. Acute vision loss. Clin Pediatr Emerg Med. 2010;11(2):137-142.
doi:10.1016/j.cpem.2010.05.001.

American Academy of Pediatrics 413


PREP® Self-Assessment PREPSA 2023

Question 122
A 3-year-old girl is brought to the emergency department (ED) for difficulty breathing. She
developed a sudden, severe sore throat and fever today. Her vital signs include a temperature of
40.1°C, heart rate of 176 beats/min, respiratory rate of 26 breaths/min, blood pressure of 76/59
mm Hg, and oxygen saturation of 91% in room air. On arrival at the ED, the girl appears toxic
and has severe respiratory distress and difficulty swallowing. She is sitting upright and holding
her head in an extended position while resting her arms on her knees. She has stridorous
respirations associated with tachypnea and suprasternal retractions.

Of the following, the BEST next step in this girl’s management is


A. administration of intravenous ceftriaxone
B. administration of intravenous corticosteroid
C. administration of nebulized racemic epinephrine
D. rapid sequence induction and intubation

American Academy of Pediatrics 414


PREP® Self-Assessment PREPSA 2023

Correct Answer: D
The girl in the vignette is displaying signs and symptoms of airway obstruction concerning for
epiglottitis. The best next step in her management is rapid sequence induction and intubation.
Other interventions, such as nebulizer treatments, intravenous corticosteroids or antibiotics, or
imaging studies should not be undertaken until a definitive airway is secured. Every effort should
be made to keep the child as calm and quiet as possible until the airway is secured.

Epiglottitis, also known as supraglottis, is a rare life-threatening condition that results in severe
swelling of the epiglottis, which can rapidly lead to complete airway obstruction (Item C122A).
The critically important first step in the management of a child with suspected epiglottitis is
obtaining a definitive and secure airway with an endotracheal tube (not a laryngeal mask
airway). The presence of stridor is a late finding and can indicate near complete airway
obstruction. The physician performing the intubation of a child with suspected epiglottitis should
have expertise with managing a difficult airway and have the proper backup personnel and
equipment in place, including the capability to establish a surgical airway if necessary.
Intubation for epiglottitis is ideally performed in a controlled setting such as an operating room.

The clinical presentation of epiglottis is an overtly toxic-appearing child with sudden onset and
rapid progression of fever, drooling, stridor, muffled voice, and difficulty swallowing. The
classic diagnostic triad, the “3 Ds,” includes drooling, dysphagia, and distress. Affected children
will often sit in the “tripod” position with their neck extended in an effort to maintain a patent
airway.

Although the introduction of the Haemophilus influenzae type b vaccine drastically reduced the
incidence of epiglottitis, H influenzae type b remains the most common cause. Other infectious
etiologies include Staphylococcus aureus, Streptococcus pneumoniae, as well as viral and fungal
infections. Epiglottitis can also be caused by noninfectious processes such as thermal injury
(from swallowing hot liquids or inhaling hot gasses), caustic ingestion, or
direct trauma.

The diagnosis of epiglottitis is made clinically in a child with classic symptoms, and prompt
intubation should be arranged; no further evaluation should be undertaken. A child with

American Academy of Pediatrics 415


PREP® Self-Assessment PREPSA 2023

suspected epiglottitis, even when calm and stable, should be kept in an upright position; an
oropharyngeal examination should not be performed, as it may upset the child and result in acute
airway obstruction. If the diagnosis of epiglottis is not likely, and a child is calm and cooperative
with mild to no respiratory distress, an upright lateral neck radiograph may be obtained to
evaluate for other diagnoses (eg, foreign body). The “thumbprint” sign is the classic finding on
lateral neck radiograph for epiglottitis, which results from a rounded, enlarged epiglottis
extending into the hypopharyngeal airway (Item C122B).

Once a secure airway has been established, intravenous antimicrobial therapy with coverage for
H influenzae type b and methicillin-resistant S aureus should be initiated. One suggested
antimicrobial regimen is a third-generation cephalosporin (eg, ceftriaxone or cefotaxime) plus
vancomycin or clindamycin. As the efficacy of corticosteroids has not been established in the
management of epiglottitis, they are not currently recommended.

PREP Pearls
• Epiglottitis is a rare life-threatening condition that results in severe swelling of the
epiglottis, which can lead to complete airway obstruction.
• The clinical presentation of epiglottis is an overtly toxic-appearing child with sudden
onset and rapid progression of fever, drooling, stridor, muffled voice, and difficulty
swallowing. The classic diagnostic triad, the “3 Ds,” includes drooling, dysphagia, and
distress.
• The critically important first step in the management of a child with suspected epiglottitis
is obtaining a definitive and secure airway with an endotracheal tube.

ABP Content Specifications(s)


• Plan the appropriate management of epiglottitis
• Plan the appropriate clinical and laboratory evaluation of epiglottitis

American Academy of Pediatrics 416


PREP® Self-Assessment PREPSA 2023

Suggested Readings
• American Academy of Pediatrics. Haemophilus influenzae infections. In: Kimberlin DW,
Barnett ED, Lynfield R, Sawyer MH, eds. Red Book: 2021–2024 Report of the
Committee on Infectious Diseases. 32nd ed. American Academy of Pediatrics; 2021.
Accessed September 1, 2022. Red Book Online.
• Conrad, CK, Cornfield DN. Airway obstruction. In: McInerny TK, Adam HM, Campbell
DE, DeWitt TG, Foy JM, Kamat DM, eds. American Academy of Pediatrics Textbook of
Pediatric Care. American Academy of Pediatrics; 2021:chap 348. Accessed September 1,
2022. Pediatric Care Online.
• Virbalas J, Smith L. Upper airway obstruction. Pediatr Rev. 2015;36(2):62-73.
doi:10.1542/pir.36.2.62.

American Academy of Pediatrics 417


PREP® Self-Assessment PREPSA 2023

Question 123
A 14-year-old adolescent boy is seen for a preparticipation physical examination to play soccer.
He is healthy with no concerns voiced by him or his parents. His physical examination findings
are normal except for a systolic ejection click heard best at the right upper sternal border. Further
inquiry reveals that the boy’s paternal grandfather had aortic valve surgery at age 55 years. The
boy is referred to a pediatric cardiologist.

Of the following, the boy’s MOST likely diagnosis is


A. bicuspid aortic valve
B. partial anomalous pulmonary venous return
C. pulmonary valve stenosis
D. ventricular septal defect

American Academy of Pediatrics 418


PREP® Self-Assessment PREPSA 2023

Correct Answers: A
The boy in the vignette has a systolic ejection click heard at the right upper sternal border,
suggestive of aortic valve pathology, and a family history (paternal grandfather) of aortic valve
pathology. Of the response choices, bicuspid aortic valve (BAV) is the most likely diagnosis. He
should be evaluated by a pediatric cardiologist; echocardiography is needed to confirm the
diagnosis. If there is no significant valve dysfunction or aortic dilation, children with BAV can
often play sports and exercise without restriction.

The boy’s physical examination findings and family history are not suggestive of the other
congenital heart diseases listed as response choices. Children with partial anomalous pulmonary
venous return have clinical evidence of increased pulmonary blood flow; the acyanotic child may
be tachypneic with hepatomegaly and a pulmonary murmur due to the relatively increased flow.
The murmur is a systolic ejection murmur best heard at the left upper sternal border. A child with
pulmonary stenosis may have few symptoms, but may have exercise intolerance and a systolic
ejection murmur best heard at the left upper sternal border. A ventricular septal defect should
present in infancy with tachypnea and symptoms of pulmonary overcirculation, depending on the
size of the defect, with a holosystolic murmur at the left sternal border.

Bicuspid aortic valve is a congenital heart disease in which the aortic valve develops with 2
leaflets instead of the normal 3. It is one of the most common congenital anomalies, affecting
~1% to 2% of the population. It is more common in males than females with a 3:1
predominance. Twenty percent to 30% of affected individuals have a family member with BAV
or an associated aortopathy. Individuals with BAV can develop aortic regurgitation, aortic
stenosis, or both. Roughly one-third will develop aortic aneurysms. Abnormalities of the aorta
itself can occur independent of valve function. While most individuals with BAV develop these
sequelae during adulthood, ~10% to 15% will do so in childhood. Life-long serial imaging is
indicated to monitor aortic valve function, ascending aortic size, and for aneurysm formation.

Individuals with BAV and significant aortic valve dysfunction, aortic dilation, or aortic
aneurysm require medical and sometimes surgical intervention. Medication to prevent high blood
pressure (eg, β-blocker) is often prescribed when there is aortic dilation; surgical intervention
may ultimately be indicated. Surgical or interventional (cardiac catheterization) procedures may
be required to relieve obstruction caused by aortic valve stenosis. When the aortic valve is
significantly regurgitant, sometimes surgical procedures to repair the valve are successful, but if
not, the valve needs to be replaced with a prosthetic or mechanical valve. Anticoagulation is
needed after valve replacement.

PREP Pearls
• Bicuspid aortic valve, in which the aortic valve develops with only 2 leaflets instead of
the normal 3, is one of the most common congenital heart defects.
• Bicuspid aortic valve occurs in males more than females and can be familial.
• Individuals with bicuspid aortic valve can develop valve dysfunction (stenosis,
regurgitation, or both), aortic dilation, or aortic aneurysm.

American Academy of Pediatrics 419


PREP® Self-Assessment PREPSA 2023

ABP Content Specifications(s)


• Understand the natural history of a bicuspid aortic valve

Suggested Readings
• Baleilevuka-Hart M, Teng BJ, Carson KA, Ravekes WJ, Holmes KW. Sports
participation and exercise restriction in children with isolated bicuspid aortic valve. Am J
Cardiol. 2020;125(11):1673-1677. doi:10.1016/j.amjcard.2020.02.039.
• Niaz T, Fernandes S, Sanders SP, Michelena H, Hagler DJ. Clinical history and
management of bicuspid aortic valve in children and adolescents. Prog Cardiovasc Dis.
2020;63:425-433. doi:10.1016/j.pcad.2020.05.012.
• Niaz T, Johnson JN, Cetta F, Olson TM, Hagler DJ. Bicuspid aortic valve in infants,
children, and adolescents: a review for primary care providers. Pediatr Rev.
2021;42(5):233-244. doi:10.1542/pir.2019-0307.
• Otto CM, Nishimura RA, Borrow RO, et al. 2020 ACC/AHA Guideline for the
Management of Patients with Valvular Heart Disease: A Report of the American College
of Cardiology/American Heart Association Joint Committee on Clinical Practice
Guidelines. J Circulation. 2021;143(5):e72-e227. doi:10.1161/CIR.0000000000000923.

American Academy of Pediatrics 420


PREP® Self-Assessment PREPSA 2023

Question 124
A 7-year-old girl is evaluated for a 1-week history of a dry, hacking cough. Today, she had an
episode of posttussive emesis. The girl appears well. She has a temperature of 37.5°C, heart rate
of 90 beats/min, respiratory rate of 18 breaths/min, and oxygen saturation of 100% in room air.
Her lungs are clear on auscultation. Bilateral subconjunctival hemorrhages and a few scattered
petechiae are noted over the neck. Her parents are concerned that the girl’s siblings, ages 6
weeks and 5 years, will catch her illness. The children are all up to date with their
immunizations.

Of the following, over the next 21 days, the MOST appropriate postexposure management is to
A. monitor only the 5-year-old sibling for symptoms
B. monitor only the 6-week-old sibling for symptoms
C. start azithromycin treatment for both siblings
D. start trimethoprim-sulfamethoxazole treatment for both siblings

American Academy of Pediatrics 421


PREP® Self-Assessment PREPSA 2023

Correct Answer: C
Pertussis postexposure prophylaxis with azithromycin is recommended for all close contacts of
the index case irrespective of immunization status. Individuals are considered close contacts if
they are either household members or asymptomatic persons who had face-to-face exposure
within 3 feet of a symptomatic individual in a childcare facility, school, or health care setting.
Exposure is defined as direct contact with respiratory, oral, or nasal secretions or sharing the
same space with the symptomatic individual for 1 hour or more. Prompt postexposure
prophylaxis is recommended, given the high transmission risk (secondary attack rate of 80% in
susceptible persons) and effectiveness of azithromycin. Trimethoprim-sulfamethoxazole is
reserved for prophylaxis in children who cannot tolerate azithromycin; it is contraindicated in
infants younger than 2 months. All exposed individuals should be monitored for symptoms for
21 days.

Pertussis is a highly contagious infection caused by the fastidious gram-negative bacillus


Bordetella pertussis. Transmission is through respiratory droplets, with the highest risk seen
during the catarrhal stage. Waning immunity from the acellular pertussis vaccine enhances the
risk in older children, adolescents, and adults. The incubation period ranges from 5 to 21 days.
The clinical presentation of pertussis infection is divided into 3 stages:

• Catarrhal: Upper respiratory symptoms (eg, rhinorrhea); lasts 1 to 2 weeks


• Paroxysmal: Repeated coughing spells followed by an inspiratory whoop in infants and
young children; lasts 2 to 4 weeks. The classic “whooping cough” is less obvious in
infants and adolescents. Infants may present with gagging, cyanosis, or apnea.
Conjunctival hemorrhages and petechiae in the upper body are common due to the severe
cough.
• Convalescent: Cough gradually improves over 4 to 12 weeks.

Serum white blood cell count is elevated due to lymphocytosis driven by pertussis toxin. The
increase in white blood cell count correlates with the severity of infection. The best laboratory
test for diagnostic confirmation is the nucleic acid amplification test (NAAT), which requires a
nasopharyngeal specimen obtained using a synthetic swab. The NAAT has a rapid turnaround
time, and should be performed within 4 weeks of symptoms. Test sensitivity, dependent on age,
is about 70% in infants and young children and less than 10% in adults. The interpretation of
results is hindered by the frequency of false positives, particularly due to cross-reactivity with
Bordetella parapertussis. Culture of the organism is 100% specific, allows strain identification,
and is used for public health purposes. However, sensitivity is much lower (difficult to grow,
particularly in immunized individuals or those taking antibiotics). The culture sample must be
obtained within the first 2 weeks of illness. Diagnostic serologic testing can be considered for a
child in the convalescent phase, however, its use is discouraged. Serum IgG against pertussis
toxin can be hard to interpret in immunized individuals. In addition, serologic testing may not be
available in commercial laboratories. Serum IgM and IgA testing should not be performed in
cases such as the child in the vignette; both lack specificity and sensitivity.

American Academy of Pediatrics 422


PREP® Self-Assessment PREPSA 2023

Treatment will only modify the clinical course if it is started in the paroxysmal phase, when it
can reduce the duration and severity of disease. Macrolides (azithromycin, clarithromycin,
erythromycin) are the first-line antibiotics. Trimethoprim-sulfamethoxazole is reserved for those
aged 2 months and older who cannot tolerate macrolides.

PREP Pearls
• Pertussis is a highly contagious infection with a typical duration of 6 to 10 weeks.
• There are 3 stages of pertussis infection: catarrhal (upper respiratory infection
symptoms), lasting 1 to 2 weeks; paroxysmal (repeated coughing spells), lasting 2 to 4
weeks; and convalescent (improving cough), lasting 4 to 12 weeks.
• Household members and close contacts of individuals diagnosed with pertussis infection
should receive postexposure prophylaxis with azithromycin, irrespective of their
immunization status.

ABP Content Specifications(s)


• Plan the appropriate management of pertussis in its various stages, including treatment
for contacts of infected patients

Suggested Readings
• American Academy of Pediatrics. Pertussis (whooping cough). In: Kimberlin DW,
Barnett ED, Lynfield R, Sawyer MH, eds.
• Red Book: 2021–2024 Report of the Committee on Infectious Diseases. 32nd ed.
American Academy of Pediatrics; 2021. Accessed September 1, 2022. Red Book Online.
• Daniels HL, Sabella C. Bordetella pertussis (pertussis). Pediatr Rev. 2018;39(5):247-257.
doi:10.1542/pir.2017-0229.
• Long SS, Edwards KM, Mertsola J. Bordetella pertussis (pertussis) and other bordetella
species. In: Long SS, ed. Principles and Practice of Pediatric Infectious Diseases. 4th ed.
Elsevier; 2012:865-873.e5.
• Sabella C. Pertussis (whooping cough). In: McInerny TK, Adam HM, Campbell DE,
DeWitt TG, Foy JM, Kamat DM, eds. American Academy of Pediatrics Textbook of
Pediatric Care. American Academy of Pediatrics; 2021:chap 310. Accessed September 1,
2022. Pediatric Care Online.
• Tiwari T, Murphy TV, Moran J. Recommended antimicrobial agents for the treatment
and postexposure prophylaxis of pertussis: 2005 CDC guidelines. MMWR Recomm Rep.
2005;54(RR-14):1-16. https://www.cdc.gov/mmwr/preview/mmwrhtml/rr5414a1.htm.

American Academy of Pediatrics 423


PREP® Self-Assessment PREPSA 2023

Question 125
A 4-day-old male neonate is seen in the office for a health supervision visit. The neonate was
born at term, had an uneventful prenatal and nursery course, and was discharged 2 days after
birth. His mother is planning to exclusively breastfeed and asks if any nutrient supplements are
recommended for her son.

Of the following, the BEST recommendation is


A. start vitamin D and iron supplementation at age 4 months
B. start vitamin D and iron supplementation now
C. start vitamin D supplementation now and iron supplementation at age 4 months
D. vitamin D and iron supplementation are not necessary

American Academy of Pediatrics 424


PREP® Self-Assessment PREPSA 2023

Correct Answer: C
The breastfed neonate in the vignette should start vitamin D supplementation now and iron at age
4 months. Although breast milk provides excellent nutrition during the first year after birth, the
vitamin D and iron content in breast milk is not adequate to meet the infant’s nutritional needs.
Vitamin D levels are low in breastmilk, especially if the mother is vitamin D deficient. Sunlight
exposure varies depending on the location and season, and cannot be depended upon to produce
adequate vitamin D for the growing infant. The American Academy of Pediatrics (AAP)
recommends supplementation of 400 IU of vitamin D to all breastfed (exclusively or partially)
neonates. Formula-fed neonates must consume 32 ounces daily to receive the recommended
amount of vitamin D.

Full-term infants have enough iron stores to last until 4 months of age. Although the
bioavailability of iron from breast milk is better than formula, the iron content in breast milk is
low; it is not sufficient to meet the infant’s requirements beyond age 6 months. Without
supplementation, depletion of iron stores in exclusively breastfed infants will lead to iron
deficiency and progress to iron-deficiency anemia. The AAP recommends iron
supplementation (1 mg/kg/day) for all exclusively breastfed infants until the intake of iron-
containing foods (eg, iron-fortified cereal, meats, lentils) is sufficient to meet requirements. For
partially breastfed infants, the intake of iron through formula and other dietary sources should be
considered prior to supplementation. As most infant formulas contain 10-12 mg/dL iron,
formula-fed infants do not need supplementation. The recommended dietary iron intake is 11
mg/day for infants aged 6 to 12 months.

PREP Pearls
• All breastfed infants should receive vitamin D supplementation of 400 IU daily starting
in the first few days after birth.
• Exclusively breastfed infants should receive iron supplementation (1 mg/kg/day) starting
at age 4 months.
• The amount of formula consumed by partially breastfed infants should be assessed for
sufficiency to meet the dietary requirements for vitamin D and iron.

ABP Content Specifications(s)


• Prevent iron and vitamin D deficiency in breastfed infants

American Academy of Pediatrics 425


PREP® Self-Assessment PREPSA 2023

Suggested Readings
• Baker RD, Greer FR; The Committee on Nutrition. Diagnosis and prevention of iron
deficiency and iron-deficiency anemia in infants and young children (0–3 years of age).
Pediatrics. 2010;126(5):1040-1050. doi:10.1542/peds.2010-2576.
• DiMaggio DM, Cox A, Porto AF. Updates in infant nutrition. Pediatr Rev.
2017;38(10):449-462. doi:10.1542/pir.2016-0239
• Iron. In: Kleinman RE, Greer FR, eds. Pediatric Nutrition. 8th ed. American Academy of
Pediatrics; 2020:561-590.
• Lawrence RM, Lawrence RA. Breastfeeding: more than just good nutrition. Pediatr Rev.
2011;32(7):267-280. doi:10.1542/pir.32-7-267.
• Wagner CL, Greer FR; American Academy of Pediatrics Section on Breastfeeding;
American Academy of Pediatrics Committee on Nutrition. Prevention of rickets and
vitamin D deficiency in infants, children, and adolescents. Pediatrics. 2008;122(5):1142-
1152. doi:10.1542/peds.2008-1862.

American Academy of Pediatrics 426


PREP® Self-Assessment PREPSA 2023

Question 126
A 1-day-old newborn was delivered at 39 weeks’ gestation via normal spontaneous vaginal
delivery to a 21-year-old primigravida mother. The pregnancy was complicated by
oligohydramnios. Prenatal ultrasonography revealed bilateral hydronephrosis. The neonate has
had 1 small void since birth. His vital signs are normal for age, and his physical examination
findings are unremarkable.

Postnatal renal ultrasonography reveals severe bilateral hydroureteronephrosis with a thick-


walled urinary bladder.

Of the following, the BEST next step in this neonate’s evaluation is to order
A. a diuretic renal scan
B. magnetic resonance imaging of the kidneys and bladder
C. repeat renal ultrasonography after 1 week
D. voiding cystourethrography

American Academy of Pediatrics 427


PREP® Self-Assessment PREPSA 2023

Correct Answer: D
The male neonate in the vignette most likely has posterior urethral valves (PUVs). This diagnosis
is supported by the history of bilateral antenatal hydronephrosis, maternal oligohydramnios, and
severe bilateral hydroureteronephrosis with a thick-walled urinary bladder on postnatal renal
ultrasonography. The best next step in his evaluation is to perform voiding cystourethrography
(VCUG). The VCUG would likely demonstrate the classic findings of a dilated posterior urethra
during the voiding phase, trabeculated bladder, and perhaps vesicoureteral reflux (VUR) (Item
C126).

A diuretic renal scan, performed for suspected ureteropelvic and ureterovesical junction
obstruction, is indicated for infants older than 2 months, when maturation of tubular function has
occurred. It is not indicated for the neonate in the vignette. Magnetic resonance imaging of the
kidneys and bladder is not a useful test for diagnosing PUVs. There is no indication for repeating
renal ultrasonography after 1 week because there is no expectation for significant change; this
will only delay confirmation of the neonate’s diagnosis and treatment.

Posterior urethral valves are abnormal mucosal folds within the lumen of the posterior urethra
that obstruct the flow of urine. This results in bladder distention and vesicoureteral reflux into the
kidney. Posterior urethral valves are the most common cause of congenital urethral obstruction in
male neonates, occurring in 1 in 5,000 to 8,000 live births. Posterior urethral valves should be
suspected in a male fetus when prenatal ultrasonography reveals bilateral hydronephrosis, a
dilated bladder, and a dilated posterior urethra. Oligohydramnios may occur in cases of severe
urinary obstruction, which can result in lung hypoplasia. The neonate may present with
abdominal distension (due to a distended urinary bladder and hydronephrosis) and difficulty
voiding. Neonates and infants with PUVs may present with acute pyelonephritis, urosepsis, weak
urinary stream, or failure to thrive. Older boys usually present with nocturnal and daytime
enuresis, voiding dysfunction, and urinary tract infections.

Neonates with suspected PUVs usually require urgent urinary catheter placement to bypass the
obstruction and drain the bladder. Cystoscopy is performed to confirm the diagnosis and ablate

American Academy of Pediatrics 428


PREP® Self-Assessment PREPSA 2023

the PUVs. A vesicostomy may be performed in premature neonates when cystoscopy cannot be
performed because of their small size. Careful monitoring for postobstructive diuresis and
electrolyte disturbances is indicated after the obstruction is relieved.

Long-term management of PUVs includes correction of metabolic abnormalities (eg, acidosis,


hyperkalemia, and hyponatremia), antibiotic prophylaxis for VUR, and treatment of bladder
dysfunction with anticholinergic medications, intermittent catheterization, and nighttime
drainage. Even with prenatal diagnosis and early intervention, children with PUVs are at
increased risk of chronic kidney disease. Regular follow-up with a pediatric nephrologist is
required for monitoring of kidney function, management of chronic kidney disease, and
assessing progression to end-stage renal disease.

PREP Pearls
• Posterior urethral valves are abnormal mucosal folds within the lumen of the posterior
urethra that obstruct the flow of urine.
• Children with suspected posterior urethral valves should have voiding
cystourethrography (VCUG) performed. A dilated posterior urethra on VCUG is
consistent with the diagnosis.
• Children with posterior urethral valves are at increased risk of chronic kidney disease,
even with prenatal diagnosis and early intervention.

ABP Content Specifications(s)


• Recognize the clinical findings associated with posterior urethral valves in children of
various ages
• Plan the appropriate long-term management of posterior urethral valves

Suggested Readings
• Janjua HS, Lam KS, Gupta V, Krishna S. Congenital anomalies of the kidneys, collecting
system, bladder, and urethra. Pediatr Rev. 2019;40(6):619-626. doi:10.1542/pir.2018-
0242.
• Nguyen HT. Obstructive uropathy and vesicoureteral reflux. In: McInerny TK, Adam
HM, Campbell DE, DeWitt TG, Foy JM, Kamat DM, eds. American Academy of
Pediatrics Textbook of Pediatric Care. American Academy of Pediatrics; 2021:chap 299.
Accessed September 1, 2022. Pediatric Care Online.
• Schlomer BJ, Copp HL. Antenatal hydronephrosis. NeoReviews. 2013;14(11):e551-e561.
doi:10.1542/neo.14-11-e551.

American Academy of Pediatrics 429


PREP® Self-Assessment PREPSA 2023

Question 127
A 2-year-old girl is seen for hives of 24 hours’ duration. The girl woke up yesterday with
scattered lesions on her torso. Since onset, the lesions have come and gone in various areas of
her torso. Two weeks ago, she was diagnosed with a viral upper respiratory tract infection and
acute otitis media for which she was prescribed amoxicillin. She finished the amoxicillin 4 days
ago. For the past 2 days, the girl has had a temperature of 38°C, mild cough, and rhinorrhea. She
has continued at her usual activity level and is eating a wide variety of foods. Her medical
history is unremarkable. Her mother has hypothyroidism and her older brother has a peanut
allergy. The girl’s vital signs are normal. Physical examination findings are remarkable only for
clear rhinorrhea and scattered 1- to 5-cm diameter hives on her torso.

Of the following the MOST likely cause of this girl’s hives is


A. amoxicillin
B. food allergy
C. hypothyroidism
D. viral illness

American Academy of Pediatrics 430


PREP® Self-Assessment PREPSA 2023

Correct Answer: D
The most likely cause for the hives exhibited by the girl in the vignette is a viral illness. Viral
infections cause almost 60% of acute urticaria seen in children, and this girl has clear symptoms
of a viral illness. Amoxicillin is a less likely cause of this girl’s urticaria because urticaria caused
by amoxicillin usually does not have a delayed onset. Urticaria from food allergies usually
presents within 2 hours of exposure. Hypothyroidism, specifically autoimmune hypothyroidism,
may predispose some children to urticaria, but in these cases, the urticaria is usually chronic,
lasting more than 6 weeks.

Urticaria can be defined as acute and chronic depending on the duration of symptoms. Causes of
acute urticaria and chronic urticaria are listed in Item C127A. The cause of chronic urticaria is
unknown in most cases. The first-line treatment is the same for both acute and chronic
urticaria—a nonsedating antihistamine such as cetirizine or loratadine. Sedating antihistamines
are second-line treatments. Children with chronic urticaria refractory to this management should
be referred to an allergist.

Chronic urticaria that occurs in response to cold temperature is unique in that it may lead to
anaphylaxis. Affected children should practice avoidance and be prescribed an epinephrine auto-
injector.

Sometimes, urticaria is accompanied by angioedema; about 40% of children with angioedema


also have urticaria. These 2 symptoms, combined with flushing and pruritus, are experienced by
about 90% of children with anaphylaxis. Causes of angioedema and anaphylaxis are listed in
Item C127B and Item C127C, respectively.

Causes of angioedema
• Animal dander (scales of shed skin)
• Exposure to water, sunlight, cold or heat
• Foods (such as berries, shellfish, fish, nuts, eggs, and milk)
American Academy of Pediatrics 431
PREP® Self-Assessment PREPSA 2023

• Insect bites
• Medicines (drug allergy) such as antibiotics (penicillin and sulfa drugs), nonsteroidal anti-
inflammatory drugs (NSAIDs), and blood pressure medicines (ACE inhibitors)
• Pollen
Autoimmune diseases such as lupus

PREP Pearls
• The most common cause of acute urticaria in children is a viral infection.
• The first-line treatment for both acute and chronic urticaria is a nonsedating antihistamine
such as cetirizine or loratadine.
• Children with cold-induced urticaria should be prescribed an epinephrine auto-injector
because of the risk of anaphylaxis.

ABP Content Specifications(s)


• Identify the etiologic agents that commonly cause urticaria, angioedema, and/or
anaphylaxis
• Recognize the signs and symptoms of chronic urticaria, and manage appropriately

Suggested Readings
• Pier J, Bingemann TA. Urticaria, angioedema, and anaphylaxis. Pediatr Rev.
2020;41(6):283-292. doi:10.1542/pir.2019-0056
• Secord E, Simon MR. Anaphylaxis. In: McInerny TK, Adam HM, Campbell DE, DeWitt
TG, Foy JM, Kamat DM, eds. American Academy of Pediatrics Textbook of Pediatric
Care. American Academy of Pediatrics; 2021:chap 350. Accessed February 8, 2022.
Pediatric Care Online.

American Academy of Pediatrics 432


PREP® Self-Assessment PREPSA 2023

Question 128
A 14-year-old adolescent boy is brought to the emergency department by ambulance after his
parents found him unresponsive in his bedroom. Upon arrival at the home, the emergency
medical services team witnessed a 5-minute generalized tonic clonic seizure for which the
paramedic administered diazepam 2 mg intravenously. The boy has a history of depression and
attention-deficit/hyperactivity disorder. His current medications include melatonin, amitriptyline,
and dexmethylphenidate.

In the emergency department, the boy has a temperature of 38.3°C, heart rate of 188 beats/min,
respiratory rate of 10 breaths/min, blood pressure of 72/45 mm Hg, and oxygen saturation of
90% in room air. He appears sleepy, without apparent seizure activity. His physical examination
findings are significant for dilated pupils; flushed, dry skin; and dry mucous membranes. He has
shallow breathing and decreased respiratory effort. The remainder of his physical examination
findings are unremarkable.

The boy is intubated, and his airway is secured. Results of a complete blood count and
comprehensive metabolic panel are normal. Shortly after intubation he is noted to have rhythm
changes on the cardiac monitor; 12-lead electrocardiography is obtained and shown (Item
Q128).

Item Q128: Electrocardiogram for the adolescent in the vignette.


Reprinted with permission from Barry D. (9) Pediatrics. 2018;142 (1_MeetingAbstract): 19

Of the following, the BEST next step in this boy’s management is to administer intravenous
A. flumazenil
B. magnesium sulfate
C. physostigmine
D. sodium bicarbonate

American Academy of Pediatrics 433


PREP® Self-Assessment PREPSA 2023

Correct Answer: D
The adolescent in the vignette is demonstrating classic anticholinergic findings (seizure;
respiratory depression; dilated pupils; dry, flushed skin; tachycardia; and arrhythmia) caused by
an overdose of amitriptyline, a tricyclic antidepressant (TCA). The treatment for TCA-induced,
wide complex ventricular tachycardia and resulting hypotension is administration of intravenous
sodium bicarbonate, with the goal of raising the serum pH to approximately 7.5.
Flumazenil, the antidote for a benzodiazepine overdose, is contraindicated in individuals with
TCA overdose as it may induce seizures. Flumazenil should also not be administered to
individuals with an unknown ingestion, as it may precipitate seizures in those who are
benzodiazepine dependent.

Physostigmine is the antidote for anticholinergic toxicity. However, in individuals with TCA
overdose, administration of physostigmine has been associated with cardiac arrest and is
therefore contraindicated.

Intravenous magnesium sulfate is the treatment for torsades de pointes. Although TCA overdoses
may cause a wide variety of complex cardiac arrhythmias, torsades de pointes is a very rare
finding; therefore, magnesium sulfate is not the best next management step for the boy in the
vignette.

Tricyclic antidepressants are used in children and adolescents to treat depression and as migraine
prophylaxis. Tricyclic antidepressant poisoning can be life-threatening; therefore, it is important
to identify the signs and symptoms of an overdose and begin treatment immediately. Signs and
symptoms of tricyclic antidepressant overdose include:
• Dry mouth
• Blurred vision
• Dilated pupils
• Respiratory depression
• Hyperthermia
• Flushing
• Decreased bowel sounds
• Neurologic
o Drowsiness
o Confusion
o Agitation
o Seizure
o Coma
• Cardiovascular
o Tachycardia
o Hypotension
o Arrhythmias (eg, sinus tachycardia, wide-complex tachycardia, atrioventricular
block, bradycardia, ventricular tachycardia, ventricular fibrillation)

American Academy of Pediatrics 434


PREP® Self-Assessment PREPSA 2023

Tricyclic antidepressant overdose blocks the fast sodium channels in the myocardium and
antagonizes central and peripheral muscarinic acetylcholine receptors, H1-histamine receptors,
central nervous system GABA 1 receptors, and peripheral α1-adrenergic receptors. Blockage of
myocardial sodium channels impairs depolarization and causes prolongation of the QT and QRS
intervals. QRS prolongation of >100 milliseconds increases the risk for seizures; prolongation
>160 milliseconds increases the risk for complex arrhythmias. Death is usually a result of
ventricular arrhythmias, which may be refractory to defibrillation.

The focus of treatment for TCA overdose is stabilization of the cardiopulmonary system, which
includes airway protection and fluid resuscitation. Seizures should be treated initially with a
benzodiazepine. An electrocardiogram should be obtained immediately when there is suspected
or confirmed ingestion of a TCA. Administration of sodium bicarbonate should be initiated in
any child with TCA ingestion who is hemodynamically unstable, acidotic, hypotensive,
exhibiting seizure activity, or has a QRS prolongation of more than 100 milliseconds. It is
administered intravenously as a rapid IV bolus of 1 to 2 mEq/kg followed by a continuous
infusion. The goal of sodium bicarbonate therapy is to achieve and maintain a serum pH of 7.5
and narrow the QRS complex.

PREP Pearls
• An electrocardiogram should be obtained immediately in a child with suspected or
confirmed ingestion of a tricyclic antidepressant.
• Sodium bicarbonate is the mainstay of treatment in children with tricyclic antidepressant
toxicity and should be initiated in any child who is hemodynamically unstable, acidotic,
hypotensive, or exhibiting seizure activity or a QRS interval prolongation of more than
100 milliseconds.
• The goal of sodium bicarbonate therapy for tricyclic antidepressant–induced QRS
interval prolongation is to narrow the QRS complex and achieve and maintain a serum
pH of 7.5.

ABP Content Specifications(s)


• Recognize the signs and symptoms of tricyclic antidepressant toxicity, and provide
appropriate initial management
• Recognize the signs and symptoms of ingestion of an anticholinergic drug, and manage
appropriately

Suggested Readings
• Fine JS, Auerbach MA, Ching KU, Fullerton KT, Weinberg ER. Poisoning. In: McInerny
TK, Adam HM, Campbell DE, DeWitt TG, Foy JM, Kamat DM, eds. American
Academy of Pediatrics Textbook of Pediatric Care. American Academy of
Pediatrics;2021:chap 369. Accessed September 1, 2022. Pediatric Care Online.
• Toce MS, Burns MM. The poisoned pediatric patient. Pediatr Rev. 2017;38(5):207-220.
doi:10.1542/pir.2016-0130.

American Academy of Pediatrics 435


PREP® Self-Assessment PREPSA 2023

Question 129
A 3-year-old girl in the pediatric intensive care unit has acute respiratory failure from respiratory
syncytial virus bronchiolitis. She is in marked respiratory distress, with a respiratory rate of 60
breaths/min and oxygen saturation of 73% in room air. She is somnolent and lethargic.
Auscultation of the lungs demonstrates diffuse, coarse rales, wheezes, and rhonchi. The decision
is made to perform rapid-sequence endotracheal intubation, after which her oxygen saturation is
98% on an FiO2 of 45%. Approximately 20 minutes after initiation of mechanical ventilation,
the girl has an acute oxygen desaturation to 75% which does not improve when 100% FiO2 is
provided by the ventilator.

Visual inspection of her chest reveals equal but restricted chest rise, and auscultation reveals
diminished breath sounds in all lung fields.

Of the following, the BEST next step in this girl’s management is


A. chest radiography
B. manual ventilation with a resuscitation bag
C. needle thoracostomy of the chest
D. suctioning of the endotracheal tube

American Academy of Pediatrics 436


PREP® Self-Assessment PREPSA 2023

Correct Answer: B
The girl in the vignette has acute respiratory failure requiring intubation and mechanical
ventilation. Given her ongoing hypoxia despite the provision of 100% FiO2 by the ventilator,
equal but diminished breath sounds, and equal chest rise, the best next step in management is to
provide manual ventilation with a resuscitation bag via the endotracheal tube to both assess and
manage the cause of ventilation failure.

Children with acute lung disease may progress to respiratory failure requiring intubation and
mechanical ventilation due to: (1) failure of oxygenation and/or (2) failure of ventilation. Failure
of oxygenation or ventilation can occur as a result of new-onset parenchymal lung disease,
cardiac dysfunction, neurologic abnormalities, or multi-organ system failure. Children may also
require intubation for airway protection, particularly when an acute underlying neurologic
impairment renders them unable to protect their airway from aspiration of oral secretions (eg,
acute head trauma or toxic ingestion).

Endotracheal intubation should only be performed by those skilled in the procedure. Although
rapid sequence endotracheal intubation can mitigate some of the complications, even in the most
skilled hands, there are risks and complications that require vigilance. After successful placement
of an endotracheal tube, complications can arise, as in the case of the girl in the vignette.
A common mnemonic used to evaluate and manage a successfully intubated patient who
subsequently develops hypoxemia is DOPE, which stands for:
• Displacement/Dislodgement
• Obstruction
• Pneumothorax
• Equipment failure

Each of these issues can be life-threatening if not addressed, but the first step in evaluating and
managing a patient who is acutely hypoxic on the ventilator is to provide manual ventilation with
a resuscitation bag.

Displacement/dislodgement can occur after successful placement of an endotracheal tube in the


trachea. The tube may migrate into one of the mainstem airways, or may become dislodged out
of the trachea into the supraglottic space. Displacement can occur with patient movement or
during the securement or re-securing of an endotracheal tube. Continuous end-tidal capnography
can yield clues to the location of an endotracheal tube and can help verify that dislodgement has
not occurred.

The endotracheal tube can become obstructed with inspissated secretions that obstruct airflow
down the tube. High airway resistance may be felt by the physician during manual ventilation
with a flow-inflating resuscitation device. Suctioning the endotracheal tube can relieve the
obstruction and improve airflow; this approach is often needed immediately after intubation in
patients with copious pulmonary secretions.

American Academy of Pediatrics 437


PREP® Self-Assessment PREPSA 2023

Pneumothorax occurs when air leaks from ruptured alveoli into the pleural space leading to
accumulation of air and compression or collapse of the affected lung. When the trapped air
cannot escape, pressure and compression can lead to decreased venous return and cardiovascular
collapse (eg, tension pneumothorax). In such cases, needle decompression is required to relieve
the tension and a thoracostomy tube is often required thereafter.

Equipment failure can occur when oxygen supply to the ventilator becomes disrupted, or when
the ventilator itself malfunctions in some way such that the patient is not adequately supported.
Although rare, equipment failure can be a cause of acute hypoxia after successful intubation.
Although chest radiography could be helpful in diagnosing the cause of acute hypoxia, the time
required would delay necessary and immediate intervention. Pneumothoraces tend to be
unilateral with findings of absent or diminished breath sounds and decreased chest rise on the
affected side only. The child in the vignette had bilateral findings, thus, needle decompression is
not the preferred response. Finally, suctioning of the endotracheal tube can alleviate mucus or
secretion-related endotracheal tube obstruction. However, given the ongoing significant oxygen
desaturations, manual ventilation with a resuscitation bag is indicated before suctioning to
attempt to restore normal oxygenation quickly.

PREP Pearls
• When acute hypoxia occurs in mechanically ventilated children, manual ventilation with
a resuscitation bag should be performed with 100% FiO2 until the underlying cause of
hypoxia can be addressed and corrected.
• Endotracheal intubation may be required for children with respiratory failure, impending
respiratory arrest, or those with neurologic impairment or severely altered mental status
where airway protective reflexes cannot be maintained.
• A common mnemonic used to evaluate and manage a successfully intubated patient who
subsequently develops hypoxemia is DOPE: Displacement/Dislodgement of the
endotracheal tube, Obstruction of the endotracheal tube, Pneumothorax, and Equipment
failure.

ABP Content Specifications(s)


• Understand the potential complications associated with endotracheal intubation

Suggested Readings
• Easley RB, Segeleon JE, Haun SE, Tobias JD. Prospective study of airway management
of children requiring endotracheal intubation before admission to a pediatric intensive
care unit. Crit Care Med. 2000;28(6):2058-2063. doi:10.1097/00003246-200006000-
00065.
• Hatch LD, Grubb PH, Lea AS, et al. Endotracheal intubation in neonates: a prospective
study of adverse safety events in 162 infants. J Pediatr. 2016;168:62-66.e6.
doi:10.1016/j.jpeds.2015.09.077.
• Page NE, Giehl M, Luke S. Intubation complications in the critically ill child. AACN
Clin Issues. 1998;9(1):25-35.

American Academy of Pediatrics 438


PREP® Self-Assessment PREPSA 2023

Question 130
A 10-year-old girl is seen for a health supervision visit. The girl recently completed a formal
psychoeducational evaluation through her school because of long-standing learning difficulties
and was diagnosed with dyslexia.

The girl starts to cry. She admits that she is worried about no longer being in the same class as
her friends and that the other students will “know I’m different because I have to go to the other
classroom.” The school has a family meeting planned to discuss how special education services
will be implemented. The girl’s mother received special education services for dyslexia when she
was in school, which prevented her from attending her favorite class, which was art. She began
investigating private school options for her daughter because she is worried that she will not
have any input into determining how educational services will be provided.

Of the following, the BEST next step in counseling this girl’s mother is to
A. encourage her to continue her search for a private school so that she can be involved in
decision-making
B. have the mother request that special education instruction take place during the girl’s
physical education class time
C. recommend that the girl be in a regular education setting with in-class supports and small
group instruction
D. recommend that the girl be in a self-contained classroom for her entire school day

American Academy of Pediatrics 439


PREP® Self-Assessment PREPSA 2023

Correct Answers: C
In 1975 the Individuals with Disabilities Education Act (IDEA) was passed through Congress,
which mandates that a free and appropriate public education be provided and that supports be
implemented in the least restrictive environment allowing children with disabilities to be
educated with children without disabilities as much as possible. For the girl in the vignette, in-
class supports along with small group instruction would be considered the least restrictive
educational environment. Pull-out instruction (having the girl leave her classroom periodically)
may be considered if the girl does not meet the goals of her individualized education plan within
the least restrictive setting.

It is important for parents and caregivers to be involved in the educational planning process
through shared decision-making. Parents should be encouraged to provide input into the
implementation of special education services. Having the girl attend a private school does not
guarantee that her mother would have more involvement in decision-making. Additionally,
children placed in a private school by their parents do not have the same entitlement to special
education services that they would receive by attending public school. A local or state agency
may place a child in private school who was previously attending public school if the public
school is unable to provide a free and appropriate education.

Placement of the girl in a self-contained classroom would be the most restrictive environment
and would not be an appropriate initial educational setting. For a child with more severe and
global disabilities, a self-contained classroom may be an appropriate setting. However,
opportunities for “main-streaming” during electives such as music, physical education, and
recess should be encouraged to provide exposure to typical peers. The girl should not be
removed from physical education as physical activity may help support self-esteem, improve
mood, and provide additional opportunity for peer interaction.

Pediatricians play an important role in not only identifying children in need of special education
services, but in the implementation and longitudinal monitoring of educational interventions.
Pediatricians should encourage parents and caregivers to take an active role in their children’s
education plans and provide education on their right to a free and appropriate public education in
the least restrictive environment.

PREP Pearls
• Children with disabilities must be educated in the least restrictive environment and, as
much as possible, with children without disabilities.
• All children are entitled to a free and appropriate public education with the school
providing enough support for the child to progress.
• Parents are key stakeholders in the design and implementation of special education
services.

ABP Content Specifications(s)


• Recognize appropriate educational settings for patients with learning disabilities, and the
various strategies utilized in those settings to circumvent weaknesses
American Academy of Pediatrics 440
PREP® Self-Assessment PREPSA 2023

Suggested Readings
• Council on Early Childhood; Council on School Health. The pediatrician’s role in
optimizing school readiness. Pediatrics. 2016;138(3):e20162293. doi:10.1542/peds.2016-
2293.
• Frankowski BL. Learning difficulty. In: McInerny TK, Adam HM, Campbell DE, DeWitt
TG, Foy JM, Kamat DM, eds. American Academy of Pediatrics Textbook of Pediatric
Care. American Academy of Pediatrics; 2021:chap 172. Accessed September 1, 2022.
Pediatric Care Online.
• Lipkin PH, Okamoto J; Council on Children with Disabilities; Council on School Health.
The Individuals with Disabilities Education Act (IDEA) for children with special
educational needs. Pediatrics. 2015;136(6):e1650-e1662. doi:10.1542/peds.2015-3409.
• Rey-Casserly C, McGuinn L, Lavin A; Committee on Psychosocial Aspects of Child and
Family Health,section on
• Developmental and Behavioral Pediatrics. School-aged children who are not progressing
academically: considerations for pediatricians. Pediatrics. 2019;144(4):e20192520.
doi:10.1542/peds.2019-2520.

American Academy of Pediatrics 441


PREP® Self-Assessment PREPSA 2023

Question 131
A 16-year-old adolescent boy is evaluated for left-sided neck swelling and pain of 5 days’
duration, headache, abdominal pain, and fever. No one else is sick at home and there has been no
recent travel. The family has 1 chihuahua and 3 kittens. On physical examination, the boy is well
appearing, with a temperature of 38.3°C. He has a tender neck mass on the left with overlying
skin redness (Item Q131A). Neck ultrasonography shows a 3.3 × 2.1–cm suppurative lymph
node in the neck left midjugular region (Item Q131B). Abdominal ultrasonography did not show
any liver or spleen involvement.

Item Q131A: Left neck mass described for the boy in the vignette. Courtesy of A. Noor

Item Q131B: Ultrasonography findings for the boy in the vignette. Courtesy of A. Noor
Laboratory testing at diagnosis reveals the following:
Laboratory Test Result
White blood cell count 15,200/µL (15.2 × 109/L)
Neutrophils 93%
Lymphocytes 5%
Monocytes 2%
Hemoglobin 13.3 g/dL (133 g/L)
Platelet count 269 × 103/µL (269 × 109/L)
C-reactive protein 1.2 mg/dL (12 mg/L)
Bartonella henselae IgG 1:64
Bartonella henselae IgM <1:16

American Academy of Pediatrics 442


PREP® Self-Assessment PREPSA 2023

A plan is made to treat the adolescent with antibiotics and draw repeat titers for Bartonella
henselae in 4 weeks.

Of the following, the BEST next step in management of this adolescent is to administer
A. amoxicillin
B. azithromycin
C. cephalexin
D. penicillin

American Academy of Pediatrics 443


PREP® Self-Assessment PREPSA 2023

Correct Answer: B
The adolescent in the vignette has unilateral cervical lymphadenopathy associated with fever.
This clinical scenario, including exposure to kittens, is suggestive of Bartonella henselae
infection or “cat-scratch disease.” The adolescent in the vignette has lymphadenopathy, fever,
headache, and abdominal pain as well as exposure to kittens, therefore, empiric treatment with
azithromycin would be appropriate. Empiric treatment with azithromycin can be considered in
situations compatible with this clinical syndrome (eg, acute lymphadenopathy and exposure to
kittens). Penicillin, amoxicillin, and first-generation cephalosporins are not recommended for
treatment of B henselae.

Bartonella henselae is a fastidious gram-negative bacillus for which cats are the natural reservoir.
Most children presenting with cat-scratch disease have had close contact with healthy cats,
particularly kittens. Most cases occur in the fall and winter.

Bartonella henselae infection has a spectrum of clinical presentations. The severity of disease is
primarily determined by the age and immune status of the child. Immunocompetent children are
generally asymptomatic or have a mild, self-limiting illness, whereas immunocompromised
children may develop disseminated infection. The most common presentation is regional
lymphadenopathy. The incubation period is 5 to 50 days from cat scratch to the appearance of
lymphadenopathy. One or more erythematous papules may appear at the inoculation site and
typically persist for 1 week and then disappear. Regional lymphadenopathy appears between 2
and 4 weeks from exposure. The lymph nodes most commonly involved are axillary, followed by
cervical, mandibular, and inguinal. In children younger than 8 years, the differential diagnosis
includes acute bacterial lymphadenopathy from Staphylococcus aureus or group A Streptococcus
species. In older children and adolescents with unilateral cervical lymphadenopathy, other causes
should be considered based on immune status and exposure.

Parinaud oculoglandular syndrome is a presentation of cat-scratch disease in which the site of


inoculation is either the eyelid or conjunctiva. It manifests as a papule and/or conjunctivitis
coupled with preauricular lymphadenitis.

Systemic B henselae is uncommon. It should be considered in children with fever of unknown


origin. Abdominal ultrasonography or computed tomography reveals multiple microabscesses or
granulomas of liver and spleen. Other rare presentations of B henselae include culture-negative
endocarditis, aseptic meningitis, neuroretinitis, osteomyelitis, pneumonia, thrombocytopenia
purpura, and erythema nodosum.

The diagnosis of cat-scratch disease is confirmed with acute and convalescent B henselae
antibody titers. During acute infection, B henselae antibody titers can be falsely negative.
Antibody levels can be measured with either a commercially available enzyme immunoassay or
indirect immunofluorescence antibody. IgG titers between 1:64 and 1:256 may represent past or
current infection and follow-up convalescent titers are recommended in 2 to 4 weeks. A rise in
antibody titers is the best evidence of infection. A single IgG titer of 1:256 is strongly suggestive
of acute catscratch disease (the boy in the vignette has a 4-week IgG titer of 1:256). An elevated
American Academy of Pediatrics 444
PREP® Self-Assessment PREPSA 2023

IgM titer might represent acute infection; however, false positives (cross reactivity with other
antigens) and false negatives (early test) are common.

Although the value of antibiotic treatment for cat-scratch disease is uncertain, a modest
improvement in lymph node size may be seen within 5 days of starting azithromycin. Painful
lymphadenopathy can be treated with needle aspiration. Disseminated disease with hepatic
involvement is treated with a combination of azithromycin plus rifampin in children younger
than 8 years.

PREP Pearls
• Cat-scratch disease is a common cause of lymphadenopathy in children, caused by the
gram-negative bacillus Bartonella henselae.
• The majority of Bartonella henselae infections are asymptomatic. The severity of disease
varies based on the age and immune status of the host, ranging from regional
lymphadenopathy to systemic disseminated infection.
• Uncomplicated cases of lymphadenopathy due to Bartonella henselae do not require
treatment. However, a 5-day course of azithromycin results in reduction of lymph node
size and hastens resolution of symptoms.

ABP Content Specifications(s)


• Recognize the clinical features associated with cat-scratch disease
• Understand the epidemiology of cat-scratch disease
• Plan appropriate management for a patient with cat-scratch disease

Suggested Readings
• American Academy of Pediatrics. Bartonella henselae (cat-scratch disease). In:
Kimberlin DW, Barnett ED, Lynfield R, Sawyer MH, eds. Red Book: 2021–2024 Report
of the Committee on Infectious Diseases. 32nd ed. American Academy of Pediatrics;
2021. Accessed September 1, 2022. Red Book Online.
• English R. Cat-scratch disease. Pediatr Rev. 2006;27(4):123-128. doi:10.1542/pir.27-4-
123.
• Han JY, Jacobs RF. Bartonella species (cat-scratch disease). In: Long SS, Prober CG,
Fischer M, eds. Principles and Practice of Pediatric Infectious Diseases. 5th ed. Elsevier;
2018:881-886.e1.
• Sandoval AC, Reyes FT, Prado MA, Peña AL, Viviani TN. Cat-scratch disease in the
pediatric population. Pediatr Infect Dis J. 2020;39(10):889-893.
doi:10.1097/INF.0000000000002708.
• Spicer KB. Cat-scratch disease. Feld LG, Mahan JD, Jackson MA, eds. Succinct
Pediatrics: Evaluation and Management of Infectious Diseases and Dermatology.
American Academy of Pediatrics; 2016:123-128.

American Academy of Pediatrics 445


PREP® Self-Assessment PREPSA 2023

Question 132
A 14-year-old adolescent girl is seen for evaluation of a 9-kg weight gain over the past year. Her
mother is concerned about a possible hormonal etiology. The adolescent reports fatigue and
depression. She often stays up late working on her computer and will snack during this time. She
does not sleep well and snores loudly. She reports occasional headaches, but no vision changes,
nausea, or vomiting. She has no muscle weakness. Menarche occurred at age 11 years, and her
menses are regular. Her mother recently underwent bariatric surgery.

On physical examination, the adolescent’s blood pressure is 132/86 mm Hg and heart rate is 86
beats/min. Her weight is 85 kg (98th percentile), height is 163 cm (65th percentile) and body
mass index is 32 kg/m2 (98th percentile). She has hyperpigmentation around her neck and in her
skin creases, and thin striae on her abdomen. The remainder of her physical examination findings
are normal.

Of the following, the BEST next step in this adolescent’s evaluation is to


A. gather a detailed dietary history
B. measure a serum morning cortisol level
C. obtain magnetic resonance imaging of the abdomen
D. obtain thyroid function tests

American Academy of Pediatrics 446


PREP® Self-Assessment PREPSA 2023

Correct Answer: A
The adolescent in the vignette has significant weight gain and obesity (body mass index greater
than the 95th percentile for age and sex). Obesity usually results from an interplay of genetic and
environmental factors. Single gene disorders and endocrine etiologies are rare. This adolescent
has multiple risk factors for developing obesity including family history (her mother underwent
bariatric surgery), media time, and sleep deprivation (staying up late, snoring suggestive of sleep
apnea). Of the response choices, the best next step in her evaluation is to gather a detailed dietary
history.

Dietary choices and portions are among the most important factors in the development of
pediatric obesity. Children and adolescents who consume high-calorie, high-fat, high-
carbohydrate, and low-fiber foods are at high risk of developing obesity. The primary
intervention for children and adolescents with obesity is assistance with healthy lifestyle changes
that result in improved diet and increased physical activity (at least 30 minutes per day 5 days
per week).

Media time has been linked to childhood obesity; children engaging in more than 4 hours of
media per day have the highest prevalence of obesity. Sleep deprivation has also been linked to
obesity, and sleep apnea exacerbates sleep deprivation. Sleep apnea is diagnosed with
polysomnography, which should be considered for this adolescent.

Cushing syndrome and other endocrine causes of obesity are rare in pediatrics. In growing
children with Cushing syndrome, weight gain is accompanied by severe linear growth
impairment. Striae are a common finding in children with Cushing syndrome, as is depression,
but both are non-specific. Proximal muscle weakness is a more specific finding.
Hyperpigmentation may be present due to adrenocorticotropic hormone (ACTH) excess in
ACTH-dependent Cushing syndrome. The adolescent in the vignette has increased pigmentation
around her neck and in her skin creases (acanthosis nigricans), which is a sign of insulin
resistance.

Magnetic resonance imaging of the abdomen to visualize the adrenal glands is not indicated for
the adolescent in the vignette. When Cushing syndrome is suspected, the diagnosis should be
confirmed biochemically before imaging studies are undertaken. Measurement of a serum
cortisol level could be considered in the evaluation of Cushing syndrome. However, measuring a
cortisol level in the morning, when levels are at their highest, is not appropriate to evaluate for
cortisol excess. Screening for Cushing syndrome should be performed by measurement of a
nighttime salivary cortisol level, 24-hour urine collection for urinary free cortisol, or
dexamethasone suppression test.

Thyroid function tests are frequently measured as part of an evaluation for weight gain in
children and adolescents. However, hypothyroidism is not associated with significant weight
gain in these age groups. Hypothyroidism is more likely to present with poor linear growth in
children who are still growing. In adolescents, the classic symptoms of hypothyroidism (eg,
fatigue, cold intolerance, irregular menses in females) may be present.
American Academy of Pediatrics 447
PREP® Self-Assessment PREPSA 2023

PREP Pearls
• Obesity usually results from an interplay of genetic and environmental factors.
• Primary hormonal causes of obesity in children and adolescents are rare.
• Healthy lifestyle changes are the most important treatment for pediatric obesity.

ABP Content Specifications(s)


• Identify the genetic and environmental risk factors for obesity, including lifestyle choices

Suggested Readings
• Ebbeling CB, Feldman HA, Osganian SK, Chomitz VR, Ellenbogen SJ, Ludwig DS.
Effects of decreasing sugar-sweetened beverage consumption on body weight in
adolescents: a randomized, controlled pilot study. Pediatrics. 2006;117(3):673680.
doi:10.1542/peds.2005-0983.
• Golden NH, Schneider M, Wood C; Committee on Nutrition; Committee on
Adolescence; Section on Obesity. Preventing obesity and eating disorders in adolescents.
Pediatrics. 2016;138(3):e20161649. doi:10.1542/peds.2016-1649.
• Reinehr T, de Sousa G, Andler W. Hyperthyrotropinemia in obese children is reversible
after weight loss and is not related to lipids. J Clin Endocrinol Metab. 2006;91(8):3088-
3091. doi:10.1210/jc.2006-0095.
• Styne DM, Arslanian SA, Connor EL, et al. Pediatric obesity—assessment, treatment,
and prevention: an Endocrine Society Clinical Practice Guideline. J Clin Endocrinol
Metab 2017; 102(3):709-757. doi:10.1210/jc.2016-2573.
• Wang YC, Gahagan S. Obesity and metabolic syndrome. In: McInerny TK, Adam HM,
Campbell DE, DeWitt TG, Foy JM, Kamat DM, eds. American Academy of Pediatrics
Textbook of Pediatric Care. American Academy of Pediatrics; 2021:chap 299. Accessed
September 1, 2022. Pediatric Care Online.

American Academy of Pediatrics 448


PREP® Self-Assessment PREPSA 2023

Question 133
A 5-day-old male neonate born at 36 weeks’ gestation is brought to the emergency department
for evaluation of a 1day history of poor feeding, decreased activity, and episodes of excessive
crying. He is exclusively breastfed. The infant was discharged home 2 days after birth with a
bilirubin level of 8 mg/dL (136.83 µmol/L). The mother’s blood group is O positive; she does
not know the neonate’s blood type. On physical examination, the neonate has a highpitched cry,
marked jaundice, increased extensor tone in the upper and lower extremities, intermittent
arching, and an exaggerated Moro reflex. There is no clonus. Laboratory results include a total
bilirubin level of 32 mg/dL (547.33 µmol/L), a blood type of A positive, and a positive direct
antiglobulin (Coombs) test result. The neonate is admitted, and phototherapy is started.

Of the following, the MOST important next step in the management of this neonate is
A. continued phototherapy
B. double volume exchange transfusion
C. intravenous fluid therapy
D. partial volume exchange transfusion

American Academy of Pediatrics 449


PREP® Self-Assessment PREPSA 2023

Correct Answer: B
The neonate in the vignette is a late preterm infant with severe hyperbilirubinemia. His physical
examination findings are consistent with phases 1 to 2 of acute bilirubin neurotoxicity. To
prevent further neurologic deterioration and decrease the risk of the long-term
neurodevelopmental sequelae of kernicterus, a double volume exchange transfusion should be
performed as soon as possible. For this neonate, the total bilirubin level for which exchange
transfusion is indicated, is 22.2 mg/dL (379.7 µmol/L); this neonate’s level is well above that
(Item C133A).

Exchange transfusion thresholds by gestational age for infants with any recognized
hyperbilirubinemia neurotoxicity risk factors other than gestational age. These thresholds are
based on expert opinion rather than strong evidence on when the potential benefits of escalation
of care exceed its potential harm. The stippled lines for the first 24 hours indicate uncertainty
because of the wide range of clinical circumstances and responses to intensive phototherapy.
Use total serum bilirubin concentrations; do not subtract direct bilirubin from the total serum
bilirubin. In rare cases of severe hyperbilirubinemia in which the direct-reacting or conjugated
bilirubin exceeds 50% of the total serum bilirubin, consult an expert. Hyperbilirubinemia
neurotoxicity risk factors include albumin <3.0 g/dL; isoimmue hemolytic disease, glucose-6-
phosphate dehydrogenase (G6PD) deficiency, or other hemolytic conditions: sepsis; or any
significant clinical instability in the previous 24 hours
Reprinted with permission from Kemper AR, Newman TB, Slaughter JL, et al. Clinical
practice guideline revision: management of hyperbilirubinemia in the newborn infant 35 or
more weeks of gestation. Pediatrics. 2022;e2022058859.

Almost all preterm neonates and approximately 60% of term neonates will develop physiologic
jaundice. This jaundice occurs due to catabolism of red blood cells (RBCs) (the half-life of RBCs
in neonates is 90 days compared with 120 days in adults), increased RBC volume, immature
hepatic conjugation, and delayed establishment of feedings, leading to reduced excretion of
bilirubin and increased enterohepatic circulation of bilirubin. Item C133B lists risk factors for

American Academy of Pediatrics 450


PREP® Self-Assessment PREPSA 2023

severe hyperbilirubinemia in neonates; the neonate in the vignette’s risk factors include
gestational age (late preterm), exclusive breastfeeding, and isoimmune hemolytic disease.

Item C133B: Risk factors for developing severe Hyperbilirubinemia.


• Lower gestational age (ie, risk increases with each additional week less than 40 wk)
• Jaundice in the first 24 h after birth
• Predischarge transcutaneous bilirubin (TcB) or total serum bilirubin (TSB) concentration
close to the phototherapy threshold
• Hemolysis from any cause, if known or suspected based on a rapid rate of increase in
TSB or TcB of > 0.3 mg/dl per hour in the first 24 h or >0.2 mg/dl per hour thereafter.
• Phototherapy before discharge
• Parent or sibling requiring phototherapy or exchange transfusion
• Family history or genetic ancestry suggestive of inherited red blood cell disorders,
including glucose-6-phosphate dehydrogenase (G6PD) deficiency
• Exclusive breastfeeding with suboptimal intake
• Down syndrome
• Macrosomic infant or a diabetic mother
Reprinted with permission from Kemper AR, Newman TB, Slaughter JL, et al. Clinical
practice guideline revision: management of hyperbilirubinemia in the newborn infant 35 or
more weeks of gestation. Pediatrics 2022;e2022058859.

Early detection and treatment of hyperbilirubinemia are important to prevent bilirubin


neurotoxicity, specifically to prevent the chronic neurologic sequelae known as bilirubin-induced
neurologic dysfunction (BIND). A high level of bilirubin or an insufficient amount of serum
protein to bind the bilirubin leads to circulating free bilirubin, which can easily penetrate the
blood–brain barrier (BBB), especially when the BBB is disturbed because of the risk factors
noted in Item C133C. This condition leads to hyperbilirubinemia neurotoxicity.

Hyperbilirubinemia Neurotoxicity Risk Factors


• Gestational age <38 wk and this risk increases with the degree of prematuritya
• Albumin <3.0 g/dL
• Isoimmune hemolytic disease (ie, positive direct antiglobulin test), G6PD deficiency, or other
hemolytic conditions
• Sepsis
• Significant clinical instability in the previous 24 h
a
Gestational age is required to identify the phototherapy thresholds and the exchange transfusion
thresholds.

Acute bilirubin encephalopathy describes the neurologic changes that occur in the first postnatal
weeks as a result of bilirubin toxicity. The clinical findings of acute bilirubin toxicity progress
through 3 phases during the first few weeks after birth (Item C133D). If the bilirubin
concentration is not decreased quickly after initial presentation, long-term

American Academy of Pediatrics 451


PREP® Self-Assessment PREPSA 2023

morbidity can result in BIND or kernicterus. The term kernicterus was used originally to
describe the staining of the brainstem nuclei and cerebellum associated with the chronic or
permanent spastic diplegia resulting from bilirubin toxicity. The neuronal injury involves the
basal ganglia, brainstem nuclei, hippocampus, and cerebellum. Bilirubininduced neurologic
dysfunction occurs in 2 phases. The first phase, seen in the first postnatal year, is characterized
by hypotonia, hyperactive deep-tendon reflexes, obligatory tonic neck reflexes, and delayed
motor skills. The second phase, seen after the first postnatal year, includes choreoathetoid
cerebral palsy, dental dysplasia, sensorineural hearing loss, and cognitive impairment.

Three factors important in classifying children and adults with kernicterus. Note that the heights of the
bars do not represent a relationship between location and time and the severity of the response, but
merely represent illustrated that three factors, severity, location and time, can be used to catagorize
clinical kernicterus.

Phototherapy should continue before and after exchange transfusion but will not rapidly decrease
the bilirubin level, which is critical to minimizing the neurotoxic effects of severe
hyperbilirubinemia. Intravenous fluid is helpful to address dehydration, if present, but it is not a
treatment for hyperbilirubinemia. Partial volume exchange transfusion is performed in neonates
with chronic anemia for whom heart failure is a concern and in neonates with symptomatic
polycythemia with a central hematocrit greater than 65% to 70% (0.65-0.70); it is not an
appropriate treatment for neonatal hyperbilirubinemia.

PREP Pearls
• A high level of bilirubin or an insufficient amount of serum protein to bind the bilirubin
leads to circulating free bilirubin, which can easily penetrate the blood–brain barrier.
• To prevent further neurologic deterioration and decrease the risk of the long-term
neurodevelopmental sequelae of kernicterus, a double volume exchange transfusion
should be performed as soon as possible when exchange transfusion is indicated.
• Some risk factors for significant neonatal hyperbilirubinemia include predischarge total
serum or transcutaneous bilirubin level close to phototherapy threshold, lower gestational
age, exclusive breastfeeding, jaundice observed in the first 24 hours, inherited red blood
cell disorders, phototherapy before discharge, hemolysis from any cause, macrosomic
infant, and Down syndrome.
American Academy of Pediatrics 452
PREP® Self-Assessment PREPSA 2023

ABP Content Specifications(s)


• Understand strategies to prevent the development of severe hyperbilirubinemia in
newborn infants
• Recognize the clinical features and sequelae of acute bilirubin encephalopathy in
newborn infants, and manage appropriately

Suggested Readings
• American Academy of Pediatrics. Hyperbilirubinemia. In: PCEP Book 3: Neonatal Care.
4th ed. American Academy of Pediatrics; 2021:253-282.
• Bahr TM, Henry E, Christensen RD, Minton SD, Bhutani VK. A new hour-specific
serum bilirubin nomogram for neonates =35 weeks of gestation. J Pediatr. 2021;236:28-
33.e1. doi:10.1016/j.jpeds.2021.05.039 .
• Kemper AR, Newman TB, Slaughter JL, et al. Clinical practice guideline revision:
management of hyperbilirubinemia in the newborn infant 35 or more weeks of gestation.
Pediatrics. 2022;e2022058859. doi:10.1542/peds.2022-058859.
• Lauer BJ, Spector ND. Hyperbilirubinemia in the newborn. Pediatr Rev. 2011;32(8):341-
349. doi:10.1542/pir.32-8-341.
• Shapiro SM. Definition of the clinical spectrum of kernicterus and bilirubin-induced
neurologic dysfunction (BIND). J Perinatol. 2005;25(1):54-9. doi:10.1038/sj.jp.7211157.

American Academy of Pediatrics 453


PREP® Self-Assessment PREPSA 2023

Question 134
A new patient is seen for a health supervision visit. The child is healthy with a normal birth
history. He is crawling and pulling himself up to stand using the couch. Occasionally, he will
stand up unassisted in the middle of the floor. He uses his thumb and index finger to pick up a
piece of breakfast cereal. He is able to find his pacifier when his mother hides it under a blanket.
He says ”mama” and “uh-oh” appropriately. He will stop and look at his parents when they say
“no.”

Of the following, this child’s development is MOST consistent with that of a


A. 9-month-old
B. 12-month-old
C. 15-month-old
D. 18-month-old

American Academy of Pediatrics 454


PREP® Self-Assessment PREPSA 2023

Correct Answer: B
The child in the vignette is at a developmental level that is most consistent with that of a 12-
month-old.

The fine motor development of a 12-month-old child should include the ability to pick things up
between thumb and pointer finger, like small bits of food. The gross motor development of a 12-
month-old child should include cruising and taking first steps. Often a child this age will stand
independently, demonstrating age-appropriate tone and balance, but may not yet be willing to
attempt the actual motion of walking. At 15 to 18 months of age we would expect a child to be
walking well independently and beginning to run and climb.

Language development at 12 months of age should include a specific word for a parent, such as
“mama,” and an appropriate response to the word “no.” At age 9 months, we expect nonspecific
use of the words “mama” or “dada,” and receptive language skills should include recognizing
their own name. By age 18 months, children should try to say greater than or equal to 3 words
besides mama and dada. By 30 months, they should say approximately 50 single words.
The social and cognitive developmental skills of a 12-month-old would include the ability to
play games with the parent/caregiver like pat-a-cake and the ability to find an object they saw
their parent hide.

In February of 2022, the Centers for Disease Control and Prevention published an updated list of
developmental milestones for pediatricians and parents to use in the developmental surveillance
of children. This revision used an evidence-informed approach to the assignment of
developmental milestones expected for children at specific ages. Eleven criteria were used to
determine milestone lists. The milestones represent the age at which the majority (75%) of
children have attained the defined skill, as opposed to the average age at which children attain
the skill.

PREP Pearls
• Gross motor development of a 12-month-old child should include cruising and taking
first steps.
• Language development at age 12 months should include a specific word for a parent,
such as mama, and understand the word “no.”
• Cognitively, a 12-month-old should be able to find an object they witnessed being
hidden, and socially they should enjoy participating in a game such as “pat-a-cake.”

ABP Content Specifications(s)


• Evaluate the cognitive and behavioral developmental progress/status of an infant at 12
months of age
• Evaluate the motor developmental progress/status of an infant at 12 months of age

American Academy of Pediatrics 455


PREP® Self-Assessment PREPSA 2023

Suggested Readings
• Dosman CF, Andrews D, Goulden KJ. Evidence-based milestone ages as a framework
for developmental surveillance. Paediatr Child Health. 2012;17(10):561-568.
doi:10.1093/pch/17.10.561.
• Scharf RJ, Scharf GJ, Stroustrup A. Developmental milestones. Pediatr Rev.
2016;37(1):25-38. doi:10.1542/pir.2014-0103
• Zubler JM, Wiggins LD, Macias MM, et al. Evidence-informed milestones for
developmental surveillance tools. Pediatrics. 2022;149(3):e2021052138.
doi:10.1542/peds.2021-052138.

American Academy of Pediatrics 456


PREP® Self-Assessment PREPSA 2023

Question 135
A 7-month-old boy is admitted to the hospital for dehydration, weight loss, vomiting, and
diarrhea. The diarrhea began approximately 4 weeks ago. He has 8 to 10 nonbloody, watery
stools daily and frequent diaper rashes, which have been difficult to clear despite frequent
application of zinc oxide diaper cream. His diet consists of cow milk formula, pureed fruits and
vegetables, and oat and wheat infant cereal. He recently began to have intermittent, nonbilious,
nonbloody emesis and decreased oral intake (both solids and formula). On physical examination,
the infant is pale and fussy but consolable. His weight is at the third percentile, length is at the
50th percentile, and weight for length is fifth percentile for age. He has tachycardia, a capillary
refill time of 2 seconds, a soft and nondistended abdomen without hepatosplenomegaly, and
perianal erythema. The remainder of his physical examination findings are normal.
The infant is given intravenous fluids and made nil per os. Stool infectious studies, including
viral and bacterial testing, are negative. His diarrhea significantly improves after 24 hours of
bowel rest.

Of the following, the BEST next step in this infant’s management is


A. fiber supplementation
B. loperamide
C. serum vasoactive intestinal peptide concentration
D. upper endoscopy

American Academy of Pediatrics 457


PREP® Self-Assessment PREPSA 2023

Correct Answer: D
The infant in the vignette has chronic diarrhea (duration >14 days) associated with failure to
thrive and therefore requires further evaluation. For a child with this clinical presentation,
without evidence of secretory diarrhea, upper endoscopy is the best management option. With
further evaluation, he was diagnosed with allergic enteropathy.

Chronic diarrhea may occur because of the following mechanisms:


• Osmotic diarrhea: unabsorbed intestinal solutes drive luminal retention of water (via
osmotic gradient)
o Examples: lactose intolerance, exocrine pancreatic insufficiency
o Diarrhea improves when unabsorbed nutrients are removed from the diet (or in nil
per os [NPO] status)
• Secretory diarrhea: excessive intestinal electrolyte and/or fluid secretion
o Examples: neuroendocrine tumors (eg, VIPoma, neuroblastoma), congenital
chloride diarrhea
o Diarrhea persists despite NPO status
o Low stool osmolar gap: Stool Total Osmolarity – 2 × [Stool Na+ + Stool K+]
• Inflammatory diarrhea: may include both secretory and osmotic mechanisms because
intestinal fluid secretion may be excessive and the inflamed bowel may not completely
absorb intestinal solutes
o Examples: infectious diarrhea (excessive intestinal fluid secretion), inflammatory
bowel disease, celiac disease, and allergic enteropathy Diarrhea may improve
with NPO status
• Diarrhea caused by dysmotility: occurs because of decreased intestinal transit time
o Examples: chronic nonspecific diarrhea, irritable bowel syndrome

Evaluation of the infant with chronic diarrhea depends on the presence (or not) of failure to
thrive. Infants with frequent, loose stools and normal nutritional status may have chronic
nonspecific diarrhea, infectious diarrhea, or, rarely, lactose intolerance. Chronic nonspecific
diarrhea is a self-limiting process characterized by multiple loose stools daily (generally worse as
the day progresses), the presence of undigested food in the stool, and/or mild abdominal
discomfort in an otherwise well child or infant. Minimizing juice intake, encouraging a high-
fiber diet or fiber supplementation, and eating a higher-fat diet (which slows intestinal transit
time) may be recommended to manage symptoms. Causes of chronic diarrhea in children with
failure to thrive (Item C135A) and without failure to thrive (Item C135B) are shown.

Infants with chronic diarrhea and failure to thrive require further evaluation, which may include:
• Stool testing
o Infectious studies
o Inflammatory markers (lactoferrin, calprotectin)
o Fecal elastase, fecal fat
o Electrolytes (total stool osmolarity, stool potassium, stool sodium)
• Serum testing for markers of inflammation (sedimentation rate, C-reactive protein),
complete blood cell count, tissue transglutaminase IgA, total IgA
American Academy of Pediatrics 458
PREP® Self-Assessment PREPSA 2023

• Assessment of nutritional status: serum albumin, fat-soluble vitamin levels (A, D, E, K),
and zinc levels
• If secretory diarrhea is suspected, evaluation may also include testing for neuroendocrine
tumors (serum concentrations of vasoactive intestinal peptide [VIP] and gastrin and urine
catecholamines).
• Endoscopic evaluation (upper endoscopy, sigmoidoscopy/colonoscopy) is often
recommended; many causes require histologic evaluation of the small intestine.
Sigmoidoscopy/colonoscopy may also be helpful to identify causes of diarrhea, including
allergic proctocolitis and inflammatory bowel disease.

American Academy of Pediatrics 459


PREP® Self-Assessment PREPSA 2023

Treatment of chronic diarrhea is based on the underlying cause. Supportive care, including
rehydration and achieving optimal nutritional status, is essential for all causes. Antidiarrheals,
including loperamide, are not indicated for acute diarrhea and should not be used for chronic
inflammatory causes of diarrhea because of the potential risk for toxic megacolon. However, on
rare occasions, they may be used (under close supervision) with chronic dysmotility-related
diarrhea or increased ostomy output.

For the infant with failure to thrive in the vignette, neither loperamide nor fiber supplementation
would be indicated. Serum VIP concentrations would not be helpful in this infant without
evidence of secretory diarrhea.

PREP Pearls
• Chronic diarrhea may be classified based on the mechanism: osmotic, secretory,
inflammatory, and dysmotility associated.
• Infants with chronic diarrhea and failure to thrive require additional evaluation, including
blood and stool studies; endoscopic evaluation may be useful to determine the cause.

ABP Content Specifications(s)


• Identify factors predisposing to the development of protracted diarrhea
• Provide appropriate management for a patient who has protracted diarrhea Plan the initial
evaluation of an infant with protracted diarrhea
• Apply age-appropriate guidelines in the use of anti-diarrhea medicines

American Academy of Pediatrics 460


PREP® Self-Assessment PREPSA 2023

Suggested Readings
• American Academy of Pediatrics. Diarrhea and steatorrhea. Point-of-Care Quick
Reference. 2014. doi:10.1542/aap.ppcqr.396060.
• CaJacob NJ, Cohen MB. Update on diarrhea. Pediatr Rev. 2016;37(8):313-322.
doi:10.1542/pir.2015-0099.
• Lo K, Sridhar S, Ali S. Chronic diarrhea and weight loss in an 18-month-old boy. Pediatr
Rev. 2021;42(9):514-518. doi:10.1542/pir.2020-000448.
• Zella GC, Israel EJ. Chronic diarrhea in children. Pediatr Rev. 2012;33(5):207-218.
doi:10.1542/pir.33-5-207.

American Academy of Pediatrics 461


PREP® Self-Assessment PREPSA 2023

Question 136
An 8-year-old girl is seen for a health supervision visit. She was recently diagnosed with a left
optic pathway glioma that is being monitored by her health care team. She is doing well in the
third grade in a general education classroom. The girl’s physical examination findings are
significant only for multiple café au lait macules (at least 6 greater than 5 mm). Sexual maturity
rating is 1 for breast and pubic hair development.

Of the following, this girl’s MOST likely diagnosis is


A. Li-Fraumeni syndrome
B. neurofibromatosis type 1
C. neurofibromatosis type 2
D. tuberous sclerosis complex

American Academy of Pediatrics 462


PREP® Self-Assessment PREPSA 2023

Correct Answer: B
The girl in the vignette has neurofibromatosis type 1 (NF1). She meets the clinical criteria for
NF1 with her optic pathway glioma and at least 6 café au lait macules larger than 5 mm.
Neurofibromatosis type 1 is a tumor predisposition syndrome inherited in an autosomal dominant
manner with complete penetrance. An example of a NF1 pedigree is shown in Item C136A.

Clinical criteria for NF1, recently revised, are below:


B. In the absence of a parent with diagnosed NF1, at least 2 of the following features are
needed to make a clinical diagnosis:
• Café au lait macules (at least 6; >5 mm prepubertal, >15 mm postpubertal) (Item
C136B)
• Freckling in the axillary or inguinal region
• Cutaneous neurofibromas (≥2)
• Lisch nodules (≥2) or choroidal abnormalities (≥2)
• Plexiform neurofibroma (≥1)
• Optic pathway glioma
• Osseous lesion (sphenoid dysplasia, anterolateral tibial bowing, or long bone
pseudoarthrosis) Pathogenic variant in the NF1 gene

C. In the presence of a parent with diagnosed NF1, 1 of the previous features is


sufficient to make a clinical diagnosis.
• Various clinical features seen in NF1 are age dependent:
• Café au lait macules and intertriginous freckling—commonly the first features
seen in young children
• Optic pathway glioma—highest risk for development at 6 to 8 years of age
• Osseous lesions—congenital and can progress over time
• Cutaneous neurofibromas—more commonly seen during puberty in both males
and females, with an increased emergence in pregnant females
• Plexiform neurofibromas of the face and neck—usually seen at birth
• Plexiform neurofibromas of other parts of the body—usually seen before
adolescence

Pediatric surveillance includes annual dilated eye examination, complete physical examination,
and developmental assessment. Although most tumors seen in individuals with NF1 are benign,
there is a 10% lifetime risk of development of a malignant peripheral nerve sheath tumor from an
underlying plexiform neurofibroma. Females with NF1 are at increased risk of developing breast
cancer. The National Cancer Comprehensive Network recommends surveillance with annual

American Academy of Pediatrics 463


PREP® Self-Assessment PREPSA 2023

mammograms starting at 30 years of age. Learning disabilities and behavioral issues, such as
attention-deficit/hyperactivity disorder, occur at a higher incidence in individuals with NF1.

Li-Fraumeni syndrome is an autosomal dominant tumor predisposition syndrome caused by a


pathogenic variant in the tumor suppressor gene TP53. The condition predisposes the individual
to a multitude of childhood and adultonset cancers, including but not limited to adrenocortical
carcinoma, embryonal rhabdomyosarcoma, soft tissue sarcoma, acute lymphocytic leukemia, and
brain tumors.

Neurofibromatosis type 2 is an autosomal dominant tumor predisposition syndrome


characterized by the development of vestibular schwannomas. Affected individuals are also at
risk for the development of cranial and peripheral nerve schwannomas, meningiomas,
ependymomas, and astrocytomas.

Tuberous sclerosis complex involves multiple organ systems and can have the following
manifestations:
• Brain: Subependymal nodules, subependymal giant cell astrocytomas, seizures, cortical
dysplasia, intellectual disability, developmental delay
• Skin: Hypomelanotic macules, shagreen patch, facial angiofibromas, confetti-type
hypopigmented skin lesions, fibrous cephalic plaque, ungual and gingival fibromas
• Kidney: Cysts, angiomyolipomas, renal cell cancer
• Heart: Rhabdomyomas
• Lungs: Lymphangioleiomyomatosis

American Academy of Pediatrics 464


PREP® Self-Assessment PREPSA 2023

The personal and family history for the girl in the vignette is not consistent with these other
tumor predisposition syndromes.

PREP Pearls
• Neurofibromatosis type 1 is an autosomal dominant tumor predisposition syndrome with
complete penetrance.
• Neurofibromatosis type 1 is characterized by development of café au lait macules,
axillary or inguinal freckling, cutaneous neurofibromas, plexiform neurofibroma, optic
pathway glioma, and osseous lesions.
• Various clinical features seen in neurofibromatosis type 1 are age dependent.

ABP Content Specifications(s)


• Recognize the inheritance pattern of neurocutaneous hamartoses (eg, neurofibromatosis)
• Recognize the clinical findings associated with neurofibromatosis in patients of various
ages

Suggested Readings
• Adam MP, Ardinger HH, Pagon RA, et al, eds. Neurofibromatosis 1. GeneReviews
[internet]. Seattle: University of Washington, Seattle; 1993-2022.
• Al-Owain M, Faden M, Chedrawi A. Visual diagnosis: a child who has hyperpigmented
spots and a forearm deformity. Pediatr Rev. 2009;30(5):182-186.
doi:10.1542/pir.30.5.182.
• Dove DE, Smith ML. Neurocutaneous syndromes. In: McInerny TK, Adam HM,
Campbell DE, DeWitt TG, Foy JM, Kamat DM, eds. American Academy of Pediatrics
Textbook of Pediatric Care. American Academy of Pediatrics; 2021:chap 297. Accessed
September 1, 2022. Pediatric Care Online.
• Korf BR, Bebin EM. Neurocutaneous disorders in children. Pediatr Rev. 2017;38(3):119-
128. doi:10.1542/pir.2015-0118.
• Legius E, Messiaen L, Wolkenstein P, et al; International Consensus Group on
Neurofibromatosis Diagnostic Criteria (I-NFDC). Revised diagnostic criteria for
neurofibromatosis type 1 and Legius syndrome: an international consensus
recommendation. Genet Med. 2021;23(8):1506-1513. doi:10.1038/s41436-021-01170-5 .
• Miller DT, Freedenberg D, Schorry E, et al; Council on Genetics, American College of
Medical Genetics and Genomics. Health supervision for children with neurofibromatosis
type 1. Pediatrics. 2019;143(5):e20190660. doi:10.1542/peds.2019-0660.

American Academy of Pediatrics 465


PREP® Self-Assessment PREPSA 2023

Question 137
An 8-month-old infant undergoing treatment for acute lymphoblastic leukemia is seen in the
emergency department for evaluation of fever, diarrhea, and a generalized macular,
erythematous, blanching, nonpruritic rash. Laboratory evaluation findings are significant for
transaminitis. After further evaluation, the infant’s findings are determined to be secondary to a
packed red blood cell transfusion administered 2 weeks ago.

Of the following, the blood product preparation strategy MOST likely to have prevented this
infant’s findings is
A. irradiation
B. leukoreduction
C. selection of the freshest product
D. washing

American Academy of Pediatrics 466


PREP® Self-Assessment PREPSA 2023

Correct Answer: A
Fever, transaminitis, diarrhea, and generalized macular erythematous, blanching, nonpruritic
rash, as seen in the infant in the vignette, are common findings in many conditions, including
viral infections and medication reactions. However, in the context of a recent blood product
(packed red blood cells [PRBCs]) transfusion, transfusionassociated graft-vs-host disease (TA-
GVHD) is this infant’s most likely diagnosis.

Acute graft-vs-host disease is a rare complication of a blood transfusion with a high mortality
rate. The findings usually develop by 2 to 4 weeks after the transfusion. The initial symptom is
often fever, which may develop in the first week. Rash and gastrointestinal symptoms and signs
(eg, anorexia, nausea, vomiting, diarrhea, hepatomegaly, transaminitis, and direct
hyperbilirubinemia) follow. Finally, pancytopenia may develop. The cause of death is usually an
overwhelming infection. Because the signs and symptoms of TA-GVHD mimic many other
conditions, delays in the diagnosis are common.

Transfusion-associated graft-vs-host disease results when donor T cells proliferate in an


immunocompromised host who is unable to reject them. Any blood product with mature,
immunocompetent donor T lymphocytes can cause TAGVHD (PRBCs, platelets, granulocytes,
and fresh, never-frozen plasma). Children at risk for TA-GVHD include those with an
immunodeficiency (eg, severe combined immunodeficiency), newborns (because of their
immature immune system), and children who have received immunosuppressive treatment (eg,
chemotherapy).

There is no adequate treatment for TA-GVHD. Medications that have been tried include
corticosteroids, cyclosporine, anti-CD3 monoclonal antibody, and serine protease inhibitors that
inhibit cytotoxic T cells. Treatment with a recently available anti–interleukin 2 receptor antibody
(daclizumab) may be considered.

Prevention of TA-GVHD is key because of the high mortality risk and lack of effective
treatment. Irradiation of PRBCs eradicates the donor lymphocytes, making TA-GVHD less
likely to occur. However, irradiation reduces the shelf-life of the PRBCs to approximately 1
month. If the PRBCs are stored after irradiation, there is an increase in the potassium content of
the product. Therefore, it is best to irradiate the blood product immediately before use, if
possible, or to use the product within 28 days of irradiation.

Fresh PRBCs have a high amount of donor T cells. Although it is preferred that young children
receive the freshest product, irradiation is needed to eliminate donor lymphocytes.
Leukoreduction, which has been almost universally adapted for all blood products, significantly
reduces the number of white blood cells in PRBCs. This practice decreases adverse transfusion
reactions (such as fevers) but does not prevent TA-GVHD. Washing of RBCs eliminates
products that accumulate during storage (eg, potassium and cytokines). It does not reduce the
amount of donor T cells, which lead to TA-GVHD.

American Academy of Pediatrics 467


PREP® Self-Assessment PREPSA 2023

PREP Pearls
• Transfusion-associated graft-vs-host disease has a high mortality rate.
• Irradiation of blood products administered to an immunodeficient individual is paramount
in preventing transfusionassociated graft-vs-host disease.

ABP Content Specifications(s)


• Recognize the clinical features of graft-versus-host disease (transfusion related)

Suggested Readings
• Lanzkowsky P, Lipton J, Fish JD. Lanzkowsky’s Manual of Pediatric Hematology and
Oncology. 6th ed. Elsevier Inc; 2016:729-732.
• Ozgonenel B, Nash TA, Rajpurkar M. Blood components for pediatric transfusions.
Pediatr Rev. 2020;41(5):259-261. doi:10.1542/pir.2018-0306.
• Schroeder ML. Transfusion-associated graft-versus-host disease. Br J Haematol.
2002;117(2):275-287. doi:10.1046/j.1365-2141.2002.03450.x.

American Academy of Pediatrics 468


PREP® Self-Assessment PREPSA 2023

Question 139
A 14-year-old adolescent boy is seen for concerns about a worsening chest deformity. He is self-
conscious about his appearance and has experienced difficulty exercising. He has no other
known health problems. On physical examination, his oxygen saturation in room air is 100% by
pulse oximetry. There is a moderately severe depression of his sternum with some molding of the
anterior ribs (Item Q139). Breath sounds are equal with no stridor or wheeze; there is good air
exchange. Cardiac sounds are displaced slightly to the left. The remainder of the adolescent’s
physical examination findings are normal.

A chest radiograph shows a long, narrow thoracic cavity with the heart slightly displaced to the
left on anteriorposterior view and the sternum displaced posteriorly on the lateral view. Complete
lung function testing including spirometry and lung volumes is normal for age and height.

Of the following, the BEST test to evaluate the impact of this adolescent’s condition on his
respiratory function is
A. cardiopulmonary exercise testing
B. computed tomography scan of the chest
C. echocardiography
D. a lung ventilation-perfusion scan

American Academy of Pediatrics 469


PREP® Self-Assessment PREPSA 2023

Correct Answer: A
The adolescent in the vignette has a pectus excavatum chest deformity that is causing him
concern about his appearance and exercise capacity. The most common chest wall deformity,
pectus excavatum often has a significant negative impact on body image and perceived exercise
capacity. Static pulmonary function testing results are rarely abnormal, as is the case for the boy
in the vignette. Cardiopulmonary exercise testing may uncover subtle deficits in exercise
capacity and is the best test to further evaluate his respiratory function.

Surgical repair of pectus excavatum has been shown to increase lung volumes, air flow, and
exercise capacity to within the normal range on pulmonary function and cardiopulmonary
exercise testing, but the greatest improvement is in body image perception and subjective
assessment of exercise performance.

The degree of pectus excavatum can be quantitated on computed tomography (CT) of the chest
using the Haller index: the ratio of the internal transverse rib-to-rib diameter to the distance
between the spine and sternum on a cross-sectional CT view (Item C139). An index of 2.5 or
less is normal; the higher the index, the worse the pectus deformity. However, the Haller index is
a poor biomarker of functional limitation; there is no direct correlation between severity as
determined by Haller index and any pulmonary function abnormalities.

More severe pectus deformities can cause displacement of the heart to the left and cardiac
conduction abnormalities. Exercise intolerance may have both cardiac and pulmonary
components. While echocardiography is indicated in the evaluation of exercise intolerance in
children and adolescents with pectus excavatum, this test will not define any functional
respiratory deficits caused by the chest wall deformity.

A lung ventilation-perfusion scan is not indicated for the adolescent in the vignette. Chest wall
deformities do not cause ventilation-perfusion mismatch.

American Academy of Pediatrics 470


PREP® Self-Assessment PREPSA 2023

The second most common chest wall abnormality is scoliosis, which has many etiologies.
Idiopathic scoliosis is rarely associated with objective pulmonary function changes until it is
very severe. A restrictive pulmonary physiology is expected in any individual with scoliosis,
with loss of lung volume on the side of the concavity and a hyperexpanded lung associated with
stretched and poorly compliant intercostal muscles on the convex side. A rotational component
and/or kyphosis worsens chest wall compliance and function, further restricting lung volume and
chest wall expansion with respiration. When scoliosis is secondary to neuromuscular conditions,
the underlying poor function of the intercostal muscles and diaphragm compounds the restrictive
pulmonary physiology. There is a direct relationship between the severity and complexity of
scoliosis and restrictive pulmonary physiology. Surgical management of severe or progressive
scoliosis can stabilize lung function and prevent further decline; however, it is rarely associated
with improvement in lung function even when the chest wall abnormalities are corrected.

Early severe scoliosis is associated with structural abnormalities of the spine (eg, hemivertebrae,
bar vertebrae) with or without rib fusions or absent ribs. Affected children may have life-
threatening pulmonary restriction from birth and require ventilator support until intervention can
be provided to stabilize and expand the chest wall. This constellation of anatomic anomalies and
severe restrictive pulmonary physiology (thoracic insufficiency syndrome) requires surgical
intervention to preserve linear growth. If linear growth is not preserved, pulmonary restriction
will worsen as the child ages. Placement of expanding rods in the spine for chest/rib stabilization
promotes ongoing skeletal growth through interval extension.

PREP Pearls
• Severe chest wall deformities can cause restrictive pulmonary physiology. The severity of
the restrictive lung disease is directly related to the complexity of the chest deformity.
• Pectus excavatum can have a significant negative impact on self-image and perceived
exercise capacity; static pulmonary function testing is usually normal.
• Surgical repair of scoliosis is not likely to improve lung function but can prevent it from
worsening.

ABP Content Specifications(s)


• Recognize the association of thoracic deformities with restrictive pulmonary disease

American Academy of Pediatrics 471


PREP® Self-Assessment PREPSA 2023

Suggested Readings
• Alapati D, Shaffer TH. Skeletal dysplasia: respiratory management during infancy.
Respir Med. 2017;131:18-26. doi:10.1016/j.rmed.2017.07.063 .
• Kelly RE Jr, Cash TF, Shamburger RC, et al. Surgical repair of pectus excavatum
markedly improves body image and perceived ability for physical activity: multicenter
study. Pediatrics. 2008;122(6):1218-1222. doi:10.1542/peds.20072723 .
• Mayer OH, Allen JL. Chest wall and spinal deformities. In: Light MJ, ed. Pediatric
Pulmonology. American Academy of Pediatrics; 2011:309-345.
• Mino J, Stallion A, Monteiro R. Pectus excavatum and pectus carinatum. In: McInerny
TK, Adam HM, Campbell DE, DeWitt TG, Foy JM, Kamat DM, eds. American
Academy of Pediatrics Textbook of Pediatric Care. American Academy of Pediatrics;
2021:chap 309. Accessed September 1, 2022. Pediatric Care Online.
• Redding GJ. Clinical issues for pediatric pulmonologists managing children with thoracic
insufficiency syndrome. Front Pediatr. 2020;8:392. doi:10.3389/fped.2020.00392.
• Yang S, Andras L, Redding G, Skaggs D. Early-onset scoliosis: a review of history,
current treatment and future directions. Pediatrics. 2016;137(1). doi:10.1542/peds.2015-
0709.

American Academy of Pediatrics 472


PREP® Self-Assessment PREPSA 2023

Question 140
A 1-month-old infant is seen for a health supervision visit. His mother is concerned about his
head shape. The infant was born at term after an unremarkable pregnancy and delivery. He has
been well and is meeting age-appropriate developmental milestones. His family history is
unremarkable. The infant’s growth parameters are normal, including head circumference. His
physical examination findings are significant only for the head shape shown in Item Q140.

Item Q140: Head shape of the infant described in the vignette.


Reprinted with permission from Dias MS, Samson T, Rizk EB, Governale LS, Richtsmeier JT and Section
on Neurologic Surgery, Section on Plastic and Reconstructive Surgery. Identifying the misshapen head.
Pediatrics. 2020;146(3):e2020015511.

Of the following, the BEST next management step for this infant is
A. helmet therapy
B. neurosurgery referral
C. physical therapy
D. reassurance

American Academy of Pediatrics 473


PREP® Self-Assessment PREPSA 2023

Correct Answer: D
The infant in the vignette has bathrocephaly resulting from a persistent mendosal suture (Item
C140A). This suture usually disappears by the time of birth but when persistent can cause a
prominent occiput. Bathrocephaly usually resolves without intervention, so the infant’s mother
can be reassured that her son’s head shape will likely normalize. He does not require helmet
therapy, referral to neurosurgery, or physical therapy for correction.

Persistent mendosal suture (arrowheads)

Bathrocephaly may be confused with sagittal synostosis, which is caused by abnormal fusion of
the 2 parietal bones (Item C140B). Sagittal synostosis results in frontal bossing, a midline
sagittal ridge, and narrowing of the biparietal diameter, features not associated with
bathrocephaly. Infants with sagittal synostosis should be referred to neurosurgery as soon as
possible (ideally by 6 to 12 weeks of age); late referral (6 to 10 months of age or later) is
associated with the need for significantly more extensive surgical correction. Helmet therapy is
usually also recommended after surgical correction of sagittal synostosis.

Item C140B: Sagittal synostosis

American Academy of Pediatrics 474


PREP® Self-Assessment PREPSA 2023

Dolichocephaly can also be confused with sagittal synostosis. This condition occurs most
frequently in premature infants in the neonatal intensive care unit. While these infants have an
elongated skull, they do not have biparietal diameter narrowing.

Early identification of a concerning head shape is crucial to ensure that infants receive treatment
before the window of opportunity closes. Furthermore, identification of benign variations in skull
shape can avoid unnecessary referrals and ease parental anxiety.

One of the most common benign variations of head shape is occipital positional deformation
(positional plagiocephaly), which most commonly results from supine positioning. Unilateral
flattening of the occiput results in plagiocephalic changes with a parallelogram head shape (Item
C140C). Bilateral flattening results in positional brachycephaly. Treatment involves
repositioning education and physical therapy. For infants with moderate to severe positional
deformation or older infants, helmet therapy can be beneficial.

Benign variations in head shape must be differentiated from certain types of craniosynostosis.
Craniosynostosis is the premature closure of 1 or more cranial sutures and abnormal fusion of the
contiguous bones that border the suture. Depending on which suture line is involved, head shape
is altered, as shown in Item C140D.

American Academy of Pediatrics 475


PREP® Self-Assessment PREPSA 2023

Unicoronal synostosis (anterior plagiocephaly) and bicoronal synostosis (anterior brachycephaly)


are caused by fusion of the frontal and parietal bones and may be mistaken as occipital positional
deformation. However, anterior plagiocephaly results in a trapezoidal head shape (Item C140E)
and the radiographic changes shown in Item C140F. Anterior brachycephaly results in changes
shown in Item C140G.

Lambdoid synostosis may also be mistaken for occipital positional deformation. This condition
is extremely rare and results in a head shape similar to a trapezoid instead of a parallelogram
(Item C140H).

Metopic synostosis results from prematurely fused frontal bones, which produces a prominent
metopic ridge and trigonocephaly (Item C140I).

American Academy of Pediatrics 476


PREP® Self-Assessment PREPSA 2023

The least common craniosynostosis is frontosphenoidal craniosynostosis and produces the


changes shown in Item C140J.

Anterior plagiocephaly

Unilateral lambdoid synostosis

Trigonocephaly
Management of most cases of craniosynostosis requires early referral to neurosurgery. Head
computed tomography is performed to assist in surgical planning. Some types of
craniosynostosis are syndromic; genetics consultation may be helpful to identify potential
involvement of other organ systems. Children with developmental concerns require
developmental/behavioral support.

Craniosynostosis is a rare cause of microcephaly, defined as a head circumference less than the
5th percentile. Common causes of microcephaly are listed in Item C140K. Children with
microcephaly, especially those with deceleration of growth, abnormal physical examination
findings, and abnormal development should be referred for further evaluation.

American Academy of Pediatrics 477


PREP® Self-Assessment PREPSA 2023

Frontosphenoidal craniosynostosis

Item C140K: Conditions Causing Microcephaly


• Primary microcephaly
o Chromosomal disorders
o Anencephaly
o Encephalocele
o Holoprosencephaly
o Agenesis of the corpus callosum
o Neuronal migration disorders
o Microcephaly vera
• Secondary microcephaly
o Intrauterine infections
o Intrauterine toxins
o Intrauterine vascular insufficiency
o Hypoxic-ischemic brain injury
o Intracranial hemorrhage
o Neonatal infections (meningitis and
o encephalitis)
o Neonatal stroke
o Chronic cardiopulmonary or renal disease
American Academy of Pediatrics 478
PREP® Self-Assessment PREPSA 2023

o Malnutrition
o Craniosynostosis

Adapted from Pina-Garza J. Fenichel’s Clinical PediatricmNeurology. 2nd ed. Amsterdam, Netherlands:
Elsevier; 013:359.

PREP Pearls
• Bathrocephaly, which is caused by a persistent mendosal suture and results in a
prominent occiput, usually resolves without intervention.
• Early identification of concerning head shape is crucial so that infants receive treatment
before the window of opportunity closes.
• Craniosynostosis is a rare cause of microcephaly.

MOCA-Peds Objective
• Evaluate an infant with microcephaly.

ABP Content Specifications(s)


• Plan the management of a patient with an abnormal head shape and/or growth (eg,
craniosynostosis, plagiocephaly, microcephaly, macrocephaly)
• Differentiate between normal and abnormal variations in head shape and/or growth (eg,
craniosynostosis, plagiocephaly, microcephaly, macrocephaly)
• Differentiate among the possible causes of abnormal head shape and/or growth (eg,
craniosynostosis, plagiocephaly, microcephaly, macrocephaly)

Suggested Readings
• Dias MS, Samson T, Rizk EB, Governale LS, Richtsmeier JT and Section on Neurologic
Surgery, Section on Plastic and Reconstructive Surgery. Identifying the misshapen head:
craniosynostosis and related disorders. Pediatrics. 2020;146(3):e2020015511.
doi:10.1542/peds.2020-015511.
• Flannery AM, Tamber MS, Mazzola C, et al. Congress of neurological surgeons
systematic review and evidence-based guidelines for the management of patients with
positional plagiocephaly: executive summary. Neurosurgery. 2016;79(5):623-624.
doi:10.1227/NEU.0000000000001426.
• Kamat DM, Ahmer A. Positional deformational plagiocephaly. In: McInerny TK, Adam
HM, Campbell DE, DeWitt TG, Foy JM, Kamat DM, eds. American Academy of
Pediatrics Textbook of Pediatric Care. American Academy of Pediatrics; 2021:chap 316.
Accessed September 1, 2022. Pediatric Care Online.
• Purugganan OH. Microcephaly. In: McInerny TK, Adam HM, Campbell DE, DeWitt TG,
Foy JM, Kamat DM, eds. American Academy of Pediatrics Textbook of Pediatric Care.
American Academy of Pediatrics; 2021:chap 178. Accessed September 1, 2022. Pediatric
Care Online.

American Academy of Pediatrics 479


PREP® Self-Assessment PREPSA 2023

Question 141
A 15-year-old adolescent boy is brought to the office for a health supervision visit. His maternal
grandfather received a kidney transplant secondary to autosomal dominant polycystic kidney
disease at age 58 years. One year ago, the adolescent underwent renal ultrasonography screening
that showed multiple bilateral cysts, the largest measuring 2 cm in the left kidney and 1.5 cm in
the right kidney. The boy and his parents have no concerns today. He is doing well in school. He
has not had headaches, dizziness, flank pain, swelling in any parts of his body, increased urinary
frequency, or dysuria.

The adolescent’s weight is at the 25th percentile, height is at the 40th percentile, and blood
pressure is 120/72 mm Hg. The remainder of his physical examination findings are
unremarkable.

Of the following, the BEST next step in this adolescent’s evaluation is to perform
A. computed tomography of the brain
B. Doppler ultrasonography of the liver
C. genetic testing
D. urinalysis with microscopy

American Academy of Pediatrics 480


PREP® Self-Assessment PREPSA 2023

Correct Answer: D
The adolescent in the vignette most likely has autosomal dominant polycystic kidney disease
(ADPKD). He has multiple cysts visualized on renal ultrasonography and a family history of
ADPKD. While he remains asymptomatic, he should be screened with annual blood pressure
measurements (to monitor for hypertension) and urinalysis with microscopy (to detect
proteinuria and hematuria). Annual blood pressure measurement and urinalysis should be
performed for all asymptomatic children with a family history of ADPKD. Serum creatinine
values are usually normal in children with ADPKD and kidney function does not typically
worsen until adulthood.

Autosomal dominant polycystic kidney disease is the most common inherited kidney disease,
with an incidence of 1 in 1,000 live births. The cysts gradually enlarge and replace the normal
renal parenchyma leading to kidney failure. The signs and symptoms of ADPKD usually appear
at age 30 to 40 years; these include hypertension, hematuria, proteinuria, flank pain, kidney
stones, and urinary tract infection. Most children with ADPKD are asymptomatic. They are
typically diagnosed incidentally on an imaging study or during screening due to family history.
Autosomal dominant polycystic kidney disease can manifest in the neonatal period with an
abdominal mass. Children may occasionally present before age 10 years with hypertension,
proteinuria, or a renal concentrating defect (eg, polyuria, polydipsia); some may experience early
progression to end-stage renal disease. Extrarenal manifestations, rarely seen in children, may
include cysts in other organs (eg, liver, pancreas, and seminal vesicle), cerebral aneurysm, mitral
valve prolapse, and colonic diverticula. Cerebral aneurysm is a rare manifestation that can
present with a life-threatening intracranial bleed during adulthood.

In children, the diagnosis of ADPKD is based on family history and the presence of one or more
cysts on renal ultrasonography. Autosomal dominant polycystic kidney disease is associated with
macroscopic cysts (>10 mm) on renal ultrasonography, while autosomal recessive polycystic
kidney disease (ARPKD) is associated with numerous microscopic cysts. While computed
tomography and magnetic resonance imaging are more sensitive for the detection of renal cysts,
ultrasonography is the preferred imaging study because of the lack of radiation exposure and
cost-effectiveness.

Genetic testing is recommended for a child for whom the diagnosis of ADPKD is unclear (eg,
equivocal imaging, inconsistent family history) or for a family member being considered for
kidney donation. Because the boy in the vignette has a strong family history of ADPKD and
characteristic renal ultrasonography findings, genetic testing is not essential and would not be the
next step in management. A computed tomography scan of the brain to screen for cerebral
aneurysm is not recommended for asymptomatic children since aneurysmal rupture is extremely
rare. Doppler ultrasonography of the liver is helpful in diagnosing portal hypertension secondary
to congenital hepatic fibrosis, which is associated with ARPKD. However, hepatic fibrosis is not
associated with ADPKD and therefore, liver ultrasonography would not be indicated for the child
in the vignette.

American Academy of Pediatrics 481


PREP® Self-Assessment PREPSA 2023

Management of ADPKD includes treatment of hypertension and/or proteinuria with angiotensin-


converting enzyme inhibitors or angiotensin receptor blockers, and treatment of urinary tract
infections, kidney stones, and extra renal complications. Long-term monitoring of renal function,
management of chronic kidney disease, and renal replacement therapy (dialysis or kidney
transplant) may be required.

PREP Pearls
• Autosomal dominant polycystic kidney disease is the most common inherited kidney
disease.
• Most children with autosomal dominant polycystic kidney disease are asymptomatic and
diagnosed incidentally on an imaging study or during screening due to family history.
• Annual blood pressure measurement and urinalysis (to monitor for proteinuria and
hematuria) should be performed for asymptomatic children with a family history of
autosomal dominant polycystic kidney disease.

ABP Content Specifications(s)


• Recognize the clinical findings associated with autosomal-dominant polycystic kidney
disease in patients of various ages
• Plan the appropriate diagnostic evaluation for a patient in whom autosomal-dominant
polycystic kidney disease is suspected

Suggested Readings
• Benun J, Lewis C. Polycystic kidney disease. Pediatr Rev. 2009;30(10):e78-e79;
discussion e79. doi:10.1542/pir.30-10-e78
• De Rechter S, Breysem L and Mekahli D. Is autosomal dominant polycystic kidney
disease becoming a pediatric disorder? Front Pediatr. 2017;5:272.
doi:10.3389/fped.2017.00272.
• Janjua HS, Lam KS, Gupta V, Krishna S. Congenital anomalies of the kidneys, collecting
system, bladder, and urethra. Pediatr Rev. 2019;40(12):619-626. doi:10.1542/pir.2018-
0242.

American Academy of Pediatrics 482


PREP® Self-Assessment PREPSA 2023

Question 142
A 14-day-old, low-birth-weight neonate undergoing parenteral antibiotic therapy for bacterial
sepsis in the neonatal intensive care unit becomes clinically unstable.

Laboratory data are shown:

Laboratory Test Result


White blood cell count 18,000/µL (18.0 × 109/L)
Hemoglobin 8.7 g/dL (87 g/L)
Hematocrit 25.8%
Platelet count 339 × 103/µL (339 × 109/L)
Sodium 143 mEq/L (143 mmol/L)
Potassium 4.8 mEq/L (4.8 mmol/L)
Chloride 108 mEq/L (108 mmol/L)
Carbon dioxide 23 mEq/L (23 mmol/L)
Blood urea nitrogen 4.0 mg/dL (1.4 mmol/L)
Creatinine <0.2 mg/dL (<17.9 µmol/L)
Glucose 174 mg/dL (9.7 mmol/L)

Gram stain of the neonate’s positive blood culture reveals fungi concerning for Candida albicans.

Parenteral antifungal therapy is started. Twenty-four hours after initiation of antifungal therapy,
the following laboratory data are obtained:
Laboratory Test Result
White blood cell count 13,000/µL (13.0 × 109/L)
Hemoglobin 8.0 g/dL (80 g/L)
Hematocrit 24.2%
Platelet count 275 × 103/µL (275 × 109/L)
Sodium 136 mEq/L (136 mmol/L)
Potassium 2.6 mEq/L (2.6 mmol/L)
Chloride 105 mEq/L (105 mmol/L)
Carbon dioxide 18 mEq/L (18 mmol/L)
Blood urea nitrogen 25 mg/dL (8.9 mmol/L)
Creatinine 0.7 mg/dL (61.9 µmol/L)
Glucose 99 mg/dL (5.5 mmol/L)

Of the following, the MOST likely antifungal agent that was started is
A. amphotericin B
B. fluconazole
C. flucytosine
D. micafungin

American Academy of Pediatrics 483


PREP® Self-Assessment PREPSA 2023

Correct Answer: A
The neonate in the vignette developed fungemia (candidemia) requiring parenteral antifungal
therapy. Within 24 hours after starting treatment the neonate appears to have significant
nephrotoxicity with acute kidney injury as evidenced by elevated serum blood urea nitrogen and
creatinine levels and hypokalemia. Of the response choices, amphotericin B is most likely to
cause acute and significant nephrotoxicity. Fluconazole, flucytosine, and micafungin are not
associated with kidney injury.

Invasive candidiasis is a leading cause of mortality in neonatal intensive care units, particularly
in extremely lowbirth-weight infants. There are multiple antifungal agents available to treat
invasive fungal infections in children; the choice of medication is dependent on the site of
infection, patient age, severity of illness, specific organism, and antimicrobial susceptibility
patterns. The 4 main classes of antifungal agents used in the treatment of systemic fungal
infections in children are macrocyclic polyenes (eg, amphotericin B), azoles (eg, fluconazole),
fluorinated pyrimidines (eg, flucytosine), and echinocandins (eg, micafungin). Each class has
advantages and disadvantages depending on the side-effect profile and target of therapy.

Amphotericin B is a macrocyclic polyene that binds to ergosterol in fungal cell membranes,


causing membrane disruption, an increase in cellular permeability, cell lysis, and cellular death.
The primary side-effect is renal toxicity, which occurs in up to 50% of children treated with this
medication; lipid preparations of amphotericin were developed to mitigate the nephrotoxic side
effects. The nephrotoxicity is characterized by renal tubular acidosis, hypokalemia, and renal
failure; potassium supplementation is often required. This toxicity is often transient and
generally reversible with changes in medication dosing and elimination of other nephrotoxic
medications. Less common adverse reactions include fever, rigors, and chills, which are often
associated with rapid infusion of amphotericin B, and may be mitigated by pre-treatment with
antipyretic and antihistamine therapy.

There are 2 subclasses of azole antifungals, imidazole and triazole. Fluconazole was the first
triazole developed and has activity against most yeasts and fungal diseases such as
histoplasmosis and blastomycosis. Newer triazoles (eg, voriconazole) have activity against
Aspergillus. All of the triazoles inhibit hepatic cytochrome p450 enzyme pathways resulting in
increased levels of other hepatically metabolized drugs. Hepatotoxicity is the most notable side
effect of triazole agents and thus liver enzymes should be monitored frequently.

Flucytosine inhibits protein and DNA synthesis following conversion to 5-fluorouracil within
fungal cells. Flucytosine is active against many yeasts and some molds; it is most often
prescribed as adjunctive therapy for cryptococcal meningitis. Flucytosine exhibits concentration-
dependent toxicity, which manifests most frequently as hepatotoxicity and bone marrow
suppression. Unlike other antifungal agents, therapeutic drug level monitoring is recommended
to prevent toxicity.

Echinocandin antifungals (eg, micafungin and caspofungin) inhibit (1,3)-β-d-Glucan synthase


activity and prevent synthesis of the fungal cell wall. These agents have fungicidal activity
American Academy of Pediatrics 484
PREP® Self-Assessment PREPSA 2023

against Candida and fungistatic activity against Aspergillus. The most common adverse reactions
include infusion reactions and transient elevation of liver enzymes.

PREP Pearls
• The primary toxic side effect of amphotericin B, including liposomal preparations, is
nephrotoxicity.
• Antifungals may be associated with hepatotoxicity or inhibition of the cytochrome p450
enzyme system, which can result in elevated levels of other medications cleared by the
liver.
• With the exception of flucytosine, therapeutic drug level monitoring is rarely necessary
when prescribing antifungal therapy.

ABP Content Specifications(s)


• Recognize the adverse effects associated with the use of various antifungal drugs

Suggested Readings
• Almirante B, Rodríguez D. Antifungal agents in neonates. Paediatr Drugs.
2007;9(5):311-321. doi:10.2165/00148581200709050-00004.
• American Academy of Pediatrics. Antifungal drugs for systemic fungal infections. In:
Kimberlin DW, Barnett ED, Lynfield R, Sawyer MH, eds. Red Book: 2021–2024 Report
of the Committee on Infectious Diseases. 32nd ed. American Academy of Pediatrics;
2021. Accessed September 1, 2022. Red Book Online.
• Downes KJ, Fisher BT, Zane NR. Administration and dosing of systemic antifungal
agents in pediatric patients. Paediatr Drugs. 2020;22(2):165-188. doi:10.1007/s40272-
020-00379-2.
• Smitherman L. In Brief: antifungal drugs. Pediatr Rev. 2016;37(6):267-268.
doi:10.1542/pir.2015-0171.

American Academy of Pediatrics 485


PREP® Self-Assessment PREPSA 2023

Question 143
A 4-year-old boy with a noncontributory medical history is brought to the emergency
department. He awoke around 3 AM crying and complaining of severe abdominal pain. The pain
persisted throughout the night. This morning he continues to complain, and his oral intake has
been poor. When asked where he hurts, the boy points to the periumbilical region. He has not
had any emesis, fever, constipation, diarrhea, rash, joint pain, cough, shortness of breath, sore
throat, testicular pain, or recent trauma.

The boy has a temperature of 36.7°C, heart rate of 106 beats/min, respiratory rate of 22
breaths/min, and blood pressure of 109/72 mm Hg. He appears uncomfortable, lying in his
mother’s arms and groaning. His abdomen is tender to palpation in the periumbilical area
without rebound tenderness or guarding. The remainder of his physical examination findings are
unremarkable.

Findings on left lateral decubitus abdominal radiography are shown in Item Q143.

Item Q143: Left lateral decubitus radiograph for the boy in the vignette.
Courtesy of T. Swan

Of the following, the BEST next step in this boy’s management is


A. admission to the hospital for serial abdominal examinations
B. close outpatient follow-up with serial abdominal radiography
C. emergency referral for endoscopic management
D. emergency referral for surgical management

American Academy of Pediatrics 486


PREP® Self-Assessment PREPSA 2023

Correct Answer: D
The boy in the vignette’s abdominal radiograph shows 3 circular radiopaque foreign bodies that
appear to be touching, raising concern that the boy ingested magnets. Because the boy is
symptomatic with multiple magnets that have passed through the stomach, the best next step in
management is emergency referral for surgical removal. The algorithm for magnet ingestion in
children is shown in Item C143. If the magnets were located within the esophagus or stomach,
emergency referral for endoscopic removal would be the appropriate next step. Neither close
outpatient follow-up with serial abdominal radiography or admission to the hospital for serial
abdominal examinations would be appropriate for this child. With ingestion of multiple magnets,
he is at risk for bowel injury or perforation if the magnets are not removed promptly.

The incidence of magnet ingestion in children has significantly increased since 2002; in
approximately 50% of cases the child has ingested 2 or more magnets. For children with known
or suspected magnet ingestion, the first step in management is to obtain plain radiographs of the
neck, chest, and abdomen to ascertain the location and number of magnets ingested. If a single
magnet was ingested and the child is asymptomatic, it may be appropriate to follow the child as
an outpatient with serial abdominal radiography until the magnet has been successfully excreted
with the stool. Endoscopic removal is necessary if the magnet is large (typically >2 cm) and
becomes impacted, is abnormally shaped, or fails to pass as expected.
American Academy of Pediatrics 487
PREP® Self-Assessment PREPSA 2023

Ingestion of multiple magnets is an emergency and warrants immediate removal regardless of


symptoms. If the magnets are located in the esophagus or stomach, endoscopic removal is
indicated. If endoscopic removal is unsuccessful, surgical removal is indicated. It is reasonable to
remove the magnets by enteroscopy or colonoscopy, or to follow the situation closely with serial
abdominal radiography if the following criteria are met:
• Multiple magnets are present beyond the stomach
• The child is asymptomatic
• There is no evidence of obstruction or perforation on abdominal radiographs

If multiple magnets are present past the stomach and the child is symptomatic, as seen in the
child in the vignette, emergency surgical removal is indicated.
The clinical presentation of a child with magnet ingestion can be variable and nonspecific,
ranging from asymptomatic to severe abdominal pain with peritonitic signs. Presenting
symptoms may include nausea, vomiting, diarrhea, abdominal pain, bloody stools, or fever.
Because of the magnets’ ability to attract to one another across bowel mucosa, failure to
immediately remove multiple ingested magnets may lead to bowel obstruction, perforation,
peritonitis, bowel ischemia, necrosis, or fistula formation.

PREP Pearls
• For children with known or suspected magnet ingestion, the first step in management is
to obtain plain radiographs of the neck, chest, and abdomen to ascertain the location and
number of magnets ingested.
• If multiple ingested magnets are present past the stomach in a symptomatic child,
emergency surgical removal is indicated.
• The clinical presentation of a child with magnet ingestion can be variable and
nonspecific, ranging from asymptomatic to severe abdominal pain with peritonitic signs.

ABP Content Specifications(s)


• Plan the management of a patient who has ingested a magnet

Suggested Readings
• Cagil Y, Diaz J, Iskowitz S, Muniz Crim AJ. Ingested foreign bodies and toxic materials:
who needs to be scoped and when? Pediatr Rev. 2021;42(6):290-301.
doi:10.1542/pir.2018-0327.
• Larsen CD, King MA. Case 3: abdominal pain and vomiting in a 4-year-old boy. Pediatr
Rev. 2015;36(3):132-134. doi:10.1542/pir.36-3-132.
• Sheridan DC, Hughes A, Horowitz BZ. Pediatric ingestions: new high-risk household
hazards. Pediatr Rev. 2021;42(1):2-10. doi:10.1542/pir.2019-0309.
• Skae CC, Parikh SR. Foreign bodies of the ear, nose, airway, and esophagus. In:
McInerny TK, Adam HM, Campbell DE, DeWitt TG, Foy JM, Kamat DM, eds.
American Academy of Pediatrics Textbook of Pediatric Care. American Academy of
Pediatrics; 2021:chap 251. Accessed September 1, 2022. Pediatric Care Online.

American Academy of Pediatrics 488


PREP® Self-Assessment PREPSA 2023

Question 144
An 8-month-old female infant is being evaluated in the emergency department for a progressive
rash and fever. The infant was seen in an urgent care center 2 weeks ago, diagnosed with
gingivostomatitis, and treated with supportive care. The oral lesions resolved over 1 week, and
she was doing well until 2 days ago when she developed a low-grade fever and a rash that started
on her trunk and spread to her extremities. The infant has a temperature of 38.7°C, a heart rate of
120 beats/min, and a respiratory rate of 30 breaths/min. She appears alert, is not in acute distress,
and appears nontoxic. There are multiple annular plaques with dusky centers on her trunk,
extensor aspects of all extremities, and palms (Item Q144). There is no conjunctival injection or
any oral lesions.

Item Q144: Findings for the infant described in the vignette.


Reprinted with permission from Ferrandiz-Pulido C, Garcia-Patos V. Arch Dis Child.
2013;98(12):999.

Of the following, the MOST accurate statement regarding this infant’s rash is that it is
A. associated with arthritis
B. characterized by intense pruritus
C. commonly triggered by infections
D. expected to resolve in 24 hours

American Academy of Pediatrics 489


PREP® Self-Assessment PREPSA 2023

Correct Answer: C
The infant in the vignette has an annular rash with dusky center following a herpes simplex virus
(HSV) gingivostomatitis infection that is suggestive of erythema multiforme (EM). Erythema
multiforme is commonly triggered by infections. It is not associated with arthritis; any associated
pruritus is mild, not intense; and erythema multiforme is expected to resolve in 2 to
3 weeks, not 24 hours.

Erythema multiforme is an immune-mediated, self-limiting disorder involving the skin and


mucous membranes. The EM rash is described as annular targetoid plaques with 3 zones: a
dusky center, surrounding pallor, and a rim of erythema. The centers of the lesions can become
bullous. The lesions symmetrically involve the extremities, trunk, palms, and soles. Individual
lesions last for 1 to 2 weeks; transient lesions are more consistent with urticaria. Pruritus, if
present, is usually mild, and systemic symptoms such as fever and myalgias may occur
especially in those with mucous membrane lesions.

Erythema multiforme can be classified as EM minor, with involvement of the skin alone, or EM
major, with mucous membrane involvement (about 50% of cases). In the past, EM was thought
to be along the spectrum of StevensJohnson syndrome but now is considered a separate
condition.

Infections trigger about 90% of EM cases, while less than 10% are attributable to medications.
The most common infectious trigger in children is HSV, followed by mycoplasma.
Antiepileptics, nonsteroidal anti-inflammatory drugs, sulfonamides, and other antibiotics are
common medication triggers of EM.

Erythema multiforme resolves spontaneously within 2 to 3 weeks and usually requires only
supportive care.

Treatment of triggering infections should be considered, when appropriate, and precipitating


medications should be discontinued. There may be recurrence of EM in 10% to 20% of cases.
Factors associated with recurrence include herpes simplex virus as the trigger and underlying
genetic predisposition.

The differential diagnosis of EM includes urticaria multiforme, serum sickness-like reactions,


vasculitis, and viral exanthems. Item C144 shows differentiating features of common conditions
considered in the differential diagnoses

American Academy of Pediatrics 490


PREP® Self-Assessment PREPSA 2023

PREP Pearls
• Erythema multiforme is an immune-mediated condition characterized by targetoid lesions
on the trunk, extremities, palms, and soles.
• Infections, especially herpes simplex virus, are a common trigger for erythema
multiforme.
• Erythema multiforme is self-limiting and resolves with supportive care within 2 to 3
weeks.

MOCA-Peds Objective
• Recognize common viral exanthems.

ABP Content Specifications(s)


• Recognize the clinical spectrum of erythema multiforme

Suggested Readings
• Henderson L, Williams JV. Drug eruptions, erythema multiforme, Stevens-Johnson
Syndrome. In: McInerny TK, Adam HM, Campbell DE, DeWitt TG, Foy JM, Kamat
DM, eds. American Academy of Pediatrics Textbook of Pediatric Care. Itasca, IL:
American Academy of Pediatrics; 2017; chap 245.
https://pediatriccare.solutions.aap.org/chapter.aspx?
sectionid=124994819&bookid=1626.
• Shah KN, Honig PJ, Yan AC. “Urticaria multiforme”: a case series and review of acute
annular urticarial hypersensitivity syndromes in children. Pediatrics. 2007;119(5):e1177-
e1183. doi:10.1542/peds.2006-1553.
• Sokumbi O, Wetter DA. Clinical features, diagnosis, and treatment of erythema
multiforme: a review for the practicing dermatologist. Int J Dermatol. 2012;51(8):889-
902. doi:10.1111/j.1365-4632.2011.05348.x.
• Trayes KP, Love G, Studdiford JS. Erythema multiforme: recognition and management.
Am Fam Physician.2019;100(2):82. https://pubmed.ncbi.nlm.nih.gov/31305041/.

American Academy of Pediatrics 491


PREP® Self-Assessment PREPSA 2023

Question 145
A 9-year-old girl is seen in the office for concerns of inattention and declining school
performance for the past 2 months. The girl has always been an excellent student, and her
teachers have never expressed behavioral concerns. The girl’s mother describes her as “very
independent” and is puzzled by these changes.

The girl’s mother is a single parent and works nights at a warehouse. She reports that a few
months ago there was a shooting in their apartment complex, and the girl “might have seen some
things she shouldn’t have.” There have been other violent incidents at their building, which she
does not think her children witnessed, and their apartment was broken into last year when they
weren’t home. The girl now refuses to leave the apartment to go to the playground or to her
friend’s apartment in their complex, startles at loud noises, and frequently asks her mother if
something bad is going to happen to them. She has nightmares and “wakes up with any little
noise.” The girl’s 3-year-old sister has begun to display aggressive behaviors at daycare and
frequent tantrums at home, and is now sleeping in her mother’s bed.

Concerns about posttraumatic stress disorder are discussed for both girls, therapy resources for
both are provided, and plans are made to see them back for follow-up in 3 months.

Of the following, the BEST additional management step for this family is to
A. advise the girls’ mother to avoid talking about the violent events to not upset the girls
B. assess how the girls’ mother is coping and provide applicable resources
C. prescribe bedtime clonidine to help the older girl sleep
D. recommend the girls’ mother take a short leave of absence to be home with her children

American Academy of Pediatrics 492


PREP® Self-Assessment PREPSA 2023

Correct Answers: B
The girl in the vignette and her younger sister are exhibiting signs of posttraumatic stress
disorder (PTSD). In addition to trauma-focused cognitive behavioral therapy for both girls, the
best management step for the family is to assess how the girls’ mother is coping and provide
applicable resources. A protective factor for children who have experienced trauma is having a
strong connection to an attentive caregiver. Providing the mother with appropriate resources for
her own treatment will help her to better attend to her children’s emotional and behavioral needs
and fully participate in therapy with them.

The diagnostic criteria for PTSD in children require symptoms exhibited within all 4 of the
following categories:
1. Intrusive thoughts or emotions
2. Avoidance of stimuli
3. Negative thoughts or emotions
4. Hypervigilance or sleep disturbance

Symptoms of PTSD present differently at different ages. Infants may exhibit sleep and feeding
disturbances, regression in milestone attainment, and impairments in attachment. Toddlers and
preschool-aged children may experience developmental regression, irritability, tantrums, and
changes in feeding, sleep, and toileting. As seen with the girl in the vignette, school-aged
children may have academic difficulties, inattention, and increased clinginess or need for parents
or caregivers. Pediatricians should assess for trauma by asking screening questions during
routine visits. When a child has behavioral changes or loss of developmental milestones, trauma
should be assessed for as part of the differential diagnosis. Trauma may include experiencing
abuse or neglect, domestic violence, community violence, severe accidents, murder, suicide, and
natural or man-made disasters. Children who experience trauma are at increased risk for chronic
health problems as adults including heart disease, diabetes, cancer, and lung disease. They also
experience higher rates of behavioral and mental health disorders (eg, anxiety, depression,
substance use).

Trauma-focused cognitive behavioral therapy is recommended to address PTSD in preschool-


and school-aged children. This therapy includes having the child discuss the event and helping
them to develop a trauma narrative to learn to manage anxiety and utilize relaxation techniques.
Having the mother take a leave of absence would likely lead to financial stress and would not be
considered a long-term solution. Use of medications such as clonidine is not recommended in the
treatment of PTSD unless symptoms are so severe that the child is unable to participate in
therapy or there are additional comorbidities. Avoiding discussion of the trauma is not
recommended; discussion is an important part of therapy.

American Academy of Pediatrics 493


PREP® Self-Assessment PREPSA 2023

PREP Pearls
• Posttraumatic stress disorder can vary in presentation depending on a child’s age and
developmental level.
• A strong parent and caregiver relationship is a protective factor for children who have
experienced trauma.
• Providing a parent or caregiver with appropriate resources for their own mental health
will help them to better attend to their children’s emotional and behavioral needs.

ABP Content Specifications(s)


• Recognize the clinical findings associated with post-traumatic stress disorder in patients
of various ages, and manage appropriately

Suggested Readings
• Cohen JA, Kolko DJ. Post-traumatic stress disorder. In: McInerny TK, Adam HM,
Campbell DE, DeWitt TG, Foy JM, Kamat DM, eds. American Academy of Pediatrics
Textbook of Pediatric Care. American Academy of Pediatrics; 2021:chap 317. Accessed
September 1, 2021. Pediatric Care Online.
• Horn SR, Feder A. Understanding resilience and preventing and treating PTSD. Harv
Rev Psychiatry. 2018;26(3):158-174. doi:10.1097/HRP.0000000000000194.
• Kelly P. Posttraumatic stress disorder. Pediatr Rev. 2012;33(8):382-383.
doi:10.1542/pir.33-8-382.
• Smith P, Dalgleish T, Meiser-Stedman R. Practitioner Review: Posttraumatic stress
disorder and its treatment in children and adolescents. J Child Psychol Psychiatry.
2019;60(5):500-515. doi:10.1111/jcpp.12983.
• Wilson HW, Joshi SV. Recognizing and referring children with posttraumatic stress
disorder: guidelines for pediatric providers. Pediatr Rev. 2018;39(2):68-77.
doi:10.1542/pir.2017-0036.

American Academy of Pediatrics 494


PREP® Self-Assessment PREPSA 2023

Question 146
A 6-month-old infant born at 29 weeks’ gestation is seen in the neonatal intensive care unit for
management of feeding difficulties. She has had a tracheoplasty for complete tracheal rings and
has chronic lung disease. The infant reached full enteral feedings 5 weeks ago and has been
slowly improving her oral intake. She currently feeds every 3 hours, taking one-third of her
caloric requirements by bottle and the remaining administered in bolus feedings through a
nasogastric tube. She has not made any progress with oral feedings in the past month despite
intensive intervention by the feeding team. The team anticipates that it will take several months
before the infant is able to take full caloric requirements by mouth. The medical team and
infant’s family, through shared decision-making, discuss plans for discharge and ongoing enteral
nutrition support. The infant’s family would like to continue with nasogastric feedings and work
on improving oral intake at home before considering placement of a gastrostomy tube. The
medical team, however, recommends gastrostomy tube placement.

Of the following, the factor MOST supportive of the medical team’s recommendation is the
A. anticipated duration of nasogastric feedings
B. decreased risk for emergency department visits
C. history of prematurity
D. history of previous tracheoplasty

American Academy of Pediatrics 495


PREP® Self-Assessment PREPSA 2023

Correct Answer: A
The infant in the vignette currently requires enteral nutrition support by nasogastric (NG) tube
due to inadequate oral intake despite appropriate feeding therapy. Because the medical team
anticipates that this level of support will be required for several months, gastrostomy tube (G-
tube) placement is indicated. Gastrostomy tube placement is generally recommended when
enteral supplementation will be needed for a long period, with recommendations varying from at
least 6 weeks to 3 months.

The infant’s history of prematurity and previous tracheoplasty do not support a preference for
NG or G-tube feeding. Nasogastric feeding is associated with fewer tube-related emergency
department visits compared with G-tube feeding, which would be a factor supporting the
family’s preference.

Enteral nutrition support can be delivered through various methods: orogastric (OG), NG,
nasojejunal (NJ), G-tube, gastrojejunostomy (GJ), and jejunostomy (Item C146A). The optimal
mode of nutrition support depends on individual patient factors. Orogastric feeding may be used
for preterm infants in the hospital to avoid nasal obstruction; they are generally not used at home
because of the increased incidence of tube displacement and potential risk for aspiration.
Nasogastric feeding is used in infants and children with normal intestinal anatomy and a need for
short-term enteral support. Long-term NG feeding can be used in some situations. For example,
individuals with Crohn disease may use exclusive enteral nutrition as treatment by placing an
NG tube nightly, administering overnight enteral nutrition, and removing the NG tube in the
morning. Gastrostomy tube placement is generally recommended for infants and children
requiring longer-term supplementation or when NG feeding is considered unsafe (eg, limited
ability to care for or replace the NG tube as necessary). Gastrostomy tube types include
percutaneous endoscopic gastrostomy tubes (PEG) or skin level gastrostomy tubes (commonly
referred to as “buttons”) (Item C146B and Item C146). Initial placement of a G-tube
(endoscopic, surgical, or radiologic) requires anesthesia; once a G-tube button is in place,
subsequent G-tube button replacements can be performed at home.

Gastric feedings (OG, NG, or G-tube) may be administered as bolus or continuous feedings.
Bolus feedings can be administered via a syringe; they are easier to administer. For children with
recurrent vomiting or difficulty tolerating feedings, continuous feedings at a slower rate may be
beneficial. It is not uncommon for children to receive feedings using both bolus feedings (during
the daytime) and continuous feedings (overnight). For infants and children requiring postpyloric
feedings (eg, severe aspiration, gastroparesis, or superior mesenteric artery syndrome), NJ, GJ, or
jejunostomy tube feedings are indicated; jejunal feedings must be administered continuously.
Nasojejunal tubes require fluoroscopic or endoscopic placement. Gastrojejunostomy tubes may
be placed by fluoroscopically advancing a dual port tube through the stomach into the jejunum.
Jejunostomy tubes are skin-level devices placed directly into the jejunum generally by surgeons
(endoscopic placement may be available).

American Academy of Pediatrics 496


PREP® Self-Assessment PREPSA 2023

Complications of enteral feeding vary based on mode of delivery (Item C146D).


Troubleshooting enteral tube feeding is described in Item C146E. Displacement of a G-tube, GJ
tube, or jejunostomy tube (inadvertent removal) needs to be addressed as soon as possible.
Urgent replacement of the tube is required, as an empty tract may stenose quickly. If the tube is
broken or will not advance through the tract, other tubes may be used as a temporary measure
while a new tube is obtained. A Foley catheter is preferred as it has a balloon to keep it in place.

American Academy of Pediatrics 497


PREP® Self-Assessment PREPSA 2023

American Academy of Pediatrics 498


PREP® Self-Assessment PREPSA 2023

PREP Pearls
• Enteral nutrition support may be provided by gastric (orogastric, nasogastric,
gastrostomy) and/or postpyloric (nasojejunal, gastrojejunal, jejunostomy) tubes.
• Gastric feedings may be administered via bolus or continuous feedings; postpyloric
feedings must be administered as continuous feedings.
• A dislodged gastrostomy, gastrojejunostomy, or jejunostomy tube must be replaced
promptly as the established tract can stenose quickly.

ABP Content Specifications(s)


• Recognize the complications associated with tube feeding
• Know when to prescribe intermittent (bolus) feeding rather than continuous tube feeding

American Academy of Pediatrics 499


PREP® Self-Assessment PREPSA 2023

Suggested Readings
• Homan M, Hauser B, Romano C, et al. Percutaneous endoscopic gastrostomy in children:
an update to the ESPGHAN position paper. J Pediatr Gastroenterol Nutr. 2021;73(3):415-
426. doi:10.1097/MPG.0000000000003207.
• Khalil ST, Uhing MR, Duesing L, Visotcky A, Tarima S, Ngheim-Rao T. Outcomes of
infants with home tube feeding: comparing nasogastric versus gastrostomy tubes. J
Parenter Enteral Nutr. 2017;41(8):1380-1385. doi:10.1177/0148607116670621.
• Scharbach K. Shared decision making around home technologies. In: McInerny TK,
Adam HM, Campbell DE, DeWitt TG, Foy JM, Kamat DM, eds. American Academy of
Pediatrics Textbook of Pediatric Care. American Academy of Pediatrics; 2021:chap 116.
Accessed September 1, 2022. Pediatric Care Online.
• Singhal S, Baker SS, Bojczuk GA, Baker RD. Tube feeding in children. Pediatr Rev.
2017;38(1):23-34. doi:10.1542/pir.2016-0096.
• Volpe A, Malakounides G. Feeding tubes in children. Curr Opin Pediatr. 2018;30(5):665-
670. doi:10.1097/MOP.0000000000000666.

American Academy of Pediatrics 500


PREP® Self-Assessment PREPSA 2023

Question 147
A 4-year-old boy (Item Q147) is seen for a health supervision visit. His mother reports reduced
sweating compared to his peers. On physical examination, the boy has sparse scalp hair, dry skin,
and a total of 2 teeth which are conical in shape. The family history is significant for oligodontia
in the mother and the similarity of the boy’s physical features to those of his maternal
grandfather.

Item Q147: Boy depicted in the vignette.


Courtesy of D. Krowchuk

Of the following, the MOST appropriate recommendation for this child at this visit is
A. avoidance of overheating
B. placement of dentures by age 20 years
C. use of methotrexate to treat alopecia
D. use of prophylactic antibiotics to prevent recurrent sinus infections

American Academy of Pediatrics 501


PREP® Self-Assessment PREPSA 2023

Correct Answer: A
The boy in the vignette has a type of ectodermal dysplasia (ED). Children with ED are at high
risk for hyperthermia and hyperpyrexia because of their decreased ability to sweat. Anticipatory
guidance regarding avoidance of overheating is critically important.

Children with ED have abnormalities in at least 2 ectodermal structures—hair, teeth, nails, or


sweat glands. There are over 200 types of ED. X-linked hypohidrotic ED (HED), the most
common type and the one that is consistent with the boy’s clinical presentation, is characterized
by the following features:
• Teeth – hypodontia (congenitally missing teeth): Teeth that erupt are abnormally formed,
with the front teeth often conical in shape (Item C147).
• Hair – hypotrichosis (sparse body and scalp hair): Hair has reduced pigmentation, is
brittle, and exhibits slow growth.
• Sweat glands – hypohidrosis (decreased ability to sweat; sweating is not totally absent
but is deficient).
• Other systems involved includes:
o Eyes: Abnormal meibomian glands can lead to dry eyes and resultant corneal
abrasion. Periorbital hyperpigmentation is commonly observed.
o Nose: Secretions can get solidified in the nasal and aural passages, with recurrent
sinus infections.
o Mouth: Decreased oral secretions can lead to dry mouth.
o Skin: Dry, fragile-appearing skin with lack of dermal ridges.
o Respiratory: Raspy voice, abnormal bronchial glands that can lead to asthma and
recurrent pneumonia.

X-linked HED is diagnosed based on clinical features and confirmed on identification of a


pathogenic variant in the EDA gene. Females typically exhibit mild manifestation(s) of the
disorder.

Management of HED includes the following:


• Teeth: Dental treatment is initiated at an early age. Dental implants in the anterior
mandibular arch have been successful in children ages 7 years and older. Dentures should
be placed at least by the time a child starts school. Children as young as 2.5 years have
achieved success with the use of dentures. Benefits of early denture use include the

American Academy of Pediatrics 502


PREP® Self-Assessment PREPSA 2023

ability to eat a wide variety of food, improvement in speech, and a positive impact on the
child’s psychological and emotional well-being.
• Hair: Care is directed toward preventing damage to the hair shaft with the use of a gentle
shampoo and conditioner. Minoxidil was used to stimulate hair growth in a single case
report in a patient with HED but has not been widely studied. Some patients choose to
wear a wig.
• Sweat glands: It is critical to avoid overheating by ensuring adequate hydration with cool
liquids, use of airconditioning when inside, a cool bath when hot, and resting in a shady
cool area when outside in the heat.
• Other system-related management is as follows:
o Eyes: Artificial lubricant drops to avoid dry eyes and resultant corneal abrasion.
o Nose: Nasal and aural secretions that have solidified must be removed using a
suction device or forceps. Adequate humidification of nasal mucosa can be
provided with saline drops.
o Mouth: An artificial saliva preparation can be used to maintain oral lubrication.
o Skin: Gentle cleansing soap and mild emollient is used.
o Respiratory: Humidification of environmental air is recommended to alleviate
symptoms of a raspy voice.

Methotrexate has not been studied as a treatment modality for alopecia in HED. There is no
evidence to suggest that prophylactic antibiotics would prevent recurrent sinus infection in
children with HED.

PREP Pearls
• Ectodermal dysplasias are a group of disorders that involve abnormalities in at least 2
ectodermal structures—hair, teeth, nails, or sweat glands.
• The main clinical features of hypohidrotic ectodermal dysplasia include hypotrichosis,
hypodontia, and hypohidrosis.
• A critical feature of the management of hypohidrosis is avoidance of overheating.

ABP Content Specifications(s)


• Recognize the clinical findings associated with ectodermal dysplasia

American Academy of Pediatrics 503


PREP® Self-Assessment PREPSA 2023

Suggested Readings
• National Foundation for Ectodermal Dysplasias. What are ectodermal dysplasias?
Accessed September 1, 2022. https://www.nfed.org/.
• Reyes-Reali J, Mendoza-Ramos MI, Garrido-Guerrero E, Méndez-Catalá CF, Méndez-
Cruz AR, Pozo-Molina G. Hypohidrotic ectodermal dysplasia: clinical and molecular
review. Int J Dermatol. 2018;57(8):965-972. doi:10.1111/ijd.14048.
• Wohlfart S, Meiller R, Hammersen J, et al. Natural history of X-linked hypohidrotic
ectodermal dysplasia: a 5-year followup study. Orphanet J Rare Dis. 2020;15:7.
doi:10.1186/s13023-019-1288-x.
• Wright JT, Fete M, Schneider H, et al. Ectodermal dysplasias: classification and
organization by phenotype, genotype, and molecular pathway. Am J Med Genet.
2019;179A:442-447. doi:10.1002/ajmg.a.61045.
• Wright JT, Grange DK, Fete M. Hypohidrotic ectodermal dysplasia. GeneReviews
[Internet]. University of Washington; 2021. Accessed September 1, 2022.
https://www.ncbi.nlm.nih.gov/books/NBK1112/.

American Academy of Pediatrics 504


PREP® Self-Assessment PREPSA 2023

Question 148
A 6-year-old girl is brought to the office for breathing problems. She has had repeated episodes
of wheezing which occurred only with viral respiratory infections. Over the past 2 months she
has been coughing at night 2 to 3 times weekly, frequently becomes short of breath while
running, and must stop and rest while playing with friends. She required a course of oral
corticosteroids for wheezing with a viral respiratory illness during the most recent winter season.
Her mother and a paternal aunt had asthma as children. The girl’s vital signs are normal. There is
no increased work of breathing or use of accessory muscles. Breath sounds are clear and equal to
auscultation with good aeration. The remainder of her physical examination findings are normal.

Of the following, the BEST treatment for this girl is a/an


A. combined inhaled steroid and formoterol inhaler used daily and as needed
B. medium-dose inhaled corticosteroid as needed
C. oral leukotriene modifier daily
D. short-acting bronchodilator inhaler as needed

American Academy of Pediatrics 505


PREP® Self-Assessment PREPSA 2023

Correct Answers: A
The girl in the vignette has mild to moderate persistent asthma, with symptoms occurring more
than twice per week but less than daily. The preferred management for persistent asthma in
children 5 years of age and older is daily and as-needed use of a combined inhaler with
corticosteroid and formoterol.

Combined inhalers containing an inhaled corticosteroid and formoterol (a long-acting β-agonist


[LABA]) are appropriate for what is known as the same maintenance and rescue therapy
(SMART) approach to asthma management. Formoterol combined with budesonide used with
the SMART approach has been well studied in the United States. This combination became the
standard of care with the publication of the 2020 National Institutes of Health (NIH) updated
asthma treatment guidelines. For children aged 5 years and older with persistent mild or
moderate asthma, using an inhaled corticosteroid (ICS) with the long-acting β-agonist (LABA)
formoterol as both maintenance and rescue therapy helps to minimize daily steroid exposure
while decreasing the likelihood that oral steroids will be needed with exacerbation. The LABA
salmeterol has a slower onset of action and is not appropriate for use as a rescue treatment. Item
C148A, Item C148B, and Item C148C outline current treatment recommendations for children
with asthma.

One of the tenets of the updated NIH asthma treatment recommendations embraces minimizing
cumulative inhaled steroid dosing. For some young children with intermittent asthma triggered
only by viral illnesses, using an ICS concurrent with the start of a respiratory tract infection
along with short-acting albuterol as needed during exacerbations is a viable approach (Item
C148A). As-needed use of a medium-dose ICS would not be appropriate for the girl in the
vignette given the frequency of her symptoms and her age.

While the most effective controller for even mild asthma is an ICS, there is a role for a
leukotriene modifier as primary treatment of mild persistent asthma in young children. Both the
NIH Asthma Guidelines and the Global Initiative for Asthma (GINA) recommendations include
montelukast as an alternative first-line controller treatment at step 2 of the treatment paradigm
for both the 0- to 4-year-old and 5- to 11-year-old age groups and as add on to daily inhaled
corticosteroids at steps 3 and 4 (Item C148A and Item C148B). However, it is important to note
that the use of montelukast was not specifically considered in the most recent update; there was
no change in recommendation regarding its use from the 2007 NIH Asthma Guidelines. Pediatric
health care professionals should be aware of the boxed warning added to the prescribing
information for montelukast by the US Food and Drug Administration in March 2020 regarding
the risk for neuropsychiatric adverse effects.

Short-acting bronchodilators (SABA) alone are not appropriate primary therapy for persistent
asthma. They are appropriate at any age as sole therapy only for intermittent asthma (step 1 of
the treatment tables) and as an addon to controller therapy for treatment of acute exacerbations.
Long-acting β-agonists are appropriate for asthma management only when combined with an
ICS for controller therapy at step 3 in children aged 0 to 4 years or for SMART at step 3 in older
children and adolescents (Item C148A, Item C148B, and Item C148C).
American Academy of Pediatrics 506
PREP® Self-Assessment PREPSA 2023

Both SABA and LABA have been associated with adverse events. Tachycardia and tremors are
common with β2agonists, particularly those that are less β2-receptor–selective than albuterol and
levalbuterol. However, there is wide variability in severity among individuals. Chronic or
excessive use may magnify these effects. Arrhythmias and severe cardiac adverse events are
more likely to be associated with the less β2-selective agents (eg, epinephrine or isoproterenol).
Long-acting β-agonists are not appropriate stand-alone therapy for asthma, but they should be
used only in conjunction with an ICS. In early marketing studies, the use of salmeterol (a LABA)
as the sole agent for the treatment of asthma was associated with an increased risk of sudden
death from asthma. The adverse outcomes were felt to be due to the use of salmeterol as a rescue
medication without concurrent antiinflammatory treatment, and failure to seek additional therapy
for exacerbations not responding to the LABA.

American Academy of Pediatrics 507


PREP® Self-Assessment PREPSA 2023

PREP Pearls
• The most effective treatment for asthma of all severities is an inhaled corticosteroid.
• Leukotriene modifiers may be used as alternative first-line therapy in young children with
mild persistent asthma. A US Food and Drug Administration boxed warning was added
to the prescribing information for montelukast regarding the risk for neuropsychiatric
adverse effects.
• Combined inhaled corticosteroids with formoterol are appropriate for the same
maintenance and rescue therapy in children and adolescents aged 5 years and older.
• Long-acting β-agonists should be used to treat asthma only in conjunction with an
inhaled corticosteroid.

MOCA-Peds Objective
• Manage a child with an acute asthma exacerbation.

ABP Content Specifications(s)


• Understand the role of leukotriene antagonists in the management of asthma
• Understand the pharmacokinetics of short- and long-acting inhaled beta-adrenergic
agonists and the risks associated with their excessive use

American Academy of Pediatrics 508


PREP® Self-Assessment PREPSA 2023

Suggested Readings
• Dinakar C. Asthma. In: McInerny TK, Adam HM, Campbell DE, DeWitt TG, Foy JM,
Kamat DM, eds. American Academy of Pediatrics Textbook of Pediatric Care. American
Academy of Pediatrics; 2021:chap 218. Accessed September 1, 2022. Pediatric Care
Online.
• Expert Panel Working Group of the National Heart, Lung, and Blood Institute (NHLBI)
administered and coordinated National Asthma Education and Prevention Program
Coordinating Committee (NAEPPCC); Cloutier MM, Baptist AP, Blake KV, et al. 2020
Focused Updates to the Asthma Management Guidelines: a Report from the National
Asthma Education and Prevention Program Coordinating Committee Expert Panel
Working Group. J Allergy Clin Immunol. 2020;146(6):1217-1270.
doi:10.1016/j.jaci.2020.10.003.
• Fitzpatrick AM, Jackson DJ, Mouger DT, et al; NIH/NHLBI AsthmaNet. Individualized
therapy for persistent asthma in young children. J Allergy Clin Immunol.
2016;138(6):1608-1618.e12. doi:10.1016/j.jaci.2016.09.028.
• Global strategy for asthma management and prevention. Global Initiative for Asthma.
2021. Accessed September 1, 2022. https://ginasthma.org/.
• Patel SJ, Teach SJ. Asthma. Pediatr Rev. 2019;40:549-567. doi:10.1542/pir.2018-0282 .

American Academy of Pediatrics 509


PREP® Self-Assessment PREPSA 2023

Question 149
A 16-year-old adolescent boy is seen for a health supervision visit. He has been healthy and is
developing normally. He is a good student and just completed a driver’s education course. Safe
driving guidelines are discussed with the boy and his mother.

Of the following, the BEST advice to provide on this topic is to recommend


A. at least 20 hours of supervised driving
B. cell phone use only when the car is stopped
C. limiting unsupervised driving to between 8 AM and 6 PM
D. no adolescent passengers for the first 6 months

American Academy of Pediatrics 510


PREP® Self-Assessment PREPSA 2023

Correct Answer: D
Of the response choices, the best advice to recommend is that the boy have no adolescent
passengers ride with him for the first 6 months.

Motor vehicle accidents are a notable cause of childhood morbidity and mortality, and relevant
anticipatory guidance should be provided at all visits beginning in infancy. The proper use of an
age-appropriate approved car safety device (eg, car seat, booster seat) decreases the risk of death
or serious injury due to motor vehicle collisions. As children become adolescents and prepare to
drive independently, there are several guidelines that improve safety.

Factors that decrease the risk of motor vehicle collisions for adolescent drivers:
• At least 50 hours of supervised driving, including 10 hours of nighttime driving
• Learner’s permit at age 16 years or older (some states offer a license at a younger age)
• Cell phone use (for both calls and text messaging) only allowed when the car is parked
• Limiting unsupervised driving to the hours of 5 AM to 10 PM
• Continue supervised driving under new conditions and in new areas, even after they
obtain an unrestricted license
• No driving after using any substances, including alcohol or marijuana
• 6 months of supervised driving before obtaining an unrestricted license
• No adolescent passengers for 6 months, then only 1 for the following 6 months

Data from the Insurance Institute for Highway Safety demonstrated that vehicular accidents
involving 16- to 17year-old drivers occurred more often when there were 3 or more passengers
in the car.

In addition to discussing driver safety with adolescents and car seat safety recommendations with
parents of young children, it is important to counsel families regarding children’s safety around
parked cars. Playing around or in a car unsupervised can be very dangerous for young children. It
is estimated that 35 to 40 people, with the majority being children, die each year from becoming
overheated in an enclosed vehicle. Five people die each year from being strangled in an
electronic window. Each year, thousands of children are injured from being driven over when a
car is moving forward. Children are difficult to be seen by a driver backing up a car or when
crouched down or sitting in front of a car; therefore, they are at risk of serious injury or death
when allowed to play around a car.

PREP Pearls
• Children should be supervised at all times around motor vehicles, including when the car
is turned off.
• Adolescents should have limits on their driver’s license or driving privileges, such as not
driving at night and initially not being permitted adolescent passengers.
• Use of cell phones should not be allowed while operating a motor vehicle unless the
vehicle is parked.

American Academy of Pediatrics 511


PREP® Self-Assessment PREPSA 2023

ABP Content Specifications(s)


• Recognize the major causes of automotive fatalities among young drivers (eg, drunk
driving)
• Understand the effects of non-crash automobile accidents in young children

Suggested Readings
• Durbin D, Hoffman B; Council on Injury, Violence, and Poison Prevention. Child
passenger safety. Pediatrics. 2018;142(5):e20182461. doi:10.1542/peds.2018-2461.
• Healthchildren.org. Teen passengers: what parents need to know. Updated July 30, 2014.
Accessed September 1, 2022.
• https://www.healthychildren.org/English/ages-stages/teen/safety/Pages/Teen-Passengers-
What-Parents-Need-toKnow.aspx.
• Healthychildren.org. Graduated driver licensing laws: information for parents. Updated
October 29, 2013. Accessed September 1, 2022.
https://www.healthychildren.org/English/ages-stages/teen/safety/Pages/Graduated-
DriverLicensing-Laws-Information-for-Parents.aspx.
• Kidsandcars.org. Accessed September 1, 2022. https://www.kidsandcars.org/.
• Mokdad A, Wolf L, Pandya S, Ryan M, Qureshi F. Road traffic accidents and disparities
in child mortality. Pediatrics. 2020;146(5):e20193009. doi:10.1542/peds.2019-3009.

American Academy of Pediatrics 512


PREP® Self-Assessment PREPSA 2023

Question 150
A 1-year-old girl with moderate atopic dermatitis is seen for a health supervision visit. Her father
has been using a low-potency topical steroid sporadically to manage her atopic dermatitis with
minimal effect. He does not want to increase the frequency or strength of the topical steroid
because he has concerns regarding side effects. He would like to explore dietary modifications to
manage her atopic dermatitis. The girl eats a wide variety of foods with no obvious correlation
between flares of her skin condition and specific foods. She has been otherwise healthy with
normal growth and development. Her physical examination findings are normal except for
diffuse patches of moderate atopic dermatitis.

Of the following, the BEST management strategy for this girl’s condition is to
A. begin vitamin D supplementation
B. optimize medical management
C. order food allergy testing
D. recommend a food elimination diet

American Academy of Pediatrics 513


PREP® Self-Assessment PREPSA 2023

Correct Answer: B
The best management strategy for the girl in the vignette is optimization of medical treatment of
her atopic dermatitis. Without an obvious correlation between flares of her atopic dermatitis and
specific foods, a food elimination diet is not indicated, and food allergy testing in this situation
has a high false-positive rate. Current evidence does not support the effectiveness of vitamin
supplementation in the management of atopic dermatitis. It will be important to share this
information with the girl’s father, and respond respectfully to his questions and concerns to
engage him willingly in effective skin care management.

Atopic dermatitis is characterized by itchy, dry, red, and scaly skin patches distributed in specific
patterns depending on age. Item C150A lists features used in the diagnosis of atopic dermatitis.
Atopic dermatitis is one part of the “atopic march,” an early phase in the sequence of atopic
conditions leading toward allergic rhinitis, asthma, and food allergies. Children with atopic
dermatitis have an increased risk of developing skin infections from Staphylococcus aureus and
group A Streptococcus due to their compromised skin barrier. Children with atopic dermatitis
may also experience an increased risk and more severe and persistent episodes of concomitant
viral skin infections such as molluscum contagiosum, coxsackie virus (Item C150B), and herpes
simplex virus (Item C150C).

Complications of atopic dermatitis extend beyond the skin. The itching associated with atopic
dermatitis may lead to poor sleep, which may in turn negatively affect school performance and
personal relationships. Some studies have correlated atopic dermatitis with anxiety, depression,
and attention-deficit/hyperactivity disorder.

Management of atopic dermatitis must include family education that emphasizes the chronic
nature of the condition and the importance of maintenance therapy. Item C150D outlines a
general approach to the management of mild and moderate to severe atopic dermatitis.
Moisturizers should be generously and frequently applied throughout the day. Topical
corticosteroids are the mainstay of treatment. Item C150E lists the potency of various topical
corticosteroids suitable for mild, moderate, or severe disease. Wet wrap therapy may improve the
efficacy of emollients and topical therapies. If topical corticosteroids are unsuccessful, topical
calcineurin inhibitors may be used as second-line treatment.

Management of complications of atopic dermatitis should target the inciting agent. Eczema
herpeticum, a disseminated form of herpes simplex 1 skin infection occurring in individuals with
atopic dermatitis, requires treatment with acyclovir and, if there is involvement near the eye,
consultation with an ophthalmologist. Bacterial superinfection of atopic dermatitis may be
decreased with bleach baths (Item C150F).

Children with refractory atopic dermatitis should be referred to a pediatric dermatologist who
may consider phototherapy or systemic therapies.

American Academy of Pediatrics 514


PREP® Self-Assessment PREPSA 2023

American Academy of Pediatrics 515


PREP® Self-Assessment PREPSA 2023

PREP Pearls
• First-line treatment of atopic dermatitis is to optimize medical management before
considering dietary modifications.
• Management of atopic dermatitis must include family education that emphasizes the
chronic nature of the condition and the importance of maintenance therapy.
• Complications of atopic dermatitis include atopic, infectious, and behavioral conditions.

ABP Content Specifications(s)


• Plan the appropriate management of atopic dermatitis
American Academy of Pediatrics 516
PREP® Self-Assessment PREPSA 2023

• Recognize complications associated with atopic dermatitis


• Recognize the clinical findings associated with atopic dermatitis

Suggested Readings
• Eichenfield LF, Wynnis LT, Berger TG, et al. Guidelines of care for the management of
atopic dermatitis: section 2. Management and treatment of atopic dermatitis with topical
therapies. J Am Acad Dermatol. 2014;71(1):116-132. doi:10.1016/j.jaad.2014.03.023.
• Eichenfield LF, Wynnis LT, Chamlin SL, et al. Guidelines of care for the management of
atopic dermatitis: section 1. Diagnosis and assessment of atopic dermatitis. J Am Acad
Dermatol. 2014;70(2):338-351. doi:10.1016/j.jaad.2013.10.010.
• Nield LS, Keri JE. Atopic dermatitis. In: McInerny TK, Adam HM, Campbell DE,
DeWitt TG, Foy JM, Kamat DM, eds. American Academy of Pediatrics Textbook of
Pediatric Care. American Academy of Pediatrics; 2021:chap 219. Accessed September 1,
2022. Pediatric Care Online .
• Waldman AR, Ahluwalia J, Udkoff J, Borok JF, Eichenfield LF. Atopic dermatitis.
Pediatr Rev. 2018;39(4)180-193. doi:10.1542/pir.2016-0169.
• Yang EJ, Sekhon S, Sanchez IM, Beck KM, Bhutani T. Recent developments in atopic
dermatitis. Pediatrics. 2018;142(4):e20181102. doi:10.1542/peds.2018-1102.

American Academy of Pediatrics 517


PREP® Self-Assessment PREPSA 2023

Question 151
A 6-year-old boy is seen for a routine health supervision visit. He has a history of an
atrioventricular septal defect repair that resulted in stenosis of his left atrioventricular valve. The
stenotic valve was replaced with a mechanical valve. He is maintained on anticoagulation, and
his international normalized ratio has been in the appropriate treatment range. He has no known
drug allergies and takes oral medications without difficulty. The boy’s physical examination
findings are normal except for mechanical cardiac sounds. He is scheduled to see his dentist next
week for a routine cleaning for which his mother requests a prescription for antibiotic
prophylaxis.

Of the following, the BEST medication to prescribe for this boy is


A. amoxicillin
B. clindamycin
C. nothing; none are indicated
D. trimethoprim-sulfamethoxazole

American Academy of Pediatrics 518


PREP® Self-Assessment PREPSA 2023

Correct Answers: A
The boy in the vignette has a mechanical heart valve, an indication for endocarditis prophylaxis
with antibiotics prior to routine dental cleaning. The regimen recommended by the American
Heart Association 2007 guideline is amoxicillin, orally, in a single dose 30 to 60 minutes prior to
the procedure. Amoxicillin is the drug of choice because it is well absorbed by the
gastrointestinal (GI) tract and has high and sustained serum levels. Item C151 lists alternative
antibiotic options for those with a penicillin allergy or inability to take medication orally. The
targeted organisms are viridans group streptococci. Clindamycin is an appropriate alternative for
those with a penicillin allergy, which the boy in the vignette does not have. Trimethoprim-
sulfamethoxazole is not indicated for endocarditis prophylaxis.

Not all children with congenital heart disease require endocarditis prophylaxis. Prophylaxis is
indicated for children with conditions that confer a high risk of an adverse outcome from
infective endocarditis. These conditions include:
• A prosthetic cardiac valve or prosthetic material used in cardiac valve repair
• A previous history of endocarditis
• Specific congenital heart disease (CHD)
o Unrepaired cyanotic cardiac lesions, including palliative shunts or conduits
o Completely repaired CHD (via surgical or interventional procedure) with
prosthetic material/device for the first 6 months after the procedure
o Repaired CHD with residual defects at or adjacent to the site of prosthetic
material which would present endothelialization
• A history of cardiac transplantation with valvulopathy

The most common procedures requiring prophylaxis are dental procedures that “involve
manipulation of gingival tissue or the periapical region of teeth or perforation of the oral
mucosa” (AHA guideline). Respiratory procedures that involve incision or biopsy of the mucosa
(eg, tonsillectomy, adenoidectomy) also require antibiotic prophylaxis (Item C151). Routine GI
and genitourinary (GU) procedures do not necessarily require prophylactic antibiotics. Children
who are undergoing GI, GU, skin, or musculoskeletal procedures for an infection may require
prophylaxis; the child’s pediatric cardiologist should be consulted regarding the need for
prophylaxis in such cases.

PREP Pearls
• Endocarditis prophylaxis is required for children with specific cardiac risk factors (eg,
prior history of endocarditis, residual defects, cyanotic disease, or prosthetic material that
is not endothelialized).
• Specific dental procedures and those causing respiratory mucosal injury are the most
common indications for endocarditis prophylaxis.
• Amoxicillin is the antibiotic of choice for endocarditis prophylaxis, administered as a
single dose 30 to 60 minutes prior to the procedure.

American Academy of Pediatrics 519


PREP® Self-Assessment PREPSA 2023

ABP Content Specifications(s)


• Plan appropriate prophylaxis for infective endocarditis

Suggested Readings
• Baltimore RS, Gewitz M, Baddour LM, et al; Kawasaki Disease Committee of the
Council on Cardiovascular Disease in the Young and the Council on Cardiovascular and
Stroke Nursing. Infective endocarditis in childhood: 2015 update: a scientific statement
from the American Heart Association. Circulation. 2015;132(15):1487-1515.
doi:10.1161/CIR.0000000000000298.
• Bragg L, Alvarez A. Endocarditis. Pediatr Rev. 2014;35(4):162-168.
doi:10.1542/pir.35.4.162.
• McCulloch MA, Gajarski RJ. Congenital and acquired heart disease. In: McInerny TK,
Adam HM, Campbell DE, DeWitt TG, Foy JM, Kamat DM, eds. American Academy of
Pediatrics Textbook of Pediatric Care. American Academy of Pediatrics; 2021:chap 234.
Accessed September 1, 2022. Pediatric Care Online.
• Wilson W, Taubert KA, Gewitz M, et al; American Heart Association Rheumatic Fever,
Endocarditis, and Kawasaki Disease Committee; American Heart Association Council on
Cardiovascular Disease in the Young; American Heart Association Council on Clinical
Cardiology; American Heart Association Council on Cardiovascular Surgery and
Anesthesia; Quality of Care and Outcomes Research Interdisciplinary Working Group.
Prevention of infective endocarditis: guidelines from the American Heart Association: a
guideline from the American Heart Association Rheumatic Fever, Endocarditis, and
Kawasaki Disease Committee, Council on Cardiovascular Disease in the Young, and the
Council on Clinical Cardiology, Council on Cardiovascular Surgery and Anesthesia, and
the Quality of Care and Outcomes Research Interdisciplinary Working Group.
Circulation. 2007;116(15):1736-1754. doi:10.1161/CIRCULATIONAHA.106.183095.

American Academy of Pediatrics 520


PREP® Self-Assessment PREPSA 2023

Question 152
A 16-year-old adolescent boy is seen for a health supervision visit. On routine adolescent
screening, he reports that he has been sexually active with 2 males for 9 months and does not use
condoms consistently. His last high-risk sexual contact was 4 weeks ago. He denies sick contacts
or injection drug use. His physical examination findings are normal. He is counseled on reducing
his risk for sexually transmitted infection (STI) and offered routine STI testing including HIV.

Of the following, the BEST HIV test to order for this adolescent is
A. DNA polymerase chain reaction
B. HIV-1/HIV-2 antibody immunoassay
C. HIV-1/HIV-2 antigen/antibody combination assay
D. RNA polymerase chain reaction

American Academy of Pediatrics 521


PREP® Self-Assessment PREPSA 2023

Correct Answers: C
The best HIV test to order for the adolescent in the vignette is the HIV-1/HIV-2 antigen/antibody
combination assay, a fourth-generation immunoassay recommended by the Centers for Disease
Control and Prevention (CDC) as the preferred initial screening test. This combination test
detects HIV viral protein (p24 antigen) as well as HIV IgM and IgG antibodies to HIV-1 and
HIV-2. The test is highly sensitive and specific in identifying individuals in the early stages of
HIV infection. The p24 antigen is detectable between weeks 2 and 3 of infection. A non-reactive
test result is reported as negative, and no additional testing is recommended unless early HIV
infection (acute retroviral syndrome) is suspected. A reactive test is followed up with a
supplemental HIV-1/HIV-2 antibody differentiation assay.

The third-generation HIV-1/HIV-2 antibody immunoassay can detect HIV IgM and/or IgG
antibodies around day 21 of infection; the test cannot differentiate between HIV-1 and HIV-2
antibodies. This test is a reasonable alternative for initial HIV testing if the fourth-generation
combination antigen/antibody immunoassay is unavailable. Rapid point-of-care antibody or
antigen/antibody tests (ie, finger-stick blood specimen or oral swab) that are sensitive, specific,
and easy to use are also available. However, the laboratory-based fourth-generation combination
immunoassay is more accurate for early HIV diagnosis. Positive results on rapid tests should
always be confirmed by laboratory testing.

Nucleic acid amplification test (NAAT) assays will detect HIV infection as early as 1 week after
acquisition, followed by the antigen-antibody combination test (2 weeks), and the IgM antibody
test (3 weeks). Various types of NAAT tests have been utilized to diagnose HIV infection such
as plasma HIV DNA polymerase chain reaction (PCR), HIV RNA qualitative PCR, and HIV
RNA quantitative PCR. The HIV DNA PCR test detects proviral DNA in infected mononuclear
cells in peripheral blood; it is the preferred diagnostic test for HIV in perinatal HIV-exposed
infants younger than 18 months (given the persistence of transplacental maternal antibodies in
the infant). The HIV RNA PCR tests detect the amount of HIV RNA in plasma. An HIV RNA
assay may be indicated in individuals with suspected acute HIV infection (acute retroviral
syndrome) and negative antibody test results. The quantitative HIV RNA PCR (viral load) test
measures the magnitude of viral replication in copies/mL and is helpful in predicting risk for
disease progression and prognosis, and monitoring response to therapy. The adolescent in the
vignette does not have symptoms and signs of acute retroviral syndrome (eg, fever,
lymphadenopathy, pharyngitis, rash, myalgia, arthralgia or gastrointestinal symptoms).
Therefore, the fourth-generation immunoassay, HIV-1/HIV-2 antigen/antibody combination
assay is the preferred initial HIV screening test.

In the United States (US), an estimated 1.2 million people are currently living with HIV/AIDS.
In 2018, there were 37,968 new HIV diagnoses in the US; of these individuals, 21% were
adolescents and youth aged 13 to 24 years. In 2018, among all people living with HIV in the US
an estimated 13.8% were unaware of their HIV diagnosis; in contrast, an estimated 44% of
adolescents and youth living with HIV were unaware of their infection. Most new HIV diagnoses
in youth are among gay and bisexual men.

American Academy of Pediatrics 522


PREP® Self-Assessment PREPSA 2023

The CDC recommends routine HIV screening at least once for individuals aged 13 to 64 years.
The US Preventive Services Task Force (USPSTF) recommends screening at least once for all
individuals 15 to 65 years old, and younger adolescents with risk factors. Routine screening for
HIV is recommended by the CDC and USPSTF for all pregnant individuals, including those
presenting in labor with unknown HIV status. The American Academy of Pediatrics
recommends offering HIV screening to all adolescents by 16 to 18 years of age, with more
frequent screening for those at continued risk of HIV acquisition. Annual HIV screening is
recommended for individuals at high risk for HIV acquisition (eg, individuals who inject drugs,
have multiple sexual partners, exchange sex for money or drugs, or have a new diagnosis of a
sexually transmitted infection [STI]).

Pediatricians play a vital role in promoting routine HIV screening among adolescents by
engaging in open conversations regarding sexuality and sexual practices while maintaining
confidentiality. Testing for HIV is voluntary. The CDC recommends that health care
professionals inform individuals that a HIV screening test will be obtained unless they refuse
(opt-out testing). A separate written consent for HIV testing is not needed, consistent with most
state laws. Adolescents <18 years may consent to testing and treatment for STIs in all 50 states,
and their confidentiality should be respected (a few states allow parental notification, but none
require it). A positive HIV test result should be communicated to the individual in person by the
health care professional. HIV is a nationally notifiable disease; health care professionals must
report a diagnosis of HIV to the local or state health department.

The adolescent in the vignette is at high risk for ongoing, significant risk of HIV acquisition and
therefore, is a good candidate for HIV preexposure prophylaxis (PrEP). PrEP involves the
provision of pre-exposure antiretroviral-based prophylaxis to HIV-seronegative individuals to
prevent viral acquisition from HIV-seropositive sexual partners. A daily oral PrEP regimen with
the fixed dose combination of tenofovir disoproxil fumarate 300 mg plus emtricitabine 200 mg is
effective and well tolerated in adolescents and adults. Adherence to the PrEP regimen is crucial
for efficacy.

PREP Pearls
• The United States Centers for Disease Control and Prevention recommends routine HIV
screening at least once for individuals aged 13 to 64 years.
• Annual HIV screening is recommended for individuals who are at high risk for HIV
acquisition.
• The preferred initial HIV screening test recommended by the Centers for Disease Control
and Prevention is the fourth-generation HIV-1/HIV-2 antigen-antibody combination
immunoassay performed on serum/plasma.

American Academy of Pediatrics 523


PREP® Self-Assessment PREPSA 2023

ABP Content Specifications(s)


• Recognize and apply ethical principles regarding the care of children and adolescents
with AIDS/HIV infection
• Identify the clinical features associated with AIDS in patients of various ages
• Plan appropriate screening for human immunodeficiency virus infection in at-risk infants
and children older than 18 months of age

Suggested Readings
• American Academy of Pediatrics. Human immunodeficiency virus infection. In:
Kimberlin DW, Barnett ED, Lynfield R, Sawyer MH, eds. Red Book: 2021–2024 Report
of the Committee on Infectious Diseases. 32nd ed. American Academy of Pediatrics;
2021. Accessed February 18, 2022. Red Book Online.
• Brady MT, Persaud D, Moss W. Human immunodeficiency virus infection and acquired
immunodeficiency syndrome. In: McInerny TK, Adam HM, Campbell DE, DeWitt TG,
Foy JM, Kamat DM, eds. American Academy of Pediatrics Textbook of Pediatric Care.
American Academy of Pediatrics; 2021:chap 268. Accessed February 18, 2022. Pediatric
Care Online.
• Branson BM, Handsfield HH, Lampe MA, Janssen RS, Taylor AW, Lyss SB, Clark JE;
Centers for Disease Control and
• Prevention (CDC). Revised recommendations for HIV testing of adults, adolescents, and
pregnant women in health-care
• settings. MMWR Recomm Rep. 2006;55(RR-14):1-17.
https://www.cdc.gov/mmwr/preview/mmwrhtml/rr5514a1.htm.
• Centers for Disease Control and Prevention; Association of Public Health Laboratories.
2018 Quick reference guide: Recommended laboratory HIV testing algorithm for serum
or plasma specimens. January 27, 2018. Accessed February 18, 2022.
https://www.cdc.gov/hiv/pdf/guidelines_testing_recommendedlabtestingalgorithm.pdf.
• Committee on Pediatric AIDS; Emmanuel PJ, Martinez J. Adolescents and HIV
infection: the pediatrician's role in promoting routine testing. Pediatrics.
2011;128(5):1023-1029. doi:10.1542/peds.2011-1761.
• Hurt CB, Nelson JAE, Hightow-Weidman LB, Miller WC. Selecting an HIV test: a
narrative review for clinicians and researchers. Sex Transm Dis. 2017;44(12):739-746.
doi:10.1097/OLQ.0000000000000719.

American Academy of Pediatrics 524


PREP® Self-Assessment PREPSA 2023

Question 153
A 7-year-old girl is seen for a health supervision visit. Results of urine dipstick testing,
performed on the previous 2 visits for a suspicion of urinary tract infection, were positive for
blood and negative for protein, and urine microscopy revealed 10 to 20 red blood cells per HPF.
She has no recent symptoms of dysuria, urgency, frequency, nocturnal enuresis, abdominal pain,
rashes, or joint pain. There is no family history of kidney disease or deafness. The girl’s weight
is at the 50th percentile and height is at the 65th percentile for age. Her blood pressure is 100/60
mm Hg. The remainder of the physical examination findings are normal.

Results of today’s urinalysis with microscopy are shown:


Test Result
Appearance Yellow
Specific gravity 1.015
Leukocyte esterase Negative
Nitrite Negative
Blood Positive
Protein Negative
Red blood cells per high-power field 20-50
White blood cells per high-power field <5

Of the following, the BEST next step in this girl’s evaluation is


A. 24-hour urine protein excretion
B. computed tomography scan of the abdomen
C. genetic testing
D. spot urine calcium:creatinine ratio

American Academy of Pediatrics 525


PREP® Self-Assessment PREPSA 2023

Correct Answer: D
The girl in the vignette has persistent asymptomatic microscopic hematuria. The most
appropriate next step in her evaluation is to order a spot urine calcium:creatinine ratio to evaluate
for hypercalciuria. Hypercalciuria, defined as a urine calcium:creatinine ratio of greater than 0.2
mg/mg or calcium excretion of greater than 4 mg/kg daily on 24hour urine collection, is a risk
factor for nephrolithiasis.

Microscopic hematuria is defined as the presence of more than 5 red blood cells per HPF on a
centrifuged urine specimen. Microscopic hematuria may be seen in 4.1% of school-aged children
on a single urine specimen but is present in only 1.1% of children on a repeat urinalysis.
Asymptomatic microscopic hematuria (ie, without associated signs and symptoms) is often
benign. Microscopic hematuria may be transiently seen in association with fever, exercise,
urinary tract infection, or trauma. Common causes of persistent microscopic hematuria (present
on urinalysis on ≥2 occasions) include hypercalciuria, thin basement membrane disease
(TBMN), IgA nephropathy, and sickle cell disease (Item C153).

A detailed history, including any family history of deafness or kidney disease, and physical
examination should be performed in children with persistent asymptomatic microscopic
hematuria. Initial laboratory evaluation should include spot urine protein:creatinine ratio, spot
urine calcium:creatinine ratio, urine culture, serum electrolyte levels, renal function tests (blood
urea nitrogen and serum creatinine measurement), and hemoglobin electrophoresis (if sickle cell
disease is suspected). Renal ultrasonography should be performed to evaluate for nephrolithiasis,
renal cyst, and congenital anomalies of the urinary tract. Children with microscopic hematuria
associated with edema, hypertension, or proteinuria should be promptly referred to a pediatric
nephrologist for evaluation for glomerulonephritis.

A 24-hour urine protein excretion measurement is not required for the child in the vignette
because her urinalysis results are negative for protein. Computed tomography scan would be
recommended for a child with persistent microscopic hematuria with associated abdominal pain
(unlike the girl in the vignette) and normal renal ultrasonography findings. The child in the

American Academy of Pediatrics 526


PREP® Self-Assessment PREPSA 2023

vignette has no family history of deafness or kidney disease, making Alport syndrome and
TBMN less likely diagnoses; thus, genetic testing is not indicated at this time.
Persistent asymptomatic microscopic hematuria has an overall good prognosis. Long-term
follow-up is required to monitor for onset of gross hematuria, proteinuria, or hypertension, which
would warrant further evaluation and management.

PREP Pearls
• Common causes of persistent microscopic hematuria include hypercalciuria, thin
basement membrane disease, IgA nephropathy, and sickle cell disease.
• Initial laboratory evaluation of persistent microscopic hematuria should include urine
protein:creatinine ratio, urine calcium:creatinine ratio, urine culture, serum electrolyte
levels, renal function tests (blood urea nitrogen and serum creatinine measurement), and
hemoglobin electrophoresis (if sickle cell disease is suspected).
• Children with microscopic hematuria associated with edema, hypertension, or proteinuria
should be promptly referred to a pediatric nephrologist.

ABP Content Specifications(s)


• Plan the appropriate clinical and laboratory evaluation of microscopic hematuria
• Formulate a differential diagnosis of persistent microscopic hematuria with and without
persistent proteinuria

Suggested Readings
• Kallash M, Rheault MN. Approach to persistent microscopic hematuria in children.
Kidney360. 2020;1:1014-1020. doi:10.34067/KID.0003222020.
• Reidy KJ, Rio MD. Hematuria. In: McInerny TK, Adam HM, Campbell DE, DeWitt TG,
Foy JM, Kamat DM, eds. American Academy of Pediatrics Textbook of Pediatric Care.
American Academy of Pediatrics; 2021:chap 160. Accessed September 1, 2022. Pediatric
Care Online.
• Viteri B, Reid-Adam J. Hematuria and proteinuria in children. Pediatr Rev.
2018;39(12):573-587. doi:10.1542/pir.2017-0300

American Academy of Pediatrics 527


PREP® Self-Assessment PREPSA 2023

Question 154
A 1-month-old infant is seen for a health supervision visit. Her parents are concerned that she
breathes faster while eating. She takes 1 to 2 ounces of standard infant formula every 2 to 3
hours and is sleeping well between feedings. The infant was born at term. There were no
complications during pregnancy or the delivery, and she was discharged from the hospital 24
hours after birth. She has had consistent growth along the 50th percentile for weight and length.
On physical examination, she has a heart rate of 150 beats/min, respiratory rate of 60
breaths/min, holosystolic murmur at the lower left sternal border, and liver edge palpable 3 cm
below the right costal margin. The remainder of her physical examination findings are normal.

Of the following, this infant’s MOST likely diagnosis is


A. aortic stenosis
B. atrial septal defect
C. pulmonary stenosis
D. ventricular septal defect

American Academy of Pediatrics 528


PREP® Self-Assessment PREPSA 2023

Correct Answers: D
The infant in the vignette has tachypnea while feeding, hepatomegaly, and a holosystolic
murmur at the lower left sternal border. These findings are consistent with a ventricular septal
defect (VSD). The flow across an atrial septal defect (ASD) itself does not create a murmur,
though the increased flow can result in a fixed split S2 and produce a murmur across the
pulmonary valve, which would be heard in mid-systole at the left upper sternal border.
Pulmonary valve stenosis would produce a systolic ejection murmur in the pulmonic location.
An aortic stenosis murmur would be heard in mid-systole, loudest at the right upper sternal
border.

As the pulmonary vascular resistance decreases through the first 1 to 2 months after birth, the
communication between the right and left ventricles from a VSD results in a left-to-right shunt.
Pulmonary blood flow increases as blood leaves the left ventricle, enters the right ventricle, and
then enters the lungs. Symptoms and signs of this increased pulmonary blood flow include
tachypnea, sweating with feedings, decreased oral intake, poor weight gain, tachycardia, and
hepatomegaly. On cardiac auscultation, a VSD murmur is heard throughout all of systole
(holosystolic) at the lower left sternal border. Its pitch is determined by the size of the defect,
with larger defects creating a lower pitch.

A VSD large enough to cause symptoms (typically moderate to large) requires closure.
Ventricular septal defects are usually closed surgically, though some are closed via a
transcatheter approach. Diuretics can provide symptom improvement while children are awaiting
surgical repair. If the VSD is left open, over time, the pulmonary vascular resistance increases
due to the excess pulmonary blood flow leading to irreversible pulmonary hypertension
(Eisenmenger syndrome).

PREP Pearls
• A ventricular septal defect creates a holosystolic murmur heard best at the left lower
sternal border.
• Symptoms and signs of a moderate to large ventricular septal defect include tachypnea,
sweating while feeding, tachycardia, and hepatomegaly.
• A moderate to large ventricular septal defect requires surgical closure and if left open can
lead to irreversible pulmonary hypertension (Eisenmenger syndrome).

ABP Content Specifications(s)


• Understand the natural history of ventricular septal defect

American Academy of Pediatrics 529


PREP® Self-Assessment PREPSA 2023

Suggested Readings
• Backer CL, Eltayeb O, Monge MC, Mazwi ML, Costello JC. Shunt lesions part I: patent
ductus arteriosus, atrial septal defect, ventricular septal defect, and atrioventricular septal
defect. Pediatr Crit Care Med. 2016;17(8 suppl 1):S302S309.
doi:10.1097/PCC.0000000000000786.
• Madriago E, Silberbach M. Heart failure in infants and children. Pediatr Rev.
2010;31(1):4-12. doi:10.1542/pir.31-1-4.
• McCulloch MA, Gajarski RJ. Congenital and acquired heart disease. In: McInerny TK,
Adam HM, Campbell DE, DeWitt TG, Foy JM, Kamat DM, eds. American Academy of
Pediatrics Textbook of Pediatric Care. American Academy of Pediatrics; 2021:chap 234.
Pediatric Care Online.
• McDaniel NL. Ventricular and atrial septal defect. Pediatr Rev. 2001;22(8):265-270.
doi:10.1542/pir.22-8-265.

American Academy of Pediatrics 530


PREP® Self-Assessment PREPSA 2023

Question 155
An 11-month-old girl is brought to the emergency department by her father for a fever of 38.8°C
at home. She was discharged from the hospital 2 months ago after spending several months there
recovering from heart surgery. Since discharge, she has had weekly fevers and was
rehospitalized for every episode. During these hospitalizations, her evaluation was unrevealing
and no fever was ever documented. The girl’s temperature taken by the triage nurse today is
normal. Except for expected postoperative changes, her physical examination findings are
unremarkable. The pediatrician entering the room noticed the father placing a heat pack on the
temperature probe. When confronted, he says the heat pack is for his own use. On further
questioning, he admits to being overwhelmed by caring for his daughter.

Of the following, the BEST next step in management for this girl is to
A. admit the child for evaluation by a multidisciplinary team
B. discharge the child with plans to follow up with her pediatrician
C. immediately separate the child from her father
D. provide an outpatient referral to a social worker

American Academy of Pediatrics 531


PREP® Self-Assessment PREPSA 2023

Correct Answer: A
The girl in the vignette has signs and symptoms that raise concerns for caregiver-fabricated
illness, including multiple presentations for medical care, inconsistent history, and reported
symptoms discordant with her physical examination findings. The best next management step is
admission to the hospital for evaluation by a multidisciplinary team and to ensure that she and
her father receive necessary support. Discharging her with plans to follow up with her
pediatrician is not likely to stop her father’s behavior as he will continue to feel overwhelmed.
Immediately separating the child from her father will most likely undermine trust and may lead
to worsening of any psychosocial stressors. Providing an outpatient referral to a social worker is
indicated, but should be done in conjunction with multidisciplinary evaluation and treatment.

Caregiver-fabricated illness, also known as medical child abuse, is defined by Roesler and Jenny
as “a child receiving unnecessary and harmful or potentially harmful medical care at the
instigation of a caretaker.” The incidence is inversely related to age. Offenders are more likely to
be female, but there is no consistent or generalizable psychological profile. Male and female
children can be victims, and siblings of victims are at high risk of being affected. Signs and
symptoms may involve any organ system and can lead to death. The psychological effects of
medical child abuse on the victim can be particularly damaging.

A high index of suspicion is required to identify medical child abuse. Evaluation includes
extensive review of the medical record and communication with the child’s primary and
specialist care teams as well as other members of the health care team. A child abuse specialist
can provide an expert, objective opinion, avoiding the risk of bias from established physician-
family relationships. Health care professionals are mandatory reporters, required to report
suspicions of any form of child abuse to child protective services as soon as possible.

PREP Pearls
• Children suspected to be victims of caregiver-fabricated illness (medical child abuse)
should be evaluated by a multidisciplinary team.
• Clinicians should have a high index of suspicion for medical child abuse and if suspected,
should request an evaluation by a multidisciplinary team.
• Health care professionals are mandatory reporters, required to report suspicions of any
form of child abuse to child protective services as soon as possible.

ABP Content Specifications(s)


• Understand the epidemiology of and the psychosocial and environmental risk factors for
caregiver-fabricated illness
• Recognize the clinical circumstances associated with caregiver-fabricated illness (eg,
caregiver seeking unnecessary, duplicative, and/or harmful medical interventions;
caregiver exaggerating, fabricating, or inducing a child's illness or symptoms)
• Plan the appropriate evaluation and management of suspected caregiver-fabricated illness

American Academy of Pediatrics 532


PREP® Self-Assessment PREPSA 2023

Suggested Readings
• Bryk M, Siegel PT. My mother caused my illness: the story of a survivor of Münchausen
by proxy syndrome. Pediatrics. 1997;100(1):1-7. doi:10.1542/peds.100.1.1.
• Flaherty EG, MacMillan HL; Committee on Child Abuse and Neglect. Caregiver-
fabricated illness in a child: a manifestation of child maltreatment. Pediatrics.
2013;132(3)590-597. doi:10.1542/peds.2013-2045.
• Jenny C, Metz JB. Medical child abuse and medical neglect. Pediatr Rev. 2020;41(2):49-
60. doi:10.1542/pir.2017-0302.
• Roesler TA, Jenny C. Medical Child Abuse: Beyond Munchausen Syndrome by Proxy.
American Academy of Pediatrics;

American Academy of Pediatrics 533


PREP® Self-Assessment PREPSA 2023

Question 156
A 16-year-old adolescent girl is seen during the winter for concerns about recurrent episodes of
blue hands and feet. These episodes have occurred at various intervals for several months and
seem to be related to cold exposure. There is no associated pain or dysfunction, and the events
resolve spontaneously in a warm environment. She takes no medications and has no history of
cardiovascular or respiratory disease. The adolescent is a ballet dancer and practices for 3 to 4
hours daily after school.

On physical examination, the adolescent is very thin, with little subcutaneous fat on her trunk.
Her weight is 45 kg, 8 kg lower than it was 6 months ago, and height is 165 cm. Her temperature
is 35.5°C, heart rate is 75 beats/min, respiratory rate is 12 breaths/min, and blood pressure is
100/60 mm Hg in the right arm, sitting; there is no change in blood pressure when supine, sitting,
or standing. Oxygen saturation by pulse oximetry is 99% in room air in each hand and foot. The
girl’s mucous membranes are pink, and there is no peripheral cyanosis. The remainder of her
physical examination findings are normal. The girl’s weight loss is discussed, and plans are made
to address this concern.

Of the following, the BEST next step in evaluating the concern for which the adolescent was
brought to the office is to
A. obtain a blood methemoglobin level
B. obtain a serum antinuclear antibody level
C. order a 72-hour Holter monitor study
D. request pictures of the next episode

American Academy of Pediatrics 534


PREP® Self-Assessment PREPSA 2023

Correct Answer: D
The girl in the vignette has acrocyanosis caused by thermodysregulation accompanying anorexia
nervosa. An 8-kg weight loss in 6 months and low body temperature in an otherwise healthy
athletic adolescent makes anorexia nervosa a likely diagnosis. Secondary acrocyanosis is a
common finding in anorexia nervosa, occurring in up to 80% of affected individuals.
Acrocyanosis is characteristically variable without clearly demarcated margins. Cold exposure is
the most common trigger. Asking the girl to provide pictures of her extremities at the time of an
episode can confirm the diagnosis. She can be reassured that the cyanosis itself is not threatening
to her health; the focus of her care can then shift to treatment of her anorexia nervosa.

Methemoglobinemia causes hemoglobin to be oxidized to the ferric (Fe3+) state, which cannot
carry oxygen to the peripheral tissues. This pathology results in persistent central cyanosis and
low peripheral oxygen saturation. Serum methemoglobin levels above 10% are associated with
cyanosis and levels above 20% are associated with dyspnea, tachycardia, and lightheadedness.

A Holter monitor study would identify an arrhythmia, but it is unlikely that repeated transient
arrhythmias would produce peripheral cyanosis without other symptoms of cardiovascular
dysfunction.

Measurement of antinuclear antibody (ANA) would be appropriate in the context of Raynaud


phenomenon because of the condition’s association with collagen vascular diseases. However,
the cyanosis of Raynaud phenomenon is usually clearly demarcated, associated with pain or
discomfort, and part of a biphasic or triphasic response. The peripheral vasculature undergoes
constriction (pallor) followed by vasodilation of the precapillary venules leading to pooling of
deoxygenated blood (cyanosis). A reactive hyperemia (plethora) of the affected area may occur
when the arterial and venous elements dilate after the cyanotic phase.

Most episodes of Raynaud phenomenon are idiopathic and primary. Secondary Raynaud is often
associated with collagen vascular diseases. These episodes may be more severe and frequent,
leading to skin breakdown. The nose, ears, and nipples can be affected in secondary Raynaud
phenomenon, and the episodes may be asymmetric, differentiating them from the primary form.
Pictures taken at the time of an episode can document the demarcated areas of skin and confirm
the diagnosis if it is not clear from the history. Evaluation for a potential underlying disorder is
appropriate, and serum ANA testing is indicated for individuals with new-onset Raynaud
phenomenon, despite the high incidence of false-positive values. Specific testing for collagen
vascular disease should be directed by the history and physical examination findings.
Capillaroscopy (not readily available in general office practice), may identify cutaneous capillary
enlargement and proliferation with or without telangiectasia. Additional associations with
secondary Raynaud phenomenon include:
• Heavy alcohol intake
• Smoking
• Caffeine ingestion
• Vasoactive medications
• Physical trauma
American Academy of Pediatrics 535
PREP® Self-Assessment PREPSA 2023

Cyanosis can be either central or peripheral. Other than methemoglobinemia from an exposure or
ingestion, central cyanosis that has no obvious source is very rare outside the newborn period.
Non-pulmonary acquired causes of central cyanosis in older children have an obvious association
and include:
• Infection/sepsis/shock
• Smoke inhalation or exposure to asphyxiating gas
• Cyanide poisoning
• Upper airway obstruction
• Impairment of the chest wall due to injury or neuromuscular dysfunction
• Cyanotic breath holding spells in toddlers that resolve after they lose consciousness

Peripheral cyanosis is characterized by normal systemic arterial oxygen saturation with increased
peripheral oxygen extraction and an increase in deoxygenated blood on the venous side of the
capillary bed. There may be perioral in addition to peripheral cyanosis, but mucous membranes
remain pink. Most peripheral cyanosis is considered primary acrocyanosis, and it is not
associated with underlying pathology. It has been proposed that individuals with recurrent
primary acrocyanosis have intermittent unexplained vasospasm of small cutaneous arteries and
compensatory dilation of capillaries and postcapillary venules.

In addition to anorexia nervosa, secondary acrocyanosis is associated with:


• Use of psychoactive medications
• Postural tachycardia syndrome (POTS)
• Infections
• Disseminated intravascular coagulation
• Spinal cord injury
• Ehler-Danlos syndrome
• Mitochondrial diseases

The likely mechanism of the acrocyanosis seen in POTS is abnormal arterial vasoconstriction,
decreased cutaneous nitric oxide production, and decreased overall blood flow to the arms and
legs. Affected individuals have gravitydependent cyanosis that is more pronounced than typically
seen with other causes of acrocyanosis.

A careful history and physical examination are important in identifying the cause of secondary
acrocyanosis. Because the events are transient and not usually present at the time of evaluation,
pictures documenting the cyanosis are very helpful in understanding the process and allowing for
reassurance as the primary intervention. No specific testing is needed to diagnose acrocyanosis
other than documentation of the process in that individual and evaluation for specific potential
triggers.

Mimics of cyanosis must be considered, especially when pictures are not available. These may
include:

American Academy of Pediatrics 536


PREP® Self-Assessment PREPSA 2023

• Pigmentary lesions
• Drug therapies with cutaneous color side effects
• Large or extensive tattoos
• Blue clothing dye
• Ingestion of blue or purple food simulating perioral cyanosis

PREP Pearls
• Primary acrocyanosis is common and benign. Up to 80% of patients with anorexia
nervosa may have acrocyanosis.
• Raynaud phenomenon is differentiated from acrocyanosis by sharp demarcation of
cyanotic skin, pain or discomfort, and a biphasic or triphasic color response (pallor,
cyanosis, plethora).
• Secondary Raynaud phenomenon can be associated with collagen vascular disease and
skin breakdown.

ABP Content Specifications(s)


• Plan the appropriate clinical and laboratory evaluation of cyanosis
• Identify the common extrapulmonary causes of cyanosis

Suggested Readings
• Fleck DE, Hoeltzel MF. Hand and foot color change: diagnosis and management. Pediatr
Rev. 2017;38(11):511-519. doi:10.1542/pir.2016-0234.
• Kaminecki I, Huang D. Methemoglobinemia. Pediatr Rev. 2021;42(3):164-166.
doi:10.1542/pir.2020-000943.
• Rome ES, Strandjord SE. Eating disorders. Pediatr Rev. 2016;37(8):323-336.
doi:10.1542/pir.2015-0180.
• Schneider M, Fisher M. Anorexia nervosa, bulimia nervosa, and other eating disorders.
In: McInerny TK, Adam HM, Campbell DE, DeWitt TG, Foy JM, Kamat DM, eds.
American Academy of Pediatrics Textbook of Pediatric Care. American Academy of
Pediatrics; 2021:chap 216. Accessed September 1, 2022. Pediatric Care Online.

American Academy of Pediatrics 537


PREP® Self-Assessment PREPSA 2023

Question 157
A 10-year-old girl with a history of recurrent otitis media is seen for follow-up after a course of
antibiotic ear drops for recurrent ear drainage and mild subjective hearing loss. She has had no
dizziness, headaches, or fever. On physical examination, her vital signs are within the normal
range. Findings on right ear otoscopic examination are shown in Item Q157. The remainder of
her physical examination findings are normal.

Of the following, the BEST description of the finding noted on this girl’s right otoscopic
examination is (a/the)
A. cholesteatoma
B. foreign body
C. head of the stapes
D. tympanosclerosis

American Academy of Pediatrics 538


PREP® Self-Assessment PREPSA 2023

Correct Answer: A
The right ear finding for the girl in the vignette is most likely a cholesteatoma, associated with
retraction of a portion of the tympanic membrane (TM). A foreign body is more likely to
protrude into the ear canal. The head of the stapes is inferior to the location of this girl’s finding.
Areas of tympanosclerosis are not usually retracted.

Cholesteatomas are collections of debris behind the TM. They most commonly form in retraction
pockets (Item C157A) that occur due to negative pressure on the TM resulting from chronic
otitis media. Less commonly, they form when debris enters through a perforation of or after
surgery involving the TM. The most common location for a cholesteatoma is the superior aspect
of the TM, called the pars flaccida (Item C157B), where the TM is thinner.

Pars Flaccida

If left untreated, cholesteatomas may grow into the middle ear and erode the ossicles. Long-term
sequelae include hearing loss, dizziness, abscess of the adjacent periosteum and/or brain tissue,
meningitis, and facial nerve palsy.

Surgical treatment performed by an otolaryngologist is required for cholesteatoma management;


due to the risk of recurrence, lifelong monitoring is necessary.

PREP Pearls
• Cholesteatomas are collections of debris that form in retraction pockets of the tympanic
membrane, most often secondary to chronic otitis media.
• If left untreated, cholesteatomas may grow into the middle ear and erode the ossicles.
• Cholesteatomas may recur after surgical removal, requiring lifelong monitoring.

ABP Content Specifications(s)


• Recognize the clinical findings associated with a cholesteatoma and the consequences if
left untreated

American Academy of Pediatrics 539


PREP® Self-Assessment PREPSA 2023

Suggested Readings
• American Academy of Otolaryngology-Head and Neck Surgery Foundation.
Cholesteatoma. In: Ishman SL, Stanley JJ, eds. Primary Care Otolaryngology. 4th ed.
Alexandria, VA: American Academy of Otolaryngology-Head and Neck Surgery
Foundation; 2019:26-28.
• Isaacson G. Diagnosis of pediatric cholesteatoma. Pediatrics. 2007;120(3):603-608.
doi:10.1542/peds.2007-0120.
• Nguyen CV, Parikh SR. Cholesteatoma. Pediatr Rev. 2008;29(9):330-331.
doi:10.1542/pir.29-9-330.

American Academy of Pediatrics 540


PREP® Self-Assessment PREPSA 2023

Question 158
A 15-month-old boy born at term is brought to the office for evaluation of seizure-like events.
His parents report episodes of shivering movements of his arms and upper body, lasting 5 to 10
seconds, occurring multiple times per day for the past few months. These events seem to happen
randomly, although they may be more frequent at mealtimes. There is no loss of consciousness
during the events, and they do not distress him or interrupt his activities. There is no significant
medical or perinatal history, he is meeting developmental milestones as expected, and there has
been no developmental regression. His physical examination findings are normal.

Of the following, the BEST next step in this boy’s evaluation is to obtain (a)
A. brain magnetic resonance imaging
B. routine electroencephalography
C. serum thyroid-stimulating hormone level
D. video recording of a characteristic event

American Academy of Pediatrics 541


PREP® Self-Assessment PREPSA 2023

Correct Answers: D
The boy in the vignette is experiencing brief episodes of shivering movements of his head and
upper extremities with no loss of consciousness. These episodes are suggestive of benign
shuddering attacks of infancy, a nonepileptic self-limited condition seen in infants and young
children. Benign shuddering attacks of infancy last several seconds, can occur multiple times per
day, and typically occur randomly but are often noticed more frequently during meal times,
diaper changes, or when the child is excited or frustrated. The diagnosis is made clinically; no
diagnostic testing is required. The best next step in the boy’s evaluation is to have the parents
obtain a video recording of a characteristic event. Video recordings can be helpful in the
diagnostic process, often eliminating the need for further evaluation such as video
electroencephalogram (EEG) monitoring.

The underlying pathophysiology of shuddering attacks is unknown. Initial studies postulated a


relationship with essential tremor, an association that has not been replicated in more recent
studies. In children with shuddering attacks, development and neurologic examination findings
are normal. Shuddering attacks can mimic seizures, and may be mistaken for tonic, myoclonic,
or absence seizure types. A careful history and home video can generally distinguish the
condition from seizures. In unusual presentations, or if doubt remains as to the nature of the
event, an attempt to capture events on video EEG monitoring is useful. In the case of shuddering
attacks, the EEG remains normal throughout, confirming their nonepileptic nature.

If benign shuddering attacks are suspected, neuroimaging and laboratory testing are not
indicated. Parental reassurance and education is a critical component of management as the
episodes can be distressing, especially if the child has frequent events.

In infants and young children, shuddering attacks represent one form of nonepileptic paroxysmal
disorders. Other nonepileptic conditions that may be mistaken for seizures commonly
encountered in this age group include: breath-holding spells, benign myoclonus of infancy,
Sandifer syndrome, benign torticollis of infancy, and rhythmic movement disorder (ie, head
banging) (Item C158). A comprehensive history with attention to the clinical features of the
events, conditions in which they occur, and provoking factors, as well as a video recording of the
events can help distinguish these conditions from seizures (Item C158). Video-EEG monitoring
remains a useful study to determine the nature of paroxysmal events in difficult cases.

PREP Pearls
• Benign shuddering attacks of infancy is a nonepileptic, self-limited condition of infancy
and early childhood characterized by brief shivering movements of the head and upper
extremities without alteration of consciousness. Episodes are more frequently noticed
during activities such as mealtime, diaper changes, or excitement/frustration.
• Distinguishing epileptic from nonepileptic conditions in infancy requires careful history
with attention to the clinical features of the events, conditions in which they occur, and
provoking factors. Video recording of events can be a useful tool for accurate diagnosis.

American Academy of Pediatrics 542


PREP® Self-Assessment PREPSA 2023

• When a diagnosis of benign shuddering attacks is suspected, parental reassurance and


education is a critical component of management. Investigative studies, such as
neuroimaging, electroencephalography, or laboratory studies are not necessary.

ABP Content Specifications(s)


• Differentiate between normal and abnormal repetitive movements during infancy

Suggested Readings
• Fine A, Wirrell EC. Seizures in children. Pediatr Rev. 2020;41(7):321-347.
doi:10.1542/pir.2019-0134.
• Nonconvulsive periodic disorders In: McInerny TK, Adam HM, Campbell DE, DeWitt
TG, Foy JM, Kamat DM, eds. American Academy of Pediatrics Textbook of Pediatric
Care. American Academy of Pediatrics; 2021:chap 179. Accessed September 1, 2022.
Pediatric Care Online.
• Tibussek D, Karenfort M, Mayatepek E, Assman B. Clinical reasoning: shuddering
attacks in infancy. Neurology. 2008;70(13):e38-e41.
doi:10.1212/01.wnl.0000306698.75592.6e.

American Academy of Pediatrics 543


PREP® Self-Assessment PREPSA 2023

Question 159
A 14-year-old adolescent boy is seen for a preparticipation physical examination. He would like
to participate in high school volleyball, wrestling, and baseball. The boy has a history of vision
impairment following an eye injury sustained as a young child. The corrected vision in his left
eye is 20/60, and his uncorrected vision in the right eye is 20/20. Recommendations for
participation and the need for protective eyewear are discussed with the boy and his family.

Of the following, it is MOST appropriate to clear this boy for participation in


A. volleyball and baseball only
B. volleyball only
C. volleyball, wrestling, and baseball
D. wrestling only

American Academy of Pediatrics 544


PREP® Self-Assessment PREPSA 2023

Correct Answer: A
The adolescent in the vignette is considered to have only 1 functional eye because the corrected
visual acuity in his left eye is less than 20/40. The American Academy of Pediatrics (AAP) and
American Academy of Ophthalmology (AAO) recommend that athletes who are functionally 1-
eyed wear protective eyewear for all sports. An eye injury resulting in vision loss in the
unaffected eye puts these athletes at risk for significant disability. Because protective eyewear
cannot be worn during wrestling, the boy in the vignette should not participate in this sport.
Combat sports (eg, martial arts and boxing) are also contraindicated for athletes who are
functionally 1-eyed.

Wrestling has been classified as a low-risk sport for eye injuries. However, in a study of eye
injuries sustained during high school and college sports, boys’/men’s wrestling was the fourth
riskiest sport for eye injuries (after women’s basketball, women’s field hockey, and men’s
basketball). Baseball is considered a high-risk sport for eye injury, and volleyball is a moderate
risk sport. However, protective eyewear is permissible and feasible during these activities.
The AAO recommends appropriate eyewear for common sports with a significant risk of eye
injury. For some sports, such as football and hockey, a face shield can be used instead of sports
goggles.

PREP Pearls
• Athletes with unilateral corrected visual acuity of less than 20/40 are considered
functionally 1-eyed.
• Athletes who are functionally 1-eyed should wear protective eyewear for all sports and
should not participate in sports where eye protection is not feasible or permitted.
• Protective eyewear is recommended for sports with a moderate to high risk for eye injury.

ABP Content Specifications(s)


• Recognize the indications for the use of protective eyewear during sports activities

Suggested Readings
• Bernhardt DT, Roberts WO; American Academy of Family Physicians. PPE:
Preparticipation Physical Evaluation. 5th ed. American Academy of Pediatrics; 2019.
• Boden BP, Pierpoint LA, Boden RG, Comstock RD, Kerr ZY. Eye injuries in high school
and collegiate athletes. Sports Health. 2017;9(5):444-449.
doi:10.1177/1941738117712425.
• Miller KN, Collins CL, Chounthirath T, Smith GA. Pediatric sports- and recreation-
related eye injuries treated in US emergency departments. Pediatrics.
2018;141(2):e20173083. doi:10.1542/peds.2017-3083.
• Turbert D. Safety glasses and protective eyewear. American Academy of
Ophthalmology. March 23, 2019. Accessed September 1, 2022. https://www.aao.org/eye-
health/tips-prevention/injuries-protective-eyewear.

American Academy of Pediatrics 545


PREP® Self-Assessment PREPSA 2023

Question 160
A 5-year-old boy is brought to the clinic for evaluation. He has had difficulty walking for the
past 2 days. He has bilateral calf pain and prefers not to walk; when he does walk, he does so
with a stiff-legged gait. Three days ago, the boy had a fever and cough, both of which have
resolved. There is no history of trauma or initiation of a new physical activity. The boy’s
physical examination findings are normal except for tenderness to palpation in both calves.

Of the following, the BEST next step in this boy’s evaluation is to order
A. antinuclear antibody and double-stranded DNA testing
B. antistreptolysin O testing
C. a creatinine phosphokinase level
D. a respiratory viral panel for influenza A/B

American Academy of Pediatrics 546


PREP® Self-Assessment PREPSA 2023

Correct Answer: C
The boy in the vignette has the classic symptoms and signs of postviral myositis (benign acute
childhood myositis [BACM]), including difficulty walking, calf pain, and calf tenderness after a
viral illness. Of the response choices, the best next step in his evaluation is to obtain a creatinine
phosphokinase (CPK) level. The CPK level is usually moderately elevated in patients with
BACM. This test is useful for excluding other diagnoses and in monitoring recovery. In a well-
appearing child with BACM, where close follow-up is feasible, laboratory testing may not be
needed.

The most dangerous sequela of BACM is rhabdomyolysis, which can lead to myoglobin release
and resulting renal failure. Rhabdomyolysis is very rare in children with BACM, and the risk of
occurrence does not correlate with the CPK level. Some experts recommend that children with
high CPK levels be monitored for dark-colored urine at home and, when noted, use urinalysis to
identify myoglobinuria. A reasonable management approach when mild signs and symptoms are
present, such as for the boy in the vignette, is observation without laboratory testing as long as
follow-up is feasible and anticipatory guidance is provided to return if symptoms worsen or
progress, weakness develops, or if dark-colored urine occurs.

Benign acute childhood myositis occurs more commonly in males. It is a self-limiting condition
that occurs in schoolaged children. Physical examination findings include bilateral, symmetric
tenderness of the lower extremities, usually over the gastrocnemius muscles, with preserved
strength. Benign acute childhood myositis is most commonly associated with influenza. Other
viruses (eg, parainfluenza, enteroviruses) and Mycoplasma pneumoniae have been reported.
Viral studies are not recommended for the boy in the vignette because the results would not alter
his management. The boy in the vignette’s findings are not consistent with a prior streptococcal
infection, so an antistreptolysin O test is not indicated.

The differential diagnosis of muscle pain or weakness in a child includes rheumatologic


disorders (eg, systemic lupus erythematosus, juvenile idiopathic arthritis), muscular dystrophies,
Guillain-Barré syndrome, and the rare autoimmune conditions juvenile dermatomyositis and
polymyositis. Both dermatomyositis and polymyositis are more common in females and typically
present with progressive proximal muscle weakness. Classic skin findings in dermatomyositis
include a heliotrope rash (Item C160A) and Gottron papules (Item C160B). These conditions
have a bimodal age of presentation at 5 to 14 years and 45 to 60 years. The incidence of both
conditions combined is approximately 1-7/100,000 children.

American Academy of Pediatrics 547


PREP® Self-Assessment PREPSA 2023

Item C160A & Item C160B Heliotrope rash and Gottron papules

A careful history and physical examination should be performed when a child has muscle pain
and/or weakness with attention to the time course (acute versus chronic), and the presence or
absence of weakness when pain is present. This information can help direct evaluation and
testing. Antinuclear antibody and double-stranded DNA testing are not indicated for the boy in
the vignette, who had acute onset of muscle pain without weakness. These tests would be
indicated if the pain was chronic and associated with progressive weakness.

PREP Pearls
• Benign acute childhood myositis is an acute myositis that occurs after a viral infection,
most often influenza.
• A child with classic signs and symptoms of benign acute childhood myositis does not
require an extensive evaluation. A creatinine phosphokinase level is usually elevated, but
rhabdomyolysis is rare.
• Rheumatological conditions (eg, polymyositis, dermatomyositis, systemic lupus
erythematosus, juvenile idiopathic arthritis), and muscular dystrophies should be
considered in a child with chronic progressive muscle pain and weakness.

ABP Content Specifications(s)


• Identify the etiology of myositis
• Plan the appropriate evaluation and management of myositis

American Academy of Pediatrics 548


PREP® Self-Assessment PREPSA 2023

Suggested Readings
• Feldman BM, Rider LG, Reed AM, Pachman LM. Juvenile dermatomyositis and other
idiopathic inflammatory myopathies of childhood. Lancet. 2008;371(9631):2201-2212.
doi:10.1016/S0140-6736(08)60955-1.
• Juvenile dermatomyositis: a clinical overview. Pediatr Rev. 1990;12(4):117-25.
doi:10.1542/pir.12-4-117.
• Magee H, Goldman RD. Viral myositis in children. Can Fam Physician. 2017;63(5):365-
368. https://pubmed.ncbi.nlm.nih.gov/28500193/.
• Quddusi FI, Youssef MJ, Davis DMR. Dermatologic manifestations of systemic diseases
in childhood. Pediatr Rev. 2021;42(12):655-671. doi:10.1542/pir.2020-000679.
• Siegel DM, Gewanter HL, Sahai S. Rheumatologic diseases. In: McInerny TK, Adam
HM, Campbell DE, DeWitt TG, Foy JM, Kamat DM, eds. American Academy of
Pediatrics Textbook of Pediatric Care. American Academy of Pediatrics; 2021:chap 324.
Accessed September 1, 2022. Pediatric Care Online.

American Academy of Pediatrics 549


PREP® Self-Assessment PREPSA 2023

Question 161
A pediatric practice is considering changing the screening tools that have been used routinely in
the office for the past 5 years. One of the physicians in the practice states that he never uses the
screening tool and instead asks parents “a couple yes-or-no questions every few visits” to
determine if there are any delays or if referral to early intervention is needed. Another physician
in the practice states that she uses the screening tool at some of the recommended time points but
does not always refer children following a positive screening test result because, “There are so
many false positives, and I don’t want to scare parents if they don’t have concerns.”

Of the following, the BEST strategy for this practice to adopt is


A. implementation of the screening tools with the highest sensitivity and specificity
B. referral of all infants and children with a positive screening test result for further
evaluation
C. surveillance with standardized tools, and parent and physician input
D. use of a screening tool when there is physician and/or parental concern

American Academy of Pediatrics 550


PREP® Self-Assessment PREPSA 2023

Correct Answer: C
Developmental surveillance is an important component of health supervision in pediatric
practice. It is a longitudinal process that includes eliciting parental concerns, monitoring
developmental milestones and behaviors, and assessing for both risk factors and protective
factors that may impact development. Developmental screening involves the use of a
standardized tool to support the identification of children who need further evaluation and may
need intervention. Developmental screening tools are not used to make a diagnosis. A number of
“false-positive” results are expected and appropriate in any screening process and are necessary
to avoid false-negative (missed positive) cases that require further evaluation and possible
treatment. Importantly, children who screen positive with a developmental screening tool but do
not qualify for early intervention services or receive a formal diagnosis, should still be
considered high risk and require close monitoring.

Informal questioning in place of developmental screening with a standardized tool can result in
the underidentification of children with developmental and behavioral disorders. Use of a
developmental screening test in isolation without eliciting parental concerns may also result in
developmental disorders being missed (false-negative) and weaken the physician–family
relationship.

The psychometric properties (reliability, sensitivity, specificity, positive-predictive value,


negative-predictive value) of screening tools are important. However, even with strong
psychometric properties, no single instrument can screen for every developmental-behavioral
disorder. It is recommended that the following types of developmental screening tools be used in
routine health supervision:
• Broadband developmental (assesses multiple areas, eg, motor, cognitive, language)
• Social emotional/behavioral
• Autism-specific

The American Academy of Pediatrics recommends formal developmental screening at:


• 9, 18, and 30 months of age for general developmental screening
• 18 and 24 months of age for autism-specific screening
• Other time points when surveillance identifies a concern or risk

Use of standardized screening tools supports the identification of children with or at risk for
developmental and behavioral disorders. It does not take the place of developmental surveillance
or clinical judgment. Additionally, when selecting a screening tool, it is important to consider
patient characteristics, socio-demographic characteristics, clinic resources (time to
complete/interpret the tool) as well as local early intervention eligibility criteria.

American Academy of Pediatrics 551


PREP® Self-Assessment PREPSA 2023

PREP Pearls
• Developmental surveillance should occur at every health supervision visit.
Developmental screening using a standardized instrument should occur at the 9-,18-, and
30-month visits; autism screening should occur at the 18and 24-month visits.
• Informal screening leads to underidentification of children with or at risk for
developmental and behavioral disorders.
• No single screening tool can identify every developmental or behavioral disorder.

ABP Content Specifications(s)


• Understand the uses and limitations of various developmental screening tools

Suggested Readings
• Lipkin PH, Macias MM; Council on Children with Disabilities, Section on
Developmental and Behavioral Pediatrics. Promoting optimal development: identifying
infants and young children with developmental disorders through developmental
surveillance and screening. Pediatrics. 2020;145(1):e20193449. doi:10.1542/peds.2019-
3449.
• Marks KP, LaRosa AC. Understanding developmental-behavioral screening measures.
Pediatr Rev. 2012;33(10):448-458. doi:10.1542/pir.33-10-448.
• Voigt RG, Macias MM, Myers SM, Tapia CD, eds. Developmental and Behavioral
Pediatrics: Section on Developmental and
• Behavioral Pediatrics American Academy of Pediatrics. 2nd ed. American ACademy of
Pediatrics; 2018. Accessed September 1, 2022.
https://publications.aap.org/aapbooks/book/668/Developmental-and-Behavioral-
PediatricsSectionon.

American Academy of Pediatrics 552


PREP® Self-Assessment PREPSA 2023

Question 162
A 2-year-old boy is evaluated in the emergency department for 5 days of fever, rash, and red
eyes. He was seen at an urgent care center 2 days ago, where he was given amoxicillin; there has
been no decrease in fever. Similar symptoms are present in several children attending his toddler
swim class. There has been no SARS-CoV-2 infection documented among his contacts in the
past 6 weeks. On physical examination, he has dry, cracked lips, and dry mucous membranes.
His conjunctivae are injected bilaterally. He has tender preauricular lymph nodes, pharyngeal
erythema, and tonsillar exudates.

Results of laboratory testing performed at the urgent care center are shown:
Laboratory Test Result
White blood cell 12,000/µL (12.0 × 109/L)
Neutrophils 25%
Lymphocytes 55%
Monocytes 18%
Eosinophils 2%
Hemoglobin 12 g/dL (120 g/L)
Platelet count 150 × 103/µL (150 × 109/L)
Group A streptococcal antigen test Negative
Group A Streptococcus culture Pending
C-reactive protein 3.5 mg/dL (35 mg/L)
SARS-CoV-2 polymerase chain reaction Not detected
Epstein–Barr virus VCA IgM <17 U/mL
Epstein–Barr virus VCA IgG >150 U/mL
Epstein–Barr virus nuclear IgG >300 U/mL
Abbreviation: VCA, viral capsid antigen.

Of the following, the MOST appropriate test to confirm this boy’s diagnosis is
A. adenovirus polymerase chain reaction
B. Epstein-Barr virus polymerase chain reaction
C. group A Streptococcus antibodies
D. SARS-CoV-2 spike protein antibodies

American Academy of Pediatrics 553


PREP® Self-Assessment PREPSA 2023

Correct Answer: A
The boy in the vignette has a clinical constellation of acute follicular conjunctivitis (Item
C162A), tender preauricular lymphadenopathy, pharyngitis, and fever, which is most consistent
with adenovirus pharyngoconjunctival fever. Adenoviral tonsillitis most often occurs in children
younger than 3 years.

It is unlikely that the boy in the vignette has group A streptococcal (GAS) tonsillitis, which is
rare in children younger than 2 years. The highest prevalence of GAS tonsillitis occurs in
children between 5 and 7 years of age and typically responds to penicillin within 24 to 48 hours.
Unlike the findings of the boy in the vignette, Epstein-Barr virus (EBV) infection is associated
with symmetric bilateral cervical lymphadenopathy (predominantly posterior). The boy’s EBV
serology is consistent with past infection, and he does not have the atypical lymphocytosis
(>10%) observed in most children younger than 4 years with acute EBV infection. The presence
of fevers for more than 3 days should raise suspicion for multisystem inflammatory syndrome in
children associated with COVID-19, which is seen 2 to 6 weeks after acute SARS-CoV-2
infection; however, this child has no known history suggestive of COVID-19 infection in that
time frame. A diagnosis of incomplete Kawasaki disease should also be considered, but this
child’s laboratory findings do not meet criteria for this diagnosis.

Adenovirus, a common cause of viral infections in young children, is a non-enveloped DNA


virus; 75% of children are seropositive by 5 years of age. It is transmitted through person-to-
person contact (touching, kissing), droplets (cough, sneeze), and fomites (unclean hands, toys).
The incubation period is 5 to 10 days. The clinical manifestations depend on the serotype, site of
infection, and the child’s immune status. Several clinical syndromes are associated with
adenoviral infection (Item C162B). Adenovirus infection in otherwise healthy children results in
organ-specific (respiratory, gastrointestinal, genitourinary) self-limiting illness. Adenovirus
conjunctivitis presents in 4 primary forms: (1) pharyngoconjunctival fever, (2) epidemic
keratoconjunctivitis, (3) conjunctivitis associated with respiratory tract infection, or (4) isolated
follicular conjunctivitis. Acute adenovirus gastroenteritis in young children typically lasts longer
than other infectious causes of diarrhea. An exanthem (urticaria rash, maculopapular rash) may
or may not accompany adenoviral clinical syndromes. Meningitis and encephalitis are
occasionally seen.

American Academy of Pediatrics 554


PREP® Self-Assessment PREPSA 2023

Immunocompromised children, particularly those with a history of hematopoietic stem cell or


solid organ transplant, are predisposed to severe, disseminated infections in the first month after
the transplant.

The preferred method for diagnosis of adenovirus infection is a molecular assay (eg, polymerase
chain reaction) test performed on a nasopharyngeal swab sample. The commonly available
respiratory multiplex viral panels include adenovirus. Treatment of immunocompetent hosts is
primarily supportive care.

American Academy of Pediatrics 555


PREP® Self-Assessment PREPSA 2023

PREP Pearls
• Adenovirus is responsible for a wide variety of infections in children, most commonly
respiratory and gastrointestinal.
• Adenovirus pharyngoconjunctival fever presents with follicular conjunctivitis, tender
preauricular lymphadenopathy, pharyngitis, and fever.
• Adenovirus is diagnosed by polymerase chain reaction performed on a nasopharyngeal
swab sample; this test is included in the available respiratory multiplex viral panels.

ABP Content Specifications(s)


• Understand the epidemiology of adenovirus
• Recognize the clinical features associated with adenovirus infection

Suggested Readings
• Allen UD, Demmler GJ. Adenoviruses. In: Long SS, ed. Principles and Practice of
Pediatric Infectious Diseases. 4th ed. Elsevier; 2012:1067-1071.
• American Academy of Pediatrics. Adenovirus infections. In: Kimberlin DW, Barnett ED,
Lynfield R, Sawyer MH, eds. Red Book: 2021–2024 Report of the Committee on
Infectious Diseases. 32nd ed. American Academy of Pediatrics; 2021. Accessed
September 1, 2022. Red Book Online.
• Chen TK, Cherry JD. Adenoviruses. In: Cherry JD, Harrison GJ, Kaplan SL, Hotez PJ,
Steinbach WJ, eds. Feigin and Cherry’s Textbook of Pediatric Infectious Diseases. 7th
ed. Saunders; 2014:1364-1383.
• Steele RW. Pharyngitis and tonsillitis. In: McInerny TK, Adam HM, Campbell DE,
DeWitt TG, Foy JM, Kamat DM, eds. American Academy of Pediatrics Textbook of
Pediatric Care. American Academy of Pediatrics; 2021:chap 311. Accessed September 1,
2022. Pediatric Care Online.

American Academy of Pediatrics 556


PREP® Self-Assessment PREPSA 2023

Question 163
A 7-year-old girl is brought to the clinic for concerns about her balance. For the past few days,
the girl has complained of the room spinning and nausea. She has 5 to 6 episodes per day, each
lasting about 1 minute, during which she grabs onto a chair or the sofa. Between episodes she
acts and feels normal. She is otherwise healthy with no history of trauma or recent viral
symptoms. Her physical examination findings are unremarkable, including her tympanic
membranes and a detailed neurologic examination.

Of the following, the BEST next step in this girl’s management is


A. an antihistamine prescription
B. head and neck imaging
C. observation
D. referral to neurology

American Academy of Pediatrics 557


PREP® Self-Assessment PREPSA 2023

Correct Answer: C
The girl in the vignette most likely has benign paroxysmal vertigo of childhood (BPVC), given
her intermittent symptoms and unremarkable physical examination findings. Benign paroxysmal
vertigo of childhood is typically a self-limited condition for which observation is recommended.
Diagnostic evaluation and referral to neurology are only recommended if symptoms worsen or
there is impairment of function. Medication management with antiemetics or antihistamines may
be warranted if episodes are long lasting, which is not yet the case for the girl in the vignette.
Although the cause of BPVC is unknown, there is usually a family history of migraine
headaches, and children with BPVC may subsequently develop migraines.

Young children may have difficulty describing vertigo, defined as a sensation of rotational
movement that usually involves the vestibular system, either peripheral (inner ear) or central
(brainstem, cerebellum, or cranial nerve VIII). History and physical examination findings
provide etiologic clues. Middle ear disease is the most common cause of vertigo in children.
Benign paroxysmal vertigo of childhood is the second most common cause of vertigo in children
aged 2 to 12 years. The diagnosis of BPVC is clinical and based the following criteria:
• Brief episodes of vertigo associated with vomiting, pallor, nystagmus, ataxia, or
fearfulness
• Normal neurologic examination, audiometric, and vestibular findings between episodes
• Symptom-free intervals
• Not attributable to another disorder

Benign paroxysmal vertigo of childhood may be considered a migraine equivalent. The


prognosis is good, usually with spontaneous resolution before adolescence.
Benign paroxysmal vertigo of childhood is a distinct entity from benign paroxysmal positional
vertigo (BPPV), which is associated with position change, can be elicited by the Dix-Hallpike
maneuver (neck is extended and turned to one side followed by quick placement in the supine
position), and is caused by calcium deposits in the posterior semicircular canal. Benign
paroxysmal positional vertigo is rare in children. Vestibular migraines may also cause vertigo.

PREP Pearls
• Benign paroxysmal vertigo of childhood is the second most common cause of vertigo in
children aged 2 to 12 years; the most common cause is middle ear disease.
• The diagnosis of benign paroxysmal vertigo is based on a history of brief episodes of
vertigo associated with vomiting, pallor, nystagmus, ataxia or fearfulness, normal
neurologic examination findings, audiometric, and vestibular findings between episodes,
symptom-free intervals, and no other attributable disorder.
• Benign paroxysmal vertigo is a self-limited condition for which the treatment is
observation. Referral to neurology for further evaluation and management may be
considered for worsening symptoms or impairment of function.

ABP Content Specifications(s)


• Recognize the clinical findings associated with benign paroxysmal vertigo

American Academy of Pediatrics 558


PREP® Self-Assessment PREPSA 2023

Suggested Readings
• Batson G. Benign paroxysmal vertigo of childhood: a review of the literature. Paediatr
Child Health. 2004;9(1):31-34. doi:10.1093/pch/9.1.31.
• Benun J. Balance and vertigo in children. Pediatr Rev. 2011;32(2):84-85.
doi:10.1542/pir.32.2.84.
• Rivera RF, Sellinger CR. Dizziness and vertigo. In: McInerny TK, Adam HM, Campbell
DE, DeWitt TG, Foy JM, Kamat DM, eds. American Academy of Pediatrics Textbook of
Pediatric Care. American Academy of Pediatrics; 2021:chap 140. Accessed September 1,
2022. Pediatric Care Online.
• van de Berg R, Widdershoven J, Bisdorff A, et al. Vestibular migraine of childhood and
recurrent vertigo of childhood: diagnostic criteria consensus document of the Committee
for the Classification of Vestibular Disorders of the Bárány Society and the International
Headache Society. J Vestib Res. 2021;31(1):1-9. doi:10.3233/VES-200003.

American Academy of Pediatrics 559


PREP® Self-Assessment PREPSA 2023

Question 164
A 16-month-old girl is seen for a health supervision visit. She eats a wide variety of foods and
drinks 40 oz of cow milk per day. On physical examination, she looks pale. Her vital signs are
normal for age. The remainder of her physical examination findings are normal.
Screening laboratory evaluation reveals a hemoglobin level of 6.4 g/dL (64 g/L) and a mean
corpuscular volume of 65 fL.

After review of the laboratory results, a more detailed history is obtained. The family recently
renovated their house. The girl often puts items into her mouth (things found on the floor and
paper items). She has not had bright red blood in her diaper or darker stools.

Of the following, the MOST likely additional laboratory findings for this girl include

Reticulocytes RDW Ferritin Soluble Transfer Receptor


A High Normal Normal Normal
B Low Normal High Low
C Low High Low High
D Low Normal Normal High

American Academy of Pediatrics 560


PREP® Self-Assessment PREPSA 2023

Correct Answers: C
The girl in the vignette has iron-deficiency anemia, the most common cause of anemia in her age
group. Her high-volume consumption of cow milk, pica, and pallor support this diagnosis.
Laboratory findings consistent with irondeficiency anemia include a low hemoglobin (Hb) level,
low mean corpuscular volume (MCV), low reticulocyte count, high red blood cell distribution
width (RDW), high soluble transferrin receptor (sTfR) level, and low ferritin level (the storage
form of iron).

Although a low reticulocyte count is an inappropriate response to anemia (the appropriate


response is increased red blood cell production), the reticulocyte count is decreased in iron-
deficiency anemia because of the lack of available iron to make new red blood cells. It is the first
value to increase once iron is supplemented. The sTfR level helps differentiate iron-deficiency
anemia (high sTIR) from anemia of chronic disease or inflammation (low sTIR).

Anemia of chronic disease or inflammation is associated with a low reticulocyte count, normal
RDW, high ferritin level, and low sTfR level. Gastrointestinal blood loss should be considered in
the differential diagnosis of any child with iron-deficiency anemia. There may be a poor
response to oral iron supplementation if there is a gastrointestinal condition; evaluation by a
gastroenterologist for an inflammatory bowel disorder may be indicated. Markers of
inflammation that can be helpful in determining the cause of anemia include elevated ferritin
level, erythrocyte sedimentation rate, C-reactive protein level, and platelet count.

Both α- and β-thalassemia traits are associated with mild microcytic anemia. In these conditions
the reticulocyte count is low, RDW is normal, ferritin level is normal, and the sTfR is high. Iron
study results are normal, and there is no improvement in the Hb level with iron supplementation.
An elevation in HbA2 on Hb electrophoresis is consistent with β-thalassemia trait; however, the
results can be falsely normal in the setting of iron deficiency. Hemoglobin electrophoresis cannot
be used to diagnose α-thalassemia trait; it can sometimes be detected on newborn screening as
Hb Barts. Genetic testing is available for both α- and β-thalassemia traits.

Lead toxicity, an uncommon cause of anemia, is associated with a high reticulocyte count,
normal RDW, normal ferritin level, and normal sTfR level. The MCV and reticulocyte count
may be normal in lead poisoning. The peripheral blood smear may show basophilic stippling.

Item C164A summarizes the laboratory findings associated with causes of microcytic anemia.

The reticulocyte count can help determine the cause of anemia, especially normocytic anemia.
An elevated reticulocyte count usually indicates destruction of red blood cells, either
intravascular or extravascular. Additional laboratory findings that support hemolysis include an
elevated indirect bilirubin level and low haptoglobin level. A positive direct antiglobulin
(Coombs) test result supports an immune hemolytic anemia. A low reticulocyte count can
indicate impaired bone marrow function, either from temporary suppression (eg, infection or
medication) or bone marrow failure.

American Academy of Pediatrics 561


PREP® Self-Assessment PREPSA 2023

Normal Hb and MCV values vary with age. It is important to use the correct reference range to
determine which laboratory findings warrant further investigation. Item C164B shows the
normal range for Hb values and Item C164C shows the normal range for MCV values based on
age.

American Academy of Pediatrics 562


PREP® Self-Assessment PREPSA 2023

American Academy of Pediatrics 563


PREP® Self-Assessment PREPSA 2023

PREP Pearls
• Iron deficiency is the most common cause of microcytic anemia in childhood.
• Children with iron deficiency should be evaluated to identify any gastrointestinal cause of
blood loss (eg, inflammatory bowel disease).

ABP Content Specifications(s)


• Recognize the laboratory findings associated with microcytic anemia
• Recognize the normal variations in hemoglobin concentration and mean cell volume
during childhood
• Distinguish between a disorder of erythrocyte production and a disorder of erythrocyte
destruction based on laboratory results

Suggested Readings
• Krishnamurti L. Iron-deficiency anemia. In: McInerny TK, Adam HM, Campbell DE,
DeWitt TG, Foy JM, Kamat DM, eds. American Academy of Pediatrics Textbook of
Pediatric Care. American Academy of Pediatrics; 2021:chap 279. Accessed September 1,
2022. Pediatric Care Online.
• Lanzkowsky P, Lipton J, Fish JD. Lanzkowsky’s Manual of Pediatric Hematology and
Oncology. 6th ed. Elsevier Inc; 2016:38,77.
• McFarren AK, Levy AS. Anemia and pallor. In: McInerny TK, Adam HM, Campbell
DE, DeWitt TG, Foy JM, Kamat DM, eds. American Academy of Pediatrics Textbook of
Pediatric Care. American Academy of Pediatrics; 2021:chap 128. Accessed September 1,
2022. Pediatric Care Online.
• Richardson M. Microcytic anemia. Pediatr Rev. 2007;28(1):5-14. doi:10.1542/pir.28-1-5.
• Zanetti R, Feldman B, Porea T. Microcytic anemia. Pediatr Rev. 2021;42(1):41-43.
doi:10.1542/pir.2019-0295.

American Academy of Pediatrics 564


PREP® Self-Assessment PREPSA 2023

Question 165
A neonate born at 31 weeks’ gestation with a birth weight of 1,500 g is admitted to the neonatal
intensive care unit. The neonate currently requires continuous positive pressure support of 5 cm
H2O and a fraction of inspired oxygen of 25%. To provide ease of access for care, the newborn
is placed under a radiant warmer.

Of the following, the MOST likely effect of this care environment on the neonate will be
A. decreased insensible water loss
B. excess weight gain
C. hyperthermia
D. hypoglycemia

American Academy of Pediatrics 565


PREP® Self-Assessment PREPSA 2023

Correct Answer: C
The most likely effect of care for a neonate under a radiant warmer is hyperthermia. Radiant
warmers, while allowing for easy access to neonates, are associated with increased insensible
water losses and, if not used properly, can result in significant deleterious effects.

All newborns, especially those born prematurely and/or at low birth weight, are at higher risk for
heat loss than older children because of neonates’ higher skin surface–area-to-weight (volume)
ratio, decreased amount of subcutaneous fat, permeable skin, and increased transdermal loss of
water and heat. These factors determine the neutral thermal environment (NTE), a set of
conditions in which oxygen consumption (metabolic demand) is minimal when body temperature
is maintained in the reference range. An increase or decrease in the NTE can have deleterious
effects on the neonate. The NTE temperature range is dependent on the gestational age, weight,
postnatal age, and clinical status of the neonate. Heat loss occurs via a combination of 4 different
phenomena: evaporation, conduction, radiation, and convection (Item Q165A).

Reprinted with permission from Roychoudhury S, Yusuf K. Thermoregulation: Advances in Preterm


Infants. Neoreviews. 2017;18(12):e695.

The NTE under a radiant warmer is managed through maintaining normal skin temperature by
use of servo control with a skin sensor probe. If the sensor probe is not properly placed or is
dislodged, it can result in hyperthermia or hypothermia. The probe should be taped to an
exposed, fleshy part of the neonate’s trunk. A probe taped over a bony prominence or a limb can
record a lower temperature than the actual general skin temperature; in response, the radiant
warmer will provide additional heat, resulting in hyperthermia. If the neonate is lying on top of
the probe, the sensor can register a temperature higher than the neonate’s actual temperature,
causing the radiant warmer to provide less heat than needed, resulting in hypothermia. The
automatic rather than manual mode is recommended to decrease the risk of hyperthermia. A

American Academy of Pediatrics 566


PREP® Self-Assessment PREPSA 2023

neonate’s insensible water loss (IWL) while under a radiant warmer can lead to weight loss;
excess weight gain and hypoglycemia are not effects of this environment.

Humidity plays an important role in the maintenance of NTE, especially in preterm neonates
because of their immature epithelial layers, larger surface area, and increased skin vascularity.
Inadequate humidity may contribute to morbidity and mortality. Thermal control is improved
and IWL is decreased when adequate humidity is present in the environment. The daily fluid
requirement for neonates, especially preterm neonates, depends largely on IWL (Item Q165B).
The major component of IWL is transepidermal loss, which varies with gestational age, postnatal
age, and ambient water vapor pressure. Insensible water loss also occurs across the upper airway
epithelium. Insensible water loss is obligate free water loss, which can make electrolyte
management a challenge. Antenatal corticosteroids decrease IWL by decreasing transepidermal
loss.

Item C165B. Estimated Newborn Fluid Requirement by Birth Weight.

Neonates cared for in double-walled incubators have less IWL compared with those cared for
with radiant warmers, single-walled incubators, or heat shields. The level of humidity under a
radiant warmer is dependent on the humidity of the room and cannot be controlled easily.
Radiant warmers can increase IWL by 50% to 80%, whereas doublewalled incubators can
decrease IWL by 30% to 50%. Double-walled incubators have dual modes: when closed, they
allow care in a humidified environment, and while open, they provide access to the neonate for
procedures, handling, and parental interactions with less control of humidification. A double-
walled incubator is the best choice of care environment for attaining a NTE for the neonate in the
vignette.

PREP Pearls
• A neutral thermal environment is important for care of newborns.
• Premature neonates should ideally be cared for in a double-walled incubator.
• If a radiant warmer is used, care must be taken to ensure that the sensor probe is properly
placed and the warmer is placed in automatic mode; careful monitoring of the neonate’s
temperature is required.

ABP Content Specifications(s)


• Recognize the hazards and benefits associated with the use of radiant warmers for
neonates

American Academy of Pediatrics 567


PREP® Self-Assessment PREPSA 2023

• Understand how prematurity and the use of radiant warmers affect insensible water loss,
especially in preterm infants
• Recognize the differences in daily fluid requirements per kilogram of body weight in
preterm and full-term infants

Suggested Readings
• American Academy of Pediatrics. Thermal environment. In: PCEP Book 1: Maternal and
Fetal Evaluation and Immediate Newborn Care. American Academy of Pediatrics;
October 2021:247-284. doi:10.1542/9781610024952-unit7.
• Pinheiro JMB. Assessment and stabilization at delivery. In: McInerny TK, Adam HM,
Campbell DE, DeWitt TG, Foy JM, Kamat DM, eds. American Academy of Pediatrics
Textbook of Pediatric Care. American Academy of Pediatrics; 2021:chap 109. Accessed
September 1, 2022. Pediatric Care Online.
• Ringer SA. Core concepts: thermoregulation in the newborn part I: basic mechanisms.
Neoreviews. 2013;14(4):e161– e167. doi10.1542/neo.14-4-e161.
• Ringer SA. Core concepts: thermoregulation in the newborn, part II: prevention of
aberrant body temperature. Neoreviews. 2013;14(5):e221–e226. doi:10.1542/neo.14-5-
e221.

American Academy of Pediatrics 568


PREP® Self-Assessment PREPSA 2023

Question 166
A 3-year-old boy is seen in the office to establish care after emigrating from Southeast Asia 2
months ago. He has had a cough for 12 months and has been eating less for the past 6 months.
He has a habit of soil pica. The family has 4 dogs and 5 cats. The boy’s physical examination
findings are remarkable for scattered wheezing in both lung fields and a liver edge palpable 2 cm
below the right costal margin. His weight is less than the third percentile. His parents report that
his weight is usually at the 25th percentile.

Laboratory data are shown:


Laboratory Test Result
White blood cell count 15,000/µL (15.0 × 109/L)
Neutrophils 45%
Lymphocytes 30%
Monocytes 5%
Eosinophils 20%
Hemoglobin 10 g/dL (100 g/L)
Platelet count 200 × 103/µL (200 × 109/L)
Alanine aminotransferase 55 U/L
Aspartate aminotransferase 65 U/L

Of the following, the BEST next step to confirm this boy’s diagnosis is to obtain
A. serum for Ascaris IgG
B. serum for Toxocara IgG
C. stool for bacterial culture
D. stool for ova and parasites

American Academy of Pediatrics 569


PREP® Self-Assessment PREPSA 2023

Correct Answer: B
The boy in the vignette has toxocariasis, a tissue roundworm infection. His signs and symptoms
(wheezing, hepatomegaly, anorexia, and weight loss), exposure to dogs and cats, habit of soil
pica, and eosinophilia are consistent with infection with the visceral larva migrans form of
Toxocara. In Southeast Asia, the seroprevalence of toxocariasis is approximately 34%. In the
United States, the seroprevalence is 5%.

The diagnosis of toxocariasis is based on compatible signs and symptoms and risk factors for
acquisition, such as exposure to dogs and cats, which are definitive hosts. Young children who
spend time in potentially contaminated areas (eg, playgrounds and sandboxes) and place soil in
their mouths are at increased risk of acquiring Toxocara infection. Toxocara infection is
confirmed with serologic testing, with an enzyme-linked immunosorbent assay that detects
Toxocara IgG levels. Stool for ova and parasites is not helpful in the diagnosis of toxocariasis
because the parasite does not complete its life cycle in the human gastrointestinal tract.

Toxocariasis, a roundworm (nematode) infection, is caused by 2 species: Toxocara canis and


Toxocara cati, parasites of dogs (puppies) and cats (kittens), respectively. Humans are
unintentional hosts.

Toxocara infection causes disease when larvae migrate to tissues and local immune reactions
result in injury. Item C166A reviews the major symptomatic presentations.

American Academy of Pediatrics 570


PREP® Self-Assessment PREPSA 2023

Albendazole is the recommended treatment for Toxocara. Mebendazole is the alternative.

Testing the serum for Ascaris IgG would not be appropriate for the boy in the vignette, who has
a clinical picture consistent with toxocariasis. Ascariasis (Ascaris lumbricoides) is the most
common intestinal roundworm infection. Its clinical features overlap with toxocariasis. However,
exposure to dogs is not a risk factor for Ascaris infection. A child typically acquires ascariasis
through ingestion of water or food contaminated with the parasite’s eggs. The diagnosis is made
by visualization of Ascaris ova (Item C166B) on stool microscopy. Serologic testing is reserved
for seroprevalence studies rather than clinical diagnosis. Cross-reactivity of Ascaris IgG with
other helminths is common.

Item C166B: Ascaris ova

The boy in the vignette does not have signs or symptoms of gastroenteritis from a bacterial
pathogen, such as Salmonella or Shigella. Therefore, obtaining a stool culture would not confirm
his diagnosis.

PREP Pearls
• Toxocariasis is a parasitic infection caused by 2 species of roundworm (nematode):
Toxocara canis and Toxocara cati, parasites of dogs (puppies) and cats (kittens),
respectively.
• Toxocara infection causes disease when larvae migrate to tissues and local immune
reactions result in injury. Major symptomatic presentations are visceral larva migrans
(young children) and ocular larva migrans (older children and adolescents).
• The diagnosis of Toxocara is confirmed by Toxocara IgG serologic testing. Stool for ova
and parasites are not helpful because the adult worm does not complete its life cycle in
the human gastrointestinal tract.

ABP Content Specifications(s)


• Understand the epidemiology of Toxocara
• Recognize the clinical features associated with Toxocara infestation

American Academy of Pediatrics 571


PREP® Self-Assessment PREPSA 2023

Suggested Readings
• Abdel-Haq N, Chearskul P, Rafee Y, Asmar BI. Parasitic infections. In: McInerny TK,
Adam HM, Campbell DE, DeWitt TG, Foy JM, Kamat DM, eds. American Academy of
Pediatrics Textbook of Pediatric Care. American Academy of Pediatrics; 2021:chap 308.
Accessed September 1, 2022. Pediatric Care Online.
• American Academy of Pediatrics. Toxocariasis. In: Kimberlin DW, Barnett ED, Lynfield
R, Sawyer MH, eds. Red Book: 2021–2024 Report of the Committee on Infectious
Diseases. 32nd ed. American Academy of Pediatrics; 2021. Accessed September 1, 2022.
Red Book Online.
• Cherry JD, Harrison GJ, Kaplan SL, Hotez PJ, Steinbach WJ. Parasitology. In: Feigin
and Cherry’s Textbook of Pediatric Infectious Diseases. 7th ed. Sauders; 2014:section
XXII.
• Dickson D, Hotez PJ. Tissue nematodes. In: Long SS, ed. Principles and Practice of
Pediatric Infectious Diseases. 4th ed. Elsevier; 2012:1334-1341.e2.
• Woodhall DM. Fiore AE, Toxocariasis: a review for pediatricians. J Pediatric Infect Dis
Soc. 2014;3(2):154-149. doi:10.1093/jpids/pit066.

American Academy of Pediatrics 572


PREP® Self-Assessment PREPSA 2023

Question 167
A pediatric resident seeks to study the impact of mother–infant separation at birth on
breastfeeding outcomes during the COVID-19 pandemic. The study population includes mothers
with SARS-CoV-2 infection (symptomatic and asymptomatic) and their asymptomatic infants in
2 different community hospitals. All mother-infant dyads were separated at one hospital. In the
other hospital, asymptomatic mother-infant dyads were allowed to room-in. The pediatric
resident collects information on breastfeeding outcomes at 1 and 6 months after discharge in
these 2 groups.

Of the following, the design of this study is BEST described as


A. case-control
B. cohort
C. cross-sectional
D. ecologic

American Academy of Pediatrics 573


PREP® Self-Assessment PREPSA 2023

Correct Answer: B
Among the response choices, the study design that best describes the research study in vignette is
a cohort study.

Clinical research is of paramount importance in the development of evidence-based medical


practice. Clinical research studies are divided into the categories of experimental and
observational. The majority of new treatments and technologies are introduced into clinical
practice after randomized prospective clinical trials (experimental research). On the other hand,
observational research examines the effects of predetermined policies, treatments, and
interventions. Item C167 provides an overview of the different types of observational studies.

In the vignette, the investigator seeks to study the impact of a predetermined policy of mother-
infant separation on breastfeeding outcomes. The 2 hospitals have different policies. One
separates mother–infant dyads if the mother is detected to have SARS-CoV-2, irrespective of
symptom status. The other hospital allows rooming in of asymptomatic mother–infant dyads.
The outcome of breastfeeding is observed overtime in the 2 groups. This is a longitudinal cohort
study design, prospective in nature.

A cohort study begins with the exposure (or risk factor); the cohort is observed over time for the
development of the outcome of interest. In contrast, a case-control study begins with the outcome
and looks back in time for the exposure/risk factor. A case-control study design is useful when
the disease outcome is rare because it begins with the selection of a population with the outcome
of interest. The case population has evidence of the outcome of interest, whereas the control
population does not. An example of a case-control study is a study of children with asthma
(cases) compared with children without asthma (controls); the study looks at whether the parents
have a history of asthma (risk factor).

A cross-sectional study collects data on both the exposure and outcome at a single point in time;
study participants are not followed longitudinally. The prevalence of exposure in patients with
and without the outcome can be generated with cross-sectional studies; however, causation
cannot be ascertained. An example of a cross-sectional study is assessment of the prevalence of
long-term COVID-19 symptoms in children at a given time.

In an ecologic study, the outcome assessment is made at a population level rather than an
individual level. For example, a study may compare the outcome rates between populations in 2
different geographic locations with different exposures.
American Academy of Pediatrics 574
PREP® Self-Assessment PREPSA 2023

In the hierarchy of levels of evidence, systematic reviews carry the highest level. A systematic
review is a
comprehensive summary of all available evidence on a specific subject. It requires a rigorous
process, which involves the inclusion of studies based on defined eligibility criteria, incorporates
critical appraisal, and uses statistical testing through meta-analysis.

Meta-analysis combines the results of several studies. This method is used to synthesize the best
available evidence using a clear and transparent approach. It considers the strengths as well as
weaknesses of each study, taking into account populations and interventions, to assess specific
outcomes.

PREP Pearls
• Clinical research is either experimental or observational. Experimental research includes
randomized prospective trials (eg, drug or vaccine clinical trials). Observational research
includes cross-sectional, case-control, and cohort studies.
• In a cohort study, the measurement of exposure and outcome is made over time
(longitudinal), whereas in a crosssectional study it is made at a specific point of time.
• A cohort study begins with exposure to a risk factor and observation for the outcome of
interest is made over time. A case-control study begins with cases (with outcome) and
controls (without outcomes) who are then assessed for the presence of the defined risk
factors/exposure.

ABP Content Specifications(s)


• Understand the uses and limitations of cross-sectional and longitudinal studies
• Understand the uses and limitations of systematic review and meta-analysis

Suggested Readings
• Hammill BG. Observational study designs. In: Lopes RD, Harrington RA. eds.
Understanding Clinical Research. McGraw Hill; 2013: chap 12.
• Hulley SB. Cummings SR, Browner WS, Grady DG. Newman TB. In: Designing
Clinical Research, 4th ed. Lippincott Williams & Wilkins; 2013: chap 7.
• Johnson SL. A question of time: Cross-sectional versus longitudinal study designs.
Pediatr Rev. 2010;31(6):250-251. doi:10.1542/pir.31.6.250.
• LeBrun DG, Kocher MS, Baldwin KD, Patel NM. How often are study design and level
of evidence misreported in the pediatric orthopaedic literature? J Pediatr Orthop. 2020;
40(5):e385-e389. doi: 10.1097/BPO.0000000000001470.
• Mason A. Clinical drug trials. Pediatr Rev. 2018;39(3):150-152. doi:10.1542/pir.2016-
0230.

American Academy of Pediatrics 575


PREP® Self-Assessment PREPSA 2023

Question 168
A 2-hour-old neonate born at 36 4/7 weeks’ gestation is examined in the newborn nursery. She
was born to a 28year-old gravida 2, para 1 woman in a normal spontaneous vaginal delivery.
Maternal prenatal testing is negative for group B Streptococcus, and rupture of membranes
occurred 20 hours before delivery. The mother had a temperature of 39.6°C measured 8 hours
before delivery and fetal tachycardia was noted at that time. The neonate has a temperature of
37.1°C, heart rate of 130 beats/min, and respiratory rate of 48 breaths/min. She appears vigorous
with a good cry and normal tone. The remainder of her physical examination findings are
normal. The mother requests early discharge.

Of the following, the BEST next management step for this neonate is to
A. consider early discharge if the neonate's vital signs are stable with next day follow-up
B. observe closely for signs of sepsis for at least 48 hours
C. obtain a complete blood cell count and C-reactive protein level, and perform blood
culture
D. obtain a complete blood cell count, perform blood culture, and start empiric antibiotics

American Academy of Pediatrics 576


PREP® Self-Assessment PREPSA 2023

Correct Answer: D
The neonate in the vignette is at risk for sepsis due to a combination of factors, late preterm
gestation, maternal fever, and prolonged rupture of membranes (PROM, >18 hours). The best
next step in management is to obtain a complete blood cell count, perform a blood culture, and
start intravenous ampicillin and gentamicin treatment. Early discharge is not the best option
because signs of sepsis may be delayed until 72 hours after birth. Close observation for 48 hours
without empiric antibiotics would not be a good choice given this neonate’s multiple risk factors
for sepsis. Although normal laboratory evaluation may seem reassuring, the white blood cell
counts and inflammatory markers may not be a reliable indicator of sepsis in the first 24 to 48
hours after birth.

Although the incidence of early-onset sepsis (EOS) has decreased over the past several years
with the use of intrapartum antibiotics, the morbidity and mortality of affected neonates remains
high. Clinicians must weigh the risk of delayed treatment of neonatal sepsis with that of
unnecessary antibiotic use.

Risk factors for EOS include prematurity, PROM, maternal colonization with group B
Streptococcus (GBS), and evidence of maternal chorioamnionitis. A suspected intra-amniotic
infection is defined by the American College of Obstetricians and Gynecologists as maternal
fever (>39°C) and at least 1 of the following: maternal leukocytosis, purulent cervical discharge,
and fetal tachycardia. If the mother meets the criteria for suspected intra-amniotic infection,
laboratory evaluation and empiric treatment with antibiotics are recommended for neonates with
any of the following:
• Signs of sepsis (eg, ill appearing)
• Gestational age <37 weeks
• Maternal PROM
• Inadequately treated maternal GBS colonization
• Evidence of maternal chorioamnionitis

Risk stratification algorithms are available to clinicians to assist with the at-risk neonate. The
EOS calculator takes into account maternal factors as well as the clinical appearance of the
neonate to guide the decision-making process. Several studies have associated use of the EOS
calculator with reduction in antibiotic use, laboratory testing, and neonatal intensive care unit
admissions, without any missed sepsis cases. The EOS calculator can be found at
https://neonatalsepsiscalculator.kaiserpermanente.org/. For the neonate in the vignette, the EOS
calculator recommends performing a blood culture and starting empiric antibiotics.

American Academy of Pediatrics 577


PREP® Self-Assessment PREPSA 2023

PREP Pearls
• Risk factors for early-onset neonatal sepsis include prematurity, evidence of maternal
chorioamnionitis, untreated maternal colonization with group B Streptococcus, and
prolonged rupture of membranes.
• Risk stratification algorithms may guide clinicians’ management of neonates at risk for
sepsis and reduce the use of unnecessary antibiotics.
• The use of intrapartum antibiotics has reduced the incidence of early-onset neonatal
sepsis.

ABP Content Specifications(s)


• Plan the management of a neonate whose mother is febrile at the time of delivery

Suggested Readings
• Jan AI, Ramanathan R, Cayabyab RG. Chorioamnionitis and management of
asymptomatic infants =35 weeks without empiric antibiotics [erratum in: Pediatrics.
2017;140(4)]. Pediatrics. 2017;140(1):e20162744. doi:10.1542/peds.2016-
• Klitzman MD, Jafri FW, Scott EK, Thomas AE, Engle WA. Implementation of early
onset sepsis calculator safely decreases antibiotic use and laboratory testing in newborns
born to mothers with chorioamnionitis. Pediatrics. 2020;146(1 MeetingAbstract):243-
244. doi:10.1542/peds.146.1_MeetingAbstract.243-a.
• Kojaoghlanian T. The newborn at risk of infection. In: McInerny TK, Adam HM,
Campbell DE, DeWitt TG, Foy JM, Kamat DM, eds. American Academy of Pediatrics
Textbook of Pediatric Care. American Academy of Pediatrics; 2021:chap 102. Accessed
September 1, 2022. Pediatric Care Online.
• Puopolo KM, Benitz WE, Zaoutis TE, AAP Committee on Fetus and Newborn, AAP
Committee on Infectious Diseases. Management of neonates born at =35 0/7 weeks’
gestation with suspected or proven early-onset bacterial sepsis. Pediatrics.
2018;142(6):e20182894. doi:10.1542/peds.2018-2894.

American Academy of Pediatrics 578


PREP® Self-Assessment PREPSA 2023

Question 169
A 5-year-old boy is seen in the office for a health supervision visit. His parents have concerns
about his behavior since starting kindergarten. He was previously cared for by a nanny who had
no behavioral concerns. His teachers report that he does not follow directions, grabs toys from
his classmates, and has frequent emotional outbursts. His behavior has worsened at home since
starting school. The boy’s parents have questions about how to most effectively control his
behavior.

Of the following, the BEST recommendation for the boy’s parents is to


A. offer tasty treats for good behavior
B. praise him for good behavior
C. take away his favorite toy for bad behavior
D. verbally reprimand him for bad behavior

American Academy of Pediatrics 579


PREP® Self-Assessment PREPSA 2023

Correct Answers: B
Providing positive praise for good behavior is the most effective discipline technique to
recommend to parents, particularly for preschool and elementary school-aged children. The boy
in the vignette just entered kindergarten and is learning to socialize with other children; it will
take time for him to understand how to interact appropriately in a group setting. Offering specific
and timely praise for targeted behaviors through a graduated token economy, a reward system in
which children earn tokens that can be exchanged for bigger rewards, will help sustain behavior
change. A sticker chart is an example of a token economy method effective for younger children.

Discipline is meant to teach acceptable behavior using age and developmentally appropriate
methods that encourage self-regulation, safety, increasing independence, and responsibility.
Effective discipline (Item C169) requires a safe, positive, respectful, nurturing, and consistent
learning environment with intentional role modeling to teach and strengthen behaviors. When an
undesirable behavior occurs, caregivers may use other discipline strategies such as ignoring the
misbehavior, redirection, explaining limits, allowing natural consequences, withdrawing
privileges, and time out. These discipline strategies are most successful in the context of a
positive, nurturing relationship between the caregiver and child, and may be used in combination
to stimulate desirable behavior.

Item C169: Effective discipline


To be effective, discipline needs to be:
• given by an adult with an affective bond to the child;
• consistent, close to the behaviour needing change;
• perceived as ‘fair’ by the child;
• developmentally and temperamentally appropriate; and
• self-enhancing, ie, ultimately leading to self-discipline.
The physician can promote effective discipline through evaluation, anticipatory guidance and
counselling.

Providing food as a reward (or withholding it for punishment) is not recommended for
behavioral management, as this can undermine healthy eating habits and create unhealthy
associations with food.

Discipline is not punishment; punishment is the utilization of a negative stimulus, such as verbal
reprimand or taking away a favorite toy, to modify undesirable behavior. Behavioral change is
more effectively achieved by encouraging the child to earn time with a favorite toy with good
behavior than the negative stimulus of creating fear that it will be taken away. During health
supervision visits, parents should be provided anticipatory guidance regarding developmentally
appropriate behaviors and offered a variety of appropriate discipline strategies according to
Bright Futures.

PREP Pearls
• Providing positive praise after good behavior is the most effective discipline technique.
• Effective discipline requires a safe, positive, respectful, nurturing, and consistent learning
environment with intentional role modeling.
American Academy of Pediatrics 580
PREP® Self-Assessment PREPSA 2023

• Discipline is not punishment; punishment is the utilization of a negative stimulus, such as


verbal reprimand or taking away a favorite toy, to modify undesirable behavior.

ABP Content Specifications(s)


• Advise parents regarding appropriate discipline and limit-setting for children of various
ages

Suggested Readings
• Hagan JF, Shaw JS, Duncan PM, eds. Bright Futures: Guidelines for Health Supervision
of Infants, Children, and Adolescents. 4th ed. American Academy of Pediatrics; 2017.
• Nieman P, Shea S; Canadian Paediatric Society, Community Paediatrics Committee.
Effective discipline for children. Paediatr Child Health. 2004;9(1):37-41.
doi:10.1093/pch/9.1.37.
• Pipan ME, Blum NJ. Basics of child behavior and primary care management of common
behavioral problems. In: Voigt RG, Macias MM, Myers SM, Tapia CD, eds. American
Academy of Pediatrics Developmental and Behavioral Pediatrics. American Academy of
Pediatrics; 2018:91-110.
• Schooler SJ. Parental monitoring and discipline in middle childhood. Pediatr Rev.
2009;30(9):366-367. doi:10.1542/pir.30-9-366.
• Sege RD, Siegel BS. Effective discipline to raise healthy children. Pediatrics.
2018;142(6):e2018112. doi:10.1542/peds.2018-3112.

American Academy of Pediatrics 581


PREP® Self-Assessment PREPSA 2023

Question 170
A 27-hour-old neonate in the newborn nursery was noted to have a malformation of her right ear.
She was born at 38 weeks’ gestation. The pregnancy was complicated by a history of maternal
type diabetes that was not well controlled. The neonate’s physical examination findings are
significant only for the appearance of her right ear (Item Q170).

Of the following, the BEST next step in the evaluation of this neonate is
A. an auditory brainstem response test
B. a chromosomal microarray
C. renal ultrasonography
D. a temporal bone computed tomography scan

American Academy of Pediatrics 582


PREP® Self-Assessment PREPSA 2023

Correct Answer: A
The neonate in the vignette has isolated right microtia, a malformation of the external ear.
Microtia commonly affects both the external ear canal and middle ear, so hearing can be
affected. Thus, the best next step is to evaluate this neonate’s hearing with an auditory brainstem
response test. This test is recommended soon after birth to evaluate inner ear function.
A chromosomal microarray is not indicated for isolated microtia. Although microtia occurs in the
context of many genetic syndromes (eg, Goldenhar, Treacher-Collins), the neonate in the
vignette does not have any associated phenotypic features. Renal ultrasonography is indicated in
the evaluation of children with microtia (in contrast to children with preauricular skin lesions and
other external ear deformities) to evaluate for associated anomalies, but is a lower priority than a
hearing evaluation. A temporal bone computed tomography scan is not indicated in the
evaluation of young children with microtia.

Microtia describes a malformation of the external ear. Anotia is a complete absence of the
external ear structures. Microtia usually occurs unilaterally and as an isolated congenital
anomaly, but can occur as part of a genetic syndrome (eg, Goldenhar and Treacher-Collins
syndromes). The prevalence of microtia varies widely geographically and is estimated at 0.83-
17.4 per 10,000 births. It is more common in males. There is an increased incidence of microtia
in the offspring of women with a history of diabetes before pregnancy.

In contrast to external ear deformities, microtia and anotia are external ear malformations and do
not improve over time. Because external ear deformities may respond to nonsurgical
interventions, early referral to a specialist (eg, pediatric otolaryngologist, plastic surgeon) is
recommended.

Treatment of microtia is aimed at hearing restoration and external structure reconstruction.


Surgical correction of microtia most often uses the child’s rib bones, which need to mature
before surgery is undertaken. Because the construction produces an adult-sized external ear, it is
often delayed until the child is 6 to 10 years old. Because malformation of the external ear is
associated with an increased incidence of mood disorders, reconstruction is an important
contributor to long-term well-being.

PREP Pearls
• Neonates with microtia, a malformation of the external ear, should have their hearing
evaluated with an auditory brainstem response test soon after birth.
• Microtia is usually an isolated congenital anomaly. Goldenhar and Treacher-Collins are
the most common associated syndromes.
• Treatment of microtia consists of hearing restoration and external structure
reconstruction.

ABP Content Specifications(s)


• Recognize disorders commonly associated with malformed external and middle ears

American Academy of Pediatrics 583


PREP® Self-Assessment PREPSA 2023

Suggested Readings
• Bhatti SL, Daly LT, Mejia M, Perlyn C. Ear abnormalities. Pediatr Rev. 2021;42(4):180-
188. doi:10.1542/pir.2019-0167.
• Centers for Disease Control and Prevention. 4.4 Congenital Malformations of the Ear
Microtia: Anotia (Q16.0, Q17.2). Accessed September 1, 2022.
https://www.cdc.gov/ncbddd/birthdefects/surveillancemanual/chapters/chapter4/chapter4-
4.html.
• Luquetti DV, Heike CL, Hing AV, Cunningham ML, Cox TC. Microtia: epidemiology
and genetics. Am J Med Genet A. 2012;158A(1):124-139. doi:10.1002/ajmg.a.34352.

American Academy of Pediatrics 584


PREP® Self-Assessment PREPSA 2023

Question 171
A 4-week-old infant is seen for a health supervision visit. His parents are concerned about
excessive crying. He sleeps well during the day, waking to feed every 3 hours, and generally has
a good temperament, but most nights he cries for 3 to 4 hours even after being fed and changed.
Afterward, he can be consoled and sleeps well. He is not gassy and does not spit up or arch his
back. His urine and stool output is normal in appearance, quantity, and frequency. The infant was
born full term via normal vaginal delivery. He has no significant medical history. His vital signs
are normal for age, growth parameters are all at the 50th percentile with normal weight gain, and
physical examination findings are normal. His parents ask when they can expect his symptoms to
improve.

Of the following, the BEST response is that this should occur by age
A. 2 months
B. 4 months
C. 6 months
D. 8 months

American Academy of Pediatrics 585


PREP® Self-Assessment PREPSA 2023

Correct Answers: B
The infant in the vignette most likely has colic. His parents can be reassured that it should
improve by the time he is 4 months of age.

Crying is a common concern for parents of young infants and understanding the normal duration
of crying can be reassuring. The normal duration of crying in the first 6 weeks after birth is about
2 to 3 hours per day. This decreases to about 1 to 2 hours per day at 10 to 12 weeks of age (Item
C171A). Healthy infants who cry more than 3 hours a day may be experiencing colic. The Rome
IV clinical research diagnostic criteria for colic require that infants must be younger than 5
months at the start and end of symptoms and that the crying must last at least 3 hours per day for
at least 3 days of a week. The full Rome IV diagnostic criteria for colic are shown in Item
C171B. Excessive crying beyond age 5 months should alert families and physicians to
investigate for other underlying medical causes.

Pediatricians can help families cope with crying by validating feelings of frustration, giving
families permission to take breaks, and reassuring them that crying in and of itself is not harmful
to infants.

Rome IV diagnostic criteria for colic


• An infant who is less than 5 months of age when the symptoms start and stop
• Recurrent and prolonged periods of infant crying, fussing, or irritability reported by caregivers
that occur without obvious cause and cannot be prevented or resolved by caregivers
• No evidence of infant failure to thrive, fever, or illness

PREP Pearls
• Infants cry about 2 to 3 hours per day for about the first 6 weeks after birth; it decreases
to about 1 to 2 hours per day by about 10 to 12 weeks of age.
• The diagnostic criteria for colic require that infants must be younger than 5 months at the
start and end of symptoms and that the crying must last at least 3 hours per day for at
least 3 days per week.
• Colic usually resolves by 4 months of age.
American Academy of Pediatrics 586
PREP® Self-Assessment PREPSA 2023

ABP Content Specifications(s)


• Recognize the frequency of crying in infants of various ages

Suggested Readings
• Baum RA. Colic. In: McInerny TK, Adam HM, Campbell DE, DeWitt TG, Foy JM,
Kamat DM, eds. American Academy of Pediatrics Textbook of Pediatric Care. American
Academy of Pediatrics; 2021:chap 231. Accessed September 1, 2022. Pediatric Care
Online.
• Cohen GM, Albertini LW. Colic. Pediatr Rev. 2012;33(7):332-333. doi:10.1542/pir.33-7-
332 .
• Wolke DW, Bilgin A, Samara M. Systematic review and meta-analysis: fussing and
crying durations and prevalence of colic in infants. J Pediatr. 2017;185:55-61.
doi:10.1016/j.jpeds.2017.02.020.
• Zeevenhooven J, Knoppen IJN, Benninga MA. The new Rome IV criteria for functional
gastrointestinal disorders in infants and toddlers. Pediatr Gastroenterol Hepatol Nutr.
2017;20(1):1-13. doi:10.5223/pghn.2017.20.1.1.

American Academy of Pediatrics 587


PREP® Self-Assessment PREPSA 2023

Question 172
The state laboratory notifies a pediatrician about a newborn screening test result that is positive
for cystic fibrosis (CF). The report indicates elevated immunoreactive trypsinogen and
identification of 2 mutations recognized as CF disease-causing genes. The neonate is the first
child for this couple, and her mother declined prenatal genetic screening for CF. The neonate
was born at term weighing 3,200 g and has been nursing well. At her 2-week health supervision
visit yesterday she had adequate weight gain and normal physical examination findings, and
there were no concerns expressed by her parents.

Of the following, the BEST next step in this neonate’s management is to


A. inform the neonate’s parents that any future children will have a positive newborn
screening result for cystic fibrosis
B. inform the neonate’s parents that she has cystic fibrosis; no additional testing is required
at this time
C. schedule the neonate for a sweat chloride analysis as soon as possible
D. test both parents for cystic fibrosis before obtaining any additional studies on the neonate

American Academy of Pediatrics 588


PREP® Self-Assessment PREPSA 2023

Correct Answer: C
The neonate in the vignette has a positive screening test result for cystic fibrosis (CF). However,
even with 2 disease-causing CF genes identified, the newborn screening test does not confirm the
diagnosis. Therefore, of the response choices, the best next step in this neonate’s management is
to schedule a sweat chloride analysis as soon as possible. A diagnostic test is required for
confirmation. Typically, a sweat chloride test is used, which measures the amount of chloride in
a sample of sweat stimulated by pilocarpine iontophoresis. The diagnosis is made by
demonstrating abnormal function of the cystic fibrosis transmembrane conductance regulator
(CFTR) protein in the affected infant, and is not dependent on parental gene carriage.

The newborn screen is designed with a high sensitivity to increase the likelihood of identifying
affected individuals. As a result, some who are not affected, including many CF gene carriers,
will screen positive. A very small percentage of neonates with positive newborn screening test
results are confirmed to have a diagnosis of CF (the actual percentage depends on the screening
algorithm used).

Cystic fibrosis has an autosomal recessive pattern of inheritance. When each parent carries an
abnormal CF gene, with every pregnancy there is a 25% chance of having a child affected with
CF (carrying the abnormal gene from each parent), a 25% chance of having a child carrying no
CF gene, and a 50% chance of having a child who is phenotypically normal but carries 1
abnormal gene (from 1 parent). Cystic fibrosis newborn screening results for future children
would be expected to be positive for affected offspring and possibly for offspring who carry 1
abnormal gene but are phenotypically normal. A child who carries no CF gene will have a
negative result on the CF newborn screening test.

Newborn screening for CF may be performed by one of several protocols. In most states,
immunoreactive trypsinogen (IRT) is measured from the blood spot card. An IRT level is
defined as elevated if it falls within the highest percentage of the values (usually the highest 5%
to 6%) for a specific day’s testing. If the IRT is elevated, a genetic panel (determined at the level
of the state laboratory) is performed using the same blood spot card. If 1 or 2 abnormal genes are
identified, the newborn screen is labeled as positive, and a second level (diagnostic) test is
indicated. Specific procedures are defined by each state for contacting the neonate’s health care
professional and/or the parents with this information. A few states use IRT determination alone
as a primary screen and defer genetic testing until after a repeat IRT is performed at about 2
weeks of age. If the second IRT is elevated, the genetic test panel is performed, and the screen is
deemed positive if 1 or 2 genes are detected. Very few states use broader and more sophisticated
genetic testing after the initial positive IRT. Regardless of the sensitivity of the screening test, a
diagnostic test is required for confirmation. Because of variability in the sweat chloride values in
the first few days after birth, the test should be performed at least 2 days after birth in an infant
weighing at least 2 kg to decrease likelihood of insufficient sweat stimulation for accurate
measurement of chloride. Premature infants should be at least 36 weeks postconceptual age and
2 kg in weight.

American Academy of Pediatrics 589


PREP® Self-Assessment PREPSA 2023

In addition to sweat chloride testing, genetic testing of the affected neonate independent of the
newborn screen is acceptable as a diagnostic test. The sweat chloride test has the added benefit
of use as a biomarker for CFTR function. It has been used as an outcome measure in studies of
oral medications that affect CFTR function. A decrease in sweat chloride concentration in
children and adolescents with CF taking these medications indicates improvement in CFTR
function and correlates with clinical improvements in lung function and nutritional status.

Newborn screening and more effective treatment methods have greatly increased the survival of
individuals with CF. The median age of survival for individuals with CF in the early 1990s was
below 30 years and has risen progressively to about 45 years as of 2019 (Item C172A). The
number of children with CF has remained fairly stable from year to year, while the number
surviving into adulthood has grown steadily. Item C172B demonstrates that by 2010, adults
made up more than 50% of individuals with CF and the proportion of affected adults continued
to increase. In the current era, highly effective modulator drugs benefit approximately 90% of CF
genotypes. The median survival age and the percentage of adults among patients with CF is
expected to increase at even higher rates. Most neonates with CF born today can expect to live
healthy and productive lives well into adulthood. However, the benefit of these new medications
is genotype-specific, and there are still some genotypes for which no modulator drugs have been
found to be effective.

PREP Pearls
• A positive newborn screen for cystic fibrosis suggests the diagnosis but must be
confirmed by a diagnostic test, preferably sweat chloride analysis.
• With an autosomal recessive inheritance pattern, when each parent carries 1 copy of an
abnormal cystic fibrosis gene, with every pregnancy there is a 25% chance of giving birth
to an affected child.
• More than 50% of individuals living with cystic fibrosis are adults, with the percentage
increasing annually.

ABP Content Specifications(s)


• Recognize the importance of planning for survival into adulthood for patients with cystic
fibrosis
• Understand the inheritance pattern associated with cystic fibrosis
• Plan the appropriate diagnostic evaluation and management of cystic fibrosis

American Academy of Pediatrics 590


PREP® Self-Assessment PREPSA 2023

Suggested Readings
• Farrell PM, White TB, Ren CL, et al. Diagnosis of cystic fibrosis: consensus guidelines
from the Cystic Fibrosis Foundation. J Pediatr. 2017;181S:S4-S15.e1.
doi:10.1016/j.jpeds.2016.09.064.
• Goetz D and Ren CL. Review of cystic fibrosis. Pediatr Ann. 2019;48(4):e154-e161.
doi:10.3928/19382359-20190327-01
• Joshi D, Ehrhardt A, Hong JS, Sorscher EJ. Cystic fibrosis precision therapeutics:
emerging considerations. Pediatr Pulmonol. 2019 ov;54 Suppl 3(Suppl 3):S13-S17.
doi:10.1002/ppul.24547.
• Sosnay PR and Farrell P. Challenges in cystic fibrosis newborn screening and
recommendations for primary care physicians. Pediatrics. 2015;136(6):1181-1184.
doi:10.1542/peds.2015-3490.
• Willey-Courand DB, Marshall BC. Cystic fibrosis. In: McInerny TK, Adam HM,
Campbell DE, DeWitt TG, Foy JM, Kamat DM, eds. American Academy of Pediatrics
Textbook of Pediatric Care. American Academy of Pediatrics; 2021:chap 239. Accessed
September 1, 2022. Pediatric Care Online.

American Academy of Pediatrics 591


PREP® Self-Assessment PREPSA 2023

Question 173
A 16-year-old adolescent girl who received a kidney transplant 2 years ago is admitted to the
hospital with temperatures up to 38.6°C at home and decreased urine output. She had been doing
well on immunosuppression medication, including tacrolimus.

On physical examination, the girl has a temperature of 37.8°C, blood pressure of 135/92 mm Hg,
heart rate of 92 beats/min, respiratory rate of 15 breaths/min, and oxygen saturation of 94% in
room air. She is non-toxic in appearance. Lower extremity edema is noted. The remainder of her
physical examination findings, including her lungs and heart, are normal.

Laboratory evaluation is significant for a tacrolimus level of 2.0 ng/ mL (goal, 5-15 ng/mL) and
a creatinine level of 2.1 mg/dL (185.6 µmol/L); 4 months earlier, her creatinine level was 1.1
mg/dL (97.2 µmol/L).

The adolescent’s mother is surprised by her low tacrolimus level as no recent dosage changes
have been made and she monitors her daughter while taking her medications. The girl states that
it has been a difficult few months, and she has struggled with depression. Her mood has been
improving since she began taking a natural remedy recommended by a friend.

Of the following, the MOST likely natural remedy this adolescent began taking is
A. ginkgo biloba
B. rosemary
C. saffron
D. St John’s wort

American Academy of Pediatrics 592


PREP® Self-Assessment PREPSA 2023

Correct Answer: D
The adolescent in the vignette most likely started taking St John’s wort, an herb with
antidepressant properties. It affects the cytochrome P-450 mixed function oxidase, which can
increase the metabolism of some medications, including immunosuppressive medications such as
tacrolimus and cyclosporine, leading to decreased drug levels. Ginkgo biloba, rosemary, and
saffron are less likely to affect tacrolimus metabolism.

Complementary and alternative medications, including herbal remedies, are increasing in use.
These supplements are often not reported to clinicians unless young patients and their families
are asked directly. Their use can cause serious drug interactions.

St John’s wort has 2 active components: hyperforin and hypericin. Hyperforin leads to the
increased metabolism of drugs (eg, immunosuppressives, antiretrovirals, and warfarin) via the
cytochrome P450 enzyme. Hypericin affects the P-glycoprotein pathway, which can decrease
digoxin, verapamil, and nifedipine levels. St John’s wort can alter the metabolism of many
medications including oral contraceptive pills, synthetic opioids, and benzodiazepines. It can also
alter pharmacodynamics, leading to serotonin syndrome for children and adolescents taking
selective serotonin reuptake inhibitors.

It is important for clinicians to ask about and discuss newly prescribed medications, over-the-
counter medications, natural remedies, and herbal medications at every patient encounter.

PREP Pearls
• Use of complementary and alternative medicine is increasing and should be discussed
during every clinical encounter.
• Complementary and alternative medicine can cause interactions with drugs such as
antiretrovirals and transplant medications.

ABP Content Specifications(s)


• Recognize potential interactions between drugs and complementary therapies

American Academy of Pediatrics 593


PREP® Self-Assessment PREPSA 2023

Suggested Readings
• Dasgupta A, Wahed A. Effect of herbal supplements on clinical laboratory test results.
Clin Chem Immunol Lab Qual Control. 2014:449-459. doi:10.1016/B978-0-12-407821-
5.00025-5.
• Gilmour J, Harrison C, Asadi L, Cohen M, Vohra S. Natural health product-drug
interactions: evolving responsibilities to take complementary and alternative medicine
into account. Pediatrics. 2011;128(suppl 4):S155-S160. doi:10.1542/peds.2010-2720C.
• Larsen G, Cash J. Drug interactions and adverse effects. In: McInerny TK, Adam HM,
Campbell DE, DeWitt TG, Foy JM, Kamat DM, eds. American Academy of Pediatrics
Textbook of Pediatric Care. American Academy of Pediatrics; 2021:chap 246. Accessed
September 1, 2022. Pediatric Care Online.
• St. John’s wort. National Center for Complementary and Integrative Health. Updated
October 2020. Accessed September 1, 2022. https://www.nccih.nih.gov/health/st-johns-
wort.

American Academy of Pediatrics 594


PREP® Self-Assessment PREPSA 2023

Question 174
A 7-year-old boy is evaluated in the outpatient clinic for a 1-year history of hematochezia and
parental concern for a hemorrhoid. He has soft, formed stools daily with bright red blood on the
outside of the stool. He is otherwise healthy. Physical examination reveals a well-appearing child
with normal weight and height for age. His abdomen is soft and nontender. Rectal examination
demonstrates a normal-appearing anus with a 1-cm, palpable, mobile, nontender mass in the
rectum.

Colonoscopy is performed and 10 pedunculated polyps (0.5-1 cm in diameter) are noted to be


scattered throughout the colon (Item Q174). All visualized polyps are successfully removed and
retrieved; each polyp demonstrates pathology consistent with a juvenile polyp.

ITEM Q174: Polyps seen on the colonoscopy of the child described in the vignette.
Courtesy of J. Sullivan

Of the following, the BEST next step in the management of this child is
A. administration of a nonsteroidal anti-inflammatory drug
B. referral for genetic testing
C. referral to a pediatric colorectal surgeon
D. repeat colonoscopy only if hematochezia recurs

American Academy of Pediatrics 595


PREP® Self-Assessment PREPSA 2023

Correct Answer: B
The child in the vignette has clinical features concerning for juvenile polyposis syndrome (JPS),
evidenced by the presence of 5 or more juvenile polyps in the colon. Because JPS is associated
with an increased risk for colorectal cancer, testing and referral to genetics for counseling is
recommended for the child and other potentially affected family members.

Solitary juvenile polyps are the most common polyps removed in children, with a peak incidence
between ages 2 to 6 years. Juvenile polyps have a smooth, friable surface that can result in
intestinal bleeding. Most of these polyps are pedunculated with a narrow stalk (Item C174A).
The most common presentation is painless rectal bleeding, which can be significant at times.
Treatment is removal of the polyp; surveillance colonoscopy is not indicated. In the early stages
of JPS, a child may present with a solitary juvenile polyp; if symptoms recur, repeat colonoscopy
would be warranted.

Several polyposis syndromes can occur in children; the most common syndromes include
familial adenomatous polyposis (FAP), JPS, Peutz-Jeghers syndrome, and Hamartoma syndrome
(Item C174B). When a polyposis syndrome is suspected, genetic counseling should be pursued.
Because this child had 5 or more juvenile polyps present, surveillance colonoscopy even in
asymptomatic individuals should be considered. Referral to a colorectal surgeon is indicated for
patients with FAP who have a significant polyp burden or if there is concern for colorectal
cancer, but would not be indicated for this child with JPS. Treatment with a nonsteroidal anti-
inflammatory drug may be recommended for those with FAP, however would not be indicated
for the treatment of juvenile polyps.

PREP Pearls
• For a child with fewer than 5 juvenile colonic polyps, endoscopic removal is
recommended; repeat colonoscopy is not indicated unless symptoms recur.
• Children with 5 or more juvenile polyps should be referred for juvenile polyposis
syndrome genetic testing.
• Pediatric polyposis syndromes often require monitoring for extraintestinal malignancies;
the involvement of genetic counselors or cancer risk teams is essential.

ABP Content Specifications(s)


• Recognize the significance of a solitary juvenile polyp
• Recognize the clinical features associated with juvenile polyposis

American Academy of Pediatrics 596


PREP® Self-Assessment PREPSA 2023

Suggested Readings
• Attard TM, Burke CA, Hyer W, et al. ACG clinical report and recommendations on
transition of care in children and adolescents with hereditary polyposis syndromes. Am J
Gastroenterol. 2021;116(4):638-646. doi:10.14309/ajg.0000000000001140.
• Larsen Haidle J, Howe JR. Juvenile polyposis syndrome. GeneReviews. [Internet].
Seattle, WA: University of Washington, Seattle; 2017.
https://www.ncbi.nlm.nih.gov/books/NBK1469.
• MacFarland SP, Zelley K, Katona BW, Wilkins BJ, Brodeur GM, Mamula P.
Gastrointestinal polyposis in pediatric patients. J Pediatr Gastroenterol Nutr.
2019;69(3):273-280. doi:10.1097/MPG.0000000000002421.

American Academy of Pediatrics 597


PREP® Self-Assessment PREPSA 2023

Question 175
An 18-month-old boy is brought to the clinic for a routine health supervision visit. He takes a
few steps on his own but cannot walk without holding onto someone or something. He tries to
eat with a spoon and scribbles. He points to show the physician something interesting, puts hands
out to be washed, looks at a few pages in a book with the reader, and helps with dressing by
pushing his arm through a sleeve. The boy tries to say 3 or more words besides “mama” and
“dada” and follows 1-step directions without any gestures.

Of the following, this boy most likely has developmental DELAY in


A. fine motor skills
B. gross motor skills
C. language-communication skills
D. social-emotional skills

American Academy of Pediatrics 598


PREP® Self-Assessment PREPSA 2023

Correct Answer: B
The boy in the vignette is 18 months old. At this age, expected gross motor skills include
walking without holding onto anyone or anything and climbing on and off a couch or chair
without help. As the boy can take a few steps but cannot walk without holding onto anyone or
anything, he most likely has delayed gross motor skills. His skills in the other developmental
domains are appropriate for his age.

The domains of development and their relevant skills are described in Item C175A. Item C175B
lists the gross motor, fine motor, cognitive, social-emotional, and language-communication
developmental milestones expected for 15- to 24-month-old children.

PREP Pearls
• The 5 domains of development include fine motor, gross motor, language, cognitive, and
social-emotional.
• At 18 months of age, expected gross motor skills include creeping down stairs, walking
without holding onto anything, seating oneself in a small chair, and throwing a ball while
standing.

ABP Content Specifications(s)


• Evaluate the motor developmental progress/status of a child at 18 months of age,
including recognition of abnormalities
• Evaluate the cognitive and behavioral developmental progress/status of a child at 18
months of age

Suggested Readings
• Feigelman S. The first year. In: Kliegman RM, Stanton BF, St Geme JW III, Schor NF,
eds. Nelson Textbook of Pediatrics. 21st ed. Elsevier; 2020.
• Scharf RJ, Scharf GJ, Stroustrup A. Developmental milestones. Pediatr Rev.
2016;37(1):25-38. doi:10.1542/pir.20140103.
• Zubler JM, Wiggins LD, Macias MM. Evidence-informed milestones for developmental
surveillance tools. Pediatrics. 2022;149(3):e2021052138. doi:10.1542/peds.2021-052138.

American Academy of Pediatrics 599


PREP® Self-Assessment PREPSA 2023

Question 176
A 14-year-old adolescent girl is brought to the emergency department for difficulty ambulating.
One week ago, she developed leg pain, primarily in her thighs and feet. Her pain progressed to
difficulty walking up and down stairs. Over the past 24 hours she has had difficulty standing and
walking, and she feels that her arms have become “clumsy.” She denies back pain, trauma, or
bowel or bladder changes. Three weeks ago, she had a mild upper respiratory infection that
resolved with supportive care.

The girl is anxious, alert, and oriented to person, place, and time. Her cranial nerve examination
findings are normal. Motor examination shows hypotonia in her bilateral lower extremities. She
is able to move her legs across the bed but not lift them against gravity, and she cannot plantar
flex or dorsiflex her ankles. Her upper extremities have normal proximal strength, with mild
distal weakness. Deep tendon reflexes are 1+ in the upper extremities, and absent in the lower
extremities. Her toes are mute to plantar stimulation. Sensation is intact; there is no sensory
deficit level. Finger-to-nose testing is normal. She cannot ambulate. The remainder of her
physical examination findings are normal.

Magnetic resonance imaging of the spine is normal.


Lumbar puncture with cerebrospinal fluid analysis shows the following data:
Laboratory Test Result
Protein 85 mg/dL (reference range,15-60 mg/dL)
Glucose 70 mg/dL (reference range, 40-80 mg/dL)
White blood cell count 8 cells/µL (reference range, 0-5 cells/µL)
Red blood cell count 0 cells/µL (reference range, 0 cells/µL)

Of the following, the BEST next management step for this adolescent is administration of
intravenous
A. botulism antitoxin
B. ceftriaxone
C. immune globulin
D. methylprednisolone

American Academy of Pediatrics 600


PREP® Self-Assessment PREPSA 2023

Correct Answer: C
The adolescent in the vignette has ascending flaccid paralysis and areflexia without evidence of
encephalopathy or myelopathy. This presentation is consistent with Guillain-Barré syndrome
(GBS). The diagnosis of GBS is clinical. While diagnostic testing can be used to exclude
alternative etiologies or provide additional supportive data, testing results can be normal,
especially early in its clinical course. The foundation of treatment is supportive, with close
monitoring for evolution to impending respiratory failure. Of the response choices,
administration of intravenous immune globulin (IVIG) is the best next management step for this
adolescent because of the severity and ongoing progression of her symptoms. Intravenous
immune globulin is an immunomodulating therapy that can reduce hospital length of stay and
time to ambulation in individuals with GBS.

Guillain-Barré syndrome is an autoimmune, inflammatory, demyelinating polyneuropathy that


can have an acutesubacute presentation characterized by ascending symmetric flaccid paralysis
and areflexia. The majority of cases occur in a post-infectious setting, classically associated with
Campylobacter jejuni. There are many subtypes of GBS; the most common form seen in North
America is acute inflammatory demyelinating radiculoneuropathy (AIDP). The Miller-Fisher
variant, a subtype characterized by the triad of ataxia, areflexia, and ophthalmoplegia, can be
mistaken for an acute cerebellar ataxia.

While GBS symptoms are typically bilateral and symmetric, the distribution can be patchy,
especially early in the disease course. Pain is a common early sign and can lead to diagnostic
confusion. Cranial nerve involvement can occur in up to 45% of children with GBS, although
pupillary involvement is rare and should prompt alternate diagnostic considerations. Autonomic
dysfunction and respiratory failure may occur; vigilance is necessary to recognize and treat them
promptly. Sphincter dysfunction is a less common feature; the presence of a sensory level on
physical examination should prompt consideration of spinal cord pathology such as transverse
myelitis or cord compression. The symptom nadir is usually reached within 2 to 4 weeks.

Guillain-Barré syndrome is a clinical diagnosis. Testing is not required to initiate treatment but
can be used to support the diagnosis or exclude alternate diagnostic considerations. Magnetic
resonance imaging (MRI) of the spine is the preferred imaging modality to exclude spinal cord
pathology. In individuals with GBS, findings on MRI with gadolinium may range from normal to
enhancement of the nerve roots. Cerebrospinal fluid (CSF) analysis, in cases of GBS, classically
demonstrates an increased total protein level with a normal white blood cell (WBC) count; CSF
leukocytosis should prompt consideration of alternate diagnoses. Electrophysiology studies (eg,
electromyography [EMG] or nerve conduction studies [NCS]) can be normal early in the disease
course and may not show abnormalities until 2 to 3 weeks after symptom onset.
Electromyography and NCS can be helpful in prognostication and distinguishing among variants.

In the absence of prospective, placebo-controlled, randomized controlled trials in children, the


treatment of GBS is guided by consensus opinion and extrapolation from adult studies.
Administration of IVIG is shown to reduce hospital length of stay and time to ambulation.
Plasmapheresis may be used in severe cases. No benefit of combining the 2 treatments has been
American Academy of Pediatrics 601
PREP® Self-Assessment PREPSA 2023

shown in adult populations. Methylprednisolone has not been shown to be effective in the
treatment of GBS. Ceftriaxone or other antibiotics are not indicated for the adolescent in the
vignette, whose CSF analysis does not indicate infection. Botulism antitoxin is used in the
treatment of botulism. Unlike the clinical presentation of the adolescent in the vignette, botulism
typically presents with a descending paralysis, usually starting with cranial nerve involvement.

PREP Pearls
• Guillain-Barré syndrome is an autoimmune, inflammatory, demyelinating
polyneuropathy that can have an acutesubacute presentation characterized by ascending
symmetric flaccid paralysis and areflexia.
• Guillain-Barré syndrome is a clinical diagnosis; testing is not required to initiate
treatment. While testing can be used to exclude alternative etiologies or provide
additional supportive data, results can be normal, especially early in its course.
• Intravenous immune globulin is an immunomodulating therapy that can shorten hospital
length of stay and time to ambulation for individuals with Guillain-Barré syndrome.

ABP Content Specifications(s)


• Recognize the clinical findings associated with Guillain-Barré syndrome and the
progression of disease
• Plan the appropriate diagnostic evaluation of Guillain-Barré syndrome, and manage
appropriately Recognize risk factors associated with Guillain-Barré syndrome

Suggested Readings
• Chung A, Deimling M. Guillain-Barré syndrome. Pediatr Rev. 2018;39(1):53-54.
doi:10.1542/pir.2017-0189.
• Evidence-based guideline: intravenous immunoglobulin in the treatment of
neuromuscular disorders: report of the Therapeutics and Technology Assessment
Subcommittee of the American Academy of Neurology. Neurology. 2012;78(13):1009-
1015. doi:10.1212/WNL.0b013e31824de293.
• Sivaswamy L. Guillain-Barré syndrome. In: McInerny TK, Adam HM, Campbell DE,
DeWitt TG, Foy JM, Kamat DM, eds.
• American Academy of Pediatrics Textbook of Pediatric Care. American Academy of
Pediatrics; 2021:chap 260. Accessed September 1, 2022. Pediatric Care Online.
• Yuki N, Hartung H. Guillain-Barré syndrome. N Engl J Med. 2012;366(24):2294-2304.
doi:10.1056/NEJMra1114525.

American Academy of Pediatrics 602


PREP® Self-Assessment PREPSA 2023

Question 177
A new test is being developed to determine the risk of developing type 1 diabetes in first-degree
relatives of those known to have type 1 diabetes. One hundred first-degree relatives have the new
test performed on them and are followed for 5 years for the development of diabetes. Fifty first-
degree relatives have a positive test result. After 5 years, 25 first-degree relatives developed type
1 diabetes, all of whom had a positive test result. None of the firstdegree relatives with negative
test results developed diabetes.

Of the following, the positive predictive value of this test is


A. 50%
B. 67%
C. 75%
D. 100%

American Academy of Pediatrics 603


PREP® Self-Assessment PREPSA 2023

Correct Answer: A
For a given diagnostic test, the ability to detect disease varies. Positive predictive value (PPV)
describes the proportion of individuals with a positive test result who have the disease. Stated
another way, PPV describes the probability of disease in a person with a positive test result.
Negative predictive value (NPV) describes the proportion of individuals with a negative test
result who do not have the disease, or the probability of not having the disease when the test
result is negative.

The predictive values of a diagnostic test depend on disease prevalence in the test population. A
higher disease prevalence (pre-test probability) correlates with a higher PPV (post-test
probability).

For the study described in the vignette, the population is first-degree relatives of individuals
known to have type 1 diabetes. The diagnostic test is a marker for risk of developing type 1
diabetes.

A 2 × 2–table can be helpful when calculating sensitivity, specificity, PPV, and NPV:

Developed Diabetes Did Not Develop Diabetes


Positive Test 25 (TP) 25 (FP)
Negative Test 0 (FN) 50(TN)
• TP = True positives
• FP = False positives
• FN - False negatives
• TN = True negatives
• Positive predictive value: TP/(TP+FP)
• Negative predictive value: TN/(FN+TN)

In this case, the PPV is the number of true positives (25) divided by all the positives detected by
the test (25+25), which equals 50%.

The NPV is the number of true negatives (50) divided by all of the negatives detected by the test
(0+50), which equals 100%.

Sensitivity and specificity describe the ability of a test to identify disease. These attributes are
defined by the test itself, independent of the patient population. Sensitivity describes the
proportion of people with a disease who test positive for that disease. Specificity describes the
proportion of people without a disease who test negative for the disease.

For the test in the vignette:


Sensitivity = TP/(TP+FN) = 25/(25+0) = 100%
Specificity = TN/(FP+TN) = 50/(25+50) = 67%

American Academy of Pediatrics 604


PREP® Self-Assessment PREPSA 2023

PREP Pearls
• Positive predictive value is the proportion of individuals with a positive test result who
have the disease (the probability of a disease in a person who has a positive test result).
• Negative predictive value is the proportion of individuals with a negative test result who
do not have the disease (the probability of not having the disease when the test result is
negative).
• Both positive and negative predictive values vary with the prevalence of disease in the
population and are not intrinsic to the test.

ABP Content Specifications(s)


• Understand positive and negative predictive values
• Understand sensitivity and specificity and how to apply them to test results

Suggested Readings
• Akobeng AK. Understanding diagnostic tests 1: sensitivity, specificity and predictive
values. Acta Paediatr. 2007;96(3):338-341. doi:10.1111/j.1651-2227.2006.00180.x.
• Carvajal DN, Rowe PC. Sensitivity, specificity, predictive values, and likelihood ratios.
Pediatr Rev. 2010;31(12):511-513. doi:10.1542/pir.31-12-511.
• Kuzma JW, Bohnenblust SE. Two-sample significance testing, point estimates, and
confidence intervals. In: Basic Statistics for the Health Sciences. 4th ed. McGraw-Hill
Higher Education; 2001:147-149.

American Academy of Pediatrics 605


PREP® Self-Assessment PREPSA 2023

Question 178
A 5-year-old boy with acute lymphoblastic leukemia is admitted to the hospital after 3 days of
diarrhea. He has had 8 small-volume, loose stools with mucus and blood during the past 24
hours. His younger brother has similar symptoms. The family visited a waterpark 2 days before
symptom onset. On physical examination, the boy’s temperature is 39°C, heart rate is 120
beats/min, respiratory rate is 28 breaths/min, blood pressure is 95/55 mm Hg, and oxygen
saturation is 99% in room air. He appears uncomfortable, and his abdomen is slightly tender.
The boy’s absolute neutrophil count today is 500/µL (0.50 × 109/L). A stool culture from 2 days
ago yields a gramnegative rod that is non–lactose fermenting, oxidase negative, and non–
hydrogen sulfide producing.

Of the following, the BEST next management step for this child is
A. intravenous rehydration without antibiotics
B. intravenous rehydration and ceftriaxone
C. oral rehydration and amoxicillin
D. oral rehydration without antibiotics

American Academy of Pediatrics 606


PREP® Self-Assessment PREPSA 2023

Correct Answer: B
The child in the vignette has symptoms of acute severe gastroenteritis, including fevers,
vomiting, and dysentery (diarrhea with blood and mucus). The Gram stain and biochemical test
characteristics on stool culture suggest infection from a Shigella species. The boy is
immunocompromised, with an absolute neutrophil count of 500/µL (0.50 × 109/L). In this
situation, intravenous hydration and empiric treatment with intravenous ceftriaxone are
recommended while awaiting culture and sensitivity data. Oral amoxicillin is not well absorbed
in cases with these clinical findings; therefore, it is not an appropriate treatment option.
Rehydration, preferably oral, is the mainstay of management in an otherwise healthy child with
mild to moderate symptoms; antibiotics would not be recommended in such cases.

Shigella is a gram-negative rod that is non–lactose fermenting, as opposed to Enterobacteriaceae


(eg, Escherichia coli), which ferment lactose. The oxidase test result for Shigella is negative
compared with Pseudomonas, which is oxidase positive. Shigella lacks urease activity and does
not produce hydrogen sulfide (colorless colonies), unlike Salmonella species, which produce
hydrogen sulfide (red-black center).

Shigella is the third most common cause of acute bacterial gastroenteritis in the United States
after nontyphoidal Salmonella and Campylobacter species. Other important causes include
enteric E coli and Yersinia. Shigella affects the large intestine. A small inoculum of only 10 to
100 organisms is required for infection. Transmission occurs through fecal-oral and person-to-
person contact. Symptoms range from loose, watery stools with no fevers to severe symptoms of
mucoid stools, bloody stools, tenesmus, and high temperatures. There is a 10% prevalence of
seizures associated with Shigella infection due to high fevers. The serum white blood cell count
may show an increase in immature neutrophils (elevated bands); however, it is not a consistent
finding. Symptoms depend on the serotype involved; Shigella sonnei primarily leads to watery
diarrhea, whereas Shigella dysenteriae and Shigella flexneri may lead to dysentery.
Gastrointestinal multiplex polymerase chain reaction assays are now routinely used to detect
organisms that cause gastroenteritis, including Shigella. Because this testing does not provide
antibiotic sensitivity information, a stool culture should be obtained for antibiotic susceptibility
testing.

Antibiotic therapy is not indicated for acute gastroenteritis in otherwise healthy children with
mild to moderate disease. In such cases, Shigella infection is managed with correction of fluids,
electrolytes, and early (<12 hours) introduction of feeding. Antibiotics are indicated for a child
with immunocompromised status and severe symptoms to prevent complications, such as
bacteremia. Antibiotic therapy reduces the duration of diarrhea by 3 days, fever by 1 day, and
shedding by 4 to 5 days compared with 4 weeks without treatment. If a sensitive strain is isolated
on culture, oral azithromycin or ciprofloxacin can be used in children who can maintain good
oral intake. Oral trimethoprim-sulfamethoxazole is an alternate option in the case of a sensitive
strain. Antibiotic therapy in children with Shiga toxin–producing E coli (ie, O157:H7) infection
may be associated with hemolytic uremic syndrome (hemolysis, thrombocytopenia, and acute
renal failure after a prodrome of diarrhea). This association has not been demonstrated with
Shiga toxin–producing S dysenteriae.
American Academy of Pediatrics 607
PREP® Self-Assessment PREPSA 2023

PREP Pearls
• Shigella is the third most common cause of acute bacterial gastroenteritis in the United
States after nontyphoidal Salmonella and Campylobacter species.
• The mainstay of management in an otherwise healthy child with mild to moderate
symptoms of Shigella infection is rehydration. Antibiotic therapy is not indicated.
• Antibiotic therapy is indicated for Shigella infection in an immunocompromised child or
a child with severe illness (dysentery and bacteremia). Intravenous ceftriaxone is the
treatment of choice.

ABP Content Specifications(s)


• Recognize the clinical features associated with Shigella infection
• Plan appropriate management for a patient with Shigella infection
• Understand the epidemiology of Shigella species

Suggested Readings
• American Academy of Pediatrics. Shigella infections. In: Kimberlin DW, Barnett ED,
Lynfield R, Sawyer MH, eds. Red Book: 2021–2024 Report of the Committee on
Infectious Diseases. 32nd ed. American Academy of Pediatrics; 2021. Accessed
September 1, 2022. Red Book Online.
• Ashkenazi S, Cleary TG. Shigella species. In: Long SS, ed. Principles and Practice of
Pediatric Infectious Diseases. 4th ed. Elsevier; 2012:819-823.e2.
• Holmes LC. Shigella. Pediatr Rev. 2014;35(6):261-262. doi:10.1542/pir.35-6-261.
• Ochoa TJ, Kosek M. Shigella. In: Feigin and Cherry’s Textbook of Pediatric Infectious
Diseases. 7th ed. Saunders; 2014:1055-1063.
• Ulshen MH. Diarrhea and steatorrhea. In: McInerny TK, Adam HM, Campbell DE,
DeWitt TG, Foy JM, Kamat DM, eds. American Academy of Pediatrics Textbook of
Pediatric Care. American Academy of Pediatrics; 2021:chap 199. Accessed September 1,
2022. Pediatric Care Online.

American Academy of Pediatrics 608


PREP® Self-Assessment PREPSA 2023

Question 179
An 8-month-old boy is seen in the emergency department after falling from his crib. A
cephalohematoma is noted on physical examination. His vital signs are appropriate for age. The
remainder of his physical examination findings are normal. His medical history includes a 2-
week neonatal intensive care unit admission for prolonged bleeding after circumcision.

Laboratory data are shown:


Laboratory Test Result
White blood cell count 9,800/µL (9.8 × 109/L)
Hemoglobin 8.5 g/dL (85 g/L)
Platelet count 90 × 103/µL (90 × 109/L)
Mean corpuscular volume 85 fL
Reticulocyte 2.5%
vWF activity 34% (reference range, 54%-152%)
vWF antigen 88% (reference range, (50%-150%)
Factor VIII assay 95% (reference range, (50%-200%)
vWF multimer assay Abnormal

Of the following, the BEST next step in management of this infant is


A. blood transfusion
B. desmopressin therapy
C. factor complex injection
D. platelet transfusion

American Academy of Pediatrics 609


PREP® Self-Assessment PREPSA 2023

Correct Answer: C
The boy in the vignette most likely has von Willebrand disease (vWD); this diagnosis is
supported by his laboratory findings of an abnormal von Willebrand panel. Item C179
summarizes the different types of vWD. Type 2B is associated with thrombocytopenia, including
neonatal thrombocytopenia, and is the most likely diagnosis for the child in the vignette.

Von Willebrand disease is an inherited bleeding disorder with qualitative or quantitative defects
in von Willebrand factor (vWF). Von Willebrand factor is a plasma protein that functions as a
carrier for factor VIII and helps bind platelets to damaged endothelium. Bleeding symptoms of
vWD include mucosal bleeding, epistaxis, and menorrhagia. Administration of antihemophilic
factor/vWF complex or another factor VIII/vWF concentrate is the treatment of choice for type
2B vWD. Acquired vWD, a decrease in vWAg in an individual without any bleeding history,
may occur in some illnesses (eg, autoimmune, some cancers) or as a side effect of medications
(eg, ciprofloxacin, valproic acid).

Desmopressin causes an increase in factor VIII activity, von Willebrand antigen, and ristocetin
cofactor activity. Desmopressin is the most common treatment for individuals with type 1 vWD,
although not every affected individual has a response. Treatment with desmopressin may cause
hyponatremia and therefore requires fluid restriction when used. The infant in the vignette does
not have type 1 vWD and would therefore not benefit from this therapy. In addition, in
individuals with type 2B vWD, desmopressin can lead to a transient decrease in platelets, which
may worsen bleeding.

American Academy of Pediatrics 610


PREP® Self-Assessment PREPSA 2023

A blood transfusion is not indicated for the infant in the vignette. Although he is anemic, there is
an appropriate reticulocyte response to the anemia, and he is hemodynamically stable.
Platelet transfusions in type 2B vWD may lead to increased binding between the platelets and the
high-molecularweight von Willebrand factor multimers. This binding leads to an increase in their
clearance from circulation. Therefore, platelet transfusion is not recommended.

PREP Pearls
• Von Willebrand factor is a plasma protein that functions as a carrier for factor VIII and
helps bind platelets to damaged endothelium.
• Von Willebrand disease type 2B may be associated with thrombocytopenia
• Desmopressin is not an appropriate treatment for type 2B von Willebrand disease and
may worsen thrombocytopenia.

ABP Content Specifications(s)


• Recognize the clinical findings associated with von Willebrand disease
• Plan the appropriate management of von Willebrand disease

Suggested Readings
• Cooper S, Takemoto C. Von Willebrand disease. Pediatr Rev. 2014;35(3):136-137.
doi:10.1542/pir.35-3-136.
• Lanzkowsky P, Lipton J, Fish JD. Lanzkowsky’s Manual of Pediatric Hematology and
Oncology. 6th ed. Elsevier Inc; 2016:121-125.
• Lichtman MA, Williams WJ, Kipps TJ, et al. Williams Manual of Hematology. 8th ed.
McGraw Hill; 2010:608-613.

American Academy of Pediatrics 611


PREP® Self-Assessment PREPSA 2023

Question 180
A neonate at term gestation was delivered vaginally 2 hours ago. His mother has a history of
genital herpes simplex virus (HSV); she was treated with appropriate antiviral suppressive
therapy during pregnancy. She had no HSV outbreaks during pregnancy and currently has no
prodromal symptoms or active HSV lesions anywhere on her body. The neonate’s Apgar scores
were 9 at both 1 and 5 minutes after birth. His physical examination findings are normal.

Of the following, the BEST next step in this neonate’s management is to


A. avoid breastfeeding
B. observe for signs of infection
C. obtain surface cultures for herpes simplex virus
D. start empiric acyclovir

American Academy of Pediatrics 612


PREP® Self-Assessment PREPSA 2023

Correct Answer: B
The neonate in the vignette is at low risk for acquiring vertically transmitted herpes simplex
virus (HSV) infection; the best next step in his management is to observe him for signs of
infection. Avoidance of breastfeeding is not recommended unless the mother has active HSV
lesions on her breast. Obtaining surface cultures for HSV and starting empiric acyclovir would
be indicated if the mother had active genital HSV lesions at the time of delivery.

Herpes simplex virus infections are common; approximately 1 of every 6 individuals aged 14 to
49 years is infected with genital herpes. Vertical transmission of HSV conveys a high risk of
morbidity and mortality to the neonate, making it critically important to appropriately manage
pregnant women with HSV. Of all neonates who contract HSV through vertical transmission,
more than 75% are born to mothers who have no known history or examination findings of
genital HSV infection.

Pregnant women who experience a primary outbreak of genital HSV at the time of delivery have
a 40% to 80% risk of vertical transmission if their neonate is delivered vaginally. Therefore, in
such cases the neonates should be delivered by cesarean section. While delivery by cesarean
section does not completely eliminate risk of vertical transmission, one study (Brown) found that
among neonates born to women with active HSV genital lesions, 1.2% of those delivered by
cesarean section became infected versus 7.7% of those delivered vaginally. Because prodromal
symptoms (eg, burning or itching prior to the appearance of a lesion) may indicate viral
shedding, cesarean delivery should also be performed in cases of pregnant women with
prodromal genital symptoms but no active lesions at the time of delivery. The American College
of Obstetricians and Gynecologists further supports offering cesarean section to women who had
their first episode of genital HSV infection at any time during the third trimester because of the
possibility of prolonged viral shedding.

Antiviral treatment should be given to pregnant women who have an outbreak at any time during
pregnancy to help reduce the duration of viral shedding. Any woman who has had a history of
genital herpes should be offered antiviral suppression starting at 36 weeks’ gestation to further
reduce the risk of vertical transmission.

PREP Pearls
• Neonates born to mothers who have a history of genital herpes simplex virus infection
but do not have active lesions or prodromal genital symptoms (eg, itching or burning) at
delivery require only observation for signs of infection.
• To reduce the risk of vertical transmission, cesarean section should be performed for
mothers who have active herpes simplex virus lesions or prodromal genital symptoms
(eg, itching or burning) at the time of delivery.

ABP Content Specifications(s)


• Understand the risk of maternal transmission of herpes simplex virus infection to
newborn infants Understand the epidemiology of herpes simplex virus

American Academy of Pediatrics 613


PREP® Self-Assessment PREPSA 2023

Suggested Readings
• American Academy of Pediatrics. Herpes simplex. In: Kimberlin DW, Barnett ED,
Lynfield R, Sawyer MH, eds. Red Book: 2021–2024 Report of the Committee on
Infectious Diseases. 32nd ed. American Academy of Pediatrics; 2021. Accessed
September 1, 2022. Red Book Online.
• Brown ZA, Wald A, Morrow A, et al. Effect of serologic status and cesarean delivery on
transmission rates of herpes simplex virus from mother to infant. JAMA.
2003;289(2):203-209. doi:10.1001/jama.289.2.203.
• Committee on Practice Bulletins. Management of genital herpes in pregnancy. Obstet
Gynecol. 2020;135(5):e193-e202. doi:10.1097/AOG.0000000000003840.
• Kojaoghlanian T. The newborn at risk of infection. In: McInerny TK, Adam HM,
Campbell DE, DeWitt TG, Foy JM, Kamat DM, eds. American Academy of Pediatrics
Textbook of Pediatric Care. American Academy of Pediatrics; 2021:chap 102. Accessed
September 1, 2022. Pediatric Care Online.

American Academy of Pediatrics 614


PREP® Self-Assessment PREPSA 2023

Question 181
A 6-year-old girl is seen for a routine health supervision visit. Her mother is concerned because
the girl has been wetting her bed for the past few weeks; she is dry during the day. She has had
no fever, increased urinary frequency or urgency, or burning with urination. The girl has been
completely toilet trained since age 4 years. She does not awaken when she wets the bed,
occasionally snores while sleeping, and recently seems tired in the morning. Her physical
examination findings are normal except for 3+ bilateral tonsillar enlargement.

Of the following, the MOST appropriate next step in management for this girl is
A. administration of antibiotics
B. institution of proper sleep hygiene
C. referral to an otolaryngologist
D. use of a bedwetting alarm

American Academy of Pediatrics 615


PREP® Self-Assessment PREPSA 2023

Correct Answer: C
The girl in the vignette has nocturnal enuresis which is likely secondary to sleep-disordered
breathing (SDB). Her additional signs and symptoms of SDB include snoring and daytime
fatigue. Given her 3+ tonsillar hypertrophy on physical examination, the most appropriate next
step in management is referral to an otolaryngologist for consideration of tonsilloadenoidectomy.

Sleep-disordered breathing, the interruption of normal sleep breathing patterns, can result in
physical and behavioral problems such as nocturnal enuresis and poor school performance. The
incidence of SDB in children, currently 1% to 6%, has been increasing in the United States.
Conditions associated with an increased risk for SDB include adenotonsillar hypertrophy,
obesity, neuromuscular diseases, and syndromes with features of macroglossia and retrognathia.

The spectrum of SDB conditions ranges from snoring without apnea to obstructive sleep apnea
(OSA) associated with poor oxygenation and ventilation. Symptoms of SDB in children include
snoring, daytime sleepiness, inattention and hyperactivity, nocturnal enuresis, and poor school
performance. Untreated chronic OSA can lead to fatigue, hypertension, cor pulmonale, right
heart failure, and neuropsychiatric symptoms.

Polysomnography helps to confirm the diagnosis and assess the severity of OSA, but it is not
always indicated before tonsillectomy and adenoidectomy in otherwise healthy children. The
2019 American Academy of Otolaryngology– Head and Neck Surgery Foundation’s Clinical
Practice Guideline: Tonsillectomy in Children (Update)—Executive Summary states that
“Before performing tonsillectomy, the clinician should refer children with obstructive
sleepdisordered breathing for polysomnography if they are <2 years of age or if they exhibit any
of the following: obesity, Down syndrome, craniofacial abnormalities, neuromuscular disorders,
sickle cell disease, or mucopolysaccharidoses… The clinician should also advocate for
polysomnography prior to tonsillectomy for obstructive sleep-disordered breathing in children
without any of the comorbidities listed for whom the need for tonsillectomy is uncertain or when
there is discordance between the physical examination and the reported severity of obstructive
sleep- disordered breathing.”

Studies report improvement in symptoms of OSA and polysomnography metrics after the
surgical procedure. Tonsillectomy and adenoidectomy is generally a safe procedure with
postoperative bleeding being the most common complication. Other complications include
infection, dehydration secondary to pain and poor oral intake, transient respiratory compromise
requiring oxygen, and rarely, velopharyngeal insufficiency.

Urinary tract infections are a common cause of secondary enuresis; urinalysis should be
performed and a urine specimen for culture obtained in suspected cases before antibiotic
treatment. However, the girl in the vignette has no other signs or symptoms suggestive of a
urinary infection, making this an unlikely diagnosis. Proper sleep hygiene is recommended for
all children but would not treat this girl’s symptoms. A bedwetting alarm is often recommended
for management of primary nocturnal enuresis. However, the girl in the vignette, with recent

American Academy of Pediatrics 616


PREP® Self-Assessment PREPSA 2023

onset of bedwetting after being toilet trained since 4 years of age, has secondary nocturnal
enuresis and would not benefit from the use of an alarm.

PREP Pearls
• Tonsillar and adenoid hypertrophy can lead to sleep-disordered breathing and obstructive
sleep apnea.
• Nocturnal enuresis can be a symptom of sleep-disordered breathing secondary to tonsillar
and adenoid hypertrophy.
• Tonsilloadenoidectomy is an appropriate first-line treatment in otherwise healthy children
with obstructive sleep apnea due to documented tonsillar and adenoid hypertrophy.

ABP Content Specifications(s)


• Understand the indications for a tonsilloadenoidectomy

Suggested Readings
• Bochner RE, Gangar M, Belamarich PF. A clinical approach to tonsillitis, tonsillar
hypertrophy, and peritonsillar and retropharyngeal abscesses. Pediatr Rev.
2017;38(2):81-92. doi:10.1542/pir.2016-0072.
• Burns JJ, Griffin G, Smith T, Slagle S, Kluka EA. Tonsillectomy and adenoidectomy. In:
McInerny TK, Adam HM, Campbell DE, DeWitt TG, Foy JM, Kamat DM, eds.
American Academy of Pediatrics Textbook of Pediatric Care. American Academy of
Pediatrics; 2021:chap 339. Accessed September 1, 2022. Pediatric Care Online.
• Gipson K, Lu M, Kinane TB. Sleep-disordered breathing in children. Pediatr Rev.
2019;40(1):3-13. doi:10.1542/pir.20180142.
• Mitchell RB, Archer SM, Ishman SL, et al. Clinical practice guideline: tonsillectomy in
children (update)—executive summary. Otolaryngol Head Neck Surg. 2019;160(2):187-
205. doi:10.1177/0194599818807917.

American Academy of Pediatrics 617


PREP® Self-Assessment PREPSA 2023

Question 182
A 2-month-old infant is seen for a health supervision visit. She has normal growth and age-
appropriate developmental milestones. The infant was diagnosed with a metabolic disorder
identified through the state newborn screening program. She was evaluated in the metabolic
clinic and started on a special formula. The infant’s mother reports that her daughter’s urine and
cerumen had a maple syrup odor that resolved after dietary intervention.

Of the following, the metabolic condition identified in this infant is a DISORDER of


A. amino acid metabolism
B. fatty acid oxidation
C. glycogen storage
D. lysosomal storage

American Academy of Pediatrics 618


PREP® Self-Assessment PREPSA 2023

Correct Answer: A
The girl in the vignette has maple syrup urine disease (MSUD), a disorder of amino acid
metabolism that can be identified on newborn screening.

Inborn errors of metabolism arise from enzyme deficiencies that lead to accumulation of toxic
substrate(s) and inadequate creation of the product(s) of the enzymatic reaction (Item C182A).
Clinical features and management of inborn errors of metabolism are outlined in Item C182B.
Clinical features in an untreated neonate with MSUD may include cerumen and urine with a
maple syrup odor, encephalopathy, and central respiratory failure leading to death. Individuals
with milder forms of MSUD can present in childhood with failure to thrive, developmental
delay, and encephalopathy during periods of stress (eg, fasting, dehydration, or illness).
Biochemical findings include increased levels of branched-chain amino acids (leucine, valine,
isoleucine) in plasma, ketonuria, and increased levels of branched-chain ketoacids in urine. A
metabolic formula managed by a dietician is the mainstay of treatment for infants. Affected
individuals require a lifelong, proteinrestricted diet that limits branched-chain amino acid intake.
Other amino acid disorders associated with an abnormal odor include phenylketonuria
(mousy/musty) and isovaleric acidemia (sweaty feet).

Newborn screening is a population-based screening program with the goal of identifying


disorders that have significantly high morbidity and/or mortality in the neonatal period and an
identified treatment. There is universal screening for some disorders across all states and the
District of Columbia; other conditions on the panel may be state specific.

Fatty acid oxidation (FAO) defects include medium-chain acyl-CoA dehydrogenase deficiency,
very long–chain acylCoA dehydrogenase deficiency, carnitine palmitoyltransferase I and II
deficiencies, and carnitine transporter defects.

The goal of treatment is to prevent fat catabolism. Management includes a low-fat diet with daily
carnitine administration, and avoidance of prolonged fasting with extra caloric intake during
illness. Management during an acute phase of illness includes administration of intravenous 10%
dextrose with electrolytes to maintain normoglycemia and prevent catabolism and intravenous
carnitine. The majority of FAO defects can be identified through newborn screening.
Glycogen storage disorders (GSDs) are a group of conditions whose clinical features vary based
on the site of abnormal glycogen metabolism (eg, liver, muscle, brain). Major features include
hypoglycemia, hepatomegaly, and myopathy. These disorders are not included in newborn
screening (except for Pompe disease, as noted below).

Lysosomal storage disorders include a wide variety of conditions characterized by accumulation


of material in the lysosomes (eg, Fabry disease, Krabbe disease, Gaucher disease,
mucopolysaccharidosis type I, and Niemann-Pick A and B). Clinical features are dependent upon
the specific enzymatic defect and the material accumulated. Multiple organ systems are usually
affected. Inclusion of these disorders on the newborn screen varies by state. Pompe disease (GSD
II, acid alpha-glucosidase deficiency) is categorized as both a GSD and lysosomal disease. It is
included in newborn screening in many states.
American Academy of Pediatrics 619
PREP® Self-Assessment PREPSA 2023

PREP Pearls
• Inborn errors of metabolism arise from enzyme deficiencies, which lead to accumulation
of toxic substrates and inadequate creation of the product(s) of the enzymatic reaction.
• Maple syrup urine disease can be identified on newborn screening. Clinical features in an
untreated neonate may include maple syrup odor in cerumen and urine, encephalopathy,
and central respiratory failure leading to death.
• Amino acid disorders associated with an abnormal odor include maple syrup urine
disease (maple syrup), phenylketonuria (mousy/musty), and isovaleric acidemia (sweaty
feet).
American Academy of Pediatrics 620
PREP® Self-Assessment PREPSA 2023

ABP Content Specifications(s)


• Recognize the clinical features associated with a disorder of amino acid metabolism other
than phenylketonuria

Suggested Readings
• Burton BK. Inborn errors of metabolism in infancy: a guide to diagnosis. Pediatrics.
1998;102(6):e69. doi:10.1542/peds.102.6.e69.
• Rice GM, Steiner RD. Inborn errors of metabolism. Pediatr Rev. 2016;37(1):3-15; quiz
16-7, 47. doi:10.1542/pir.2014-0122
• Rios A, Shur N. Specific congenital metabolic diseases. In: McInerny TK, Adam HM,
Campbell DE, DeWitt TG, Foy JM, Kamat DM, eds. American Academy of Pediatrics
Textbook of Pediatric Care. American Academy of Pediatrics; 2021:chap 106. Accessed
September 1, 2022. Pediatric Care Online.
• Strauss KA, Puffenberger EG, Carson VJ. Maple syrup urine disease. GeneReviews
[Internet]. University of Washington; 2020. Accessed September 1, 2022.
https://www.ncbi.nlm.nih.gov/books/NBK1319/.

American Academy of Pediatrics 621


PREP® Self-Assessment PREPSA 2023

Question 183
A pediatrician is giving a presentation at a conference for elementary school teachers in the local
school district. The topic is protecting children from sexual abuse. A teacher asks how to identify
when a child has been sexually abused. In response, the pediatrician describes several scenarios
and asks the participants to select the one that should raise concern for abuse.

Of the following, the participants’ BEST response would be


A. a 5-year-old and 6-year-old are looking at each others’ genitals in the playground
B. a 6-year-old is frequently found watching pornography in the school library
C. a 6-year-old girl refuses to be walked to the bathroom by a male teacher
D. a 10-year-old is looking at books about puberty and reproduction in the corner of the
school library

American Academy of Pediatrics 622


PREP® Self-Assessment PREPSA 2023

Correct Answers: B
Of the response choices, the situation most concerning that a child has been sexually abused is a
6-year-old frequently found watching pornography in the school library. Young children
discussing explicit sexual activities or participating in sexualized behaviors that are not
developmentally typical warrant an evaluation.

Behaviors reflecting sexual curiosity may be developmentally normal. Preschool children


undressing in front of each other, touching their own genitals, or attempting to see another peer
unclothed are developmentally normal behaviors. Similarly, a preadolescent showing interest in
puberty and the mechanics of reproduction is also developmentally normal. A 6-year-old girl
refusing to be walked to the bathroom by a male teacher is not cause for concern; this caution
likely reflects developmentally normal feelings of modesty.

Sexual abuse must be considered when behaviors deviate from developmentally expected sexual
curiousity. Young children exhibiting sexualized behaviors, mimicking sexual acts, or discussing
explicit sexual activities is concerning. No one behavior is diagnostic of sexual abuse, and
developmentally incongruent sexualized behaviors may also represent access to inappropriate
media or witnessing sexual activities in the home.

Non-specific complaints can also be an indicator of sexual abuse. Children may respond to abuse
with new nonspecific complaints such as abdominal pain, headaches, nocturnal enuresis,
encopresis, and sleep disturbance.

The prevalence of sexual abuse of children is difficult to accurately measure. The prevalence of
sexual abuse as reported to professionals or investigators is estimated at 2.4/1,000 children,
whereas a survey of children and their caretakers elicited a prevalence of 82/1,000 children. Not
all cases are disclosed to community or health professionals. Delayed disclosures of abuse are
common. Children with developmental disabilities, a past history of sexual exploitation, and
those lacking a protective parent are at increased risk for sexual abuse.

Teachers and physicians are mandated reporters for suspected sexual abuse. In instances of
concern without disclosure, discussion of the presenting symptoms with a child abuse specialist
may be helpful. Mandated reporters are protected from liability if reporting is made in good
faith.

• Parents and teachers can help prevent sexual abuse by teaching children:
• Appropriate boundaries with regards to others touching their bodies
• Correct vocabulary for all parts of their bodies, including the genitalia
• That they may choose with whom they do or do not feel comfortable spending time
• That adults should not ask them to keep inappropriate secrets
• To inform their parents if an adult tries to touch them or talks with them in a way that
makes them uncomfortable

American Academy of Pediatrics 623


PREP® Self-Assessment PREPSA 2023

A child discovered to be a victim of abuse should be referred to a skilled therapist to help the
child and their family manage the emotional consequences of the abuse.

PREP Pearls
• The prevalence of sexual abuse of children is difficult to accurately measure; delayed
disclosure of abuse is common.
• Developmentally precocious sexual behavior is concerning for sexual abuse.
• No single sexual behavior is diagnostic of sexual abuse.

ABP Content Specifications(s)


• Understand the behavioral and emotional consequences of child sexual abuse
• Understand the epidemiology of and the psychosocial and environmental risk factors for
sexual abuse

Suggested Readings
• Committee on Child Abuse and Neglect. Guidelines for the evaluation of sexual abuse of
children. Pediatrics. 1991;87(2):254-260. doi:10.1542/peds.87.2.254.
• Fortin K, Jenny C. Sexual abuse. Pediatr Rev. 2012;33(1):19-32. doi:10.1542/pir.33-1-
19.
• Leventhal JM, Asnes AG. Sexual abuse of children. In: McInerny TK, Adam HM,
Campbell DE, DeWitt TG, Foy JM, Kamat DM, eds. American Academy of Pediatrics
Textbook of Pediatric Care. American Academy of Pediatrics; 2021:chap 329. Accessed
September 1, 2022. Pediatric Care Online.

American Academy of Pediatrics 624


PREP® Self-Assessment PREPSA 2023

Question 184
A 3-year-old girl is brought to the office for evaluation of burning with urination for 2 days. She
has no urinary urgency, frequency, nighttime or daytime accidents, back pain, fever, rash, or
joint pains. She has a soft bowel movement every day. She has been toilet-trained since 2 years
of age. She has bubble baths 3 times a week and showers on most other days. The girl’s weight is
at the 50th percentile, height is at the 75th percentile, and her blood pressure is 90/60 mm Hg.
Her physical examination findings, including the genitourinary examination findings, are normal.

Urinalysis with microscopy results are shown:


Test Result
Appearance Yellow
Specific gravity 1.015
Leukocyte esterase Negative
Nitrite Negative
Blood Positive
Protein Negative
Red blood cells 10-20/HPF
White blood cells <5/HPF

Of the following, the MOST likely diagnosis for this girl is


A. chemical urethritis
B. dysfunctional voiding
C. urethral prolapse
D. urinary tract infection

American Academy of Pediatrics 625


PREP® Self-Assessment PREPSA 2023

Correct Answer: A
The girl in the vignette most likely has chemical urethritis caused by frequent bubble baths. This
diagnosis is supported by her symptom of dysuria and the urinalysis results showing microscopic
hematuria without pyuria. Dysuria is defined as pain, discomfort, or burning during urination.
Dysuria is a common symptom seen with urinary tract infection, inflammation, irritation, trauma,
obstruction, and several systemic illnesses. The causes of dysuria in different age groups are
shown (C184).

A detailed history can help focus the differential diagnosis for a child with dysuria.
• The presence of other urinary symptoms (eg, hematuria, frequency, urgency, foul-
smelling urine, and nocturnal and/or daytime enuresis) may support an infectious cause.
• Recent use of new detergents, soaps, ointments, or frequent bubble baths may point
toward chemical urethritis.
• A history of trauma to the genital area (eg, mechanical irritation from underwear fabric,
masturbation, or sexual activity), with consideration of the age group, can help
differentiate the cause.
• A history of withholding of urine, enuresis, and/or constipation may suggest
dysfunctional voiding.
• A family history of nephrolithiasis may suggest hypercalciuria.
• A history of fever may suggest acute pyelonephritis, pelvic inflammatory disease, or
systemic disease.
• A history of conjunctival inflammation, rash, joint pain, or oral ulcer may suggest a
diagnosis of reactive arthritis, Stevens-Johnson syndrome, or Behçet disease.

The physical examination of a child with dysuria should include the following:
• Examination of the lower back for markers of spinal dysraphism (eg, sacral dimple, cleft,
tufts of hair, or lipoma)
• Assessment of lower limb strength and reflexes (diminished strength or reflexes may
suggest tethered spinal cord)
• Abdominal examination to assess for flank or suprapubic mass (urinary tract obstruction),
flank tenderness (acute pyelonephritis), and suprapubic tenderness (cystitis)
• Genital examination to assess for discharge, labial adhesions, mucosal protrusion from
the urethral orifice (urethral prolapse), phimosis, or abnormalities of urethral meatus
(hypospadias or stenosis)

American Academy of Pediatrics 626


PREP® Self-Assessment PREPSA 2023

Laboratory evaluation of a child with dysuria should be based on the history and physical
examination findings. Evaluation may include urinalysis with microscopy, urine culture, urethral
smear, and/or examination of a vaginal discharge specimen for sexually transmitted infections. In
children with recurrent urinary tract infections or persistent dysuria symptoms,
cystourethrography or cystoscopy may be required to evaluate for urethral stricture.

Urinary tract infection is the most common cause of dysuria. Because the child’s urinalysis result
is negative for leukocyte esterase, nitrite, and white blood cells, urinary tract infection is an
unlikely diagnosis. Dysfunctional voiding is unlikely for the child in the vignette because she has
no history of constipation or associated urinary symptoms. Urethral prolapse typically presents
with dysuria, blood spotting on the diaper or underwear, and protrusion of urethral mucosa; the
child in the vignette has normal genital examination findings, making this diagnosis unlikely.

PREP Pearls
• Dysuria is a common symptom seen with urinary tract infection, inflammation, irritation,
trauma, obstruction, and several systemic illnesses.
• Chemical urethritis can be caused by detergents, soaps, ointments, and bubble baths.

ABP Content Specifications(s)


• Recognize the etiology of dysuria in patients of various ages
• Plan the appropriate diagnostic evaluation of dysuria in patients of various ages

Suggested Readings
• Goilav B, Kaskel FJ. Dysuria. In: McInerny TK, Adam HM, Campbell DE, DeWitt TG,
Foy JM, Kamat DM, eds. American Academy of Pediatrics Textbook of Pediatric Care.
American Academy of Pediatrics; 2021:chap 144. Accessed September 1, 2022. Pediatric
Care Online.

American Academy of Pediatrics 627


PREP® Self-Assessment PREPSA 2023

Question 185
An 8-year-old boy is brought to the emergency department with right elbow pain after falling
from the monkey bars at school. He appears distressed, is holding his right arm, and is crying
with severe pain. On physical examination, he has significant swelling and deformity of his right
elbow. Intravenous access is obtained and analgesics are administered. A right upper extremity
radiograph is obtained (Item Q185).

Of the following, the BEST next step in management for this boy is to
A. attempt reduction under conscious sedation
B. consult orthopedic surgery
C. perform a thorough neurovascular examination
D. place a long-arm posterior splint

American Academy of Pediatrics 628


PREP® Self-Assessment PREPSA 2023

Correct Answers: C
The radiograph of the boy in the vignette demonstrates a type III (completely displaced)
supracondylar fracture. Any significantly displaced fracture in a child warrants immediate
assessment for neurovascular compromise with a thorough neurovascular examination of the
affected extremity. If the child displays signs of neurovascular compromise, urgent reduction
under conscious sedation or general anesthesia is indicated. If there are no signs of neurovascular
compromise, the next management steps include orthopedic surgery consultation and placement
in a long-arm posterior splint until the specialist is consulted. However, these steps should not be
undertaken until both neurologic and vascular status of the extremity is completely assessed.

Supracondylar humeral fractures are among the most common fractures seen in children. They
occur most often in children aged 5 to 7 years and typically result from a fall landing directly on
the elbow or on an outstretched arm with the elbow hyperextended.

There are 3 types of supracondylar humerus fractures:


Type I: Non-displaced or minimally displaced
Type II: Displaced with intact posterior cortex
Type III: Completely displaced with no cortical contact

Because of the close proximity to the neurovascular bundle of the arm, there is an increased risk
of neurovascular compromise and compartment syndrome with type II and type III fractures.
Supracondylar humerus fractures may cause injury to the brachial artery, anterior interosseous
nerve, radial nerve, ulnar nerve, or median nerve. The most commonly injured nerve is the
anterior interosseous nerve; next is the radial nerve. Item C185 describes findings associated
with neurovascular injuries in supracondylar humerus fractures. Other orthopedic injuries that
can cause neurovascular compromise include dislocation of the elbow or any fracture that is
significantly displaced or angulated.

Type I supracondylar humerus fractures can be treated with cast or splint immobilization and
follow-up care with an orthopedic surgeon on an outpatient basis. Type II and III supracondylar
humerus fractures require urgent orthopedic evaluation for operative management. The goals of
appropriate and timely treatment of supracondylar fractures are to avoid neurologic or vascular
issues and to prevent long-term angular or extension deformities.

PREP Pearls
• Any significantly displaced fracture in a child warrants immediate assessment for
neurovascular compromise with a thorough neurovascular examination of the affected
extremity.
• There are 3 types of supracondylar humerus fractures. There is an increased risk of
neurovascular compromise and compartment syndrome with type II and type III
supracondylar humerus fractures due to the displacement.
• The most commonly injured nerve in supracondylar humerus fractures is the anterior
interosseous nerve.

American Academy of Pediatrics 629


PREP® Self-Assessment PREPSA 2023

ABP Content Specifications(s)


• Recognize the bone and joint injuries that commonly affect the vasculature

Suggested Readings
• Dinolfo EA. Fractures. Pediatr Rev. 2004;25(6):218-219. doi:10.1542/pir.25-6-218.
• Huurman WW, Ginsburg GM. Musculoskeletal injury in children. Pediatr Rev.
1997;18(12):429-440. https://pubmed.ncbi.nlm.nih.gov/9401402/.
• Strahlman RS. Fractures and dislocations. In: McInerny TK, Adam HM, Campbell DE,
DeWitt TG, Foy JM, Kamat DM, eds. American Academy of Pediatrics Textbook of
Pediatric Care. American Academy of Pediatrics; 2021:chap 252. Accessed September 1,
2022. Pediatric Care Online.

American Academy of Pediatrics 630


PREP® Self-Assessment PREPSA 2023

Question 186
A 7-year-old girl is seen for concerns about rapid growth. She has not had headaches, vision
changes, increased thirst, or increased urination. Her family has not noted any breast
development, vaginal bleeding, or vaginal discharge. Review of the girl’s growth chart shows
that she had been growing along the 25th percentile for height for age, but in the last year, her
height increased to the 95th percentile. Her weight is at the 60th percentile and body mass index
is at the 15th percentile. On physical examination, her heart rate is 140 beats/min and her blood
pressure is 120/50 mm Hg. She appears tall, thin, and “jittery.” Her neck appears full. Sexual
maturity rating is 1 for breast development and pubic hair. The remainder of her physical
examination findings are normal.

Of the following, the test MOST likely to reveal this girl’s diagnosis is a/an
A. 17-hydroxyprogesterone level
B. insulin-like growth factor 1 level
C. luteinizing hormone level
D. thyroid-stimulating hormone level

American Academy of Pediatrics 631


PREP® Self-Assessment PREPSA 2023

Correct Answer: D
The girl in the vignette, with linear growth acceleration, tachycardia, hypertension, goiter, and
tremulousness, has hyperthyroidism. Her thyroid-stimulating hormone production would likely
be completely suppressed due to thyroid hyperfunction, and her free thyroxine level would be
elevated. Elevated levels of thyroid hormone increase linear growth through a direct stimulatory
effect on the growth plates and enhanced growth hormone (GH) secretion from the pituitary
gland. In contrast, poor linear growth is frequently seen with longstanding hypothyroidism.

17-Hydroxyprogesterone, an adrenocortical hormone produced in response to


adrenocorticotropic hormone (ACTH) stimulation, is the hormone measured when screening for
congenital adrenal hyperplasia (CAH). Children with the non-classic form of 21-hydroxylase
deficiency (the most common form of CAH) typically present at school-age with signs of
adrenarche and accelerated growth. Bone age is advanced and precocious puberty may develop.
Congenital adrenal hyperplasia is an unlikely diagnosis for the girl in the vignette because she
does not have any signs of adrenarche or early pubertal development.

Insulin-like growth factor 1 (IGF1), produced in the liver in direct response to GH stimulation,
acts on the pediatric growth plate causing children to grow. Although rare in children, GH excess
results in increased IGF1 levels, growth acceleration, and tall stature. Growth hormone excess
may be seen in the setting of other conditions (eg, neurofibromatosis type 1 with optic glioma). If
GH excess is suspected, an oral glucose tolerance test should be performed; the diagnosis is
supported if GH is not suppressed in response to a glucose load. Growth hormone excess is
unlikely for the girl in the vignette because her growth acceleration is better explained by
hyperthyroidism.

Luteinizing hormone (LH) is the main pituitary hormone that induces puberty in children, acting
on the gonads to cause testosterone and estradiol production. The first sign of puberty in females
is thelarche (breast development). The first sign of puberty in males is testicular growth. The
pubertal growth spurt occurs in early puberty for girls, whereas boys are typically midway
through puberty before experiencing their growth spurt. Precocious puberty is defined as breast
development in girls prior to age 8 years or testicular growth in boys prior to age 9 years. Pubic
and axillary hair growth may not be signs of true puberty. A pubertal growth spurt is an unlikely
cause of growth acceleration for the girl in the vignette because she does not have any signs of
puberty by report or on physical examination.

Obesity is a common cause of tall stature due to overnutrition. Familial (or constitutional) tall
stature is the most common etiology of tall stature. Children with familial tall stature have a
height that is appropriate for their family pattern and have normal linear growth velocity.
Pathological tall stature is more likely in the setting of increased growth velocity. Rapid growth
may suggest a hormonal etiology.

Genetic syndromes, such as Marfan, Klinefelter (in males), Sotos, or Weaver, should be
considered in children with tall stature. Children with Sotos and Weaver syndromes are born
large for gestational age and have large head size.
American Academy of Pediatrics 632
PREP® Self-Assessment PREPSA 2023

PREP Pearls
• Thyroid disease should be considered in the setting of abnormal growth.
• Thyroid hormone has a direct stimulatory effect on the growth plates and enhances
growth hormone secretion from the pituitary gland; poor growth is often seen with
longstanding hypothyroidism.
• The increased growth velocity of hormonally related tall stature can be differentiated
from the normal growth velocity of non-hormonal causes of tall stature.

ABP Content Specifications(s)


• Differentiate among the causes of tall stature
• Plan the appropriate diagnostic evaluation of tall stature

Suggested Readings
• Alarcon G, Figueredo V, Tarkoff J. Thyroid disorders. Pediatr Rev. 2021;42(11):604-
618. doi:10.1542/pir.2020-001420.
• Jospe N. Hyperthyroidism. In: McInerny TK, Adam HM, Campbell DE, DeWitt TG, Foy
JM, Kamat DM, eds. American Academy of Pediatrics Textbook of Pediatric Care.
American Academy of Pediatrics; 2021:chap 270. Accessed September 1, 2021. Pediatric
Care Online.
• Srinivasan S, Misra M. Hyperthyroidism in children. Pediatr Rev. 2015;36(6):239-248.
doi:10.1542/pir.36-6-239.
• Zargham S, Crotty JE. Tall stature. Pediatr Rev. 2014;35(12):538-539; discussion 539.
doi:10.1542/pir.35-12-538.

American Academy of Pediatrics 633


PREP® Self-Assessment PREPSA 2023

Question 187
A 10-year-old girl is seen for a health supervision visit. Her parents are concerned because their
daughter can bend backward at the elbows and knees to a degree that seems unnatural and she is
“double-jointed.” They want to make sure that it is safe for her to participate in sports. The girl is
active in both soccer and gymnastics without pain or limitations. On physical examination, the
girl demonstrates 15 degrees of elbow extension and 15 degrees of knee extension bilaterally.
She is able to touch her palms to the floor without bending her knees but does not have signs of
increased joint mobility in her hands or wrists. The remainder of her physical examination
findings are normal.

Of the following, the BEST next step in this girl’s evaluation and management is to

A. reassure the family about her physical examination findings


B. recommend that she discontinue gymnastics due to risk of joint injury
C. refer her to a geneticist for evaluation for a connective tissue disorder
D. refer her to a physical therapist for generalized strengthening

American Academy of Pediatrics 634


PREP® Self-Assessment PREPSA 2023

Correct Answer: A
The girl in the vignette exhibits excess mobility of the elbows, knees, and spine. She does not
have any joint pain or limitations in her activities. This type of joint mobility is a normal finding
in school-aged children. The best next step is to reassure the family about her physical
examination findings. She does not need any additional evaluation, treatment, or restriction of
her activities.

The terms “benign joint hypermobility” or “generalized joint hypermobility” describe increased
joint mobility in the absence of pain, dislocation, or disability. Historically, the phrase “joint
hypermobility syndrome” was used to characterize the combination of hypermobility and
arthralgias. In 2017, the criteria for Ehlers-Danlos syndromes (EDS) were revised to reflect the
wide variety of clinical presentations and molecular diagnoses. Ehlers-Danlos, hypermobility
type, describes individuals with joint hypermobility, arthralgias, and/or atraumatic joint
dislocations, skin hyperextensibility, striae, and atrophic scarring. The diagnosis of EDS,
hypermobility type, is made clinically. There is no genetic test or known molecular basis for this
condition. The hypermobility subtype of EDS encompasses individuals with a heterogenous
group of clinical findings and symptoms. The term “hypermobility spectrum disorders” (HSD) is
now applied to people with hypermobility with pain and/or joint dislocations who do not meet
the criteria for EDS.

Children with EDS, hypermobility type, can benefit from a physical therapy program that
emphasizes strengthening to support the affected joints. Physicians should counsel patients with
EDS to avoid extremes of joint movement. Individuals with severe symptoms and joint
dislocations may need to discontinue sports, such as gymnastics, that require stretching joints to
their limits. Genetic evaluation should be considered for children and adolescents with signs and
symptoms of a connective tissue disorder.

The Beighton score is a measure of joint hypermobility. One point is awarded for each of the
following features:
• Greater than 10 degrees of extension at the elbows on each side
• Greater than 10 degrees of extension at the knees on each side
• Extension of the 5th finger at the metacarpophalangeal joint beyond 90 degrees on each
side
• Ability to touch the thumb to the palmar aspect of the wrist on each side Ability to touch
one’s palms to the floor without bending the knees

The maximum Beighton score is 9. A score of 4 or 5 or greater typically determines


hypermobility in adults. There is no clear consensus on the Beighton cutoff score that determines
hypermobility in children. Because children have more joint mobility than adults, Beighton
scoring does not have adequate specificity to confirm hypermobility conditions in children. A
score of 6 or greater is the proposed cutoff for diagnosis of hypermobility in children.
However, a recent study of Australian children showed that over one-quarter of girls and over
10% of boys had a Beighton score of 6 or higher. In addition, the Beighton score only measures

American Academy of Pediatrics 635


PREP® Self-Assessment PREPSA 2023

mobility in a subset of joints. Therefore, children with hypermobility and atraumatic dislocations
of the shoulders and patellas may have a low Beighton score.

PREP Pearls
• Joint hypermobility that is not associated with pain or activity limitation is a normal
finding in school-aged children.
• Ehlers-Danlos syndrome, hypermobility type, describes individuals with joint
hypermobility, arthralgias, and/or atraumatic joint dislocations, and other criteria, such as
skin hyperextensibility, striae, and atrophic scarring.
• Ehlers-Danlos syndrome, hypermobility type, is diagnosed clinically.

ABP Content Specifications(s)


• Recognize the clinical findings associated with hypermobility syndrome and manage
appropriately

Suggested Readings
• Malfait F, Francomano C, Byers P, et al. The 2017 international classification of the
Ehlers-Danlos syndromes. Am J Med Genet C Semin Med Genet. 2017;175(1):8-26.
doi:10.1002/ajmg.c.31552.
• Morris SL, O'Sullivan PB, Murray KJ, Bear N, Hands B, Smith AJ. Hypermobility and
musculoskeletal pain in adolescents. J Pediatr. 2017;181:213-221.e211.
doi:10.1016/j.jpeds.2016.09.060.
• Siegel DM, Marston B. Joint pain. In: McInerny TK, Adam HM, Campbell DE, DeWitt
TG, Foy JM, Kamat DM, eds. American Academy of Pediatrics Textbook of Pediatric
Care. American Academy of Pediatrics; 2021:chap 171. Accessed September 1, 2022.
Pediatric Care Online.
• Smits-Engelsman B, Klerks M, Kirby A. Beighton score: a valid measure for generalized
hypermobility in children. J Pediatr. 2011;158(1):119-123, 123.e111-e114.
doi:10.1016/j.jpeds.2010.07.021.

American Academy of Pediatrics 636


PREP® Self-Assessment PREPSA 2023

Question 188
A 13-year-old adolescent girl is seen in the clinic for evaluation of irregular menstrual bleeding.
She had menarche at age 11 years and has monthly cycles lasting 5 to 7 days. Two months ago,
she began having light vaginal bleeding between cycles and a small amount of vaginal discharge.
She has had 2 lifetime male sexual partners and does not use condoms. Her last sexual encounter
was 2 months ago. Physical examination is significant for normal-appearing external genitalia
with a mild non-odorous vaginal discharge. She requests no speculum examination due to
extreme discomfort. The remainder of her physical examination findings are normal.
A urine pregnancy test result is negative. Urine is sent for sexually transmitted infection testing.
When the adolescent is counseled on safe sexual practices, she states that her partner “does not
like condoms” and has refused to use them. She declines a prescription for birth control because
her partner has told her about negative side effects. Her caregivers are not aware of her sexual
practices.

Of the following, the BEST next step is this adolescent’s management is to


A. ask her open-ended questions about her relationship to elicit coercion or abuse
B. inform her caregiver about concerns for her risky sexual behaviors
C. involve a social worker to make a report to child protective services
D. treat her empirically for sexually transmitted infections with appropriate antibiotics

American Academy of Pediatrics 637


PREP® Self-Assessment PREPSA 2023

Correct Answer: A
The sexually active adolescent in the vignette describes a partner who refuses to wear condoms
and does not support her autonomy to choose contraception. These behaviors are examples of
reproductive coercion, an aspect of intimate partner violence (IPV). Of the response choices, the
best next management step is to ask for more information about her relationship in an open,
nonjudgmental, and confidential manner. The age of her partner is a key piece of information to
elicit. Health care professionals should be aware of their state’s laws regarding legal age of
consent and definitions of statutory rape. Local legal and social services can provide assistance.
The adolescent should also be provided information on healthy and unhealthy relationships and
resources for getting help.

Intimate partner violence is physical, emotional, or sexual violence between 2 individuals in a


relationship. Intimate partner violence in adolescents (“teen dating violence”) occurs in
relationships where at least one partner is a minor. Reproductive coercion is an aspect of IPV in
which one partner discourages the other to appropriately use condoms or other contraception in
an attempt to deny autonomy and exert control. Worldwide, 1 in 3 adolescents aged 15 to 19
years have at least one lifetime experience of IPV. Red flags for exposure to IPV include early
age of sexual initiation, somatic complaints, substance use, mood changes, multiple sexually
transmitted infections (STIs), and family history of IPV.

While the adolescent in the vignette’s young age and high-risk behaviors are concerning, it is not
the best next step to inform her caregivers, especially without her consent. It is also not yet
necessary to involve social services to report this relationship to authorities. More information
should be obtained first. Informing the adolescent’s caregivers or child protective services
without appropriate clarification may inadvertently place the adolescent in danger and may cause
the adolescent to lose trust and not return for care.

The adolescent in the vignette’s irregular menstrual bleeding and vaginal discharge are
concerning for an STI. Rather than treating empirically, the most appropriate course of action is
to await results of the STI testing and provide treatment for positive results. The adolescent
should be treated empirically for STIs if she is aware that her partner had a positive test result or
if she can not return for care due to transportation or other concerns. She should also be given
information on pre-exposure prophylaxis (PrEP) for prevention of HIV infection.

PREP Pearls
• Intimate partner violence should be discussed with adolescents through education on
healthy and unhealthy relationships using confidentiality and open-ended questions.
• Red flags for exposure to intimate partner violence (IPV) include early age of sexual
initiation, somatic complaints, substance use, mood changes, multiple sexually
transmitted infections, and family history of IPV.
• Health care professionals should be aware of their state’s laws on age of consent and
partner with social services when appropriate.

American Academy of Pediatrics 638


PREP® Self-Assessment PREPSA 2023

MOCA-Peds Objective
• Recognize indications for sexually transmitted infection testing in prepubertal and
peripubertal children.

ABP Content Specifications(s)


• Recognize and apply ethical principles regarding the issue of intimate-partner violence
• Recognize and apply ethical principles regarding the issues of physical and mental abuse

Suggested Readings
• Knapp P. Pediatric history: assessing the social environment. In: McInerny TK, Adam
HM, Campbell DE, DeWitt TG, Foy JM, Kamat DM, eds. American Academy of
Pediatrics Textbook of Pediatric Care. American Academy of Pediatrics; 2021:chap 14.
Accessed September 1, 2022. Pediatric Care Online.
• Rome E, Miller E. Intimate partner violence in the adolescent. Pediatr Rev.
2020;41(2):73-80. doi:10.1542/pir.2018-
• Schaff EA. Contraception and abortion. In: McInerny TK, Adam HM, Campbell DE,
DeWitt TG, Foy JM, Kamat DM, eds. American Academy of Pediatrics Textbook of
Pediatric Care. American Academy of Pediatrics; 2021:chap 123. Accessed September 1,
2022. Pediatric Care Online.

American Academy of Pediatrics 639


PREP® Self-Assessment PREPSA 2023

Question 189
A 3-year-old girl is admitted to the pediatric intensive care unit (PICU) after being found face
down in a backyard pool in the summer after an unknown but prolonged period of time. She
underwent cardiopulmonary resuscitation at the scene without return of spontaneous circulation.
The girl was transported to the emergency department where resuscitation continued for another
45 minutes, and ultimately the return of spontaneous circulation was established. She was
intubated and mechanically ventilated, and vasopressors were started for fluid refractory
hypotension.

In the PICU, her vital signs are a heart rate of 60 beats/min, temperature of 37.4°C, respiratory
rate in sync with the ventilator at 20 breaths/min, and blood pressure of 120/75 mm Hg. The
patient is in a comatose state with a Glascow coma scale score of 3, no spontaneous respiratory
effort, and no response to external stimulation. There are no palpable skull defects. Findings on
brain computed tomography scan are shown in Item Q189.

Item Q189: Computed tomography of the brain.


Courtesy of J. Kane

Of the following, the MOST likely physiologic mechanism for the girl’s current condition is
A. fresh water intoxication
B. hypoxic brain injury
C. spinal cord injury
D. status epilepticus

American Academy of Pediatrics 640


PREP® Self-Assessment PREPSA 2023

Correct Answer: B
The most likely mechanism to explain the girl in the vignette’s current condition is hypoxic brain
injury with cerebral edema as a result of prolonged hypoxia or anoxia from drowning. Drowning
is defined as a process resulting in primary respiratory impairment from submersion/immersion
in a liquid medium. The primary physiologic consequence of drowning results from hypoxic-
ischemic and reperfusion injuries of the brain. As shown in the computed tomography of the
brain (Item C189), prolonged cerebral hypoxia can lead to cerebral edema, with loss of gray-
white matter differentiation, and in the most severe cases, cerebral herniation, and brain death.
For children, the entire drowning process from immersion to cardiac arrest may be as short as
30-60 seconds.

Fresh water intoxication may cause severe electrolyte derangements, including hyponatremia;
this usually results in acute hyponatremic seizures. Cerebral edema can ensue, but this condition
would be extremely rare and is not the preferred response. There are no signs of acute traumatic
brain injury to explain the child’s condition. Although subclinical status epilepticus can cause a
comatose state, the known history of drowning makes hypoxia a more likely mechanism of
injury.

Severe consequences of drowning occur when children develop hypoxia as a result of no gas
exchange across the pulmonary alveolar membranes. As hypoxemia increases, there is a
concomitant decrease in cardiac output. Children then develop bradycardia with progression to
fatal arrhythmias or pulseless electrical activity. There is a decrease in pulmonary compliance as
surfactant is washed out of the alveoli with resultant mismatch in ventilation and perfusion,
leading to intrapulmonary shunt and worsening hypoxia. Acute hypoxic/ischemic brain injury
ensues as a function of the overall duration and degree of hypoxia. Watershed regions of the
brain are affected first, but with severe hypoxia, global cortical injury ensues. In severe cases,
cytotoxic edema from cellular death occurs causing cerebral edema, which may lead to cerebral
herniation and brain death. Unlike acute traumatic brain injury, there are no medical or surgical
interventions shown to reverse, improve, or abort the development of cerebral edema resulting
from hypoxic brain injury outside of the neonatal population.

Rapid rescue from the water and immediate cardiopulmonary resuscitation (CPR) is critical for
meaningful neurologic survival in drowning victims. Unlike other causes of cardiac arrest,
resuscitation should begin with airway and breathing as opposed to initiating chest compressions
first. Although children found unresponsive in pools are often placed in c-collar neck
immobilization, the risk of cervical spine injury is exceptionally rare, and immobilization should
only be used when the history or clinical scenario is high-risk for cervical injury. Children with
any degree of hypoxia should be provided supplemental oxygen to normalize oxygen saturations
and avoid hyperoxia. Non-invasive or invasive mechanical ventilation may be required.
Normalization of hemodynamics is important to maintain adequate cerebral perfusion pressure.
Hypotension may result from hypovolemia, myocardial injury and ischemia, or loss of peripheral
vascular tone.

American Academy of Pediatrics 641


PREP® Self-Assessment PREPSA 2023

Hypothermia should be corrected; nearly all drownings, regardless of location or seasonality,


result in hypothermia. Hypothermia reflects the water temperature, duration of submersion, and
environmental exposure after removal from the water. Hypoxic arrest is common in warm water
drownings. Very cold water (defined as less than 5°C) can result in immediate syncope and
dysrhythmias. Ice water, in contrast to cold water, may offer the only survival advantage for
drowning victims. The 2017 Guidelines for Uniform Reporting of Data From Drowning made a
distinction between cold water and icy water, noting that icy water may increase the time of
brain survival. Thus, the temperature of the water, and not the child’s body temperature on
arrival, may impact neurological outcomes. Recent data demonstrate that if water temperature is
greater than 6°C, survival/resuscitation is extremely unlikely if submerged longer than 30
minutes. If the water temperature is 6°C or below, survival/resuscitation is extremely unlikely if
submerged longer than 90 minutes. Although it is challenging to set firm limits for the duration
of submersion after which CPR should be limited, if the victim has submerged in warm water
(>5°C) for longer than 30 minutes, there is practically no chance of meaningful neurologic
survival, and the result of ongoing or prolonged resuscitation will be a persistent vegetative state
or death.

PREP Pearls
• In drowning, acute hypoxic/ischemic brain injury ensues and is a function of the overall
duration and degree of hypoxia.
• In severe hypoxia, cytotoxic edema from cellular death occurs, causing cerebral edema
which may lead to cerebral herniation and brain death.
• In drowning, if water temperature is warmer than 5°C, survival is extremely unlikely if
submerged longer than 25-30 minutes.

ABP Content Specifications(s)


• Recognize cerebral edema in an asphyxiated patient

Suggested Readings
• Frankel LR. Drowning and near drowning (submersion injuries). In: McInerny TK, Adam
HM, Campbell DE, DeWitt TG, Foy JM, Kamat DM, eds. American Academy of
Pediatrics Textbook of Pediatric Care. American Academy of Pediatrics; 2021:chap 356.
Accessed September 1, 2022. Pediatric Care Online.
• McCallin TE, Morgan M, Hart MLI, Yusuf S. Epidemiology, prevention, and sequelae of
drowning. Pediatr Rev. 2021;42(3):123-132. doi:10.1542/pir.2019-0150.
• Semple-Hess J, Campwala R. Pediatric submersion injuries: emergency care and
resuscitation. Pediatr Emerg Med Pract. 2014;11(6):1-21; quiz 21-2.
https://pubmed.ncbi.nlm.nih.gov/25090739/.
• Suominen P, Baillie C, Korpela R, Rautanen S, Ranta S, Olkkola KT. Impact of age,
submersion time and water temperature on outcome in near-drowning. Resuscitation.
2002;52(3):247-54. doi:10.1016/s0300-9572(01)00478-6.

American Academy of Pediatrics 642


PREP® Self-Assessment PREPSA 2023

Question 190
A 17-year-old adolescent boy is evaluated for several months of generalized abdominal pain,
diarrhea, flatulence, and bloating. He has 2 to 3 loose, watery, nonbloody stools daily (most
commonly after lunch and dinner), without nocturnal stooling. He is otherwise well. His father
has had similar symptoms for many years. For breakfast, the adolescent usually eats toast and
peanut butter with fruit. At lunch, he usually eats a ham and cheese sandwich, chips, a salad with
ranch dressing and cheese, and chocolate milk. Dinners are often fast food options after sports
games, typically large sandwiches and chocolate milk or chicken nuggets, french fries, and a
milkshake. He snacks on fruit and nuts during the day. On physical examination, the boy appears
healthy with a weight of 64 kg (50th percentile for age), height of 175 cm (50th percentile for
age), and body mass index of 21 kg/m2 (50th percentile for age). His abdomen is soft and
nontender, but distended and tympanitic. His perianal examination findings are normal.

Additional testing reveals:


Laboratory Test (Stool) Result
Fecal occult blood test Negative
Ova and parasite examination Negative
Lactoferrin Negative
pH 5.0
Bacterial pathogen testing Negative

Of the following, the test MOST likely to identify the cause of the adolescent’s symptoms is
A. colonoscopy
B. esophageal pH impedance study
C. lactose breath test
D. wheat IgE concentration

American Academy of Pediatrics 643


PREP® Self-Assessment PREPSA 2023

Correct Answers: C
The adolescent male in the vignette has signs and symptoms consistent with carbohydrate
malabsorption (watery and acidic stools, bloating, and flatulence). He reports an increase in
symptoms after eating lactose-containing foods; thus, lactose intolerance is very likely and can
be further investigated with a lactose breath test. Colonoscopy, esophageal pH impedance study,
or wheat IgE testing would not be useful to identify lactose intolerance or other causes of
carbohydrate malabsorption.

Starches (amylopectin and amylose), lactose, and sucrose are the most common forms of
ingested carbohydrates.

Salivary and pancreatic amylases are secreted upon carbohydrate ingestion and break down
starches to maltose, maltotriose, and branched glucose polymers (dextrins). Carbohydrates are
further digested to monosaccharides in the intestinal brush border in the small bowel by
disaccharidase enzymes:
• Sucrase-isomaltase
o Maltose/isomaltose → glucose
o Sucrose → glucose + fructose
• Lactase
o Lactose → glucose + galactose
• Glucoamylase
o Glucose polymers → glucose
o Monosaccharides (glucose, fructose, and galactose) are then transported through
carrier proteins into enterocytes.

When carbohydrates are not fully digested and absorbed, water is drawn into the intestinal lumen
and colonic bacteria produce excessive gas (hydrogen, methane, carbon dioxide) and acids (eg,
acetic acid, butyric acid), resulting in diarrhea, flatulence, abdominal cramping, bloating, and
acidic stool (<5.5; normal stool pH is generally 6.0-7.5).

Carbohydrate malabsorption can occur due to deficiencies in brush border enzymes, small bowel
intestinal inflammation, or short bowel syndrome. The malabsorption may be chronic (eg,
congenital disaccharidase deficiencies or short bowel syndrome) or transient (eg, intestinal
infection or inflammation). When the small intestine is inflamed, lactase is the most common
enzyme to be affected. When the intestinal injury resolves, disaccharidase function normalizes
and carbohydrate absorption returns. Lactase deficiency primarily occurs in older teenagers and
adults; congenital lactase deficiency is very uncommon. Lactose intolerance may be diagnosed
clinically (with improvement in symptoms after removal of dietary lactose and recurrence of
symptoms following reintroduction of lactose), use of lactose breath hydrogen testing, or
measurement of duodenal lactase concentration (obtained during an upper endoscopy).

Sucrase-isomaltase deficiency is a congenital brush border enzyme deficiency, diagnosed during


infancy due to the development of watery diarrhea after introduction of sucrose (which may be

American Academy of Pediatrics 644


PREP® Self-Assessment PREPSA 2023

present in some infant formulas, multivitamin suspension, and baby foods). It is diagnosed by
assessing the duodenal sucrase-isomaltase concentration.

PREP Pearls
• Carbohydrate malabsorption should be suspected in children with watery stools,
abdominal pain, bloating, flatulence, and acidic stools.
• Causes of carbohydrate malabsorption include intestinal injury, brush border enzyme
deficiencies, and short bowel syndrome.
• Lactase deficiency primarily occurs in older teenagers and adults; congenital lactase
deficiency is very uncommon.

ABP Content Specifications(s)


• Recognize the clinical features associated with a carbohydrate malabsorption disorder

Suggested Readings
• Ammoury RF, Croffie JM. Malabsorptive disorders of childhood. Pediatr Rev.
2010;31(10):407-415; quiz 415-416.
• doi:10.1542/pir.31-10-407.
• Szilagyi A, Ishayek N. Lactose intolerance, dairy avoidance, and treatment options.
Nutrients. 2018;10(12):1994. doi:10.3390/nu10121994.

American Academy of Pediatrics 645


PREP® Self-Assessment PREPSA 2023

Question 191
A previously healthy, 16-year-old adolescent boy is seen in the emergency department. Two
days ago, he fell from his bicycle and sustained a minor injury to his left arm. For 1 day he has
had pain, redness, and swelling of his left upper extremity and fever up to 39.4°C. The pain and
swelling have increased in severity over the past few hours, with spread of the redness to his
shoulder (Item Q191). He has no sick contacts, exposure to pets or animals, or recent travel.
The adolescent appears toxic, with a temperature of 39.1°C, heart rate of 154 beats/min,
respiratory rate of 42 breaths/min, and blood pressure of 83/36 mm Hg. Musculoskeletal
examination is significant for exquisite tenderness and tense swelling of his left arm with a
purplish discoloration of the skin. The remainder of his physical examination findings are
unremarkable.

A complete blood count shows a white blood cell count of 23,000/µL (23 × 109/L) with 97%
neutrophils, a hemoglobin level of 10.9 g/dL (109 g/L), and a platelet count of 76 × 103/µL (76 ×
109/L). After a blood sample is drawn for culture, intravenous vancomycin and ceftazidime are
administered.

Item Q191: Left upper extremity findings for the boy described in the vignette.
Reprinted with permission from C Prober. Copyright C Prober.

Of the following, the BEST next step in this adolescent’s management is


A. administration of clindamycin, intravenously
B. magnetic resonance imaging of the left arm
C. plain radiography of the left arm
D. urgent surgical debridement

American Academy of Pediatrics 646


PREP® Self-Assessment PREPSA 2023

Correct Answer: D
The ill-appearing adolescent in the vignette has a clinical picture suggestive of necrotizing
fasciitis (NF) following a history of recent trauma. Given the significant mortality and morbidity
associated with NF, surgical exploration for debridement is urgently indicated. Necrotizing
fasciitis is a life-threatening pyogenic infection of the subcutaneous tissues and fascia
characterized by rapid spread along the fascial planes and thrombosis of the nutrient vessels,
leading to ischemia and necrosis. The trunk and lower limbs are most commonly affected in
children. Pain disproportionate to local findings is the classical presentation and should alert the
physician to consider the diagnosis of NF; signs of systemic toxicity are often noted.

Necrotizing fasciitis is unusual in children; the incidence is 0.08 to 0.13 per 100,000 children per
year. Case-fatality rates are high (10%). Group A Streptococcus (GAS) is a well-recognized
causative pathogen. Escherichia coli and other gram-negative rods, and Staphylococcus aureus,
including methicillin-resistant S aureus (MRSA), have also been implicated. Infection can be
polymicrobial and include anaerobes (eg, Peptostreptococcus and Bacteroides fragilis),
especially when the perineum is involved. In immunocompromised hosts with neutropenia, NF
due to Pseudomonas aeruginosa and Clostridium septicum has been reported. Myonecrosis
caused by Clostridium perfringens may complicate contaminated surgical or traumatic wounds.
While affected individuals often report a history of trauma, other risk factors for NF include
eczema, surgery, immune deficiency, and diabetes. Prior to the availability of varicella vaccine,
NF and other forms of invasive group A streptococcal infection (eg, toxic shock syndrome,
bacteremia without a source, pneumonia) occurred in 15% to 30% of children with varicella
disease. In cases of NF associated with streptococcal toxic shock syndrome, pathogen virulence
(eg, pyrogenic exotoxin A) and host susceptibility factors may contribute to the disease severity.

The pathogenesis of NF involves entry of bacteria into the subcutaneous tissue via a breach in
the skin barrier or by hematogenous spread. Early clinical findings include severe pain and
tenderness on palpation of the affected area. There may be associated swelling and erythema.
Fever and other non-specific complaints (eg, vomiting, myalgia, and diarrhea) may be noted.
Early diagnosis of NF may be challenging as the disease can mimic cellulitis or a viral illness
when there is no local evidence of skin injury or infection. However, pain in the affected area
disproportionate to local findings is a crucial, early clue for NF; systemic toxicity is another red
flag, if present. As the disease progresses, the affected area may become markedly swollen
followed by the appearance of hemorrhagic bullae (or blisters), crepitus, and gangrene. A rapidly
progressive clinical course with sepsis, septic shock, and multisystem involvement is a frequent
occurrence in cases of NF caused by GAS strains.

The clinical suspicion of NF is a surgical emergency. Surgical intervention may range from
aggressive debridement of all devitalized tissue to amputation, based on the operative findings of
disease severity. In many cases, repeated resection of necrotic tissue is necessary for source
control and removal of devitalized tissue. Tissue specimens must be submitted to the laboratory
for bacterial cultures and histopathology. An incisional frozen-section biopsy may be considered
in cases in which the diagnosis is uncertain.

American Academy of Pediatrics 647


PREP® Self-Assessment PREPSA 2023

Performance of imaging studies should never delay surgical exploration when NF is suspected.
Imaging studies may be warranted when the clinical diagnosis is in doubt. Plain radiographs are
not recommended, as they may appear normal in cases of NF. Contrast-enhanced computed
tomography (CT) is very sensitive and specific, and is the imaging modality of choice to confirm
the diagnosis of NF. Extensive soft tissue involvement and presence of gas may be noted on CT.
Magnetic resonance imaging is very sensitive and specific, but challenging to obtain in the
emergency setting.

Management of NF requires a multidisciplinary team involving infectious disease specialists,


surgeons, and critical care specialists. In addition to early surgical debridement, aggressive
critical care support and empiric broadspectrum antimicrobial therapy is recommended.
Antibiotic coverage should include gram-positive cocci (including MRSA), gram-negative rods,
and anaerobic pathogens. Culture with sensitivities is used to guide specific antibiotic choice
once results are available. Treatment of GAS infection includes high-dose intravenous penicillin
plus a proteinsynthesis inhibitor (eg, clindamycin). Clindamycin is recommended due to its anti-
toxin activity and other properties (eg, long postantibiotic effect unaffected by inoculum size
compared to penicillin). Adjunctive immunomodulatory therapy with intravenous
immunoglobulin may improve outcomes in cases of streptococcal toxic shock syndrome
associated with NF.

PREP Pearls
• Necrotizing fasciitis is a severe, life-threatening pyogenic infection of the subcutaneous
tissues and fascia characterized by rapidly progressive spread of infection along the
fascial planes, resulting in tissue necrosis and often sepsis.
• Recent trauma, burns, surgery, and varicella infection are common risk factors for
necrotizing fasciitis in children.
• Necrotizing fasciitis is a surgical emergency. Urgent surgical exploration with aggressive
debridement of devitalized tissue is required.

ABP Content Specifications(s)


• Recognize the clinical findings associated with necrotizing fasciitis

American Academy of Pediatrics 648


PREP® Self-Assessment PREPSA 2023

Suggested Readings
• American Academy of Pediatrics. Group A streptococcal infections. In: Kimberlin DW,
Barnett ED, Lynfield R, Sawyer MH, eds. Red Book: 2021–2024 Report of the
Committee on Infectious Diseases. 32nd ed. American Academy of Pediatrics; 2021.
Accessed September 1, 2022. Red Book Online.
• Diab J, Bannan A, Polity T. Necrotizing fasciitis. BMJ. 2021;369:m1428
doi:10.1136/bmj.m1428.
• Marathe K, Williams JV. Bacterial skin infections. In: McInerny TK, Adam HM,
Campbell DE, DeWitt TG, Foy JM, Kamat DM, eds. American Academy of Pediatrics
Textbook of Pediatric Care. American Academy of Pediatrics; 2021:chap 222. Accessed
September 1, 2022. Pediatric Care Online.
• Nelson GE, Pondo T, Toews KA, et al. Epidemiology of invasive group A Streptococcal
infections in the United States, 2005-2012. Clin Infect Dis. 2016;63(4):478-486.
doi:10.1093/cid/ciw248.
• Noor A, Krilov LR. Necrotizing fasciitis. Pediatr Rev. 2021;42(10):573-575.
doi:10.1542/pir.2020-003871.
• Schröder A, Gerin A, Firth GB, et al. A systematic review of necrotising fasciitis in
children from its first description in 1930 to 2018. BMC Infect Dis. 2019;19(1):317.
doi:10.1186/s12879-019-3941-3.

American Academy of Pediatrics 649


PREP® Self-Assessment PREPSA 2023

Question 192
A 3-month-old infant is brought to the office for concerns about noisy breathing, poor feeding,
and slow growth. He has had noisy breathing since 1 month of age that is worse with feeding and
while sleeping on his back. He nurses poorly and is not growing as well as expected. The feeding
difficulties and noisy breathing have been worsening over the past month. He has not had fever
or other signs of illness. His prenatal and birth history are unremarkable; birth weight was 3.02
kg.

On physical examination, the infant weighs 4.2 kg. His temperature is 37°C, heart rate is 120
beats/min, respiratory rate is 30 breaths/min when upright and calm, and oxygen saturation is
97% in room air. When supine, he has variable inspiratory stridor with use of accessory muscles
and intercostal retractions. Breath sounds are clear with good aeration throughout. The remainder
of his physical examination findings are unremarkable.

Of the following, the BEST description of this infant’s airway compromise is


A. dynamic bronchial obstruction
B. dynamic laryngeal obstruction
C. fixed nasal obstruction
D. fixed tracheal obstruction

American Academy of Pediatrics 650


PREP® Self-Assessment PREPSA 2023

Correct Answer: B
The infant in the vignette has laryngomalacia, a dynamic airway obstruction at the level of the
larynx. Most infants with laryngomalacia have noisy breathing that is worse when supine and
sometimes worse when feeding. This infant has critical laryngomalacia with increased work of
breathing that is interfering with feeding and growth. His inspiratory stridor localizes the site of
obstruction to above the glottis, and his varying signs and symptoms corroborate a dynamic
rather than fixed process.

Dynamic bronchial obstruction (eg, bronchomalacia) is characterized by expiratory wheezing


that is variable rather than constant. Fixed tracheal obstruction (eg, tracheal stenosis) is
characterized by biphasic turbulent airflow with inspiratory stridor or stertor (lower-pitched
stridor) and expiratory wheezing, sometimes described as biphasic wheezing. The noisy
breathing can vary with the amount of activity or depth of respiration, but typically not with
position. The expiratory component of either a fixed tracheal or bronchial obstruction is
characterized by a monophonic wheeze, indicating a single point of obstruction; this
characteristic differentiates it from the polyphonic or musical wheezing of diffuse small airway
obstruction (asthma).

A fixed nasal obstruction (eg, choanal stenosis or atresia) in an infant is potentially life-
threatening, as infants are obligate nose breathers. These conditions are identified in the newborn
period because of respiratory distress and inability to feed and breathe at the same time. There is
use of accessory muscles and significantly increased work of breathing as the infant attempts to
inspire through a narrowed or obstructed nasal airway.

Upper airway obstruction in infants can occur at the level of the nose, pharynx, larynx, or
trachea. Potentially life-threatening obstructions are identified in the perinatal period. Other than
laryngomalacia, which often does not become symptomatic for several weeks after birth, or
vocal cord paralysis secondary to thoracic surgery in the newborn period, airway obstruction in
infants is most often associated with a congenital anomaly. Conditions that are not immediately
critical may be identified early but not become physiologically important for several weeks or
months. Item C192A outlines common causes of upper airway obstruction in infants and
children. Acute upper airway obstruction at any age is a medical emergency. Item C192B
localizes the site of obstruction for the most common airway obstructive processes.

American Academy of Pediatrics 651


PREP® Self-Assessment PREPSA 2023

In older children, infectious processes, foreign body obstruction, and airway injuries are more
prominent etiologies of airway obstruction. Viral croup is rarely life-threatening, but acute
epiglottitis and bacterial tracheitis may be lifethreatening in preschool and school-age children.
Chronic tonsillar hypertrophy is common and accompanied by symptoms of dynamic pharyngeal
obstruction during sleep. Children with preexisting neuromuscular diseases contributing to poor
pharyngeal tone are at increased risk for dynamic airway obstruction with sleep, even without
tonsillar hypertrophy. Often the poor airway tone becomes more pronounced as the child ages
and the underlying neurologic dysfunction progresses. Any subacute or chronic airway
obstruction may become severe and even potentially life-threatening with superimposition of an
acute infectious or inflammatory process on an already compromised airway.

PREP Pearls
• Upper airway obstruction at or above the glottis is associated with inspiratory stridor;
obstruction below the glottis is associated with biphasic or expiratory wheezing.
• Nasal obstruction in infants (eg, choanal atresia) is life-threatening, as infants are obligate
nose breathers. Acute upper airway obstruction at any age is a medical emergency.
• Airway obstruction can be fixed or dynamic; either type can be worsened by
superimposed infection or inflammation.

ABP Content Specifications(s)


• Identify the age-specific upper airway factors that may lead to respiratory distress

American Academy of Pediatrics 652


PREP® Self-Assessment PREPSA 2023

Suggested Readings
• Chinnadurai S, Goudy SL. Neonatal airway obstruction: an overview of diagnosis and
treatment. NeoReviews. 2013;14(3):e128-e137. doi:10.1542/neo.14-3-e128 .
• Conrad C, Cornfield DN Airway obstruction. In: McInerny TK, Adam HM, Campbell
DE, DeWitt TG, Foy JM, Kamat DM, eds. American Academy of Pediatrics Textbook of
Pediatric Care. American Academy of Pediatrics; 2021:chap 239. Accessed September 1,
2022. Pediatric Care Online.
• Eskander A, de Almeida JR, Irish JC. Acute upper airway infection. New Engl J Med.
2019;381: 1940-1949. doi:10.1056/NEJMra1811697.
• Vicenio AG, Parikh S. Laryngomalacia and tracheomalacia: common dynamic airway
lesions. Pediatr Rev. 2006;27(4):e33-e35; discussion e35. doi:10.1542/pir.27-4-e33.
• Virbalas J, Smith L. Upper airway obstruction. Pediatr Rev, 2015;35(6):62-72; quiz 73.
doi:10.1542/pir.36-2-62.
• Vo P, Kharasch VS. Respiratory failure. Pediatr Rev. 2014;35(11):476-484; quiz 485-6.
doi:10.1542/pir.35-11-476.

American Academy of Pediatrics 653


PREP® Self-Assessment PREPSA 2023

Question 193
A 2-year-old boy with recently diagnosed autism spectrum disorder is brought to the clinic for
concerns about his behavior. When he does not get his way, he throws tantrums and bangs his
head repeatedly against a wall. He does not sleep well, sometimes taking hours to fall asleep
after bedtime. He eats well but has intermittent episodes of nonbloody diarrhea that last 1 day
without associated abdominal pain. The boy has not yet started early intervention services or
applied behavior analysis therapy. His mother has been reading about autism spectrum disorder
and asks about a dietary supplement that can help with his behavioral problems.

Of the following, the MOST appropriate supplement to recommend is


A. cannabidiol oil
B. melatonin
C. probiotics
D. vitamin B6

American Academy of Pediatrics 654


PREP® Self-Assessment PREPSA 2023

Correct Answer: B
The boy in the vignette has recently diagnosed autism spectrum disorder (ASD) and is exhibiting
a number of common comorbid conditions, such as gastrointestinal complaints, sleep
disturbance, and behavioral problems, including self-injurious behaviors. Pediatricians should be
knowledgeable about the common coexisting medical and behavioral health conditions seen in
children with ASD (Item C193). Behavioral, developmental, and educational interventions are
the mainstay of treatment for ASD, including early intervention services for those younger than 3
years, and referral to the local school district for those 3 years and older. Additional interventions
include applied behavior analysis, parent management training, classroom-based models,
developmental relationship-focused interventions, naturalistic developmental behavioral
interventions, and speech and language therapy.

Many parents of children with autism spectrum disorder include complementary health
approaches to the management of their child’s symptoms. It is important to validate the parents’
concerns and support their desire to help their child. However, there is a lack of high-quality
research (eg, randomized controlled trials) evaluating the effectiveness of complementary health
approaches for children with ASD. Of the existing research, the only rigorously studied dietary
supplement that may improve symptoms for children with ASD is melatonin, which may be
beneficial for sleep disturbances. For the other dietary supplements listed as response choices,
there is insufficient evidence to recommend their use as supplements for modifying behavior in
children with ASD.

American Academy of Pediatrics 655


PREP® Self-Assessment PREPSA 2023

American Academy of Pediatrics 656


PREP® Self-Assessment PREPSA 2023

Although there is a paucity of high-quality research studies evaluating complementary


approaches to modifying ASD behaviors, many families may still choose to use supplements. It
is important to validate the family’s desire to use these complementary therapies. Physicians
should consider a "no harm" approach to support families by investigating the family’s preferred
therapy to ensure it will not cause harm; if no significant risk of harm is found, the physician can
support a trial of the therapy and advise the family to discontinue it if there is no positive effect.
If no safety data are available or there is concern of harm, physicians should make this clear to
families.

PREP Pearls
• Children with autism spectrum disorder are at increased risk for gastrointestinal
complaints, sleep disturbance, and self-injurious behaviors.
• Behavioral, developmental, and educational interventions are the mainstay of treatment
for autism spectrum disorder.
• Many parents of children with autism spectrum disorder include complementary health
approaches in the
• management of their child’s symptoms. It is important to validate parents’ concerns and
support their desire to help their child using a therapeutic trial of a “no harm” approach.

ABP Content Specifications(s)


• Evaluate available data regarding dietary or controversial perceptual/therapeutic
interventions for children with learning and behavioral problems, and provide appropriate
guidance while understanding a family’s motivation for seeking such treatment

Suggested Readings
• Kryszak E, Mulick JA, Butter EM. Autism spectrum disorder. In: McInerny TK, Adam
HM, Campbell DE, DeWitt TG, Foy JM, Kamat DM, eds. American Academy of
Pediatrics Textbook of Pediatric Care. American Academy of Pediatrics; 2021:chap 221.
Accessed September 1, 2022. Pediatric Care Online.
• Long M, Register-Brown K. Autism spectrum disorder. Pediatr Rev. 2021;42(7):360-374.
doi:10.1542/pir.2020-000547.
• Myers SM, Challman TD. Autism spectrum disorder. In: Voigt RG, Macias MM, Myers
SM, Tapid CD, eds. Developmental and Behavioral Pediatrics. 2nd ed. American
Academy of Pediatrics; 2018:407-475.
• National Center for Complementary and Integrative Health. Autism spectrum disorder
and complementary health approaches. National Institutes of Health. April 2021.
Accessed September 1, 2022. https://www.nccih.nih.gov/health/providers/digest/autism-
spectrum-disorder-and-complementary-health-approaches.

American Academy of Pediatrics 657


PREP® Self-Assessment PREPSA 2023

Question 194
The initial physical examination of a 12-hour-old neonate born at 39 weeks’ gestation reveals the
finding seen in Item Q194. The stretched penile length is 3.8 cm, and the scrotum is well
developed with bilateral descended testes. The remainder of his physical examination findings
are normal.

Item Q194: Finding for the neonate described in the vignette.


Reprinted with permission from Vricella GJ, Coplen DE. Neoreviews. 2017;18(6):e383.

Of the following, the BEST next step in this neonate’s management is to


A. defer circumcision
B. obtain a pediatric endocrinology consultation
C. obtain pelvic ultrasonography
D. obtain an urgent pediatric urology consultation

American Academy of Pediatrics 658


PREP® Self-Assessment PREPSA 2023

Correct Answer: A
The neonate in the vignette has distal shaft hypospadias (Item C194A). As in this case, a dorsal
hooded prepuce is often seen. Distal shaft hypospadias, which accounts for approximately 90%
of cases, is usually surgically repaired at 6 to 12 months of age. Circumcision, if desired, should
be deferred because the foreskin tissue may be used in the hypospadias repair. Parents should be
reassured that circumcision can be performed at the time of hypospadias repair.

Referral to a pediatric urologist is recommended, but an urgent urology consultation is not


indicated. The neonate in the vignette’s stretched penile length is normal for age (mean [SD], 3.9
[0.8] cm in term infants), and both testes are descended in a normal scrotum, making a disorder
of sexual development (DSD) unlikely; therefore, pelvic ultrasonography and a pediatric
endocrinology consultation are not indicated.

Hypospadias is due to incomplete development of the anterior urethra, resulting in a urethral


meatus located anywhere along the ventral penis from the proximal glans to the perineum. The
location of the urethral meatus is used to classify hypospadias into 3 subgroups: distal (glandular
or subcoronal), medial (distal penile or midshaft), or proximal (penoscrotal or perineal) (Item
C194B). Proximal hypospadias may be associated with chordee, a curvature of the phallus
(usually ventral); chordee is present in most cases of proximal hypospadias but may occur with
any subtype. The physical examination of a neonate with hypospadias should focus on
identifying other associated abnormalities, the exact location of the meatus, assessment of the
foreskin, and the presence of chordee.

Hypospadias is the most common anomaly of the male genitourinary tract, with an incidence of 1
in 200 male infants. Risk factors for hypospadias include a history of paternal hypospadias,
advanced maternal age, preexisting maternal diabetes, prenatal exposure to cigarette smoke or
pesticide, fetal growth restriction, and in vitro fertilization. Although most cases of hypospadias
are idiopathic, when hypospadias is associated with micropenis (phallus size <2.5 cm in term
male), with or without cryptorchidism, a DSD should be considered. In such cases, a karyotype
should be performed to confirm male genotype, and a multidisciplinary interprofessional team
evaluation and management approach may be indicated that involves pediatric endocrinology,
pediatric urology, gynecology, genetics, psychologists, and ethics teams.

Surgical repair of idiopathic cases of hypospadias without DSD or other genitourinary


abnormalities is usually performed between 6 and 12 months of age. The goal of repair is a penis
with normal appearance and function, which includes a urethral opening close to the ventral tip
of the penis, a properly directed urinary stream, and a penis that straightens on erection. In most
cases of hypospadias (especially idiopathic distal shaft subtype), parents and caregivers can be
reassured that surgical repair generally results in an acceptable cosmetic and functional penis.

American Academy of Pediatrics 659


PREP® Self-Assessment PREPSA 2023

PREP Pearls
• Circumcision should be deferred for neonates with hypospadias because the foreskin tissue
may be used in the surgical repair.
• Ninety percent of hypospadias cases are the distal shaft subtype.
• A neonate with hypospadias and micropenis, with or without undescended testes, should
have an evaluation for disorders of sexual development.

MOCA-Peds Objective
• Recognize and manage hypospadias in a neonate.

ABP Content Specifications(s)


• Recognize disorders associated with hypospadias
• Plan the appropriate management of hypospadias

Suggested Readings
• Diaz A, Lipman Diaz EG. Disorders of sex development. Pediatr Rev. 2021;42(8):414-
426. doi:10.1542/pir.2018-0183.
• Janjua HS, Lam SK, Gupta V, Krishna S. Congenital anomalies of the kidneys, collecting
system, bladder, and urethra. Pediatr Rev. 2019;40(12):619-626. doi:10.1542/pir.2018-
0242.
• Madhok N, Scharbach K, Shahid-Saless S. Hypospadias. Pediatr Rev. 2009;30(6):235-
237. doi:10.1542/pir.30.6.235.
• Snodgrass WT. Consultation with the specialist: hypospadias. Pediatr Rev.
2004;25(2):63-67. doi:10.1542/pir.25.2.63.
• Vricella GJ, Coplen DE. Neonatal urogenital issues: evaluation and management.
Neoreviews. 2017;18(6):e372-e385. doi:10.1542/neo.18-6-e372.

American Academy of Pediatrics 660


PREP® Self-Assessment PREPSA 2023

Question 195
A 7-year-old boy is seen for a health supervision visit. His adoptive mother has concerns about
his behavior. In his second grade classroom he distracts other children and has emotional
outbursts when his teacher tries to redirect him. During recess he gets into arguments and has
physical altercations with his peers. He reads at grade level and is capable of doing his
homework, but his mother has trouble getting him to start it. She also feels he needs closer
supervision at home compared to his peers to ensure his safety.
The boy was born at 39 weeks’ gestation and has a history of intrauterine methamphetamine
exposure. He was in foster care for 12 months before being re-unified with his biological mother.
At age 24 months he was found unkempt and without adult supervision in an automobile that
appeared to be his residence. He was placed in a new foster home, which subsequently resulted
in adoption. The boy’s physical examination findings are unremarkable.

Of the following, the MOST likely etiology of this boy’s behavior is


A. attention-deficit/hyperactivity disorder
B. complex trauma
C. conduct disorder
D. a learning disability

American Academy of Pediatrics 661


PREP® Self-Assessment PREPSA 2023

Correct Answers: B
Of the response choices, the most likely etiology of the boy in the vignette’s behavior is complex
trauma, defined as exposure to multiple traumatic events (eg, maltreatment). There are
significant long-term effects of these exposures. This type of trauma is often interpersonal in
nature, such as abuse or long-term neglect from a parent or caretaker, with associated disruption
in primary caregiver attachment. Childhood neglect is the most common form of childhood
maltreatment; 75% of reports to child protective services are for concerns of neglect.

Neglect is the failure of a parent or caregiver to provide for a child’s basic needs. Neglect can
result in a predictable change in neurologic functioning as the child develops. Symptoms include:
hypervigilance, anxiety, inattention, impulsiveness, emotional dysregulation, and difficulty
forming relationships.

Although some of the boy in the vignette’s symptoms are suggestive of attention-
deficit/hyperactivity disorder (ADHD), his history of neglect is more likely the causative factor.
There is a clear overlap of these symptoms with the diagnostic criteria for ADHD; however, the
constellation of symptoms paired with the boy’s history makes complex trauma a more accurate
diagnosis and will guide a more targeted management plan.

A diagnosis of conduct disorder requires evidence of aggression toward people or animals,


destruction of property, and serious violation of rules. This diagnosis is not supported by this
boy’s behavior. Likewise, the boy’s behavior and school performance are not consistent with a
learning disability.

PREP Pearls
• Complex trauma involves multiple traumatic events often associated with parental or
caregiver neglect.
• Neglect is the most common form of child maltreatment.
• Attention-deficit/hyperactivity disorder may be comorbid with complex trauma disorder;
it is important to recognize the trauma disorder to ensure comprehensive treatment.

ABP Content Specifications(s)


• Understand the emotional and behavioral consequences of neglect
• Recognize the historical and clinical findings associated with child neglect, including
findings associated with physical neglect or neglect of medical care

American Academy of Pediatrics 662


PREP® Self-Assessment PREPSA 2023

Suggested Readings
• Council on Foster Care; Adoption, and Kinship Care; Committee on Adolescence, and
Council on Early Childhood. Health care issues for children and adolescents in foster care
and kinship care. Pediatrics. 2015;136(4):e1131-e1140. doi:10.1542/peds.2015-2655.
• Dubowitz H, Finkel MA. Physical abuse and neglect. In: McInerny TK, Adam HM,
Campbell DE, DeWitt TG, Foy JM, Kamat DM, eds. American Academy of Pediatrics
Textbook of Pediatric Care. American Academy of Pediatrics; 2021:chap 367. Accessed
September 1, 2022. Pediatric Care Online.
• Dubowitz H, Giardino A, Gustavson E. Child neglect: guidance for pediatricians. Pediatr
Rev. 2000;21(4):111-116. doi:10.1542/pir.21-4-111.
• Jenny C, Metz JB. Medical child abuse and medical neglect. Pediatr Rev. 2020;41(2):49-
60. doi:10.1542/pir.2017-0302.
• Wilson HW, Joshi SV. Recognizing and referring children with posttraumatic stress
disorder: guidelines for pediatric providers. Pediatr Rev. 2018;39(2):68-77.
doi:10.1542/pir.21-4-111.
• Copyright © 2023 American Academy of Pediatrics. All rights reserved.
• Child Abuse and Neglect

American Academy of Pediatrics 663


PREP® Self-Assessment PREPSA 2023

Question 196
A term newborn male, intubated for respiratory distress, is in the neonatal intensive care unit.
Physical examination shows a cleft palate, hooded eyelids, prominent nasal bridge, bulbous nose,
and micrognathia. The remainder of his physical examination findings are normal. Significant
findings on further evaluation include: tetralogy of Fallot, absent thymus, and a total serum
calcium level of 7.8 mg/dL (1.9 mmol/L).

Of the following, this neonate’s MOST likely diagnosis is


A. 22q11.2 deletion syndrome
B. trisomy 13
C. trisomy 21
D. Turner syndrome

American Academy of Pediatrics 664


PREP® Self-Assessment PREPSA 2023

Correct Answer: A
The neonate in the vignette has 22q11.2 deletion syndrome (22q11.2 DS) or velocardiofacial
syndrome. Clinical features of 22q11.2 DS are listed in Item C196.

Clinical features of 22q11.2 DS


• Congenital heart disease (in 64% of individuals), particularly conotruncal defects (e.g.,
ventricular septal defect, tetralogy of Fallot, interrupted aortic arch, truncus arteriosus)
• Palatal abnormalities (in 67%) including velopharyngeal insufficiency, submucosal cleft palate,
bifid uvula, cleft palate, and hypernasal speech, dysphagia
• Laryngotracheoesophageal abnormalities including vascular ring, laryngeal web,
laryngotracheomalacia, and subglottic stenosis
• Gastrointestinal anomalies including constipation with or without structural gastrointestinal
anomalies (e.g., anteriorly placed/imperforate anus, esophageal atresia, jejunal atresia,
intestinal malrotation, Hirschsprung disease), accessory spleens, diaphragmatic hernia,
umbilical hernia, and inguinal hernia
• Immune deficiency (in 77%) (e.g., frequent infections, thymic hypoplasia)
• Autoimmune disorders (e.g., juvenile rheumatoid arthritis, Grave's disease, vitiligo)
• Ophthalmologic findings including tortuous retinal vessels, ptosis, posterior embryotoxon,
sclerocornea, coloboma, cataract, anophthalmia, and strabismus
• Other craniofacial features (e.g., hooded eyelids, ear anomalies, prominent nasal bridge, bulbous
nose, micrognathia, asymmetric crying facies, craniosynostosis)
• Hearing loss (sensorineural and/or conductive)
• CNS abnormalities including hypotonia in infancy, microcephaly, polymicrogyria, and seizures
(idiopathic or associated with hypocalcemia)
• Developmental delay and/or learning difficulties (in 70%-90%), especially a nonverbal learning
disability
• Psychiatric illness including autism spectrum disorder (20% of children), schizophrenia (25% of
adults), attention deficit disorder, anxiety, perseveration, and difficulty with social interactions
• Early-onset Parkinson disease
• Skeletal anomalies including occipital-cervical anomaly, scoliosis, rib and vertebral anomalies,
clubfoot, and polydactyly
• Genitourinary tract anomalies including renal anomalies (in 16%) (e.g., hydronephrosis, renal
agenesis, multicystic/dysplastic kidney), cryptorchidism, and hypospadias

The diagnosis of 22q11.2 DS is confirmed by chromosomal microarray (CMA). Testing with


fluorescent in situ hybridization (FISH) for the 22q11.2 critical region is reserved for neonates in
critical condition with classic clinical features for whom a quick result is needed. Fluorescent in
situ hybridization for the 22q11.2 DS critical region can miss distal deletions, hence CMA is the
diagnostic test of choice. 22q11.2 DS is de novo in around 90% of affected individuals.
Recurrence risk is 50%, inherited in an autosomal dominant manner.

Trisomy 21 is characterized by a distinctive facial profile with up-slanted palpebral fissures,


midface hypoplasia, and epicanthal folds. Clinical features include congenital heart defect, single
transverse palmar crease, and a wide space between the first and second toes. Common cardiac
defects include atrioventricular septal defect, ventricular septal defect, and secundum septal
defect. Hypoparathyroidism and hypocalcemia are not associated features.
American Academy of Pediatrics 665
PREP® Self-Assessment PREPSA 2023

Turner syndrome is a sex chromosome aneuploidy characterized by webbed neck, down slanted
palpebral fissures, shield chest, wide-set nipples, and pectus excavatum. Common cardiac
defects include coarctation of the aorta and bicuspid aortic valve.

Clinical features of trisomy 13 include small for gestational age, failure to thrive, microcephaly,
cutis aplasia, cleft palate, holoprosencephaly, and postaxial polydactyly. Tetralogy of Fallot can
be seen in trisomy 13. However, the neonate in the vignette has hypocalcemia and an absent
thymus; his clinical features make 22q11.2 DS a more likely diagnosis.

PREP Pearls
• 22q11.2 deletion syndrome is characterized by conotruncal heart defects, cleft palate,
immune deficiency,
• dysmorphic facial features, and endocrine abnormalities (eg, hypoparathyroidism leading
to hypocalcemia).
• The diagnostic test of choice for 22q11.2 deletion syndrome is a chromosomal
microarray. Fluorescent in situ hybridization for the 22q11.2 critical region is reserved
for a critically ill neonate with classic clinical features in whom a rapid turnaround time
is required.

ABP Content Specifications(s)


• Recognize the features of velocardiofacial syndrome and plan appropriate diagnostic
evaluation

Suggested Readings
• LaTuga MS, McDonald M, Jiang YH. Common congenital anomalies. In: McInerny TK,
Adam HM, Campbell DE, DeWitt TG, Foy JM, Kamat DM, eds. American Academy of
Pediatrics Textbook of Pediatric Care. American Academy of Pediatrics; 2021:chap 96.
Accessed September 1, 2022. Pediatric Care Online.
• McDonald-McGinn DM, Hain HS, Emanuel BS, et al., eds. 22q11.2 deletion syndrome.
GeneReviews [Internet]. University of Washington, Seattle; 2021.
https://www.ncbi.nlm.nih.gov/books/NBK1523/.
• Saul RA. Specific genetic conditions. In: AAP Committee on Genetics. Medical Genetics
in Pediatric Practice. American Academy of Pediatrics; 2014:chap 9.

American Academy of Pediatrics 666


PREP® Self-Assessment PREPSA 2023

Question 197
A 5-day-old male neonate is brought to the emergency department for evaluation of orange-red
urine spots in his diaper (Item Q197). He was born at 38 weeks’ gestation by uncomplicated
vaginal delivery. He has no history of fever, decreased activity, vomiting, or diarrhea. He is
exclusively breastfed. The neonate’s temperature is 37.5°C, heart rate is 136 beats/min,
respiratory rate is 36 breaths/min, and blood pressure is 70/40 mm Hg. He is alert and active; the
remainder of his physical examination findings are unremarkable. A stool guaiac test result is
negative.

Item Q197: Diaper from the neonate described in the vignette.


Reprinted with permission from Whattoexpect.com

Results of urinalysis with microscopy are shown:


Urine Result
Specific gravity 1.010
Leukocyte esterase Negative
Nitrite Negative
Blood Negative
Protein Negative
Red blood cells 1-2/HPF
White blood cells <5/HPF

Of the following, the neonate’s MOST likely diagnosis is


A. hemoglobinuria
B. hypercalciuria
C. uric acid crystalluria
D. urinary tract infection

American Academy of Pediatrics 667


PREP® Self-Assessment PREPSA 2023

Correct Answer: C

American Academy of Pediatrics 668


PREP® Self-Assessment PREPSA 2023

Question 198
The physicians at a pediatric clinic are interested in investing in a screening tool for adolescent
depression. The tool screens for symptoms of depression including changes in sleep, appetite,
energy level, and behavior, and for suicidal ideation. For 3 months, pediatricians in the practice
used the screening tool while continuing their standardized clinical assessment for depression
using a validated interview approach. Results from both the clinical assessment and the screening
tool were compared to determine if the tool can be used in lieu of the longer, more time-
consuming, standardized interview.

Of the following, this project is BEST designed to measure


A. convergent validity
B. internal consistency reliability
C. predictive validity
D. test-retest reliability

American Academy of Pediatrics 669


PREP® Self-Assessment PREPSA 2023

Correct Answer: A
The project described in the vignette is designed to measure convergent validity. Validity
evaluates the extent to which a measurement or finding is accurate or true (ie, does it accurately
measure what it is intended to measure?). The 2 major categories of validity are internal validity
and external validity. Internal validity examines the ability of a tool to accurately measure the
intended condition. External validity or generalizability, measures the extent to
which the tool can be used in a broader population. For example, if the depression screening tool
was only tested for adolescents between the ages of 16 and 18 years of age, generalizability
seeks to evaluate if the test can be used in a broader age range.

Two types of internal validity include:


• Convergent validity: compares 2 ways of measuring the same thing to determine if they
obtain the same result. For example, the project in the vignette was designed to measure
whether the shorter screening tool yielded the same results as the longer standardized
clinical interview.
• Predictive validity: evaluates the accuracy of a tool to measure a future event or
outcome. For example, the pediatric practice could perform a chart review of all
adolescents screened for depression using the new screening tool and determine how
many developed a depressive episode within a specified time period.

Reliability of a tool or measurement evaluates its ability to replicate findings. For example, if the
same depression screening tool is administered several times to the same individual or group
would it produce the same results?

Types of reliability include:


• Test-retest reliability: measures the ability of an instrument or tool to produce the same
results with
• repeated measurements. Administration of the tool to the same individual or sample
group and comparison of those results provides a measurement of test-retest reliability.
• Interrelater reliability: measures the ability of 2 individuals to independently produce
the same results. For example, are the results from the depression screening tool the same
when administered by 2 different individuals?
• Internal consistency reliability: is a measure of how consistent the items within a single
instrument are with each other.

PREP Pearls
• Validity evaluates the extent to which a measurement is accurate or true. Internal validity
is the ability of a test to declare a causal relationship between a measurement and
outcome. External validity (generalizability) is the ability of the tool, finding, or
measurement to be applied outside the study setting.
• Reliability is the ability to replicate the same results. Test-retest reliability measures the
ability of an instrument or tool to produce the same results with repeated measurements.
Interrelater reliability measures the ability for 2 individuals to independently produce the
same results.
American Academy of Pediatrics 670
PREP® Self-Assessment PREPSA 2023

MOCA-Peds Objective
• Understand factors that influence generalizability of a research study.

ABP Content Specifications(s)


• Understand generalizability and how it relates to validity
• Understand validity and how it might be compromised
• Understand reliability and how it might be compromised

Suggested Readings
• Copeland-Linder N. Research and statistics: reliability and validity in pediatric practice.
Pediatr Rev. 2009;30(7):278-279. doi:10.1542/pir.30.7.278.
• Shenoi R, Linakis J, Bromberg J, et al; Pediatric Emergency Care Applied Research
Network. Predictive validity of the CRAFFT for substance use disorder. Pediatrics.
2019;144(2):e20183415. doi:10.1542/peds.2018-3415.
• Spirito A, Bromberg J, Casper TC, et al; Pediatric Emergency Care Applied Research
Network. Reliability and validity of a two-question alcohol screen in the pediatric
emergency department. Pediatrics. 2016;13(6):e20160691.
• doi:10.1542/peds.2016-0691.
• Sullivan GM. A primer on the validity of assessment instruments. J Grad Med Educ.
2011;3(2):119-120. doi:10.4300/JGME-D-11-00075.1.

American Academy of Pediatrics 671


PREP® Self-Assessment PREPSA 2023

Question 199
A full-term, 2-day-old male neonate in the newborn nursery has a platelet count of 48.0 × 103/µL
(48.0 × 109/L). A complete blood cell count was ordered after his parents reported that his older
sister had thrombocytopenia at birth. The sister is now doing well without any medical problems.
The neonate’s physical examination findings are normal.

Of the following, the BEST next step in this neonate’s management is


A. head ultrasonography
B. intravenous immune globulin administration
C. observation
D. platelet administration

American Academy of Pediatrics 672


PREP® Self-Assessment PREPSA 2023

Correct Answer: A
The neonate in the vignette has moderate thrombocytopenia (platelet count <50.0 × 103/µL [50.0
× 109/L]) but is otherwise stable with normal physical examination findings. The family history
of thrombocytopenia makes neonatal (or fetal) alloimmune thrombocytopenia (NAIT) a likely
diagnosis. Head ultrasonography is indicated at this time due to his level of thrombocytopenia
and the family history of thrombocytopenia in a previous pregnancy.

Neonatal alloimmune thrombocytopenia occurs as a result of a difference between the human


platelet antigens (HPAs) of the mother and father. In response to transplacental exposure, when
the fetus’ platelets have the father’s HPA, the mother’s immune system recognizes the fetus’
antigen as foreign and forms a platelet antibody towards the fetus’ platelets. The maternal IgG
antibody can then cross the placenta and destroy the fetus’ platelets. Affected neonates may have
signs and symptoms of moderate to severe thrombocytopenia (eg, petechiae, bruising, or
bleeding) and have otherwise normal findings, making genetic or infectious causes of
thrombocytopenia less likely. Intracranial hemorrhage may occur in 10% to 20% of cases of
NAIT and can be severe. Therefore, a neonate with moderate to severe thrombocytopenia
requires assessment for intracranial bleeding with head ultrasonography. Additional brain
imaging may be indicated if there are concerning signs or symptoms suggestive of intracranial
bleeding.

A neonate with moderate to severe thrombocytopenia without any plausible explanation should
be evaluated for NAIT. The evaluation may consist of detection of maternal antibodies in the
mother or fetus; however, not all antibodies are known. Therefore, the best way to diagnose
NAIT is by determining a difference in the HPA profile of the mother and father.
A family history of NAIT in a previous pregnancy may warrant antenatal treatment during
subsequent pregnancies to prevent a potentially fatal outcome; there is significant risk that
thrombocytopenia during subsequent pregnancies may be more severe and occur earlier.
Antenatal treatment of NAIT with intravenous immune globulin (IVIG) with or without steroids
has had some success. Intrauterine platelet transfusions have also been tried, however, this
treatment presents risks to the fetus, including possible death.

Postnatal treatment of NAIT with significant bleeding or severe thrombocytopenia includes


platelet transfusions (preferably with a matched donor). If maternal platelets are used, they must
be washed to remove the antibody. The neonate in the vignette has moderate thrombocytopenia
and does not have signs or symptoms of bleeding, so platelet transfusion is not indicated.
Intravenous immune globulin is a treatment option for NAIT. However, as the neonate in the
vignette is clinically stable and can have his platelet counts monitored closely, treatment with
IVIG is not indicated at this time but can be administered in the future if warranted. Observation
alone is not appropriate management for the neonate in the vignette. It is crucial to recommend
that the parents undergo testing to determine their HPA profile so they can be counseled
appropriately about the risk of NAIT with future pregnancies.

American Academy of Pediatrics 673


PREP® Self-Assessment PREPSA 2023

Because NAIT results from transplacental crossing of an IgG antibody, the neonate’s platelet
count should improve to normal within about a month (the half-life of IgG). Affected neonates
require close monitoring until the platelet count increases to a normal level.

Neonatal autoimmune thrombocytopenia occurs when a mother with autoimmune


thrombocytopenia has antiplatelet antibodies that cross the placenta and destroy the fetus’
platelets. In this case, the mother may also have thrombocytopenia, whereas in NAIT the
maternal platelet count remains normal.

PREP Pearls
• Neonatal (fetal) alloimmune thrombocytopenia should be ruled out in any neonate with
unexplained, moderate to severe thrombocytopenia.
• Head ultrasonography should be ordered for a neonate with moderate to severe
thrombocytopenia to evaluate for evidence of intracranial bleeding.
• Neonatal (fetal) alloimmune thrombocytopenia can be more severe and occur earlier in
future pregnancies; antenatal treatment may be warranted.

ABP Content Specifications(s)


• Understand the significance of neonatal thrombocytopenia in multiple siblings

Suggested Readings
• Fernández KS, de Alarcón P. Neonatal thrombocytopenia. Neoreviews. 2013;14(2):e74-
e82. doi:10.1542/neo.14-2-e74.
• Haley KM. Platelet disorders. Pediatr Rev. 2020;41(5):224-235. doi:10.1542/pir.2018-
0359.
• Lanzkowsky P, Lipton J, Fish JD, eds. Lanzkowsky’s Manual of Pediatric Hematology
and Oncology. 6th ed. Elsevier Inc; 2016:239-254.
• Morrone K. Thrombocytopenia in the newborn. Neoreviews. 2018;1(1):e34e41.
doi:10.1542/neo.19-1-e34.

American Academy of Pediatrics 674


PREP® Self-Assessment PREPSA 2023

Question 200
A 15-year-old adolescent girl is seen for a preparticipation physical examination before the start
of her high school basketball season. The girl’s twin sister tore her anterior cruciate ligament
(ACL) while playing soccer last year. The girl and her family would like advice on how to
prevent ACL injuries. On physical examination, the girl has generalized joint hypermobility with
15 degrees of knee extension bilaterally. She also has flexible pes planovalgus (ankle pronation).
The remainder of her physical examination findings are normal.

Of the following, the measure MOST likely to decrease this girl’s risk of injury is
A. participation in a neuromuscular training program
B. performance of stretching exercises prior to sports practice
C. use of a hinged knee brace during sports
D. use of shoe inserts with arch supports

American Academy of Pediatrics 675


PREP® Self-Assessment PREPSA 2023

Correct Answer: A
The girl in the vignette exhibits anatomic factors that increase her risk of anterior cruciate
ligament (ACL) injury.

She has generalized joint hypermobility with laxity of the ligaments around the knees and
flexible pes planovalgus. Additionally, she has a twin sister who tore her ACL; genetic factors
are thought to play a role in ACL injury. These intrinsic features are not modifiable risk factors.
Of the response choices, only participation in a neuromuscular training program has been shown
to lower the risk of ACL injury.

Anterior cruciate ligament injuries often occur while engaging in sports that involve jumping
and/or sudden changes in running direction. Most ACL tears are non-contact injuries. Adolescent
girls have higher rates of ACL injury as compared to adolescent boys. The rate of ACL injury in
high school girls’ basketball is nearly 5 times higher than in boys’ basketball; girls are 3 times
more likely than boys to tear their ACL playing soccer. Sex differences in biomechanics with
jumping and cutting maneuvers play a role in the higher rates of ACL tears in female athletes.
Neuromuscular training programs teach exercises that increase strength, improve proprioception,
and provide feedback on proper form while landing from a jump, squatting, and other sport-
specific motions.

The typical treatment of a torn ACL is surgical reconstruction of the ligament. Young athletes
who tear their ACL have a dramatically increased risk of developing arthritis; surgical
reconstruction does not mitigate this risk. Additionally, an individual who suffers an ACL tear
has an increased risk of both a repeat tear on the ipsilateral side and a new tear on the
contralateral side. For young athletes at high risk of ACL tear, the American Academy of
Pediatrics recommends that pediatricians provide counseling about the benefits of neuromuscular
training programs.

Preventative bracing for those at increased risk of ACL tear and the use of shoe inserts for
individuals with flexible flat feet (pes planovalgus or ankle pronation) have not been shown to
reduce rates of ACL injury. Static stretching prior to exercising is not recommended as this may
increase the risk of joint injuries.

PREP Pearls
• Adolescent girls have higher rates of anterior cruciate ligament tears than boys.
• Neuromuscular training programs have been shown to decrease the rate of anterior
cruciate ligament injuries.

ABP Content Specifications(s)


• Understand the role of conditioning in preventing injuries in athletes of various ages

American Academy of Pediatrics 676


PREP® Self-Assessment PREPSA 2023

Suggested Readings
• Coleman N. Sports injuries. Pediatr Rev. 2019;40(6):278-290. doi:10.1542/pir.2018-
0221.
• Internal derangement of the lnee. In: Sarwark JF, LaBella CR. Pediatric Orthopaedics and
Sports Injuries: A Quick Reference Guide. 3rd ed. American Academy of Pediatrics;
2021:355-375.
• Joseph AM, Collins CL, Henke NM, Yard EE, Fields SK, Comstock RD. A multisport
epidemiologic comparison of anterior cruciate ligament injuries in high school athletics. J
Athl Train. 2013;48(6):810-817. doi:10.4085/1062-6050-48.6.03.
• Kannikeswaran N, Suresh S. Sports musculoskeletal injuries. In: McInerny TK, Adam
HM, Campbell DE, DeWitt TG, Foy JM, Kamat DM, eds. American Academy of
Pediatrics Textbook of Pediatric Care. American Academy of Pediatrics; 2021:chap 348.
Accessed September 1, 2022. Pediatric Care Online.
• LaBella CR, Hennrikus W, Hewett TE. Anterior cruciate ligament injuries: diagnosis,
treatment, and prevention. Pediatrics. 2014;133(5):e1437-e1450. doi:10.1542/peds.2014-
0623.
• Wolf S. ACL injuries in young athletes. American Academy of Pediatrics. Updated
March 29, 2019. Accessed September 1, 2022.
https://www.healthychildren.org/English/health-issues/injuries-emergencies/sports-
injuries/Pages/ACLInjuries.aspx.

American Academy of Pediatrics 677


PREP® Self-Assessment PREPSA 2023

Question 201
A neonate is evaluated in the newborn nursery 2 hours after birth. She was born at 36 weeks’
gestation to a 24-yearold woman with a history of gestational diabetes and smoking during
pregnancy. The mother’s membranes ruptured 20 hours before delivery. The infant’s Apgar
scores were 7 and 9 at 1 and 5 minutes, respectively. The neonate’s weight is 1,860 g (3rd
percentile), length is 44 cm (10th percentile), and head circumference is 32 cm (25th percentile).
She is alert, active, and pink with no respiratory distress. The remainder of her physical
examination findings are normal.

Of the following, the MOST likely cause of this neonate’s low birthweight is
A. a chromosomal abnormality
B. fetal hypoxia
C. low insulin level
D. sepsis

American Academy of Pediatrics 678


PREP® Self-Assessment PREPSA 2023

Correct Answers: B
The neonate in the vignette is small for gestational age (SGA), most likely due to maternal
smoking during pregnancy. Small for gestational age is defined as a birthweight below the 10th
percentile for gestational age. Etiologic factors in SGA include conditions causing placental
insufficiency, congenital viral infections, chromosomal and genetic conditions, and toxins such
as cigarette smoke, alcohol, and illicit drugs.

Nicotine readily crosses the placenta and is found in higher concentrations in the fetus than in the
mother. Nicotine affects oxygen delivery to the fetus, causing a state of chronic hypoxia. Women
who smoke during pregnancy are twice as likely to have an SGA infant compared with women
who have never smoked. The effects of nicotine are dose dependent, with an odds ratio of 2.5 for
women who smoke more than 10 cigarettes per day. Smoking is also associated with decreased
length and head circumference. In addition to growth restriction, nicotine use during pregnancy
has been associated with anatomic abnormalities such as cleft lip/palate, neural tube defects,
gastroschisis, jejunal/ileal atresias, and cryptorchidism. Postnatally, neonates may exhibit signs
of withdrawal such as irritability, hyperexcitability, and tremors. Prenatal and postnatal smoking
exposure has been associated with sudden infant death syndrome. Long-term cognitive effects,
learning disabilities, and poor school performance are all seen in school-age children exposed to
nicotine prenatally. E-cigarettes and nicotine replacement therapies also pose a risk to the fetus.
Second- and third-hand smoking exposure can cause similar effects as first-hand smoking.

The neonate in the vignette does not have clinical features suggestive of a chromosomal
abnormality. The neonate’s mother had gestational diabetes, which is associated with high levels
of insulin and upregulation of insulin-like growth factors in the fetus, leading to fetal
macrosomia. Although the neonate is at risk for sepsis due to prematurity and prolonged rupture
of membranes, she does not have any symptoms or signs of sepsis, and sepsis would not explain
the finding of SGA.

PREP Pearls
• Maternal smoking during pregnancy causes fetal growth restriction and is a common
cause for small-for-gestational age neonates.
• Nicotine use during the first trimester of pregnancy is associated with anatomic
abnormalities in the fetus (eg, cleft lip/palate, neural tube defects, gastroschisis,
jejunal/ileal atresia, and cryptorchidism).
• E-cigarettes and nicotine replacement products have the same toxic effects as smoking
during pregnancy.

ABP Content Specifications(s)


• Recognize the effects of maternal smoking on a fetus

American Academy of Pediatrics 679


PREP® Self-Assessment PREPSA 2023

Suggested Readings
• Bailey NA, Diaz-Barbosa M. Effect of maternal substance abuse on the fetus, neonate,
and child. Pediatr Rev. 2018;39(11):550-559. doi:10.1542/pir.2017-0201.
• Jackson K, Harrington JW. SGA and VLBW infants: outcomes and care. Pediatr Rev.
2018;39(7):375-377. doi:10.1542/pir.2017-0091.
• Jansson LM, Velez ML. Infants of drug-dependent mothers. Pediatr Rev. 2011;32(1):5-
12, quiz 12-13. doi:10.1542/pir.32-1-5.
• Nafday SM. Abnormalities of fetal growth. In: McInerny TK, Adam HM, Campbell DE,
DeWitt TG, Foy JM, Kamat DM, eds. American Academy of Pediatrics Textbook of
Pediatric Care. American Academy of Pediatrics; 2021:chap 98. Accessed September 1,
2022. Pediatric Care Online.
• Quelhas D, Kompala C, Wittenbrink B, et al. The association between active tobacco use
during pregnancy and growth outcomes of children under five years of age: a systematic
review and meta-analysis. BMC Public Health. 2018;18(1):1372. doi:10.1186/s12889-
018-6137-7.
• Zhou S, Rosenthal DG, Sherman S, Zelikoff J, Gordon T, Weitzman M. Physical,
behavioral, and cognitive effects of prenatal tobacco and postnatal secondhand smoke
exposure. Curr Probl Pediatr Adolesc Health Care. 2014;44(8):219241.
doi:10.1016/j.cppeds.2014.03.007.

American Academy of Pediatrics 680


PREP® Self-Assessment PREPSA 2023

Question 202
A 6-month-old boy is brought to the emergency department for excessive crying over the past 4
hours. He began crying abruptly when his mother was walking him in his stroller and has been
inconsolable and rubbing his eyes since then. He has had no fevers or recent illness. The boy’s
physical examination findings are normal except for mild injection of his left conjunctiva and
clear tearing from both eyes. Fluorescein stain is applied to both eyes, and results of a Wood
lamp examination of the left eye are shown in Item Q202.

Of the following, the MOST appropriate therapy for this boy is application of
A. an eye patch for 72 hours
B. steroid ophthalmic drops
C. tetracaine ophthalmic drops
D. topical ophthalmic antibiotic

American Academy of Pediatrics 681


PREP® Self-Assessment PREPSA 2023

Correct Answers: D
The boy in the vignette has a corneal abrasion (Item C202). The most appropriate treatment is
application of topical ophthalmic antibiotic ointment or drops to the affected eye until the
abrasion is healed. Corneal abrasions are very common in childhood. Typical signs and
symptoms include pain, tearing, redness, blurry vision, photophobia, and foreign body sensation.
Children may also have mild eyelid swelling or redness resultant from mechanical irritation of
rubbing the eye. In infants and very young children the only symptom of a corneal abrasion may
be fussiness or excessive crying.

Corneal abrasions are defects in the corneal epithelium. Tetracaine eye drops may be used for
anesthesia to facilitate the fluorescein stain eye examination but should not be prescribed for
home use; tetracaine overuse can cause corneal cell death and inhibit corneal healing.
Application of an eye patch for corneal abrasions is usually not recommended. Corticosteroid
eye drops are not indicated to treat corneal abrasions; they may delay healing and increase the
risk of infection.

Careful examination of the eye is required for any child with eye pain, eye injury, or excessive
crying. A topical anesthetic, such as tetracaine, may be used to anesthetize the eye for proper
examination. The eye should be gently opened, with care taken to avoid any pressure on the
globe. The eye should be carefully examined for retained foreign bodies; this includes eversion
of both the upper and lower eyelids and examination for foreign bodies within the globe.
Evaluation for globe rupture, hyphema, and lacerations should be performed prior to staining the
eye with fluorescein. The eye should be stained with fluorescein, and an examination performed
with a cobalt blue light, using a standard ophthalmoscope or a Wood lamp. Areas of abnormality
will fluoresce green. Multiple linear abrasions of the cornea suggest that an upper lid foreign
body may be embedded in the conjunctiva causing repeated abrasions with each blink of the eye.
Corneal abrasions heal very quickly. Treatment includes a topical antibiotic eye ointment or
drop. Oral non-steroidal anti-inflammatory medications may be administered as needed to treat
pain in children older than 6 months.

Children with a corneal abrasion should be re-examined in 24 hours. Urgent ophthalmologic


consultation is indicated if the abrasion is large or centrally located (impairing the central visual
axis), corneal infiltrates are seen, if there are active signs of infection, or the abrasion is not
healed after 3 to 4 days of treatment.

American Academy of Pediatrics 682


PREP® Self-Assessment PREPSA 2023

PREP Pearls
• Signs and symptoms of a corneal abrasion include pain, tearing, redness, blurry vision,
photophobia, and foreign body sensation; it may present as excessive fussiness or crying
in young infants.
• The treatment of a corneal abrasion is application of topical ophthalmic antibiotic
ointment or drops to the affected eye until the abrasion is healed.
• Tetracaine eye drops should not be prescribed for home use; tetracaine overuse can cause
corneal cell death and inhibit corneal healing.
• Suspected corneal abrasions should be stained with fluorescein, and an examination
performed with a cobalt blue light, using a standard ophthalmoscope or a Wood lamp.

ABP Content Specifications(s)


• Plan the appropriate clinical evaluation of a suspected foreign body in the eye
• Recognize the clinical findings associated with corneal abrasion in patients of various
ages
• Plan the appropriate initial and follow-up management of a corneal abrasion

Suggested Readings
• Bohra LI, Rao RC. Ocular trauma. In: McInerny TK, Adam HM, Campbell DE, DeWitt
TG, Foy JM, Kamat DM, eds. American Academy of Pediatrics Textbook of Pediatric
Care. American Academy of Pediatrics; 2021:chap 300. Accessed September 1, 2021.
Pediatric Care Online .
• Browner EA. Corneal abrasions. Pediatr Rev. 2012;33(6):285-286.
doi:10.1542/pir.33.6.285.
• Stout AU. Corneal abrasions. Pediatr Rev. 2006;27(11):433-434.
doi:10.1542/pir.27.11.433.

American Academy of Pediatrics 683


PREP® Self-Assessment PREPSA 2023

Question 203
A 4-hour-old male neonate is admitted to the neonatal intensive care unit for cyanosis. The
maternal history is unremarkable, and the mother received good prenatal care. He was born by
spontaneous vaginal delivery at 39 weeks’ gestation. He has a heart rate of 160 beats/min,
respiratory rate of 50 breaths/min, and blood pressure of 65/35 mmHg. Pulse oximetry readings
are 85% on all 4 extremities in room air. His lungs are clear to auscultation, there is no increased
work of breathing. A high-pitched systolic ejection murmur is heard over the left upper sternal
border, and good pulses are palpated throughout. Physical examination findings are otherwise
unremarkable. A prostaglandin infusion is started.

Of the following, this neonate’s MOST likely diagnosis is


A. aortic valve stenosis
B. mitral valve stenosis
C. pulmonary valve stenosis
D. tricuspid valve stenosis

American Academy of Pediatrics 684


PREP® Self-Assessment PREPSA 2023

Correct Answer: C
The neonate in the vignette has cyanosis, no respiratory distress, low pulse oximetry readings in
all extremities, and a systolic ejection murmur in the pulmonic location. These findings are most
consistent with pulmonary valve stenosis. If the neonate had aortic valve stenosis, the murmur
would be more prominent at the right upper sternal border (aortic valve position), and the right
upper extremity saturation would be higher than the lower extremity saturation. Stenosis of either
atrioventricular valve (mitral or tricuspid) would result in a diastolic murmur.

Pulmonary valve stenosis is typically an acyanotic cardiac lesion. However, in a neonate with
severe stenosis that limits pulmonary blood flow or one that has right ventricular pressures high
enough to result in right-to-left shunting at the atrial level, desaturation can occur. In that
situation, prostaglandin infusion should be initiated to open or maintain a patent ductus
arteriosus. In the event that the pressure gradient across the pulmonary valve is severe or critical
(ductal dependent), the neonate should undergo intervention to open the valve in order to
decrease the right ventricular pressure. This procedure is typically done in the cardiac
catheterization laboratory using a balloon-tipped catheter (ie, balloon valvuloplasty).

Neonates and infants can have severe pulmonary valve stenosis and not be desaturated. On
physical examination, a loud systolic ejection murmur will be heard in the pulmonic position
(left upper sternal border). Children who have mild to moderate pulmonary valve stenosis may
be asymptomatic but will have a pulmonic murmur. The degree of pulmonary valve stenosis can
increase over time. Symptoms such as fatigue, decreased oral intake, and mild exertional
dyspnea can develop as the degree of stenosis worsens. These children should be referred to a
pediatric cardiologist for management.

PREP Pearls
• The murmur of pulmonary valve stenosis is a middle- to high-pitch systolic ejection
murmur best heard at the left upper sternal border.
• Neonates with critical pulmonary stenosis will be desaturated due to limited pulmonary
blood flow and right-to-left shunting, primarily at the atrial level.
• Neonates with critical and severe pulmonary stenosis should undergo balloon
valvuloplasty.

ABP Content Specifications(s)


• Plan appropriate initial management of severe pulmonary valve stenosis

American Academy of Pediatrics 685


PREP® Self-Assessment PREPSA 2023

Suggested Readings
• Cuypers JAAE, Witsenburg M, van der Linde D, et al. Pulmonary stenosis: update on
diagnosis and therapeutic options. Heart. 2013;99(5):339-347. doi:10.1136/heartjnl-2012-
301964 .
• Mack G, Silberbach M. Aortic and pulmonic stenosis. Pediatr Rev. 2000;21(3):79-85.
doi:10.1542/pir.21-3-79.
• McCulloch MA, Gajarski RJ. Congenital and acquired heart disease. In: McInerny TK,
Adam HM, Campbell DE, DeWitt TG, Foy JM, Kamat DM, eds. American Academy of
Pediatrics Textbook of Pediatric Care. American Academy of Pediatrics; 2021:chap 234.
Accessed September 1, 2021. Pediatric Care Online.
• Menashe V. Heart murmurs. Pediatr Rev. 2007;28(4)e19-e22. doi:10.1542/pir.28-4-e19.

American Academy of Pediatrics 686


PREP® Self-Assessment PREPSA 2023

Question 204
A 9-month-old infant is evaluated in the clinic for an itchy rash of 3 days’ duration. The infant
had a similar rash at ages 2 and 6 months. Each time, the rash resolved after a few days with the
use of moisturizers. He is otherwise healthy. His medical history is significant only for
physiologic jaundice on day 5 after birth requiring phototherapy and a brief viral illness at 4
months of age. The family history is notable for allergic rhinitis in the father. The infant was
exclusively breastfed until 4 months of age and currently is taking breast milk, pureed foods, and
infant cereals. On physical examination, an erythematous maculopapular rash with scaling and
excoriations is seen on his face (Item Q204) and the extensor aspects of the arms, legs, and
wrists.

Of the following, the MOST likely predisposing factor for this infant’s condition is
A. allergic rhinitis in the father
B. breastfeeding
C. jaundice requiring phototherapy
D. prior viral illness

American Academy of Pediatrics 687


PREP® Self-Assessment PREPSA 2023

Correct Answers: A
The infant described in the vignette has a history and physical examination findings consistent
with infantile atopic dermatitis. Atopic dermatitis (AD) is a chronic skin condition with
inflammation presenting as erythema and pruritus that is characterized by exacerbations and
remissions. The prevalence of atopic dermatitis is increasing worldwide, especially in better
resourced countries; most affected individuals develop the condition during infancy. Atopic
dermatitis is grouped under the atopic spectrum of diseases which includes food allergies,
allergic rhinitis, and asthma.

Several factors predispose an individual to developing AD. Genetic predisposition plays an


important role in the development of AD. Children with 1 family member having a condition in
the atopic spectrum are 2 to 3 times more likely to develop AD and 3 to 5 times more likely
when 2 or more family members are affected. More than 40 genetic mutations are known to be
associated with an increased incidence of atopic conditions. The filaggrin gene (FLG) is
responsible for maintaining normal skin function and protection against environmental trauma.
Mutations in FLG impair the skin barrier, permitting allergen invasion and sensitization leading
to atopic disease. Environmental factors, such as urban dwelling (increased exposure to
pollution/smoke), climate, and humidity increase the risk of developing atopic diseases. The
increased incidence of AD in better resourced countries led to the “hygiene hypothesis,” which
describes an advantage of early exposure to certain infections and microbes involving the
immune system, which decreases the child’s inflammatory state.

The impact of breastfeeding on the risk of atopic diseases is controversial. Although past
evidence demonstrated a decreased risk of atopic disease in individuals who were breastfed, a
recent systematic review suggests that there is no association between breastfeeding and atopy.
Viral infections may be associated with wheezing, but are not the most likely factor to increase
the risk of atopic dermatitis in this infant. Jaundice and phototherapy do not lead to
increased risk of atopic diseases.

PREP Pearls
• Atopic dermatitis is a chronic inflammatory skin condition characterized by
exacerbations and remissions.
• The prevalence of atopic diseases is increasing, especially in better-resourced countries.
• Genetic predisposition plays a strong role in the development of atopic dermatitis and
other allergic conditions.

ABP Content Specifications(s)


• Recognize the non-environmental factors (eg, genetics, diet, infection) that influence the
incidence and severity of atopy in infants and children

American Academy of Pediatrics 688


PREP® Self-Assessment PREPSA 2023

Suggested Readings
• Garg N, Silverberg JI. Epidemiology of childhood atopic dermatitis. Clin Dermatol.
2015;33(3):281-288. doi:10.1016/j.clindermatol.2014.12.004.
• Lin B, Dai R, Lu L, Fan X, Yu Y. Breastfeeding and atopic dermatitis risk: a systematic
review and meta-analysis of prospective cohort studies. Dermatology. 2020;236(4):345-
360. doi:10.1159/000503781.
• Portelli MA, Hodge E, Sayers I. Genetic risk factors for the development of allergic
disease identified by genome-wide association. Clin Exp Allergy. 2015;45(1):21-31.
doi:10.1111/cea.12327.
• Waldman AR, Ahluwalia J, Udkoff J, Borok JF, Eichenfield LF. Atopic dermatitis.
Pediatr Rev. 2018;39(4)180-193. doi:10.1542/pir.2016-0169.

American Academy of Pediatrics 689


PREP® Self-Assessment PREPSA 2023

Question 205
A 17-year-old adolescent girl is seen in the emergency department. She has a 1-month history of
worsening severe early morning headaches, tinnitus, and intermittent blurring of her vision. She
was sent for evaluation by her ophthalmologist who noted bilateral papilledema and visual acuity
of 20/30 in both eyes on examination that morning. She is otherwise healthy and takes no
medications.

In the emergency department, the adolescent’s vital signs are normal. Her weight is 110 kg and
body mass index is 41.4 kg/m2. With the exception of bilateral papilledema, her physical
examination findings are normal. Brain magnetic resonance imaging with and without contrast
and magnetic resonance venography findings are normal. A lumbar puncture is performed and
reveals an opening pressure of 36 cm H2O and normal cerebrospinal fluid laboratory results.

Of the following, the BEST next step in this adolescent’s acute management is
A. administration of acetazolamide
B. counseling on healthy weight loss
C. neurosurgery consultation
D. referral back to ophthalmology

American Academy of Pediatrics 690


PREP® Self-Assessment PREPSA 2023

Correct Answer: A
The adolescent in the vignette has idiopathic intracranial hypertension (IIH; previously known as
pseudotumor cerebri); administration of acetazolamide is the best next step in her initial acute
management. Idiopathic intracranial hypertension is a syndrome of unknown etiology; it is a
diagnosis of exclusion. The pathophysiology is poorly understood. A proposed mechanism is
blockage of cerebrospinal fluid (CSF) absorption at the arachnoid villi secondary to cerebral
venous hypertension from transverse venous sinus stenosis.

Children and adolescents with IIH often have signs and symptoms of increased intracranial
pressure, including:
• Symptoms:
o Headache
o Vision disturbances (ie, transient visual obscurations)
o Pulsatile tinnitus
o Diplopia

• Signs:
o Papilledema (Item C205A)
o Cranial nerve VI palsies
o Visual field defects

Some asymptomatic children are identified incidentally when papilledema is noted on routine
ophthalmological examination. Demographic studies suggest 3 distinct pediatric IIH populations:
1) Prepubertal children with a normal body mass index (BMI)
2) Pubertal/early adolescent children (often with overweight or obesity)
3) Late adolescent (often with obesity) (more common in females than males)

Diagnostic evaluation of IIH involves exclusion of alternate causes of increased intracranial


pressure (Item C205B) and documentation of an elevated opening pressure on lumbar puncture.
Once papilledema is identified, magnetic resonance imaging (MRI) of the brain with contrast and
magnetic resonance venography (MRV) should be obtained to exclude alternate causes such as
hemorrhage, space-occupying lesion, hydrocephalus, Chiari I malformation, or cerebral venous
sinus thrombosis. Subtle findings on MRI, such as an empty sella turcica or dilation or tortuosity
of the optic nerve sheaths can support a diagnosis of IIH but are not required or specific. Lumbar
puncture should be performed with opening pressure measurement to confirm the presence of
American Academy of Pediatrics 691
PREP® Self-Assessment PREPSA 2023

increased intracranial pressure (ICP) and for collection of CSF for analysis (cell count, glucose,
protein, electrolyte levels) to exclude alternate diagnoses. A CSF opening pressure >28 cm H2O
(lumbar puncture with sedation) or >25 cm H2O (lumbar puncture with minimal or no sedation)
is considered abnormal.

Causes of Increased Intracranial Pressure in Children


HEAD TRAUMA
• Cerebral edema
• Intracerebral hemorrhage
• Extracerebral hemorrhage (subdural, epidural)
VASCULAR CAUSES
• Arterial or venous infarctions
• Intracerebral hemorrhage
• Dural sinus thrombosis
• Subarachnoid hemorrhage
• Vascular anomalies (vein of Galen malformation,
arteriovenous malformations)
NEOPLASTIC CAUSES
• Primary brain tumors
• Metastatic (intracerebral, meningeal infiltration)
• Hydrocephalus (congenital or acquired,
communicating or noncommunicating)
• Pseudotumor cerebri (benign intracranial
hypertension)
• Central nervous system infections
• Meningitis (bacterial, fungal, mycobacterial)
• Encephalitis (focal or diffuse)
• Abscess
METABOLIC CAUSES
• Inborn errors of metabolism (hyperammonemia)
• Hepatic encephalopathy
• Diabetic ketoacidosis
• Renal failure
• Reye syndrome
• Status epilepticus
• Hypoxic-ischemic encephalopathy
• Fluid-electrolyte abnormalities (hyponatremia,
hypernatremia)
STRUCTURAL CAUSES
• Craniosynostosis

The management of IIH has 2 goals: 1) preservation of vision and 2) acute symptom relief. First-
line medical management is the initiation of a carbonic anhydrase inhibitor (eg, acetazolamide).
Acetazolamide is believed to decrease CSF production and act as a mild diuretic, providing
symptom relief and reduction in intracranial pressure. Acetazolamide is generally well tolerated
with common side effects of weight loss, paresthesias, dysgeusia, nausea, vomiting, and diarrhea.
Topiramate, a mild carbonic anhydrase inhibitor used in migraine prophylaxis, has similar
American Academy of Pediatrics 692
PREP® Self-Assessment PREPSA 2023

efficacy to acetazolamide in the treatment of mild to moderate IIH and is sometimes better
tolerated. Common side effects of topiramate include decreased appetite, weight loss, mental
fogginess, and paresthesias. Renal stones and acute angle-closure glaucoma are rare but
significant potential adverse effects. Other diuretics (eg, furosemide) are sometimes used in
combination with a carbonic anhydrase inhibitor for synergistic effect for refractory IIH patients.

Data are limited regarding the effect on IIH of weight loss in children with obesity. Pediatric
recommendations are extrapolated from treatment studies of adults with IIH in which healthy
weight loss demonstrated benefit and potential remission. For many individuals in these studies,
loss of 6% to 10% of their initial body weight led to remission in the long term, in particular in
those with an elevated BMI. However, weight loss is not practical or appropriate for acute short-
term management of IIH due to the time to effect and should not be the sole intervention at time
of diagnosis.

Surgical interventions are considered for children with severe, rapidly progressing IIH in which
vision is acutely threatened or in the setting of medical therapy failure. The 3 most common
surgical interventions are: 1) CSF diversion with shunting, 2) optic nerve fenestration, and 3)
transverse venous sinus stenting. In the case of an acute threat to vision, optic nerve fenestration
has been shown to be effective for vision preservation. In the vignette, the adolescent’s vision is
not acutely threatened, and she has not undergone medical management; thus, ophthalmologic or
neurosurgical consultation for surgical intervention is not yet indicated.

Treatment efficacy is monitored in the long term by improvement or resolution of symptoms and
serial ophthalmological examinations that evaluate the fundi, visual acuity, and visual fields.
Timing of follow-up is tailored to the individual and management is adjusted based on the
clinical course and responsiveness to treatment.

PREP Pearls
• Idiopathic intracranial hypertension is a syndrome of unknown etiology; it is a diagnosis
of exclusion. Signs and symptoms of increased intracranial pressure include headache,
visual obscurations, pulsatile tinnitus, diplopia, papilledema, cranial nerve VI palsies, and
visual field defects.
• The management of idiopathic intracranial hypertension has 2 goals: 1) preservation of
vision and 2) acute symptom relief. A carbonic anhydrase inhibitor (eg, acetazolamide) is
the first-line medical therapy in conjunction with weight loss (for children with obesity);
surgical intervention is reserved for fulminant presentations or treatment failure.
• Children and adolescents with idiopathic intracranial hypertension require long-term
ophthalmological follow-up with serial ophthalmological examinations that evaluate the
fundi, visual acuity, and visual fields.

ABP Content Specifications(s)


• Identify the risk factors for pseudotumor cerebri

American Academy of Pediatrics 693


PREP® Self-Assessment PREPSA 2023

Suggested Readings
• Aylward SC, Reem RE. Pediatric intracranial hypertension. Pediatr Neurol. 2017;66:32-
43. doi:10.1016/j.pediatrneurol.2016.08.010.
• Friedman D, Liu G, Digre K. Revised diagnostic criteria for the pseudotumor cerebri
syndrome in adults and children. Neurology. 2013;81(13):1159-1165.
doi:10.1212/WNL.0b013e3182a55f17.
• Shah MM, Smyth MD. Increased intracranial pressure. In: McInerny TK, Adam HM,
Campbell DE, DeWitt TG, Foy JM, Kamat DM, eds. American Academy of Pediatrics
Textbook of Pediatric Care. American Academy of Pediatrics; 2021:chap 364. Accessed
September 1, 2022. Pediatric Care Online.

American Academy of Pediatrics 694


PREP® Self-Assessment PREPSA 2023

Question 206
A 13-year-old adolescent girl is seen in the office for evaluation of right knee pain and swelling.
The day prior, she twisted her knee while rising from a chair. She felt something shift in her knee
and had a sudden onset of pain. Within an hour, she developed knee swelling. On physical
examination, she is unable to fully bend or straighten her right knee, and there is a large joint
effusion. There is tenderness around the patella, and she is very apprehensive with attempts to
move her patella. There is no ligamentous laxity with the Lachman test or with valgus or varus
stress. The remainder of her physical examination findings are normal.

Of the following, this girl’s MOST likely diagnosis is


A. anterior cruciate ligament tear
B. medial meniscus tear
C. patella dislocation
D. patella tendon rupture

American Academy of Pediatrics 695


PREP® Self-Assessment PREPSA 2023

Correct Answer: C
The girl in the vignette felt something shift in her knee and experienced a rapid onset of pain and
swelling after a seemingly mild twisting injury. She has a large effusion and apprehension with
patellar motion but no anterior cruciate ligamentous laxity. The most likely diagnosis is a patellar
dislocation.

Patellar dislocation is a common acute knee injury in adolescents. The term dislocation is used
when the patella leaves the trochlear groove completely. Subluxation describes a partial shift of
the patella. Internal rotation of the femur with the foot planted is a common mechanism for
patellar dislocation. The patella typically dislocates laterally, then reduces back into place
spontaneously. When the dislocation occurs, bone fragments may be sheared off the front of the
femoral trochlea or the back of the patella. Typically, affected individuals report a feeling of
something popping or shifting in the front of the knee. Swelling often develops within hours of
injury.

Children with patellar dislocation often have noticeable effusion, tenderness around the patella,
and apprehension with manipulation of the patella. Clinicians should obtain radiographs of the
knee, including a sunrise view, to assess the shape of the trochlea and look for associated loose
bodies. Magnetic resonance imaging (MRI) may be needed for children with mechanical
symptoms, such as catching or locking, or persistent swelling and pain 4 weeks after the injury.
Initial dislocations can be treated conservatively with rest, a patella-stabilizing brace (Item
C206A), and physical therapy. Referral to an orthopedic surgeon is indicated for those with
osteochondral fractures or repeat dislocations.

The femoral trochlea acts like a pocket for the patella. Individuals with a shallow trochlea and a
patella that tilts to the side (Item C206B) are at higher risk for patellar dislocation than those
with a deeper trochlea and a patella seated centrally (Item C206C). About one-third of
individuals with a patellar dislocation will have a repeat dislocation.

Anterior cruciate ligament (ACL) tears are also common acute knee injuries in adolescents. The
clinical presentation for an ACL tear can be very similar to that seen with patellar dislocation.
Athletes often report a twisting mechanism. A valgus stress applied to the knee can also cause
both patellar dislocation and an ACL tear. A “pop” at the time of injury and rapid onset of
swelling frequently accompany ACL injuries. The girl in the vignette does not have ligamentous
laxity on physical examination, making ACL injury less likely. Both ACL injuries and repeated
patellar dislocations can increase the risk of early osteoarthritis. Knee injury prevention warm-up
programs have demonstrated efficacy in reducing knee injuries in young athletes.

A medial meniscal tear can occur with a twisting knee injury. However, the mechanism of injury
in young athletes is typically more forceful than that described in the vignette. Adolescents with
meniscus tears tend to have tenderness over the joint line rather than around the patella.

Tendon rupture is a very rare injury in children and adolescents. On physical examination, an
individual with a patella tendon rupture will have an elevated patella and be unable to actively
American Academy of Pediatrics 696
PREP® Self-Assessment PREPSA 2023

extend the knee. The girl in the vignette does not have an elevated patella, making this diagnosis
unlikely.

PREP Pearls
• Patellar dislocations and anterior cruciate ligament tears can both present in adolescents
with rapid onset of pain and swelling following a twisting knee injury.
• Knee injury prevention warm-up programs have been shown to reduce rates of knee
injury in young athletes.
• Patella tendon rupture is very uncommon in the pediatric population.

ABP Content Specifications(s)


• Recognize the historical and clinical findings associated with subluxation of the patella

Suggested Readings
• Fithian DC, Paxton EW, Stone ML, et al. Epidemiology and natural history of acute
patellar dislocation. Am J Sports Med. 2004;32(5):1114-1121.
doi:10.1177/0363546503260788.
• Internal Derangement of the Knee (Knee Injury). In: Sarwark JF, LaBella CR. Pediatric
Orthopaedics and Sports Injuries: A Quick Reference Guide. 3rd ed. American Academy
of Pediatrics; 2021:355-373.
• Jaquith BP, Parikh SN. Predictors of recurrent patellar instability in children and
adolescents after first-time dislocation. J Pediatr Orthop. 2017;37(7):484-490.
doi:10.1097/BPO.0000000000000674.
• Kannikeswaran N, Suresh S. Sports musculoskeletal injuries. In: McInerny TK, Adam
HM, Campbell DE, DeWitt TG, Foy JM, Kamat DM, eds. American Academy of
Pediatrics Textbook of Pediatric Care. American Academy of Pediatrics; 2021:chap 334.
Accessed September 1, 2022. Pediatric Care Online.
• LaBella CR, Hennrikus W, Hewett TE. Anterior cruciate ligament injuries: diagnosis,
treatment, and prevention. Pediatrics. 2014;133(5):e1437-e1450. doi:10.1542/peds.2014-
0623.

American Academy of Pediatrics 697


PREP® Self-Assessment PREPSA 2023

Question 207
A right orchiectomy and left orchidopexy were performed on a 1-day-old boy for in utero right
testicular torsion. Postoperatively, he was returned to the neonatal intensive care unit with
normal vital signs in room air. Six hours after surgery, he was minimally responsive and
hypotonic, with pinpoint pupils. A narcotic overdose was suspected. A review of the medication
list revealed that while in the postanesthesia care unit the neonate received 3.6 mg of intravenous
morphine for pain instead of 0.36 mg. The resident gave a verbal order, documented in the
medical record, for 0.1 mg/kg of morphine. The neonate’s weight is 3.6 kg. Appropriate doses of
naloxone were administered twice during a 4-hour period. Five hours after treatment with
naloxone, he was awake, breastfeeding, and active. The medical error was disclosed to the
parents by the attending physician.

Of the following, the health care team’s BEST next step in response to this event is to
A. implement a policy of no verbal orders
B. implement an error prevention campaign
C. notify the risk management team
D. provide support and counseling

American Academy of Pediatrics 698


PREP® Self-Assessment PREPSA 2023

Correct Answer: D
In the United States, 1% to 3% of pediatric hospital admissions are complicated by medical
errors. Medication error is the most common adverse event in the pediatric population, with an
error present in more than 5% of pediatric medication orders, in part due to weight-based dosing.
It is imperative for health care practitioners to be aware of the potential for medical error and to
be prepared, when an error occurs, to disclose the error to the patient and family. Training in
error disclosure methods is helpful in developing the skills to best deliver this difficult news.
Practitioners have an obligation to disclose an adverse event caused by a medical error. Failure to
disclose can erode trust in the patient–practitioner relationship. Disclosure also provides the
opportunity to identify and address problems to prevent error recurrence, thereby leading to
improved patient safety. Since the publication of the National Academy of Medicine (formerly
Institute of Medicine) 1999 report, “To Err is Human: Building a Safer Health System,” there is
not only a greater national awareness of patient safety risks but also a shift toward transparency
and systems error prevention.

The emotional and psychological impact of a medical error reaches not only the patient and
family but also the health care practitioners involved. A health care practitioner may become the
“second victim,” experiencing emotional distress after a patient adverse event. Therefore, it is
imperative to provide support and counseling to the patient, family, and health care practitioner;
institutions should ensure that there are mechanisms in place for this. Approximately 50% of
health care practitioners will experience a “second victim” phenomenon in their career. Although
disclosure is often stressful for the practitioner, in the long term it may help alleviate guilt and
anxiety surrounding the error and decrease the risk of “second victim”–related trauma.

Although a no verbal orders policy might appear desirable, in certain situations a strict policy can
delay urgently needed care. Therefore, most hospitals have policies that limit the use of verbal
orders but permit them in specific circumstances. Notifying risk management of an error may be
advisable, but providing support should be the immediate priority. An error prevention campaign
may be recommended as an outcome of a root cause analysis but will happen later in the process.

PREP Pearls
• Medication errors are the most common adverse event in pediatrics.
• Disclosure of an adverse event to a patient and family is imperative and should occur at
the earliest opportunity.
• Support and counseling should be provided to the patient, family, and health care
practitioner after an adverse event. The practitioner is often the “second victim,”
experiencing emotional distress following a patient adverse event.

ABP Content Specifications(s)


• Use appropriate means to disclose medical errors to patients
• Apply appropriate methods of support for patients and their families after an error
producing medical harm occurs
• Use appropriate methods of support for physicians and other health-care providers after
an error producing medical harm occurs
American Academy of Pediatrics 699
PREP® Self-Assessment PREPSA 2023

Suggested Readings
• Busch IM, Moretti F, Campagna I, et al. Promoting the psychological well-being of
healthcare providers facing the burden of adverse events: a systematic review of second
victim support resources. Int J Environ Res Public Health. 2021;18(10):5080.
doi:10.3390/ijerph18105080.
• Edrees H, Connors C, Paine L, Norvell M, Taylor H, Wu AW. Implementing the RISE
second victim support programme at the Johns Hopkins Hospital: a case study. BMJ
Open. 2016;6(9):e011708. doi:10.1136/bmjopen-2016-011708.
• McDonnell WM, Altman RL, Bondi SA, et al; Committee on Medical Liability and Risk
Management, Council on Quality Improvement and Patient Safety. Disclosure of adverse
events in pediatrics. Pediatrics. 2016;138 (6):e20163215. doi:10.1542/peds.2016-3215.
• Mueller BU, Neuspiel DR, Fisher ERS; Council on Quality Improvement and Patient
Safety, Committee on Hospital Care. Principles of pediatric patient safety: reducing harm
due to medical care. Pediatrics. 2019;143(2):e20183649.
• doi:10.1542/peds.2018-3649.
• Shaikh SK, Cohen SP. Disclosure of medical errors. Pediatr Rev. 2020;41(1):45–47.
doi:10.1542/pir.2018-0228.

American Academy of Pediatrics 700


PREP® Self-Assessment PREPSA 2023

Question 208
A 16-year-old adolescent boy is seen for follow-up after hospital admission for acute onset of
abdominal pain, nausea, and hematemesis. His evaluation included an upper endoscopy which
revealed acute hemorrhagic erosive gastritis. The adolescent was started on acid-suppressing
medication and discharged from the hospital after symptom resolution. He has a history of
asthma, for which he uses fluticasone and montelukast daily, and a history of headaches, for
which frequent use of acetaminophen is reported. In addition, he has a history of alcohol and
marijuana use; on the day the symptoms began, he had consumed multiple alcoholic beverages
and used marijuana heavily.

Of the following, the substance MOST likely to have caused the adolescent’s endoscopy finding
is
A. acetaminophen
B. alcohol
C. marijuana
D. montelukast

American Academy of Pediatrics 701


PREP® Self-Assessment PREPSA 2023

Correct Answer: B
The adolescent in the vignette developed acute onset of hematemesis, and endoscopy
demonstrated acute hemorrhagic erosive gastritis. Of the substances listed, alcohol is most likely
to cause these findings. Use of acetaminophen, montelukast, and/or marijuana are not associated
with this endoscopic finding.

Alcohol can induce esophageal and gastric injury with both acute and chronic use. In the
esophagus, alcohol can decrease both lower esophageal sphincter tone and esophageal motility.
These changes result in more frequent and longer episodes of gastroesophageal reflux.
Presenting symptoms can include regurgitation, heartburn, epigastric pain, and/or vomiting.
Alcohol decreases gastric motility, which allows for more direct contact of alcohol on the gastric
epithelium and stimulates gastric acid secretion (particularly fermented alcoholic beverages
including beer and wine). At high doses, alcohol causes direct gastric epithelial injury through
release of inflammatory mediators, resulting in cell necrosis and mucosal injury. With ingestion
of large amounts of alcohol, severe acute hemorrhagic erosive gastritis can occur, presenting as
nausea, abdominal pain, and hematemesis. It is imperative to ask about alcohol use when
evaluating adolescents exhibiting these symptoms.

Treatment for alcohol-related esophagitis and/or gastritis includes avoidance of alcohol, the
administration of acidsuppressing medication, and supportive care. For severe gastrointestinal
bleeding, therapeutic endoscopy (with use of modalities such as argon plasma coagulation) can
be used to achieve hemostasis.

PREP Pearls
• Symptoms of alcohol-induced gastritis can include nausea, vomiting, abdominal pain,
and hematemesis.
• Alcohol use should be considered in children or adolescents exhibiting nausea, vomiting,
abdominal pain, and/or hematemesis.
• Treatment of alcohol-induced gastritis includes acid suppression and avoidance of
alcohol.

ABP Content Specifications(s)


• Recognize the clinical features associated with alcohol-induced gastritis, and manage
appropriately

Suggested Readings
• Franke A, Teyssen S, Singer MV. Alcohol-related diseases of the esophagus and
stomach. Dig Dis. 2005;23:204-213. doi:10.1159/000090167.
• Ryan SA, Kokotailo P; Committee on Substance Use and Prevention. Alcohol use by
youth. Pediatrics.
• 2019;144(1):e20191357. doi:10.1542/peds.2019-1357.

American Academy of Pediatrics 702


PREP® Self-Assessment PREPSA 2023

Question 209
An 11-month-old boy was admitted to the hospital with fever of 6 days’ duration, conjunctival
injection, and rash. Echocardiography showed a small pericardial effusion, normal coronary
vessels, and normal ventricular function. He was diagnosed with Kawasaki disease and treated
with intravenous immunoglobulin and high-dose aspirin. He is currently afebrile, doing well, and
ready for discharge to home on low-dose aspirin.

Of the following, the BEST next step in this boy’s management is to


A. administer inactivated influenza vaccine prior to discharge
B. administer the measles-mumps-rubella and varicella vaccines at age 12 months
C. advise his parents to monitor the number of urine-filled diapers each day
D. restrict his formula intake to no more than 16 oz/day

American Academy of Pediatrics 703


PREP® Self-Assessment PREPSA 2023

Correct Answer: A
The boy in the vignette is being treated with low-dose aspirin for Kawasaki disease. Of the
response choices, the best next management step is to administer an inactivated influenza
vaccine prior to hospital discharge. Children taking aspirin are at risk for developing Reye
syndrome if they contract influenza or varicella infection. Although the risk is minimal, there
have been case reports of Reye syndrome associated with Kawasaki disease and aspirin use. The
American Heart Association guidelines recommend that infants and children (aged 6 months and
older) who are prescribed aspirin receive an inactivated influenza vaccine prior to hospital
discharge, if not already vaccinated for the influenza season. Children on long-term aspirin
therapy should receive the influenza vaccine annually. Parents of children taking aspirin should
be counseled regarding the symptoms of Reye syndrome and advised to seek immediate medical
attention if these symptoms develop.

Reye syndrome is an acute inflammatory encephalopathy characterized by impaired


consciousness, elevated liver transaminases, elevated ammonia levels, and fatty infiltration of the
liver. To meet criteria for Reye syndrome, the findings cannot be explained by another condition.
There is a strong association with preceding viral infections, especially influenza and varicella,
and the use of salicylates. It has been postulated that Reye syndrome occurs in children who have
an underlying genetic predisposition. The incidence of Reye syndrome has significantly
decreased since 1986 when labeling of all aspirin products with the Reye syndrome warning
resulted in cessation of aspirin use as an antipyretic in children. However, it is important to be
aware of this condition, due to the high morbidity and mortality risk.

Currently, aspirin use in pediatrics is almost exclusively limited to Kawasaki disease and
multisystem inflammatory syndrome in children (MIS-C). Upon diagnosis, children with
Kawasaki disease are prescribed high-dose aspirin for the anti-inflammatory effect, followed by
4 to 6 weeks of low-dose aspirin for the antiplatelet effect. If, at 4- to 6week follow-up
echocardiography shows no coronary artery changes, aspirin is discontinued. Children with
coronary artery changes are continued on longer courses of aspirin based on risk stratification.
Live virus vaccines, including measles, mumps, rubella, and varicella, should be deferred for 11
months following
intravenous immunoglobulin therapy for Kawasaki disease. This interval is recommended for an
optimal immune response to live vaccines after clearance of passive antibodies.
The live-virus varicella vaccine also confers a risk of Reye syndrome for children treated with
long-term aspirin therapy. When these children are due for a varicella vaccine, the risks and
benefits of the vaccine versus contracting varicella infection should be assessed; physicians may
consider switching to another antiplatelet agent for 6 weeks following varicella vaccination.
There is no indication for fluid restriction or monitoring of urine output for children with
Kawasaki disease with normal cardiac function.

American Academy of Pediatrics 704


PREP® Self-Assessment PREPSA 2023

PREP Pearls
• Children taking aspirin are at risk of developing Reye syndrome if they contract influenza
or varicella infection.
• Reye syndrome is characterized by altered mental status associated with significant
elevation of liver transaminases and ammonia level.
• Children with Kawasaki disease who are prescribed aspirin therapy should receive an
inactivated influenza vaccine prior to hospital discharge.

ABP Content Specifications(s)


• Recognize the risks associated with the use of aspirin

Suggested Readings
• American Academy of Pediatrics. Active immunization after receipt of immune globulin
or other blood products. In: Kimberlin DW, Barnett ED, Lynfield R, Sawyer MH, eds.
Red Book: 2021–2024 Report of the Committee on Infectious Diseases. 32nd ed.
American Academy of Pediatrics; 2021. Accessed September 1, 2022. Red Book Online.
• American Academy of Pediatrics. Kawasaki disease. In: Kimberlin DW, Barnett ED,
Lynfield R, Sawyer MH, eds. Red Book: 2021–2024 Report of the Committee on
Infectious Diseases. 32nd ed. American Academy of Pediatrics; 2021. Accessed
September 1, 2022. Red Book Online.
• Ferretti S, Gatto A, Curatola A, Pansini V, Graglia B, Chiaretti A. Atypical Reye
syndrome: three cases of a problem that pediatricians should consider and remember.
Acta Biomed. 2021;92(S1):e2021110. doi:10.23750/abm.v92iS1.10205.
• McCrindle BW, Rowley AH, Newburger JW, et al. Diagnosis, treatment, and long-term
management of Kawasaki disease: a scientific statement for health professionals from the
American Heart Association. Circulation. 2017;135(17):e927-e999.
doi:10.1161/CIR.0000000000000484.
• Wei CM, Chen HL, Lee PI, Chen CM, Ma CY, Hwu WL. Reye's syndrome developing in
an infant on treatment of Kawasaki syndrome. J Paediatr Child Health. 2005;41(5-6):303-
304. doi:10.1111/j.1440-1754.2005.00617.x.

American Academy of Pediatrics 705


PREP® Self-Assessment PREPSA 2023

Question 210
A 4-week-old male infant is evaluated for rapid breathing which has been present since birth. He
had a respiratory rate of 50 breaths/min at his 2-week visit. He is otherwise healthy, nursing well,
and growing appropriately. He does not have cyanosis or color change with feeding or sleeping.
There is no family history of metabolic disease, and his newborn screening results are normal.
On physical examination, the infant is alert and comfortable. His temperature is 36.5°C, heart
rate is 120 beats/min, respiratory rate is 55 breaths/min (counted over a full minute), and oxygen
saturation by pulse oximetry is 99% in room air. His breath sounds are clear with good air
exchange. There is no retraction, nasal flaring, or use of accessory muscles of respiration. The
remainder of his physical examination findings are normal.

Of the following, the BEST next step is to


A. begin inhaled albuterol treatments
B. monitor with home pulse oximetry for 1 month
C. order echocardiography and chest computed tomography
D. reassure the infant’s mother and reevaluate in 1 month

American Academy of Pediatrics 706


PREP® Self-Assessment PREPSA 2023

Correct Answer: D
The infant in the vignette has a respiratory rate that is at the upper end of the normal range for his
age; there are no findings to suggest underlying pathology. The best next step is to reassure the
mother that he is not in danger, and schedule follow-up for reevaluation. This scenario is not
uncommon and often precipitates an unnecessary referral to a specialist.

There is nothing in the infant’s history or physical examination findings to suggest a cardiac
abnormality, and his oxygen saturation is normal. With no history of cyanosis or color change,
further evaluation of the heart or lung structure with echocardiography or computed tomography
is not indicated. There is no wheezing or other indicator of airway reactivity, so treatment with
albuterol is not appropriate. With no history of cyanosis, a normal physical examination, and
normal pulse oximetry, home measurement of oxygen saturation is not indicated.

The range of normal respiratory rates in children varies from infancy to adolescence; the range of
normal is broadest for infants (Item C210). Respiratory rates at or slightly above 60 breaths/min
can be normal in infants 3 to 4 months of age (90th percentile). At 1 year of age, the 90th
percentile rate is above 50 breaths/min and by 4 years of age the 90th percentile is close to 30
breaths/min.

Multiple factors may affect respiratory rate. Exercise, fever, and anxiety can increase respiratory
rate. Dysregulation of the central nervous system can increase or decrease respiratory rate. The
complex response to metabolic acidosis includes an increase in respiratory rate. Oxygen
desaturation is usually accompanied by a compensatory increase in respiratory rate.

American Academy of Pediatrics 707


PREP® Self-Assessment PREPSA 2023

PREP Pearls
• There is a broad range of normal respiratory rates with the widest variation in the first 2
years after birth.
• Multiple factors affect respiratory rate including body temperature, anxiety, metabolic
dysfunction, and central neurologic dysfunction.

ABP Content Specifications(s)


• Recognize normal breathing patterns in patients of various ages
• Recognize the various factors that influence respiratory rate

Suggested Readings
• Bloomfield D. Tachypnea. Pediatr Rev. 2002;23(8):294-295; discussion 294-295.
doi:10.1542/pir.23-8-294.
• Davies P, Maconochie I. The relationship between body temperature, heart rate and
respiratory rate in children. Emerg Med J. 2009;26(9):641-643.
doi:10.1136/emj.2008.061598.
• Fleming S, Thompson M, Stevens R, et al. Normal ranges of heart rate and respiratory
rate in children from birth to 18 years of age: a systematic review of observational
studies. Lancet. 2011;377(9770):1011-1018. doi:10.1016/S01406736(10)62226-X.
• Homnick D. Pediatric physical examination: interpretation of findings. In: McInerny TK,
Adam HM, Campbell DE, DeWitt TG, Foy JM, Kamat DM, eds. American Academy of
Pediatrics Textbook of Pediatric Care. American Academy of Pediatrics; 2021:chap 16.
Accessed September 1, 2021. Pediatric Care Online.

American Academy of Pediatrics 708


PREP® Self-Assessment PREPSA 2023

Question 211
A 2-month-old boy is brought to the office for a health supervision visit. He was born to a 30-
year-old mother by an uncomplicated vaginal delivery. Prenatal ultrasonography revealed
multiple noncommunicating cysts on the left kidney; the right kidney appears normal. The family
history is negative for cystic kidney disease. He is exclusively breastfed and has 6 to 8 wet
diapers every day. The boy’s weight is at 50th percentile and height is at the 50th to 75th
percentile. His vital signs are normal for age, and the remainder of his physical examination
findings are normal.

Renal ultrasonography after birth revealed multiple noncommunicating cysts of various sizes
occupying the left kidney without normal intervening renal parenchyma. The right kidney
appears normal.

Of the following, the BEST next step in this infant’s management is to


A. order a diuretic renal scan
B. perform serial renal ultrasonography
C. refer for surgical nephrectomy
D. start trimethoprim-sulfamethoxazole prophylaxis

American Academy of Pediatrics 709


PREP® Self-Assessment PREPSA 2023

Correct Answer: B
The infant in the vignette has a left multicystic dysplastic kidney (MCDK) and a normal-
functioning right kidney. The best next step in this infant’s management is serial renal
ultrasonography to monitor for involution of the MCDK and compensatory hypertrophy of the
normal-functioning kidney.

Multicystic dysplastic kidney is the most common cause of congenital solitary kidney. The
prevalence of MCDK is 1 in 4,300 live births. It is more common in males, has a predisposition
for the left side, and is nonhereditary. In most cases, multicystic dysplastic kidney is diagnosed
on prenatal ultrasonography. Multicystic dysplastic kidney may present in the newborn as an
abdominal mass. Some cases are diagnosed incidentally by imaging performed for unrelated
conditions or during evaluation for renal involvement in the context of a genetic syndrome.
Postnatal ultrasonography characteristically shows multiple noncommunicating cysts of different
sizes with dysplastic renal parenchyma and an atretic ureter.

Most MCDKs involute by a median age of 5.5 years without any intervention. The contralateral
kidney is usually normal in size and structure and gradually develops compensatory hypertrophy.
Renal function is normal with a solitary functioning kidney. Annual screening for hypertension
and proteinuria should be performed. Serial renal ultrasonography (at birth and 6 months, 2
years, 5 years, 10 years, and 15 years of age) is indicated to monitor for involution of the MCDK
and compensatory hypertrophy or any abnormalities of the contralateral kidney.

Evaluation of the contralateral kidney may demonstrate vesicoureteral reflux (VUR) in 15% to
28% and ureteropelvic junction (UPJ) obstruction in 4% of children with a MCDK. A voiding
cystourethrogram (to evaluate for VUR) and diuretic renal scan (to evaluate for UPJ obstruction)
are indicated in children with urinary tract infection or hydronephrosis of the contralateral
kidney. Because the child in the vignette has a normal right kidney, neither a
diuretic renal scan nor trimethoprim-sulfamethoxazole prophylaxis to prevent urinary tract
infection is indicated.

Surgical nephrectomy is not routinely performed for MCDK because the risk of malignant
transformation is low. The procedure is only indicated if there is noninvolution or an increase in
the size of the MCDK.

PREP Pearls
• Multicystic dysplastic kidney may be diagnosed on prenatal ultrasonography or
incidentally by imaging performed for unrelated conditions or may present as an
abdominal mass in the newborn.
• Renal ultrasonography of a multicystic dysplastic kidney characteristically reveals
multiple noncommunicating cysts of different sizes with dysplastic renal parenchyma and
an atretic ureter.
• Serial renal ultrasonography is indicated to monitor for involution of a multicystic
dysplastic kidney and compensatory hypertrophy or any abnormalities of the contralateral
kidney.
American Academy of Pediatrics 710
PREP® Self-Assessment PREPSA 2023

ABP Content Specifications(s)


• Recognize the clinical findings associated with multicystic dysplastic kidney in patients
of various ages

Suggested Readings
• Janjua HS, Lam KS, Gupta V, Krishna S. Congenital anomalies of the kidneys, collecting
system, bladder, and urethra. Pediatr Rev. 2019;40(6):619-626. doi:10.1542/pir.2018-
0242.
• Jawa NA, Rosenblum ND, Radhakrishnan S, Pearl RJ, Levin L, Matsuda-Abedini M.
Reducing unnecessary imaging in children with multicystic dysplastic kidney or solitary
kidney. Pediatrics. 2021;148(2):e2020035550. doi:10.1542/peds.2020-035550.
• Nguyen HT. Obstructive uropathy and vesicoureteral reflux. In: McInerny TK, Adam
HM, Campbell DE, DeWitt TG, Foy JM, Kamat DM, eds. American Academy of
Pediatrics Textbook of Pediatric Care. American Academy of Pediatrics; 2021:chap 299.
Accessed September 1, 2022. Pediatric Care Online.

American Academy of Pediatrics 711


PREP® Self-Assessment PREPSA 2023

Question 212
A 17-year-old adolescent boy is seen for a preparticipation sports physical evaluation for
clearance to play on the school soccer team. He is accompanied by his mother, who waits in a
separate room while his interview is conducted. He has no medical concerns. He has a history of
mild intermittent asthma and last used his albuterol inhaler 6 months ago. The adolescent’s vital
signs are normal for age, and his body mass index is at the 55th percentile. His physical
examination findings are normal.

As a part of the clinic protocol, a substance use screening tool reveals a high score. After
viewing his screening results, additional history questions are posed to the adolescent. He reports
drinking alcohol twice per week to relax after school and 6 to 8 beers every weekend at parties.
Last weekend, he drove his friend home after drinking alcohol but states that he “just had 2 or 3.”
He also admits to regularly taking pills at parties that are called “bars,” which he thinks might be
alprazolam. He uses a nicotine vape pen daily and intermittently uses marijuana. He denies the
use of any pain pills or other prescription drugs, or the use of intravenous drugs. He denies
experiencing any blackouts. His mother does not know about his alcohol use and is only aware
of his vaping.

Of the following, the BEST next step in this adolescent’s management is to


A. discuss with him that his drug and alcohol use must be disclosed to his mother
B. perform a urine drug screening test to identify any other drug use that he did not share
C. refrain from clearing him for soccer participation until he agrees to a drug counseling
program
D. set a goal to have him cut down on alcohol and nicotine and return in 3 months

American Academy of Pediatrics 712


PREP® Self-Assessment PREPSA 2023

Correct Answers: A
Confidentiality is a core tenet of adolescent and young adult health care. Adolescents should
have access to services including sexually transmitted infections treatment, mental health care,
contraception, and substance use treatment. Confidentiality provides adolescents with autonomy
in their health care decision-making and enhances the likelihood that they will be open and
honest regarding their health concerns. At the same time, family members are vitally important
in the care of adolescents and young adults. Physicians should encourage healthy communication
between adolescents and their caregivers.

The best time to introduce the importance of confidentiality in adolescent and young adult care is
at the first medical visit wherein a preadolescent patient is interviewed separately from the
caregiver or when an adolescent is initiating care with a new physician. One recommended
approach is to explain that the information shared between patient and physician is private and
confidential; however, caregivers must be informed if something is discussed that may cause
imminent danger or harm to them or to someone else.

The boy in the vignette has concerning alcohol use with frequent “binge drinking” (4 or more
standard drinks within a 2-hour period for females; 5 or more standard drinks for males), driving
under the influence of alcohol, and mixing sedatives (benzodiazepines) with alcohol. His
behaviors may imminently lead to danger to himself or others. Of the response options, the next
best step in his management would be to inform his mother about concerns regarding his
drug and alcohol use. One way to begin this conversation with the adolescent in the vignette
would be, “I’m grateful for your honesty with me today. I am concerned about your use of drugs
and alcohol, especially mixing sedatives with alcohol, which could lead to overdose or death.
Because these are serious issues that could put you at risk of harm, I have to share my concerns
with your parent. What do you think is the best plan for sharing this with your parent today?”
Most adolescent patients understand the need to break confidentiality when the reasoning is
explained. An adolescent may be less resistant to sharing information with a parent if it is clear
what exactly will be disclosed, who will give the details (ie, the patient or the physician), and
how it will be disclosed. The adolescent may desire that certain details be left out of the shared
information to protect friends or others. It is important to note that adolescents who need
treatment for substance use typically need parental involvement to follow through on necessary
referrals, obtain needed medications, and assist in supporting them at home.

While the adolescent in the vignette likely needs referral to a drug treatment program, his
clearance to play soccer should not depend on this factor. Extracurricular activities are typically
protective factors for adolescents and can be viewed as a motivator for him to decrease his drug
and alcohol use. The physician and adolescent should set a goal to cut down on his drug and
alcohol use, but he should be seen for follow-up much sooner than 3 months given his high-risk
behaviors.

Urine drug screening tests are not routinely recommended in the care of adolescents with
substance use problems in an outpatient setting and should only be performed with the
knowledge and consent of the adolescent. Urine drug screening tests are fraught with false-
American Academy of Pediatrics 713
PREP® Self-Assessment PREPSA 2023

positive and false-negative results and, thus, should not be used to “find” other drugs that the
patient may be using. The best way to elicit an accurate substance use history is to establish trust
with the adolescent through confidential care; the history is more accurate in most cases than a
standard urine drug screening test done in the office. A comprehensive urine drug screening test
performed with mass spectroscopy can identify all substances in the patient’s urine but typically
takes 1 to 2 weeks to result and is not useful in many clinical settings.

PREP Pearls
• Confidentiality is a vital part of adolescent care because of the need to foster autonomy,
establish trust, and identify sensitive and important health concerns.
• There are limits on confidentiality when the adolescent or others are in imminent danger.
The best way to involve parents (ie, “break confidentiality”) is to ask the adolescent how
they wish for you as the provider to share the information, including specific details they
would like kept private, if possible.

ABP Content Specifications(s)


• Understand when a parent must be notified about an adolescent’s medical condition

Suggested Readings
• Ford C, English A, and Sigman G. Confidential health care for adolescents: position
paper of the Society for Adolescent Medicine. J Adol Health. 2004;35:160-167.
doi:10.1016/j.jadohealth.2004.03.002.
• Levy S, Bagley S. Substance use: initial approach in primary care. In: McInerny TK,
Adam HM, Campbell DE, DeWitt TG, Foy JM, Kamat DM, eds. American Academy of
Pediatrics Textbook of Pediatric Care. American Academy of Pediatrics; 2021:chap 198.
Accessed September 1, 2022. Pediatric Care Online.
• Levy SJL, Williams JF; Committee on Substance Use and Prevention, Substance use
screening, brief intervention, and referral to treatment. Pediatrics.
2016;139(1):e20161211. doi:10.1542/peds.2016-1211.

American Academy of Pediatrics 714


PREP® Self-Assessment PREPSA 2023

Question 213
A 16-year-old adolescent girl with systemic lupus erythematosus is seen in the emergency
department for shortness of breath and light-headedness. Two weeks ago she had a cough,
congestion, low-grade fever, and rhinorrhea that resolved. She appears ill with a temperature of
36.5°C, heart rate of 110 beats/min, and respiratory rate of 25 breaths/min. Lung examination
reveals clear breath sounds without rales or wheezing; aeration is better on the right compared to
the left. She has mild abdominal tenderness, and a fluid wave is noted. Her pulses are weak, and
she has 1+ distal edema. Chest radiography results are shown in Item Q213.

Item Q213: Chest radiograph of the girl in the vignette. Courtesy of J. Kane

Of the following, the MOST likely additional finding for this girl is (a)
A. loud ejection murmur over the right sternal border
B. low central venous pressure
C. quiet precordium with distant heart sounds
D. systolic hypertension in the upright position

American Academy of Pediatrics 715


PREP® Self-Assessment PREPSA 2023

Correct Answer: C
The girl in the vignette is displaying signs and symptoms that cause concern for pericardial
tamponade. She has shortness of breath, nonfocal lung examination findings, an abdominal fluid
wave, diminished pulses, and distal edema. Chest radiography shows cardiomegaly and a pleural
effusion. With her medical history of systemic lupus erythematosus (SLE), and physical
examination and imaging findings concerning for serositis (abdominal fluid wave and pleural
effusion), pericardial tamponade and pericardial effusion are high on the differential diagnosis
list. Of the response choices, the most likely additional finding is a quiet precordium with distant
heart sounds.

Pericardial tamponade occurs when fluid accumulates in the virtual space between the pericardial
sac and the heart. Although chest radiography may reveal an enlarged cardiac silhouette
suggestive of this condition, bedside ultrasonography or echocardiography is used to make a
definitive diagnosis of pericardial effusion and assess for tamponade physiology. The fluid-filled
pericardial sac causes a diminished quality to heart sounds (quiet and distant), often described as
“muffled.” The pressure exerted by the pericardial fluid can compress the right atrium, restricting
filling. As a result, right atrial pressure and central venous pressure increase, limiting forward
flow into the heart. Thus, central venous pressure would be high, not low, for the girl in the
vignette.

Because left-sided stroke volume is dependent on right-sided filling, cardiac output will fall if
the tamponade physiology is not relieved. Children are often rate and stroke-volume dependent
for cardiac output and, unlike adults, do not augment cardiac contractility well. Thus, a child’s
tolerance of cardiac tamponade is much lower than that of an adult, which makes any cardiac
tamponade physiology a life-threatening emergency in children.

Classically, the signs of acute cardiac tamponade include the Beck triad: distended jugular veins,
muffled heart sounds, and hypotension (thus, hypertension would not be seen in the girl in the
vignette). However, the complete triad is rarely observed in children and adolescents, and may
not be present in cases where there is slow or chronic accumulation of fluid. Jugular venous
distension is rarely observed in young children and may be hard to see.
Hypotension is a late and ominous sign of impending cardiovascular collapse. Although
subjective in nature, heart sounds may sound distant on auscultation of the precordium in
children with cardiac tamponade.

Recognition of the physical examination findings of cardiac tamponade in children can be


challenging; a high index of suspicion based on history and risk factors is required. Risk factors
include chest trauma, central venous catheters in or near the heart, recent cardiac surgery,
malignancy, collagen vascular disease, severe renal disease, systemic infection with capillary
leak, and hypothyroidism.

Rheumatic heart disease occurring after streptococcal infection may result in aortic valve
insufficiency with a loud systolic ejection murmur. However, the adolescent in the vignette had a

American Academy of Pediatrics 716


PREP® Self-Assessment PREPSA 2023

recent upper respiratory tract infection, not streptococcal pharyngitis, making the finding of a
murmur associated with rheumatic heart disease unlikely.

Definitive management of pericardial tamponade requires evacuation of the pericardial fluid with
drain placement. Nonsteroidal antiinflammatory drugs may be used in cases where an
inflammatory response is suspected to be the underlying cause, particularly after cardiac surgery.
Acute management of tamponade physiology requires increasing the circulating venous volume
to overcome the increased right atrial pressure and promote forward flow through the heart. For
those with unstable hemodynamics, administration of parenteral isotonic fluid boluses while
preparing for drain placement may be life-saving. Although individuals with cardiac tamponade
may appear to be in acute heart failure, diuretics are contraindicated because lowering central
venous pressure may lead to acute cardiovascular collapse.

PREP Pearls
• A high index of suspicion is required to diagnose pericardial tamponade in children.
Bedside ultrasonography or echocardiography are useful in detecting pericardial fluid and
assessing for tamponade.
• Risk factors for pericardial tamponade include chest trauma, central venous catheters in
or near the heart, recent cardiac surgery, malignancy, collagen vascular disease, severe
renal disease, systemic infection with capillary leak, and hypothyroidism.
• Initial stabilization of individuals with pericardial tamponade requires resuscitation with
parenteral boluses of isotonic fluid to maintain cardiac output.

ABP Content Specifications(s)


• Recognize the clinical findings associated with pericardial tamponade

Suggested Readings
• Bolin EH, Tang X, Lang SM, Daily JA, Collins RT. Characteristics of non-postoperative
pediatric pericardial effusion: a multicenter retrospective cohort study from the Pediatric
Health Information System (PHIS). Pediatr Cardiol. 2018;39(2):347-353.
doi:10.1007/s00246-017-1762-y.
• LeWinter MM. Acute pericarditis. N Engl J Med. 2014;371(25):2410-2416.
doi:10.1056/NEJMcp1404070.
• McCulloch MA, Gajarski RJ. Congenital and acquired heart disease. In: McInerny TK,
Adam HM, Campbell DE, DeWitt TG, Foy JM, Kamat DM, eds. American Academy of
Pediatrics Textbook of Pediatric Care. American Academy of Pediatrics; 2021:chap 234.
Accessed September 1, 2022. Pediatric Care Online.
• Tunuguntla H, Jeewa A, Denfield SW. Acute myocarditis and pericarditis in children.
Pediatr Rev. 2019;40(1):14-25. doi:10.1542/pir.2018-0044.

American Academy of Pediatrics 717


PREP® Self-Assessment PREPSA 2023

Question 214
A 1-year-old boy is brought to the emergency department with burns on his hands. His mother
reports that while she was at work, the boy’s father called to tell her that the boy was sitting on
the kitchen counter next to the stove and accidentally put his hands in a pan of hot water. On
physical examination, the boy’s vital signs include a temperature of 36.5°C, heart rate of 155
beats/min, respiratory rate of 20 breaths/min, blood pressure of 90/65 mm Hg, and oxygen
saturation of 97% in room air. He is crying and appears to be in pain. There are circumferential
burns to both hands and wrists with blistering (Item Q214). The remainder of his physical
examination findings are unremarkable. Fluid resuscitation is initiated, and analgesics are
administered intravenously.

Of the following, the BEST next step in this boy’s management is to


A. administer a dose of intravenous antibiotics
B. call the local child abuse hotline to report suspected child abuse
C. debride the burn of loose and blistered skin and apply a dressing
D. transfer him to the nearest burn center

American Academy of Pediatrics 718


PREP® Self-Assessment PREPSA 2023

Correct Answer: B
The boy in the vignette has circumferential deep burns to both hands in a stocking and glove
pattern that is highly suspicious for child abuse. This pattern results from one or both hands or
feet being immersed and held in hot water. The best next step in this boy’s management is to
contact the local child abuse hotline to report suspected child abuse. It is not the role of health
care professionals to directly investigate child abuse. Rather, health care professionals are
required by law to report findings concerning for all forms of child abuse (eg, physical abuse,
sexual abuse, neglect). Every state has mandatory reporting laws, and physicians are mandatory
reporters.

Treatment of burns with prophylactic antibiotics is not recommended. Although the boy will
require debridement and transfer to a burn center for further care of the burns, this process will s

It is important to recognize history and physical examination findings that are suspicious for
child abuse. Careful attention must be paid to the reported history of the presenting injury within
the context of the child’s age, physical capabilities, and developmental stage. Item C214A
outlines helpful criteria for determining whether an injury should prompt investigation for child
abuse. A reported history of the event that has inconsistencies, does not adequately explain the
physical examination findings, or is not consistent with the child’s age or developmental stage
should be considered red flags. The boy in the vignette has deep bilateral burns in a glove
distribution with clear demarcation of the burn that is not consistent with the reported history of
him reaching into a pot of hot water.

Criteria for Consideration to Initiate a Child Physical Abuse Assessment


• Age and Development
• Nonmobile infant with any injury
• Injury in nonverbal child
• Injury inconsistent with child’s ability
• Statement of harm from a verbal child
• Injury
• Any injury in a nonmobile infant
• Uncommon in age group
• Occult finding
• Mechanism not plausible
• Multiple injuries, including involvement of multiple organs
• Injuries of differing ages
• Pattern of increasing frequency or severity of injury over time
• Patterned cutaneous lesions
• Bruises to torso, ear, or neck in child younger than age 4 years
• Burns to genitalia, stocking or glove distribution, branding, or pattern
• History
• Chief complaint does not contain caregiver concern for an injury
• and plausible history
• Caretaker response not commensurate to injury
• Unexplained delay in seeking care
• Lack of, inconsistent, or changing history
American Academy of Pediatrics 719
PREP® Self-Assessment PREPSA 2023

• Inconsistencies or discrepancies in histories provided by


• involved caretakers

Characteristics suggestive of burns caused by abuse include:


• Delayed presentation
• History not matching the burn injury
• Deep or extensive burns
• Burns in a patterned shape (such as an iron, cigarette)
• Stocking or glove distribution pattern on the extremities
• Bilateral burns of extremities
• Clear demarcation of burn edges without splash marks (Item C214B vs Item C214C)

Item C214D outlines a suggested protocol for evaluating suspected child abuse. Health care
professionals must always consider child abuse in the differential diagnosis when a child
presents with an injury.

Protocol for the Evaluation of Suspected Child Physical Abuse


• History of Present Illness
• Interview primary caretakers separately; note historian’s ability to provide history
• Ask caretakers about age-appropriate developmental abilities of child. Observe child if
possible
• Develop a timeline from when the child was last agreed upon to be in his or her usual
state of good health and note the following:
▪ Onset of symptoms and progression
▪ The patient’s observed mental status and activity level. Ask specifically about
how the child appeared at time of hand off between caretakers
• Note if there were any witnesses, photos taken of child, or other corroborating
information
• Social History
• List all adults having access to the child, including age, relationship, and contact
information
• List all children, including age and relationship; identify in which home they reside
• Note history of drug or alcohol abuse, intimate partner violence, mental illness, prior
history of involvement with child protective services
• Relevant Past Medical History
• Skeletal trauma: child or family history of bone disease, diet history
o Abusive head trauma (AHT) and cutaneous injuries: child or family history of bleeding
diathesis, eg, prolonged bleeding after circumcision, umbilical cord removal, or surgery
or as a result of past injuries
• Physical Examination
• Examine closely for possible intraoral injuries such as frenulum tears; explore all
unexposed surfaces: behind ears, genital region, and bottoms of feet
• Growth chart: obtain prior growth data, and with regard to AHT, note trajectory of head
circumferences
• Photodocumentation

American Academy of Pediatrics 720


PREP® Self-Assessment PREPSA 2023

o If photos are obtained, document in the medical record details of the photos taken,
including location of injuries, number of photos taken, date, and photographer
• If photodocumentation is unavailable, use a body diagram noting all cutaneous lesions
by size, location, and color
• Evaluation
• Indicated laboratory and imaging studies for current illness or injury
• Studies to assess occult injuries, such as skeletal survey
o Communication with appropriate subspecialists regarding findings and treatment,
including child abuse pediatricians when appropriate for referral and consultation
• Mandated Reporting and Safety
• Develop dialogue to inform parents about mandated reporting, safety, and reason for
report
• Ensure that forms and phone numbers for reporting are accessible
• Establish office process for specific scenarios with regard to obtaining imaging and
laboratory studies and process for transfer to appropriate
facility for evaluation and treatment, including protocol for accessing expertise of child
abuse pediatrician
o Facilitate thorough sibling assessment, including appropriate imaging, laboratory
studies, and interpretation; establish protocol to ensure results of sibling assessments are
communicated to others in the investigation, including primary care clinician
• Ensure medical record and photodocumentation accessibility for investigators (consent
not required after report to child welfare)
• Discuss disposition of, medical follow-up, and supportive services for patient with child
welfare case worker

PREP Pearls
• It is important to recognize history and physical examination findings that are suspicious
for child abuse. Careful attention must be paid to the history of the presenting injury in
the context of the child’s age, physical capabilities, and developmental stage.
• Burns with well-demarcated areas of similiar depth without splash or drip marks, or
“stocking or glove” distribution are highly suggestive of child abuse.
• As mandated reporters, physicians are required to make a report to the local child
protective services if there is reasonable suspicion that an injury is the result of abuse or
neglect.

ABP Content Specifications(s)


• Recognize the historical, clinical, and radiologic findings associated with inflicted
injuries (bruises, burns, fractures, intracranial injuries, and intra-abdominal injuries) and
differentiate them from those caused by accidental trauma and other non-traumatic causes

American Academy of Pediatrics 721


PREP® Self-Assessment PREPSA 2023

Suggested Readings
• Asnes AG, Leventhal JM. Managing child abuse: general principles. Pediatr Rev.
2010;31(2):47-55. doi:10.1542/pir.31-2-47
• Dubowitz H, Finkel MA. Physical abuse and neglect. In: McInerny TK, Adam HM,
Campbell DE, DeWitt TG, Foy JM, Kamat DM, eds. American Academy of Pediatrics
Textbook of Pediatric Care. American Academy of Pediatrics; 2021:chap 367. Accessed
September 1, 2022. Pediatric Care Online.
• Glick JC, Lorand MA, Bilka KR. Physical abuse of children. Pediatr Rev.
2016;37(4):146-158. doi:10.1542/pir.2015-0012.
• Sirotnak AP, Grigsby T, Krugman RD. Physical abuse of children. Pediatr Rev.
2004;25(8): 264-277. doi:10.1542/pir.25-8-264

American Academy of Pediatrics 722


PREP® Self-Assessment PREPSA 2023

Question 215
A 4-month-old, full-term male infant with myelomeningocele, who underwent repair on the
second day after birth, is seen for a routine health supervision visit. He has been doing well
overall and is followed in the multidisciplinary myelomeningocele clinic. He can coo, laugh,
grasp at toys, and bring his hands to his mouth. He is not yet rolling but can push himself up on
his forearms when placed prone. He has started vomiting with feedings over the past week. He is
not choking or gagging on feedings. The infant’s head circumference growth chart is shown in
Item Q215.

Of the following, the MOST likely condition causing this infant’s symptoms and signs is
A. Chiari II malformation
B. dysgenesis of the corpus callosum
C. hydrocephalus
D. polymicrogyria

American Academy of Pediatrics 723


PREP® Self-Assessment PREPSA 2023

Correct Answer: C
The infant in the vignette has an increasing head circumference and intermittent vomiting, which
should raise concern for evolving increased intracranial pressure. With his history of
myelomeningocele, these symptoms and signs are concerning for the development of
hydrocephalus, a common comorbid condition. Hydrocephalus affects 60% to 95% of children
with myelomeningocele, especially with higher-level lesions; ongoing surveillance for the
development of signs and symptoms of hydrocephalus is important for prompt recognition and
treatment.

Myelomeningocele is a neural tube defect in which a portion of the spinal cord and nerves within
the meningeal sac protrude through a defect in the vertebral column. Affected children
experience motor and sensory dysfunction below the affected level of the spinal cord, the
severity of which varies with the lesion level. Additional brain, spine, vertebral, and soft tissue
abnormalities can accompany myelomeningocele (Item C215).

The most common brain malformation accompanying myelomeningocele is a Chiari II


malformation, in which there is downward displacement of part of the brainstem and cerebellum.
Symptoms may include dysphagia, choking, hoarseness, stridor, central apnea, disordered
breathing, opisthotonus, and in rare cases, sudden death. Many children with this condition are
asymptomatic.

Hydrocephalus commonly develops in children with myelomeningocele due to abnormal


cerebrospinal fluid flow. Symptoms and signs may include a bulging or tense fontanelle, rapidly
accelerating head circumference, lethargy, paralysis of upward gaze (sunsetting eyes), cranial
nerve dysfunction, vomiting, headache, and irritability. Presence of these symptoms and signs
should prompt urgent imaging and referral to pediatric neurosurgery for definitive treatment,
such as placement of a ventriculoperitoneal shunt.

American Academy of Pediatrics 724


PREP® Self-Assessment PREPSA 2023

Brain malformations, such as polymicrogyria, agenesis or dysgenesis of the corpus callosum,


cortical dysplasia, frontal pachygyria, and small cerebellum, can accompany myelomeningocele.
The presence of a brain malformation increases the risk of epilepsy and/or intellectual disability.
Although myelomeningocele is a nonprogressive condition, the potential for interval
development of complications related to common comorbid malformations necessitates
multidisciplinary surveillance and care coordination. A change in a child’s functional status
should prompt urgent investigation, as children with myelomeningocele remain at risk for life-
threatening complications (eg, hydrocephalus).

PREP Pearls
• Myelomeningocele is a neural tube defect in which a portion of the spinal cord and
nerves within the meningeal sac protrude through a defect in the vertebral column.
Affected children experience motor and sensory dysfunction below the affected level of
the spinal cord, the severity of which varies with the lesion level.
• Malformations of the brain, spinal cord, soft tissues, and vertebral column can
accompany myelomeningocele; Chiari II malformations, hydrocephalus, and
malformations of the brain are common.
• Hydrocephalus is a life-threatening condition which can present with a bulging or tense
fontanelle, rapidly accelerating head circumference, lethargy, paralysis of upward gaze
(sunsetting eyes), cranial nerve dysfunction, vomiting, headache, and irritability.

ABP Content Specifications(s)


• Recognize other abnormalities commonly associated with myelomeningocele

Suggested Readings
• Burke R, Liptak G; Council on Children With Disabilities. Providing a primary care
medical home for children and youth with spina bifida. Pediatrics. 2011;128(6):e1645-
e1657. doi:10.1542/peds.2011-2219.
• Houtrow A. Spina bifida. In: McInerny TK, Adam HM, Campbell DE, DeWitt TG, Foy
JM, Kamat DM, eds. American Academy of Pediatrics Textbook of Pediatric Care.
American Academy of Pediatrics; 2021:chap 332. Accessed September 1, 2022. Pediatric
Care Online.
• Liptak G, Dosa N. Myelomeningocele. Pediatr Rev. 2010;31(11):443-450.
doi:10.1542/pir.31.11.443.
• Lu V, Niazi T. Pediatric spinal cord diseases. Pediatr Rev. 2021;42(9):486-499.
doi:10.1542/pir.2020-000661.

American Academy of Pediatrics 725


PREP® Self-Assessment PREPSA 2023

Question 216
A full-term neonate is examined in the newborn nursery. He was delivered via cesarean section
with a birth weight of 7 lb, 10 oz. On physical examination, he is in no distress and has
macrocephaly, posteriorly rotated ears, a symmetric defect of both upper extremities with intact
thumbs, and a 3/6 mid-diastolic murmur best heard at the lower left sternal border. The
remainder of his physical examination findings are unremarkable.

Results of a complete blood cell count are shown:


Laboratory Test Result
White blood cell count 7,400/µL (7.4 × 109/L)
Neutrophils 59%
Lymphocytes 33%
Monocytes 7%
Eosinophils 1%
Hemoglobin 14.4 g/dL (144 g/L)
Platelet count 30 × 103/µL (30 × 109/L)
Mean corpuscular volume 100 fL

Of the following, the BEST treatment for this neonate’s condition is


A. administration of corticosteroids
B. administration of intravenous immunoglobulin
C. bone marrow transplantation
D. platelet transfusion

American Academy of Pediatrics 726


PREP® Self-Assessment PREPSA 2023

Correct Answer: D
The neonate in the vignette has thrombocytopenia absent radius syndrome (TAR). This condition
is characterized by severe thrombocytopenia and physical examination findings, including
skeletal abnormalities of the upper (Item C216) or lower extremities, short stature, cardiac
abnormalities (most commonly septal defects), and characteristic craniofacial features
(posteriorly rotated ears, micrognathia, and macrocephaly). Thrombocytopenia absent radius
syndrome is a congenital thrombocytopenia that is usually inherited in an autosomal recessive
manner. It presents in infancy with thrombocytopenia, which may result in bleeding if not
identified and treated promptly.

The mainstay of treatment for TAR syndrome is supportive care with platelet transfusions.
Irradiated, single-donor platelets should be administered to prevent adverse effects. Because the
thrombocytopenia associated with TAR syndrome usually resolves after 1 year of age, definitive
treatment with bone marrow transplantation is not warranted. Bone marrow transplantation is the
recommended treatment for some congenital causes of thrombocytopenia (eg, congenital
amegakaryocytic thrombocytopenia and Wiskott-Aldrich syndrome).

Thrombocytopenia may be described as mild (50,000/µL–150,000/µL), moderate (<50,000/µL),


or severe (<30,000/µL). Signs and symptoms of bleeding (eg, bruises, petechiae, and epistaxis)
become evident with moderate to severe thrombocytopenia. Spontaneous bleeding, unprovoked
by trauma, is more common in severe thrombocytopenia, especially with platelet counts less than
10,000/µL. Significant bleeding at higher platelet counts should raise concern for other bleeding
disorders, such as disorders of platelet function. A rapidly enlarging hematoma is indicative of a
significant bleeding disorder (eg, hemophilia) and requires prompt intervention and management.
American Academy of Pediatrics 727
PREP® Self-Assessment PREPSA 2023

Administration of intravenous immunoglobulin is used to treat neonatal alloimmune


thrombocytopenia but is not recommended for this neonate with TAR. Corticosteroids are used
to treat immune causes of thrombocytopenia, such as immune (idiopathic) thrombocytopenia but
would not be appropriate treatment for this neonate who does not have an immune (idiopathic)
thrombocytopenia.

PREP Pearls
• Thrombocytopenia absent radius syndrome causes congenital thrombocytopenia, which
may be severe.
• Treatment of severe thrombocytopenia associated with thrombocytopenia absent radius
syndrome is supportive with administration of irradiated, single-donor platelets as
needed.

ABP Content Specifications(s)


• Plan appropriate management of thrombocytopenia associated with TAR syndrome
• Understand the hematologic significance of a rapidly enlarging hematoma
• Recognize the laboratory findings associated with thrombocytopenia

Suggested Readings
• de Ybarrondo L, Barratt MS. Thrombocytopenia absent radius syndrome. Pediatr Rev.
2011;32(9):399-400; discussion 400. doi:10.1542/pir.32-9-399.
• Figueiredo L, Levy AS. Petechiae and purpura. In: McInerny TK, Adam HM, Campbell
DE, DeWitt TG, Foy JM, Kamat DM, eds. American Academy of Pediatrics Textbook of
Pediatric Care. American Academy of Pediatrics; 2021:chap 181. Accessed September 1,
2022. Pediatric Care Online.
• Haley KM. Platelet disorders. Pediatr Rev. 2020;41(5):224-235. doi:10.1542/pir.2018-
0359.
• Lanzkowsky P, Lipton J, Fish JD. Lanzkowsky’s Manual of Pediatric Hematology and
Oncology. 6th ed. Elsevier Inc; 2016:239-254.
• Morrone K. Thrombocytopenia in the newborn. Neoreviews. 2018;19(1):e34–e41.
doi:10.1542/neo.19-1-e34 .

American Academy of Pediatrics 728


PREP® Self-Assessment PREPSA 2023

Question 217
A 17-year-old immunocompetent adolescent is being evaluated for a painful vesicular rash of 3
days’ duration over the tip of the nose and mid-forehead (Item Q217). His 7-year-old sister who
lives in the same household received a hematopoietic stem cell transplant 6 months ago and
continues to receive immunosuppressive treatment.

Item Q217: Rash on the face of the patient described in the vignette.
Courtesy of A. Noor

Of the following, the BEST next step in management for this family is

Response Choice Management of 17-year-old Management of 7-year-old


Row A Ophthalmology referral Varicella-zoster immune globulin
Row B Otolaryngology referral Varicella-zoster immune globulin
Row C Infectious diseases referral Valacyclovir prophylaxis
Row D Immunology referral Valacyclovir prophylaxis

A. Row A
B. Row B
C. Row C
D. Row D

American Academy of Pediatrics 729


PREP® Self-Assessment PREPSA 2023

Correct Answer: A
Painful vesicles on the tip of the nose and a vesicular rash over the forehead meet the criteria for
Hutchinson sign, defined as varicella-zoster virus (VZV) infection involving the nasociliary
nerve (a branch of the ophthalmic nerve, cranial nerve V, innervating the nose, eyelid, and
conjunctiva). In cases of herpes zoster ophthalmicus, individuals with Hutchinson sign have a
3.4-fold higher incidence of ocular inflammation and 4-fold higher incidence of corneal
denervation. Prompt ophthalmologic consultation is recommended to evaluate for ocular
involvement and assess the extent of infection. Herpes zoster ophthalmicus, VZV involvement of
the ophthalmic division of the fifth cranial nerve, is a potentially sight-threatening condition.
Otolaryngologists, infectious disease specialists, and immunologists may also be involved in the
care of affected children.

The adolescent’s 7-year-old sister is considered severely immunocompromised, because she is a


recent hematopoietic stem cell transplant recipient. As she is a member of the adolescent’s
household and the exposure to VZV is within 10 days, administration of varicella-zoster immune
globulin is recommended at a dose of 125 units per 10 kg, intramuscularly (preferably within 96
hours). If varicella-zoster immune globulin is not available, oral valacyclovir can be
administered for 22 days from the time of exposure. Chemoprophylaxis with valacyclovir can be
considered for immunocompetent hosts (eg, older adolescents, adults) or mildly
immunocompromised hosts with no prior history of VZV immunity (lack of varicella-zoster
immunization or natural infection) within 7 days of exposure.

Varicella-zoster is a herpesvirus 3. Primary infection results in chickenpox (varicella),


characterized by a pruritic vesicular erythematous rash in different stages at the same time
(papules, vesicles, and crusted lesions). At the time of primary infection, VZV has a propensity
to the development of latent infection in the sensory ganglia.

Reactivation results in herpes zoster (shingles) with skin rash and acute neuritis. The typical
presentation of shingles is painful grouped vesicles on an erythematous base, limited to 1 to 2
sensory dermatomes (commonly over the thorax or lumbar region).

Infected individuals are contagious within 24 to 48 hours before the appearance of rash.
Recommendations for postexposure management are based on age, susceptibility to varicella
infection (no prior evidence of varicella infection or varicella vaccination), and the immune
status of the host (Item C217).

American Academy of Pediatrics 730


PREP® Self-Assessment PREPSA 2023

Prevention for children exposed to varicella-zoster virus.


a
Persons who receive bone marrow transplants should be considered nonimmune regardless of previous
history of varicella disease or varicella vaccination in themselves or in their donors.
b
Immunocompromised children include those with congenital or acquired T-lymphocyte
immunodeficiency, including leukemia, lymphoma, and other malignant neoplasms affecting the
bone marrow or lymphatic system; children receiving immunosuppressive therapy, including $2
mg/kg/day of systemic prednisone (or its equivalent) for $14 days; all children with HIV infection
regardless of CD4 %; and all bone marrow transplant patients regardless of pretransplant
immunity status.
American Academy of Pediatrics 731
PREP® Self-Assessment PREPSA 2023

c
No postexposure prophylaxis, but age-appropriate vaccination still recommended for protection against
subsequent exposures. If the exposure occurred during an outbreak, 2-dose vaccination is
recommended for preschool-aged children under 4 years of age for outbreak control.
d
If 1 prior dose of varicella vaccine has been received, a second dose should be administered at $4 years
of age. If the exposure occurred during an outbreak, a second dose is recommended for
preschool-aged children under 4 years of age
for outbreak control.
e
Contraindications include patients who are allergic to a vaccine component, or who are
immunocompromised (see above footnote), or pregnant. Caution should be used in patients on
salicylates. Vaccine may not be as effective if patient has recently received IGIV, whole blood, or
plasma transfusions, and for this reason it is recommended that varicella vaccine be withheld for
3–11 months, depending on the dose, after administration of these products.
f
VariZIG was approved by the United States Food and Drug Administration in December 2012. The
product is manufactured by Cangene Corporation (Winnipeg, Canada) and distributed in the
U.S. by FFF Enterprises (Temecula, California; 800-843-7477; www.fffenterprises.com).
g
If VariZIG and IGIV are not available, some experts recommend prophylaxis with oral acyclovir (20
mg/kg per dose administered four times per day, with a maximum daily dose of 3,200 mg) or oral
valacyclovir (if .3 months of age; 20 mg/kg per dose administered 3 times per day, with a
maximum daily dose of 3,000 mg) beginning 7 to 10 days after exposure and continuing
for 7 days

PREP Pearls
• Reactivation of varicella-zoster virus (shingles) in children typically presents as painful
grouped vesicles over an erythematous base, limited to 1 to 2 sensory dermatomes.
• Presence of varicella-zoster virus vesicles over the tip of the nose and forehead
(Hutchinson sign) indicates a high risk of ocular involvement.
• Severely immunocompromised children exposed to varicella-zoster virus (primary or
reactivation) benefit from passive prophylaxis with varicella zoster immune globulin if
there is a high likelihood of exposure (household member), high risk of subsequent
infection, and predisposition to severe complications.

ABP Content Specifications(s)


• Plan appropriate antiviral therapy for normal and immunocompromised patients who
have varicella-zoster virus infection
• Plan the appropriate management of an immunocompromised patient exposed to varicella

American Academy of Pediatrics 732


PREP® Self-Assessment PREPSA 2023

Suggested Readings
• American Academy of Pediatrics. Varicella-zoster virus infections. In: Kimberlin DW,
Barnett ED, Lynfield R, Sawyer MH, eds. Red Book: 2021–2024 Report of the
Committee on Infectious Diseases. 32nd ed. American Academy of Pediatrics; 2021.
Accessed September 1, 2022. Red Book Online.
• Arvin AM. Varicella-zoster virus. In: Long SS, Prober CG, Fischer M. Principles and
Practice of Pediatric Infectious Diseases. 5th ed. Elsevier; 2018; chap 205:1065-1073.
• Blair RJ. Varicella zoster virus. Pediatr Rev. 2019;40(7):375-377. doi: 10.1542/pir.2017-
0242.
• Gershon, A., Breuer, J., Cohen, J. et al. Varicella zoster virus infection. Nat Rev Dis
Primers. 2015;1. doi: 10.1038/nrdp.2015.16.
• Kimberlin DW, Price N. Chickenpox. In: McInerny TK, Adam HM, Campbell DE,
DeWitt TG, Foy JM, Kamat DM, eds. American Academy of Pediatrics Textbook of
Pediatric Care. American Academy of Pediatrics; 2021:chap 227. Accessed September 1,
2022. Pediatric Care Online.

American Academy of Pediatrics 733


PREP® Self-Assessment PREPSA 2023

Question 218
A 9-year-old boy with attention-deficit/hyperactivity disorder and a learning disorder is brought
to the office by his mother for a scheduled medication follow-up visit. His 4th grade teacher
recently implemented a reward system at school for when he remains seated appropriately during
class and for completing his work. The new system is working well, and the boy’s mother no
longer receives phone calls or emails regarding his behavior at school.

The boy’s behavior continues to be challenging at home. His mother provides praise when he
completes a task the first time she asks and has stopped “giving in” when he cries for more time
on his tablet device. She wants to implement a reward system to help with homework
completion. The boy’s grandmother, who cares for him in the afternoons, is not interested in
using a reward system. According to his mother, “if he starts crying about homework, his
grandmother will yell at him.” The boy is not completing his homework and is now refusing to
go to his grandmother’s house. His mother is worried that this is having a negative impact on the
boy’s relationship with his grandmother. The grandmother has suggested that his medication
dose be increased.

Of the following, the BEST next step in this boy’s management is to recommend that the
A. boy attend an after-school program that provides homework assistance
B. grandmother leave the room until the boy completes his homework
C. grandmother spend 10 to 15 minutes daily engaging him in an activity he enjoys
D. medication dose be increased to help with emotional regulation

American Academy of Pediatrics 734


PREP® Self-Assessment PREPSA 2023

Correct Answer: C
The family in the vignette is struggling with behavior management surrounding homework
completion. Behavior modification consists of modifying a child’s environment with positive
and negative consequences that reinforce desired behaviors and decrease unwanted behaviors.
For behavior modification to be successful, parents and caregivers must be able to attend to their
child to provide positive reinforcement and praise. This approach is currently difficult for the
grandmother, whose interaction with the boy has largely become a daily battle over homework
completion. Recommending that the grandmother spend 10 to 15 minutes of quality time doing
an activity the boy enjoys will provide an opportunity for them to experience positive
interactions and facilitate the implementation of other behavior modifications.

Behavior modification increases desired behaviors through positive reinforcement. Positive


reinforcement may include praise, rewards, and/or a token reward system. The praise should be
focused on the behavior and be appropriate for the child’s developmental level. For example, “I
like how you focused on your math problems instead of talking to your brother.”

Negative reinforcement involves taking away something unpleasant in order to reinforce the
desired behavior (eg, the grandmother will not yell at the boy in the vignette if he does not cry
about his homework). This approach is less effective than positive reinforcement.
Recommending that the boy attend an after school program so his homework does not need to be
completed at home (avoiding a meltdown) is a form of negative reinforcement; it would not
address the concerns about the boy’s relationship with his grandmother, and may limit the
consistency of consequences between school and home. For behavior modification to be
successful, consistency across environments is key; all adults (parents, caregivers, teachers)
should be utilizing the same strategies.

Another aspect of behavior modification is ignoring or disciplining inappropriate behaviors.


Recommending that the grandmother ignore the boy’s crying may be useful in some situations
(eg, when his video game is turned off because it is bedtime), but would not be appropriate in
this situation given the boy’s history of attentiondeficit/hyperactivity disorder (ADHD) and
learning difficulties.

Behavior modification does not change the underlying learning or behavior disorder, but acts to
change the environment and how parents and caregivers respond to both positive and negative
behaviors. Behavior modification strategies can be helpful in children with ADHD, learning, and
behavior problems by providing structure and consistent consequences. For most affected
children, behavior modification alone is not enough. For many schoolage children with moderate
to severe ADHD, medication (typically a stimulant medication) and behavior therapy (which
may include behavior modification strategies) is recommended.

While many children with ADHD have difficulty with emotional regulation, increasing the boy’s
current medication dose does not address the specific behavior that is occurring (crying during
homework time in an attempt to get out of doing it). The boy’s ADHD symptoms are much
improved in school on his current medication dose, therefore one must consider the impact of the
American Academy of Pediatrics 735
PREP® Self-Assessment PREPSA 2023

environment on his behavior and approach management from that context. For children with a
specific learning disorder, school interventions, including modifications and accommodations,
are frequently indicated. Homework may need to be modified or reduced to be appropriate for
the child’s academic and cognitive functioning. Behavior modification may play a role in
providing structure and positive reinforcement for completion of work.

It is important for pediatricians to ask about behavior concerns during health supervision visits
and provide support and referrals when appropriate. Identifying environmental factors preceding
the behavior and the parent’s/caregiver’s response to the behavior are important to consider
when providing recommendations. Ongoing behavior therapy or counseling for mild behaviors,
and medication for moderate to severe or aggressive behaviors may also be indicated.

PREP Pearls
• Behavior modification works by changing a child’s environment to encourage and
reinforce desired behaviors while ignoring or disciplining inappropriate behaviors.
• Praise and positive reinforcement are more effective when there is a positive and
supportive relationship between a child and their parents/caregivers.
• Behavior modification can be helpful in children with learning and behavior disorders but
by itself may not be enough.

ABP Content Specifications(s)


• Understand the advantages and limitations of behavior modification approaches in the
overall management of learning and behavioral problems

Suggested Readings
• Kavan MG, Saxena SK, Rafiq N. General parenting strategies: practical suggestions for
common child behavior issues. Am Fam Physician. 2018;97(10):642-648.
https://www.aafp.org/afp/2018/0515/p642.html.
• Wolraich ML. Addressing behavior problems among school-aged children: traditional
and controversial approaches. Pediatr Rev. 1997;18(8):266-270. doi:10.1542/pir.18-8-
266.

American Academy of Pediatrics 736


PREP® Self-Assessment PREPSA 2023

Question 219
A 14-year-old adolescent boy is seen for a health supervision visit. He has a history of exercise-
induced asthma and had an elevated blood pressure at a visit 6 months ago. He participates in a
community baseball league. Today, his physical examination findings are significant for a blood
pressure of 125/79 mm Hg (measured by manual sphygmomanometer), a body mass index of
35.9 kg/m2, (138% of the 95th percentile) and acanthosis nigricans around his neck. The
remainder of his physical examination findings are unremarkable.

Of the following, the BEST next step in this adolescent’s management is to


A. order a fasting glucose level, basic metabolic panel, lipid panel, liver transaminases, and
urinalysis
B. provide nutrition counseling and reassurance that sports participation will lead to weight
loss
C. recommend that he discontinue sports participation pending evaluation of his
hypertension
D. refer him for ambulatory blood pressure monitoring

American Academy of Pediatrics 737


PREP® Self-Assessment PREPSA 2023

Correct Answer: A
The adolescent in the vignette meets criteria for the diagnosis of obesity with a body mass index
(BMI) ≥95th percentile for age and sex. He has evidence of comorbidities, including insulin
resistance (acanthosis nigricans) and hypertension. The best next step in his management is to
order a fasting glucose level, basic metabolic panel, lipid panel, liver transaminases, and
urinalysis to screen for additional comorbidities associated with obesity and hypertension.
Current clinical guidelines recommend screening children and adolescents with a BMI ≥95th
percentile for diabetes (fasting glucose level, hemoglobin A1c, or 2-hour oral glucose tolerance
test), hyperlipidemia (lipid panel), and fatty liver disease (alanine aminotransferase and aspartate
aminotransferase levels). A basic metabolic panel and urinalysis is indicated for all adolescents
with hypertension.

Body mass index is a tool used to categorize an individual’s risk for comorbidities and
complications associated with overweight and obesity. Pediatricians have the opportunity to
monitor growth over time and provide counseling regarding lifestyle changes to help children
achieve and maintain a healthy weight.

A BMI at ≥95th percentile in children over age 2 years defines obesity. Overweight is defined as
a BMI between the 85th to <95th percentile. Body mass index is calculated using the formula:
weight (kg)/[height (m)]2. When the BMI is significantly above the 95th percentile, it is useful to
express it as a percent of the 95th percentile BMI (eg, a BMI of 138% indicates a BMI that is
38% greater than the BMI defined as the 95th percentile). Expressing high BMI levels in this
manner facilitates monitoring of changes in BMI percentile over time. A BMI ≥120% of the 95th
percentile is considered severe obesity. Online calculators are available that calculate high BMI
levels as a percent of the 95th percentile.

Body mass index is a screening tool. Body mass index has limitations in individuals with high
muscle mass. The advantages of BMI over other direct measures of adiposity are its non-invasive
nature, ease of calculating, and the availability of standards for age. Body mass index for age
correlates well with subcutaneous fat mass and total body fatness in adolescents. Although BMI
is not a perfect proxy for adiposity, the measurement of BMI for age has been correlated with the
comorbidities of hyperlipidemia, hypertension, and insulin resistance.

Identification of elevated BMI, categorization of risk, and further evaluation is important because
comorbidities pose significant health risks. These comorbidities include diabetes, hypertension,
polycystic ovary syndrome, hyperlipidemia, and fatty liver disease.

Based on the 2017 American Academy of Pediatrics revised hypertension guidelines the
adolescent in the vignette’s blood pressure is elevated. It is important that this measurement be
confirmed using a manual sphygmomanometer with an appropriately sized cuff. Because the
adolescent in the vignette had an elevated blood pressure 6 months ago, the guidelines
recommend measurement of both an upper and lower extremity blood pressure at this visit. If
there is no discrepancy in these measurements, the boy’s blood pressure should be measured
again in 6 months. If his blood pressure remains elevated for 12 months he should be referred for
American Academy of Pediatrics 738
PREP® Self-Assessment PREPSA 2023

ambulatory blood pressure monitoring. Because this boy has elevated blood pressures
documented for only 6 months, it is too soon to refer him for ambulatory blood pressure
monitoring.

To decrease their risk of cardiovascular disease and diabetes, children with obesity should be
encouraged to make lifestyle changes that promote a decrease in their BMI percentile. Sports
participation provides an important opportunity for physical activity. The adolescent in the
vignette does not have any contraindications to sports participation and should be encouraged to
participate. However, sports participation alone does not guarantee weight loss. In addition to
physical activity, recommended lifestyle changes would include a healthy diet, adequate sleep,
and stress reduction.

PREP Pearls
• Body mass index is an important screening tool and assists in assessing an individual's
risk for weight-related comorbidities.
• Children with a body mass index in the overweight (85th to <95th percentile) or obesity
(≥95th percentile) categories should be screened for diabetes, hyperlipidemia, and fatty
liver disease. A body mass index greater than 99th percentile may be best understood
when described as the percentage greater than the 95th percentile.
• Children with obesity, including those with comorbidities, should be encouraged to
continue or begin sports participation.

ABP Content Specifications(s)


• Understand the importance of body mass index (BMI) in identifying obesity and
overweight

Suggested Readings
• Barlow SE; Expert Committee. Expert committee recommendations regarding the
prevention, assessment, and treatment of child and adolescent overweight and obesity:
summary report. Pediatrics. 2007; 120 (Suppl 4):S164-S192. doi:10.1542/peds.2007-
2329C.
• Cuda SE, Censani M. Pediatric obesity algorithm: a practical approach to obesity
diagnosis and management. Front Pediatr. 2019;6:431. doi:10.3389/fped.2018.00431.
• Flynn JT, Kaelber DC, Baker-Smith CM, et al; Subcommittee on Screening and
Management of High Blood Pressure in Children. Clinical practice guideline for
screening and management of high blood pressure in children and adolescents. Pediatrics.
2017;140(3):e20171904. doi:10.1542/peds.2017-1904.
• Gulati AK, Kaplan DW, Daniels SR. Clinical tracking of severely obese children: a new
growth chart. Pediatrics. 2012;130(6):1136-40. doi:10.1542/peds.2012-0596.
• Styne DM, Arslanian SA, Connor EL, et al. Pediatric obesity-assessment, treatment, and
prevention: an Endocrine Society clinical practice guideline. J Clin Endocrinol Metab.
2017;102(3):709-757. doi:10.1210/jc.2016-2573.

American Academy of Pediatrics 739


PREP® Self-Assessment PREPSA 2023

Question 220
Adverse events related to insulin medication errors are tracked at a hospital. In an effort to
reduce these events, the hospital quality office appoints a committee tasked with developing a
quality improvement initiative.

Of the following, the committee’s BEST first step is to


A. establish improvement measures
B. examine the current processes
C. select changes to be implemented
D. set the specific aim of the project

American Academy of Pediatrics 740


PREP® Self-Assessment PREPSA 2023

Correct Answer: D
Of the response choices, the best first step for the committee in the vignette is to set the aim of
the quality improvement project. The Langley Model for Improvement uses a series of Plan-Do-
Study-Act (PDSA) cycles as the basis for continuous improvement and is a common approach to
quality improvement in health care. The first step of this model is to form a team, in this case a
committee, to work on the project. The team should review what the project is trying to
accomplish (ie, set the aim or goal of the project) and consider what system(s) will be affected by
the proposed changes. There should be a team member representing each aspect of the relevant
process.

After forming the team, a specific aim of the project should be set. The aim should be time-
specific and measurable. The patient population is also determined in this step. A potential aim
for the project in the vignette is to reduce adverse events related to insulin errors in the hospital
by 80%.

After setting the aim of the project, the team should determine what will be measured to
determine if the proposed changes are effective or not. For the project in the vignette, the team
may choose to measure the number of hypoglycemic events due to improper insulin
administration and/or the number of patient safety reports related to insulin errors.

The team must then examine the current processes in place to determine potential changes. The
next step is to select changes to be implemented. For example, if in the current state one nurse
confirms an insulin order, a proposed change may be to have 2 nurses independently confirm the
insulin dose. When determining proposed changes, the team should ensure that the change is not
too large, or it may be difficult for the system as a whole to adopt if it is found to be effective.

The proposed changes must then be tested with PDSA cycles. In the Plan stage, the team
determines the specific data to be collected and predicts what will happen with the change. In the
Do stage, the change is provisionally implemented prior to full-scale roll-out. Data are analyzed
in the Study stage and compared to predictions made in the Plan stage. In the Act stage,
modifications to the change are made, and the cycle is repeated with another test of the modified
change.

Once the PDSA cycles have yielded the desired results, the change is then formalized on a larger
scale. For example, the project may start with patients with diabetes in one unit of the hospital.
The PDSA cycles can be repeated to determine if the change is successful on this larger scale,
such as an entire floor of the hospital. If so, the change can be applied to patients with diabetes
hospital-wide.

American Academy of Pediatrics 741


PREP® Self-Assessment PREPSA 2023

PREP Pearls
• Quality improvement projects should follow a stepwise process for continuous
improvement.
• Once an improvement change is proposed, it can be tested with successive Plan-Do-
Study-Act cycles, modified, retested, and eventually implemented.

ABP Content Specifications(s)


• Identify the components of the Langley Model for Improvement

Suggested Readings
• Bartman T, McClead RE. Core principles of quality improvement and patient safety.
Pediatr Rev. 2016;37(10):407-417. doi:10.1542/pir.2015-0091.
• How to improve. Institute for Healthcare Improvement. 2022. Accessed September 1,
2022. http://www.ihi.org/resources/Pages/HowtoImprove/default.aspx.
• Langley GL, Moen R, Nolan KM, Nolan TW, Norman CL, Provost LP. The
Improvement Guide: A Practical Approach to Enhancing Organizational Performance.
2nd ed. Jossey-Bass Publishers; 2009.
• Schriefer J, Leonard MS. Patient safety and quality improvement: an overview of QI.
Pediatr Rev. 2012;33(8):353-359; quiz 359-360. doi:10.1542/pir.33-8-353.

American Academy of Pediatrics 742


PREP® Self-Assessment PREPSA 2023

Question 221
A pediatrician is urgently called to the delivery room to examine a newborn who had been
delivered precipitously by a mother who did not receive any prenatal care. The infant is
breathing comfortably in room air. Physical examination reveals a term-appearing infant with the
abdominal findings shown (Item Q221).

Item Q221: Findings for the infant described in the vignette.


Reprinted with permission from Chabra S. Neoreviews. 2006; 7(8): e423

Of the following the MOST appropriate next management step for this neonate is to

A. call a surgical consult immediately


B. cover the exposed bowel with warm, saline-soaked gauze
C. perform endotracheal intubation and administer positive-pressure ventilation
D. wrap the exposed bowel with a sterile compression bandage

American Academy of Pediatrics 743


PREP® Self-Assessment PREPSA 2023

Correct Answer: B
The neonate in the vignette has gastroschisis, an abdominal wall defect lateral to the umbilicus
with exposed bowel. In most cases, this condition will be identified prenatally, and the delivery
room staff will be prepared ahead of time to manage the care of the neonate. In addition to
providing regular delivery room care, it is critically important to protect the bowel from injury
due to fluid loss by keeping it covered with warm, soaked gauze and putting the lower half of the
body (from the upper abdomen down) in a bowel bag.

A pediatric surgical consultation will be needed but not immediately in the delivery room.
Surgical treatment consists of either primary closure or a staged approach with silo placement to
protect the bowel while it is slowly introduced into the abdomen (usually performed when there
is no room in the underdeveloped abdominal cavity for the bowel or other extra-abdominal
organs) and subsequent closure. The neonate is breathing comfortably in room air, so there is no
need for endotracheal intubation and positive-pressure ventilation. Wrapping the bowel in a
compression bandage will cause further injury to the bowel.

The two most common abdominal wall defects in the newborn are gastroschisis and
omphalocele. Item C221 reviews the important differences between these diagnoses.

PREP Pearls
• In a neonate with gastroschisis, it is critically important in the delivery room to protect
the exposed bowel from further injury due to fluid loss.
• Surgical treatment of gastroschisis consists of either primary closure, or a staged
approach with silo placement to protect the bowel while it is slowly introduced into the
underdeveloped abdomen, and subsequent closure.
• Gastroschisis may be associated with intestinal atresia.

ABP Content Specifications(s)


• Plan the appropriate evaluation and management of a newborn infant who has abdominal-
intestinal wall defect

American Academy of Pediatrics 744


PREP® Self-Assessment PREPSA 2023

Suggested Readings
• Chabra S. Management of gastroschisis: prenatal, perinatal, and neonatal. Neoreviews.
2006;7(8):e419-e427. doi:10.1542/neo.7-8-e419.
• Kastenberg ZJ, Dutta S. Ventral abdominal wall defects. Neoreviews 2013;14(8):e402-
e411. doi:10.1542/neo.14-8-e402.
• Mansfield SA, Jancelewicz T. Ventral abdominal wall defects. Pediatr Rev.
2019;40(12):627-635. doi:10.1542/pir.2018-
• Slater BJ, Pimpalwar A. Abdominal wall defects. Neoreviews. 2020;21(6):e383-e391.
doi:10.1542/neo.21-6-e383.

American Academy of Pediatrics 745


PREP® Self-Assessment PREPSA 2023

Question 222
An 18-year-old young woman is brought to the emergency department by ambulance after being
“found down” at her grandparents’ home while visiting them during summer break. She was
found lying next to several empty liquor bottles. Since she was found, she has had no vomiting
and received no medications. On physical examination, the adolescent is minimally responsive to
painful stimuli. Vital signs are a temperature of 36.2°C, heart rate of 61 beats/min, respiratory
rate of 12 breaths/min, blood pressure of 110/60 mm Hg, and oxygen saturation of 98% in room
air. Her weight is 60 kg. Her pupils are equal, 3 mm, and reactive. The remainder of her physical
examination findings are unremarkable.

Of the following, the BEST next step in this adolescent’s management is


A. administration of charcoal by nasogastric tube
B. administration of naloxone intramuscularly
C. intubation for airway protection before performing urgent head computed tomography
D. obtaining rapid serum glucose testing and ethanol, acetaminophen, and salicylate levels

American Academy of Pediatrics 746


PREP® Self-Assessment PREPSA 2023

Correct Answer: D
Of the response choices, the best next step for the adolescent in the vignette is obtaining rapid
serum glucose testing and ethanol, acetaminophen, and salicylate levels. The most common
laboratory abnormality found in ethanol ingestion, and one requiring urgent treatment, is
hypoglycemia. It is important to obtain ethanol and possible coingestant levels rapidly to direct
further management.

The adolescent’s vital signs and clinical status do not meet the criteria for urgent intubation,
though head computed tomography may be necessary after initial laboratory testing. She does
not have pinpoint pupils, and her oxygenation level is adequate; therefore, administration of
naloxone is not the most appropriate next step. However, given that coingestion of alcohol and
opiates is potentially deadly, administering naloxone should be considered to reverse a possible
opioid overdose. Naloxone does not treat the effects of alcohol. Charcoal is not recommended in
this situation because of alcohol’s rapid gastrointestinal absorption and the risk of vomiting and
aspiration with ethanol ingestion.

Alcohol is one of the most common substances used by adolescents. It has been implicated in the
three leading causes of adolescent mortality: motor vehicle crashes, homicide, and suicide. Most
adolescents ingest alcohol through “binge drinking,” defined as 5 standard drinks for young men
or 4 standard drinks for young women ingested within a 2-hour period; this typically results in a
blood alcohol content equal to or greater than 0.08% (80 mg/dL), the legal definition of
intoxication.

The adolescent in the vignette had a blood alcohol content almost 5 times the legal limit. Her
clinical presentation was consistent with acute ethanol intoxication (Item C222): somnolence,
American Academy of Pediatrics 747
PREP® Self-Assessment PREPSA 2023

mild respiratory depression, and lownormal pulse and blood pressure. Her pupils were not
pinpoint, as would occur in opioid intoxication. However, it is vital to test for potential
coingestants in the medical evaluation.

Initiation of alcohol use early in adolescence is a risk factor for problematic alcohol use and
alcohol use disorder in adulthood. Screening, brief intervention, and referral to treatment for
alcohol and other drug use is recommended for adolescents in primary care settings.

PREP Pearls
• Alcohol or ethanol intoxication is managed with supportive care and includes rapid
evaluation for hypoglycemia, electrolyte derangements, and coingestant levels.
• Adolescent alcohol use increases the risk of alcohol use disorders as adults, and thus
requires early intervention and possible referral to treatment.

ABP Content Specifications(s)


• Identify the major physiologic consequences associated with alcohol use in adolescence
• Recognize the major behavioral consequences of alcohol use in adolescence

Suggested Readings
• Adger H, Saha S. Alcohol use disorders in adolescents. Pediatr Rev. 2013;34(3):103-114.
doi:10.1542/pir.34-3-103.
• American Academy of Pediatrics. Adolescent alcohol and substance use and abuse. In:
Tanski S, Garfunkel LC, Duncan
• PM, Weitzman M, eds. Performing Preventive Services: A Bright Futures Handbook.
American Academy of Pediatrics; 2010.
• Bagley S, Levy S. Substance use disorders. In: McInerny TK, Adam HM, Campbell DE,
DeWitt TG, Foy JM, Kamat DM, eds. American Academy of Pediatrics Textbook of
Pediatric Care. American Academy of Pediatrics; 2021:chap 336. Accessed September 1,
2022. Pediatric Care Online.
• Committee on Substance Use and Prevention. Substance use screening, brief
intervention, and referral to treatment. Pediatrics. 2016;138(1):e20161210.
doi:10.1542/peds.2016-1210.
• Hadland SE, Knight JR, Harris SK. Alcohol use disorder: a pediatric-onset condition
needing early detection and intervention. Pediatrics. 2019;143(3):e20183654.
doi:10.1542/peds.2018-3654.

American Academy of Pediatrics 748


PREP® Self-Assessment PREPSA 2023

Question 223
A previously healthy, 14-year-old adolescent girl is brought to the office for evaluation. She has
a 4-day history of worsening right ankle pain and a 2-day history of fever and limp. She is now
unable to bear weight. One week ago, she fell off her bicycle and sustained a minor bruise to her
right ankle. Her immunizations are up to date.

On physical examination, the adolescent is alert and ill appearing, with a temperature of 38.2°C,
heart rate of 106 beats/min, respiratory rate of 18 breaths/min, and blood pressure of 116/66 mm
Hg. There is tenderness to palpation over the superior and medial regions of her right ankle, but
no warmth, redness, or swelling. The remainder of her physical examination findings are normal.

Laboratory data are shown:


Laboratory Test Result
White blood cell count 19,000/µL (19.0 × 109/L)
Segmented neutrophils 84%
Lymphocytes 13%
Monocytes 3%
C-reactive protein 7.9 mg/dL (79 mg/L)

Of the following, the BEST next step in this adolescent’s evaluation is to order
A. a blood culture
B. computed tomography of the right ankle and distal tibia
C. serum procalcitonin level
D. ultrasonography of the right ankle

American Academy of Pediatrics 749


PREP® Self-Assessment PREPSA 2023

Correct Answer: A
The adolescent in the vignette has acute onset of fever, limp, right ankle bony tenderness, and an
elevated Creactive protein (CRP) level after an ankle injury. These signs and symptoms are
consistent with acute hematogenous osteomyelitis (AHO).

Osteomyelitis is defined as acute when it is diagnosed within 4 weeks of sign or symptom onset
in a previously uninfected bone. Osteomyelitis in children often results from hematogenous
spread of primary bacteremia to the bony metaphysis. Osteomyelitis can also result from direct
inoculation of bacteria (eg, due to trauma or surgical procedures) or contiguous spread from
adjacent infected soft tissue or joint fluid.

The 2021 Pediatric Infectious Disease Society and the Infectious Disease Society of America
clinical practice guidelines on the diagnosis and management of AHO recommend that initial
laboratory evaluation of children with suspected AHO should include a CRP level and blood
culture. The CRP level is elevated in up to 98% of children with AHO. A blood culture is
positive in approximately one-third of children with AHO, typically demonstrating bacterial
growth within 12 to 14 hours of incubation. Most children with AHO have a normal white blood
cell count, and there are insufficient data to recommend measurement of procalcitonin as part of
initial evaluation of AHO. Radiography of the affected bone can identify or exclude other
diagnoses (eg, fracture), but it will not show specific findings of AHO until at least 2 weeks after
infection. Magnetic resonance imaging (MRI) with contrast is the imaging modality of choice to
confirm the diagnosis of AHO. Magnetic resonance imaging is recommended over other imaging
modalities for evaluation of AHO because MRI can detect abnormalities within 24 to 48 hours of
infection, has better diagnostic accuracy, lacks ionizing radiation, detects complications (eg,
subperiosteal abscess, soft tissue abscesses, sinus tracts), and provides superior anatomic details
compared to plain radiographs, bone scans, ultrasonography, and computed tomography.

In children, the incidence of AHO is approximately 1.3 cases per 100,000 population annually.
The long bones, especially the femur, tibia, and humerus, are most commonly affected. Short or
non-tubular bones are affected in 10% to 25% of cases. Usually a single bone is affected;
multiple bones may be involved in 5% of cases. The clinical manifestations of AHO vary with
age. Neonates may have fever, swelling, and inflammation of the affected extremity, extreme
irritability when the affected extremity is palpated or moved, or pseudoparalysis. Infants may
present with high-grade fever, extremity swelling and redness, or refusal to bear weight or crawl.
Older children and adolescents present with fever, decreased motion of the affected extremity,
limp, and focal or point tenderness of the affected area.

The pathogen most commonly associated with AHO in every age group is Staphylococcus
aureus (including methicillin-resistant S aureus [MRSA]), followed by group A Streptococcus.
In infants and children aged <5 years, Kingella kingae is also an important pathogen. In
neonates, group B Streptococcus, S aureus, and gram-negative enteric bacilli (eg, Escherichia
coli, Salmonella spp) are important pathogens. Children with sickle cell disease are at risk for
osteomyelitis due to Salmonella spp.

American Academy of Pediatrics 750


PREP® Self-Assessment PREPSA 2023

In ill-appearing children with presumed AHO, antibiotic therapy targeted against S aureus should
be started empirically. In well-appearing children with AHO, antibiotic therapy may be withheld
until a diagnostic bone aspiration or biopsy is performed. Selection of an empiric anti-
staphylococcal agent should be guided by local antimicrobial susceptibility data; clindamycin or
vancomycin is indicated in regions with a high prevalence of MRSA (defined as >10% of all
community-acquired S aureus isolates). Cefazolin or oxacillin are reasonable treatment options
in geographic areas with low MRSA prevalence.

Response to antibiotic therapy is monitored by clinical assessment in conjunction with trending


CRP levels. Treatment of hospitalized children with suspected or proven AHO should be
initiated with parenteral antibiotic therapy, with transition to oral antibiotic therapy when they
have demonstrated a good clinical and laboratory response (eg, reduction in pain and swelling,
absence of fever, improvement in bone and joint function, decline in CRP level by 50% or 2-3
mg/dL [20-30 mg/L]). The prolonged use of intravenous antibiotic therapy in AHO is associated
with central venous catheter–associated complications (eg, thrombosis) leading to substantial
morbidity and high costs. A 3- to 4-week total duration of antibiotic therapy is recommended for
children with uncomplicated AHO. Imaging after the completion of therapy is not recommended
for children with uncomplicated AHO.

Surgical intervention is indicated for complicated AHO (eg, subperiosteal abscess, soft tissue
abscess). A longer duration of intravenous and total antibiotic therapy may be necessary for
children with clinical complications associated with AHO (eg, bacteremia for 3 or more days,
bone abscess requiring >1 surgical debridement procedure, venous thrombosis, physeal
involvement, pathologic fracture, or disseminated multi-organ infection (pneumonia,
pyomyositis, endocarditis). When children with complicated AHO are ready for discharge,
transition to outpatient parenteral antibiotic therapy is recommended.

PREP Pearls
• The initial laboratory evaluation of children with suspected acute hematogenous
osteomyelitis should include a Creactive protein level and blood culture. Magnetic
resonance imaging is the imaging modality of choice to confirm the diagnosis.
• The pathogen most commonly associated with acute hematogenous osteomyelitis in
every age group is Staphylococcus aureus (including methicillin-resistant S aureus);
group B Streptococcus and Kingella kingae are important pathogens in neonates and
young children, respectively.
• Transition to oral antibiotic therapy is recommended for hospitalized children with
suspected or proven acute hematogenous osteomyelitis who have demonstrated a good
clinical and laboratory response to initial intravenous
• antibiotic therapy.

American Academy of Pediatrics 751


PREP® Self-Assessment PREPSA 2023

ABP Content Specifications(s)


• Recognize the clinical findings associated with osteomyelitis in various anatomic
locations
• Plan the appropriate diagnostic evaluation of osteomyelitis, with attention to the sequence
with which positive findings become evident on imaging studies
• Plan the appropriate management of osteomyelitis in patients of various ages

Suggested Readings
• American Academy of Pediatrics. Staphylococcus aureus. In: Kimberlin DW, Barnett
ED, Lynfield R, Sawyer MH, eds. Red Book: 2021–2024 Report of the Committee on
Infectious Diseases. 32nd ed. American Academy of Pediatrics; 2021. Accessed
September 1, 2022. Red Book Online.
• Donaldson N, Sanders J, Child J, Parker S. Acute hematogenous bacterial osteoarticular
infections in children. Pediatr Rev. 2020;41(3):120-126. doi:10.1542/pir.2018-0201.
• Woods CR, Bradley JS, Chatterjee A, et al. Clinical practice guidelines by the Pediatric
Infectious Diseases Society and the Infectious Diseases Society of America: 2021
guidelines on diagnosis and management of acute hematogenous osteomyelitis in
pediatrics. J Pediatr Infect Dis Soc. 2021;10(8):801-844. doi:10.1093/jpids/piab027.

American Academy of Pediatrics 752


PREP® Self-Assessment PREPSA 2023

Question 224
A community-based study was conducted among women to determine the risk for cervical
intraepithelial neoplasia (CIN) and cancer. Women aged 18 to 45 years who had not received the
human papillomavirus (HPV) vaccine were included in the study. After informed consent, the
study participants completed a structured questionnaire on sexual behavior and reproductive
health. The women also underwent a gynecological examination and samples were collected for
cervical cytology and HPV DNA polymerase chain reaction (PCR).

Data were collected at enrollment and at yearly intervals for 10 years. Of the 16,239 enrolled
women, 149 women developed CIN3 or cancer during the 10-year follow-up period. Of the 149
women with CIN3 or cancer, 107 (71.8%) had evidence of cervical cytologic abnormalities (ie,
atypical squamous cells or worse) and/or a positive HPV test. Women with a negative
Papanicolaou test result and negative HPV DNA PCR assay results at baseline were found to
have a low risk for developing CIN3 and cancer during the first 48 months of follow-up.

Of the following, this study is BEST described as a


A. case-control study
B. cohort study
C. cross-sectional study
D. randomized controlled trial

American Academy of Pediatrics 753


PREP® Self-Assessment PREPSA 2023

Correct Answer: B
The study in the vignette is best described as a prospective cohort study. In this study, a cohort of
women was defined by the presence or absence of risk factors (cervical cytology abnormalities,
human papillomavirus DNA detected on polymerase chain reaction) and followed prospectively
over time for development of disease (cervical intraepithelial neoplasia or cancer).

Studies may be classified as descriptive or analytic. Descriptive studies describe general


characteristics and distribution of disease, especially in the context of person, place, and time.
Cross-sectional surveys, case reports, and case series are examples of descriptive studies. In
cross-sectional studies, data are collected regarding health status or condition and exposures at
one point in time. Cross-sectional studies can determine disease prevalence.

Possible risk factors for a disease may be compared between the group with disease and the
group without disease. The strengths of cross-sectional studies include their quick and easy
implementation and relatively low cost. The weaknesses of a cross-sectional study design are
potential recall bias in surveys and inability to confirm an etiologic association warranting the
need for an analytic study to determine causality.

In analytic studies, there is an explicit comparison between an exposure and a health outcome (or
disease). An investigator can determine whether or not risk of a health outcome (or disease) is
different between persons exposed or not exposed to a risk factor or intervention. Analytic
studies test hypotheses by using comparison groups.

Analytic studies may be categorized as:


• Observational studies (eg, cohort studies, case-control studies)
• Randomized clinical trials (eg, intervention studies)

Cohort studies allow for measurement of multiple data points over time and can be prospective
or retrospective.

Cohort studies group subjects by exposure(s) and measure relative risk (RR) of disease for the
given exposure. Prospective cohort studies follow cohorts of patients from initial exposure(s)
through development of health outcome or disease (or not). For retrospective cohort studies, data
on exposure(s) and presence or absence of health outcome/disease exist at the time of data
collection. Cohort studies can determine causality and monitor trends, and are suitable for
outbreak investigations in defined populations. Cohort studies are expensive, time-consuming,
and not suitable for studying rare diseases. The comparison group should be as similar as
possible to the exposed group with respect to known factors related to the disease, except for the
exposure of interest. Potential bias in cohort studies include loss to follow-up.

Case-control studies are retrospective studies comparing cases and matched controls. Cases are
persons who have already developed health outcomes or diseases. Controls are persons who do
not have the health outcome or disease. Controls should be comparable to the case population
using the same exclusions and restrictions as case selection. The differences in the frequency of
American Academy of Pediatrics 754
PREP® Self-Assessment PREPSA 2023

exposures is then compared in these groups retrospectively. Because case-control studies begin
with known cases, only an odds ratio (OR) for exposure may be calculated.

Compared to other study types, case-control studies are generally completed more rapidly, easier
to conduct, and are less expensive. Case-control studies are useful for the study of rare diseases,
new diseases, or diseases with long latency. Weaknesses of the case-control study design include
difficulty determining the timing of exposure, inability to determine relative risk, and potential
bias in the selection of cases and controls. Examples of biases include refusal to participate
(selection bias) during selection of controls or diseased persons being more likely to remember
an exposure than a healthy person (recall bias).

Randomized controlled trials (RCT) (eg, intervention studies) are useful to evaluate:
• Benefits or harm of an intervention
• New drugs or other comparable treatment options of disease

Randomization minimizes potential bias. Blinding also helps to ensure an unbiased assessment.
However, RCTs are expensive and complex to implement. In addition, selection of placebo in
RCTs may raise ethical issues. The relatively small sample and rigid enrollment criteria may
prevent inclusion of subjects most at risk to an adverse

PREP Pearls
• Cohort studies allow for measurement of multiple data points over time and can be
prospective or retrospective.
• Cohort studies can determine causality and monitor trends, and are suitable for outbreak
investigations in defined populations.
• Cohort studies are expensive, time-consuming, and not suitable for studying rare
diseases; potential bias in cohort studies includes nonresponse and losses to follow-up.

ABP Content Specifications(s)


• Understand the uses and limitations of cohort studies
• Understand the uses and limitations of case-control studies
• Understand the uses and limitations of case reports/series and anecdotal evidence

Suggested Readings
• Hartung DM, Touchette D. Overview of clinical research design. Am J Health Syst
Pharm. 2009;66(4):398-408. doi:10.2146/ajhp080300.
• Johnson SL. A question of time: Cross-sectional versus longitudinal study designs.
Pediatr Rev. 2010;31(6):250-251. doi:10.1542/pir.31-6-250.
• Palaia A. Research and statistics: study design and data sources. Pediatr Rev.
2013;34(8):371-372. doi:10.1542/pir.348-371.
• Perry-Parrish C, Dodge R. Research and statistics: validity hierarchy for study design and
study type. Pediatr Rev. 2010;31(1):27-9. doi:10.1542/pir.31-1-27.

American Academy of Pediatrics 755


PREP® Self-Assessment PREPSA 2023

Question 225
A 2-day-old neonate born at 35 weeks’ gestation is seen in the newborn nursery for a 12-hour
history of persistent yellow-stained emesis, intermittent irritability, and abdominal distention.
The neonate has hyperbilirubinemia, currently requiring phototherapy, and is exclusively
breastfed. There have been 2 meconium smears since birth. On physical examination, the
neonate is not in any acute distress and has normal vital signs for age. The abdomen is soft and
distended with a palpable upper abdominal loop. Digital rectal examination findings are normal
and results in passage of dark meconium.

Abdominal radiography (Item Q225A) reveals multiple dilated loops of bowel. A contrast
enema study (Item Q225B) reveals a transition to dilated colon at the splenic flexure with
multiple filling defects consistent with the presence of meconium.

Item Q225A: Abdominal radiograph for the neonate described in the vignette. Courtesy of J. Sullivan

Item Q225B: Contrast enema findings for the neonate described in the vignette. Courtesy of J. Sullivan

Of the following, the test MOST likely to reveal the cause of this neonate’s vomiting is
A. abdominal ultrasonography
B. lumbar puncture
C. rectal suction biopsy
D. upper gastrointestinal series

American Academy of Pediatrics 756


PREP® Self-Assessment PREPSA 2023

Correct Answer: C
The neonate in the vignette has persistent vomiting, abdominal distention with a palpable bowel
loop, and multiple dilated loops of bowel noted on radiography. This presentation is concerning
for an intestinal obstruction, even if the emesis is not bilious (green). The contrast enema
findings are consistent with Hirschsprung disease (HD); thus, a rectal suction biopsy is indicated
as the next diagnostic step, and of the response choices is the test most likely to reveal the
diagnosis. For this neonate, the biopsy demonstrated an absence of ganglion cells and confirmed
the diagnosis of HD.

Intestinal obstruction should be considered when a neonate has bilious emesis, a history of
maternal polyhydramnios, abdominal distention, and/or failure to pass meconium within 24 to 48
hours after birth. The majority (94% to 95%) of healthy term infants pass stool within 24 hours
after birth; all healthy term infants should pass stool within the first 48 hours after birth. It is
important to note that neonates with intestinal obstruction may not exhibit all of these signs.
Causes of neonatal intestinal obstruction include pyloric stenosis or small intestinal atresia,
intestinal malrotation/volvulus, meconium ileus, meconium plug syndrome, HD, and anorectal
abnormalities. For premature neonates, necrotizing enterocolitis should also be considered.
When an intestinal obstruction is suspected, initial management steps include a thorough
physical examination (including anorectal examination), fluid resuscitation as necessary, and
urgent pediatric surgical consultation.

Decompression of the bowel with a nasogastric tube may be indicated in neonates with distress
or severe abdominal distention. Plain abdominal radiography should be performed to assess for
proximal or distal bowel obstruction. Infants in distress in whom obstruction or perforation is
suspected may need urgent operative management without further imaging.

Radiographic findings in proximal small bowel obstruction can include the “double bubble” sign,
due to a dilated stomach and proximal small bowel with paucity of gas in the remainder of the
abdomen. Multiple dilated loops of small bowel or a dilated colon with a paucity of rectal air can
be seen with a distal bowel obstruction. Upper gastrointestinal fluoroscopy is indicated when a
proximal bowel obstruction is suspected in a clinically stable child. When the clinical scenario
suggests a distal bowel obstruction, a contrast enema is indicated. For some conditions, the
enema may be both diagnostic and therapeutic (eg, meconium plug syndrome). If HD is
suspected based on contrast enema findings, the diagnosis should be confirmed with rectal
suction biopsy.

The neonate in the vignette has findings that are most consistent with a distal intestinal
obstruction (multiple dilated loops, abnormal contrast enema), for which abdominal
ultrasonography and upper gastrointestinal series are not indicated. Although neonates with
meningitis can present with vomiting, this diagnosis would not explain the abnormal findings on
abdominal examination and radiographic studies; thus, a lumbar puncture is not indicated.

American Academy of Pediatrics 757


PREP® Self-Assessment PREPSA 2023

PREP Pearls
• Intestinal obstruction should be suspected in neonates with bilious (green) emesis, failure
to pass meconium by 24 to 48 hours after birth, abdominal distention, and/or history of
maternal polyhydramnios. The absence of 1 or more of these signs and symptoms does
not exclude neonatal intestinal obstruction.
• Initial management steps when a neonatal bowel obstruction is suspected include a
thorough physical examination, fluid resuscitation as needed, and urgent pediatric
surgical consultation.
• When intestinal obstruction is suspected, clinically unstable neonates may require urgent
operative management without definitive imaging results.

ABP Content Specifications(s)


• Plan the evaluation of the acute onset of vomiting as a result of obstruction in children of
various ages

Suggested Readings
• Burge DM. The management of bilious vomiting in the neonate. Early Hum Dev.
2016;102:41-45. doi:10.1016/j.earlhumdev.2016.09.002.
• Juang D, Snyder CL. Neonatal bowel obstruction. Surg Clin North Am. 2012;92(3):685-
711, ix-x. doi:10.1016/j.suc.2012.03.008.
• McCollough M, Sharieff GQ. Abdominal surgical emergencies in infants and young
children. Emerg Med Clin North Am. 2003;21(4):909-935. doi:10.1016/s0733-
8627(03)00090-7.
• Midura D, Statter MB. Surgical emergencies in the pediatric office. Pediatr Rev.
2022;43(3):148-159. doi:10.1542/pir.2020-004916.
• Mino J, Monteiro R, Stallion A. Gastrointestinal obstruction. In: McInerny TK, Adam
HM, Campbell DE, DeWitt TG, Foy JM, Kamat DM, eds. American Academy of
Pediatrics Textbook of Pediatric Care. American Academy of Pediatrics; 2021:chap 257.
Accessed September 1, 2022. Pediatric Care Online.
• Shields TM, Lightdale JR. Vomiting in children. Pediatr Rev. 2018;39(7):342-356.
doi:10.1542/pir.2017-0053.

American Academy of Pediatrics 758


PREP® Self-Assessment PREPSA 2023

Question 226
A 4-year-old girl is evaluated for a 2-day history of left eye redness and eyelid matting upon
awakening. She has occasional left eye itching. She has been otherwise healthy without fever or
rhinorrhea. The girl attends preschool and cannot return until cleared by a doctor.
On physical examination, there is left bulbar and tarsal conjunctival erythema with a small
amount of green discharge at the corner of the left eye. There is purulent fluid visible behind her
left tympanic membrane. The remainder of her physical examination findings are unremarkable.

Of the following, the MOST likely etiology of this girl’s findings is


A. adenovirus type 7
B. coxsackie virus A24
C. Haemophilus influenzae non-typeable
D. Moraxella catarrhalis

American Academy of Pediatrics 759


PREP® Self-Assessment PREPSA 2023

Correct Answer: C
The girl in the vignette has conjunctivitis-otitis syndrome. Non-typeable Haemophilus influenzae
is the most common pathogen associated with this syndrome and is therefore the most likely
etiology of this girl’s findings. The Haemophilus influenzae childhood vaccine protects against
Haemophilus influenzae type b; it does not provide protection against non-typeable Haemophilus
influenzae. Oral amoxicillin-clavulanate is the first-line treatment for conjunctivitis-otitis
syndrome.

Most childhood conjunctivitis has an infectious etiology; the remainder usually have an allergic
cause. There is conflicting evidence as to whether children are more likely to have bacterial or
viral causes of conjunctivitis; many sources report that bacteria may cause up to 80% of
infectious conjunctivitis in children.

Bacterial conjunctivitis is commonly bilateral with purulent eye discharge; there may be a
concurrent otitis media. The most common bacteria that cause conjunctivitis in children are
Staphylococcus epidermidis, Streptococcus pneumoniae, and Haemophilus influenzae;
Moraxella catarrhalis is an uncommon cause.

Viral conjunctivitis typically begins unilaterally and may spread to the other eye. The eye
discharge is watery or mucoid. There is often a concurrent viral infection. Adenovirus is the most
common viral etiology in children. Subtype 7 is associated with pharyngoconjunctival fever.
Coxsackie virus A24 is associated with an acute hemorrhagic conjunctivitis that is often
accompanied by fever and headache. Otitis-conjunctivitis syndrome generally does not have a
viral cause.

There are currently no rapid, reliable methods available to the primary care physician for
definitively distinguishing bacterial from viral etiologies of conjunctivitis. Health care
professionals frequently face pressure from parents, schools, and daycare settings to treat
affected children with antibiotics so that they may quickly return to school programs.
Conjunctivitis of either etiology resolves without treatment in the majority of cases. A 2012
Cochrane review found that topical antibiotic administration resulted in a slightly more rapid
resolution of uncomplicated bacterial conjunctivitis.

Prevention of the spread of conjunctivitis is most reliably managed by good hand hygiene.
Frequent hand washing and limiting the touching and rubbing of eyes should be encouraged.
Many school districts and daycare facilities exclude children with conjunctivitis until resolution
of the eye discharge or for 24 hours following the initiation of antibiotics. However, evidence
supports children with conjunctivitis being allowed to return to school sooner as long as they do
not have fever or other systemic symptoms and are able to participate in school activities.

American Academy of Pediatrics 760


PREP® Self-Assessment PREPSA 2023

PREP Pearls
• Non-typeable Haemophilus influenzae is the most common etiology of conjunctivitis-
otitis syndrome.
• There is conflicting evidence as to whether children are more likely to have bacterial or
viral causes of conjunctivitis; many sources report that bacteria may cause up to 80% of
infectious conjunctivitis in children.
• Both bacterial and viral conjunctivitis are self-limiting conditions. Good hand hygiene is
the most effective way to prevent spread.

MOCA-Peds Objective
• Evaluate and manage a patient with conjunctivitis.

ABP Content Specifications(s)


• Plan the appropriate management of conjunctivitis, including prevention of spread to
others

Suggested Readings
• Lavrich JB, Heersink S. Red eye/pink eye. In: McInerny TK, Adam HM, Campbell DE,
DeWitt TG, Foy JM, Kamat DM, eds. American Academy of Pediatrics Textbook of
Pediatric Care. American Academy of Pediatrics; 2021:chap 188. Accessed September 1,
2022. Pediatric Care Online.
• Richards A, Guzman-Cottrill JA. Conjunctivitis. Pediatr Rev. 2010;31(5):196-208.
doi:10.1542/pir.31-5-196.
• Sheikh A, Hurwitz B, van Schayck CP, McLean S, Nurmatov U. Antibiotics versus
placebo for acute bacterial conjunctivitis. Cochrane Database Syst Rev.
2012;(9):CD001211. doi:10.1002/14651858.CD001211.pub3.
• Shekhawat NS, Shtein RM, Blachley TS, Stein JD. Antibiotic prescription fills for acute
conjunctivitis among enrollees in a large United States managed care network.
Ophthalmology. 2017;124(8):1099-1107. doi:10.1016/j.ophtha.2017.04.034.

American Academy of Pediatrics 761


PREP® Self-Assessment PREPSA 2023

Question 227
A 3-year-old girl is being evaluated for an abnormal speech pattern. Her speech is hypernasal,
and air escapes from her nose when she talks. She has a history of frequent ear infections. On
physical examination, the girl has a long face, widely spaced eyes, and a bifid uvula with a ridge
at the end of the hard palate. The remainder of the girl’s physical examination findings are
normal.

Of the following, the BEST next step is to evaluate this girl for
A. cardiac abnormalities
B. gastroesophageal reflux
C. hearing loss
D. immunodeficiency

American Academy of Pediatrics 762


PREP® Self-Assessment PREPSA 2023

Correct Answers: A
The girl in the vignette has a bifid uvula and clinical findings suggestive of a submucosal cleft
palate (SMCP), including hyper-nasal speech, air escape from her nose, and a palatal ridge. The
additional findings of hypertelorism and long face suggest an underlying genetic syndrome such
as Loeys-Dietz or DiGeorge syndrome. DiGeorge syndrome is less likely because there is no
history of neonatal hypocalcemia or infections due to T-cell defects.

The girl in the vignette most likely has Loeys-Dietz syndrome (LDS). Loeys-Dietz syndrome is
an autosomal dominant condition with characteristic vascular findings (aortic aneurysms, arterial
tortuosity), craniofacial findings (hypertelorism, bifid uvula, cleft palate, craniosynostosis), and
musculoskeletal findings (pectus excavatum or carinatum, arachnodactyly, joint laxity). The
diagnosis of LDS is made when the characteristic clinical features are present and/or genetic
testing. Loeys-Dietz syndrome is associated with pathologic variants in the TGFBR1, TGFBR2,
SMAD3, and TGFB2 genes. Individuals with suspected LDS should be screened for aortic root
dilatation and monitored for progression and development of aortic dissection. Contact sports
and isometric exercises should be restricted due to the risk of increasing systemic vascular
resistance.

A bifid uvula can be an isolated finding or associated with a SMCP. Bifid uvula and SMCP are
associated with several genetic syndromes with clinically significant cardiac anomalies (Item
C227). Therefore, children with a bifid uvula should undergo a comprehensive cardiac
evaluation whether or not there are abnormal cardiovascular findings on physical examination.

List of associated syndromes/malformations


• Down syndrome
• Pierre-Robin-Sequence
• Ulrich-Turner-syndrome
• Roberts-syndrome
• Klippel-Feil-syndrome
• Cornelia de Lange syndrome
• Di George syndrome
• Unknown syndrome

An SMCP is the result of a fusion defect of the soft palate muscles that occurs during
embryogenesis. It can be difficult to detect on physical examination. Findings suggestive of a
SMCP include a bifid uvula, a translucent line in the middle of the hard palate, and a palpable
notch at the end of the hard palate. Due to incomplete palate closure, affected children
commonly present with nasal regurgitation during feedings, recurrent otitis media due to
eustachian tube dysfunction, and velopharyngeal insufficiency (VPI). Velopharyngeal
insufficiency is characterized by hypernasal speech. It is important to recognize bifid uvula or
SMCP prior to adenoidectomy as affected individuals are at significantly increased risk for VPI,
a known postoperative complication of the procedure.

American Academy of Pediatrics 763


PREP® Self-Assessment PREPSA 2023

Gastroesophageal reflux secondary to hypotonia, and hearing loss due to chronic middle ear
effusions, may occur in children with bifid uvula or SMCP; evaluation may be indicated, but
evaluation for cardiac abnormalities takes precedence. Immunodeficiency and SMCP are
associated with DiGeorge syndrome, but the girl in the vignette does not have a history of
neonatal hypocalcemia and infections typically seen with characteristic T-cell defects.

PREP Pearls
• Bifid uvula may be associated with a submucosal cleft palate.
• Bifid uvula is associated with several genetic syndromes; all children with a bifid uvula
should be evaluated for cardiac anomalies.
• Chronic recurrent ear infections, nasal regurgitation, and velopharyngeal insufficiency
are commonly seen in children with submucosal cleft palate.

ABP Content Specifications(s)


• Identify conditions associated with a bifid uvula

Suggested Readings
• Abbott MA. Cleft lip and palate. Pediatr Rev. 2014;35(5):177-181. doi:10.1542/pir.35-5-
177.
• Baum ED. Tonsillectomy and adenoidectomy and myringotomy with tube insertion.
Pediatr Rev. 2010;31(10):417-425; quiz 426. doi:10.1542/pir.31-10-417.
• Kim AY, Bodurtha JN. Dysmorphology. Pediatr Rev. 2019;40(12):609-618.
doi:10.1542/pir.2018-0331.
• Loeys BL, Dietz HC. Loeys-Dietz syndrome. GeneReviews [Internet]. University of
Washington; 2018. Accessed September 1, 2022.
https://www.ncbi.nlm.nih.gov/books/NBK1133/.
• Reiter R, Brosch S, Wefel H, Schlömer G, Haase S. The submucous cleft palate:
diagnosis and therapy. Int J Pediatr Otorhinolaryngol. 2011;75(1):85-88.
doi:10.1016/j.ijporl.2010.10.015.

American Academy of Pediatrics 764


PREP® Self-Assessment PREPSA 2023

Question 228
A 17-year-old, previously healthy adolescent boy is brought to the emergency department with
lower extremity weakness. He was walking into his house after checking the mailbox when he
experienced the onset of bilateral lower extremity weakness, which he describes as a heaviness.
His weakness progressively worsened over several hours to the point that he could not stand up
from the dinner table. The boy recalls a self-limited episode of blurred vision 6 months ago, but
no other episodes of focal neurological deficits.

On physical examination, the adolescent’s vital signs are normal. His mental status and cranial
nerve examination findings are normal. Motor examination reveals normal upper extremity
strength, tone, and reflexes; lower extremities are flaccid with 0/5 strength, absent reflexes, and
hypotonia. His toes are upgoing to plantar stimulation. He has a sensory level at his mid-thorax,
below which he is unable to reliably feel light touch, pin prick, temperature, or vibration. He has
normal finger to nose testing but cannot perform heel-to-shin testing due to weakness. He is
unable to ambulate. The remainder of his physical examination findings are normal.

Of the following, the BEST next step in this adolescent’s management is


A. electromyography and nerve conduction studies
B. lumbar puncture with cerebrospinal fluid analysis
C. spine computed tomography
D. spine magnetic resonance imaging

American Academy of Pediatrics 765


PREP® Self-Assessment PREPSA 2023

Correct Answer: D
The adolescent in the vignette is experiencing an acute myelopathy, a neurological emergency
requiring urgent neuroimaging to evaluate for spinal cord compression or inflammation.
Magnetic resonance imaging (MRI) of the spine with and without contrast is the best test to
evaluate the spinal cord and associated structures. The presence of a compressive lesion requires
neurosurgical intervention.

Causes of acute and subacute myelopathies are shown in Item C228.

Recognition of acute spinal cord dysfunction is critical to timely diagnosis and treatment.

Common symptoms and signs of an acute myelopathy include:


• Bilateral motor and/or sensory symptoms
• Bladder and/or bowel dysfunction
• Neck and back pain

In the acute phase, children and adolescents with myelopathy can present with a flaccid paralysis
and areflexia below the level of the lesion, leading to diagnostic confusion due to the absence of
upper motor neuron signs. Careful history and physical examination can aid in determining the
appropriate evaluation to distinguish a peripheral nervous system condition, such as Guillain-
Barré syndrome, from myelopathy, and guide the management plan.

In the setting of acute-subacute myelopathy, MRI of the spine is the most precise diagnostic test.
Computed tomography of the spine is helpful to look at bony structures but does not provide
sufficient information about the cord and soft tissues. Lumbar puncture is a useful ancillary test,
in particular in inflammatory disorders, and is often part of a tiered evaluation; however, it
should not be prioritized over imaging in a patient with acute spinal cord dysfunction.
Electromyography and nerve conduction studies can be useful in prognostication and monitoring
progression of peripheral nervous system disorder. They provide ancillary data in the subacute
phase but are often normal in the acute phase.

Management of myelopathy is aimed at the underlying cause. In early inflammatory myelitis,


MRI of the spine can be normal; empiric treatment should be initiated if there is a high index of
suspicion and alternative etiologies have been excluded. To prevent complications, monitoring
and supportive care should include:
• Frequent bladder scans with intermittent catheterization
• Monitoring for the development of autonomic dysfunction
• Prevention of deep venous thrombosis
• Prevention of decubitus ulcer formation Psychological support

American Academy of Pediatrics 766


PREP® Self-Assessment PREPSA 2023

PREP Pearls
• Acute spinal cord dysfunction is a neurological emergency requiring careful history,
physical examination, and urgent neuroimaging to evaluate for cord compression.
Magnetic resonance imaging of the spine with and without contrast is the best test to
evaluate the spinal cord and associated structures.
• Common symptoms and signs of an acute myelopathy include: bilateral motor and/or
sensory symptoms, bladder and/or bowel dysfunction, and neck/back pain.
• In the acute phase, children and adolescents with myelopathy can present with a flaccid
paralysis and areflexia below the level of the lesion; this may lead to diagnostic confusion
due to the absence of upper motor neuron signs.

ABP Content Specifications(s)


• Plan the initial neurodiagnostic evaluation of acute spinal cord dysfunction

Suggested Readings
• Absoud?M, Greenberg?BM, Lim?M, Lotze?T, Thomas?T, Deiva?K. Pediatric transverse
myelitis. Neurology. 2016;87(9 suppl 2):S46-S52.
doi:10.1212/WNL.0000000000002820.
• Huh?Y, Park?EJ, Jung?JW, Oh?S, Choi?SC. Clinical insights for early detection of acute
transverse myelitis in the emergency department. Clin Exp Emerg Med. 2015;2(1):44-50.
doi:10.15441/ceem.14.034.
• Lu V, Niazi T. Pediatric spinal cord diseases. Pediatr Rev. 2021 42(9):486-499.
doi:10.1542/pir.2020-000661.

American Academy of Pediatrics 767


PREP® Self-Assessment PREPSA 2023

Question 229
A 12-year-old boy is brought to the emergency department after being bitten by a snake while
hunting in Florida. Photos show a dark brown snake with diamond-shaped marks on its back
(Item Q229). The boy is ill-appearing and pale. His vital signs are a temperature of 36.7°C, heart
rate of 140 beats/min, respiratory rate of 18 breaths/min, and blood pressure of 102/65 mm Hg.
Physical examination reveals 2 small puncture wounds on his lower calf, which is swollen, red,
ecchymotic, and extremely tender to palpation. The erythema is extending up his calf toward his
knee. Shortly after arrival, the boy begins to complain of nausea and has 1 episode of emesis.

Of the following, the BEST next step in the management of this boy is to
A. administer crotalidae polyvalent immune fab
B. apply ice to the puncture wounds
C. apply a tourniquet to the upper thigh to prevent the spread of venom
D. place the leg in a splint and elevate the leg above the level of the patient’s heart

American Academy of Pediatrics 768


PREP® Self-Assessment PREPSA 2023

Correct Answer: A
The boy in the vignette is displaying signs and symptoms of severe envenomation from a snake
bite. Item C229A shows an eastern diamondback rattlesnake. Because the boy is displaying
systemic symptoms as well as evidence of a severe local reaction, crotalidae polyvalent immune
fab (antivenom) should be administered immediately. Applying a tourniquet or ice to the wound
are not recommended treatments. It is appropriate to splint an affected limb, but it should be
positioned below the level of the heart, not above. The recommended first aid for snakebites is
outlined in Item C229B.

There are 2 families of poisonous snakes in the United States, Crotalidae (pit vipers) and
Elapidae. Members of the Crotalidae family include rattlesnakes, cottonmouths, water
moccasins, and copperheads. Members of the Elapidae family include coral snakes and cobras.
Over 90% of poisonous snake bites in the US are from Crotalidae. Item C229C lists the
differences between snakes in the Crotalidae and Elapidae families.

First Aid for Snakebites (Pre-Hospital care)


1. Don’t waste significant time trying to identify the snake. If possible and you are able to keep
yourself and others safe and out of the snake’s striking range, take a photo of the snake to
capture approximate size and characteristics.
2. Move the patient as little as possible.
3. Mark the victim’s skin with an indelible pen to indicate the area of swelling and the time. Repeat
this every 15 minutes.
5. Remove rings, watches, and constrictive clothing.
4. Immobilize the affected limb by splinting as if for a fracture, and keep the limb below the level of
the heart if possible.
5. Regardless of early symptoms, transport the victim to the nearest medical facility at a safe speed.
6. Avoid the use of ice (tissue damage), tourniquet/pressure bandages, aspirin (anticoagulation),
alcohol or sedative drugs (vasodilation), or stimulants such as caffeine (acceleration of venom
absorption).
7. Do not use any extraction devices or have anyone attempt to suck out the venom. This will
prevent further damage to tissue as well as preventing a second exposure (caregiver) to the
venom.
8. As soon as possible, start basic life support, including volume expansion and Trendelenburg
position for patients with hypotension

Signs and symptoms of a crotaline envenomation typically develop within 2 to 6 hours. These
include severe pain, redness, swelling, and ecchymosis near the envenomation site (local tissue
toxicity) and may progress to systemic symptoms including nausea, vomiting, weakness, metallic
taste in mouth, coagulation abnormalities, tachycardia, hypotension, bleeding gums,
hematemesis, hematuria, bloody stools, or cardiovascular collapse.

Envenomation from snakes from the Elapidae family is neurotoxic. Signs and symptoms may be
delayed up to several hours. There is little to no tissue reaction at the envenomation site, but
patients may experience paresthesia or weakness around the site. Progressive neurologic
symptoms may develop and can include tremors, lethargy, increased salivation, cranial nerve

American Academy of Pediatrics 769


PREP® Self-Assessment PREPSA 2023

palsies, vision changes, ptosis, difficulty speaking or swallowing, difficulty breathing, or even
respiratory paralysis.

Initial first aid should be administered to children who sustain a snake bite. This includes
keeping the child calm with as little movement or agitation as possible to prevent further
dissemination of the venom. The affected extremity should be splinted in a position of comfort
and kept below the level of the heart; the envenomation site should be marked with a pen to
allow tracking of local tissue changes. The wound should be gently cleaned with soap and water.
Application of a tourniquet to reduce arterial supply to the limb is not recommended. Application
of ice, the use of suction to remove venom, or incision of the envenomation site have not been
demonstrated to have any beneficial effect. Children should be immediately transported to the
nearest medical facility.

The boy in the vignette has sustained an envenomation from a rattlesnake and is displaying signs
of severe local tissue damage as well as systemic toxicity. Commercially available antivenom
should be administered immediately. Providing antivenom is the most important life-saving step
for this boy. Pit viper envenomation can cause bleeding, platelet dysfunction, and disseminated
intravascular coagulation in addition to the local tissue damage of the envenomation site. A
complete blood cell count, partial thromboplastin time, prothrombin time, international
normalized ratio, fibrinogen level, and D-dimer level should be obtained initially and repeated
every 6 hours to monitor for hematoxicity.

Crotalidae polyvalent immune fab is the standard of treatment for children displaying severe
local reactions, systemic symptoms, or rapid progression of symptoms. The initial dosing for
children is 4 to 6 vials. The affected child requires close frequent monitoring every hour for signs
and symptoms and should be given 4 to 6 additional vials of antivenom each hour until there is
slowing or cessation of envenomation effects as evidenced by clinical and laboratory evaluation.
After control of signs and symptoms has been established, the patient may require up to 3
maintenance doses of antivenom. Antivenom administration can cause anaphylactic or serum
sickness reactions; the child must be monitored closely for evidence of a reaction.

Children with pit viper envenomation with minimal or no evidence of progressive symptoms
should not be given antivenom but should be carefully monitored in an inpatient setting.
Prophylactic antibiotics are not indicated in the treatment of snake bites.

It should be noted that because of the relatively higher ratio of venom to body mass in children,
small children are at increased risk for toxic effects from snake envenomation.

American Academy of Pediatrics 770


PREP® Self-Assessment PREPSA 2023

PREP Pearls
• Crotalidae polyvalent immune fab is the standard of treatment for children with pit viper
snake bite displaying severe local reactions, systemic symptoms, or rapid progression of
symptoms. Initial dosing is 4 to 6 vials.
• The 2 families of poisonous snakes in the United States are Crotalidae (pit vipers) and
Elapidae (eg, coral snakes and cobras).
• Crotalidae envenomations cause severe local tissue damage and can progress to systemic
hematotoxicity (eg, nausea, vomiting, hypotension, coagulation abnormalities); Elapidae
envenomations are neurotoxic with little to no local tissue damage.

ABP Content Specifications(s)


• Plan the appropriate management of a snake bite

Suggested Readings
• Banerji S, Bronstein AC. Envenomations. In: McInerny TK, Adam HM, Campbell DE,
DeWitt TG, Foy JM, Kamat DM, eds. American Academy of Pediatrics Textbook of
Pediatric Care. American Academy of Pediatrics; 2021:chap 358. Accessed September 1,
2022. Pediatric Care Online.
• Bond GR. Snake, spider, and scorpion envenomation in North America. Pediatr Rev.
1999;20(5):147-151. doi:10.1542/pir.20-5-147.

American Academy of Pediatrics 771


PREP® Self-Assessment PREPSA 2023

Question 230
A 10-year-old boy is seen in the office for 2 days of left ear pain. He has a history of mild
intermittent asthma but has not needed albuterol in over 1 year. He has no upper airway
symptoms, cough, or fever. The boy was camping last week near a lake where he engaged in
swimming and water sports. On physical examination, the boy has pain with traction or
movement of his left outer ear. Findings on otoscopic examination of the left ear are shown in
Item Q230. The remainder of his physical examination findings are normal.

Of the following, the BEST treatment for this boy is


A. oral analgesics and ear canal barrier protection
B. topical acetic acid 2% solution
C. topical antifungal drops
D. topical combined antibiotic and steroid drops

American Academy of Pediatrics 772


PREP® Self-Assessment PREPSA 2023

Correct Answers: D
The boy in the vignette has no symptoms beyond ear pain, had a recent experience with
swimming and water sports, and has otoscopic examination findings (Item C230) typical for
otitis externa (external canal edema and debris); this scenario is consistent with a diagnosis of
otitis externa. Swimmers have a 5 times increased incidence of otitis externa. The differential
diagnosis of an ear canal with this appearance includes otitis media with tympanic membrane
rupture, skin conditions such as atopic dermatitis or psoriasis, and a missed foreign body in the
canal causing inflammation.

Otitis externa is caused by bacteria 90% of the time, most commonly Pseudomonas aeruginosa
and Staphylococcus aureus. A 2010 Cochrane review showed that compared to topical
antibiotics alone, combined topical antibiotics and steroids had greater efficacy, and acetic acid
solution had less efficacy in the treatment of otitis externa. Thus, the best treatment for the boy in
the vignette is topical combined antibiotic and steroid drops. However, because the evidence
distinguishing efficacy of these treatments is limited, treatment choice should take into
consideration cost, potential for side effects, and ease of compliance. Ototoxic preparations (eg,
aminoglycosides) should not be instilled in the ear canal without first visualizing an intact
tympanic membrane.

It may be necessary to remove debris from the ear canal for topical treatments to be effective.
This cleansing, if needed, should be performed by an otolaryngologist with the appropriate tools
and ability to visualize the canal clearly to avoid injury.

Fungal otitis externa can occur but is much less common than bacterial infection. Thus, topical
antifungal drops are not an appropriate first-line treatment. Oral analgesics and ear canal barrier
protection alone would not be an appropriate treatment for otitis externa.

PREP Pearls
• Otitis externa often presents with isolated ear pain after a recent exposure to water (eg,
swimming, water sports).
• Topical combined antibiotic and steroid drops is the first-line treatment of otitis externa.
• Ototoxic preparations (eg, aminoglycosides) should not be instilled in the ear canal
without first visualizing an intact tympanic membrane.

ABP Content Specifications(s)


• Recognize pathogens commonly associated with otitis externa
• Recognize the clinical findings associated with foreign body in the external ear canal
• Plan the appropriate initial and prophylactic management of otitis externa Formulate a
differential diagnosis of otitis externa

American Academy of Pediatrics 773


PREP® Self-Assessment PREPSA 2023

Suggested Readings
• Kaushik V, Malik T, Saeed SR. Interventions for acute otitis externa. Cochrane Database
Syst Rev. 2010;(1):CD004740. doi:10.1002/14651858.CD004740.pub2.
• Long M. Otitis externa. Pediatr Rev. 2013;34(3):143-144. doi:10.1542/pir.34-3-143.
• Tan TQ. Otitis media and otitis externa. In: McInerny TK, Adam HM, Campbell DE,
DeWitt TG, Foy JM, Kamat DM, eds. American Academy of Pediatrics Textbook of
Pediatric Care. American Academy of Pediatrics; 2021:chap 305. Accessed September 1,
2022. Pediatric Care Online.
• Wiegand S, Berner R, Schneider A, Lundershausen E, Dietz A. Otitis externa. Dtsch
Arztebl Int. 2019;116(13):224-234. doi:10.3238/arztebl.2019.0224.

American Academy of Pediatrics 774


PREP® Self-Assessment PREPSA 2023

Question 231
A 3-year-old boy recently adopted from India is seen for profuse watery diarrhea. The boy has
HIV infection. His recent laboratory testing showed an HIV viral load of 250,000 copies/mL and
an absolute CD4 cell count of 200/µL. He is not receiving antiretroviral therapy. The diarrhea
started 3 days after the family visited a recreational waterpark. On physical examination, the
boy's temperature is 38.5°C, heart rate is 90 beats/min, respiratory rate is 22 breaths/min, blood
pressure is 100/65 mm Hg, and oxygen saturation is 99% in room air. Stool microscopy reveals
acid-fast oocysts.

Of the following, the BEST next step in this boy’s management is to administer
A. albendazole
B. antiretroviral therapy
C. metronidazole
D. nitazoxanide

American Academy of Pediatrics 775


PREP® Self-Assessment PREPSA 2023

Correct Answer: B
The child in the vignette has a low CD4 cell count because of untreated HIV infection. The
presence of acid-fast oocysts in the stool is consistent with cryptosporidiosis. Diarrheal illness
from Cryptosporidium species is associated with recreational water parks and contaminated
drinking water. In a child with HIV infection, prompt initiation of antiretroviral therapy (eg,
dolutegravir, emtricitabine, and tenofovir combination) to improve the CD4 cell count results in
resolution of diarrheal symptoms. Treatment is generally not necessary in otherwise healthy
children; however, treatment with oral nitazoxanide is considered in children without HIV
infection for severe symptoms (>10 loose stools per 24 hours) or persistent symptoms (>2
weeks).

Albendazole is an antiparasitic agent with activity against select protozoa and intestinal and
extraintestinal roundworms; it is not effective against Cryptosporidium. Metronidazole, a
nitroimidazole, treats anaerobic infections and giardiasis but is not active against
Cryptosporidium.

Cryptosporidium is an intracellular protozoan. Most human infections are caused by


Cryptosporidium parvum and Cryptosporidium hominis species. Although infection occurs in all
age groups, the incidence is highest in children 1 to 4 years of age. Infection occurs after
ingestion of spores. The incubation period is 2 to 10 days. Clinical manifestations include
profuse watery diarrhea, occasionally associated with mucus and rarely with blood. Almost half
of the children will have abdominal cramps, nausea, and vomiting. Fevers may or may not be
present. In immunocompetent children, the infection is asymptomatic and self-limiting (resolving
in 2-3 weeks). In children with immune deficiencies, malnutrition, solid organ or stem cell
transplantation, or untreated HIV infection, a prolonged and debilitating condition can occur.
Biliary tract disease is frequently reported in immunocompromised children with fevers, right
upper quadrant pain, jaundice, and diarrhea.

Methods of diagnostic testing for Cryptosporidium include stool microscopy, enzyme


immunoassay, and polymerase chain reaction (PCR) test. Cryptosporidium oocysts can be
identified using a modified Ziehl-Nelson stain (acid-fast) or an immunofluorescence assay. The
routine stool for ova and parasites test does not include Cryptosporidium. Therefore, when the
diagnosis is suspected, specific testing for Cryptosporidium should be requested. At least 3 stool
samples should be collected on separate days because shedding can be intermittent.
Cryptosporidium testing is included in the multiplex gastrointestinal PCR panel.

PREP Pearls
• Cryptosporidium is an intracellular parasite responsible for diarrheal illness.
• Most Cryptosporidium infections are asymptomatic and self-limiting in
immunocompetent children. Severe or protracted illness is seen in children with untreated
HIV, solid or stem cell transplantation, or immunodeficiency.
• In a child with HIV infection, prompt initiation of antiretroviral therapy can lead to
resolution of diarrhea due to Cryptosporidium.

American Academy of Pediatrics 776


PREP® Self-Assessment PREPSA 2023

ABP Content Specifications(s)


• Understand the epidemiology of Cryptosporidium infection
• Recognize the clinical features associated with Cryptosporidium infection, including
diarrhea in an immunocompromised host

Suggested Readings
• American Academy of Pediatrics. Cryptosporidiosis. In: Kimberlin DW, Brady MT,
Jackson MA, Long SS, eds. Red Book: 2018-2021 Report of the Committee on Infectious
Diseases. 31st ed. Itasca, IL: American Academy of Pediatrics; 2018:288-290. Red Book
Online .
• Flynn PM. Cryptosporidium species. In: Long SS, ed. Principles and Practice of Pediatric
Infectious Diseases. 4th ed. Amsterdam, the Netherlands: Elsevier; 2012:1269-1271.e2.
https://doi.org/10.1016/B978-1-4377-2702-9.00263-4.
• Kalyoussef S, Goldman D, Adam HM. Giardiasis and cryptosporidiosis. Pediatr Rev.
2010;31(2):81–82. doi:10.1542/pir.31-2-81.
• Ochoa TJ, White AC Jr. Cryptosporidiosis. In: Feigin and Cherry’s Textbook of Pediatric
Infectious Diseases. 7th ed. Philadelphia, PA: Saunders; 2014:2139-2144.
• Ulshen MH. Diarrhea and steatorrhea. In: McInerny TK, Adam HM, Campbell DE,
DeWitt TG, Foy JM, Kamat DM, eds. American Academy of Pediatrics Textbook of
Pediatric Care. 2nd ed. Itasca, IL: American Academy of Pediatrics; 2017:1267-1281.
Pediatric Care Online.

American Academy of Pediatrics 777


PREP® Self-Assessment PREPSA 2023

Question 232
A term female neonate is brought to the clinic for a health supervision visit. Her parents report
that their 4-month-old son died from sudden unexplained infant death syndrome 2 years ago.
They ask how best to prevent a similar event happening in their daughter.

Of the following, the BEST recommendation for this neonate’s parents is to


A. avoid feeding her with a bottle
B. avoid smoke exposure
C. place her to sleep in the side position
D. use home apnea and bradycardia monitoring

American Academy of Pediatrics 778


PREP® Self-Assessment PREPSA 2023

Correct Answer: B
Of the response choices, the best recommendation for the parents in the vignette to reduce the
risk of sudden infant death syndrome (SIDS) is to avoid smoke exposure. Smoke exposure in an
infant’s environment is a major risk factor for SIDS. Although not one of the response choices,
placing the neonate to sleep supine is a major preventative measure. The prone or side sleep
positions are associated with an increased risk of SIDS. Parents often perceive the supine
position for infant sleep as conferring an increased risk of choking or aspiration, but there is no
evidence to support this concern.

Bottle feeding has not been associated with an increased risk of SIDS, although breast milk
without the introduction of formula provides enhanced protection. Home apnea and bradycardia
monitoring has not been associated with a reduction in the risk of SIDS, even for families with a
previous child who died from SIDS. Its use is limited to premature infants with a history of
apnea of prematurity and infants with central apnea, cardiac arrhythmia, or other identifiable risk
of requiring cardiopulmonary resuscitation.

According to the American Academy of Pediatrics 2016 Technical Report, SIDS and Other
Sleep-Related Infant Deaths: Evidence Base for 2016 Updated Recommendations for a Safe
Infant Sleeping Environment, “Sudden unexpected infant death (SUID), also known as sudden
unexpected death in infancy (SUDI), is a term used to describe any sudden and unexpected
death, whether explained or unexplained (including sudden infant death syndrome [SIDS] and
ill-defined deaths), occurring during infancy. After case investigation, SUID can be attributed to
causes of death such as suffocation, asphyxia, entrapment, infection, ingestions, metabolic
diseases, and trauma (unintentional or nonaccidental). SIDS is a subcategory of SUID and is a
cause assigned to infant deaths that cannot be explained after a thorough case investigation
including autopsy, a scene investigation, and review of clinical history.”

Sudden infant death syndrome rates have declined by more than 50% since the early 1990s, in
large part due to the “Back to Sleep” campaign, now known as “Safe to Sleep,” through which
infants were first recommended to sleep supine. However, SIDS remains the leading cause of
death for infants under 1 year of age. Pediatricians should recommend that infants be placed to
sleep supine on a firm surface with only a fitted sheet without bumpers, pillows, soft or loose
bedding, or stuffed animals. Infants should not sleep in swings, slings, strollers, or infant carriers,
or on couches or armchairs. While bed-sharing may increase breastfeeding duration, the risk of
SIDS associated with bed-sharing outweighs the benefits, and infants should be returned to their
crib to sleep. Pacifier use may reduce the risk of SIDS.

American Academy of Pediatrics 779


PREP® Self-Assessment PREPSA 2023

PREP Pearls
• Smoke exposure in an infant’s environment is a major risk factor for sudden infant death
syndrome.
• Infants should be placed to sleep supine on a firm surface with only a fitted sheet without
bumpers, pillows, or soft or loose bedding to decrease the risk of sudden infant death
syndrome.
• Breast milk without the introduction of formula provides enhanced protection against
sudden infant death syndrome.

ABP Content Specifications(s)


• Counsel parents regarding prevention of sudden infant death syndrome
• Identify risk factors associated with sudden infant death syndrome

Suggested Readings
• Behnam-Terneus M, Clemente M. SIDS, BRUE, and safe sleep guidelines. Pediatr Rev.
2019;40(9):443-455. doi:10.1542/pir.2017-0259.
• Ellis C, Pease A, Garstang J, Watson D, Blair PS, Fleming PJ. Interventions to improve
safer sleep practices in families with children considered to be at increased risk for
sudden unexpected death in infancy: a systematic review. Front Pediatr. 2022;9:778186.
doi:10.3389/fped.2021.778186.
• Moon RY, Task Force on Sudden Infant Death Syndrome. SIDS and other sleep-related
infant deaths: evidence base for 2016 Updated Recommendations for a Safe Infant
Sleeping Environment. Pediatrics. 2016;138(5):e20162940. doi:10.1542/peds.2016-2940.
• Shapiro-Mendoza CK, Palusci VJ, Hoffman B, et al; AAP Task Force on Sudden Infant
Death Syndrome, Council on Child Abuse and Neglect, Council on Injury, Violence, and
Poison Prevention, Section on Child Death Review and Prevention, National Association
of Medical Examiners. Half century since SIDS: a reappraisal of terminology. Pediatrics.
2021;148(4):e2021053746. doi:10.1542/peds.2021-053746.

American Academy of Pediatrics 780


PREP® Self-Assessment PREPSA 2023

Question 233
A 15-year-old adolescent girl is seen in the clinic for follow-up of anemia. She was initially
evaluated 1 month ago for a 6-month history of fatigue. At that visit, her hemoglobin level was
9.0 g/dL (90 g/L), and she was prescribed a daily iron supplement. The adolescent often skips
breakfast and goes with her friends to the corner store for “lunch,” which usually consists of
chips and iced tea. She eats a home-cooked dinner with her parents every evening. She has
regular menstrual periods which she describes as moderately heavy, requiring more than 4 pads
during the day and lasting 7 days. The adolescent’s physical examination findings are normal
except for pallor of her palpebral conjunctiva. A repeat hemoglobin level today is 9.2 g/dL (92
g/L).

Of the following, the BEST next step in this adolescent’s management is to


A. assist her with strategies to improve medication adherence
B. obtain laboratory data to evaluate her iron levels
C. provide her with a detailed list of iron-containing foods
D. start an oral contraceptive to prevent excessive menstrual blood loss

American Academy of Pediatrics 781


PREP® Self-Assessment PREPSA 2023

Correct Answer: B
The adolescent in the vignette has anemia and multiple risk factors for iron deficiency including
a diet deficient in iron-containing foods and heavy monthly menses. However, her hemoglobin
level did not improve 1 month after starting an iron supplement. The best next management step
is to obtain laboratory testing to evaluate her iron levels to confirm the diagnosis of iron-
deficiency anemia.

Most likely, the lack of improvement in the adolescent’s hemoglobin level is due to non-
adherence with her iron supplement. However, it is important to ensure that there is not an
alternative etiology of her anemia before focusing solely on methods to increase her iron intake
(eg, a detailed list of iron-containing foods, strategies to improve medication adherence).
Decreasing her menstrual flow with an oral contraceptive is unlikely to correct the current iron
deficiency when there is poor dietary iron intake.

Adolescent females are at risk for iron deficiency due to monthly blood loss from menstruation
as well as from having an iron-deficient diet. An intake of 15 mg of iron per day is recommended
for adolescent females to maintain their iron stores and prevent anemia. A normal hemoglobin
level for an adolescent is 12-15 g/dL (120-150 g/L).

Iron is absorbed by the duodenum and the jejunum. Heme iron from meat is the most easily
absorbed form of dietary iron. Ferric iron, the most common dietary form of iron, must be
chelated in order to be absorbed in the small intestine. Iron is required for hemoglobin
production. Anemia resulting from its deficiency can have wideranging effects on a child’s
health; iron-deficiency anemia in infancy has been associated with lower standardized test scores
at age 11 to 14 years.

Symptoms and signs of iron-deficiency anemia include fatigue, inattention, and difficulty with
concentration. Pica and restless leg syndrome are less common symptoms. Restless leg syndrome
is a sensation of discomfort in the lower limbs that is relieved by leg movement. The sensation is
often worse when lying still, attempting to sleep.

PREP Pearls
• An intake of 15 mg of iron per day is recommended for menstruating adolescent females
to maintain their iron stores and prevent anemia.
• Iron-deficiency anemia can cause fatigue, poor attention and concentration, restless leg
syndrome, pica, and negative neurodevelopmental outcomes.
• The diagnosis of iron-deficiency anemia should be confirmed with iron levels if there is
no response to iron supplementation.

ABP Content Specifications(s)


• Identify non-hematologic effects of iron deficiency
• Understand the etiology of iron deficiency

American Academy of Pediatrics 782


PREP® Self-Assessment PREPSA 2023

Suggested Readings
• Krishnamurti L. Iron-deficiency anemia. In: McInerny TK, Adam HM, Campbell DE,
DeWitt TG, Foy JM, Kamat DM, eds. American Academy of Pediatrics Textbook of
Pediatric Care. American Academy of Pediatrics; 2021:chap 279. Accessed September 1,
2022. Pediatric Care Online.
• Tong S, Vichinsky E. Iron deficiency: implications before anemia. Pediatr Rev.
2021;42(1):11-20. doi:10.1542/pir.2018-
• Wang M. Iron deficiency and other types of anemia in infants and children. Am Fam
Physician. 2016;93(4):270-278. https://pubmed.ncbi.nlm.nih.gov/26926814/.

American Academy of Pediatrics 783


PREP® Self-Assessment PREPSA 2023

Question 234
A 17-year-old adolescent girl is evaluated for a 3-week history of foul-smelling vaginal
discharge and mild vaginal itching. She has had 1 lifetime sexual partner and uses condoms
inconsistently. She was evaluated at the local health department 1 week ago where testing results
for sexually transmitted infections, including chlamydia, gonorrhea, trichomonas, and HIV, were
negative. On speculum examination, there is a homogeneous, white vaginal discharge with a
fish-like odor, and a normal cervix. There is no tenderness on the bimanual examination. The
remainder of her physical examination findings are normal. The vaginal discharge is further
evaluated.

Of the following, the MOST likely laboratory finding for this adolescent is

A. >10 white blood cells/HPF on a wet preparation


B. budding hyphae on a 10% potassium hydroxide preparation
C. multiple clue cells on a wet preparation
D. vaginal pH <4.5 on pH paper

American Academy of Pediatrics 784


PREP® Self-Assessment PREPSA 2023

Correct Answers: C
The adolescent in the vignette has signs and symptoms consistent with bacterial vaginosis (BV).
The gold standard test for BV is the Nugent score of a vaginal fluid Gram stain. In clinical
practice, BV can be diagnosed by the presence of at least 3 of the following Amsel criteria:
• Homogeneous, thin vaginal discharge that smoothly coats the vaginal walls
• Clue cells (vaginal epithelial cells covered with adherent bacteria) on microscopic
examination (Item C234A)
• pH of vaginal fluid >4.5 (normal vaginal pH is 3.8 to 4.3)
• Fishy odor of vaginal discharge before or after addition of 10% potassium hydroxide
(KOH; ie, the whiff test)

Photomicrograph of a vaginal smear specimen depicts 2 epithelial cells: a normal cell and an epithelial
cell with its exterior covered by bacteria, giving the cell a roughened, stippled appearance known as a
clue cell. These are typically seen in bacterial vaginosis. (Reprinted with permission from the Centers for
Disease Control and Prevention’s Public Health Image Library. Source of original image: M. Rein,
1978.)

Of the response choices, the presence of clue cells on wet preparation is the finding most
consistent with BV and, therefore, the most likely laboratory finding for the adolescent in the
vignette. Bacterial vaginosis infection leads to an abnormally high vaginal pH and is not
associated with an elevated number of white blood cells (WBCs) on wet preparation. Vaginal
wet preparations showing more than 10 WBCs/HPF are associated with sexually transmitted
infections (STIs) that cause cervicitis or severe vaginitis. Budding hyphae visualized after
addition of 10% KOH to the wet preparation is consistent with a Candida infection. Potassium
hydroxide causes cell lysis and destruction, facilitating the identification of budding hyphae and
yeast cells. The adolescent in the vignette does not have signs or symptoms consistent with
Candida vaginitis, which typically presents with a thick, white vaginal discharge and itching. In
cases of BV, the addition of 10% KOH to the wet preparation elicits a fish-like odor (ie, whiff
test).

Bacterial vaginosis, the most common cause of vaginal discharge worldwide, is due to an
overgrowth of anaerobic bacteria (eg, Gardnerella vaginalis, Prevotella species) which displaces
the normal vaginal flora (ie, Lactobacillus). It rarely occurs in women who have never had
vaginal sex; it is associated with having multiple male sex partners, female partners, inconsistent
condom use, and douching. Young women with BV infection are more likely to acquire
an STI including HIV.

American Academy of Pediatrics 785


PREP® Self-Assessment PREPSA 2023

Treatment for BV is recommended for symptomatic women (Item C234B). Female partners of
women with BV should be tested for BV. It is not recommended to test male partners of women
with BV.

It is important to note that clindamycin cream is oil based and might weaken latex condoms and
diaphragms for 5 days after use. A disulfiram-like reaction has been described with concurrent
use of alcohol and metronidazole or tinidazole. However, recent evidence does not support this
concern; refraining from alcohol use while taking metronidazole (or tinidazole) is unnecessary.

American Academy of Pediatrics 786


PREP® Self-Assessment PREPSA 2023

American Academy of Pediatrics 787


PREP® Self-Assessment PREPSA 2023

PREP Pearls
• Bacterial vaginosis is associated with a homogeneous thin vaginal discharge, fishy odor,
abnormally high vaginal pH, and the presence of clue cells on a wet preparation.
• First-line treatment for bacterial vaginosis is metronidazole orally or intravaginally.
• All adolescents with bacterial vaginosis should be tested for sexually transmitted
infections including gonorrhea, chlamydia, trichomonas, and HIV.

ABP Content Specifications(s)


• Recognize the clinical findings associated with bacterial vaginosis

Suggested Readings
• Dinerman LM, Joffe A. Vaginal discharge. In: McInerny TK, Adam HM, Campbell DE,
DeWitt TG, Foy JM, Kamat DM, eds. American Academy of Pediatrics Textbook of
Pediatric Care. American Academy of Pediatrics; 2021:chap 205. Accessed September 1,
2022. Pediatric Care Online.
• Lemly D, Gupta N. Sexually transmitted infections part 2: discharge syndromes and
pelvic inflammatory disease. Pediatr Rev. 2020;41(10):522-537. doi:10.1542/pir.2019-
0078.
• Workowski, KA, Bachmann LH, Chan, PA, et al. Sexually Transmitted Infections
Treatment Guidelines, 2021. MMWR Recomm Rep. 2021;70(No. RR 3-99). Accessed
September 1, 2022. https://www.cdc.gov/std/treatmentguidelines/bv.htm.

American Academy of Pediatrics 788


PREP® Self-Assessment PREPSA 2023

Question 235
A 7-year-old girl is seen in the clinic to establish care. Review of her vaccination records reveals
that she received the diphtheria, tetanus, pertussis (DTaP) vaccine at 2, 4, and 6 months but did
not receive subsequent boosters. In order to enter school, her tetanus, diphtheria, and pertussis
vaccination status must be up to date.

Of the following, the MOST appropriate vaccine to administer today is


A. diphtheria and tetanus (DT)
B. diphtheria, tetanus, pertussis (DTaP)
C. tetanus and diphtheria (Td)
D. tetanus, diphtheria, pertussis (Tdap)

American Academy of Pediatrics 789


PREP® Self-Assessment PREPSA 2023

Correct Answer: D
The girl in the vignette is not fully vaccinated against tetanus, diphtheria, and pertussis and
should receive a tetanus, diphtheria, pertussis (Tdap) vaccine. In the United States, 7-year-olds
are considered fully vaccinated against these pathogens if they have had 5 valid doses of
diphtheria, tetanus, pertussis (DTaP) (typically administered at ages 2, 4, 6, and 15 to 18 months,
and 4 to 6 years) meeting the required minimum intervals between doses, or 4 valid
doses of DTaP if the fourth dose was administered at age 4 years or older. Because the girl
received only 3 doses of DTaP, she is currently due for an additional diphtheria, tetanus,
pertussis vaccine as recommended by the Centers for Disease Control and Prevention. For
children and adolescents aged 7 to 18 years who are not fully vaccinated with DTaP, the
preferred first dose in the catch-up series is Tdap; if subsequent doses are needed, either tetanus
and diphtheria (Td) or Tdap are recommended.

Children aged 7 to 9 years who receive Tdap as a catch-up dose should also receive the routine
adolescent dose of Tdap at age 11 to 12 years. Children aged 10 years who receive Tdap do not
need the routine dose of Tdap at age 11 to 12 years.

If DTaP is inadvertently administered to children aged 7 to 9 years, it may count as part of the
catch-up series, and the routine adolescent Tdap dose should be administered at age 11 to 12
years. If DTaP is inadvertently administered to children aged 10 to 18 years, the dose of DTaP
may be counted as the adolescent Tdap booster.

PREP Pearls
• During childhood, a total of 6 doses of a pertussis-containing vaccine are recommended:
diphtheria, tetanus, pertussis (DTaP) in a 5-dose series, typically at ages 2, 4, 6, and 15 to
18 months, and 4 to 6 years; and 1 dose of tetanus, diphtheria, pertussis (Tdap) at age 11
to 12 years.
• For children and adolescents aged 7 to 18 years who are not fully vaccinated against
tetanus, diphtheria, and pertussis, the preferred first vaccine in the catch-up series is
tetanus, diphtheria, pertussis (Tdap).

MOCA-Peds Objective
• Manage children who are behind on vaccines or have an unknown vaccination record.

ABP Content Specifications(s)


• Know the indications, contraindications, schedules, and possible complications
associated with pertussis vaccine

American Academy of Pediatrics 790


PREP® Self-Assessment PREPSA 2023

Suggested Readings
• American Academy of Pediatrics. Children who received immunizations outside the
United States or whose immunization status is unknown or uncertain. In: Kimberlin DW,
Barnett ED, Lynfield R, Sawyer MH, eds. Red Book: 2021–2024 Report of the
Committee on Infectious Diseases. 32nd ed. American Academy of Pediatrics; 2021.
Accessed September 1, 2022. Red Book Online.
• American Academy of Pediatrics. Pertussis (whooping cough). In: Kimberlin DW,
Barnett ED, Lynfield R, Sawyer MH, eds. Red Book: 2021–2024 Report of the
Committee on Infectious Diseases. 32nd ed. American Academy of Pediatrics; 2021.
Accessed September 1, 2022. Red Book Online.
• Daniels HL, Sabella C. Bordetella pertussis (pertussis). Pediatr Rev. 2018;39(5):247-257.
doi:10.1542/pir.2017-0229.
• Humiston SG, Atkinson WL, Rand C, et al. Immunizations. In: McInerny TK, Adam
HM, Campbell DE, DeWitt TG, Foy JM, Kamat DM, eds. American Academy of
Pediatrics Textbook of Pediatric Care. American Academy of Pediatrics; 2021:chap 20.
Accessed September 1, 2022. Pediatric Care Online.
• Wodi PA, Ault K, Hunter P, McNally V, Szilagyi PG, Bernstein H. Advisory Committee
on Immunization Practices Recommended Immunization Schedule for Children and
Adolescents Aged 18 Years or Younger—United States, 2021.MMWR Morb Mortal
Wkly Rep. 2021;70(6);189-192.
https://www.cdc.gov/mmwr/volumes/70/wr/mm7006a1.htm.

American Academy of Pediatrics 791


PREP® Self-Assessment PREPSA 2023

Question 236
A term neonate had Apgar scores of 8 and 9 at 1 and 5 minutes, respectively, after birth. After
delivery, he was active and crying, with room air oxygen saturations ranging from 93% to 97%.
Approximately 10 minutes after birth, when he was quiet and calm, the neonate became
cyanotic, his oxygenation saturation level decreased to 65% to 70%, and he developed
significant respiratory distress with subcostal retractions and nasal flaring. He was treated with
bag/mask positive-pressure ventilation using inspired fractionated oxygen of 100% without
significant improvement in clinical status. He was, therefore, intubated, and positive-pressure
ventilation was continued. The neonate’s clinical status improved almost immediately after
intubation. His oxygen saturation increased to 100%; oxygen supplementation was quickly
weaned and discontinued. He was placed on continuous positive airway pressure of 5 cm H2O.
He is now pink, well perfused, and breathing comfortably in room air, with oxygen saturations of
95% to 98%.

Of the following, the BEST next step in this neonate’s management is to


A. obtain an arterial blood gas
B. obtain echocardiography
C. pass a nasogastric tube through both nares
D. place an umbilical venous catheter

American Academy of Pediatrics 792


PREP® Self-Assessment PREPSA 2023

Correct Answer: C
The neonate in the vignette most likely has bilateral choanal atresia. After delivery, he had no
respiratory distress and had normal oxygen saturation levels. He quickly developed oxygen
desaturation and respiratory distress when he became quiet and calm. The best next step in his
management would be to pass a nasogastric tube through both nares. In the case of bilateral
choanal atresia, the physician would be unable to pass the tube through either naris.

Because infants up to 2 to 3 months of age are obligate nose breathers, any obstruction (Item
C236A) of the nares will cause respiratory distress. When neonates with nasal obstruction are
crying, they are able to breathe through an open mouth, but when they are calm and quiet, they
cannot breathe in air.

Causes of Neonatal Nasal Obstruction


• Congenital narrowing
o Choanal atresia
o Choanal stenosis
o Piriform aperture stenosis
o Binden syndrome (nasal/midface hypoplasia and nasal passage stenosis)
• Congenital tumors/cysts
o Nasolacrimal duct cyst (nasocystocele)
o Hemangioma
o Other vascular malformations
o Encephalocele/meningoencephalocele
o Glioma
o Heterotopic brain tissue
o Dermoid cyst
o Teratoma
o Hamartoma
• Genetic
o Fetal alcohol syndrome
o CHARGE association
o Treacher-Collins syndrome
o Crouzon syndrome
o Apert syndrome
• Inflammatory
o Upper respiratory tract infection
o Respiratory syncytial virus infection
o Gastroesophageal reflux disease
o Recurrent vomiting from any cause
o Allergic (cow milk or soy protein) rhinitis
o Congenital syphilis (“snuffles”)
o Chlamydial nasopharyngitis
• Iatrogenic
o Nasotracheal tube pressure
o Nasal suctioning irritation
o Rhinitis medicamentosa
o Maternal ingestion of drugs (eg, methyldopa)
American Academy of Pediatrics 793
PREP® Self-Assessment PREPSA 2023

• Metabolic
o Hypothyroidism
• Trauma
o Intrauterine pressure on the nasal tip (positional asymmetry)
o Nasal septal dislocation
• Malignant
o Lymphoma
o Nasopharyngeal rhabdomyosarcoma
• Foreign body

The neonate’s clinical status markedly improved once he was provided a stable airway through
an endotracheal tube, which is suggestive of nasal obstruction rather than a cardiac lesion;
therefore, echocardiography is not immediately indicated. An arterial blood gas and an umbilical
venous catheter may be needed as part of ongoing care but would not be prioritized over the
attempt to pass a nasogastric tube.

Choanal atresia is a congenital blockage in the passages between the nasal cavity and the
nasopharynx, with an incidence of 1 per 8,000 live births. It can be unilateral or bilateral.
Choanal stenosis, with partial blockage, can also occur. Bilateral choanal atresia causes
significant respiratory distress in the neonate. In contrast, unilateral choanal atresia or choanal
stenosis may not be detected in the newborn period. Choanal atresia may be bony and
membranous (mixed type; 70%) or bony only (30%); few, if any, are purely membranous. The
diagnosis is usually confirmed with computed tomography of the nasopharynx and nasal cavity
(Item C236B). Bilateral choanal atresia requires surgical repair.

Axial CT scan of unilateral choanal atresia

A neonate or infant with choanal atresia or stenosis should undergo evaluation for associated
anomalies, such as those seen in CHARGE syndrome. CHARGE syndrome is an autosomal
dominant genetic condition caused by a pathologic variant in the CHD7 gene. The incidence is
approximately 1 in 10,000 to 15,000 live births. The clinical features of CHARGE syndrome are
variable and may include: Coloboma, Heart defects, Atresia or stenosis of the choanae,
Retardation of growth and development, Genitourinary anomalies, and Ear anomalies (including
hearing loss). Patients with CHARGE syndrome also have central nervous system involvement
that leads to neurodevelopmental delays, distinguishing it from VACTERL (Vertebral defects,
Anal atresia, Cardiac defects, Tracheo-Esophageal fistula, Renal anomalies, and Limb
abnormalities) association.

American Academy of Pediatrics 794


PREP® Self-Assessment PREPSA 2023

PREP Pearls
• Bilateral choanal atresia in a neonate is a medical emergency because neonates are
obligate nose breathers.
• Passage of a nasogastric tube through both nares should be attempted in a neonate with
suspected choanal atresia.
• A neonate with choanal atresia or stenosis should undergo evaluation for associated
anomalies, such as those seen in CHARGE syndrome (Coloboma, Heart defects, Atresia
or stenosis of the choanae, Retardation of growth and development, Genitourinary
anomalies, and Ear anomalies [including hearing loss]).

ABP Content Specifications(s)


• Plan the appropriate evaluation of choanal atresia
• Recognize the clinical findings associated with both unilateral and bilateral choanal
atresia
• Understand the association of choanal atresia with other congenital anomalies

Suggested Readings
• Hudson A, Trider C-L, Blake K. CHARGE syndrome. Pediatr Rev. 2017;38(1):56–59.
doi:10.1542/pir.2016-0050.
• Kaplan J, Hudgins L. Neonatal presentations of CHARGE syndrome and
VATER/VACTERL association. Neoreviews. 2008;9(7):e299–e304. doi:10.1542/neo.9-
7-e299.
• Olnes SQ, Schwartz RH, Bahadori RS. Consultation with the specialist: diagnosis and
management of the newborn and young infant who have nasal obstruction. Pediatr Rev.
2000;21(12):416-420. doi:10.1542/pir.21.12.416.

American Academy of Pediatrics 795


PREP® Self-Assessment PREPSA 2023

Question 237
A 15-year-old adolescent girl is evaluated in the clinic for a 4-week history of worsening back
pain. She has competed in gymnastics for the past 5 years but is currently limited by pain,
especially with back extension. There is no history of trauma. She does not have numbness,
tingling, or changes in her bowel or bladder function. On physical examination, the adolescent
has pain with back extension. Back flexion and rotation are normal and pain free. She has normal
strength in her arms and legs. The remainder of the adolescent’s physical examination findings
are unremarkable.

Of the following, the BEST imaging modality to evaluate this adolescent’s findings is
A. a bone scan
B. computed tomography
C. magnetic resonance imaging
D. plain radiography

American Academy of Pediatrics 796


PREP® Self-Assessment PREPSA 2023

Correct Answer: D
The adolescent in the vignette is a gymnast and has pain with back extension. Spondylolisthesis
is the most likely diagnosis. This condition can usually be identified on plain radiography. If
plain radiographs are normal and there is a high degree of suspicion, magnetic resonance
imaging (MRI) is the next best option. Computed tomography (CT) and a bone scan may be
indicated in the evaluation of back pain due to other etiologies (eg, primary or metastatic bone
tumor and injury) but would not be the best imaging options to evaluate the adolescent in the
vignette.

Spondylosis is a fracture of the vertebral pars interarticularis. Spondylolisthesis is a fracture with


anterior displacement of the vertebral bodies. Spondylosis and spondylolisthesis typically occur
in adolescents. Activities associated with repetitive extension, twisting, and axial loading (eg,
gymnastics, dance, football, baseball, weighted squats) predispose children and adolescents to
this injury. The pain is typically associated with back extension because the injury is located in
the posterior spine. Plain radiography of the spine (standing anterior-to-posterior and lateral
lumbar views) is recommended as the initial imaging evaluation. If plain radiography is not
revealing, MRI is the next best imaging study given the lack of radiation and ability to identify
stress fractures. The initial treatment of spondylosis and spondylolisthesis is supportive with
nonsteroidal anti-inflammatory drugs (NSAIDs), limiting back extension, 6 weeks of rest, and
physical therapy to increase core strength. Children and adolescents with spondylolisthesis may
require bracing or spinal fusion if significant displacement and/or neurological symptoms are
present.

The etiology of back pain in children varies by age group. Signs and symptoms associated with
significant pathology include pain that wakes the child from sleep, pain that radiates to the limbs,
age younger than 4 years, fever, and neurological symptoms (eg, weakness, numbness, difficulty
walking, changes in bowel or bladder function).

Physical examination findings can help identify the location of the injury. Pain with flexion is
associated with injury to anterior back structures (eg, vertebral bodies, disks, muscles,
ligaments). Pain with extension is associated with more posterior injuries (eg, facet joint, pedicle,
spinous processes). Pain with rotation that occurs on the contralateral side is associated with
muscle and ligament injury of the back, hamstrings, and hip flexors. Pain with rotation that
occurs on the ipsilateral side is more commonly associated with bone injury.

Physical examination should also include evaluation for asymmetry of the shoulders and iliac
crest, leg length, and musculature of the back. The spinal processes should be palpated for
tenderness and step-offs, and the paraspinal muscles should be evaluated for tone and tenderness.
Tenderness over the costovertebral angle is more often caused by renal disease than back
pathology.

The differential diagnosis for back pain in children includes malignancy (bony or hematologic)
neoplasms, osteomyelitis, discitis, autoimmune disorders, and rheumatologic disorders. The risk
of autoimmune and rheumatologic disorders is increased in those with a family history of these
American Academy of Pediatrics 797
PREP® Self-Assessment PREPSA 2023

conditions. Cauda equina syndrome can present with loss of feeling or tingling in the legs,
especially in the medial aspects, with or without bowel or bladder symptoms.

Back pain in infants and young children usually indicates serious pathology. Infection (eg,
discitis, osteomyelitis) can present with fever, elevated white blood cell count, and elevated
inflammatory markers. Magnetic resonance imaging with contrast is recommended to diagnose
these infections. Staphylococcus aureus is the most common bacterial etiology. Treatment is with
intravenous antibiotics. Consideration for trauma, including nonaccidental trauma, is important
in this age group. A careful physical examination and history are crucial. Malignancies
associated with back pain that occur in this age group include leukemia and lymphoma. A
complete blood cell count with differential, to identify blasts, and a bone marrow biopsy are
indicated if malignancy is suspected.

Like all bones in children, there are growth plates in the vertebrae, both in the inferior and
superior aspects. These ossify around 4 years of age and close around 18 years of age. These
areas are weaker, making them prone to injury. Injury in the posterior vertebral rim is most
common; this can be associated with pain during valsalva due to intermittent nerve impingement.
The differential diagnosis of back pain in an adolescent includes infection (including sexually
transmitted infections, tuberculosis), rheumatologic disorders, malignancy, and injury (eg, stress
fracture, acute fracture, muscle strain and sprain, disc herniation). Adolescents should be
evaluated for scoliosis at health supervision visits and when presenting with back pain. Scoliosis
should be suspected when an abnormal lateral curvature in the spine is persistent when standing
and with forward flexion. The diagnosis is confirmed by measurement of a Cobb angle of greater
than 10 degrees on spine radiography obtained while the child is standing.

Primary bony neoplasms, benign and malignant types, can cause bone pain in children. Osteoid
osteoma is associated with pain after physical activity that is alleviated by nonsteroidal anti-
inflammatory drugs (NSAIDs). These sclerotic lesions are typically small (<1.5 cm) and can be
challenging to identify on plain radiographs. They are better visualized on bone scan or CT.
These are not malignant lesions; the treatment is surgical resection or radiofrequency ablation.
Osteoblastomas are large (>2 cm) cystic lesions that can rarely become malignant. The pain
associated with an osteoblastoma is not typically alleviated with NSAIDs. Ewing sarcoma and
osteosarcoma are rare, malignant neoplasms. They typically affect the anterior spine and can lead
to cord impingement with associated neurological symptoms. Systemic symptoms, such as
fatigue, weight loss, and fever, may be present. On plain radiography, they appear as destructive
lesions. Computed tomography or MRI are often required for staging and further evaluation.

Children and adolescents with a family history of autoimmune diseases, especially those
associated with the HLA B27 allele, are at risk for sacroiliitis (sacroiliac joint) and ankylosing
spondylitis (spinal). These diagnoses are also associated with iritis and Crohn disease. Affected
children typically have pain on forward flexion and impaired range of motion. Plain radiography
often shows a “bamboo spine.” Early iliitis may not be visualized on plain radiographs; if there is
high suspicion, MRI of the pelvis with and without contrast should be obtained. The initial

American Academy of Pediatrics 798


PREP® Self-Assessment PREPSA 2023

treatment includes NSAIDs and rest. These children should be referred to a rheumatologist for
further evaluation and treatment.

PREP Pearls
• Signs and symptoms associated with pathological causes of back pain include pain that
wakes the child from sleep, pain that radiates to the limbs, age younger than 4 years,
fever, and neurological symptoms (eg, weakness, numbness, difficulty walking, changes
in bowel or bladder function).
• The differential diagnosis of for back pain in children includes bone neoplasm (osteoid
osteoma, and osteoblastoma), hematologic malignancy, infection, autoimmune and or
rheumatologic disorders (especially in those with a family history), and bone and/or
muscle injury.
• In infants, causes of back pain include infection, trauma (including nonaccidental
trauma), and malignancy; in adolescents, causes include infection (including sexually
transmitted infections and tuberculosis), rheumatologic disorders, disc herniation, muscle
strains, and acute or stress fractures.

ABP Content Specifications(s)


• Formulate a differential diagnosis of back pain in children of various ages
• Plan the appropriate management of back pain
• Plan the appropriate evaluation of back pain

Suggested Readings
• American Academy of Orthopaedic Surgeons. Back pain in children. Accessed
September 1, 2022. https://orthoinfo.aaos.org/en/diseases--conditions/back-pain-in-
children/.
• Brenner JS, Smith DV. Back pain. In: McInerny TK, Adam HM, Campbell DE, DeWitt
TG, Foy JM, Kamat DM, eds. American Academy of Pediatrics Textbook of Pediatric
Care. American Academy of Pediatrics; 2021:chap 131. Accessed September 1, 2022.
Pediatric Care Online .
• DeWolfe C. Back pain. Pediatr Rev. 2002;23(6):221. doi:10.1542/pir.23-6-221.
• Lamb M, Brenner JS. Back pain in children and adolescents. Pediatr Rev.
2020;41(11):557-569. doi:10.1542/pir.2019-0051

American Academy of Pediatrics 799


PREP® Self-Assessment PREPSA 2023

Question 238
A 12-year-old boy is brought to the office by his aunt to establish care. His mother has a history
of substance use, and the boy was removed from her home at age 3 years by child protective
services for neglect. The boy lived with his grandmother until a year ago when she could not take
care of him any longer because of his behavior. The boy has been living with his aunt since then.
The boy’s aunt reports years of behavioral problems including frequent tantrums, breaking of
toys/electronics, and “constant talking back and arguing” with his grandmother and teachers. He
has been diagnosed with attentiondeficit/hyperactivity disorder (ADHD). The boy told his aunt
that he “hates everyone” and “hates school.” He has been suspended from school multiple times
for offenses including a physical altercation with another student, spraypainting a school sign,
and stealing another student’s tablet. When confronted about these incidents he will lie or blame
someone else.

The boy’s aunt brought a stimulant medication prescription bottle to the visit and states, “He
takes this for his ADHD, but it does nothing for his behavior.” She admits that she frequently
wonders “whether it’s too late to make a difference” and if she can continue to take care of the
boy.

Of the following, the BEST next step(s) in this boy’s evaluation is to


A. ask his aunt to complete a release-of-information form to obtain prior medical records
B. call his grandmother to obtain additional medical and behavioral history
C. find an inpatient psychiatric facility to evaluate and diagnose him
D. interview him privately and obtain standardized behavioral questionnaires from his
teachers

American Academy of Pediatrics 800


PREP® Self-Assessment PREPSA 2023

Correct Answers: D
The boy in the vignette likely meets criteria for conduct disorder based on the behaviors reported
by his aunt. To confirm the diagnosis and determine the severity of his behaviors and level of
impairment, information from multiple sources should be obtained. These sources include
parents/caregivers, teachers/school personnel, and the child. Standardized rating scales
completed by his teachers may provide useful information and can support the assessment for co-
occurring conditions.

A diagnosis of conduct disorder, according to the Diagnostic and Statistical Manual of Mental
Disorders, 5th Edition (DSM-5) is made for children who demonstrate at least 3 of 15 criteria in
the following categories:
• Aggression to people or animals
• Destruction of property
• Deceitfulness
• Theft
• Serious violations of rules

The behaviors must have occurred within the past 12 months with at least 1 behavior occurring
within the past 6 months.

Children with oppositional defiant disorder (ODD) display an irritable or angry mood,
argumentative or defiant behaviors, and vindictiveness. They may exhibit the following
behaviors:
• Not following rules
• Arguing with parents or authority figures
• Excessive anger

The behaviors seen in ODD are frequently less severe than those in conduct disorder and do not
include aggressive behaviors toward people or animals, destruction of property, or theft. Based
on the history provided, the boy in the vignette likely met criteria for ODD prior to
demonstrating behaviors consistent with conduct disorder. Both diagnoses can be made in the
same individual when criteria are met for both.

Comorbidities frequently seen with both ODD and conduct disorder include attention-
deficit/hyperactivity disorder (ADHD), depression, anxiety, and learning disorders. As part of
the initial evaluation, co-occurring conditions must be assessed for and treated appropriately.
Treatment of ADHD with psychotropic medication such as stimulants may be indicated.
The major component of treatment for both ODD and conduct disorder is behavioral therapy.
This therapy may include parent management training, cognitive behavioral therapy, and family
counseling. Additionally, school and community-based support are vital and may include an
individualized education plan, social skills programming, tutoring, and mentoring. Long-term
outcomes for both conditions are improved with early identification and implementation of
interventions.

American Academy of Pediatrics 801


PREP® Self-Assessment PREPSA 2023

Obtaining prior medical records for the boy in the vignette, while helpful, would not be as
important at this time as obtaining information about his current behavior. Although contacting
the boy’s grandmother may provide additional details regarding his history of neglect and early
temperament as well as his developmental history, this is not a necessary next step in his
evaluation as his aunt has provided a behavioral history that supports a diagnosis of conduct
disorder. Referral to a psychiatric facility or specialist should be reserved for children who have
made physical threats, when there are concerns for the safety of family members, or when
behaviors result in involvement of the legal system or school expulsion.

PREP Pearls
• Oppositional defiant disorder and conduct disorder can be diagnosed in the same
individual; behaviors seen in conduct disorder are more severe and can include
aggressive, destructive, and deceitful behaviors.
• Management of oppositional defiant disorder and conduct disorder includes assessment
for and treatment of cooccurring conditions (eg, attention-deficit/hyperactivity disorder,
depression, anxiety) with medication and/or behavioral interventions as appropriate.

ABP Content Specifications(s)


• Differentiate the findings associated with oppositional defiant or conduct disorder from
those of temperamental variations
• Plan the appropriate evaluation of oppositional defiant or conduct disorder
• Plan the appropriate management of oppositional defiant or conduct disorder

Suggested Readings
• Diagnostic and Statistical Manual of Mental Disorders, 5th Edition: DSM-5. American
Psychiatric Association; 2013.
• Albright MB, Maurasse SL, Ablon JS. Oppositional defiant disorder. In: McInerny TK,
Adam HM, Campbell DE, DeWitt TG, Foy JM, Kamat DM, eds. American Academy of
Pediatrics Textbook of Pediatric Care. American Academy of Pediatrics; 2021:chap 348.
Accessed September 1, 2022. Pediatric Care Online.
• Blair RJ, Leibenluft E, Pine DS. Conduct disorder and callous-unemotional traits in
youth. N Engl J Med. 2014;371(23):2207-2216. doi:10.1056/NEJMra1315612.
• Sanders LM, Schaechter J, Serwint JR. Conduct disorder. Pediatr Rev. 2007;28(11):433-
434. doi:10.1542/pir.28-11-433.
• Zahrt DM, Melzer-Lange MD. Aggressive behavior in children and adolescents. Pediatr
Rev. 2011;32(8):325-332. doi:10.1542/pir.32-8-325.

American Academy of Pediatrics 802


PREP® Self-Assessment PREPSA 2023

Question 239
A 6-year-old girl is seen in the office for evaluation of recurrent urinary tract infections (UTIs).
She has had 4 UTIs in the past 6 months. With each infection, she was seen with dysuria, urinary
urgency, and suprapubic pain. She has not had fever, nausea, vomiting, back pain, diarrhea, or
hematuria. During each episode, a urinalysis was significant for 2+ leukocyte esterase and
positive for nitrites, and each urine culture yielded more than 100,000 CFU of Escherichia coli.
After appropriate treatment of each infection, her symptoms resolved, posttreatment urinalysis
results were normal, and a urine culture result was negative. Renal ultrasonography and a
voiding cystourethrography had normal results. The girl is developmentally appropriate. She has
a bowel movement once a day that is usually hard and painful. Her review of systems is
otherwise negative. The girl’s vital signs, growth parameters, and physical examination findings
are normal.

Of the following, the BEST next step in this girl’s management is to


A. administer prophylactic antibiotics
B. optimize bowel regimen
C. provide reassurance
D. refer to a urologist

American Academy of Pediatrics 803


PREP® Self-Assessment PREPSA 2023

Correct Answer: B
The girl in the vignette has recurrent urinary tract infections (UTIs) most likely secondary to
bowel bladder dysfunction; therefore, the best next management step is to optimize her bowel
regimen. Prophylactic antibiotics will not address her underlying condition (chronic
constipation), are unlikely to prevent her UTIs in the long term, and may contribute to antibiotic
resistance. Referral to a urologist may be necessary if she continues to have recurrent UTIs after
optimization of her bowel regimen. Reassurance alone is inadequate because she is likely to have
recurrent UTIs until her chronic constipation is successfully treated.

Urinary tract infections occur when bacteria ascend the urethra and cause bladder inflammation.
If the bacteria ascend into the kidneys, pyelonephritis or renal abscess may result. If the bacteria
enter and spread through the bloodstream, urosepsis and meningitis may occur. Neonates and
immunocompromised children are at higher risk for systemic infection.

Bowel and bladder dysfunction is a common but underrecognized cause of recurrent UTIs.
Bowel dysfunction results in constipation that can cause obstruction of bladder drainage with
inadequate emptying, leading to retention of bacteria. Many families do not recognize
constipation symptoms. A thorough evaluation, including a bowel pattern diary, may be
necessary to reveal signs and symptoms of bowel bladder dysfunction.

Renal scarring may occur after recurrent UTIs but does not usually lead to long-term sequelae in
the absence of anatomical abnormalities of the urinary system. Children with high-grade
vesicoureteral reflux, posterior urethral valves, or other obstructive urinary anomalies can
develop chronic kidney disease with repeated or inadequately treated UTIs.

PREP Pearls
• Bowel and bladder dysfunction resulting in inadequate bladder emptying is a common
but underrecognized cause of recurrent urinary tract infections.
• Renal scarring may occur after recurrent urinary tract infections but does not usually lead
to long-term sequelae in the absence of anatomical urinary system abnormalities.
• If bacteria enter the bloodstream from the urinary tract, urosepsis and meningitis may
occur; neonates and immunocompromised children are at higher risk for systemic
infection.

ABP Content Specifications(s)


• Understand the natural history of urinary tract infection

American Academy of Pediatrics 804


PREP® Self-Assessment PREPSA 2023

Suggested Readings
• Balighian E, Burke M. Urinary tract infections in children. Pediatr Rev. 2018;39(1):3-12.
doi:10.1542/pir.2017-0007.
• Kaufman J, Temple-Smith M, Sanci L. Urinary tract infections in children: an overview
of diagnosis and management. BMJ Paediatrics Open. 2019;3(1):e000487.
doi:10.1136/bmjpo-2019-000487.
• Mattoo TK, Shaikh N, Nelson CP. Contemporary management of urinary tract infection
in children. Pediatrics. 2021;147(2):e2020012138. doi:10.1542/peds.2020-012138.
• Nanda G, Jahnukainen T, Vats A. Urinary tract infections. In: McInerny TK, Adam HM,
Campbell DE, DeWitt TG, Foy JM, Kamat DM, eds. American Academy of Pediatrics
Textbook of Pediatric Care. American Academy of Pediatrics; 2021:2742756. Accessed
September 1, 2022. Pediatric Care Online.

American Academy of Pediatrics 805


PREP® Self-Assessment PREPSA 2023

Question 240
A term female newborn is seen in the neonatal intensive care unit. She undergoes intubation
shortly after delivery for respiratory distress. She is small for gestational age with a birthweight
of 2,000 g. Physical examination findings include cleft lip and palate, a scalp defect,
microcephaly, and polydactyly of the hands bilaterally. Two-dimensional echocardiography
shows an atrial septal defect, and brain magnetic resonance imaging shows holoprosencephaly.

Of the following, the results of this neonate’s karyotype would MOST likely be
A. 45,X
B. 47,XX, +13
C. 47,XX, +18
D. 47,XX, +21

American Academy of Pediatrics 806


PREP® Self-Assessment PREPSA 2023

Correct Answer: B
The neonate in the vignette has findings consistent with trisomy 13; testing would most likely
result in a karyotype of 47,XX, +13. Neonates with trisomy 13 exhibit intrauterine growth
restriction and are small for gestational age. Clinical features may include:
• Midline defects: Cleft lip and/or palate, cyclopia
• Brain anomalies: Holoprosencephaly, microcephaly
• Scalp defect: Cutis aplasia
• Cardiac defects: Atrial septal defect, ventricular septal defect, tetralogy of Fallot, double
outlet right ventricle
• Eye anomalies: Micro-ophthalmia, anophthalmia
• Hernia: Inguinal, umbilical
• Renal: Polycystic kidney, hydronephrosis, horseshoe kidney
• Musculoskeletal: Postaxial polydactyly, congenital talipes equinovarus

A karyotype is the test of choice to diagnose and distinguish trisomy 13 from other aneuploidies.
In addition to confirmation of the diagnosis, a karyotype can detect a balanced translocation.
This information is needed for accurate recurrence risk estimation for parents. Item C240
compares the clinical features of trisomies 13, 18, and 21, and Turner syndrome.

Turner syndrome (45,X) is characterized by lymphedema, dysmorphic facial features including


down-slanted palpebral fissures, webbed neck, shield chest, and wide-spaced nipples. Congenital
anomalies can involve the cardiac, genital, and renal systems; short stature is evident in early
childhood.

Trisomy 18 (47,XX, +18) is characterized by dysmorphic features that include small jaw,
clenched hands with overlapping fingers (index overlapping the third, fifth overlapping the
fourth finger), short sternum, and rocker bottom feet. Intrauterine growth restriction is noted, and
affected children have congenital cardiac, ocular, and renal anomalies.

Trisomy 21 (47,XX, +21) has a distinctive facial profile with up-slanted palpebral fissures,
midface hypoplasia, and epicanthal folds. Clinical features include congenital heart defects,
single transverse palmar crease, and a wide space between the first and second toes.

PREP Pearls
• Trisomy 13 is characterized by midline defects such as cleft lip and/or palate,
holoprosencephaly, small for gestational age status, postaxial polydactyly, and cutis
aplasia.
• Neonates with trisomy 13 typically have midline and scalp defects; those with trisomy 18
typically have rocker bottom feet and clenched hands with overlapping fingers.
• A karyotype is used to make the diagnosis of trisomy 13.

MOCA-Peds Objective
• Evaluate and manage an infant with polydactyly.

American Academy of Pediatrics 807


PREP® Self-Assessment PREPSA 2023

ABP Content Specifications(s)


• Recognize the clinical features associated with trisomy 13

Suggested Readings
• AAP Committee on Genetics; Saul RA, ed. Overview of genetic testing. In: Medical
Genetics in Pediatric Practice. American Academy of Pediatrics; 2013:chap 10.
• Cereda A, Carey JC. The trisomy 18 syndrome. Orphanet J Rare Dis. 2012;7:81.
doi:10.1186/1750-1172-7-81.
• Levy P, Marion M. Trisomies. Pediatr Rev. 2018;39(2):104-106. doi:10.1542/pir.2016-
0198.
• Parisi MA. Genetics over the life cycle. In: AAP Committee on Genetics; Saul RA, ed.
Medical Genetics in Pediatric Practice. American Academy of Pediatrics; 2013:chap 2.

American Academy of Pediatrics 808


PREP® Self-Assessment PREPSA 2023

Question 241
A 16-year-old adolescent boy is brought to the emergency department for evaluation of right
shoulder pain. He was playing baseball and collided with another player while sliding into a
base. He felt immediate pain in his right shoulder and had limited movement of his right arm due
to the pain. On physical examination, there is tenderness to palpation over his right clavicle with
crepitus and swelling of the mid-clavicular area. Adduction and flexion of the right shoulder are
limited due to pain. There is no skin tenting. His right arm is neurovascularly intact. The
remainder of the adolescent’s physical examination findings are unremarkable. Appropriate
analgesia is administered, and right shoulder and clavicle radiographs are obtained (Item Q241A
and Item Q241B).

Of the following, the BEST next step in this adolescent’s management is

A. closed reduction, analgesia, immobilization, and follow-up with an orthopedic surgeon in


4 weeks
B. rest, ice, compression bandage, analgesia, and follow-up with an orthopedic surgeon in 2
weeks
C. urgent referral to an orthopedic surgeon for open reduction and internal fixation
D. use of arm sling, analgesia, and follow-up with the pediatrician in 2 weeks

American Academy of Pediatrics 809


PREP® Self-Assessment PREPSA 2023

Correct Answer: D
The adolescent in the vignette has a fracture of the middle third of his clavicle. Clavicle shaft
fractures occur commonly in children after a fall, direct trauma to the shoulder, or during the
birthing process. Most clavicle fractures can be treated conservatively with use of an arm sling or
figure-of-eight splint, analgesia, and follow-up with a pediatrician.

Clavicle fractures accompanied by sternoclavicular dislocation require closed reduction in


consultation with an orthopedic surgeon; these fractures are uncommon. Although rest, ice,
compression, and analgesia are appropriate adjuvant therapies for clavicle fractures, orthopedic
follow-up is not routinely indicated, particularly for fractures of the middle third of the clavicle.
The need for open reduction and internal fixation for clavicle fractures in children is
controversial, with little evidence to support this as a routine approach. Urgent orthopedic
consultation should be obtained for open fractures, sternoclavicular dislocation, distal displaced
fractures, tenting of the skin over the clavicle, or evidence of neurovascular compromise. The
adolescent in the vignette has examination findings and radiographs that do not demonstrate any
of these concerning findings; urgent referral to an orthopedic surgeon is not warranted.

The Allman classification of clavicular fractures places fractures into 3 groups based on location:
Group I: fracture of the middle third of the clavicle (most common)
Group II: fracture of the distal third of the clavicle
Group III: fracture of the proximal third of the clavicle

Fractures of the distal third (group II) of the clavicle can be difficult to distinguish from
acromioclavicular (AC) separations. They are both associated with pain, tenderness, and swelling
in the same location. Most non-displaced fractures of the distal clavicle and minor AC separation
injuries can be treated conservatively with rest and immobilization in an arm sling. Angulated or
displaced fractures of the distal clavicle require urgent referral to an orthopedic surgeon for
further evaluation and management.

Fractures of the proximal third of the clavicle (group III) are the least common type. They are
often associated with serious injury, and therefore require thorough evaluation for associated
chest, lung, neck, and vascular injuries. Group III fractures are difficult to identify on plain
radiography. If plain radiographs appear normal but clinical suspicion of fracture remains high,
computed tomography should be performed to evaluate for fracture, dislocation, and associated
intrathoracic or vessel injury.

PREP Pearls
• The middle third of the shaft is the most common location of a clavicle fracture; it can be
treated conservatively with use of an arm sling, analgesia, and follow-up with a
pediatrician.
• Urgent orthopedic surgery consultation is necessary for an open clavicle fracture,
sternoclavicular dislocation, displaced fractures of the distal third of the clavicle, tenting
of the skin over the clavicle, or evidence of neurovascular compromise.

American Academy of Pediatrics 810


PREP® Self-Assessment PREPSA 2023

• Fractures of the proximal third of the shaft are the least common type of clavicular
fracture and are often associated with serious chest, lung, neck, and vascular injury.

ABP Content Specifications(s)


• Recognize the clinical and radiographic findings associated with a fracture of the
clavicle, and manage appropriately
• Recognize the clinical and radiographic findings associated with acromioclavicular
separation

Suggested Readings
• Carson S, Woolridge DP, Colletti J, Kilgore K. Pediatric upper extremity injuries. Pediatr
Clin North Am. 2006;53(1):41-67, v. doi:10.1016/j.pcl.2005.10.003.
• Strahlman RS. Fractures and dislocations. In: McInerny TK, Adam HM, Campbell DE,
DeWitt TG, Foy JM, Kamat DM, eds. American Academy of Pediatrics Textbook of
Pediatric Care. American Academy of Pediatrics; 2021:chap 252. Accessed September 1,
2022. Pediatric Care Online.
• Vargas-Vila MA, Mehlman CT, Pennock AT. The community orthopaedic surgeon
taking trauma call: pediatric midshaft clavicle fracture pearls and pitfalls. J Orthop
Trauma. 2019;33 Suppl 8:S1-S5. doi:10.1097/BOT.0000000000001546.

American Academy of Pediatrics 811


PREP® Self-Assessment PREPSA 2023

Question 242
A previously healthy, 3-year-old boy is brought to an urgent care center with a 2-day history of
fever, right leg and hip pain, and a limp. Today, he is unable to bear weight on the right leg. One
week ago, the boy and several family members had a dry cough and nasal congestion. There is
no history of trauma. His immunizations are up to date.
The boy is irritable with a temperature of 39.4°C, heart rate of 164 beats/min, respiratory rate of
32 breaths/min, and blood pressure of 110/56 mm Hg. He is lying on the examination table with
his right lower extremity abducted and externally rotated. He cries when his right hip is
internally or externally rotated. There is no swelling, redness, or warmth over the right hip. The
remainder of the boy’s physical examination findings are normal.

Laboratory data are shown:


Laboratory Test Result
White blood cell count 21,000/µL (21.0 × 109/L)
Segmented neutrophils 76%
Lymphocytes 20%
Monocytes 4%
C-reactive protein 11.0 mg/dL (110 mg/L)
Erythrocyte sedimentation rate 50 mm/h

A radiograph of the right hip is normal.

Of the following, the BEST next step in this boy’s evaluation is


A. a blood culture
B. magnetic resonance imaging
C. a respiratory virus panel
D. ultrasonography

American Academy of Pediatrics 812


PREP® Self-Assessment PREPSA 2023

Correct Answer: D
In the vignette, the boy’s signs and symptoms of fever, limp, pain with internal and external
rotation of the right hip, leukocytosis, and elevated C-reactive protein (CRP) level support a
diagnosis of pyogenic arthritis of the hip joint. The best next step in his evaluation is
ultrasonography of the right hip to evaluate for an effusion. If an effusion is detected, the joint
should be aspirated as soon as possible to confirm the diagnosis and obtain fluid to send for cell
count and culture. A synovial fluid white blood cell count (WBC) of ≥50,000/µL (≥50.0 × 109/L)
with neutrophilic predominance is highly suggestive of pyogenic arthritis. Urgent orthopedic
consultation, surgical drainage, and irrigation of the hip joint space is required given the risk of
aseptic necrosis of the femoral head from increased intra-articular pressure and compromised
blood flow. Joint fluid cultures are positive for a bacterial pathogen in 60% of cases. Polymerase
chain reaction of the synovial fluid may be positive for fastidious organisms such as Kingella
kingae.

The laboratory evaluation of suspected pyogenic arthritis should also include a complete blood
count, blood culture, and CRP level. A blood culture is positive for a bacterial pathogen in 40%
of cases of pyogenic arthritis. While results of a blood culture may be helpful in guiding
antibiotic treatment, making the diagnosis and surgical draining of a septic joint are the first
priority. Measurement of CRP is recommended at baseline and serially to monitor treatment
response. A peripheral WBC is elevated in 50% of children with osteoarticular infections.

Plain radiography has low sensitivity in the diagnosis of pyogenic arthritis, but results may
demonstrate soft tissue swelling and widening of the joint space when a moderate-sized effusion
is present. It is helpful in excluding fracture as a cause of the child’s symptoms. Magnetic
resonance imaging (MRI) is very sensitive for the diagnosis of pyogenic arthritis and can detect
adjacent bone and soft tissue infection and cartilage injury. However, ultrasonography is
preferred over MRI as the diagnostic imaging modality for pyogenic arthritis of the hip because
of its relatively low cost, lack of sedation requirement, and sensitivity for detecting effusion.
Additionally, ultrasonography can be used to guide aspiration of the affected joint. Magnetic
resonance imaging is indicated if pelvic osteomyelitis, pyomyositis, or abscess is suspected.

The incidence of pyogenic arthritis in children is 5.5 to 12 cases per 100,000 population. Most
cases occur in children younger than age 10 years (median age, 2 years). Pyogenic arthritis in
children most often results from hematogenous spread of infection. Other causes include direct
inoculation of bacteria from trauma or surgery, and contiguous spread from adjacent bone.

Staphylococcus aureus (including methicillin-resistant Staphylococcus aureus [MRSA]) is the


most common pathogen causing pyogenic arthritis in all age groups. In neonates, group B
Streptococcus and enteric gram-negative bacilli are significant pathogens. In infants and children
aged ≤5 years, Kingella kingae is an important pathogen, followed by Streptococcus pneumonae,
group A Streptococcus (GAS) and Haemophilus influenzae type B (in unimmunized children).
Staphylococcus aureus and GAS are the most common pathogens in children aged 6 years and
older. In adolescents, infection with Neisseria gonorrhoeae should be considered.

American Academy of Pediatrics 813


PREP® Self-Assessment PREPSA 2023

Children with pyogenic arthritis usually present with fever. Swelling, warmth, and tenderness of
the affected joint may be noted. External signs of inflammation may be absent in pyogenic
arthritis of the hip joint; affected infants and children may rest in a frog-legged position (ie,
abduction and external rotation). A single joint is affected in up to 90% of cases. Polyarticular
disease is most commonly associated with arthritis caused by S aureus, GAS and N gonorrhoeae.
Lower extremity joints are involved in 80% of cases. The knee joint is most commonly affected,
followed by the hip and ankle.

The differential diagnosis of pyogenic arthritis includes transient synovitis, cellulitis, traumatic
arthritis, juvenile idiopathic arthritis, and leukemia. Transient synovitis of the hip should be
considered for the boy in the vignette with fever and limp given the history of a preceding viral
upper respiratory tract infection; however, his high fever, leukocytosis, and significant elevation
of CRP favors the diagnosis of pyogenic arthritis. A respiratory virus panel is not indicated for
the boy in the vignette; he does not currently have respiratory symptoms and the results would
not change his management. The most urgent aspect of his care at this time is evaluation for and
treatment of his likely bacterial hip infection. The modified Kocher criteria can help distinguish
pyogenic arthritis from transient synovitis of the hip:

Criteria:
• Non-weight bearing
• Temperature >38.5°C
• Erythrocyte sedimentation rate >40 mm/h, or CRP >0.25 mg/dL (>2.5 mg/L) WBC
>12,000/µL (12.0 × 109/L)

Predictive value for the diagnosis of a pyogenic joint based on number of criteria met:
• 1 criterion = 3%
• 2 criteria = 40%
• 3 criteria = 93%
• 4 criteria = 99%

In children with presumed pyogenic arthritis, empiric antibiotic therapy targeted against S aureus
and Streptococcus species is recommended. Empiric antibiotic coverage for K kingae infection
should also be initiated in children younger than age 3 years. In geographic areas with a high
prevalence of MRSA, empiric therapy should include clindamycin or vancomycin. After initial
intravenous antibiotic therapy, transition to oral antibiotics is recommended for hospitalized
children once discharge criteria are met: afebrile, clinical improvement, and a decline in CRP. A
3- to 4-week total duration of antibiotic therapy is recommended for pyogenic arthritis due to S
aureus and gram-negative bacteria. A 2- to 3-week antibiotic course may be adequate for
pyogenic arthritis caused by S pneumoniae, GAS, and K kingae.

American Academy of Pediatrics 814


PREP® Self-Assessment PREPSA 2023

PREP Pearls
• Children with pyogenic arthritis are typically seen with acute onset of fever, limp, pain,
and limited range of motion of the affected joint, and an elevated C-reactive protein level.
• A synovial fluid white blood cell count of ≥50,000/µL (≥50.0 × 109/L) with neutrophilic
predominance is highly suggestive of pyogenic arthritis and requires emergency surgical
drainage.
• In children with presumed pyogenic arthritis, empiric antibiotic therapy targeted against
Staphylococcus aureus and Streptococcus species is recommended. Empiric antibiotics to
treat Kingella kingae infection should be initiated in children younger than age 3 years.

ABP Content Specifications(s)


• Plan the appropriate diagnostic evaluation of pyogenic arthritis in patients of various ages
• Recognize the clinical findings of pyogenic arthritis or arthritis associated with rheumatic
fever in patients of various ages
• Plan the appropriate antimicrobial management of pyogenic arthritis

Suggested Readings
• American Academy of Pediatrics. Staphylococcus aureus. In: Kimberlin DW, Barnett
ED, Lynfield R, Sawyer MH, eds. Red Book: 2021–2024 Report of the Committee on
Infectious Diseases. 32nd ed. American Academy of Pediatrics; 2021. Accessed
September 1, 2022. Red Book Online .
• Arnold JC, Bradley JS. Osteoarticular infections in children. Infect Dis Clin North Am.
2015;29(3):557-574. doi:10.1016/j.idc.2015.05.012.
• Donaldson N, Sanders J, Child J, Parker S. Acute hematogenous bacterial osteoarticular
infections in children. Pediatr Rev. 2020;41(3):120-136. doi:10.1542/pir.2018-0201.
• Herman MJ, Martinek M. The limping child. Pediatr Rev. 2015;36(5):184-195; quiz 196-
167. doi:10.1542/pir.36-5-184.
• Yee-Guardino S, Goldfarb J. Septic arthritis. In: McInerny TK, Adam HM, Campbell
DE, DeWitt TG, Foy JM, Kamat DM, eds. American Academy of Pediatrics Textbook of
Pediatric Care. American Academy of Pediatrics; 2021:chap 328. Accessed September 1,
2022. Pediatric Care Online.

American Academy of Pediatrics 815


PREP® Self-Assessment PREPSA 2023

Question 243
A 14-year-old adolescent girl is seen in the emergency department for evaluation of worsening
epigastric abdominal pain of 6 hours’ duration. She has had several episodes of nonbilious,
nonbloody emesis and states that deep inspiration is difficult due to pain. She denies dysphagia,
diarrhea, and constipation. She has had recurrent episodes of similar, but less severe, epigastric
pain over the past month. The adolescent has had a history of dysmenorrhea since her onset of
menses at age 13 years, for which she takes ibuprofen a few times monthly. She denies the use of
any other medication. She had an appendectomy for acute appendicitis at age 8 years.
The girl is afebrile with a heart rate of 100 beats/min, respiratory rate of 18 breaths/min, weight
of 80 kg, height of 155 cm, and a body mass index of 32 kg/m2. She appears uncomfortable but
not in acute distress. There is scleral icterus. Her abdomen is soft, nondistended, and without
hepatosplenomegaly. There is tenderness to palpation in the epigastrium, and palpation of the
right upper quadrant during inspiration results in worsening pain. Her extremities are warm, well
perfused, and without edema.

Of the following, the factor MOST likely contributing to this girl’s underlying condition is
A. age of menarche
B. nonsteroidal anti-inflammatory drug use
C. obesity
D. previous appendectomy

American Academy of Pediatrics 816


PREP® Self-Assessment PREPSA 2023

Correct Answer: C
The adolescent in the vignette has abdominal pain, a positive Murphy sign (pain associated with
palpation of the right upper quadrant during inspiration), and scleral icterus; this clinical scenario
is suggestive of choledocholithiasis. Of the response options, obesity is the factor that places the
girl at risk for the development of cholelithiasis. Age of menarche, use of nonsteroidal anti-
inflammatory drugs, and history of previous appendectomy are not risk factors for the
development of cholelithiasis.

Children with cholelithiasis can be asymptomatic or can develop symptoms including sharp and
severe right upper quadrant pain that may radiate to the right shoulder (most commonly occuring
after eating fatty foods), vomiting, and nausea. Ultrasonography is the imaging modality of
choice to diagnose cholelithiasis. Management of symptomatic cholelithiasis includes surgical
consultation with consideration for elective cholecystectomy.

Risk factors for cholelithiasis in children include:


• Congenital anomalies of the gallbladder
• Cystic fibrosis
• Gilbert syndrome
• Hemolytic anemia
• Ileal disease or history of ileal resection
• Medication use (eg, ceftriaxone)
• Obesity
• Progressive familial intrahepatic cholestasis (PFIC) type 2
• Prolonged parenteral nutrition use in critically ill preterm infants

Choledocholithiasis, a gallstone present in the common bile duct, presents with right upper
quadrant pain (similar to cholelithiasis), jaundice (elevated conjugated serum bilirubin), and even
acholic stools. The presence of jaundice in a child with abdominal pain should prompt
consideration for choledocholithiasis. Acute pancreatitis can also ensue, evidenced by epigastric
pain and an elevated lipase level. The presence of fever and/or elevated white blood cell count
raises suspicion for acute cholangitis. Ultrasonography is the initial imaging modality of choice
to make this diagnosis. However, the presence of bowel gas can interfere with the detection of a
distal common bile duct stone; thus, magnetic resonance cholangiopancreatography may be
indicated in children when ultrasonography demonstrates dilation of the common bile duct
without evidence of cholelithiasis. Children with choledocholithiasis warrant gastroenterology
and pediatric surgery consultations.

Acute cholecystitis may occur with or without cholelithiasis (acalculous cholecystitis). Most
commonly, a gallstone becomes impacted in the cystic duct (before entering the common bile
duct) leading to gallbladder inflammation. Signs and symptoms of cholecystitis include right
upper quadrant pain and tenderness (with a positive Murphy sign), nausea, and vomiting. Fever
and an elevated white blood cell count may be present. The diagnosis is confirmed with
ultrasonography, demonstrating a thickened gallbladder wall, an enlarged gallbladder, and
cholelithiasis.
American Academy of Pediatrics 817
PREP® Self-Assessment PREPSA 2023

Management includes bowel rest, consideration for intravenous antibiotics, and cholecystectomy
after the acute phase of illness. The presentation of acute acalculous cholecystitis (AAC) is
similar to that seen in children with gallstones. The diagnosis is confirmed with ultrasonography
demonstrating a thickened gallbladder wall, gallbladder sludge, and pericholecystic fluid. Risk
factors for AAC include severe infection or an inflammatory state (eg, sepsis, trauma, systemic
vasculitis). Management of AAC includes bowel rest, nutritional support, intravenous antibiotics,
and treatment of the underlying condition; cholecystectomy is rarely indicated.

PREP Pearls
• Cholelithiasis can present as postprandial right upper quadrant pain, vomiting, and
nausea; obesity is a risk factor for the development of gallstones in children.
• Complications of cholelithiasis include cholecystitis, choledocholithiasis, and acute
pancreatitis.
• Choledocholithiasis should be considered in children with right upper quadrant pain,
nausea, vomiting, and jaundice. The presence of fever should raise concern for acute
cholangitis.

ABP Content Specifications(s)


• Identify the risk factors associated with the development of cholelithiasis
• Recognize the clinical features associated with choledocholithiasis
• Recognize the clinical features associated with cholecystitis

Suggested Readings
• Goldman DA. Gallbladder, gallstones, and diseases of the gallbladder in children. Pediatr
Rev. 2020;41(12):623-629. doi:10.1542/pir.2019-0077.
• Rothstein DH, Harmon CM. Gallbladder disease in children. Semin Pediatr Surg.
2016;25(4):225-231. doi:10.1053/j.sempedsurg.2016.05.005.

American Academy of Pediatrics 818


PREP® Self-Assessment PREPSA 2023

Question 244
A 4-year-old girl is brought to the clinic for evaluation of a wet-sounding, harsh cough. The girl
has a 3-day history of clear rhinorrhea and non-productive cough without fever or respiratory
distress. The mother has been giving her daughter antibiotics left over from a sibling’s ear
infection and would like her daughter to continue antibiotics because she is worried the illness
will progress.

Review of the girl’s medical record reveals frequent clinic visits for mild viral illnesses. She has
never been diagnosed with pneumonia or asthma. She was born at 27 weeks’ gestation, required
a 109-day hospitalization in the neonatal intensive care unit, and was discharged on oxygen via
nasal cannula, which was discontinued at age 6 months. Her immunizations are up to date, and
she has normal growth and development.

The girl’s vital signs are normal for age. Her physical examination findings are unremarkable
except for mild, clear rhinorrhea.

Of the following, the MOST appropriate next step in this girl’s management is to
A. elicit the mother’s specific concerns regarding her daughter’s health
B. obtain a chest radiograph to evaluate for underlying lung pathology
C. prescribe antibiotics to treat bacterial pneumonia
D. provide reassurance and recommend supportive care

American Academy of Pediatrics 819


PREP® Self-Assessment PREPSA 2023

Correct Answer: A
The girl in the vignette is experiencing vulnerable child syndrome (VCS), first described in 1964
as a condition in which a child is perceived as having a higher risk of medical, behavioral, or
developmental problems than is warranted due to the child experiencing a serious or life-
threatening event or medical condition in the past. The girl’s prematurity and prolonged neonatal
intensive care unit admission are risk factors for VCS. Her neonatal history and history of
frequent office visits for mild viral illness, in addition to her mother’s request for antibiotics to
prevent illness progression in the context of the girl’s mild symptoms and relatively
unremarkable physical examination findings, make VCS likely. Of the response choices, eliciting
the mother’s specific concerns about her daughter’s health, including her perception of her
daughter’s vulnerability to illness, is the most appropriate next step in management. Laboratory
testing, imaging studies, or a prescription of antibiotics, are not warranted. Reassurance and
supportive care may also be part of the management plan, but the best next step would be to
elicit concerns and discuss the perception of the child’s vulnerability to illness.

Vulnerable child syndrome is a maladaptive family dynamic. The child’s perceived vulnerability
can have long-term, negative psychological effects for both the child and the parent. Vulnerable
child syndrome may result in overuse of medical services, unnecessary medical interventions,
inappropriate use of medications, problems with parent–child attachment, and behavioral and
developmental problems in children.

A variety of risk factors may predispose families to develop VCS (eg, prematurity, history of
feeding problems, maternal history of miscarriages). Pediatricians should approach families with
VCS in a trauma-informed and empathetic manner while simultaneously challenging the
erroneous beliefs through rapport building, establishing a trusting relationship, and creating a
safe setting so that families feel respected while being challenged.

Acknowledging and validating past experiences and fears is the first step, followed by clear
communication regarding any risk associated with the prior event, and providing tools for
parents to use when concerns develop.

PREP Pearls
• The vulnerable child syndrome develops when a child is inappropriately perceived as
being at increased risk for illness or harm due to a previous health threat.
• Vulnerable child syndrome results in maladaptive family behaviors within the family,
including overuse of medical services, unnecessary medical interventions, inappropriate
use of medications, problems with parent–child attachment, and behavioral and
developmental problems in children.
• When vulnerable child syndrome is suspected, pediatricians should communicate clearly
and directly about the effect of the prior threat to health on the family’s perception of the
child’s current health status.

American Academy of Pediatrics 820


PREP® Self-Assessment PREPSA 2023

ABP Content Specifications(s)


• Provide anticipatory guidance to prevent vulnerable child syndrome
• Understand factors predisposing to vulnerable child syndrome

Suggested Readings
• Bernbaum JC. Follow-up care of the graduate from neonatal intensive care. In: McInerny
TK, Adam HM, Campbell DE, DeWitt TG, Foy JM, Kamat DM, eds. American
Academy of Pediatrics Textbook of Pediatric Care. American Academy of Pediatrics;
2021:chap 114. Accessed September 1, 2022. Pediatric Care Online.
• Chambers PL, Mahabee-Gittens EM, Leonard AC. Vulnerable child syndrome, parental
perception of child vulnerability, and emergency department usage. Pediatr Emerg Care.
2011;27(11):1009-1013. doi:10.1097/PEC.0b013e318235bb4f.
• Goodpasture M, Sinal SH. Münchausen syndrome by proxy: medical child abuse. In:
McInerny TK, Adam HM, Campbell DE, DeWitt TG, Foy JM, Kamat DM, eds.
American Academy of Pediatrics Textbook of Pediatric Care. American Academy of
Pediatrics; 2021:chap 292. Accessed September 1, 2022. Pediatric Care Online .
• Horwitz SMC, Storfer-Isser A, Kerker BD, et al. A model for the development of
mothers' perceived vulnerability of preterm infants. J Dev Behav Pediatr.
2015;36(5):371-380. doi:10.1097/DBP.0000000000000173.
• Pearson SR, Boyce WT. Consultation of the specialist: the vulnerable child syndrome.
Pediatr Rev. 2004;25(10):345-349. doi:10.1542/pir.25-10-345.
• Schmitz K. In brief: vulnerable child syndrome. Pediatr Rev. 2019;40(6):313-315.
doi:10.1542/pir.2017-0243.

American Academy of Pediatrics 821


PREP® Self-Assessment PREPSA 2023

Question 245
A 7-year-old girl is brought to the office by her foster mother to establish care. The girl requires
constant supervision and assistance with most activities of daily living. Her foster mother
reports, “She acts like my 4-year-old. I have to help dress her and she never sits still.” She is
struggling academically; a full educational evaluation is scheduled.
During the visit, the girl is very active and has difficulty remaining seated. She is able to provide
her full name and age and speaks in 3- to 4-word sentences. Her height and weight are below the
10th percentile for age. The girl’s physical examination findings are remarkable for short
palpebral fissures, a thin upper lip, and a smooth philtrum. The remainder of her physical
examination findings are normal.

Of the following, the MOST likely historical finding for this girl is maternal
A. alcohol use
B. anticonvulsant medication use
C. cytomegalovirus infection
D. exposure to cat feces

American Academy of Pediatrics 822


PREP® Self-Assessment PREPSA 2023

Correct Answers: A
The girl in the vignette has impairments in cognitive and adaptive functioning consistent with an
intellectual disability and facial features characteristic of fetal alcohol syndrome. The following
3 criteria must be met to diagnose fetal alcohol syndrome:
• Prenatal and/or postnatal growth deficiency
• Characteristic facial features including reduced palpebral fissure length, smooth philtrum,
and thin upper lip border
• Neurological or neurobehavioral deficits

When some but not all of these criteria are met, a diagnosis of fetal alcohol spectrum disorder is
appropriate. All children with prenatal alcohol exposure are at risk for neurobehavioral disorders
including attentiondeficit/hyperactivity disorder, anxiety, and mood disorders. They are also at
higher risk for substance use and suicide.

The etiologies of intellectual disability include genetic disorders (eg, trisomy 21, fragile X
syndrome) and environmental causes (eg, teratogens and perinatal infections). A thorough
history and physical examination should be conducted for all children with or suspected of
having intellectual disability. A detailed prenatal, birth, family, medical, and psychosocial
history should be obtained. A complete physical examination, with special attention to skin
findings, unusual facies, skeletal abnormalities, head circumference, and growth can support and
differentiate among the possible causes. For many individuals with intellectual disability a
specific etiology is not identified.

Alcohol exposure is the most common teratogenic cause of intellectual disability. Prenatal
exposure to specific anticonvulsant drugs has also been associated with birth defects and
intellectual disability. Phenytoin is associated with fetal hydantoin syndrome, which is
characterized by growth deficiency, cleft lip and palate, underdeveloped fingernails and toenails,
and developmental delay. Use of valproate during pregnancy increases the risk for neural tube
defects, cleft lip and palate, and developmental delay. The girl in the vignette does not have any
physical findings associated with maternal anticonvulsant use in pregnancy.

Intrauterine infections associated with intellectual disability include toxoplasmosis, rubella,


cytomegalovirus, and herpes simplex virus (TORCH infections). Affected neonates may have
retinopathy, intracranial calcifications, microcephaly, macrocephaly due to hydrocephalus,
sensorineural hearing loss, and rash or skin lesions (eg, a “blueberry muffin rash” with blue or
purple marks on the skin). The infant in the vignette does not have any of the stigmata of
TORCH infection. Congenital toxoplasmosis infection occurs with maternal primary infection
during pregnancy, most commonly due to handling of cat feces or cat litter, or eating raw or
undercooked meat or eggs. Congenital toxoplasmosis may not be apparent immediately
following delivery; subclinical or late manifestations may include jaundice, retinitis,
hydrocephalus, intracranial calcifications, and seizures.

American Academy of Pediatrics 823


PREP® Self-Assessment PREPSA 2023

PREP Pearls
• Etiologies of intellectual disability include genetic disorders (eg, trisomy 21, fragile X
syndrome), teratogen exposure (eg, alcohol, anticonvulsant drugs), and perinatal
infections (eg, toxoplasmosis, rubella, cytomegalovirus).
• Alcohol is the most common teratogenic cause of intellectual disability; intellectual
disability is one of the characteristic features of fetal alcohol spectrum disorder.
• A thorough history and physical examination can help differentiate among the causes of
intellectual disability; for many individuals with intellectual disability a specific etiology
is not identified.

MOCA-Peds Objective
• Understand the clinical features, approach to evaluation, and differential diagnosis of
fetal alcohol spectrum disorder.
• ABP Content Specifications(s)
• Identify common teratogenic causes of intellectual disabilities
• Identify common infectious causes of intellectual disabilities

Suggested Readings
• Flore LA. Fetal alcohol spectrum disorders. In: McInerny TK, Adam HM, Campbell DE,
DeWitt TG, Foy JM, Kamat DM, eds. American Academy of Pediatrics Textbook of
Pediatric Care. American Academy of Pediatrics; 2021:chap 250. Accessed September 1,
2022. Pediatric Care Online.
• Ostrander B, Bale JF. Congenital and perinatal infections. Handb Clin Neurol.
2019;162:133-153. doi:10.1016/B978-0444-64029-1.00006-0.
• Phelps RA, Cohen WI. Intellectual disability. In: McInerny TK, Adam HM, Campbell
DE, DeWitt TG, Foy JM, Kamat DM, eds. American Academy of Pediatrics Textbook of
Pediatric Care. American Academy of Pediatrics; 2021:chap 278. Accessed September 1,
2022. Pediatric Care Online.
• Purugganan O. Intellectual disabilities. Pediatr Rev. 2018;39(6):299-309.
doi:10.1542/pir.2016-0116.
• Williams JF, Smith VC; Committee on Substance Abuse. Fetal alcohol spectrum
disorders. Pediatrics. 2015;136(5):e1395e1406. doi:10.1542/peds.2015-3113.

American Academy of Pediatrics 824


PREP® Self-Assessment PREPSA 2023

Question 246
A 16-year-old adolescent boy is seen for evaluation of testicular swelling. He was born at 28
weeks’ gestation and has cerebral palsy with spastic quadriplegia and a seizure disorder. At 9
months of age he underwent a bilateral inguinal hernia repair with right orchiopexy. His mother
initially noticed right testicular swelling 2 months ago during a diaper change, and the testicle
has continued to increase in size. He does not have pain, fever, vomiting, diarrhea, hematuria, or
urinary symptoms. Physical examination findings are significant for spasticity of his extremities,
nontender right scrotal swelling, and a 4-cm, firm, nontender mass in the right testicle. The
remainder of his physical examination findings are unremarkable.

Of the following, the risk factor MOST likely to be associated with this boy’s findings is
A. cerebral palsy
B. cryptorchidism
C. seizure disorder
D. urinary incontinence

American Academy of Pediatrics 825


PREP® Self-Assessment PREPSA 2023

Correct Answer: B
The adolescent in the vignette has a painless testicular mass that is increasing in size. Testicular
cancer is the most likely diagnosis. Cryptorchidism is a major risk factor for the development of
testicular cancer. Infants with cryptorchidism, such as the boy in the vignette who had an
orchiopexy at 9 months of age, are up to 8 times more likely to develop testicular cancer even
after the testis is placed securely in the scrotum. Cryptorchidism is the risk factor most likely
associated with this adolescent’s testicular cancer. It is important to note that premature infants
have a high incidence of cryptorchidism. Cerebral palsy, seizure disorder, and urinary
incontinence are not associated with an increased risk of developing testicular cancer.

Cryptorchidism is also more common in infants with trisomy 21 and other genetic syndromes.
Individuals with a disorder of sexual differentiation and an intraabdominal gonad are at increased
risk of developing gonadal cancer if Y chromosome material is present. Additional risk factors
for testicular cancer include a family history of testicular cancer, HIV infection, and Klinefelter
syndrome.

The evaluation of a painless testicular mass includes Doppler ultrasonography. The differential
diagnosis of a testicular mass and expected findings on ultrasonography are outlined in Item
C246.

Additional imaging to assess the primary mass or evaluate for metastatic disease may include
magnetic resonance imaging or computed tomography of the abdomen and pelvis and chest
radiography.

Serum tumor marker levels (eg, α-fetoprotein and human chorionic gonadotropin) can be helpful
in the diagnosis of testicular cancer. They may be elevated in yolk sac tumors but not teratomas.

The treatment of testicular cancer is resection (orchiectomy). For nonmalignant masses,


testicular-sparing procedures may be considered. Adjuvant chemotherapy may be indicated,
depending on the stage, histologic subtype, and site of origin of the tumor.

PREP Pearls
• A painless testicular mass is concerning for testicular cancer.
• Cryptorchidism is associated with an increased risk of testicular cancer, even after
orchiopexy.
• Doppler ultrasonography should be obtained to evaluate a testicular mass.

ABP Content Specifications(s)


• Recognize risk factors associated with testicular cancer
• Plan the appropriate evaluation of a testicular mass

American Academy of Pediatrics 826


PREP® Self-Assessment PREPSA 2023

Suggested Readings
• Blair RJ. Testicular and scrotal masses. Pediatr Rev. 2014;35(10):450-451.
doi:10.1542/pir.35-10-450.
• Lanzkowsky P, Lipton J, Fish JD. Lanzkowsky’s Manual of Pediatric Hematology and
Oncology. 6th ed. Elsevier Inc; 2016:555-568.
• Palmer LS. Scrotal swelling and pain. In: McInerny TK, Adam HM, Campbell DE,
DeWitt TG, Foy JM, Kamat DM, eds. American Academy of Pediatrics Textbook of
Pediatric Care. American Academy of Pediatrics; 2021:chap 190. Accessed September 1,
2022. Pediatric Care Online.
• Testicular tumors: what radiologists need to know--differential diagnosis, staging, and
management. Radiographics. 2015;35(2):400-415. doi:10.1148/rg.352140097.
• Wibmer AG, Vargas HA. Imaging of testicular and scrotal masses: the essentials. In:
Hodler J, Kubik-Huch RA, von Schulthess GK, eds. Diseases of the Abdomen and Pelvis
2018-2021: Diagnostic Imaging - IDKD Book [Internet]. Springer; 2018:chap 24.
doi:10.1007/978-3-319-75019-4_24.
• Wu WJ, Gitlin JS. The male genital system. Pediatr Rev. 2020;41(3):101-111.
doi:10.1542/pir.2017-0316.

American Academy of Pediatrics 827


PREP® Self-Assessment PREPSA 2023

Question 247
A 4-month-old infant is brought to the emergency department after 2 episodes in the last 24
hours of color change, limpness, and cessation of breathing. His mother gave rescue breaths
during the last episode and reports that he was limp for more than a minute. There was a similar
but less frightening episode 1 week ago that resolved when she blew in his face. He has been
more fussy than usual in the last few days but has not shown any other signs of illness.
The infant was born at term after an uncomplicated pregnancy. His newborn screening results
were normal. At each of the recommended routine health supervision visits he was noted to be
growing and developing well. The infant breastfeeds or takes expressed breast milk. He is not in
daycare and stays with his mother’s boyfriend while she works an evening shift.

On physical examination, the infant is alert and breathing normally. His temperature is 37°C,
heart rate is 140 beats/min, respiratory rate is 25 breaths/min, and oxygen saturation is 99% in
room air by pulse oximetry. There is a bruise on his right upper arm that his mother attributes to
a bite from an older sibling. The remainder of his physical examination findings are normal.

Of the following, the BEST next step in this infant’s management is to


A. obtain computed tomography scan of the head
B. obtain a nasal swab for respiratory syncytial virus
C. order home apnea monitoring for 30 days
D. reassure the mother and schedule follow-up for the next day

American Academy of Pediatrics 828


PREP® Self-Assessment PREPSA 2023

Correct Answer: A
The infant in the vignette has had 3 episodes that meet criteria for a brief resolved unexplained
event (BRUE). Because of the recurrence, duration of >1 minute, and concerning history and
physical examination findings, he does not meet criteria for lower-risk BRUE and further
evaluation is indicated. Reassurance and follow-up is not appropriate. Potential life-threatening
etiologies for his presentation include central nervous system (CNS) abnormalities and abusive
head trauma. Thus, the best next step in his management is to obtain computed tomography of
the head. A high index of suspicion for nonaccidental trauma should be maintained. Concerning
findings include the infant’s fussiness and the bruise on his arm in the context of an otherwise
unexplained recurrent potentially life-threatening event.

Nothing in the infant’s history or physical examination suggests a viral respiratory infection, so a
nasal swab for respiratory syncytial virus is not indicated. Home apnea monitoring is not
appropriate at this time. Further evaluation would be required to identify an underlying
abnormality for which home monitoring is appropriate.

Up to 43% of all infants may have a BRUE in the first year after birth, defined as having one or
more of the following components:
• Cyanosis or pallor
• Absent, decreased, or irregular breathing
• Marked change in alertness or level of responsiveness

A thorough history, social history, and physical examination are required for infants with a
suspected diagnosis of BRUE. For those meeting the American Academy of Pediatrics criteria
for lower-risk BRUE, reassurance and follow-up are appropriate. Criteria for lower-risk BRUE
include (all criteria must be met):
• Age >60 days
• Gestational age ≥32 weeks
• Single episode without recurrence (first ever and not occurring in clusters)
• Duration <1 minute
• No cardiopulmonary resuscitation required by a trained medical professional
• No concerning historical features
• No concerning physical examination findings

Infants not meeting lower-risk criteria should be admitted for monitoring and further assessment.
Some infants diagnosed with a lower-risk BRUE may later be diagnosed with a serious
condition, most often seizure disorder, abusive head trauma, or airway anomaly (eg,
laryngomalacia).

Apnea or changes in respiratory pattern are a major component of BRUE and of great concern to
parents and caregivers. Apnea is defined as a cessation of airflow which may be central,
obstructive, or mixed. Pathologic apnea lasts more than 20 seconds and may be associated with
changes in color or tone, or with bradycardia.

American Academy of Pediatrics 829


PREP® Self-Assessment PREPSA 2023

Periodic breathing is often confused with apnea; it is characterized by 3 or more respiratory


pauses of >3 seconds with <20 seconds of normal respiration between the pauses. Periodic
breathing is common in neonates and represents the effect of neurologic immaturity on
respiratory control. It is not associated with apneas of >20 seconds or a change in color or tone.
Apnea of prematurity occurs in premature infants and is often associated with color change and
bradycardia. It is common in neonates born at <29 weeks’ gestation and weighing <1,000 g, but
also occurs in later-gestation neonates. It usually resolves spontaneously by 42 weeks
postconceptual age. Caffeine is commonly used as a respiratory stimulant to treat apnea of
prematurity.

Other causes of apnea and dysregulated breathing in infants include seizure disorders, CNS
anomalies, and airway anomalies. Nasal and upper airway anomalies are usually symptomatic in
the first few days after birth. Neonates are obligate nose breathers and nasal obstruction can be a
life-threatening emergency. Inborn errors of metabolism, electrolyte abnormalities, and
infections may also contribute to apnea in neonates. Central nervous system abnormalities
usually present as central apnea, with diminished or absent respiratory drive. Airway
obstructions are associated with obstructive apnea. Specific evaluation and management should
be driven by the prenatal and postnatal history and findings on physical examination, including
careful airway, cardiac, and neurologic examination.

In older infants, respiratory infections and CNS trauma are more likely etiologies of apnea than
are congenital anomalies. Apnea and respiratory dysregulation are most apparent during sleep.
In preschool and school-aged children, obstructive sleep apnea (OSA) from tonsillar and/or
adenoidal hypertrophy is the most common etiology of apnea. In children with obesity, OSA can
occur in the absence of enlarged tonsils or adenoids. Apnea and respiratory dysfunction can be
indolent and may be uncovered only with careful questioning about sleep pattern, snoring,
morning headaches, fatigue, or behavior symptoms. Children with OSA can have learning
difficulties, attention-deficit/hyperactivity disorder, and other behavioral manifestations of poor
sleep.

Children with underlying neurologic abnormalities associated with poor pharyngeal airway tone
that worsens with sleep (eg, muscular dystrophies) have a high likelihood of OSA even without
the anatomic burden of enlarged upper airway lymphoid tissue. Additionally, children with
achondroplasia, trisomy 21, Prader-Willi syndrome, and craniofacial syndromes are at high risk
for sleep disordered breathing, including apnea. They are also at risk for an abnormal central
drive for breathing when sleeping and combined central and obstructive apnea.

Formal testing with overnight polysomnography is required to quantitate the severity of OSA
and differentiate obstructive from mixed central and obstructive sleep apnea. For children with
hypertrophy of the tonsils and/or adenoids, adenotonsillectomy is the first-line treatment for
OSA. Children with severe OSA, underlying neurologic disorders, skeletal dysplasias,
pulmonary hypertension, craniofacial anomalies, hypotonia, and marked obesity are at risk for
respiratory compromise following adenotonsillectomy and must be closely monitored post-
procedure. They should have a follow-up polysomnography several weeks after the procedure to
American Academy of Pediatrics 830
PREP® Self-Assessment PREPSA 2023

document improvement and/or resolution of OSA. Children whose OSA is not amenable to
adenotonsillectomy or not adequately relieved after the procedure, may need nocturnal
respiratory support with continuous positive airway pressure (CPAP). Bilevel positive airway
pressure (BiPAP) with a back-up guaranteed respiratory rate may be required for children with
mixed central and obstructive apnea.

PREP Pearls
• For infants with new or recurrent episodes of apnea with color change and/or loss of tone,
without evidence of an underlying condition, a high index of suspicion must be
maintained for non-accidental head trauma.
• The most common cause of obstructive sleep apnea in children is tonsillar and/or
adenoidal hypertrophy.
• Children with neuromuscular disorders, skeletal dysplasias, and craniofacial anomalies
are at high risk for complicated obstructive sleep apnea.

ABP Content Specifications(s)


• Plan appropriate management for apnea of various etiologies
• Plan the appropriate clinical and diagnostic evaluation of apnea of various etiologies

Suggested Readings
• Behnam-Terneus M, Clements M. SIDS, BRUE and safe sleep guidelines. Pediatr Rev.
2019; 40(9):443-453. doi:10.1542/pir.2017-0259.
• Gipson K, Lu M, Kinane TB. Sleep disordered breathing in children. Pediatr Rev.
2019;40(1):3-12. doi:10.1542/pir.2018-
• Katz SL. Assessment of sleep disordered breathing in pediatric neuromuscular diseases.
Pediatrics. 2009;123 Suppl 4:S222-S225. doi:10.1542/peds.2008-2952E.
• Rafe K, Blaisdell CJ. Apparent life-threatening event. In: McInerny TK, Adam HM,
Campbell DE, DeWitt TG, Foy JM, Kamat DM, eds. American Academy of Pediatrics
Textbook of Pediatric Care. American Academy of Pediatrics; 2021:chap 217. Accessed
September 1, 2022. Pediatric Care Online.
• Tieder JS, Bonkowsky JL, Etzel RA, et al; Subcommittee on Apparent Life-Threatening
Events. Brief resolved unexplained events (formerly apparent life-threatening events) and
evaluation of lower risk infants. Pediatrics. 2016;137(5):e20160590.
doi:10.1542/peds.2016-0590.

American Academy of Pediatrics 831


PREP® Self-Assessment PREPSA 2023

Question 248
A 4-month-old infant who is new to the practice is seen for a health supervision visit. This visit
is his first follow-up since leaving the newborn nursery. The infant was born at term without
complications. His physical examination findings are significant only for the extremity
abnormality shown in Item Q248.

Item Q248: Findings for the infant described in the vignette. Courtesy of R. Carl

Of the following, the BEST next management step for this infant is to
A. have him return for reevaluation in 1 month
B. obtain bilateral foot and ankle radiographs
C. refer him to a pediatric orthopedic surgeon
D. refer him to physical therapy

American Academy of Pediatrics 832


PREP® Self-Assessment PREPSA 2023

Correct Answer: C
The infant in the vignette has congenital idiopathic talipes equinovarus, commonly called
clubfoot (Item C248A). The best next step in management is to refer him to a pediatric
orthopedic surgeon for treatment. This condition will not improve without treatment, which
should begin as soon as possible, so having him return for reevaluation in 1 month is not the
preferred response. Imaging is not routinely recommended for this condition, thus foot and ankle
radiographs would not be the appropriate next step. Physical therapy alone will not correct the
deformity; timely treatment by a pediatric orthopedic surgeon is required.

Congenital idiopathic talipes equinovarus is the most common congenital foot deformity that
requires treatment. The mnemonic CAVE is used to describe the features: cavus (high arch)
(Item C248B), adducted position of the forefoot (Item C248C), hindfoot varus (heel curves in
towards the midline) (Item C248D), and equinus (ankle fixed in a plantarflexed position) (Item
C248E). Treatment should ideally begin in the first month after birth for healthy term infants.
The gold standard for clubfoot care is known as the Ponseti method. This approach involves
serial casting (Item C248F) to stretch the foot from an adducted/inwardly rotated position to an
abducted/outwardly rotated position. After casting, most infants require an Achilles tenotomy, a
minor outpatient procedure to release the Achilles tendon. After surgery, a brace is used to
maintain the corrected position.

Item C248B: Cavus positioning.


Courtesy of BG Smith. Lower extremity disorders in children and adolescents. Pediatr Rev.
2009;30(8):288.

American Academy of Pediatrics 833


PREP® Self-Assessment PREPSA 2023

Item C248C: Metatarsus adductus.


Courtesy of BG Smith. Lower extremity disorders in children and adolescents. Pediatr Rev.
2009;30(8):289

Item C248D: Hindfoot varus positioning.


Courtesy of BG Smith. Lower extremity disorders in children and adolescents. Pediatr Rev.
2009;30(8):288.

Item C248E: Equinus positioning.


Courtesy of BG Smith. Lower extremity disorders in children and adolescents. Pediatr Rev.

American Academy of Pediatrics 834


PREP® Self-Assessment PREPSA 2023

2009;30(8):288.

Item C248F: Typical appearance of an infant clubfoot cast.


Courtesy of R. Carl

The cause of congenital idiopathic talipes equinovarus is unclear. About one-quarter of affected
children have a family history of the condition. Clubfoot is not thought to be due to crowding of
the feet in utero. However, early restriction in fetal movement may play a role for some children.
The deformity can typically be seen on prenatal ultrasonography by 14 weeks’ gestation.
Histologic examination of the muscle in affected children demonstrates abnormal muscle fibers
or connective tissue in the muscles below the knee. These changes are likely secondary to a
combination of genetic, environmental, and/or neurological factors.

PREP Pearls
• Clubfoot is an inward rotational anomaly of the foot at the ankle with an associated
increase in the midfoot crease
• and a shortened calf muscle on the affected side.
• The mnemonic CAVE is used to describe the features of clubfoot: cavus (high arch),
adducted position of the forefoot, hindfoot varus (heel curves in towards the midline),
and equinus (ankle stuck in a plantarflexed position).
• It is important to promptly refer infants with clubfoot to a pediatric orthopedic surgeon.

ABP Content Specifications(s)


• Recognize the clinical findings associated with clubfoot and the need for prompt referral

American Academy of Pediatrics 835


PREP® Self-Assessment PREPSA 2023

Suggested Readings
• Cady R, Hennessey TA, Schwend RM; Section on Orthopedics, Diagnosis and treatment
of idiopathic congenital clubfoot. Pediatrics. 2022;149(2):e2021055555.
doi:10.1542/peds.2021-055555 .
• Clubfoot. In: Sarwark JF, LaBella CR. Pediatric Orthopaedics and Sport Injuries: A
Quick Reference Guide. 3rd ed. American Academy of Pediatrics; 2021:505-510.
• Roye B, Hyman J, Roye D. Congenital idiopathic talipes equinovarus. Pediatr Rev.
2004;25(4):124-130. doi:10.1542/pir.25-4-124.
• Weintraub B. Foot and leg problems. In: McInerny TK, Adam HM, Campbell DE,
DeWitt TG, Foy JM, Kamat DM, eds. American Academy of Pediatrics Textbook of
Pediatric Care. American Academy of Pediatrics; 2021:chap 154. Accessed September 1,
2022. Pediatric Care Online.

American Academy of Pediatrics 836


PREP® Self-Assessment PREPSA 2023

Question 249
A 14-year-old adolescent girl is brought to the emergency department after she sustained a head
injury when she collided with another player while playing soccer. She had a brief loss of
consciousness at the time of the event and later developed somnolence, headache, and vomiting.
In the emergency department, other than a tired appearance, the adolescent’s physical
examination findings are normal. Head computed tomography (CT) scan without contrast
demonstrates a hyperdensity in the right frontal lobe with no mass effect. Brain magnetic
resonance imaging and angiography, performed to further evaluate the frontal lobe CT finding,
demonstrate a tightly packed tangle of blood vessels, with clearly visualized flow voids, within
the right frontal lobe without evidence of hemorrhage.

Of the following, the MOST likely diagnosis for this adolescent is


A. arachnoid cyst
B. arterio-venous malformation
C. cavernous malformation
D. meningioma

American Academy of Pediatrics 837


PREP® Self-Assessment PREPSA 2023

Correct Answer: B
The adolescent in the vignette has imaging findings consistent with a cerebral arterio-venous
malformation (AVM). Arterio-venous malformations are congenital high-flow vascular
malformations in which there is a direct connection between the arteries and veins and no
intervening capillary bed. The majority of AVMs are caused by sporadic pathologic genetic
variants resulting in dysregulated vasculogenesis. However, there are genetic syndromes
associated with development of AVMs, most commonly hereditary hemorrhagic telangiectasias.

The majority of cerebral AVMs are diagnosed incidentally on imaging performed for an
unrelated reason but can also be discovered during the diagnostic evaluation for seizures,
headaches, or progressive focal neurological deficits or acutely, in the setting of intracranial
hemorrhage. There is a bimodal peak in the age at diagnosis: childhood (age 010 years) and
young to mid-adulthood (age 30 to 50 years). Manifestations of symptomatic AVMs vary based
on age, location, size, and the features of the lesion itself.

The diagnosis of an AVM is made on neuroimaging. Initial identification is typically on brain


magnetic resonance imaging (MRI) or head computed tomography (CT) scan. Angiographic
imaging (magnetic resonance angiography or CT angiography) will provide a more detailed
image of the AVM size and features. Digital subtraction angiography confirms the diagnosis and
aids in treatment planning and assessing prognosis. Surgical or interventional procedures are
indicated in the setting of hemorrhage due to a ruptured AVM. For non-ruptured AVMs, based
on data in the adult population, conservative medical management is recommended for the
prevention of death or symptomatic stroke (Mohr).

An arachnoid cyst is a fluid-filled sac, often congenital, that occurs in the arachnoid membrane.
Arachnoid cysts are common and often discovered incidentally on imaging. Depending on the
location of the cyst, symptomatic children can present with headache, seizures, developmental
delay, increased intracranial pressure, or “bobble-head syndrome.”

Cavernous malformations (CM) are slow-flow vascular malformations consisting of a tangled,


enlarged, abnormal capillary bed without intervening brain parenchyma. Cavernous
malformations may be familial or sporadic. Hemorrhage may occur. Common symptoms of CM
include headache, seizures, vision changes, and balance problems. On MRI, cavernous
malformations have a “popcorn” or “mulberry-like” cluster of tiny blood vessels with
surrounding hemosiderin deposits. Unlike AVMs, there is no intervening brain parenchyma. The
imaging appearance of the child in the vignette’s vascular lesion is not consistent with a CM.

Meningiomas are tumors of the meninges. They are classified as benign, atypical, or malignant.
Symptoms vary based on tumor location. On MRI, meningiomas appear as dural-based, extra-
axial masses that are well circumscribed and homogeneous.

American Academy of Pediatrics 838


PREP® Self-Assessment PREPSA 2023

PREP Pearls
• Arterio-venous malformations are congenital, high-flow, vascular malformations in
which there is a direct connection between the arteries and veins and no intervening
capillary bed.
• The majority of arterio-venous malformations are diagnosed incidentally, during a
diagnostic evaluation for seizures, headaches, or progressive focal neurological deficits,
or acutely, in the setting of intracranial hemorrhage.
• Manifestations of symptomatic arteriovenous malformations vary based on age, location,
size, and the features of the lesion itself.

ABP Content Specifications(s)


• Recognize the clinical features associated with arteriovenous malformations in patients of
various ages

Suggested Readings
• Lee GL, Trevino J, Andrasik W, Chen AYY. Vascular anomalies. In: McInerny TK,
Adam HM, Campbell DE, DeWitt TG, Foy JM, Kamat DM, eds. American Academy of
Pediatrics Textbook of Pediatric Care. American Academy of Pediatrics; 2021:chap 95.
Accessed September 1, 2022. Pediatric Care Online.
• Mohr JP, Overbey JR, Hartmann A, et al; ARUBA co-investigators. Medical
management with interventional therapy versus medical management alone for
unruptured brain arteriovenous malformations (ARUBA): final follow-up of a
multicentre, non-blinded, randomised controlled trial. Lancet Neurol. 2020;19(7):573-
581. doi:10.1016/S1474-4422(20)30181-2.
• Solomon R, Connolly E. Arterio-venous malformations of the brain. N Engl J Med.
2017;376(19):1859-1866. doi:10.1056/NEJMra1607407.

American Academy of Pediatrics 839


PREP® Self-Assessment PREPSA 2023

Question 250
An 11-year-old boy is seen for a health supervision visit. His family history is significant for 2
relatives who suffered myocardial infarctions in their 50s and 2 grandparents with type 2
diabetes mellitus. On physical examination, the boy’s blood pressure is 104/62 mm Hg and body
mass index is 23.8 kg/m2 (96th percentile). He has acanthosis nigricans over the nape of his
neck. His sexual maturity rating is 2 for pubic hair and genital development. The remainder of
his physical examination findings are normal.

A fasting laboratory evaluation reveals the following:


Laboratory Test Result
Total cholesterol 220 mg/dL (5.7 mmol/L)
Triglycerides 120 mg/dL (1.4 mmol/L)
High-density lipoprotein (HDL) cholesterol 22 mg/dL (0.6 mmol/L)
Low-density lipoprotein (LDL) cholesterol 150 mg/dL (3.9 mmol/L)
Glucose 92 mg/dL (5.1 mmol/L)
Aspartate aminotransferase 22 U/L
Alanine aminotransferase 24 U/L

Of the following, the BEST next step in this boy’s management is to initiate
A. atorvastatin
B. lifestyle modification
C. metformin
D. orlistat

American Academy of Pediatrics 840


PREP® Self-Assessment PREPSA 2023

Correct Answer: B
The boy in the vignette has multiple risk factors for the development of atherosclerotic
cardiovascular disease, including dyslipidemia (high total and low-density lipoprotein [LDL]
cholesterol levels, a borderline high triglyceride level, and a low high-density lipoprotein [HDL]
cholesterol level), obesity (body mass index [BMI] ≥95th percentile for age and sex), evidence of
insulin resistance (acanthosis nigricans), and a family history of type 2 diabetes and myocardial
infarctions at a young age. The best next management step is to initiate therapeutic lifestyle
modification specific to each modifiable risk factor.

Universal lipid screening is recommended once between ages 9 and 11 years and again between
ages 17 and 21 years in order to screen for familial hypercholesterolemia. For children aged 10
years and older with a BMI ≥95th percentile, current clinical guidelines recommend undergoing
laboratory screening with a lipid panel, and fasting glucose, alanine aminotransferase (ALT), and
aspartate aminotransferase (AST) levels. Those in higher-risk categories may require earlier or
more frequent screening. Item 250A outlines cardiovascular risk factors. The boy in the vignette
has a positive family history (myocardial infarction in males <55 years old or females <65 years
old), and 2 moderate-level risk factors (BMI ≥95th to <97th percentile, and an HDL level <40
mg/dL [<1.0 mmol/L]). Once children and adolescents are risk stratified and screened for other
comorbidities, specific goals can be set for lifestyle modification and other treatment modalities.
A treatment algorithm is displayed in Item 250B. Consideration for lipid-lowering medical
therapy is based on cardiovascular risk factors and specific underlying disease processes.
Pharmacologic therapy should not replace lifestyle change; they should be used in tandem.

AS indicates aortic stenosis; CKD, chronic kidney disease; FH, familial hypercholesterolemia; HCM,
hypertrophic cardiomyopathy; IBD, inflammatory bowel disease; JIA, juvenile rheumatoid arthritis;
Lp(a), lipoprotein (a); NAFLD, nonalcoholic fatty liver disease; PCOS, polycystic ovarian syndrome;
SLE, systemic lupus erythematosus; s/p, status post; T1DM, type 1 diabetes mellitus; T2DM, type 2
diabetes mellitus; and zMax, maximum z score at any time during the course of illness.

American Academy of Pediatrics 841


PREP® Self-Assessment PREPSA 2023

Statin therapy (eg, atorvastatin) is recommended for those aged 10 years and older whose LDL
cholesterol level remains above goal after lifestyle modification. For those with a family history
of cardiovascular disease, 1 high-level risk factor, or 2 or more moderate-level risk factors, the
threshold for starting statin therapy is an LDL cholesterol level ≥160 mg/dL (4.1 mmol/L). The
treatment goal is an LDL level <130 mg/dL (3.4 mmol/L). For those at higher risk, the threshold
for statin treatment is an LDL level ≥130 mg/dL (3.4 mmol/L). The treatment goal is an LDL
level <100 mg/dL (2.6 mmol/L). Statins inhibit hydroxymethylglutaryl-coenzyme A (HMG-
CoA) reductase, the ratelimiting enzyme in cholesterol synthesis, resulting in decreased
endogenous production of cholesterol, upregulation of LDL receptors in the liver, and increased
clearance of LDL cholesterol.

Metformin is approved by the US Food and Drug Administration (FDA) for the treatment of type
2 diabetes in individuals aged 10 years and older. It works by decreasing hepatic glucose
synthesis and increasing insulin sensitivity. Some studies have suggested a slight improvement in
BMI. However, this reduction in BMI is not sufficient to significantly reduce cardiovascular risk
and metformin has no direct effect on lipid levels. Although the boy in the vignette does have
evidence of insulin resistance (acanthosis nigricans), he does not have diabetes, so metformin is
not indicated.

Until 2020, orlistat was the only FDA-approved medication for weight loss in children and
adolescents 12 years of age or older (glucagon-like peptide-1 receptor agonists have now been
approved). This intestinal lipase inhibitor reduces absorption of dietary fats. However, the side
effect of oily stools makes medication adherence difficult. The 11-year-old boy in the vignette is
younger than the FDA-approved age for any weight loss medication, so orlistat is not indicated.

American Academy of Pediatrics 842


PREP® Self-Assessment PREPSA 2023

High-risk pediatric populations: risk stratification and treatment.


Risk-stratification and treatment algorithm for high-risk pediatric populations. Directions: Step 1: Risk
stratification by disease process. Step 2: Assess all cardiovascular risk factors. If there are ≥2
comorbidities, assign patient to the next-higher-risk tier for subsequent management. Step 3: Tier-specific
treatment cut points are defined. Step 4: Initial therapy: For High Risk, initial management is therapeutic
lifestyle change plus disease-specific management. For Moderate Risk and At Risk groups, initial
management is therapeutic lifestyle change Step 5: For Moderate and At Risk groups, if goals are not
met, consider medication. *A 1c >7.0% in individuals with diabetes mellitus. %ile indicates percentile for
age and sex; BMI, body mass index; BP, blood pressure; CAD, coronary artery disease; CV,
cardiovascular; CVD, cardiovascular disease; ESRD, end-stage renal disease; FG, fasting glucose; FH,
familial hypercholesterolemia; HgbA1c, hemoglobin A1c; and LDL, low-density lipoprotein

PREP Pearls
• The first line-intervention for children with hyperlipidemia is healthy lifestyle change.
• Pharmacologic therapy with statins is recommended to treat hyperlipidemia in those who
do not achieve low-density lipoprotein cholesterol goals (defined by risk stratification)
after lifestyle modification.

American Academy of Pediatrics 843


PREP® Self-Assessment PREPSA 2023

ABP Content Specifications(s)


• Understand the recommendations of the National Cholesterol Education Program for
Children
• Know the risk factors associated with hypercholesterolemia/hyperlipidemia

Suggested Readings
• Bamba V. Update on screening, etiology, and treatment of dyslipidemia in children. J
Clin Endocrinol Metab. 2014;99(9):3093-3102. doi:10.1210/jc.2013-3860.
• Barlow SE; Expert Committee. Expert committee recommendations regarding the
prevention, assessment, and treatment of child and adolescent overweight and obesity:
summary report. Pediatrics. 2007;120 Suppl 4:S164-S192. doi:10.1542/peds.2007-
2329C.
• de Ferranti SD, Steinberger J, Ameduri R, et al. Cardiovascular risk reduction in high-
risk pediatric patients: a scientific statement from the American Heart Association.
Circulation. 2019;139(13):e603-e634. doi:10.1161/CIR.0000000000000618.
• Expert Panel on Integrated Guidelines for Cardiovascular Health and Risk Reduction in
Children and Adolescents; National Heart, Lung, and Blood Institute. Expert panel on
integrated guidelines for cardiovascular health and risk reduction in children and
adolescents: summary report. Pediatrics. 2011;128 Suppl 5(Suppl 5):S213-S256.
doi:10.1542/peds.20092107C.
• Magge SN, Goodman E, Armstrong SC; Committee on Nutrition; Section on
Endocrinology, Section on Obesity. The metabolic syndrome in children and adolescents:
shifting the focus to cardiometabolic risk factor clustering. Pediatrics.
2017;140(2):e20171603. doi:10.1542/peds.2017-1603.
• Stewart J, McCallin T, Martinez J, Chacko S, Yusuf S. Hyperlipidemia. Pediatr Rev.
2020;41(8):393-402. doi:10.1542/pir.2019-0053.

American Academy of Pediatrics 844


PREP® Self-Assessment PREPSA 2023

Question 251
A 14-year-old adolescent girl is seen in the emergency department for evaluation of lower
abdominal pain. The pain radiates to her lower back and is associated with nausea. She had an
episode of vomiting immediately before coming to the emergency department. Menarche
occurred 6 months ago, and she has had monthly cycles with bleeding lasting 5 to 7 days. Her
last menstrual period began yesterday. She has never had sexual intercourse and denies dysuria
and vaginal discharge. She takes no medications. The girl’s vital signs are normal for age. Her
physical examination findings are significant only for mild tenderness to palpation of the lower
abdomen without rebound or guarding. The result of a urine pregnancy test is negative.

Of the following, the BEST next step in this adolescent’s management is to


A. perform a speculum examination with a chaperone
B. perform transabdominal pelvic ultrasonography
C. prescribe a combined oral contraceptive pill
D. prescribe a nonsteroidal anti-inflammatory medication

American Academy of Pediatrics 845


PREP® Self-Assessment PREPSA 2023

Correct Answers: D
The adolescent in the vignette is experiencing lower abdominal pain during menstruation
consistent with dysmenorrhea. Primary dysmenorrhea, menstrual pain in the absence of any
underlying pathologic process, is the most common type of dysmenorrhea in adolescents. The
first-line treatment of primary dysmenorrhea is a nonsteroidal anti-inflammatory medication
(NSAID) such as ibuprofen or naproxen sodium.

The evaluation of primary dysmenorrhea does not require imaging (eg, pelvic ultrasonography)
unless there are clinical signs of pelvic pathology. With no history of sexual activity and
reassuring physical examination findings, a pelvic examination with a speculum is not indicated
for the girl in the vignette. Although combined oral contraceptive pills improve dysmenorrhea
symptoms, they are not a first-line option for an adolescent who is not sexually active.
Primary dysmenorrhea can be associated with back pain, nausea, vomiting, diarrhea, and
headaches. Empiric NSAIDs can be started for primary dysmenorrhea. For best results, treatment
with NSAIDs (eg, ibuprofen, naproxen sodium) should begin 1 to 2 days before and continue for
2 to 3 days after menses onset. Maintenance of good hydration is important when taking
NSAIDs.

Hormonal suppression can be achieved with combined oral contraceptive pills, the combined
hormonal patch, the progestin rod implant, or depot medroxyprogesterone acetate. The selection
of a hormonal option depends on patient preferences after a discussion of risk, benefits, and side
effects.

If symptoms do not improve after 3 to 6 months of consistent treatment with NSAIDs and/or
hormonal suppression, the patient should be evaluated for secondary causes of menstrual pain,
(eg, ovarian cysts, pelvic inflammatory disease, pelvic adhesions, congenital or acquired
obstruction, uterine growths). Endometriosis is the most common secondary cause in
adolescents. Pelvic ultrasonography may be indicated in the evaluation of secondary
dysmenorrhea. Pelvic examination with a speculum (with a chaperone present) and testing for
sexually transmitted infections should be performed in sexually active adolescents with severe
dysmenorrhea.

PREP Pearls
• Primary dysmenorrhea is the most common type in adolescents and is not associated with
pelvic pathology.
• Primary dysmenorrhea can be treated empirically with nonsteroidal antiinflammatory
medications. Hormonal suppression may also be indicated, depending on patient
preference and sexual activity.
• The most common cause of secondary dysmenorrhea in adolescents is endometriosis.

ABP Content Specifications(s)


• Recognize the gynecologic etiologies of acute and chronic abdominal pain

American Academy of Pediatrics 846


PREP® Self-Assessment PREPSA 2023

Suggested Readings
• American College of Obstetrics and Gynecologists. ACOG Committee Opinion No. 760:
dysmenorrhea and endometriosis in the adolescent. Obstet Gynecol. 2018;132(6):e249-
e258. doi:10.1097/AOG.0000000000002978.
• Dinerman LM. Dysmenorrhea. In: McInerny TK, Adam HM, Campbell DE, DeWitt TG,
Foy JM, Kamat DM, eds. American Academy of Pediatrics Textbook of Pediatric Care.
American Academy of Pediatrics; 2021:chap 141. Accessed September 1, 2022. Pediatric
Care Online.
• Gray SH. Menstrual disorders. Pediatr Rev. 2013;34(1):6-17. doi:10.1542/pir.34-1-6.
• Ryan S. The treatment of dysmenorrhea. Pediatr Clin North Am. 2017;64(2):331-342.
doi:10.1016/j.pcl.2016.11.004.

American Academy of Pediatrics 847


PREP® Self-Assessment PREPSA 2023

Question 252
A 16-year-old adolescent with a 7-year history of type 1 diabetes is seen for concerns about more
frequent hypoglycemia detected on their continuous glucose monitor for the past 2 weeks. Their
hemoglobin A1c level was 7.8% 2 months ago, and there have not been any recent changes in
their insulin regimen or diet. They have participated in dance for the past 10 years, but missed
multiple classes recently because of easy fatigability and low blood glucose values. They deny
recent illness or depression.

On physical examination, the adolescent’s heart rate is 102 beats/min, blood pressure is 96/58
mm Hg, and oxygen saturation is 99% in room air. Weight is 2.3 kg less than at a visit 2 months
ago. There is hyperpigmentation in the creases of the hands. The remainder of the physical
examination findings are unremarkable.

Of the following, this adolescent’s MOST likely diagnosis is


A. autoimmune adrenalitis
B. celiac disease
C. an eating disorder
D. hypothyroidism

American Academy of Pediatrics 848


PREP® Self-Assessment PREPSA 2023

Correct Answer: A
The adolescent in the vignette has autoimmune adrenalitis, or Addison disease. The history of
type 1 diabetes (autoimmune risk factor), new hypoglycemia, fatigue, tachycardia, low blood
pressure, and hyperpigmentation in the hand creases are all suggestive of primary adrenal
insufficiency. The diagnosis would need to be confirmed by laboratory evaluation.
Hypoglycemia in children and adolescents with type 1 diabetes is commonly caused by too high
of an insulin dose for the basal requirements, carbohydrate intake, and/or activity level. The
adolescent in the vignette, however, has new hypoglycemia despite a long history of diabetes and
no recent changes to the insulin regimen or diet. A decreased activity level would be expected to
cause higher blood glucose levels, not lower. Thus, an alternate underlying cause for this
adolescent’s hypoglycemia should be sought.

Addison disease occurs in 1% or less of children and adolescents with type 1 diabetes, but can
have severe consequences if not detected and treated in a timely fashion. Autoimmune
destruction of the adrenal cortex results in decreased production of cortisol and aldosterone.
Impaired cortisol production can cause fatigue, low blood pressure, abdominal pain, nausea, and
vomiting. The secondary increase in adrenocorticotropic hormone (ACTH) level results in skin
hyperpigmentation. Decreased aldosterone levels may result in dehydration, low serum sodium
level, and high serum potassium level.

Cortisol is a counter-regulatory hormone secreted in response to a falling serum glucose in order


to maintain euglycemia. It acts on the liver to increase gluconeogenesis and decrease glycogen
synthesis. When individuals with type 1 diabetes develop Addison disease, they lose the ability
to counter-regulate their exogenous insulin, placing them at increased risk for hypoglycemia. The
treatment of Addison disease includes replacement of glucocorticoid with hydrocortisone and
mineralocorticoid with 9α-fludrocortisone.

Hypoglycemia in individuals with type 1 diabetes can also be caused by celiac disease. Celiac
disease occurs in up to 16% of individuals with type 1 diabetes. Malabsorption of carbohydrates
that have been planned for in insulin dosing can result in relative overdosing of insulin and
subsequent hypoglycemia. Classically, children and adolescents with celiac disease report
abdominal pain and diarrhea when consuming foods with gluten, which was not noted in this
adolescent’s history. Celiac disease can present with weight loss, but it would not explain the
adolescent’s other physical examination findings. The definitive diagnosis of celiac disease is
made with intestinal biopsy. Children and adolescents undergoing evaluation for celiac disease
should not initiate a gluten-free diet until the biopsy is completed as the lack of gluten may
decrease inflammation and cause false negative results. The treatment of celiac disease is to
consume a gluten-free diet.

Eating disorders occur with increased frequency in adolescents with type 1 diabetes and can
cause weight loss, hypoglycemia, tachycardia, and low blood pressure. However, the adolescent
in the vignette’s hyperpigmentation is not consistent with an eating disorder.
Autoimmune hypothyroidism is the most common autoimmune condition that occurs in
individuals with type 1 diabetes. Symptoms and signs may include fatigue, cold intolerance, dry
American Academy of Pediatrics 849
PREP® Self-Assessment PREPSA 2023

skin, and in females, irregular menses. Some individuals have a goiter and in severe cases,
depression may develop. Autoimmune thyroid disease does not cause low blood glucose values.

PREP Pearls
• Additional autoimmune conditions, including autoimmune hypothyroidism and celiac
disease, commonly occur in individuals with type 1 diabetes.
• New onset of celiac disease and Addison disease can cause unexpected hypoglycemia in
children and adolescents with type 1 diabetes.
• Hypoglycemia in children and adolescents with type 1 diabetes is commonly caused by
too high of an insulin dose for the basal requirements, carbohydrate intake, and/or
activity level.

MOCA-Peds Objective
• Evaluate and manage a child with hypoglycemia.

ABP Content Specifications(s)


• Plan the appropriate management of hypoglycemia in a patient with type 1 diabetes and
other autoimmune disorders (eg, celiac disease, Hashimoto thyroiditis)

Suggested Readings
• American Diabetes Association Professional Practice Committee; Draznin B, Aroda VR,
Bakris G, et al.14. Children and Adolescents: Standards of Medical Care in Diabetes-
2022. Diabetes Care. 2022;45(Supplement_1):S208-S231. doi:10.2337/dc22-S014.
• Auron M, Raissouni N. Adrenal insufficiency. Pediatr Rev. 2015;36(3):92-102.
doi:10.1542/pir.36-3-92.
• Hughes JW, Riddlesworth TD, DiMeglio LA, Miller KM, Rickels MR, McGill JB.
Autoimmune diseases in children and adults with type 1 diabetes from the T1D exchange
clinic registry. J Clin Endocrinol Metab. 2016;101(12):4931-4937. doi:10.1210/jc.2016-
2478.
• Javed A, Schwenk WF, Tebben P. Diabetes mellitus. In: McInerny TK, Adam HM,
Campbell DE, DeWitt TG, Foy JM, Kamat DM, eds. American Academy of Pediatrics
Textbook of Pediatric Care. American Academy of Pediatrics; 2021:chap 241. Accessed
September 1, 2022. Pediatric Care Online.
• Speiser PW. Adrenal dysfunction. In: McInerny TK, Adam HM, Campbell DE, DeWitt
TG, Foy JM, Kamat DM, eds. American Academy of Pediatrics Textbook of Pediatric
Care. American Academy of Pediatrics; 2021:chap 211. Accessed September 1, 2022.
Pediatric Care Online.

American Academy of Pediatrics 850


PREP® Self-Assessment PREPSA 2023

Question 253
A 7-year-old child is brought to the office for a left-sided neck mass present for the past 7 weeks.
The neck mass was evaluated 4 weeks ago and was treated with a 7-day course of amoxicillin.
Vital signs are normal for age. There is a mildly tender, non-fluctuant, left-sided cervical neck
mass. The size of the mass is unchanged from 4 weeks ago. The remainder of the physical
examination findings are unremarkable.

Of the following, the BEST next step in this child’s management is


A. complete blood count and viral serology testing
B. placement of a purified protein derivative test
C. referral to a surgeon for biopsy of the mass
D. ultrasonography of the mass

American Academy of Pediatrics 851


PREP® Self-Assessment PREPSA 2023

Correct Answer: A
The child in the vignette has chronic cervical lymphadenopathy. Lymphadenopathy persisting for
>6 weeks is considered chronic, 2 to 6 weeks is subacute, and <2 weeks is acute. The best next
step in the evaluation of this child’s chronic cervical lymphadenopathy is to obtain a complete
blood count, inflammatory markers, and Epstein– Barr virus (EBV), cytomegalovirus (CMV),
and Bartonella henselae serology.

Infection is by far the most common cause of lymphadenopathy in children. Reactive adenopathy
from a viral infection most commonly presents as acute lymphadenopathy. Infectious causes of
chronic lymphadenopathy include B henselae (cat scratch disease), Mycobacterium tuberculosis,
atypical mycobacteria, specific viral etiologies (eg, EBV, CMV, HIV), fungal infections, and
parasitic infection (eg, toxoplasmosis).

Cat scratch disease is caused by B henselae and passed to humans through cat saliva. The host
cat, or more commonly kitten, is usually asymptomatic. After contact with the infected animal,
the majority of affected children develop a vesicle at the inoculation site followed by unilateral
tender lymphadenopathy. Approximately 26% involve the upper extremities and 18% involve the
neck or jaw. Lymphadenopathy can persist for months.

The initial evaluation of chronic lymphadenopathy in children should focus on the most common
infectious causes. It is common for pediatricians to treat children with acute lymphadenitis
empirically for bacterial lymphadenitis, although monitoring without treatment is appropriate
considering the high incidence of reactive adenopathy in young children with viral upper
respiratory infections. In cases of chronic adenopathy, such as the child in the vignette, initial
laboratory evaluation for common infections such as B henselae, EBV, and CMV is indicated.
Mycobacterial infections should be considered if viral testing is negative and lymphadenopathy
persists. Atypical mycobacteria are more common than M tuberculosis and will not always result
in a positive purified protein derivative skin test. Fine-needle aspiration of the lymph node may
be needed to make the diagnosis. Likewise, HIV testing should be considered if testing for more
common viral etiologies fails to determine a specific diagnosis.

Neoplastic causes of chronic lymphadenopathy in children are much less common than
infectious etiologies but must be considered. In children younger than 6 years,
rhabdomyosarcoma and non-Hodgkin lymphoma are the most common neoplasms causing
enlarged lymph nodes. In children older than 6 years, Hodgkin lymphoma is the most common.
The initial evaluation of chronic lymphadenopathy should also include a complete blood count
and inflammatory markers.

Ultrasonography of an enlarged lymph node may assist in identifying an abscess, defining the
anatomy of the mass, differentiating a malignant versus benign process, and developing
strategies for biopsy. Ultrasonography will not determine the specific etiology of
lymphadenopathy and is therefore not the best next step in management for the child in the
vignette.

American Academy of Pediatrics 852


PREP® Self-Assessment PREPSA 2023

PREP Pearls
• Reactive cervical lymphadenopathy from a viral infection is the most common cause of
lymphadenopathy in children.
• Epstein–Barr virus, cytomegalovirus, and Bartonella henselae (cat scratch disease) are
common causes of chronic cervical lymphadenopathy in children.
• Neoplasms are rare in children, but must be considered in the differential diagnosis of
chronic lymphadenopathy. Non-Hodgkin lymphoma is more common in children
younger than 6 years, and Hodgkin lymphoma is more common in children older than 6
years.

ABP Content Specifications(s)


• Plan the appropriate management of chronic cervical lymphadenopathy
• Identify the age-related etiology of chronic cervical lymphadenopathy

Suggested Readings
• American Academy of Pediatrics. Bartonella henselae (cat-scratch disease). In:
Kimberlin DW, Barnett ED, Lynfield R, Sawyer MH, eds. Red Book: 2021–2024 Report
of the Committee on Infectious Diseases. 32nd ed. American Academy of Pediatrics;
2021. Accessed September 1, 2022. Red Book Online.
• Nield LS, Kamat D. Lymphadenopathy in children: when and how to evaluate. Clin
Pediatr (Phila). 2004;43(1):25-33. doi:10.1177/000992280404300104.
• Weinberg GA, Segel GB, Hall CB. Lymphadenopathy. In: McInerny TK, Adam HM,
Campbell DE, DeWitt TG, Foy JM, Kamat DM, eds. American Academy of Pediatrics
Textbook of Pediatric Care. American Academy of Pediatrics; 2021:chap 239. Accessed
September 1, 2022. Pediatric Care Online.
• Weinstock MS, Patel NA, Smith LP. Pediatric cervical lymphadenopathy. Pediatr Rev.
2018;39(9):433-443. doi:10.1542/pir.2017-0249.

American Academy of Pediatrics 853


PREP® Self-Assessment PREPSA 2023

Question 254
A 14-day-old neonate born at 26 weeks’ gestation is in the neonatal intensive care unit. He
remains intubated, is receiving mechanical ventilatory support, and has a fractional inspired
oxygen concentration of 80% to 90%. He is receiving nothing by mouth because of gastric
feeding intolerance. On physical examination, he is tachycardic with a continuous machinelike
murmur audible throughout the precordium. Chest radiography shows increased pulmonary
vascular markings bilaterally. His creatinine level has been slowly increasing during the last 2
days. The decision is made to treat the neonate’s condition pharmacologically.

Of the following, the BEST treatment for this neonate is


A. acetaminophen, by activating calcium channels and vasoconstricting
B. acetylsalicylic acid, by blocking calcium channels and vasodilating
C. ibuprofen, by activating cyclooxygenase-1 and increasing prostaglandin synthesis
D. indomethacin, by inhibiting cyclooxygenase-1 and decreasing prostaglandin synthesis

American Academy of Pediatrics 854


PREP® Self-Assessment PREPSA 2023

Correct Answers: D
The premature neonate in the vignette has signs of pulmonary overcirculation (prolonged need
for mechanical ventilatory support and increased pulmonary vascular markings on chest
radiograph) and decreased systemic perfusion (tachycardia, increasing creatinine levels, and
feeding intolerance) caused by a patent ductus arteriosus (PDA).

The ductus arteriosus is a fetal blood vessel that connects the pulmonary artery to the aorta. It
directs oxygenated blood from the heart away from the restricted fetal lungs and out to the body.
Because the blood oxygen content increases during typical fetal to postnatal transition, the
systemic vascular resistance increases and the pulmonary vascular resistance decreases; as the
pressure differential across the PDA resolves, the shunting of blood decreases, aiding in closure
of the ductus arteriosus.

Persistent patency of a ductus arteriosus is common in premature neonates and increases their
risk of several morbidities (eg, bronchopulmonary dysplasia, intraventricular hemorrhage, and
feeding intolerance) and mortality. Postnatally, a PDA results in increased pulmonary blood flow
because it typically shunts left to right (aorta to pulmonary). This pulmonary blood flow increase
results in interstitial pulmonary edema and tachypnea in neonates with lung disease associated
with prematurity. A prolonged need for respiratory support may result. Overall cardiac output
increases and tachycardia develops as a compensatory mechanism to maintain adequate end-
organ perfusion. Ultimately, volume overload to the heart may occur and systemic end organ
perfusion can be compromised, leading to a hemodynamically significant PDA.

Although often a clinical diagnosis, echocardiography is generally used to confirm the presence
of a PDA. Indomethacin, by inhibiting cyclooxygenase-1 (COX-1) and decreasing prostaglandin
synthesis, is the best treatment for this neonate.

Nonsteroidal anti-inflammatory drugs (NSAIDs) inhibit the COX-1 enzyme, leading to


decreased synthesis of prostaglandins as well as constriction and possible closure of a PDA.
Conversely, a continuous infusion of prostaglandin can reopen or maintain the patency of a
ductus arteriosus and is used to manage critical congenital heart disease.

Indomethacin and ibuprofen are NSAIDs used to pharmacologically close a PDA. Both decrease,
not increase, prostaglandin synthesis.

Acetaminophen is also used for PDA closure; it inhibits prostaglandin synthesis at the peroxidase
enzyme region, leading to vasoconstriction, but does not activate calcium channels.

Acetylsalicylic acid inhibits the COX-1 enzyme nonselectively but has not been used to treat
PDAs because it is less effective than other options.

A PDA can also be closed surgically or by cardiac catheterization with a device. There are
potential complications of any method of PDA closure. Although pharmacologic treatment of a
hemodynamically significant PDA (hsPDA) is the method generally recommended in the first 2
American Academy of Pediatrics 855
PREP® Self-Assessment PREPSA 2023

weeks after birth, consensus on further treatment strategies, timing of treatment, and the
definition of a hsPDA remains controversial. As such, significant variability exists among
physicians in the management of a persistently PDA. Contraindications to pharmacological
management with NSAIDs include oliguria, necrotizing enterocolitis, bleeding disorders and
currently receiving steroid therapy. In these situations acetaminophen can be considered.

PREP Pearls
• The ductus arteriosus is a blood vessel that anatomically connects the pulmonary artery to
the aorta to allow blood to bypass the lungs in fetal life.
• Nonsteroidal anti-inflammatory drugs (indomethacin and ibuprofen) and acetaminophen
decrease synthesis of prostaglandins, leading to vasoconstriction and subsequent closure
of a patent ductus arteriosus.
• There is significant practice variation regarding treatment of a hemodynamically
significant persistently patent ductus arteriosus.

ABP Content Specifications(s)


• Plan appropriate initial management of patent ductus arteriosus in an infant born
prematurely

Suggested Readings
• Bardanzellu F, Neroni P, Dessì A, Fanos V. Paracetamol in patent ductus arteriosus
treatment: efficacious and safe? Biomed Res Int. 2017;2017:1438038.
doi:10.1155/2017/1438038.
• Benitz WE; Committee on Fetus and Newborn. Patent ductus arteriosus in preterm
infants. Pediatrics. 2016;137(1):e20153730. doi:10.1542/peds.2015-3730.
• Eilers LF, Kyle WB, Allen HD, Qureshi AM. Patent ductus arteriosus. Pediatr Rev.
2021;42(11):632-634. doi:10.1542/pir.2020-004564.
• Runte KE, Flyer JN, Edwards EM, et al. Variation of patent ductus arteriosus treatment in
very low birth weight infants. Pediatrics. 2021;148(5):e2021052874.
doi:10.1542/peds.2021-052874.
• Sankar MN. Bombal S. Benitz WE. PDA: to treat or not to treat. Congenit Heart Dis.
2019;14:46-51. doi:10.1111/chd.12708.
• Sutton NJ, Walsh CA. The newborn with a heart murmur or cyanosis. In: McInerny TK,
Adam HM, Campbell DE, DeWitt TG, Foy JM, Kamat DM, eds. American Academy of
Pediatrics Textbook of Pediatric Care. American Academy of Pediatrics; 2021:chap 239.
Accessed September 1, 2022. Pediatric Care Online.

American Academy of Pediatrics 856


PREP® Self-Assessment PREPSA 2023

Question 255
An 18-month-old boy is seen for a health supervision visit. His parents report that he sat at 12
months of age and crawled 15 months of age. The boy started to exhibit seizures at 10 months of
age, which have become more frequent. He recently stopped crawling. Physical examination
findings are significant for hypotonia and an exaggerated startle response. A dilated funduscopic
examination shows a cherry-red spot on the retina.

Of the following the diagnosis MOST consistent with the boy’s clinical features is
A. Fabry disease
B. mucopolysaccharidosis type I
C. Pompe disease
D. Tay-Sachs disease

American Academy of Pediatrics 857


PREP® Self-Assessment PREPSA 2023

Correct Answer: D
The boy in the vignette, with gross motor delay and regression, seizures, hypotonia, exaggerated
startle response, and cherry-red spot on his retina, has Tay-Sachs disease (TSD). Tay-Sachs
disease is part of a continuum of disorders caused by deficiency of the enzyme β-hexosaminidase
A (HEXA), which is needed to break down GM2 ganglioside. In the absence of this enzyme,
GM2 ganglioside is aggregated in the lysosomes of brain and nerve cells, leading to the clinical
features of the disease. The spectrum of disorders caused by HEXA deficiency includes acute
infantile, subacute juvenile, and late-onset TSD. Clinical features expressed depend on the
amount of enzyme present. The child in the vignette is seen with acute infantile TSD.
Clinical features of acute infantile TSD include:
• Early-onset hypotonia and gross motor delay
• Sustained ankle clonus, hyperreflexia, increased or exaggerated startle response
• Regression of motor skills at 6 to 9 months
• Decreased attention
• Cherry-red spot of the fovea centralis of the macula of the retina
• Seizure onset at around 1 year of age
• No hepatomegaly or splenomegaly
• Brain magnetic resonance imaging: ventricular enlargement starting at 18 months of age

Acute infantile TSD has early onset, rapid progression, and death before age 5 years.

Subacute juvenile TSD has a later onset, with survival documented into late childhood or
adolescence.

Symptoms of late-onset TSD generally present in the late teenage years to young adulthood as a
slowly progressive neurologic disease.
Diagnosis of TSD is established based on the clinical features and identification of 2 disease-
causing changes in the HEXA gene. Tay-Sachs disease is inherited in an autosomal recessive
manner.

Treatment of TSD is supportive, including management of seizures, intervention therapies for


developmental skills, insertion of a gastrostomy tube due to feeding difficulties and risk of
aspiration, and involvement of a palliative care team to assist with various ongoing needs.
Fabry disease is an X-linked lysosomal storage disorder that presents with sweating
abnormalities, severe pain crises in the extremities, corneal and lenticular opacities, vascular
cutaneous lesions (angiokeratomas), cardiomyopathy, and renal failure. Developmental delay
and regression, and a cherry-red spot in the retina are not seen in Fabry disease. Treatment is
available via enzyme replacement therapy (ERT) and substrate reduction therapy.

Pompe disease is an autosomal recessive lysosomal storage disorder that presents as either
infantile-onset Pompe disease (IOPD) or late-onset Pompe disease (LOPD). Clinical features of
IOPD include marked hypotonia, failure to thrive, and hypertrophic cardiomyopathy within the
first year after birth. Clinical features of LOPD include proximal muscle weakness with
respiratory insufficiency, without cardiac involvement. Pompe disease is managed with ERT.
American Academy of Pediatrics 858
PREP® Self-Assessment PREPSA 2023

Although developmental delay can be seen, seizures and a cherry-red spot on the retina are not
features of Pompe disease.

Mucopolysaccharidosis type I (Hurler syndrome) is characterized by macrocephaly, coarse facial


features, corneal clouding, frequent respiratory tract infections, hepatosplenomegaly, and
progressive skeletal dysplasia. The child in the vignette does not have these features.

PREP Pearls
• Tay-Sachs disease is an autosomal recessive disorder characterized by gross motor delay,
seizures, developmental regression, increased startle response, and a cherry-red spot on
the retina.
• Management of Tay-Sachs disease is supportive with interventional therapies, control of
seizures, and nutritional support.

ABP Content Specifications(s)


• Recognize the clinical features associated with Tay-Sachs disease
• Plan the appropriate immediate and long-term management of Tay-Sachs disease, while
considering the long-term prognosis

Suggested Readings
• Green CL. Metabolic disorders beyond the newborn period. In: McInerny TK, Adam
HM, Campbell DE, DeWitt TG, Foy JM,
• Kamat DM, eds. American Academy of Pediatrics Textbook of Pediatric Care. American
Academy of Pediatrics; 2021:chap 291. Accessed September 1, 2022. Pediatric Care
Online.
• Rice GM, Steiner RD. Inborn errors of metabolism (metabolic disorders). Pediatr Rev.
2016;37(1):3-17. doi:10.1542/pir.2014-0122.
• Toro C, Shirvan L, Tifft C, eds. Hexa disorders. GeneReviews [Internet]. University of
Washington; 2021. Accessed September 1, 2022.
https://www.ncbi.nlm.nih.gov/books/NBK1218/.

American Academy of Pediatrics 859


PREP® Self-Assessment PREPSA 2023

Question 256
A 5-year-old boy is brought to the emergency department with a nosebleed. On arrival, he has
active bleeding from the right naris. Pressure is applied for 10 minutes, and the bleeding
resolves. The boy’s mother reports that he has frequent nosebleeds. They occur at least once per
month and usually resolve with applied pressure. He does pick his nose frequently, and his
mother is concerned about the number of nosebleeds he is experiencing. There is no family
history of bleeding disorders. The boy has a temperature of 37.4°C, heart rate of 84 beats/min,
respiratory rate of 20 breaths/min, blood pressure of 108/70 mm Hg, and oxygen saturation of
99% in room air. His physical examination findings are normal except for dried blood in the
anterior portion of the right naris.

Of the following, the BEST next step in this boy’s management is to


A. obtain a complete blood count and coagulation studies
B. perform nasal cautery
C. provide reassurance and recommend conservative measures
D. refer to an otolaryngologist

American Academy of Pediatrics 860


PREP® Self-Assessment PREPSA 2023

Correct Answer: C
The boy in the vignette experienced a nosebleed that resolved after a brief period of applying
pressure. The best next management step is to provide reassurance and discuss conservative
measures that include applying pressure in the appropriate location for 5 to 10 minutes without
interruption, frequent moisturizing of the nasal mucosa with nasal saline or ointment,
humidifying the air in the home, and discouraging nose picking.

Nasal bleeding is common in children and frequently causes parental anxiety. The boy in the
vignette has normal vital signs, and the nasal bleeding was easily controlled with appropriate
pressure. No additional acute management is required. Nasal packing and nasal cautery should
be considered when bleeding continues despite attempts to control bleeding with appropriate
pressure. Routine laboratory evaluation (eg, complete blood count, coagulation studies) is not
necessary unless the child is experiencing frequent or prolonged nasal bleeding or has skin
examination findings concerning for a bleeding disorder (eg, unusual ecchymosis, petechiae).
The boy in the vignette does not have abnormal skin findings or family history of a bleeding
disorder, and he is experiencing infrequent nosebleeds. Referral to an otolaryngologist is
generally not required unless a nosebleed cannot be stopped or there are recurrent nosebleeds
requiring frequent trips to the pediatrician’s office or emergency department after appropriate
attempts to stop them at home.

The etiologies of epistaxis vary; a thorough history and physical examination should be
performed to identify the cause. The differential diagnosis of epistaxis includes trauma,
inflammation, dry air, neoplasms, intranasal drugs, structural abnormalities, and bleeding
disorders. Item C256A lists potential causes of epistaxis in children.

Nosebleeds are divided into 2 categories, anterior and posterior, based on the location of the
bleeding. Anterior nosebleeds are more common and are much easier to visualize and control.
The bleeding most commonly occurs at the Kiesselbach plexus (Kiesselbach triangle), a vascular
plexus located at the anteroinferior aspect of the nasal septum. The acute management of nasal
bleeding includes pressure application by squeezing the nasal alae against the nasal septum and
gentle elevation of the child’s head. If bleeding continues, application of a topical decongestant
or topical vasoconstrictor (eg, oxymetazoline or epinephrine), or nasal cauterization of the
bleeding locus may be necessary. If these measures are not effective, nasal packing should be
placed, and immediate evaluation by an otolaryngologist is warranted. Item C256B outlines a
treatment algorithm for epistaxis.

American Academy of Pediatrics 861


PREP® Self-Assessment PREPSA 2023

American Academy of Pediatrics 862


PREP® Self-Assessment PREPSA 2023

PREP Pearls
• Nosebleeds are common in children and most can be treated with conservative
management such as applying pressure in the appropriate location for 5 to 10 minutes
without interruption, moisturizing the nasal mucosa with nasal saline or ointment,
providing humidified air in the home, and discouraging nose picking.
• The differential diagnosis of epistaxis includes trauma, inflammation, dry air, neoplasms,
intranasal drugs, structural abnormalities, and bleeding disorders.
• Nosebleeds are divided into 2 categories, anterior and posterior, based on the location of
the bleeding. Anterior nosebleeds are the most common type and are much easier to
visualize and control than posterior nasal bleeds.

ABP Content Specifications(s)


• Formulate a differential diagnosis of epistaxis and manage appropriately
• Plan the appropriate evaluation of the various manifestations of epistaxis

American Academy of Pediatrics 863


PREP® Self-Assessment PREPSA 2023

Suggested Readings
• Mulbury PE. Recurrent epistaxis. Pediatr Rev. 1991;12(7):213-217. doi:10.1542/pir.12-7-
213.
• Schechter MB, Stevens DM. Epistaxis. In: McInerny TK, Adam HM, Campbell DE,
DeWitt TG, Foy JM, Kamat DM, eds. American Academy of Pediatrics Textbook of
Pediatric Care. American Academy of Pediatrics; 2021:chap 146. Accessed September 1,
2022. Pediatric Care Online.
• Tunkel DE, Anne S, Payne SC, et al. Clinical practice guideline: nosebleed (epistaxis).
Otolaryngol Head Neck Surg. 2020;162(1_suppl):S1-S38.
doi:10.1177/0194599819890327.

American Academy of Pediatrics 864


PREP® Self-Assessment PREPSA 2023

Question 257
A previously healthy, 16-month-old girl is brought to the emergency department by ambulance
for burns on her face and mouth sustained from drinking a lye solution that her mother uses to
make soap. The girl’s mother found her crying in great pain with redness and swelling around
her mouth. She rinsed the affected area with water and called 911. In the emergency department
the girl’s vital signs include a temperature of 37.5°C, heart rate of 165 beats/min, respiratory rate
of 38 breaths/min, and oxygen saturation of 98% in room air. On physical examination she has
significant facial irritation and lip edema (Item Q257), vocal hoarseness, drooling, nasal flaring,
and audible stridor. The remainder of her physical examination findings are unremarkable.

Item Q257: Facial burns and lip edema after alkali exposure.
Courtesy of J. Kane

Of the following, the BEST next step in this girl’s management is


A. chest radiography
B. endotracheal intubation
C. esophagogastroduodenoscopy
D. nasogastric tube placement

American Academy of Pediatrics 865


PREP® Self-Assessment PREPSA 2023

Correct Answer: B
The girl in the vignette ingested lye, a caustic alkali, and is displaying signs and symptoms of
upper airway edema (mouth and lip swelling, vocal hoarseness) and impending airway
compromise (drooling, stridor). The best next management step is endotracheal intubation to
secure her airway.

Caustic ingestions immediately affect the eyes, skin, airway, and gastrointestinal tract. Evidence
of a significant ingestion includes swelling of the tongue and mouth, drooling, and vomiting.
Direct alkali contact with tissues during swallowing or subsequent emesis causes edema, which
often involves the upper airway. Edema of the upper airway results in stridor, respiratory
distress, and vocal hoarseness. Because of the risk of rapid progression of upper airway and
oropharyngeal edema, when children develop an inability to handle oral secretions or voice
changes, clinicians should have a high index of suspicion for impending airway compromise.
Any airway compromise must be immediately addressed, as progression to complete obstruction
can be life-threatening. Chest radiography may be needed to assess for air leak from an
esophageal or tracheal perforation, but stabilization of the airway should occur before a chest
radiograph is obtained.

Caustic-induced injuries in children represent a serious public health issue in the United States.
Thousands of children with caustic ingestions are hospitalized annually, with most injuries
resulting from the natural exploratory behavior of young children. Rarely, pediatric ingestions
are the result of intentional inflicted injury or attempted selfharm.

Alkalis are commonly found in household cleaning products. Sodium hydroxide is the active
ingredient in alkalinebased cleaners such as drain cleaner, dishwasher detergent, and many
kitchen- or oven-cleaning products. Upon ingestion, the alkaline agent bonds with tissue
proteins, leading to liquefaction necrosis. Because of a strong surface tension, alkaline fluid will
stay on the tissue for a prolonged period (in contrast with acids). As such, tissue injury is
ongoing until the alkali is neutralized, which can take several days. Thus, compared to acid
ingestions, alkali ingestions tend to result in more significant injury and tissue damage.

Initial evaluation and resuscitation efforts after caustic ingestion should focus on injuries to the
oropharynx and airway. Subsequent care should address any skin and/or eye injuries from
splashes or spills. Affected clothing should be removed and the skin completely exposed. Any
areas of skin with suspected or confirmed alkali contact should undergo high-volume irrigation
with water. If the eyes are affected, saline irrigation should begin immediately, and urgent
consultation with an ophthalmologist is indicated.

Gastric decontamination with activated charcoal or induced vomiting are contraindicated in all
caustic ingestions. Activated charcoal does not absorb alkali agents and residual charcoal is
likely to obscure subsequent endoscopic visualization of esophageal or gastric erosions. Emesis
can increase alkali exposure and cause additional injury. There is no evidence to support the
efficacy of blind nasogastric-tube insertion to maintain esophageal patency, and there is a high
risk of complications (eg, esophageal rupture).
American Academy of Pediatrics 866
PREP® Self-Assessment PREPSA 2023

Burns on the cheeks, lips, or in the oropharynx may not accurately reflect the presence or scope
of injury to the lower gastrointestinal tract. Historically, esophagogastroduodenoscopy (EGD)
was performed in all children with unintentional alkali ingestion; however, current guidelines
support a watchful observation period for asymptomatic or minimally symptomatic children.
Children with vomiting and drooling, or with stridor alone are more likely to have severe injuries
and warrant evaluation with EGD. When indicated, endoscopy is usually performed within the
first 48 hours after the ingestion; identified lesions are graded by a standard scoring system
reflecting the degree of tissue damage and the prognosis.

There is no evidence to support routine administration of proton-pump inhibitors or antibiotic


agents in children with caustic ingestions. Antibiotics may be indicated in the event of direct
lung injury, sepsis, or if perforation is suspected; however, no prospective trial has evaluated the
isolated use of antibiotics (in the absence of documented infection) with respect to the prevention
of perforation, or prevention of esophageal strictures. There are conflicting data regarding the
use of corticosteroids after caustic ingestion with some evidence suggesting benefit in children
with respiratory symptoms. The only other medications that have been studied specifically for
treatment of caustic alkali ingestions are sucralfate and mitomycin C; sparse data suggest that
each of these medications may have some effectiveness in the prevention of esophageal stricture
formation.

PREP Pearls
• Alkali ingestions cause tissue liquefaction necrosis resulting in greater injury compared
to acid ingestions.
• Symptoms of airway injury due to alkali ingestion, including drooling, mouth pain, lip
swelling, and stridor, should raise concern for impending airway compromise; a secure
airway should be ensured.
• Symptomatic, clinically stable children should have esophagogastroduodenoscopy
performed within 48 hours of alkali ingestion; current guidelines support a watchful
observation period for asymptomatic or minimally symptomatic children.

ABP Content Specifications(s)


• Recognize the signs and symptoms of ingestion of a caustic substance, and manage
appropriately

American Academy of Pediatrics 867


PREP® Self-Assessment PREPSA 2023

Suggested Readings
• Ali Zargar S, Kochar R, Mehta S, Mehta SK. The role of fiberoptic endoscopy in the
management of corrosive ingestion and modified endoscopic classification of burns.
Gastrointest Endosc. 1991;37(2):165-169. doi:10.1016/s00165107(91)70678-0 .
• Bird JH, Kumar S, Paul C, Ramsden JD. Controversies in the management of caustic
ingestion injury: an evidence-based review. Clin Otolaryngol. 2017;42(3):701-708.
doi:10.1111/coa.12819.
• Hoffman RS, Burns MM, Gosselin S. Ingestion of caustic substances. N Engl J Med.
2020;382:1739-1748. doi:10.1056/NEJMra1810769.
• Kay M, Wyllie R. Caustic ingestions in children. Curr Opin Pediatr. 2009;21(5):651-654.
doi:10.1097/MOP.0b013e32832e2764.
• Ricca, RL, Drugas GT. Esophageal caustic injury. In: McInerny TK, Adam HM,
Campbell DE, DeWitt TG, Foy JM, Kamat DM, eds. American Academy of Pediatrics
Textbook of Pediatric Care. American Academy of Pediatrics; 2021:chap 359. Accessed
September 1, 2022. Pediatric Care Online.
• Sheridan DC, Hughes A, Horowitz BZ. Pediatric ingestions: new high-risk household
hazards. Pediatr Rev. 2021;42(1):2-10. doi:10.1542/pir.2019-0309.

American Academy of Pediatrics 868


PREP® Self-Assessment PREPSA 2023

Question 258
An 11-year-old, previously healthy boy is brought to the office for evaluation of fever, malaise,
rhinorrhea, and a cough for 7 days. Two days ago, he developed painful blisters on his lips and
oral mucous membranes. He is not taking any medications. The boy’s temperature is 38.4°C,
heart rate is 96 beats/min, respiratory rate is 42 breaths/min, and oxygen saturation is 92% in
room air. He has ulcerative mucosal lesions involving the lips and oral mucous membranes
(Item Q258), bilateral non-purulent conjunctivitis, pharyngeal erythema without exudates, and
bilateral diffuse crackles on lung auscultation. Skin examination is notable for a few target
lesions on the lower extremities. The remainder of his physical examination findings are
unremarkable.

Item Q258: Lip and oral mucous membrane findings for the boy in the vignette.
Reprinted with permission from ER Chulie Ulloa. Copyright ER Chulie Ulloa.

Of the following, the MOST likely cause of this boy’s illness is


A. adenovirus
B. coxsackievirus
C. herpes simplex virus
D. Mycoplasma pneumoniae

American Academy of Pediatrics 869


PREP® Self-Assessment PREPSA 2023

Correct Answer: D
The boy in the vignette has a respiratory tract infection associated with conjunctivitis, oral
mucositis, and rash. This presentation is consistent with Mycoplasma pneumoniae–induced rash
and mucositis (MIRM).

Mycoplasma pneumoniae is a common cause of upper and lower respiratory tract infection in
school-aged children and young adults. Infection is acquired via aerosolized droplets. Severe
pulmonary disease may occur in children with sickle cell disease, trisomy 21, immunodeficiency
disorders, and chronic cardiac or pulmonary disease.

Approximately 10% of individuals infected with M pneumoniae develop a nonspecific


maculopapular rash. M pneumoniae also causes other distinct mucocutaneous eruptions
including erythema multiforme, Stevens-Johnson syndrome, and MIRM.

Mycoplasma-induced rash and mucositis is characterized by painful, erosive lesions involving 2


or more mucosal sites (Item C258). The skin lesions in MIRM are target or vesiculobullous,
typically few in number (may be absent), and localized to the distal extremities. The rash is
preceded by a respiratory tract infection of approximately 1-week duration. The clinical course is
generally mild, although the oral lesions may be painful and limit oral intake. Severe cases may
require hospital admission for hydration and pain control. The recurrence rate is less than 10%.
In contrast to MIRM, M pneumoniae–associated Stevens-Johnson syndrome is characterized by
generalized skin lesions with predominant involvement of the trunk and face and extensive
mucosal involvement affecting 2 or more sites. Individuals with Stevens-Johnson syndrome or
toxic epidermal necrolysis (often a drug-related reaction to sulfonamides, anticonvulsants, non-
steroidal anti-inflammatory agents) have systemic illness with a complicated and prolonged
clinical course.

Antimicrobial therapy with a macrolide (eg, azithromycin, clarithromycin) may be indicated for
hospitalized children with moderate to severe M pneumoniae pneumonia or other severe
manifestations, including MIRM. In addition to macrolides, steroid therapy may shorten the
clinical course of MIRM especially if the oral lesions are severe and painful.

Adenovirus typically presents with high fever, exudative pharyngitis, and purulent conjunctivitis.
It does not cause the blistering mucositis described in the vignette.

The clinical manifestations of coxsackievirus infection may include pharyngitis and stomatitis
(ie, “herpangina”), but the site of mucosal lesions is the anterior pillars, posterior palate, and
pharynx. Other manifestations of coxsackie infection include hand-foot-mouth disease (sites of
vesicular lesions include the tonsillar fauces, buccal mucosa, and tongue), gastroenteritis, non-
specific exanthems, acute hemorrhagic conjunctivitis, pleurodynia, and myopericarditis.
In infants and young children beyond the neonatal period, primary herpes simplex virus (HSV)
infection may cause gingivostomatitis characterized by fever, malaise, tender submandibular
lymphadenopathy, and vesicular lesions involving the gingiva, buccal mucosa, tongue, and lips.
Both HSV and M pneumonia can cause erythema multiforme, a polymorphous exanthem with a
American Academy of Pediatrics 870
PREP® Self-Assessment PREPSA 2023

characteristic doughnut-shaped target lesion. However, the 1-week prodrome of cough, malaise,
and fever prior to onset of oral mucositis, and the presence of sparse skin lesions is more
suggestive of MIRM.

PREP Pearls
• Approximately 10% of individuals infected with Mycoplasma pneumoniae develop a
nonspecific maculopapular rash.
• Mycoplasma pneumoniae–induced rash and mucositis (MIRM) is an extrapulmonary
manifestation characterized by painful erosive lesions involving 2 or more mucosal sites;
skin lesions (eg, target lesions, bullae) may be absent or sparse.
• Treatment with azithromycin and steroid therapy may shorten the clinical course of
Mycoplasma pneumoniae– induced rash and mucositis.

ABP Content Specifications(s)


• Plan the appropriate diagnostic evaluation of Mycoplasma pneumoniae infection
• Plan appropriate management for a patient with Mycoplasma pneumoniae infection

Suggested Readings
• American Academy of Pediatrics. Mycoplasma pneumoniae and other mycoplasma
species infections. In: Kimberlin DW, Barnett ED, Lynfield R, Sawyer MH, eds. Red
Book: 2021–2024 Report of the Committee on Infectious Diseases. 32nd ed. American
Academy of Pediatrics; 2021. Accessed September 1, 2022. Red Book Online.
• Canavan TN, Mathes EF, Frieden I, et al. Mycoplasma pneumoniae-induced rash and
mucositis as a syndrome distinct from Stevens–Johnson syndrome and erythema
multiforme: a systematic review. J Am Acad Dermatol. 2015;72(2):239245.
doi:10.1016/j.jaad.2014.06.026 .
• Krafft C, Christy C. Mycoplasma pneumonia in children and adolescents. Pediatr Rev.
2020; 41(1):12-19. doi:10.1542/pir.2018-0016.
• Ramien ML. Reactive infectious mucocutaneous eruption: Mycoplasma pneumoniae-
induced rash and mucositis and other parainfectious eruptions. Clin Exp Dermatol.
2021;46(3):420-429. doi:10.1111/ced.14404.

American Academy of Pediatrics 871


PREP® Self-Assessment PREPSA 2023

Question 259
A 5-day-old neonate is seen in the office with bilateral profuse, yellow eye discharge. The
neonate was born at home through a planned home delivery. The mother and newborn did not
receive any medications during or after delivery. He has been breastfeeding well. The eye
discharge started about 2 days ago; this morning, his mother noticed that his eyelids were getting
swollen and there was a small amount of bloody discharge from the left eye.

Of the following, the neonate’s condition could have been PREVENTED by administration of
A. erythromycin 0.5% ophthalmic ointment, to both eyes of the neonate at birth
B. intramuscular vitamin K, 1 mg, to the neonate at birth
C. oral acyclovir, 10 mg/kg every 8 hours, to the neonate starting at age 24 hours
D. oral ampicillin, 500 mg, every 6 hours to the mother, starting at the onset of labor

American Academy of Pediatrics 872


PREP® Self-Assessment PREPSA 2023

Correct Answers: A
The neonate in the vignette has ophthalmia neonatorum due to Neisseria gonorrhoeae, which
typically presents 2 to 5 days after birth. Prophylactic erythromycin ophthalmic ointment, which
prevents gonococcal ophthalmia from N gonorrhoeae, was not administered to the newborn at
the time of delivery. Erythromycin is the only medication approved by the US Food and Drug
Administration for ophthalmia neonatorum prophylaxis. Mother-to-newborn transmission rates
of gonococcal infection range from 30% to 50% in those without ocular prophylaxis.

Untreated gonococcal ophthalmia neonatorum can result in corneal scarring and blindness.
Treatment of gonococcal ophthalmia neonatorum consists of a single dose of ceftriaxone (25-50
mg/kg, intravenous [IV] or intramuscular [IM]) or cefotaxime (100 mg/kg, IV or IM) if
ceftriaxone is contraindicated because of simultaneous administration of intravenous calcium.
The neonate should be evaluated for disseminated infection (including sepsis, arthritis, and
meningitis), and infectious disease consultation should be considered. An ophthalmologic
consultation and follow-up should also be considered.

Erythromycin prophylaxis does not prevent Chlamydia trachomatis conjunctivitis in the


newborn. Vitamin K 1 mg IM injection after birth prevents hemorrhagic disease of the newborn.
There is no indication for administration of ampicillin to the mother or oral acyclovir to the
neonate in this vignette.

PREP Pearls
• Administration of erythromycin ophthalmic ointment at the time of delivery prevents
gonococcal ophthalmia from Neisseria gonorrhoeae.
• Mother-to-newborn transmission rates of gonococcal infection range from 30% to 50% in
those without ocular prophylaxis.

ABP Content Specifications(s)


• Plan appropriate eye prophylaxis for a newborn infant

Suggested Readings
• American Academy of Pediatrics. Neonatal ophthalmia. In: Kimberlin DW, Barnett ED,
Lynfield R, Sawyer MH, eds. Red Book: 2021–2024 Report of the Committee on
Infectious Diseases. 32nd ed. American Academy of Pediatrics; 2021. Accessed
September 1, 2022. Red Book Online.
• Smith-Norowitz TA, Ukaegbu C, Kohlhoff S, Hammerschlag MR. Neonatal prophylaxis
with antibiotic containing ointments does not reduce incidence of chlamydial
conjunctivitis in newborns. BMC Infect Dis. 2021;21(1):270. doi:10.1186/s12879-021-
05974-3.
• US Preventive Services Task Force. Ocular prophylaxis for gonococcal ophthalmia
neonatorum: US Preventive Services Task Force Reaffirmation Recommendation
Statement. JAMA. 2019;321(4):394-398. doi:10.1001/jama.2018.21367.
• Warren JB, Phillipi CA. Care of the well newborn. Pediatr Rev. 2012;33(1):4-18.
doi:10.1542/pir.33.1.4.
American Academy of Pediatrics 873
PREP® Self-Assessment PREPSA 2023

Question 260
A 16-year-old adolescent boy is evaluated in the office for exercise-induced cough, upper chest
tightness, and difficulty catching his breath. When he was in preschool, the boy was treated with
an albuterol inhaler for wheezing with viral respiratory infections. He had no further breathing
difficulties until recently, since he began training for cross country running. He was previously
moderately active, but cross country running is a higher intensity exercise for him. His symptoms
occur shortly after beginning to run and worsen with the intensity of the workout. His symptoms
are affecting his ability to compete. The boy tried a friend’s albuterol inhaler in an attempt to
treat his symptoms, without relief. He also tried using the inhaler before running, but it did not
prevent his symptoms. On physical examination, the boy looks fit and well. He has normal vital
signs for age, and normal lung and cardiovascular findings. Office spirometry results are normal.

Of the following, the BEST next step in this boy’s management is to


A. begin daily inhaled corticosteroids
B. counsel him regarding albuterol inhaler technique
C. recommend that he choose another sport
D. schedule a treadmill exercise provocation test

American Academy of Pediatrics 874


PREP® Self-Assessment PREPSA 2023

Correct Answers: D
The boy in the vignette has exercise-related respiratory symptoms that are neither relieved nor
prevented by the use of an albuterol inhaler. The boy’s symptoms are most consistent with
exercise-induced laryngeal obstruction (EILO). The best next step in this boy’s management is to
schedule a treadmill exercise provocation test, the best way to reproduce the situation in which
he has symptoms and objectively monitor the effect of submaximal exercise on his lung function.
If the boy has exercise-induced asthma (EIA), he should have at least partial relief of symptoms
or some degree of symptom prevention with the use of an albuterol inhaler, even if his technique
is not perfect. Although assessing his inhaler technique would be beneficial, this evaluation
would likely delay his diagnosis and appropriate management.

Characteristics of the boy’s symptom complex suggest a diagnosis other than EIA. His
symptoms begin shortly after the intensity of running increases, sooner than expected for EIA; he
feels the tightness in his upper chest and reports difficulty “catching his breath,” which suggests
an inspiratory obstruction more than the small airway expiratory obstruction seen in asthma.
Differentiating EIA from EILO, also known as vocal cord dysfunction, is often very difficult;
both conditions may coexist. Many children and adolescents report the inspiratory sounds of
stridor or stertor as “wheezing,” and unless pointed questions are asked about the timing and
nature of the respiratory noise, the physician may fail to recognize that the symptoms are not
typical of asthma.

A treadmill exercise provocation test with spirometry can determine the presence or absence of
EIA. Findings consistent with EIA include a decrease in forced expiratory volume in 1 second
(FEV1) by 15% or more from baseline after 5 to 6 minutes of submaximal exercise, defined by
attaining a heart rate of 80% of predicted for age (220 beats/min minus age in years) and
maintaining it for the defined exercise period. The classic pattern of spirometry is normal
function before and immediately after exercise with a decrease in FEV1 documented 5 to 30
minutes after completion of the exercise. Cough, audible expiratory wheeze, and generalized
chest tightness may or may not be present.

Children and adolescents with EILO will often develop cough and upper chest or neck tightness
shortly after attaining submaximal exercise rate, or before reaching the goal workload. They will
often request that the challenge be stopped and be very symptomatic despite showing normal
expiratory airflow on physical examination and spirometry. In situations with a high suspicion
for EILO, a speech therapist or otolaryngologist may be involved to do a concurrent flexible
nasopharyngoscopy to assess for paradoxical vocal cord or vocal fold motion that correlates with
the symptoms. A high index of suspicion for EILO should be maintained for the child or
adolescent with respiratory symptoms after a partial or fully completed exercise challenge and no
evidence of airflow obstruction. Item C260A compares the characteristics of EIA and EILO, and
Item C260B shows typical flow volume loops on spirometry for these conditions.

Pre-exercise inhaled bronchodilator therapy is the appropriate treatment for isolated EIA with no
other trigger. Inhaled corticosteroids are generally not needed in cases of isolated EIA with no
other triggers or signs of persistent asthma. Treatment of EILO is nonpharmacologic, with voice
American Academy of Pediatrics 875
PREP® Self-Assessment PREPSA 2023

control training via biofeedback and breathing exercises to maintain upper airway and vocal cord
patency both at rest and with exercise.

Some children and adolescents with persistent asthma do not recognize their symptoms other
than with exercise. The boy in the vignette has normal baseline spirometry, making persistent
asthma unlikely. If the boy had evidence of obstruction on his baseline spirometry with
bronchodilator reversibility, he would be defined as having persistent asthma with exercise as a
symptomatic trigger. In that situation, daily treatment with either an inhaled corticosteroid or a
combined corticosteroid/long-acting β-agonist inhaler would be appropriate, and exercise
provocation would not be indicated unless the boy failed to obtain relief with appropriate
treatment for his airway reactivity. Use of daily inhaled steroids would also be appropriate if the
boy had symptoms at least 2 to 3 times weekly with triggers other than exercise (consistent with
persistent asthma).

It is not necessary to recommend that the boy in the vignette give up running, as either EIA or
EILO can be treated adequately, enabling him to participate at the level he desires.

PREP Pearls
• Exercise-induced asthma is characterized by onset several minutes after completion of
submaximal exercise and is relieved or prevented by bronchodilator inhalation.
• A treadmill exercise provocation test is the most appropriate way to document the
presence and severity of exerciseinduced asthma.
• Exercise-induced laryngeal obstruction is associated with neck or upper chest tightness
and inspiratory stridor or stertor that may be incorrectly interpreted as wheezing. There is
a direct relationship between exercise intensity and symptoms, which are not relieved by
albuterol.

ABP Content Specifications(s)


• Recognize the clinical features associated with exercise-induced asthma

Suggested Readings
• Boulet LP, O’Byrne PM. Asthma and exercise-induced bronchoconstriction in athletes. N
Engl J Med. 2015;372(7):641648. doi:10.1056/NEJMra1407552.
• Nielsen EW, Hull JH, Backus V. High Prevalence of exercise-induced laryngeal
obstruction in athletes. Med Sci Sports Exerc. 2013;45(11):2030-2035.
doi:10.1249/MSS.0b013e318298b19a.
• Patel SJ, Teach SJ. Asthma. Pediatr Rev. 2019;40(11):549-567. doi:10.1542/pir.2018-
0282.
• Shay EO, Sayad E, Milstein CF. Exercise-induced laryngeal obstruction (EILO) in
children and young adults: from referral to diagnosis. Laryngoscope. 2020;130(6):E400-
E406. doi:10.1002/lary.28276.
• Tilles SA. Exercise-induced respiratory symptoms: an epidemic among adolescents. Ann
Allergy Asthma Immunol. 2010;104(5):361-367. doi:10.1016/j.anai.2009.12.008.

American Academy of Pediatrics 876


PREP® Self-Assessment PREPSA 2023

Question 261
A 5-year-old boy taking valproic acid for a known seizure disorder is being seen for his routine
health supervision visit. He has been healthy overall and is thriving, with no recent seizure
activity. He had a viral upper respiratory illness 6 months ago. There are no pertinent findings on
the boy’s physical examination. His vital signs are normal for age. There is no palpable
lymphadenopathy or hepatosplenomegaly as well as no rash.

A screening complete blood cell count is obtained.


Laboratory Test Result
White blood cell count 4,800/µL (4.8 × 109/L)
Neutrophils 21.2%
Lymphocytes 63.5%
Monocytes 13.2%
Eosinophils 2.1%
Hemoglobin 12.3 g/dL (123 g/L)
Platelet count 184 × 103/µL (184 × 109/L)
Mean corpuscular volume 76 fL
Reticulocyte 1%
Absolute reticulocyte count 48 × 103/µL

Of the following, the best NEXT step in management is


A. administration of intravenous antibiotics
B. observation with repeat complete blood cell count in 4 weeks
C. prescription for a course of oral antibiotics
D. referral to pediatric hematology/oncology for administration of growth colony-stimulating
factor

American Academy of Pediatrics 877


PREP® Self-Assessment PREPSA 2023

Correct Answer: B
The boy in the vignette has mild neutropenia (0.212 × 4,800 = ~1,018/µL), as defined by an
absolute neutrophil count (ANC) less than 1,500/µL but above 1,000/µL. Neutropenia may be
divided into mild (ANC 1,000 to 1,500/µL), moderate (ANC 500 to <1,000/µL), and severe
(ANC <500/µL) categories, with some literature including a very severe category (ANC
<200/µL).

Mild neutropenia in an otherwise healthy individual does not require immediate intervention.
The complete blood cell count should be obtained after a short interval (ie, 2-4 weeks); if the
neutropenia persists, further investigation may be required. This approach would be the best next
management step for the child in the vignette. Treatment is not needed at this time so the
administration of oral or intravenous antibiotics or growth colony-stimulating factor would not
be appropriate.

Neutropenia has many causes, which can be broadly divided into acquired or intrinsic. Most
acquired causes are transient. Intrinsic causes require further investigation. The most common
acquired cause of neutropenia is transient bone marrow suppression due to an infection, usually
viral. This is usually the result of high neutrophil use along with slowed production. Some
bacterial infections may cause neutropenia due to redistribution of neutrophils from the
peripheral blood to the vessel wall, increased use at the site of infection, or decreased cell
production. Some individuals with a Duffy null [Fy(a-b-)] phenotype, which is protective against
malaria, may have a benign form of neutropenia.

A viral illness may also cause neutropenia through the formation of anti–granulocyte antibodies
which can lead to splenic clearance of antibody-coated neutrophils or complement-mediated cell
destruction. The lack of antibody detection does not completely rule out this disorder.

Various medications, including antiepileptic drugs, can cause neutropenia (or myelosuppression
of all cell lines). In such cases, management requires consideration of risk versus benefit. If the
child has mild neutropenia with wellcontrolled seizures due to the medication, then it is usually
safe to continue the medication with close monitoring. However, if the neutropenia is severe,
switching to a different antiepileptic medication is advised. Chemotherapeutic agents are also
known to cause myelosuppression.

In the setting of severe neutropenia, the family should be instructed to avoid the insertion of
anything per rectum, including rectal temperatures, as this can cause seeding of bacteria into the
bloodstream. The febrile child with severe neutropenia requires immediate intervention including
evaluation, performing a blood culture, and prompt administration of broad-spectrum
intravenous antibiotics.

American Academy of Pediatrics 878


PREP® Self-Assessment PREPSA 2023

PREP Pearls
• Temporary viral suppression is one of the most common causes of acquired neutropenia
and is transient.
• Antiepileptic medications may cause neutropenia or pancytopenia.

ABP Content Specifications(s)


• Understand the association of drug therapy with neutropenia
• Recognize the association of common viral infections with transient neutropenia

Suggested Readings
• Atallah-Yunes SA, Ready A, Newburger PE. Benign ethnic neutropenia. Blood Rev.
2019;37:100586. doi:10.1016/j.blre.2019.06.003 .
• Nagalapuram V, McCall D, Palabindela P, et al. Outcomes of isolated neutropenia
referred to pediatric hematologyoncology clinic. Pediatrics. 2020;146(4):e20193637.
doi:10.1542/peds.2019-3637.
• Walkovich K, Boxer LA. How to approach neutropenia in childhood. Pediatr Rev.
2013;34(4):173-184. doi:10.1542/pir.344-173.

American Academy of Pediatrics 879


PREP® Self-Assessment PREPSA 2023

Question 262
A term neonate is admitted to the neonatal intensive care unit for further evaluation of limb
abnormalities. Physical examination of the upper extremities reveals internal rotation of the
shoulders, extension of the elbows, flexion and ulnar deviation of the wrists, and finger
contractures (Item Q262). Examination of the lower extremities reveals dislocation of the hips,
extension of the knees, and bilateral clubfoot. The remainder of the neonate’s physical
examination findings are unremarkable.

Item Q262: Upper extremity of the neonate described in the vignette.


Reprinted with permission from Cherian L, Rosenberg JJ. Visual diagnosis:2010;31(9):385.

Of the following, the neonate’s MOST likely diagnosis is


A. amnion rupture sequence
B. arthrogryposis
C. Pierre-Robin sequence
D. prune-belly syndrome

American Academy of Pediatrics 880


PREP® Self-Assessment PREPSA 2023

Correct Answer: B
The physical examination findings of the neonate in the vignette are most consistent with
arthrogryposis (also known as arthrogryposis multiplex congenita), a disorder characterized by
congenital contractures that affect 2 or more areas of the body. An isolated congenital
contracture, such as isolated clubfoot, is not considered arthrogryposis. Arthrogryposis is
associated with more than 400 conditions; however, less than half of cases have a known
underlying genetic cause.

The clinical features of arthrogryposis result from decreased or absent fetal joint movement in
utero. Impairment of joint mobility in utero can occur from restricted space, neuromuscular
disorders, vascular compromise, skeletal dysplasia, connective tissue disorders, metabolic
derangement, maternal illness, or teratogenic exposure.

The 2 most common forms of arthrogryposis are amyoplasia (congenital absence or


underdevelopment of muscles) and distal arthrogryposis. The neonate in the vignette has
amyoplasia. The clinical features of amyoplasia are outlined in Item C262.

The main goal of management of amyoplasia is to enhance the following functions:


• Joint mobilization
• Splints for improved position and function
• Physical and occupational therapy
• Surgery for persistent deformities

Distal arthrogryposes are a group of conditions that primarily affect the distal joints with
minimal to no involvement of the proximal joints. These disorders exhibit variability in their
expression. To date, 10 types of distal arthrogryposes have been identified with an autosomal
dominant inheritance pattern.

Amnion rupture sequence occurs when early rupture of the amnion leads to the production of
fibrous cords that constrict and damage parts of the growing fetus. Postnatal examination of the
placenta and membranes reveals the fibrous bands, which can sometimes be seen on prenatal
ultrasonography.

Clinical features of amnion rupture sequence may include the following:


• Amputation of a digit or limb
• Constriction of the umbilical cord
• Constrictions around a digit or limb
• Fused digits

Amniotic rupture sequence can cause decreased fetal movement and deformational defects.
However, multiple joint involvement in the absence of other characteristic findings makes
amniotic rupture sequence an unlikely diagnosis for the neonate in the vignette.
Classic features of the Pierre-Robin sequence (PRS) include micrognathia and glossoptosis
(retraction of the tongue), with or without cleft palate. These features are not found on the
American Academy of Pediatrics 881
PREP® Self-Assessment PREPSA 2023

physical examination of the neonate in the vignette. Joint contractures and dislocations are not
associated with PRS.

Prune-belly syndrome is characterized by early urethral obstruction leading to bladder and


abdominal distention. Urethral obstruction can be caused by posterior urethral valves, urethral
atresia, or bladder neck or distal urethral obstruction. Bladder distention can lead to hydroureter
and renal dysplasia. Abdominal distention leads to abdominal muscle deficiency and excess
abdominal skin. Prune-belly syndrome is named for the appearance of the abdomen once the
obstruction is relieved. Multiple joint contractures are not associated with prune-belly syndrome,
making it an unlikely diagnosis for the neonate in the vignette.

PREP Pearls
• Arthrogryposis is a disorder characterized by congenital contractures that affect more
than 1 area of the body.
• The clinical features of arthrogryposis result from decreased or absent fetal joint
movement in utero.
• The 2 most common forms of arthrogryposis are amyoplasia (congenital absence or
underdevelopment of muscles) and distal arthrogryposis.

ABP Content Specifications(s)


• Recognize the clinical features associated with arthrogryposis

Suggested Readings
• Bamshad M, Van Heest AE, Pleasure D. Arthrogryposis: a review and update. J Bone
Joint Surg. 2009;91(sppl 4):40-46. doi:10.2106/JBJS.I.00281.
• Hall JG. Arthrogryposis: diagnostic approach to etiology, classification, genetics, and
general principles. Eur J Med Genet. 2014;57(8):464-472.
doi:10.1016/j.ejmg.2014.03.008.
• Jones K, Jones M, del Campo M. Unusual brain and/or neuromuscular findings with
associated defects. In: Smith’s Recognizable Patterns of Human Malformation. 8th ed.
Elsevier; 2021.
• Rosen O, Marion RW, Samanich JM. Common congenital anomalies. In: McInerny TK,
Adam HM, Campbell DE, DeWitt TG, Foy JM, Kamat DM, eds. American Academy of
Pediatrics Textbook of Pediatric Care. American Academy of Pediatrics; 2021:chap 96.
Accessed September 1, 2022. Pediatric Care Online.

American Academy of Pediatrics 882


PREP® Self-Assessment PREPSA 2023

Question 263
A 9-month-old infant with biliary atresia (BA) is seen for a health supervision visit. She was
diagnosed with BA at age 3 months and underwent Kasai portoenterostomy following diagnosis.
Over the past few months, the infant has been hospitalized for 2 episodes of cholangitis, has
remained cholestatic, and has been closely followed by a pediatric hepatology team. On physical
examination, the infant’s sclera are icteric, and her abdomen is protuberant, with a fluid wave
and a hard liver edge palpable below the right costal margin. Her spleen is palpable 5 cm below
the left costal margin. Her extremities appear thin. The remainder of her physical examination
findings are unremarkable.

The infant was seen in the hepatology clinic last week; the hepatology team noted the same
anthropometric measurements and physical examination findings.

Of the following, the anthropometric measurement (z-score) that BEST reflects this infant’s
nutritional status is
A. abdominal girth
B. mid–upper arm circumference
C. weight for age
D. weight for length

American Academy of Pediatrics 883


PREP® Self-Assessment PREPSA 2023

Correct Answer: B
The infant in the vignette has chronic cholestatic liver disease (with evidence of hypersplenism
and ascites) as a result of biliary atresia. Assessment of this infant’s nutritional status is essential.
Infants and children with chronic cholestatic liver disease are at high risk for malnutrition, and
malnutrition is associated with increased morbidity and mortality. Anthropometric
measurements, including weight, length or height, weight for length (WFL) or body mass index
(BMI), mid–upper arm circumference (MUAC), and triceps skinfold (TSF), can be used to assess
nutritional status. In infants and children with organomegaly and/or ascites, weight and
WFL/BMI can overestimate nutritional status (and thus falsely reassure health care professionals
and families). In these situations, MUAC (an indicator of lean and adipose tissue mass) and TSF
(an indicator of adipose tissue mass) more accurately reflect nutritional status. Thus, of the
response choices, MUAC is the best anthropometric indicator of nutritional status for the infant
in the vignette at this time. There are World Health Organization MUAC and TSF standards for
age. Measurements of abdominal girth are not indicated for assessment of nutritional status.

Performance of MUAC and TSF measurements require a MUAC measuring tape and skinfold
caliper, respectively, and a health care professional (most often a dietitian) trained in these
techniques. For children with liver disease, these measurements are commonly obtained as part
of a multi-disciplinary hepatology clinic visit; they are not usually performed in a general
pediatrics outpatient setting (eg, primary care physician’s office).

Measurement of MUAC can also be used as a screening tool for assessment of malnutrition
during large-scale humanitarian relief efforts due to its low cost and ease of implementation.
Color-coded (for degree of malnutrition) MUAC measuring tapes are available.
Malnutrition in pediatric cholestatic chronic liver disease is multifactorial, resulting from
suboptimal caloric intake, nutrient maldigestion and malabsorption, and altered metabolism.
Anorexia and early satiety are common, caused by organomegaly, ascites, as well as altered
intestinal motility.

Maldigestion of fat can occur as a result of decreased bile flow. Fat malabsorption may occur
secondary to enteropathy associated with portal hypertension. The undigested and malabsorbed
fat remains in the bowel, resulting in diarrhea, steatorrhea, and loss of fat calories and fat-soluble
vitamins (A, D, E, and K). Deficiencies in fat-soluble vitamins and essential fatty acids can
occur; appropriate supplementation is required. Symptoms of fatsoluble vitamin deficiencies and
supplementation recommendations are listed in Item C263.

Additionally, infants and children with chronic cholestatic liver disease are hypermetabolic,
requiring 120% to 170% of predicted caloric needs for age, and experience altered carbohydrate
and protein metabolism. It is imperative that a multidisciplinary team with experience in
pediatric chronic cholestatic liver diseases (including dietitians, social workers, and nurses) be
closely involved in their management.

Infants and children with chronic cholestatic liver disease are at risk for linear growth failure.
While malnutrition is a significant factor in growth impairment, poor linear growth may persist
American Academy of Pediatrics 884
PREP® Self-Assessment PREPSA 2023

even with adequate nutritional support. Growth hormone resistance, due to inadequate hepatic
production of insulin-like growth factor-1, also contributes to their growth failure.

PREP Pearls
• Monitoring nutritional and growth status is essential for infants and children with chronic
cholestatic liver disease.
• Mid–upper arm circumference and triceps skin fold are helpful anthropometric measures
to assess the nutritional status of infants and children with organomegaly and/or ascites,
including chronic cholestatic liver disease.
• Linear growth failure in infants and children with chronic cholestatic liver disease is
multifactorial, including nutritional status and growth hormone resistance.

American Academy of Pediatrics 885


PREP® Self-Assessment PREPSA 2023

ABP Content Specifications(s)


• Understand the nutritional causes of growth failure associated with chronic cholestatic
disease, including the effects of nutrient digestion and absorption

Suggested Readings
• Boster JM, Feldman G, Mack CL, Sokol RJ, Sundaram SS. Malnutrition in biliary
atresia: assessment, management, and outcomes. Liver Transpl. 2022;28(3):483-492.
doi:10.1002/lt.26339.
• Cameron R, Kogan-Liberman D. Nutritional considerations in pediatric liver disease.
Pediatr Rev. 2014;35(11):493-496. doi:10.1542/pir.35-11-493.
• Tang MN, Adolphe S, Rogers SR, Frank DA. Failure to thrive or growth faltering:
medical, developmental/behavioral, nutritional, and social dimensions. Pediatr Rev.
2021;2(11):590-603. doi:10.1542/pir.2020-001883.
• Tessitore M, Sorrentino E, Schiano Di Cola G, Colucci A, Vajro P, Mandato C.
Malnutrition in pediatric chronic cholestatic disease: an up-to-date overview. Nutrients.
2021;13(8): 2785. doi:10.3390/nu13082785.
• WHO child growth standards: head circumference-for-age, arm circumference-for-age,
triceps skinfold-for-age and subscapular skinfold-for-age: methods and development.
World Health Organization. November 12, 2007. Accessed September 1, 2022.
https://www.who.int/publications/i/item/9789241547185.

American Academy of Pediatrics 886


PREP® Self-Assessment PREPSA 2023

Question 264
A 15-year-old adolescent boy is seen for a sports preparticipation evaluation to play competitive
baseball. He has no significant medical history and is not taking any medications. His family
history is negative for kidney disease. The boy’s weight is at the 75th percentile and height is at
the 90th percentile for age. His blood pressure is 110/70 mm Hg. The remainder of his physical
examination findings are normal. A urine sample was sent to the laboratory for testing after the
results of a dipstick test in the office were positive for protein.

The results of urine testing are shown:


Test Result
Appearance Yellow
Specific gravity 1.015
Leukocyte esterase Negative
Nitrite Negative
Blood Negative
Protein 2+
Red blood cells/HPF <5
White blood cells/HPF <5
Urine protein 60 mg/dL
Urine creatinine 100 mg/dL (8,840 µmol/L)

Of the following, the BEST next step in management of this adolescent is to


A. order a 24-hour urine protein excretion test
B. order a first morning urine protein:creatinine ratio
C. provide clearance for sports participation
D. provide referral for a pediatric nephrologist

American Academy of Pediatrics 887


PREP® Self-Assessment PREPSA 2023

Correct Answer: B
The adolescent boy in the vignette has asymptomatic proteinuria, as suggested by the findings of
2+ protein and a urine protein:creatinine ratio (UPCR) of 0.6 mg/mg on random urinalysis. The
best next step in his evaluation is to obtain a first morning UPCR to confirm orthostatic
proteinuria. If the first morning UPCR is normal (≤0.2 mg/mg), clearance for sports participation
can be given. If the first morning UPCR is greater than 0.2 mg/mg, the boy should be referred to
a pediatric nephrologist, and clearance for sports participation should be withheld pending that
evaluation.

Proteinuria is defined as a urine protein excretion greater than 100 mg/m2 daily or a UPCR of
greater than 0.2 mg/mg on a spot urine sample. Urine dipstick testing is commonly used to
screen for proteinuria; results are reported as negative, trace, 1+ (30 mg/dL), 2+ (100 mg/dL), 3+
(300 mg/dL), and 4+ (2,000 mg/dL). Nephroticrange proteinuria is defined as a urine protein
excretion greater than 1,000 mg/m2 daily or a UPCR of greater than 2 mg/mg on a spot urine
sample. Although 24-hour urine collection is generally considered an accurate method for
measuring and calculating protein excretion, it is cumbersome in children and at risk of error
because of overcollection or undercollection. As a result, 24-hour urine collection for protein
excretion is not commonly performed in children.

Proteinuria can be classified as transient, orthostatic, or persistent. Transient proteinuria is


commonly associated with fever, exercise, or stress. With orthostatic proteinuria (commonly
seen in adolescents), there is increased excretion of protein in the standing position, which
normalizes when the child is recumbent. In orthostatic proteinuria, a random urine sample will be
positive for protein on urine dipstick testing and the UPCR may be greater than 0.2 mg/mg, but
on a first morning urine specimen the UPCR will be less than or equal to 0.2 mg/mg. Persistent
proteinuria may have a glomerular or tubular origin. The broad list of causes of persistent
proteinuria is given in Item C264.

The initial evaluation of a child with proteinuria includes a thorough history, including recent
illness, fever, rash, joint pain, urine output, urine color, weight gain or loss, history of urinary
tract infection, and family history of deafness or kidney disease. Physical examination should
include blood pressure measurement and evaluation for edema, ascites, and pallor. Children with
persistent nephrotic-range proteinuria can develop hypoalbuminemia and edema and present with
nephrotic syndrome. Laboratory evaluation for children with persistent proteinuria should
include serum electrolyte, blood urea nitrogen, serum creatinine, serum albumin, cholesterol, and
serum complement measurement; antistreptolysin O titer; antinuclear antibody test; hepatitis
serologic testing; and a complete urinalysis with a first morning UPCR.

Children with orthostatic proteinuria should have a first morning urinalysis checked at each
health supervision visit to monitor for new findings suggestive of renal disease. Children with
persistent proteinuria; proteinuria associated with hematuria, hypertension, or azotemia;
proteinuria with a family history of renal disease, deafness, or autoimmune conditions; or
proteinuria associated with comorbidities, including prematurity, obesity, and diabetes, should be
referred to a pediatric nephrologist.
American Academy of Pediatrics 888
PREP® Self-Assessment PREPSA 2023

American Academy of Pediatrics 889


PREP® Self-Assessment PREPSA 2023

American Academy of Pediatrics 890


PREP® Self-Assessment PREPSA 2023

PREP Pearls
• Proteinuria is defined as a urine protein excretion greater than 100 mg/m2 daily or urine
protein:creatinine ratio of greater than 0.2 mg/mg on a spot urine sample.
• In orthostatic proteinuria, a random urine sample will be urine dipstick test positive for
protein and the urine protein:creatinine ratio may be greater than 0.2 mg/mg, but on a
first morning urine specimen the protein:creatinine ratio will be less than or equal to 0.2
mg/mg.
• Children with orthostatic proteinuria should have a first morning urinalysis checked at
each health supervision visit.

ABP Content Specifications(s)


• Recognize the clinical features associated with hypoproteinemia
• Identify the possible causes of proteinuria
• Plan the appropriate clinical and laboratory evaluation of proteinuria

Suggested Readings
• Viteri B, Reid-Adam J. Hematuria and proteinuria in children. Pediatr Rev.
2018;39(12):573-587. doi:10.1542/pir.2017-0300
• Woroniecki RP, Singer PS. Proteinuria. In: McInerny TK, Adam HM, Campbell DE,
DeWitt TG, Foy JM, Kamat DM, eds. American Academy of Pediatrics Textbook of
Pediatric Care. American Academy of Pediatrics; 2021:chap 183. Accessed September 1,
2022. Pediatric Care Online.

American Academy of Pediatrics 891


PREP® Self-Assessment PREPSA 2023

Question 265
A 5-year-old girl and her 13-year-old brother are brought to the office by their parents for a
health supervision visit. Their father feels that his daughter is underweight and his son is at a
healthy weight. Their mother disagrees and feels that their daughter is at a healthy weight and
their son is overweight. The family eats a vegetarian diet with a wide variety of foods. The boy
eats significantly more grains and is more sedentary than the girl. On physical examination, the
girl’s weight and height are at the 50th percentile for age, and her body mass index (BMI) is at
the 60th percentile, consistent with her previous growth pattern. She has a sexual maturity rating
of 1. The boy’s weight is at the 80th percentile for age, increased from the 60th percentile 1 year
ago, and his height is at the 50th percentile, consistent with his previous measurements. His
current BMI is at the 92nd percentile. He has a sexual maturity rating of 2. The remainder of the
siblings’ physical examination findings are normal.

Of the following, the BEST next step in these siblings’ management is to


A. explore the parents’ expectations of their children’s growth and development
B. perform laboratory evaluation for vitamin and mineral deficiencies for both children
C. provide reassurance that the girl’s weight is normal and the boy’s height percentile will
catch up to his weight
D. recommend that the girl increase her intake of grains and the boy increase his physical
activity level

American Academy of Pediatrics 892


PREP® Self-Assessment PREPSA 2023

Correct Answer: A
The parents of the siblings in the vignette have different perceptions regarding the weight status
of their children. The best next step in their management is to explore the parents’ expectations
of their children’s growth and development. Reassuring them that their daughter’s weight is
normal and that their son’s height percentile will catch up to his weight does not address their
concerns. Recommending that their daughter increase her intake of grains and their son increase
his physical activity level is also incomplete advice. Performing laboratory evaluation for
vitamin and mineral deficiencies may be appropriate given the family’s vegetarian diet, but the
parents’ concerns should be explored first.

Nutrition and growth are often major concerns of families and can be influenced by multiple
cultural factors, such as socioeconomic status, generational customs, living environment, and
peer influences. Even individual family members from the same general culture can have
different perspectives because of personal beliefs and experiences. For example, further inquiry
into the concerns of the family in the vignette may reveal that the mother has a family history of
medical complications from obesity, whereas the father may have had a family history of
medical complications from being underweight. Similarly, some families believe that a small
amount of excess weight is a sign of good health, whereas others may believe that any excess
weight is dangerous.

There can also be considerable dietary variation among families following specific diets (eg,
vegetarian, halal, and kosher). These diets and their variations may result in nutritional
deficiencies. For example, some vegetarian diets may be deficient in vitamin B12 and zinc. Diets
that are nutritionally adequate in one environment may be nutritionally inadequate in another
where the usual foods are less available.

Being mindful of the diversity and cultural importance of individual families’ dietary practices
can help pediatricians promote healthy nutrition and growth.

PREP Pearls
• Exploring parents’ expectations of their children’s growth and development is usually the
best first step in addressing concerns about nutrition.
• Nutrition and growth are often influenced by multiple cultural factors, such as
socioeconomic status, generational customs, living environment, and peer influences.
• Specific cultural diets and their variations may result in nutritional deficiencies that may
depend on the families’ living environment.

ABP Content Specifications(s)


• Understand the family and cultural determinants that influence dietary practices and
nutrition

American Academy of Pediatrics 893


PREP® Self-Assessment PREPSA 2023

Suggested Readings
• Andrewski E, Cheng K, Vanderpool C. Nutritional deficiencies in vegetarian, gluten-free,
and ketogenic diets. Pediatr Rev. 2022;43(2):61-70. doi:10.1542/pir.2020-004275.
• Promoting healthy nutrition. In: Hagan JF, Shaw JS, Duncan PM, eds. Bright Futures:
Guidelines for Health Supervision of Infants, Children, and Adolescents. 4th ed.
American Academy of Pediatrics; 2017:167-192.
• Promoting healthy weight. In: Hagan JF, Shaw JS, Duncan PM, eds. Bright Futures:
Guidelines for Health Supervision of Infants, Children, and Adolescents. 4th ed.
American Academy of Pediatrics; 2017:151-166.
• Tang MN, Adolphe S, Rogers SR, Frank DA. Failure to thrive or growth faltering:
medical, developmental/behavioral, nutritional, and social dimensions. Pediatr Rev.
2021;42(11):590–603. doi:10.1542/pir.2020-001883.
• US Department of Agriculture and US Department of Health and Human Services.
Dietary Guidelines for Americans, 2020-2025. 9th ed. US Dept of Agriculture; 2020.

American Academy of Pediatrics 894


PREP® Self-Assessment PREPSA 2023

Question 266
A 16-year-old adolescent girl is seen in the office for a preparticipation physical evaluation
(PPE) prior to her school softball season. She also participates in competitive weight lifting. She
has a family history of hypertension. Review of medical records from her prior visits reveals
blood pressure measurements of 132/93 mm Hg and 134/91 mm Hg at her most recent visits. Her
blood pressure on the day of the PPE is 131/92 mm Hg. The remainder of her history and
physical examination findings are unremarkable.

Of the following, the BEST approach to this girl’s sports participation is to


A. clear her for both softball and weight lifting
B. clear her for softball but not for weight lifting
C. clear her for weight lifting but not for softball
D. not clear her for sports participation until she is evaluated for hypertension

American Academy of Pediatrics 895


PREP® Self-Assessment PREPSA 2023

Correct Answer: A
The girl in the vignette has stage I hypertension. Athletes with stage I hypertension can be
cleared for participation in all types of sports provided they do not have other conditions that
would limit participation. The girl in the vignette can be cleared for both softball and weight
lifting.

To meet the criteria for stage I hypertension, children younger than 13 years must have blood
pressure measurements at or above the 95th percentile but lower than the 95th percentile plus 12
mm Hg. For children 13 years and older, adult hypertension criteria are used; stage 1
hypertension is defined as having blood pressure measurements from 130/80 to 139/89 mm Hg
The girl in the vignette has had repeated measurements that meet these criteria; evaluation for
secondary causes of hypertension is indicated, including urinalysis, a basic metabolic panel, and
lipid profile. Other testing may be warranted depending on findings from a detailed history and
physical examination. Item C266A details the criteria for elevated blood pressure and
hypertension in children and adolescents.

Children and adolescents with stage II hypertension should not participate in sports with a high
static component such as weight lifting, martial arts, and gymnastics. Athletes with stage II
hypertension can resume participation in high static component sports once evaluation has been
completed, treatment has been instituted, and blood pressure is brought into the normal range.
Softball has a low static component and a moderate dynamic component. Therefore, athletes
with stage II hypertension can be cleared for softball participation. Item C266B illustrates the
classification of sports by static and dynamic components.

American Academy of Pediatrics 896


PREP® Self-Assessment PREPSA 2023

PREP Pearls
• Young athletes with stage I hypertension can participate in sports without restrictions.
• Children and adolescents with untreated stage II hypertension should be restricted from
sports with a high static
• component (eg, weight lifting, martial arts, and gymnastics).
• Criteria for stage I hypertension in children younger than 18 years require blood pressure
measurements at or above the 95th percentile but lower than the 95th percentile plus 12
mm Hg.

ABP Content Specifications(s)


• Identify which sports are appropriate for athletes with various conditions that may limit
sports participation

American Academy of Pediatrics 897


PREP® Self-Assessment PREPSA 2023

Suggested Readings
• Bernhardt DT, Roberts WO, American Academy of Family Physicians. PPE:
Preparticipation Physical Evaluation. 5th ed. American Academy of Pediatrics; 2019.
• Bernhardt DT. Sports preparticipation physical evaluation. In: McInerny TK, Adam HM,
Campbell DE, DeWitt TG, Foy JM, Kamat DM, eds. American Academy of Pediatrics
Textbook of Pediatric Care. American Academy of Pediatrics; 2021:chap 17. Accessed
September 1, 2022. Pediatric Care Online.
• Flynn JT, Kaelber DC, Baker-Smith CM, et al. Clinical Practice Guideline for Screening
and Management of High Blood Pressure in Children and Adolescents. Pediatrics.
2017;140(3):e20171904. doi:10.1542/peds.2017-1904.
• McCambridge TM, Benjamin HJ, Brenner JS, et al. Athletic participation by children and
adolescents who have systemic hypertension. Pediatrics. 2010;125(6):1287-1294.
doi:10.1542/peds.2010-0658.
• National High Blood Pressure Education Program Working Group on High Blood
Pressure in Children and Adolescents. The fourth report on the diagnosis, evaluation, and
treatment of high blood pressure in children and adolescents. Pediatrics. 2004;114(2
Suppl 4th Report):555-576. doi:10.1542/peds.114.S2.555.

American Academy of Pediatrics 898


PREP® Self-Assessment PREPSA 2023

Question 267
After transitioning care from the neonatal intensive care unit to the general nursery, a near-term
neonate was inadvertently given an incorrect dose of heparin through a peripherally inserted
central catheter. The neonate experienced clinically significant hemorrhage and required a blood
transfusion. The event was reported through the patient safety reporting system, and an
interdisciplinary team was assembled to identify the contributing factors leading to the
medication error. The team thoroughly analyzed the event, and established and prospectively
analyzed a new medication administration process using evidence-based methods to ensure the
least possible risk of future harm.

Of the following, the MOST likely methodology used to analyze the new process before roll-out
was
A. failure modes and effects analysis
B. key driver diagrams
C. Plan-Do-Study-Act cycles
D. root cause analysis

American Academy of Pediatrics 899


PREP® Self-Assessment PREPSA 2023

Correct Answer: A
The interdisciplinary team in the vignette most likely used failure mode and effect analysis
(FMEA) to establish and assess the new medication administration processes in the newborn
nursery before implementation. Although root cause analysis of the medication error likely
contributed to development of the new medication administration process, FMEA provides a
more comprehensive, prospective assessment of new processes.

Failure mode and effect analysis is used in patient safety science to prospectively assess complex
processes by identifying the specific risk factors (failure modes) that may result in harm, often
before the new process is implemented. Once identified, these risk factors can be ranked
according to their likelihood of causing harm, or by the degree or severity of potential harm. The
underlying concept of this methodology is that risk is related not only to the likelihood of a
failure occurring, but also to the severity of the consequence of the error. Each failure mode can
be assigned a numerical score or weight that objectively quantifies its priority for a mitigation
strategy. The overall benefit of an FMEA is that it provides a method for prospective analysis of
risk in a complex system before the system has been activated and thus can avoid potential errors
and harms. It also allows for the possibility of reducing the need to revise the process after
implementation because of unforeseen circumstances.

The first widely known use of FMEA was by the United States military. The National
Aeronautics and Space Administration adopted the methodology as a crucial project-planning
technique for the Apollo spacecraft mission, and continues to employ FMEA for spacecraft
mission planning. Failure mode and effect analysis is widely used by the commercial aviation
industry to assess aircraft safety. This methodology is now used across the health care arena.
Key driver diagrams are used by improvement teams as graphical displays of the relationship
between the overall aim of a project and the key/primary drivers that contribute directly to
achieving that aim. This methodology facilitates a shared understanding of areas that may require
process design or modification in order to achieve the project objectives. Key driver diagrams
are generally a small part of an overall improvement process and may contribute to the
development of new processes. Key driver diagrams are not used to prospectively analyze the
likely success or failure of a new process, and thus is not the preferred response.

Plan-Do-Study-Act (PDSA) cycles are used to test a change in a process that already exists. Each
PDSA cycle tests the change through a series of steps including: developing a plan to assess the
impact of the new process (Plan), starting the new process (Do), observing and analyzing the
impact of the new process (Study), and reviewing the results of the analysis and determining
what, if any, modifications should be made to optimize the new process (Act). Although PDSA
cycles are often used to design and implement a new process, the general utility of a PDSA cycle
is more often to make changes to a process that needs revision or adjustments. Thus, this is not
the preferred response.

A root cause analysis (RCA) is a retrospective tool used to identify the root cause(s) of an error
that has already occurred. Root cause analysis often involves the process of asking “why?” up to
5 times in series. When a problem is identified, RCA will ask, “Why did this happen?” and rather
American Academy of Pediatrics 900
PREP® Self-Assessment PREPSA 2023

than stopping at the answer to this first question, asks, “Why did that happen?” again and again
until the theoretical root cause of the error has been identified. Root cause analysis cannot be
used prospectively to assess risk of error or harm.

PREP Pearls
• Failure mode and effect analysis is a prospective process that can identify latent risks of
patient harm and identify opportunities for improvement before errors occur.
• Key driver diagrams are graphical displays of the relationship between the overall aim of
a project and the key/primary drivers that contribute directly to achieving that aim.
• A root cause analysis is a retrospective tool used to identify the root cause(s) of an error
that has already occurred.

ABP Content Specifications(s)


• Understand the relationship between the detection of a medical error and the ability to
discover and affect improvements

Suggested Readings
• Bartman T, McClead RE. Core principles of quality improvement and patient safety.
Pediatr Rev. 2016;37(10):407-417. doi:10.1542/pir.2015-0091.
• Lago P, Bizzarri G, Scalzotto F, et al. Use of FMEA analysis to reduce risk of errors in
prescribing and administering drugs in paediatric wards: a quality improvement report.
BMJ Open. 2012;2(6):e001249. doi:10.1136/bmjopen-2012-001249.
• Mueller BU, Neuspiel DR, Stucky Fisher ERS; Council on Quality Improvement and
Patient Safety, Committee on Hospital Care. Principles of pediatric patient safety:
reducing harm due to medical care. Pediatrics. 2019;143(2):e20183649.
doi:10.1542/peds.2018-3649.
• Schriefer J, Leonard MS. Patient safety and quality improvement: an overview of QI.
Pediatr Rev. 2012;33(8):353-360. doi:10.1542/pir.33.8.353.

American Academy of Pediatrics 901


PREP® Self-Assessment PREPSA 2023

Question 268
A 2-year-old boy is evaluated for leg pain. He was adopted at 15 months of age and did not walk
until 18 months of age. The boy has not had any fractures. He eats a balanced diet that includes
table foods and cow milk. The boy’s adoptive parents know little about his biological family
other than that the biological mother is very short. The boy’s physical examination findings are
significant only for bowing of both legs. A right knee radiograph shows osteopenia and widened
and frayed metaphyses.

Laboratory data are shown:


Laboratory Test Result
Calcium 9.2 mg/dL (2.3 mmol/L) (reference range, 9.1-10.5 mg/dL [2.27-
2.63 mmol/L])
Phosphate 1.3 mg/dL (0.42 mmol/L) (2.8-6.2 mg/dL [0.9-2.0 mmol/L])
Intact parathyroid 45 pg/mL (reference range, 15-65 pg/mL)
hormone
25-hydroxyvitamin D 27 ng/mL (67.4 nmol/L) (reference range, 30-100 ng/mL [74.9-
249.6 nmol/L)
1,25- 50 pg/mL (120 pmol/L) (reference range, 20-80 pg/mL [48-192
dihydroxyvitamin D pmol/L])
Alkaline phosphatase 780 U/L (reference range,100-400 U/L)

Of the following, the MOST likely cause of this boy’s findings is


A. abnormal calcium-sensing receptor
B. hypoparathyroidism
C. vitamin D deficiency
D. X-linked hypophosphatemia

American Academy of Pediatrics 902


PREP® Self-Assessment PREPSA 2023

Correct Answer: D
The boy in the vignette has hypophosphatemic rickets due to X-linked hypophosphatemia
(XLH), the most common inherited form of rickets. Children with XLH typically have symptoms
and signs of rickets (eg, leg bowing in ambulating children, bone pain, pathologic fractures) and
short stature. Dental abscesses are common. X-linked hypophosphatemia is inherited in an X-
linked dominant fashion. The mother of the boy in the vignette is likely affected by XLH given
her short stature.

X-linked hypophosphatemia is caused by pathologic variants in the PHEX gene, which results in
a high fibroblast growth factor 23 (FGF23) level. Fibroblast growth factor 23 is produced in
osteocytes and osteoblasts and causes renal phosphate wasting by inhibiting phosphate
reabsorption in the kidney. Conditions associated with elevated FGF23 levels result in a low
serum phosphate level and an inappropriately normal 1,25-dihydroxyvitamin D level. Low serum
phosphate levels normally stimulate 1α-hydroxylase in the kidney to increase 1,25-
dihydroxyvitamin D levels, but this response does not occur in XLH. Parathyroid hormone
(PTH) levels are usually normal in XLH because calcium levels are normal. The alkaline
phosphatase level is elevated due to the high bone turnover state.

The traditional treatment of XLH is oral phosphate and calcitriol (1,25-dihydroxyvitamin D)


replacement. In 2018, an anti-FGF23 monoclonal antibody (burosumab) was approved by the US
Food and Drug Administration for the treatment of XLH in children.

In addition to FGF23, PTH and 1,25-dihydroxyvitamin D (calcitriol) are important regulators of


serum phosphate levels. Parathyroid hormone also causes renal phosphate wasting. Calcitriol
acts to absorb phosphate from the gut.

Conditions associated with high PTH levels typically result in low serum phosphate levels. The
boy in the vignette does not have an elevated PTH level, so his hypophosphatemia is not
explained by hyperparathyroidism.

Hypoparathyroidism, in which PTH levels may be inappropriately “normal,” would result in a


low serum calcium and elevated serum phosphate level.

The calcium-sensing receptor (CaSR) is located in the parathyroid cells and is the main mediator
of PTH secretion from the parathyroid glands. Loss-of-function pathologic variants in the CaSR
gene result in inappropriately elevated PTH levels due to an inability to adequately detect
calcium in the blood. This hyperparathyroidism results in the classic pattern of hypercalcemia
and hypophosphatemia. Depending on the pathologic variant, familial hypocalciuric
hypercalcemia, a benign condition, or the more serious neonatal severe hyperparathyroidism,
results. The boy in the vignette does not have an elevated PTH or calcium level, making a
pathologic variant in the CaSR unlikely.

Vitamin D deficiency can result in various calcium and phosphate patterns depending on the
severity. In severe vitamin D deficiency, both calcium and phosphate levels are low due to an
American Academy of Pediatrics 903
PREP® Self-Assessment PREPSA 2023

inability to absorb these minerals from the intestine. Parathyroid hormone is also elevated, which
exacerbates hypophosphatemia by increasing phosphate wasting in the kidney. The boy in the
vignette does not have a 25-hydroxyvitamin D level consistent with vitamin D deficient rickets
and his PTH level is not elevated.

PREP Pearls
• The metabolic abnormalities associated with X-linked hypophosphatemia are due to
elevated levels of fibroblast growth factor 23 (FGF23), which causes renal phosphate
wasting.
• X-linked hypophosphatemia is characterized by a low phosphate and inappropriately
normal 1,25-dihydroxyvitamin D levels; a low serum phosphate level normally stimulates
1α-hydroxylase in the kidney to increase 1,25dihydroxyvitamin D levels.
• The traditional treatment of X-linked hypophosphatemia is oral phosphate and calcitriol
(1,25-dihydroxyvitamin D) replacement. A monoclonal antibody against FGF23
(burosumab) is available to aid management of the condition.

ABP Content Specifications(s)


• Recognize the clinical and laboratory features associated with hypophosphatemia Plan
the appropriate management of familial hypophosphatemic rickets

Suggested Readings
• Kinoshita Y, Fukumoto S. X-linked hypophosphatemia and FGF23-related
hypophosphatemic diseases: prospect for new treatment. Endocr Rev. 2018;39(3):274-
291. doi:10.1210/er.2017-00220.
• Underland L, Markowitz M, Gensure R. Calcium and phosphate hormones: vitamin D,
parathyroid hormone, and fibroblast growth factor 23. Pediatr Rev. 2020;41(1):3-11.
doi:10.1542/pir.2018-0065.
• US Department of Health & Human Services; National Institutes of Health; NCATS. X-
linked hypophosphatemia. Genetic and Rare Diseases Information Center. Updated
February 23, 2018. Accessed September 1, 2022.
https://rarediseases.info.nih.gov/diseases/12943/x-linked-hypophosphatemia.

American Academy of Pediatrics 904


PREP® Self-Assessment PREPSA 2023

Question 269
A girl is seen for a health supervision visit. She can skip and hop on one foot several times. She
is able to write her first name, count to 10, and name 10 colors. She enjoys rhyming words and
tells a story with proper sequence.

Of the following, this girl is also MOST likely able to


A. copy a flag
B. remember her telephone number
C. tandem walk
D. tie her shoelaces

American Academy of Pediatrics 905


PREP® Self-Assessment PREPSA 2023

Correct Answer: B
The girl in the vignette has developmental milestones consistent with a 5-year-old. In addition to
the milestones described, she should be able to remember her telephone number. The late
preschool and early childhood years (5 to 6 years) are characterized by rapid growth in language
skills, cognition, and imaginative play. Parents play an important role in school readiness and
transition by establishing consistent routines, encouraging reading, acknowledging fears, and
bolstering self-esteem.

Developmental milestones expected of a 5-year-old include the ability to balance on one foot for
about 8 seconds and hop about 15 times. Children at age 5 years can copy a triangle, draw a
person with 8 to 10 body parts, selfdress, count to 10, name 10 colors, and remember their phone
number and address. Parents should instruct their children to memorize their telephone number
for safety purposes.

Language and communication skills at age 5 years should include a more than 2,000-word
vocabulary and use of complete sentences to tell stories with a beginning, middle, and end. Five-
year-old children have groups of friends, are able to apologize for mistakes, enjoy rhyming
words, and understand adjectives. Imagination, fantasy, and creativity emerge at this age.
Copying a flag and tying shoe laces are milestones typical of a 6-year-old child. In addition, 6-
year-olds are able to tandem walk, look both ways before crossing the street, draw a diamond,
and write their first and last name. Sixyear-old children are able to understand seasons, know the
days of the week, and count to 20. They are able to distinguish fantasy from reality and make
meaningful relationships with their peers including wanting to please them.

As children progress through the late elementary school years (ages 8 to 10 years), they develop
strong peer relationships, an increased ability to problem solve, and understanding of abstract
concepts, and they begin competitive participation in organized activities.

PREP Pearls
• The ability to memorize a telephone number is a developmental milestone typical of 5-
year-old children. Parents should instruct their children to memorize their telephone
number for safety purposes.
• Language and communication milestones of a 5-year-old include a vocabulary of >2,000
words, complete sentence formation, and sequential storytelling.
• Imaginative play and creativity emerge at ages 5 to 6 years.

ABP Content Specifications(s)


• Evaluate the cognitive and behavioral developmental progress/status of a child at 5 years
of age
• Evaluate the motor developmental progress/status of a child at 5 years of age

Suggested Readings
• Gerber RJ, Wilks T, Erdie-Lalena C. Developmental milestones: motor development.
Pediatr Rev. 2010;31(7):267-277; quiz 277. doi:10.1542/pir.31-7-267.
American Academy of Pediatrics 906
PREP® Self-Assessment PREPSA 2023

• Hagan JF, Shaw JS, Duncan PM, eds. Bright Futures: Guidelines Pocket Guide:
Guidelines for Health Supervision of Infants, Children, and Adolescents. 4th ed.
American Academy of Pediatrics; 2017.
• Scharf RJ, Scharf GJ, Stroustrup A. Developmental milestones. Pediatr Rev.
2016;37(1):25-37. doi:10.1542/pir.20140103.

American Academy of Pediatrics 907


PREP® Self-Assessment PREPSA 2023

Questions Per Speciality

Adolescent
13,32,50,71,111,138,212,234,251
Allergy and Immunology
31,82,127,148,176,204,260
Behavioral and Mental Health Issues
33,102,105,115,145,158,169,171,238
Blood and Neoplastic Disorders
4,26,65,72,75,100,108,137,164,179,199,216,233,261
Cardiovascular Disorders
15,36,73,95,117,120,123,151,154,203,254
Child abuse and neglect
58,155,183,195,214
Collagen Vascular and Other Multisystem Disorders
10,74,187,
Critical care
2,129,189,213,221,
Disorders of Cognition, Language, and Learning
11,35,53,88,193,218,245
Disorders of the Eye
55,202,226
Emergency Care
80,185,229,241
Endocrine disorders
68,86,119,186,268
ENT Disorders
14,21,66,98,118,157,163,170,181,227,230,236,256
Ethics for Primary Pediatricians
43,152,188
Fetus and Newborn Infant
1,8,22,29,49,67,84,99,107,113,165,168,201,221,259
Fluids and electrolytes
25,44,54,78,264
Gastroenterology
5,16,57,112,135,174,190,208,225,243,
Genetics
63,136,196,240
Genital system
18,194,246
Growth and development
40,83,125,132,134,140,161,175,219,269
Infectious Diseases
24,45,51,56,64,76,89,103,110,114,124,131,162,178,180,217,231,258
American Academy of Pediatrics 908
PREP® Self-Assessment PREPSA 2023

Metabolic
42,182,255
MSK
70,87,93,106,160,223,237,242,248,262
Neurology
17,38,46,59,79,96,101,205,215,228,249
Nutritional Disorders
7,61,146,263,265
Patient safety
6,37,94,207
Pharmacology
23,104,173,209
Poisoning
28,81,128,143,257,
Preventive Pediatrics
3,48,121,149,235,250,
Psychosocial issues
109,244
Renal
19,41,62,92,126,141,153,184,197,211,239,
Research
69,167,177,198,224,267,
Respiratory
9,52,85,90,122,139,156,172,192,210,232,247
Skin disorders
144,147,150,191
Sport Medicine
20,39,60,77,91,116,159,200,206,266
Substance abuse
12,97,222

American Academy of Pediatrics 909

You might also like